Case Studies in Pain Management (Cambridge Medicine) [1 ed.] 1107682894, 9781107682894, 2014006285, 9781316057568, 9781107281950

Edited by internationally recognized pain experts, this unique book describes 73 real life clinical cases, each followed

1,586 135 6MB

English Pages [545] Year 2014

Report DMCA / Copyright

DOWNLOAD FILE

Polecaj historie

Case Studies in Pain Management (Cambridge Medicine) [1 ed.]
 1107682894, 9781107682894, 2014006285, 9781316057568, 9781107281950

  • Commentary
  • TRUE PDF

Table of contents :
Cover
Half-title
Title page
Copyright information
Dedication
Table of contents
Contributors
Foreword
Section 1 Neurological Disorders
Chapter 1 Postherpetic neuralgia
Case study
1. What are the basic facts regarding postherpetic neuralgia, varicella-zoster virus, and shingles?
2. What are the basic features of postherpetic neuralgia?
3. Why are there are so many challenges with regard to postherpetic neuralgia treatment options?
4. What are the guidelines for postherpetic neuralgia management?
Criteria for recommendations varies
5. Are there any systematic reviews and meta-analysis data on postherpetic neuralgia treatments?
6. Are there any gaps in the Postherpetic Neuralgia Treatment Guidelines?
7. Are there any new clinical data on postherpetic neuralgia treatments?
High-concentration (8%) topical capsaicin patch
Gastroretentive gabapentin
Gabapentin enacarbil
Pregabalin combination therapeutic approaches
8. What are key considerations in choosing postherpetic neuralgia treatments?
Tolerability
Dosing and onset of analgesia
Are there challenging subsets of patients and guideline gaps in subpopulations of patients with postherpetic neuralgia?
The older patient
Renal/hepatic impairment
Patient with anxiety and depression
The patient with a history of substance abuse
Miscellaneous considerations
9. What are the key summarized points for treatment of postherpetic neuralgia?
References
Chapter 2 Patient with spinal cord injury pain
Case study
1. What is this patient's diagnosis?
2. How many spinal cord injury (SCI) patients are there, and how common is chronic pain in this population?
3. What are commonly experienced types and examples of pain conditions in SCI patients?
4. What types of neuropathic pain are experienced in SCI patients?
5. What form does visceral pain take in SCI patients?
6. What other painful conditions are SCI patients prone to?
7. What other conditions must the pain provider be aware of?
8. What is the relevance of heterotopic ossification?
9. What work-up might be helpful to clarify pain diagnosis in an SCI patient?
10. What are the pharmacologic treatments available for patients with SCI pain?
11. Are there interventional treatments that have been shown to be beneficial for SCI patients?
12. What are the non-pharmacologic treatments effective for SCI pain?
13. Are there particular issues or sensitivities that you need to be aware of when working with SCI patients?
Summary
References
Chapter 3 Patient with poststroke pain
Case study
1. What is the diagnosis explaining this patient's pain?
2. How common is poststroke pain?
3. What is the incidence of CPSP (epidemiology)?
4. What painful conditions are commonly seen in poststroke patients?
5. What area of the brain injury correlates with the development of central poststroke pain?
6. What is the likely mechanism of poststroke pain?
What is the likely mechanism of CPSP?
What is the likely mechanism of shoulder pain?
What is known about spasticity after stroke?
What is known about headache poststroke?
7. What is the work-up of poststroke pain?
Diagnosis of CPSP
Hemiplegic shoulder pain work-up
8. What pharmacologic treatments are available for poststroke pain?
What treatments are available for CPSP?
What are the treatments available for hemiplegic shoulder pain?
9. Are there any interventional treatments that have been tried for the treatment of patients with poststroke pain?
10. What are some special concerns in poststroke pain?
Conclusion/summary
References
Chapter 4 Patient with brachial plexopathy
Case study
1. What is brachial plexopathy?
2. Describe the anatomy of the brachial plexus
3. Clinical classification of brachial plexopathies
4. What is the epidemiology of plexopathies?
5. What are the causes and mechanisms of brachial plexopathies?
6. What happens to nerves in plexopathy?
7. Describe the clinical presentation of brachial plexopathies
8. What is the differential diagnosis?
9. How do you diagnose nerve root avulsion/brachial plexopathies?
10. What other disease processes mimic traumatic nerve root avulsion?
11. How should I treat this patient?
12. Are there any complications to worry about following treatment?
13. What is the prognosis of brachial plexopathies?
14. What are the social considerations in brachial plexopathy patients?
Conclusions
References
Chapter 5 Phantom limb pain
Case study
1. What is postamputation pain?
2. What is phantom limb pain?
3. Demographics/epidemiology of phantom pain?
4. What are the indications for amputation?
5. What are the risk factors for phantom limb pain?
6. What are the clinical symptoms and signs of phantom limb pain?
7. Describe the pathophysiology of phantom limb pain
8. How is PLP diagnosed?
9. How is phantom limb pain managed?
10. How to prevent phantom limb pain
11. How successful are these treatment modalities?
12. How does this impact a person's life?
Conclusions
References
Chapter 6 Patient with post-thoracotomy pain
Case study
1. What is the differential diagnosis?
2. What is a neuroma?
3. Describe the clinical exam and how would one evaluate the differential diagnoses?
4. How do you diagnose the conditions listed in the differential diagnosis?
5. How should one treat this condition?
6. Are there potential complications from injections?
7. What are the outcomes?
References
Chapter 7 Complex regional pain syndrome
Case study
1. What are the differential diagnoses in this case?
2. What is complex regional pain syndrome?
3. What are the classification and diagnostic criteria of CRPS?
4. How does one make the diagnosis of CRPS?
5. What is the treatment approach for CRPS?
6. What interventional methods are available to treat CRPS?
7. What is the course of CRPS?
References
Chapter 8 Diabetic neuropathy
Case study
1. How prevalent is this disease presentation? Could you explain some of the epidemiologic features of this disease? Are there any cost concerns?
2. What are some of the other conditions that could have the same presentation?
3. How would you differentiate diabetic neuropathy from some of these other disorders?
4. What diagnostic studies would you obtain?
5. How would you make a diagnosis of diabetic neuropathy?
6. What are some of the other presentations in diabetic neuropathy?
7. What are the EMG findings in diabetic neuropathy?
8. What is Seddon's and Sunderland's classification for peripheral nerve injuries?
9. What is the pathophysiology of PDPN?
10. What are some treatment modalities you would offer to this patient?
11. Disease-modifying medications
a-Lipoic acid
Protein kinase C inhibitors
Polyol pathway
Advanced glycation end products
12. Symptomatic treatment
Anticonvulsants
Antidepressants
Local anesthetics
Topical agents
Combination therapies
Interventional therapy
Spinal cord stimulation
Intrathecal medication devices
Deep brain stimulation
Surgery
Physical therapy
Psychologic treatment
Complementary and alternative medicine
New developing drugs
13. What are some of the complications from the treatment modalities?
14. The patient has been unable to return to work. How would you improve this?
15. Describe the conclusions you would draw from this case?
References
Chapter 9 Alcohol-induced neuropathy
Case study
1. What are some of the epidemiologic considerations for this disease? What is the financial burden it imposes on society?
2. What is the pathophysiology of the disease?
3. What are some other clinical conditions that may present in a similar way?
4. What are some of the clinical features of alcohol-induced neuropathy?
5. What are other tests including lab tests and imaging that may aid in your diagnostic work-up? How would you make a diagnosis of alcohol neuropathy?
6. Briefly describe the therapeutic options available to treat and manage alcohol neuropathy
Alpha-lipoic acid
Acetyl-L-carnitine
Vitamin E
Anticonvulsants
Antidepressants
Topical agents
Opioids
Physical therapy
Occupational therapy
Psychotherapy
6. Describe any treatment-related complications
7. Patient refuses to stop drinking and using marijuana. Does that impose any ethical issues?
8. What conclusions could you draw from this case?
References
Chapter 10 HIV neuropathy
Case study
1. How prevalent is this disease presentation? Could you explain some of the epidemiologic features of this disease? Are there any cost concerns?
2. What is the differential diagnosis of HIV neuropathy?
3. Describe the clinical presentations of HIV neuropathy
Distal symmetric polyneuropathy (DSP)
ARV-associated neuropathy
Progressive polyradiculopathy
Mononeuropathy multiplex
4. How would you make a diagnosis of HIV neuropathy?
Brief peripheral neuropathy screen
Total neuropathy score
5. Describe some of the laboratory tests that would aid in your diagnosis
Quantitative sensory testing
Nerve conduction studies
EMG
Nerve biopsy
Skin biopsy
Measures of small-fiber neuropathy
Utah Early Neuropathy Scale
Quantitative Sudomotor Axon Reflex Test
Autonomic function testing
6. Are there any imaging tests that would help in the diagnosis?
7. What is the pathophysiology of this disease?
8. What treatment modalities exist currently for this condition?
Pharmacologic treatment
Anticonvulsants
Antidepressants
Topical agents
Opioids
Smoked cannabis
Antiretroviral therapy
Nerve growth factor
Prosapeptide
What conclusions can be made for the treatment of HIV-associated neuropathy?
Complementary and alternative medicine
Psychotherapy/cognitive behavioral therapy
Interventional techniques
9. Are there any special concerns that you might have in the care of patients with HIV neuropathy?
10. What conclusions would you draw from this case?
References
Section 2 Spinal Disorders
Chapter 11 Cervicogenic headache
Case study
1. What is the differential diagnosis?
2. What are the differentiating features of the cervical spine pain syndromes?
3. What are some of the legal ramifications encountered with workplace and motor vehicle injuries?
4. What are the differing roles of clinical assessment between a clinical practitioner and independent medical assessment?
5. How do clinical assessments differ among practitioners in the diagnosis of cervicogenic headache?
6. What symptoms can be associated with cervicogenic headaches?
7. What are some of the different subsets in cervicogenic headache?
8. What are the available treatments for cervicogenic headache and occipital neuralgia?
References
Chapter 12 Cervical stenosis and myelopathy
Case study
1. Cervical x-rays taken 6 weeks earlier reveal
2. Assessment
3. Plan
4. MRI cervical spine following encounter reveals
5. What is the differential diagnosis?
6. What is the definition of cervical spondylotic myelopathy?
7. What is the epidemiology of CSM?
8. What are the clinical manifestations of CSM?
9. What are the physical examination findings in a patient with CSM?
10. What are the imaging studies available to evaluate for CSM?
11. What radiologic criteria are used to diagnose stenosis?
12. What additional studies may be of benefit in the evaluation of CSM?
13. What is the natural history of patients with CSM?
14. What are the risk factors associated with the development of CSM?
15. What is the pathology behind the development of CSM?
16. How is CSM managed non-operatively?
17. What are the surgical options for management of CSM?
18. How does ossification of posterior longitudinal ligament cause CSM?
19. How does rheumatoid arthritis cause CSM?
20. What is the expected neurologic outcome after decompression surgery?
Cited References
References
Chapter 13 Thoracic outlet syndrome (TOS): an enigma in pain medicine
Case study
1. What is thoracic outlet syndrome?
2. What are the types of thoracic outlet syndrome?
3. What is the epidemiology of TOS?
4. What is thoracic outlet?
5. What is the clinical classification of the brachial plexus?
6. Discuss the etiology of TOS
7. Discuss and differentiate clinical presentations by the type of TOS
8. Discuss the differential diagnosis of TOS
9. What is double crush syndrome in TOS?
10. Discuss the relevant physical examination findings in TOS
11. What are the diagnostic modalities of TOS?
12. What are the conservative management strategies of TOS?
13. Discuss surgical management strategies of TOS
14. What are the treatment outcomes for TOS?
15. What could be the impact of TOS on a person's life?
Conclusion
References
Chapter 14 Patient with cervical radiculopathy
Case description
1. Provide a differential diagnosis for the patient´s complaint
2. Who is affected by cervical radiculopathy?
3. Describe the likely anatomical causes of cervical radiculopathy and the pathophysiologic mechanisms for the observed symptoms
4. What factors from the history and physical examination support the diagnosis?
5. What is the value of diagnostic testing and imaging in evaluating a patient with cervical radiculopathy?
6. What is the efficacy of non-surgical treatment strategies?
7. Describe the benefits and risks of interventional strategies to treat cervical radiculopathy
8. Discuss the role for a surgical consultation
9. Compare the anticipated outcomes with conservative, interventional, and surgical approaches
References
Chapter 15 Patient with axial neck pain
Case study
1. What is the differential diagnosis?
2. What is the mechanism of injury in this patient?
3. Why is this condition occasionally misdiagnosed?
4. Describe the anatomy and pathophysiology of the cervical facet (zygopophysial) joints
5. How to diagnose cervical axial pain?
6. What are the treatment options for axial neck pain?
7. Procedural description
8. What are the possible complications from these procedures?
9. What are the outcomes with facet joint procedures?
10. Intradiscal procedures
11. What are the possible complications from a cervical intradiscal procedure?
References
Chapter 16 Patient with thoracic spine pain
Case study
1. What are the anatomical structures that can produce pain in the thoracic region?
2. What are the key features in the differential diagnosis of thoracic pain?
3. What is ankylosing spondylitis?
4. What is Tietze syndrome?
5. What is DISH?
6. What is Scheuermann's kyphosis? Describe the end plate changes of this disease
7. What is posterior rami syndrome (dorsal ramus syndrome, Ma
8. How would you manage patients with thoracic pain?
Conservative approaches
Interventional approaches
References
Chapter 17 Patient with lumbar disc herniation
Case study
1. What is the differential diagnosis?
2. What is a lumbar disc herniation?
3. Describe the pathogenesis of lumbar radiculopathy secondary to an LDH
4. Discuss clinical symptoms, signs, imaging, and diagnostic testing for an LDH
5. Discuss conservative treatment options
6. Discuss surgical approaches to the lumbar disc herniation
References
Chapter 18 Patient with lumbar facet-mediated pain
Case study
1. What is the differential diagnosis?
2. What is the mechanism of pain generation in this patient?
3. What is the difference between the axial back pain and radicular pain?
4. Describe the anatomy and pathophysiology of the lumbar facet (zygapophysial) joint pain?
5. How to diagnose lumbar axial pain?
6. What are the treatment options for axial low back pain due to the facets?
Conservative approaches
Interventional treatments
7. Procedural description
8. What are the possible complications from these procedures?
9. What are the outcomes with facet joint procedures?
References
Chapter 19 Discogenic pain in the setting of lumbar spondylosis
Case study
1. What is the differential diagnosis?
2. What is the anatomy of a healthy intervertebral disc?
3. What risk factors predispose patients to develop degenerative disc disease?
4. What is the pathophysiology of degenerative disc disease?
5. What is the role of radiographic imaging in discogenic pain?
6. Is there a gold standard for the diagnosis of discogenic pain?
7. Does the spread of the contrast on a discogram tell us anything?
8. What are the complications of discography?
9. What treatment modalities are available for discogenic pain?
References
Chapter 20 Unusual pain syndromes: epidural lipomatosis
Case study
1. What is the differential diagnosis?
2. What are the most common symptoms of idiopathic epidural lipomatosis?
3. What is a proper approach to these symptoms?
4. What are the findings during a physical examination in epidural lipomatosis?
5. What is the pathophysiology of idiopathic epidural lipomatosis?
6. What is the most reliable diagnostic modality?
7. How should this patient be treated?
8. What is the long-term outcome after treatment?
Summary
References
Chapter 21 Unusual pain syndromes: Bertolotti´s syndrome
Case study
1. What is Bertolotti's syndrome?
2. Describe the pathophysiology of Bertolotti's syndrome
3. What is Castellvi's classification?
4. How does Bertolotti's syndrome manifest clinically?
5. How do you diagnose a Bertolotti's syndrome?
6. How should I treat this patient?
7. What are the outcomes?
References
Chapter 22 Unusual pain syndromes: Baastrup's disease/interspinous bursitis
Case study
1. What is Baastrup's disease?
2. Describe the epidemiology of Baastrup's disease
3. Describe the anatomy and pathophysiology of Baastrup's disease
4. How to diagnose Baastrup's disease?
5. What are the differential diagnoses of Baastrup's disease?
6. How should you treat a patient with BD/ISB?
References
Chapter 23 Lumbar spinal stenosis and neurogenic claudication
Case study
1. What is spinal stenosis?
2. What is the Verbiest syndrome?
3. What is the natural history of spinal stenosis?
4. What is the anatomical basis of spinal stenosis?
5. How is lumbar spinal stenosis classified radiologically?
6. How is the diagnosis of spinal stenosis made?
7. What are the differential diagnoses of lumbar spinal stenosis?
a. Lumbosacral radicular pain secondary to nerve root impingement
b. Referred pain from adjacent anatomic structures
c. Lumbar vertebral compression fracture
d. Intermittent claudication secondary to peripheral vascular disease
e. Peripheral neuropathy
f. Visceral referred pain
g. Other differential diagnoses
8. Are conservative treatments effective in lumbar spinal stenosis?
9. What is the next step if simple conservative options are not effective?
10. Neuroplasty therapy (adhesiolysis)
11. Minimally invasive lumbar decompression
12. Interspinous spacers
13. How effective is decompression for the treatment of LSS?
14. How would you manage this patient?
References
Chapter 24 Management of the patient with postlaminectomy pain syndrome
Case study
1. What is postlaminectomy pain syndrome?
2. What is the etiology of postlaminectomy pain syndrome?
3. How is postlaminectomy pain syndrome diagnosed?
4. What interventional treatments may be utilized?
Lumbar epidural steroid injections and PLPS
Caudal epidural injection and PLPS
Facet and medial branch interventions
Adhesiolysis
Spinal cord stimulation
Pharmacologic management
Intrathecal drug delivery
Interdisciplinary management
5. Complications/Conclusions
References
Chapter 25 A patient with a lumbar compression fracture
Case study
1. What is the differential diagnosis?
2. What is the difference between a simple vertebral compression fracture and a burst fracture?
3. What is the pathogenesis and pathophysiology of a vertebral compression fracture? Is osteoporosis a factor?
4. How is vertebral compression fracture diagnosed?
5. How are vertebral compression fractures treated? Describe a conservative and an interventional treatment
6. What is the controversy surrounding vertebral augmentation?
References
Chapter 26 Sacroiliac joint pain and arthritis
Case study
1. What are the differential diagnoses in this patient?
2. What are the referred pain patterns that are important in the differential diagnosis?
3. What anatomic considerations help to explain the variable presentation of SI joint pain?
4. What are the physical exam findings that would suggest SI joint pain?
5. Does imaging play a large role in the diagnostic work-up of this patient?
6. How is the diagnosis of SI joint pain made?
7. What are the conservative management options in patients with SI joint pain?
8. When considering SI joint injection, what factors should be kept in mind?
9. Is there a long-term therapeutic solution to SI joint pain?
References
Uncited References
Chapter 27 Sacral insufficiency fracture and treatment options
Case study
1. What is the differential diagnosis?
2. What risk factors predispose patients to develop sacral insufficiency fractures?
3. Why is this condition overlooked?
4. Describe the anatomy and pathophysiology of a SIF
5. How do you diagnose a SIF?
6. Is there any other diagnostic testing that should be done?
7. How should I treat this patient?
Conservative approaches
Procedural
8. Are there any complications to worry about following sacroplasty or sacral kyphoplasty?
9. What are the outcomes?
References
Chapter 28 Skeletal metastases and treatment options
Case study
1. What is the differential diagnosis?
2. What is this?
3. How do you diagnose this?
4. Why is treatment or palliation of this lesion important?
5. The patient presented with skeletal pain shortly after being diagnosed with recurrent bronchial stump cancer. Why is this?
6. How would you treat this pain?
7. Is there a role for targeted therapy in this population?
8. What types of targeted therapy for skeletal metastases are available?
9. What is the mechanism of rapid pain relief?
10. Are there any complications to worry about following vertebroplasty or kyphoplasty?
11. What can one conclude?
References
Chapter 29 Fibromyalgia and opioid-induced hyperalgesia
Case study
1. What is the differential diagnosis?
2. What is fibromyalgia? How do you diagnose it? Are there diagnostic and/or clinical criteria?
3. How do you differentially diagnose fibromyalgia from similar problems?
4. What is the treatment for fibromyalgia? Is it ethical to prescribe opioids in the treatment of fibromyalgia?
Case follow-up
5. Does the worsening of her pain despite opioid therapy make you question your diagnosis? Are there any other diagnoses to consider at this point?
References
Section 3 Musculoskeletal Pain
Chapter 30 Patient with myofascial pain syndrome: focus on functional restoration
Case study
1. What happens when an employee files a claim through workers' compensation?
2. Why do some patients have an attorney?
3. What are the different types of disability benefits and how is candidacy for these benefits determined?
4. What are pharmacologic options for our patient going forward?
Medications
5. Are there interventional options for our patient?
6. What type of physical therapy is best?
7. Is psychologic therapy necessary for this patient?
8. What is a functional restoration program?
References
Chapter 31 Spinal manipulation, osteopathic manipulative treatment, and spasticity
Case study
1. Introduction to spinal manipulation
2. Contraindications to spinal manipulation
3. Current literature considerations
4. Introduction to spasticity
5. Spasticity management
Disclosure
References
Chapter 32 Patient with ankle pain
Case study
1. What is the differential diagnosis?
2. What risk factors predispose patients to have a chronic ankle sprain?
3. Why is this condition overlooked?
4. Describe the anatomy and pathophysiology of a chronic ankle sprain
5. How do you diagnose a chronic ankle sprain?
6. How should I treat this patient?
Conservative approaches
Procedural
Corticosteroid injections
Viscosupplementation/hyaluronic acid injections
Acupuncture
Dry-needling
Radiofrequency ablation/radiofrequency coblation
Regenerative injection procedures
Prolotherapy
Platelet-based procedures
Surgical treatment options
Open surgical repair for chronic ankle instability/hypermobility
Open surgical repair for chronic ankle pain, OA/other causes
References
Chapter 33 Patient with lateral epicondylosis or other focal tendinopathy
Case study
1. What is the differential diagnosis?
2. What risk factors predispose patients to developing sacral insufficiency fractures?
3. Describe the anatomy and pathophysiology of a lateral epicondylosis
4. How do you diagnose a lateral epicondylosis?
5. Is there any other diagnostic testing that should be done?
6. How should I treat this patient?
Conservative approaches
Procedural
References
Chapter 34 Knee osteoarthritis with emphasis on percutaneous regenerative medicine
Case study
1. What are the most common etiologies of chronic knee pain?
2. What is knee osteoarthritis and what is the pathophysiologic basis for its development?
3. What are the common causes of KOA?
4. What other type(s) of arthropathy need to be ruled out and how do they differ from OA?
5. What does a pertinent exam of the knee consist of in a patient that is suspected of having knee osteoarthritis?
6. What imaging is currently used to diagnose KOA and is it adequate?
7. Are there any labs that need to be ordered when working up a patient with knee pain?
8. What are the traditional non-surgical treatment options for early KOA?
Diet and exercise
Physical therapy
Pharmacologic intervention
Orthotics
Psychology therapy
Traditional percutaneous injections
9. What are the innovative non-surgical management options for early KOA?
10. What are the traditional surgical treatment options for early KOA?
11. What are innovative surgical treatment options for KOA?
12. Coalescence of surgery and injections
References
Section 4 Visceral Pain
Chapter 35 Patient with chronic abdominal pain from pancreatitis
Case study
1. What is the difference between acute and chronic pancreatitis?
2. What is the most common cause of chronic pancreatitis and which demographics are usually affected?
3. Describe the typical presentation of pain in chronic pancreatitis
4. What is the pathogenesis of pain in chronic pancreatitis?
5. Which other clinical manifestations may contribute to the overall pain experience of patients?
6. Which diagnostic studies are of value for diagnosis and staging of chronic pancreatitis?
7. What is the first step in treatment of pain in chronic pancreatitis?
8. Which are the initial steps in conservative management of chronic pancreatitis?
9. What are the medical treatment options available for treatment of chronic pancreatitis?
10. Can COX-2 inhibitors be given for long-term pain management in chronic pancreatitis?
11. Which are the most common adjuvants used in medical treatment of chronic pancreatitis?
12. What are the indications for the use of opioids in chronic pancreatitis?
13. What is the reasoning behind the use of antioxidants in the treatment of chronic pancreatitis?
14. When are interventional options indicated and which procedures can be used?
15. What is the evidence behind the use of radiofrequency ablation and spinal cord stimulation for treatment of pain in chronic pancreatitis?
16. What are the endoscopic and surgical treatment options available?
References
Chapter 36 Patient with chronic pelvic pain from endometrial fibrosis
Case study
1. What is chronic pelvic pain?
2. Describe the epidemiology and prevalence of endometriosis
3. What is the pathophysiology of endometriosis?
4. What is the mechanism of pain associated with endometriosis?
5. How is endometriosis presented clinically?
What is the differential diagnosis of chronic pelvic pain in females?[17]
How to establish the diagnosis of CPP due to endometriosis?
How to manage CPP due to endometriosis?
Treatment of endometriosis
Hormonal treatments
Surgical treatment
Managing CPP associated with endometriosis
Medical management
Interventional and surgical management
References
Chapter 37 Patient (male) with chronic pelvic pain from interstitial cystitis
Case study
1. What is the innervation of the male pelvis?
2. What is the differential diagnosis for a male presenting with chronic pelvic pain?
3. How are prostate syndromes classified? What is the epidemiology of chronic prostatitis?
4. How should male CPP be evaluated?
History
Physical exam
Laboratory
Additional studies
5. What non-pharmacologic treatment options are available to patients with CPPS?
6. What pharmacologic agents are used in the treatment of CPPS?
7. What interventional procedures can be performed in the treatment of CPPS?
8. How are the iliohypogastric, ilioinguinal, genitofemoral, and pudendal nerve blocks performed?
Iliohypogastric and ilioinguinal nerve block
Genitofemoral nerve block
Pudendal nerve block
Superior hypogastric plexus block
9. What other treatment options are available if pharmacotherapy and targeted nerve blocks have failed to provide adequate pain relief?
Neuromodulation
Intrathecal drug therapy
10. How do you monitor the progress of treatment in CPPS?
11. What is the treatment course in CPPS?
References
Chapter 38 Chronic rectal pain
Case study
1. What is the differential diagnosis for rectal pain?
2. Describe the neuroanatomy of the pelvic viscera including the rectum
3. What are some unique features of rectal innervation and how do they contribute to rectal pain?
4. What is the typical work-up for rectal pain?
5. What are the pathophysiology and diagnostic criteria of coccygodynia?
6. What are the pathophysiology and diagnostic criteria of proctalgia fugax?
7. What are the pathophysiology and diagnostic criteria of radiation proctitis?
8. What are some treatment modalities of rectal pain?
9. What are some topical medications used to treat rectal pain?
10. When is the ganglion impar block indicated and how is it performed?
11. When is the pudendal nerve block indicated and how is it performed?
12. When is the inferior hypogastric plexus block indicated and how is it performed?
13. How is BOTOX used to treat rectal pain?
14. How can neuromodulation be used to help manage rectal pain?
References
Chapter 39 Pain in pregnancy
Case study
1. What are the common etiologies for pain in the pregnant patient?
Low back pain and disc disease
Pelvic girdle pain
Pubic symphysis separation (pubic diastasis, symphysiolysis, osteitis pubis)
Hip pain and transient osteoporosis of the hip
Knee pain and patellofemoral disorder
Leg cramps
Abdominal wall pain
2. How does the evaluation for low back pain compare from the parturient to the non-parturient?
3. What pharmacotherapy for pain is safe to give in pregnancy?
4. What role do non-pharmacologic therapies have in treatment for pain in pregnancy?
Exercise and physical therapy
Acupuncture
Pelvic belt and pillow
Transcutaneous electronic nerve stimulation
Osteopathic manipulative treatment
5. What role do non-opioid medications have in treating pain in pregnancy?
Acetaminophen
NSAIDs
Benzodiazepines
Antidepressants
Anticonvulsants
Local anesthetics
Skeletal muscle relaxants
Steroids
6. What role do opioids have in treating pain in pregnancy? If started on opioids, what is the risk to the mother and the fetus, both in gestation and the peripartum stage?
7. What interventions are available to the parturient? What are the risks associated with pain procedures for the parturient and the fetus?
8. What would be the suggested overall therapeutic management for the parturient with lower back pain?
References
Chapter 40 Postpartum pain
Case study
1. What are common etiologies of back pain in a postpartum patient?
2. What is the correlation, if any, between neuraxial analgesia and chronic back pain?
3. What other pain syndromes are common or unique to the postpartum female?
Chronic pelvic girdle pain
Meralgia paresthetica
Chronic pelvic pain
Chronic pain after cesarean delivery
Chronic perineal pain
4. What are common risk factors for chronic postpartum pain?
Pre-delivery
During delivery
Post-delivery
5. How should a patient with postpartum pain be evaluated?
Targeted pain history
Targeted physical examination
Imaging and other diagnostic modalities
6. What non-pharmacologic treatment options for postpartum pain are available in a breastfeeding female?
Physical therapy
Pelvic belts
Acupuncture
TENS
Biofeedback and relaxation training
7. What pharmacologic agents are available for treatment of postpartum pain in a breastfeeding female?
Acetaminophen
NSAIDs
Local anesthetics
Steroids
Antidepressants
Benzodiazepines
Opioids
8. What medications should be used with caution in the breastfeeding mother?
Gabapentin
Pregabalin
Antidepressants
Lithium
9. What guidelines can a physician follow in management of postpartum pain in a breastfeeding mother?
References
Section 5 Headaches and Facial Pain
Chapter 41 Patient with migraine headaches
Case study
1. Discuss the epidemiology of migraine
2. Discuss the pathogenesis of migraine
3. Clinical assessment and associated signs
International Headache Society Diagnosis Criteria for Migraine
Migraine without aura (MO) diagnostic criteria[10]
Migraine with aura (MA) diagnostic criteria[10]
4. Counseling on long-term risks of migraine
Stroke
5. Diagnostic evaluation
6. Medications: triptans
7. Medications: other non-opioid abortive treatments
8. Medications: opioids and butalbital-containing combination pills
Beta-blockers (e.g., propranolol, atenolol, nadolol, timolol)
Anticonvulsants (e.g., valproate, topiramate, gabapentin)
Calcium channel blockers (i.e., verapamil, amlodipine)
Tricyclic antidepressants (i.e., amitriptyline, nortriptyline, desipramine)
Selective SNRIs (i.e., duloxetine, milnacipram)
Treatment of chronic migraine
9. Functional measures for outcomes: headache diary
10. Interventional approaches: BOTOX, peripheral nerve blocks, peripheral nerve stimulation (including occipital nerve stimulation and supraorbital nerve stimulation)
11. Preventive measures: physical therapy, modalities, and nutrition
12. Thoughts from an expert, e.g., overlooked areas
References
Chapter 42 Patient with cluster headache
Case study
1. What is the epidemiology and clinical presentation of cluster headache?
2. How is cluster headache diagnosed?
3. What are the diagnostic criteria for cluster headache?
4. What are the diagnostic pitfalls?
5. What is the differential diagnosis?
6. Does the specific diagnosis matter?
7. What risk factors predispose patients to develop cluster headaches?
8. Why is this condition overlooked?
9. What is the anatomy and pathophysiology of a cluster headache?
10. Is there any other diagnostic testing that should be done?
11. How is cluster headache treated?
Pharmacotherapy
Acute treatment
Preventive treatments
Procedural
Occipital nerve block
Neurostimulation
12. How should I treat this patient?
13. Are there any complications to worry about using DHE (dihydroergotamine) or triptans for acute treatment of cluster headache?
14. What are the outcomes of treatment of cluster headache patients?
References
Chapter 43 Patients with tension headaches
Case study
1. What is the differential diagnosis?
2. What is the epidemiology of these headaches?
3. Describe the clinical presentation
4. What is the pathophysiology of tension-type headache?
5. Treatment
6. Conclusion
References
Chapter 44 Pain management in trigeminal neuralgia: clinical case illustrations
Introduction
Clinical cases
1. What are the history, incidence, and epidemiology of trigeminal neuralgia?
2. What are the features of trigeminal neuralgia?
3. What is the etiology and pathophysiology of trigeminal neuralgia?
4. How is a diagnosis of TN made?
5. What are the distinct management categories for trigeminal neuralgia?
6. What is microvascular decompression for trigeminal neuralgia?
7. What are some of the advantages of microvascular decompression for TN?
8. Are there any other alternative techniques for trigeminal neuralgia?
9. What are the key points regarding trigeminal neuralgia for the clinician?
Disclosure
References
Chapter 45 Patient with chronic glossopharyngeal neuralgia/post-tonsillectomy pain
Case study
1. What is glossopharyngeal neuralgia?
2. Describe the signs and symptoms
3. Discuss the etiology and pathophysiology
4. Diagnosis
5. Treatment
References
Chapter 46 Patient with sphenopalatine neuralgia
Case study
1. What is the differential diagnosis?
2. What is Sluder neuralgia or sphenopalatine ganglion neuralgia?
3. What is the anatomy of the sphenopalatine ganglion?
4. What are the major nerve structures involved with the sphenopalatine ganglion?
5. What is the proposed etiology of sphenopalatine neuralgia?
6. What is the clinical presentation of sphenopalatine neuralgia?
7. What is the recommended work-up for suspected sphenopalatine neuralgia?
8. What is the recommended medical management of sphenopalatine neuralgia?
9. What are the interventional therapies for treatment of sphenopalatine neuralgia?
10. What are the complications of the sphenopalatine block?
Summary
References
Chapter 47 Temporomandibular joint disorders
Case study
1. What is temporomandibular joint disorder (TMD) and how does it typically present?
2. What are the relevant anatomical structures related to TMD?
3. What is the cause of pain in TMDs?
4. How common is TMD?
5. What would be included in the differential diagnosis?
6. How is TMD evaluated and diagnosed?
7. What are the potential treatments for TMD?
Conclusion
References
Section 6 Cancer Pain
Chapter 48 Cancer pain
Case study
1. What is the etiology of cancer pain?
2. Could there be other non-cancer causes for the patient's pain?
3. What is the epidemiology of cancer pain?
4. Are there any risk factors for the development of cancer pain?
5. What analgesic treatments are available to relieve this cancer pain?
6. What is the World Health Organization Analgesic Ladder for the treatment of cancer pain?
7. What analgesic medications should I use to treat this cancer pain?
Non-steroidal anti-inflammatory drugs and acetaminophen
Adjuvant analgesics
Opioids
8. What if our patient has intermittent pain in spite of regular opioid therapy?
9. My patient is unable to take oral medications. What should I now do?
10. What opioid side effects are common in the chronic treatment of cancer pain?
11. Are there any non-drug therapies for cancer pain management?
12. Would a "nerve block" be helpful for this patient?
13. What is involved with a celiac plexus neurolytic block?
Conclusions
References
Chapter 49 Patient presents with pancreatic cancer with persistent pain despite all other treatments
Case study
1. What barriers to patient care might an interventional pain physician face in cross-disciplinary care?
2. What are common sources of pain in the pancreatic cancer patient?
3. What interventional techniques may be used?
Blocks
Spinal cord stimulation
Neuraxial drug administration
4. What are the complications/contraindications to neuraxial analgesia?
References
Chapter 50 Pain management in hematological cancer: clinical case illustrations
Case studies
1. Introduction
2. Discussion
Conclusion
References
Chapter 51 Patient with metastatic breast cancer who had a mastectomy complicated by lymphedema
Case study
1. What is the prevalence of upper-body morbidity and lymphedema after breast cancer?
2. What are known risk factors for the development or exacerbation of upper-body morbidity and lymphedema after breast cancer?
3. What is the differential diagnosis for pain syndromes after breast cancer?
4. What are some clinical manifestations of lymphedema after breast cancer?
5. How is lymphedema diagnosed?
6. For patients at risk for lymphedema who do not have the condition, how should they be educated regarding preventive care?
7. What are the components of the "gold standard" treatment regimen for established lymphedema?
8. What other treatments can be considered for lymphedema?
Intermittent external pneumatic compression
Weight loss
Low-level laser therapy
Surgery
9. What rare complications of lymphedema should practitioners be aware of?
10. Conclusions - how would you treat this patient in the vignette?
References
Section 7 Special Topics
Chapter 52 A 57-year-old male with chronic pain syndrome, anxiety disorder, and hypertension is seeking mental health counseling
Why is it important to address the risk of suicide?
The patient doesn't demonstrate any suicidal thoughts, has a good support system at home, and doesn't have any concrete executable plans for self harm. You now feel comfortable discussing more conventional mental health strategies. What do you recommend?
Cognitive behavioral therapy
Relaxation training
Psychoanalysis
Psychiatric approach
During this discussion, the patient presents a newspaper article touting the benefits of biofeedback. He says that one of his friends mentioned hypnosis. Can you explain these modalities to the patient?
What is biofeedback?
What is hypnosis?
Inpatient versus outpatient psychologic therapy?
References
Chapter 53 Pediatric, infant, and fetal pain
Case study 1
1. Would this fetus require analgesia?
2. How can anesthetics/analgesics be delivered to a fetus?
Case study 2
1. How would you assess pain in this patient?
2. How would you treat this infant´s pain?
Case study 3
1. How would you assess this patient´s pain?
2. How would you manage her pain?
Case study 4
1. What is your differential diagnosis?
2. How does complex regional pain type 1 present in children?
3. How would you treat this patient?
Case study 5
1. Is this patient's presentation consistent with functional abdominal pain? Are there any concerning features in this patient's presentation?
2. What diagnostic studies are indicated in this patient?
3. How is FAP treated in children?
References
Chapter 54 Patient with hearing impairment and chronic pain
Case study
1. What should the provider be aware of when communicating with the hearing impaired?
2. Why use sign language?
3. How do you evaluate the deaf pain patient?
References
Chapter 55 Complementary and alternative medicine
Case study
1. How effective are conventional therapies for chronic low back pain?
2. What is complementary and alternative medicine, and who uses it?
3. Discuss the effectiveness of CAM therapies for low back pain
Acupuncture
Massage
Meditation
Autogenic training
Progressive muscle relaxation
Tai chi
Manipulation
Yoga
Biofield therapies
Balneotherapy
Behavioral therapies
Prolotherapy
Neuroreflexotherapy
Herbal therapy
4. What is integrative medicine?
5. What information would you discuss with this patient seeking to use complementary and alternative medicine?
References
Chapter 56 Ethical issues in the substance abusing pain patient
Case study
1. What is the scope of the public health problem related to the use of prescription opioids for pain?
2. What are the differential diagnoses of aberrant opioid-taking behaviors?
3. What are some common terminologies related to drug abuse and addiction?
4. Describe the basic neurobiologic process underlying addiction
5. What are the ethical principles that guide the use of opioids in pain control?
6. How can opioids be safely used for chronic pain?
7. Considering the complexity and risks, why not just avoid offering opioid therapy altogether?
References
Chapter 57 Approach to the patient with abnormal drug screen
Case study 1
1. Clinical assessment
What is the differential diagnosis for a patient with a positive drug screen?
What information exists regarding the epidemiology of drug abuse in the chronic pain population?
What are the clinical implications of drug misuse in this patient population?
2. Overview of drug testing
What modalities and assays are available for clinical drug testing?
What limitations should be considered when interpreting urine drug test results?
What guidelines exist regarding drug testing in the clinical setting?
3. Clinical management
How would the results of this urine drug test affect your future management of this patient?
Case study 2
1. Clinical assessement
What are this patient's risk factors for illicit substance abuse?
What is the differential interpretation of these results?
2. Illicit drug abuse and detection
What are the clinical trends and implications of substance abuse?
What is the time frame for detection of marijuana in urine drug testing? What is the time frame for detection of other drugs with abuse potential?
Would you include marijuana in your routine clinical drug screening? Why or why not?
3. Clinical management
What are the next steps in your management of this patient?
Case study 3
1. Clinical assessment
What is the differential interpretation of these findings?
What steps would you take to determine a diagnosis?
2. Clinical management
What are your next steps in management of this patient?
References
Chapter 58 Physician exposed to excessive radiation
Interventional pain physician has to go to occupational medicine office regarding radiation exposure
1. What is the reason for the state inspection?
2. What are the OSHA standards regarding ionizing radiation?
3. Who sets and enforces the medical radiation dosage standards?
4. What are the penetrating characteristics of various radiation beams?
5. What are the biologic effects of ionizing radiation?
6. What are the maximum permissible doses (MPD) of radiation?
7. What can I do to reduce my exposure to radiation while performing the fluoroscopic procedures?
8. What are the recommendations by US Environmental Protection Agency regarding radiation exposure?
Summary
References
Chapter 59 Patient becomes paralyzed following a lumbar transforaminal epidural steroid injection
Case study
1. What are the possible complications after a transforaminal epidural steroid injection?
2. What is the vascular anatomy of the blood supply to the spinal cord?
3. Where is the location of the arteries in the foramen?
4. What is the pathogenesis and mechanism of injury leading to paraplegia?
5. What changes in technique have been proposed to improve the safety of TFESI?
6. What are the pros and cons of the suggested technique modifications?
7. What is the treatment of paraplegia secondary to infarction?
8. What ethical and legal ramifications are there surrounding this procedure and the possible complications?
References
Chapter 60 Postepidural steroid injection paraplegia
Case study
1. What is the differential diagnosis?
2. Review the anatomy and vasculature of the epidural space
3. If the patient developed lower extremity paralysis immediately following the epidural steroid injection, which diagnosis would be highest on the differential?
4. If the patient underwent a transforaminal epidural steroid injection, describe the most favorable needle placement to minimize the risk of these serious postinjection symptoms?
5. Which of the following symptoms should be the most worrisome for the clinician?
6. What safety measures could the physician have taken to try to prevent such complications secondary to the epidural steroid injection?
7. What is the most reliable way to diagnose an epidural hematoma?
8. How should the clinician interpret the results, depending on when the modality was utilized following symptom onset?
9. What is the treatment for an epidural hematoma?
Acute management
Chronic management
10. What is the role of intravenous steroids in acute spinal cord injury?
References
Chapter 61 Complications: patient with dural puncture following cervical interlaminar epidural steroid injection
Case study
1. What is the differential diagnosis?
2. What are the signs and symptoms of a postdural puncture headache?
3. Describe the anatomy and pathophysiology for postdural puncture headaches
4. How is the diagnosis of a postdural puncture headache made in this patient?
5. What are the non-interventional treatments for a postdural puncture headache?
6. What are the interventional treatments for a postdural headache and how are they performed?
7. What are the factors to avoid a dural puncture?
8. What are the complications from a dural puncture?
References
Chapter 62 Complications: a patient with serotonin syndrome
Case study
What is serotonin syndrome?
Which drugs have been implicated in SS?
How is SS diagnosed?
What is the differential diagnosis?
References
Chapter 63 Office-based buprenorphine to wean patients off opioids
Case study
1. How common is opioid dependence?
2. What is the best approach to address opioid dependence in the USA?
3. What are the risk factors for opioid addiction?
4. How do you diagnose opioid dependence?
5. What are some of the features that make buprenorphine a desirable option for the treatment of opioid addiction?
6. What are the pharmacokinetics and pharmacodynamics of buprenorphine and the best form of administration?
7. Is dose adjustment required for buprenorphine in the presence of hepatic and renal disease?
8. What forms of buprenorphine are available for treatment of opioid dependence?
9. Why is buprenorphine not used via oral route?
10. How can I get certified to prescribe buprenorphine for this indication?
11. How should I start this patient on buprenorphine?
12. What is the standard induction protocol?
13. What is the safety of using buprenorphine in the treatment of opioid dependence?
14. Can you use buprenorphine for pain control?
15. What are the clinical outcomes using buprenorphine?
Summary
References
Chapter 64 Patient on chronic opioids who wants to have anesthesia-assisted detoxification
Case study
1. What are the various terms related to addiction and withdrawal?
2. What are the indications for anesthesia-assisted opiate detoxification (AAOD)?
3. What are the contraindications to anesthesia-assisted opiate detoxification?
Contraindications include:
4. What pre-assessment is needed for anesthesia-assisted detoxification?
Pre-anesthetic testing
4. What is the actual methodology to perform AAOD?
Premedication
Monitoring
Induction and maintenance
4. How do you know when the detoxification is complete?
What post-procedure monitoring is required?
5. What complications can occur in AAOD?
6. How does AAOD compare to traditional methods of detoxification?
7. How can the opiate-dependent patient maintain abstinent state?
8. Does AAOD actually lead to prolonged abstinence?
9. What evidence-based reviews have been conducted of AAOD?
10. Are there potential benefits of AAOD?
11. Should this procedure be conducted at all?
12. What ethical conundrums occur with AAOD?
13. What is the role of clonidine in mitigating withdrawal?
References
Chapter 65 Munchausen syndrome and pain
Case study
What is the origin and history of this condition?
How is this diagnosed?
What are the treatment options?
References
Chapter 66 Insomnia and chronic pain
Case study
1. How common are sleep disorders among chronic pain patients?
2. What type of sleep disorder is this patient struggling with?
3. What information is needed to assist in the diagnosis and treatment of this sleep problem?
Interview
Polysomnography
Sleep diary
Psychometric tools
4. Are there any drugs that may be contributing to his sleep problem?
5. What are some initial recommendations you might make to address sleep difficulties?
6. Are there any medications you might consider to assist with his sleep?
Prescription
Non-prescription
7. What other non-pharmacologic treatments are available?
8. Conclusions
References
Chapter 67 Opioid-induced constipation
Case study
1. What is the differential diagnosis?
2. What is the prevalence of opioid-induced constipation?
3. Describe the relevant anatomy of the bowel as it relates to opioid-induced constipation
4. Describe the opioid receptors in the bowel and its relation to the physiology of the bowel
5. Describe the clinical presentation of opioid-induced constipation
6. What is the role of the patient´s history in determining constipation caused by opioid use?
7. What is the benefit of the physical examination in a patient with opioid-induced constipation?
8. Is there a role for imaging in evaluating opioid-induced constipation?
9. How important is fluid intake for treatment of opioid-induced constipation?
10. What is the roll of diet and fiber?
11. Are fiber-based laxatives safe for patient´s with fecal impaction?
12. Is exercise effective?
13. Why are stimulants effective in opioid-induced constipation?
14. What is the mechanism of action of docusate and is it effective for opiate-induced constipation?
15. What is the mechanism of action of osmotic laxatives such as lactulose and sorbitol?
16. What is the mechanism of action of magnesium sulfate?
17. Should suppositories and enemas be considered as first-line treatment modalities?
18. Is there a role for opioid receptor antagonists in the treatment of opioid-induced constipation?
19. What is opioid rotation?
Conclusion
References
Chapter 68 Complications: vasovagal response during pain procedures
Case study
1. What is the differential diagnosis?
2. What is a vasovagal response?
3. How does a vasovagal response occur?
4. What are the risk factors/causative factors for a vasovagal response during pain procedures?
5. What is the association of vasovagal response with pain procedures?
6. In which type of pain procedures are vasovagal responses more common?
7. How do you diagnose a vasovagal response?
8. How would you treat a patient for a vasovagal response during a pain procedure?
References
Chapter 69 Acute pain management: patient-controlled analgesia
Case study
1. What is patient-controlled analgesia?
2. How does an IV PCA work?
3. Why is an IV PCA beneficial?
4. What are some patient selection criteria for appropriate use of an IV PCA?
5. When would a continuous infusion on an IV PCA be appropriate?
6. What is the ideal opioid for an IV PCA?
7. How do I initially program a PCA in my opioid-naive patients?
8. How do patient characteristics such as age and weight affect IV PCA use?
9. What if the patient continues to have pain while on the PCA?
10. What are the side effects of an IV PCA?
11. How safe is an IV PCA?
12. What are the limitations of an IV PCA?
13. Are there ways to improve the efficacy of IV PCA use?
14. Can a PCA mask postoperative complications?
15. How does the cost of IV PCA compare to conventional dosing?
References
Chapter 70 Acute pain management: PCEA/continuous epidural catheters
Case study
1. What are the indications for placing an epidural catheter for postoperative analgesia?
2. What postoperative pain treatment options are available for patients undergoing thoracotomy?
3. What are the benefits of a thoracic epidural patient-controlled analgesia as an analgesic technique?
4. How should an epidural catheter be placed? What techniques can be used for assistance with a difficult placement?
5. What are some risks of an epidural placement?
6. What are some medications used for epidural analgesia?
7. What are the options for infusion strategies?
8. What special considerations exist for the use of neuraxial analgesia in patients with multiple sclerosis?
9. For what other procedures might epidural analgesia be beneficial?
10. What are the contraindications for placing an epidural catheter?
Reference
Chapter 71 New vistas: continuous peripheral catheters/regional anesthesia in postoperative pain management
Case study
1. What are some of the important epidemiologic considerations in TKA?
2. What is the rationale for standardizing the perioperative pain management pathway for TKA patients?
3. What is a multimodal analgesic protocol?
4. What are potential components of an effective multimodal analgesic protocol?
Local anesthetics
Opioid analgesics
Non-opioid systemic analgesics
5. What are some special considerations for patients with chronic pain and chronic opioid therapy?
6. How does the diagnosis of obstructive sleep apnea (OSA) influence the analgesic protocol?
Conflict of interest
References
Chapter 72 Methadone and treatment of chronic pain
Case study
1. What are the clinical uses of methadone?
2. What about using methadone in the chronically ill patient with HIV?
3. How does her history of substance abuse change methadone prescribing?
4. What is the cost of methadone?
5. How common is accidental overdose in methadone use?
6. What are other risks of methadone use?
7. What are drugdrug interactions of methadone?
8. What is the role of genetic testing?
9. What is the metabolism and mechanism of methadone?
10. What are the advantages of methadone in this patient?
11. What are the risks of methadone in this patient?
12. What are the ethical considerations?
13. What should be the initial approach to this patient?
References
Chapter 73 Drug testing
Case study
1. Why is it important to do drug testing in patients with chronic pain managed with opioids?
2. What methods are available for urine drug testing?
3. How accurate are the urine drug tests?
4. How long will a urine drug test remain positive for opioids, benzodiazepines, and barbiturates?
5. How long will a urine drug test remain positive for drugs of abuse?
6. This patient tells you he ran out of his medication and last took his oxycodone 2 days ago. Would you expect his urine drug test to be positive or negative?
7. Can urine drug testing tell you how much of a given drug the patient is currently taking?
8. What medications and substances can trigger a false positive urine drug test result?
9. How reliable is a physician's history or "gut instinct" in predicting aberrant drug use?
10. What are the most common inappropriate drugs found on a urine drug testing?
11. Is drug testing mandatory or required by law?
12. What would you do if the urine drug test is unexpectedly positive or unexpectedly negative?
References
Index

Citation preview

CAMBRIDGE

Case Studies in Pain Management

Case Studies in Pain Management Edited by

Alan David Kaye, MD, PhD, DABA, DABPM, DABIPP

Professor and Chairman, Department of Anesthesiology, Director of Interventional Pain Services, LSU School of Medicine, New Orleans, LA, USA

Rinoo V. Shah, MD, MBA, DABPMR, DABIPP

Interventional Pain Physician and Minimally Invasive Spine Specialist at Guthrie Clinic, Sayre, PA, USA

University Printing House, Cambridge CB2 8BS, United Kingdom Cambridge University Press is part of the University of Cambridge. It furthers the University’s mission by disseminating knowledge in the pursuit of education, learning and research at the highest international levels of excellence. www.cambridge.org Information on this title: www.cambridge.org/9781107682894 © Cambridge University Press 2015 This publication is in copyright. Subject to statutory exception and to the provisions of relevant collective licensing agreements, no reproduction of any part may take place without the written permission of Cambridge University Press. First published 2015 Printed and bound in the United Kingdom by TJ International Ltd. Padstow, Cornwall A catalog record for this publication is available from the British Library Library of Congress Cataloging in Publication data Case studies in pain management / edited by Alan David Kaye, Rinoo V. Shah. p. ; cm. Includes bibliographical references and index. ISBN 978-1-107-68289-4 (Pbk.) I. Kaye, Alan David, editor. II. Shah, Rinoo V., editor. [DNLM: 1. Pain Management–Case Reports. 2. Analgesics– therapeutic use–Case Reports. 3. Pain–etiology–Case Reports. WL 704.6] RB127 6160 .0472–dc23 2014006285 ISBN 978-1-107-68289-4 Paperback Cambridge University Press has no responsibility for the persistence or accuracy of URLs for external or third party internet websites referred to in this publication, and does not guarantee that any content on such websites is, or will remain, accurate or appropriate. ............................................................................................ Every effort has been made in preparing this book to provide accurate and up-to-date information which is in accord with accepted standards and practice at the time of publication. Although case histories are drawn from actual cases, every effort has been made to disguise the identities of the individuals involved. Nevertheless, the authors, editors and publishers can make no warranties that the information contained herein is totally free from error, not least because clinical standards are constantly changing through research and regulation. The authors, editors and publishers therefore disclaim all liability for direct or consequential damages resulting from the use of material contained in this book. Readers are strongly advised to pay careful attention to information provided by the manufacturer of any drugs or equipment that they plan to use.

To my parents: my mother Florence Feldman, the former Fania Eichenblat, who despite having a lifetime of chronic pain has never given up in her efforts to make a good life for her children and whose kindness and love I can never adequately repay and my father Joel Kaye, the former Joseph Krakower, for providing me with thousands of enlightening lessons in life and for helping to shape me into the man I am today. To my step parents: Andrea Bennett-Kaye and the late Gideon Feldman, who helped raise me, providing love, support, kindness, and wisdom over the last 30 plus years. To my wife: Dr. Kim Kaye, and my children, Aaron Kaye and Rachel Kaye, for making each day worth living and for giving me balance, support, and inspiration for all that I do in life. Alan D. Kaye, MD, PhD, DABA, DABPM, DABIPP To my parents: my mom, Rajul Shah, and my dad, Vasant Shah, for their unconditional love, patience, dedication, industriousness, kindness, and hard work. They always deserved better, but handled what they had with what gives essence to the meaning to life. I am the luckiest child in all of humanity to have them as my parents. To my wife: Dr. Kejal Shah and my Children, Maaya Shah, Diyaa Shah, and Dev-‘Dehdoo’ for their unconditional love, patience, and for being the source of my eternal happiness and drive. They provide inspiration and keep me focused not only during the good and bad times, but during the ‘meh’ times. They remind me how every second is precious in cosmological time. Rinoo V. Shah, MD, MBA, DABIPP, DABPMR Both Dr. Kaye and Dr. Shah wish to thank Dr. Gabor Racz, Dr. Prithvi Raj, Dr. Miles Day, and Dr. Leland Lou who helped train both of them in the field of pain management at Texas Tech Health Sciences Center in Lubbock, Texas

Contents List of contributors xi Foreword by Laxmaiah Manchikanti

xvi

Section 1 – Neurological Disorders

12. Cervical stenosis and myelopathy 95 Santhosh A. Thomas and Garett J. Helber

1.

Postherpetic neuralgia 1 Alan David Kaye and Charles E. Argoff

2.

Patient with spinal cord injury pain 16 Daniel Krashin, Natalia Murinova, and Alan David Kaye

13. Thoracic outlet syndrome (TOS): an enigma in pain medicine 102 Narendren Narayanasamy and Rahul Rastogi

3.

Patient with poststroke pain 22 Natalia Murinova, Claire Creutzfeldt, Daniel Krashin, and Alan David Kaye

4.

Patient with brachial plexopathy 30 Jonathan Chang and Rahul Rastogi

5.

Phantom limb pain 38 Jonathan Chang and Rahul Rastogi

6.

Patient with post-thoracotomy pain Rinoo V. Shah

7.

Complex regional pain syndrome Gaurav Jain and Nashaat N. Rizk

8.

Diabetic neuropathy 52 Gulshan Doulatram and Tilak Raj

9.

43

46

10. HIV neuropathy 72 Gulshan Doulatram, Tilak Raj, and William Yancey

11. Cervicogenic headache 81 Eric R. Helm and Nashaat N. Rizk

15. Patient with axial neck pain Vikram B. Patel

109

116

16. Patient with thoracic spine pain 123 Ankit Maheshwari and Jianguo Cheng 17. Patient with lumbar disc herniation Julian Sosner

Alcohol-induced neuropathy 64 Gulshan Doulatram, Tilak Raj, and Ankur Khosla

Section 2 – Spinal Disorders

14. Patient with cervical radiculopathy Robert B. Bolash and Jianguo Cheng

131

18. Patient with lumbar facet-mediated pain 137 Vikram B. Patel 19. Discogenic pain in the setting of lumbar spondylosis 144 James Kelly and Jianguo Cheng 20. Unusual pain syndromes: epidural lipomatosis 152 Vikram B. Patel 21. Unusual pain syndromes: Bertolotti’s syndrome 155 Jiang Wu and Jianguo Cheng 22. Unusual pain syndromes: Baastrup’s disease/interspinous bursitis 159 Jijun Xu and Jianguo Cheng 23. Lumbar spinal stenosis and neurogenic claudication 164 Ike Eriator and Zachariah Chambers

vii

Contents

24. Management of the patient with postlaminectomy pain syndrome Jay S. Grider

174

25. A patient with a lumbar compression fracture 182 Nihir Waghela and Magdalena Anitescu 26. Sacroiliac joint pain and arthritis Garrett LaSalle and Jianguo Cheng

195

Section 4 – Visceral Pain 35. Patient with chronic abdominal pain from pancreatitis 253 Rodrigo A. Benavides Corder and Jianguo Cheng 36. Patient with chronic pelvic pain from endometrial fibrosis 261 Maged Guirguis and Jianguo Cheng

27. Sacral insufficiency fracture and treatment options 202 Rinoo V. Shah

37. Patient (male) with chronic pelvic pain from interstitial cystitis 266 John Hau, Michael Truong, Eric S. Hsu, and Irene Wu

28. Skeletal metastases and treatment options 207 Rinoo V. Shah

38. Chronic rectal pain 275 Brandon A. Van Noord, Irene Wu, and Eric S. Hsu

29. Fibromyalgia and opioid-induced hyperalgesia 214 Grace Chen and Elliot Palmer

Section 3 – Musculoskeletal Pain 30. Patient with myofascial pain syndrome: focus on functional restoration 223 Tracy P. Jackson

39. Pain in pregnancy 282 Eugene Garvin, Jakun Ing, Irene Wu, and Eric S. Hsu 40. Postpartum pain 290 Jeffry Chen, Eric S. Hsu, and Irene Wu

Section 5 – Headaches and Facial Pain 41. Patient with migraine headaches 297 Natalia Murinova, Daniel Krashin, and Andrea Trescot

31. Spinal manipulation, osteopathic manipulative treatment, and spasticity 230 Monika A. Krzyzek, John P. McCallin, Justin B. Boge, Dean Hommer, Prasad Lakshminarasimhiah, Rebekah L. Nilson, and Brandon J. Goff

42. Patient with cluster headache 307 Natalia Murinova, Daniel Krashin, and Andrea Trescot

32. Patient with ankle pain 235 Jose E. Barreto and Thomas K. Bond

44. Pain management in trigeminal neuralgia: clinical case illustrations 316 Joaquin Maury, Alan David Kaye, and Harry J. Gould, III

33. Patient with lateral epicondylosis or other focal tendinopathy 240 Jose E. Barreto and Jeff Ericksen 34. Knee osteoarthritis with emphasis on percutaneous regenerative medicine 243 Jason Tucker, Christopher Centeno, and Jeff Ericksen

viii

43. Patients with tension headaches 312 Natacha Telusca, Chrystina Jeter, and Kingsuk Ganguly

45. Patient with chronic glossopharyngeal neuralgia/post-tonsillectomy pain 325 A. Raj Swain 46. Patient with sphenopalatine neuralgia 329 Mohit Rastogi, Natalia Murinova, and Alan David Kaye

Contents

47. Temporomandibular joint disorders Timothy Furnish

333

Section 6 – Cancer Pain 48. Cancer pain 341 Paul A. Sloan 49. Patient presents with pancreatic cancer with persistent pain despite all other treatments 352 Jay S. Grider 50. Pain management in hematological cancer: clinical case illustrations 358 Quan D. Le, Alan David Kaye, and Harry J. Gould, III 51. Patient with metastatic breast cancer who had a mastectomy complicated by lymphedema 367 Arash Asher and Jack Fu

Section 7 – Special Topics 52. A 57-year-old male with chronic pain syndrome, anxiety disorder, and hypertension is seeking mental health counseling 373 Natacha Telusca and Kingsuk Ganguly 53. Pediatric, infant, and fetal pain 379 Christine Greco and Soorena Khojasteh 54. Patient with hearing impairment and chronic pain 388 Mohit Rastogi 55. Complementary and alternative medicine 390 Ike Eriator and Jinghui Xie 56. Ethical issues in the substance abusing pain patient 399 Ike Eriator, Lori Hill Marshall, and Donald Penzien 57. Approach to the patient with abnormal drug screen 408 Jeffrey Hopcian and Magdalena Anitescu

58. Physician exposed to excessive radiation Vikram B. Patel

417

59. Patient becomes paralyzed following a lumbar transforaminal epidural steroid injection 423 Scott E. Glaser 60. Postepidural steroid injection paraplegia Annemarie E. Gallagher and Devin Peck

429

61. Complications: patient with dural puncture following cervical interlaminar epidural steroid injection 435 Niteesh Bharara and Frank J. E. Falco 62. Complications: a patient with serotonin syndrome 439 Natalia Covarrubias, Amirpasha Ehsan, and Danielle Perret Karimi 63. Office-based buprenorphine to wean patients off opioids 442 Natalia Murinova, Daniel Krashin, Cliff Gevirtz, and Alan David Kaye 64. Patient on chronic opioids who wants to have anesthesia-assisted detoxification 447 Cliff Gevirtz and Alan David Kaye 65. Munchausen syndrome and pain 456 Santhosh A. Thomas and Sachin K. Bansal 66. Insomnia and chronic pain 459 Mark Etscheidt and Paul A. Sloan 67. Opioid-induced constipation 467 John Michels, Hamilton Chen, Danielle Perret Karimi, and Justin Hata 68. Complications: vasovagal response during pain procedures 473 Frank J. E. Falco and Nomen Azeem 69. Acute pain management: patient-controlled analgesia 476 Nyla Azam and Devin Peck 70. Acute pain management: PCEA/continuous epidural catheters 482 Qiao Guo, Minyi Tan, and Devin Peck

ix

Contents

71. New vistas: continuous peripheral catheters/ regional anesthesia in postoperative pain management 491 Michael R. Rasmussen and Edward R. Mariano 72. Methadone and treatment of chronic pain 498 Daniel Krashin, Natalia Murinova, and Andrea Trescot

x

73. Drug testing 504 Steven Michael Lampert, Richard D. Urman, and Alan David Kaye

Index

508

Contributors

Magdalena Anitescu, MD Associate Professor of Anesthesia and Critical Care, Department of Anesthesia and Critical Care, University of Chicago, Chicago, IL, USA Charles E. Argoff, MD Professor, Department of Neurology, Albany School of Medicine, Albany, NY, USA Arash Asher, MD Assistant Professor at the Cedars-Sinai Medical Center, Los Angeles, CA, USA Nyla Azam, MD New York Presbyterian Hospital, Weill Cornell Medical Center, New York, NY, USA Nomen Azeem, MD 22 Chateau Mouton Drive, Kenner, LA, USA; 3439 Prytania Street Suite 501, New Orleans, LA, USA Sachin K. Bansal, MD Interventional Physiatrist, Castle Orthopedics and Sports Medicine, S.C., Aurora, IL, USA Jose E. Barreto, MD, PT TotalCare Health & Wellness Medical Center, Lafayette, LA, USA Rodrigo A. Benavides Corder, MD Clinical Fellow of Pain Medicine, Cleveland Clinic Foundation, Cleveland, OH, USA Niteesh Bharara Physiatrist and Interventional Pain Management Specialist, Virginia Spine Institute, Reston, VA, USA Justin B. Boge, DO Department of Pain Management, San Antonio Military Medical Center, Fort Sam, Houston, TX, USA

Robert B. Bolash, MD Clinical Fellow in Pain Management, Cleveland Clinic, Cleveland, OH, USA Thomas K. Bond, MD, MS Board Certified, Sports Medicine, President/Owner, TotalCare Health & Wellness Medical Center, Lafayette, LA, USA Christopher Centeno Board certified in Physical Medicine and Rehabilitation, Board Certified in Pain Medicine, Centeno-Schultz Clinic, Broomfield, CO, USA Zachariah W. Chambers MD Interventional Pain Physician, Centennial Spine and Pain Center, Las Vegas, NV, USA Jonathan Chang, MD Fellow, Pain Management, Department of Anesthesiology, Washington University School of Medicine, Saint Louis, MO, USA Grace Chen, MD Division of Pain Management, Oregon Health and Science University, Portland, OR, USA Hamilton Chen, MD UC Irvine Center for Pain Management, University of California, CA, USA Jeffry Chen, MD UCLA Department of Anesthesiology, Santa Monica, CA, USA Jianguo Cheng, MD, PhD Professor of Anesthesiology and Director of Pain Medicine Fellowship Program, Cleveland Clinic Foundation, Cleveland, OH, USA

xi

List of contributors

Natalia Covarrubias Department of Physical Medicine and Rehabilitation, The University of California at Irvine, Irvine, CA, USA Claire J. Creutzfeldt, MD Department of Neurology, University of Washington, Seattle, WA, USA Gulshan Doulatram, MD Department of Anesthesiology, University of Texas, Galveston, TX, USA Amirpasha Ehsan, MD Department of Physical Medicine and Rehabilitation, The University of California at Irvine, Irvine, USA Ike Eriator, MD, MPH Professor of Anesthesiology, University of Mississippi Medical Center, Jackson, MS, USA Jeff Ericksen, MD Division of Regenerative Medicine, Kaplan Center for Integrative Medicine and Department of Physical Medicine and Rehabilitation, Virginia VA Medical Center, McLean, VA, USA Mark Etscheidt, PhD Associate Professor of Anesthesiology, University of Kentucky Medical Center, KY, USA Frank J. E. Falco, MD Mid-Atlantic Spine and Pain Physicians, USA Jack Fu, MD Associate Professor, Department of Palliative Care & Rehabilitation Medicine, Section of Physical Medicine & Rehabilitation, University of Texas M.D. Anderson Cancer Center, Houston, TX, USA Timothy Furnish, MD Assistant Clinical Professor of Anesthesiology, UC San Diego Health System, CA, USA Annemarie E. Gallagher, MD New York Presbyterian Hospital and Weill Cornell Medical Center, New York, NY, USA Kingsuk Ganguly, MD Anesthesiology, Pain and Perioperative Medicine, Stanford School of Medicine, Stanford, CA, USA

xii

Eugene Garvin UCLA Department of Anesthesiology, Los Angeles, CA, USA Cliff Gevirtz, MD Department of Anesthesiology at LSU School of Medicine, New Orleans, LA, USA Scott E. Glaser, MD, DABIPP President, Pain Specialists of Greater Chicago, Chicago, IL, USA Lieutenant Colonel Brandon J. Goff, DO Department of Pain Management, San Antonio Military Medical Center, Fort Sam, Houston, TX, USA Harry J. Gould, III, MD, PhD Professor of Neurology and Neuroscience, Louisiana State University Health Sciences Center – New Orleans, New Orleans, LA, USA Christine Greco, MD, FAAP Children’s Hospital Boston and Harvard Medical School, Boston, MA, USA Jay S. Grider, DO, PhD Associate Professor of Anesthesiology, Division Chief, Pain and Regional Anesthesia, and Medical Director, UK HealthCare Pain Services, Lexington, KY, USA Maged Guirguis, MD Clinical Fellow in Pain Management, Cleveland Clinic, Cleveland, OH, USA Qiao Guo, MD New York Presbyterian Hospital, Weill Cornell Medical Center, New York, NY, USA Justin Hata, MD Chief, UC Irvine Pain Medicine Division, University of California, CA, USA John Hau UCLA Department of Anesthesiology, Los Angeles, CA, USA Garett J. Helber, DO Staff Physician, Cleveland Clinic – Neurological Institute, Cleveland, OH, USA

List of contributors

Eric R. Helm, MD Division of Pain Medicine, Department of Anesthesiology, University of Pittsburgh Medical Center, Pittsburgh, PA, USA

Soorena Khojasteh, MD Children’s Hospital Boston and Harvard Medical School, Boston, MA, USA

Lori Hill Marshall, MD Medical Director, Premier Pain Care, P. C., Jackson, MS, USA

Ankur Khosla Fellow, Pain Fellowship Program, Department of Anesthesiology, University of Texas Medical Branch, Galveston, TX, USA

Lieutenant Colonel Dean Hommer, MD Physical Medicine & Rehabilitation Service, San Antonio Military Medical Center, Fort Sam, Houston, TX, USA

Daniel Krashin Departments of Psychiatry and Pain & Anesthesia, Harborview Medical Center, University of Washington, Seattle, WA, USA

Jeffrey Hopcian, MD Fellow, Division of Pain Management, Department of Anesthesia and Critical Care, University of Chicago Medical Center, Chicago, IL, USA

Captain Monika A. Krzyzek, DO Department of Pain Management, San Antonio Military Medical Center, Fort Sam, Houston, TX, USA

Eric S. Hsu, MD Clinical Professor, Department of Anesthesiology, David Geffen School of Medicine, University of California Los Angeles, Santa Monica, CA, USA

Major Prasad Lakshminarasimhiah, MD Physical Medicine & Rehabilitation Service, San Antonio Military Medical Center, Fort Sam, Houston, TX, USA

Jakun Ing UCLA Department of Anesthesiology, Los Angeles, CA, USA

Steven Michael Lampert, MD Fellow, International Pain Management, Department of Anesthesiology, Perioperative and Pain Medicine, Brigham & Women’s Hospital/Harvard Medical School, Boston, MA, USA

Tracy P. Jackson, MD Assistant Professor Anesthesiology and Pain Medicine and Program Director, Multidisciplinary Pain Medicine Fellowship, Vanderbilt University, Nashville, TN, USA Gaurav Jain, MD Division of Pain Medicine, Department of Anesthesiology, University of Pittsburgh Medical Center, Pittsburgh, PA, USA Chrystina Jeter, MD Resident, Anesthesiology, Stanford University Medical Center, Stanford, CA, USA Alan David Kaye, MD, PhD Professor and Chairman, Department of Anesthesiology, Director of Interventional Pain Services, and Professor of Pharmacology, LSU School of Medicine, New Orleans, LA, USA James Kelly, MD Clinical Fellow of Pain Medicine, Cleveland Clinic Foundation, Cleveland, OH, USA

Garrett LaSalle, MD Clinical Fellow in Pain Management, Cleveland Clinic, Cleveland, OH, USA Quan D. Le, MD Departments of Physical Medicine and Rehabilitation and Pain Mastery Center of Louisiana, Louisiana State University Health Sciences Center, New Orleans, LA, USA Ankit Maheshwari, MD Chief Fellow in Pain Management, Cleveland Clinic, Cleveland, OH, USA Edward R. Mariano, MD, MAS Chief, Anesthesiology and Perioperative Care Service, Veterans Affairs Palo Alto Health Care System, Palo Alto, and Associate Professor, Department of Anesthesiology, Perioperative and Pain Medicine, Stanford University School of Medicine, Stanford, CA, USA

xiii

List of contributors

Joaquin Maury, MD Departments of Neurology and Pain Mastery Center of Louisiana, Louisiana State University Health Sciences Center, New Orleans, LA, USA Captain John P. McCallin, MD Physical Medicine & Rehabilitation Service, San Antonio Military Medical Center, Fort Sam, Houston, TX, USA John Michels, MD UCI Center for Pain Management, University of California Irvine, Irvine, CA, USA Natalia Murinova, MD Department of Neurology, University of Washington, Seattle, WA, USA Narendren Narayanasamy, MD Fellow, Pain Management, Department of Anesthesiology, Washington University School of Medicine, Saint Louis, MO, USA

Tilak Raj Dept of Anesthesiology, The University of Oklahoma Health Sciences Center, Oklahoma City, OK, USA Michael R. Rasmussen, MD Regional Anesthesiology and Acute Pain Medicine Fellow, Department of Anesthesiology, Perioperative and Pain Medicine, Stanford University School of Medicine, Stanford, CA, USA Mohit Rastogi, MD Clinical Lecturer, Division of Pain Medicine, Department of Anesthesiology, University of Michigan Hospital, Ann Arbor, MI, USA Rahul Rastogi, MD Associate Professor, Program Director Pain Fellowship, Division of Pain, Department of Anesthesiology, Washington University School of Medicine, Saint Louis, MO, USA

Rebekah L. Nilson, PT Department of Pain Management, San Antonio Military Medical Center, Fort Sam, Houston, TX, USA

Nashaat N. Rizk, MD Associate Professor, Department of Anesthesiology, Fellowship Program Director, Division of Pain Medicine, Department of Anesthesiology, University of Pittsburgh Medical Center, Pittsburgh, PA, USA

Elliot Palmer, MD Pain Medicine Fellow, Oregon Health and Science University, Portland, OR, USA

Rinoo V. Shah, MD, MBA Interventional Pain Physician and Minimally Invasive Spine Specialist at Guthrie Clinic, Sayre, PA, USA

Vikram B. Patel, MD FIPP DABIPP Phoenix Interventional Center for Advanced Learning, Algonquin, IL, USA

Paul A. Sloan, MD Professor of Anesthesiology, University of Kentucky, Lexington, KY, USA

Devin Peck, MD Assistant Professor of Anesthesiology; Director, Tri-Institute Pain Fellowship, New York Presbyterian Hospital, Weill Cornell Medical Center, New York, NY, USA

Julian Sosner, MD, FIPP Associate Clinical Professor, New York Medical College, Valhalla; Attending Physician, Department of Pain Medicine and Palliative Care, Beth Israel Medical Center and Mount Sinai Medical System, New York; and Director, New York Interventional Pain Medicine Service, PC, New York, NY, USA

Donald B. Penzien, PhD Professor of Psychiatry and Human Behavior, University of Mississippi Medical Center, Jackson, MS, USA Danielle Perret Karimi, MD Department of Anesthesiology and Perioperative Care and Department of Physical Medicine and Rehabilitation, The University of California at Irvine, Irvine, CA, USA

xiv

A. Raj Swain MD Chief – Pain Management, Berger Hospital and Fayette County Memorial Hospital, OH, USA Minyi Tan, MD New York Presbyterian Hospital, Weill Cornell Medical Center, New York, NY, USA

List of contributors

Natacha Telusca, MD, MPH Anesthesia Resident, PGY4, Department of Anesthesiology, Pain and Perioperative Medicine, Stanford University School of Medicine, Stanford, CA, USA Santhosh A. Thomas, DO, MBA Staff Physician, Cleveland Clinic – Neurological Institute, Associate Medical Director, Richard E. Jacobs Health Center, and Medical Director – Center for Spine Health at Richard E. Jacobs Health Center, Cleveland, OH, USA Andrea Trescot, MD Pain and Headache Center, Eagle River, AK, USA Michael Truong UCLA Department of Anesthesiology, Los Angeles, CA, USA Jason Tucker Virginia Commonwealth University Department of Physical Medicine and Rehabilitation, Richmond, VA, USA Richard D. Urman, MD, MBA, CPE Assistant Professor of Anesthosia, Director, Procedural Sedation, Management, and Safety, Brigham and Women’s Hospital, Harvard Medical School, Boston, MA, USA

Brandon A. Van Noord UCLA Department of Anesthesiology, Los Angeles, CA, USA Nihir Waghela, MD Fellow, Division of Pain Management, Department of Anesthesia and Critical Care, University of Chicago Medical Center, Chicago, IL, USA Irene Wu UCLA Department of Anesthesiology, Santa Monica, CA, USA Jiang Wu, MD Clinical Fellow of Pain Medicine, Cleveland Clinic Foundation, Cleveland, OH, USA Jijun Xu, MD, PhD Clinical Fellow of Pain Medicine, Cleveland Clinic Foundation, Cleveland, OH, USA Jinghui Xie, MD, PhD Physician, Careone Pain Management, Advanced Interventional Pain Management, USA William Yancey, MD Fellow, Pain Fellowship Program, Department of Anesthesiology, University of Texas Medical Branch, Galveston, TX, USA

xv

Foreword

Case studies in pain management Pain management is a dynamic and evolving specialty. The diagnosis and treatment of pain-related conditions have changed extensively in recent years. Major changes include not only surgical advances applying minimally invasive techniques and multidisciplinary approaches, but also multiple interventional techniques based on evidence. Numerous publications have described the anatomy, physiology, pathology, and technical aspects of interventional techniques. Other texts have been written describing various non-interventional modalities including pharmacology, psychology, behavioral aspects, and drug therapy. In recent years, pain medicine and interventional pain management have seen a substantial growth in the publication of journals and books. However, there remains a major gap in pain management case studies. This text by Alan Kaye and Rinoo Shah is a monumental accomplishment toward fulfilling this goal. This text extensively describes neurological disorder cases in various categories. However, it does not stop with neuropathic pain cases; this text’s 72 chapters address various spinal disorders, musculoskeletal pain, visceral pain,

xvi

headache and facial pain, cancer pain, and multiple special topics and complications. Our understanding of a multitude of cases has grown over the years based on evolving evidence. The authors of the various cases in this text are of the highest caliber and are drawn from the highest levels of academia, research, and private practice. This text is directed not just to practitioners, but more importantly, to all types of clinicians engaged in managing painful conditions. Once again, the contributing authors are appreciated for undertaking such a monumental task and providing a different perspective in managing painful conditions with the publication of Case Studies in Pain Management. Laxmaiah Manchikanti, MD Chairman of the Board and Chief Executive Officer, ASIPP and SIPMS Medical Director, Pain Management Center of Paducah Clinical Professor Anesthesiology and Perioperative Medicine University of Louisville, Kentucky

Section 1 Chapter

1

Neurological Disorders

Postherpetic neuralgia* Alan David Kaye and Charles E. Argoff

Case study A 78-year-old male with a history of postherpetic neuralgia (PHN) as well as hypertension presents to your office with complaints of moderate to severe pain (intensity 7/10) along the right T8 dermatome. He experienced acute herpes zoster (shingles) in this region 3 years ago and was treated at that time with acyclovir and analgesics. The pain never dissipated and for the past 3 years he has been treated with a variety of medications, including immediate-release gabapentin, nortriptyline, and the 5% lidocaine patch as well as unsuccessful treatment with various nerve blocks and a trial of spinal stimulation.

1. What are the basic facts regarding postherpetic neuralgia, varicella-zoster virus, and shingles? Postherpetic neuralgia is a chronic painful complication of shingles, originating with the varicella-zoster virus (VZV), the same virus that causes chicken pox. Approximately, 98% of adults have been exposed to VZV, mostly as children. Reactivation of VZV can occur decades after initial exposure to the virus. Shingles occurs in approximately 1 million people/ year in the USA alone and thus, it is the neurological disease with the highest incidence in the USA. There is a one out of three lifetime incidence in the general population of developing shingles, with increasing * Some of the material presented in this chapter was previously reviewed and published by the authors in Harden RN, Kaye AD, Kintanar T, Argoff CE. 2013. Evidence-based guidance for the management of postherpetic neuralgia in primary care. Postgrad Med 125(4):191–202. doi: 10.3810/pgm.2013.07.2690.

incidence in the elderly. Between 40% and 50% of the people who develop shingles are older than 60 years of age and between 10% and 20% develop PHN.[1,2] PHN results from damage to sensory neurons caused by reactivation of VZV. In PHN, residual nerve fibers appear to become hyperexcitable, resulting in persistent and unpredictable neural signaling, producing a pain state that is often difficult to manage. PHN is described as the pain that persists 3 months or more beyond the healing of herpes zoster blisters and approximately 15% of people who have had shingles ultimately develop PHN. In the USA, this translates to approximately 150 000 new cases annually.[3]

2. What are the basic features of postherpetic neuralgia? Symptoms of PHN may last indefinitely. Risk factors for PHN include female gender, advanced age, presence of painful VZV prodrome, greater VZV rash severity or significant pain, elevated fever in the acute phase of the VZV episode, and sensory dysfunction in the affected dermatome. As with VZV, PHN disproportionately affects older patients.[4] In one study, the overall incidence of PHN was 18% in all adults, but increased to 33% for those  79 years.[5]

3. Why are there are so many challenges with regard to postherpetic neuralgia treatment options? Numerous pharmacologic options for PHN have been extensively studied in randomized controlled studies, and several guidelines regarding the pharmacologic treatment of PHN itself exist. Treatment success must overcome a series of barriers. First, the PHN patient

Case Studies in Pain Management, ed. Alan David Kaye and Rinoo V. Shah. Published by Cambridge University Press. © Cambridge University Press 2015.

1

Chapter 1: Postherpetic neuralgia

Table 1.1. Summary of treatment guidelines for PHN*

Alpha-2 delta ligands† ‡

NeuPSIG (2010)

EFNS (2010)

CPS (2007)

AAN (2004)

1st line

1st line

1st line

1st line

TCAs

1st line

1st line

1st line

1st line

Topical lidocaine

1st line

1st line

2nd line

1st line

Opioids (including tramadol)

2nd line

2nd line

3rd line

1st line

Topical capsaicin (0.025–0.075%)

2nd line

3rd line

Not described

Not described



3rd line

Not described

Not described

§

Topical capsaicin (8%)

* Except for the AAN guidelines, all review neuropathic pain in general but make specific mention of PHN within the guidelines. All lines of therapy refer to role in PHN specifically. † Gabapentin immediate release and pregabalin. At the time of publications of these guidelines, gastroretentive gabapentin was not available. ‡ Nortriptyline, amitriptyline, desipramine, imitriptyline. NeuPSIG distinguishes between secondary amine TCAs (nortriptyline and desipramine) and tertiary amine TCAs (amitriptyline, imitriptyline) and recommends the former due to superior tolerability. § Topical capsaicin (high concentration, 8%) was approved on November 16, 2009, shortly before publication of the guidelines.

population is frequently older. As with any older population, medical comorbidities and multidrug regimens may affect the choice of drug therapy. Second, not infrequently, payors may limit treatment options or require a step approach mandating failure with certain generic medications, often used in an offlabel manner (including Medicare Part D providers), before paying for potentially more appropriate, as well as potentially higher cost, options. This often results not only in the use of medications that are not specifically Food and Drug Administration (FDA) approved for the treatment of PHN being used before those that are and for which there may be more data to guide treatment, but also potentially a greater likelihood of failure of treatment and its resulting impact on the patient with PHN. Third, the process required to optimize treatment for most medications used to treat PHN to ameliorate adverse effects may require long titration periods, demanding patience and education on the part of both the physician and the patient. Fourth, assuming the physician can overcome the first three barriers, the patient has, based upon the best available guidelines, literally at best, a 50/50 chance of achieving clinically meaningful pain relief (considered 30% pain intensity reduction) with little chance of predicting who will respond to a particular treatment.

4. What are the guidelines for postherpetic neuralgia management? Over the past decade, several organizations have published guidelines either devoted exclusively to PHN or

2

describing PHN in the context of neuropathic pain conditions in general.[6–9] A summary of their recommendations is found in Table 1.1. Each of the guidelines recognize the alpha-2 delta ligands, tricyclic antidepressants (TCAs), opioids, and tramadol as systemic options and topical lidocaine as a non-systemic approach for the treatment of localized PHN. Alpha-2 delta ligands and TCAs are typically recommended as first- or second-line status in the guidelines, and opioids and tramadol are often relegated to secondor third-line although under certain circumstances, first-line. At the time the Special Interest Group on Neuropathic Pain of the International Association for the Study of Pain (NeuPSIG) guidelines were written, the topical capsaicin (8%) patch was recognized as an emerging therapy with insufficient evidence to make a recommendation. In addition, a gastroretentive form of gabapentin as well as a form of gabapentin which is in fact a prodrug were not addressed. Table 1.2 shows the NeuPSIG guidelines. Additional evidence for the use of these agents is now available. The NeuPSIG[7] and the European Federation of Neurological Societies (EFNS)[8] guidelines are the most recently published. Both review the treatment of neuropathic pain in general, but also include specific mention of PHN. The Canadian Pain Society (CPS), published in 2007, likewise makes specific mention of PHN within the context of overall neuropathic pain. The American Academy of Neurology (AAN), in contrast, published a specific PHN guideline in 2004; however, new published evidence has become available since 2004.

Chapter 1: Postherpetic neuralgia

Table 1.2. Summary of NeuPSIG Guidelines for PHN*



Begin treatment with one or more of the following:    

  

Secondary amine TCA (nortriptyline, desipramine) Alpha-2 delta ligand (gabapentin, pregabalin) Topical lidocaine (for patients with localized PHN) alone or in combination with another therapy Opioids or tramadol for patients with acute exacerbations requiring prompt relief (used alone or in combination with one other firstline therapy)

If pain relief is partial (average pain  4 out of 10), add one of the other first-line therapies If no or inadequate pain relief (< 30% reduction at target dosage) after an adequate trial,† switch to another first-line option If first-line single-agent or combination therapy fails, consider second- or third-line options

* Modified from table 1 in Ref.[7]. † Some drugs such as immediate-release gabapentin and TCAs require long duration of up to 8 weeks.

Criteria for recommendations varies The NeuPSIG guidelines rated a medication first line if it has proven effective in multiple randomized controlled studies (RCTs) and the results are consistent with the authors’ clinical experience; second-line status if efficacy has been established in multiple RCTs but the authors had reservations about the use of the medication relative to first-line options; third-line if efficacy was shown in only one RCT or if the results of two or more RCTs were inconsistent, “but the authors thought that in selected circumstances the medication may be a reasonable treatment option.”[8: p. S4] In contrast, the EFNS rate medications having “established” efficacy based on class I or class II evidence, with class I defined as “an adequately powered prospective, randomized, controlled clinical trial with masked outcome assessment in a representative population or an adequately powered systematic review of prospective randomized controlled clinical trials with masked outcome assessment in representative populations” (Table 1.1).[10] In addition, class I studies must have all of the following: (a) randomization concealment, (b) clearly defined primary outcome(s), (c) clearly defined exclusion/ inclusion criteria, (d) adequate accounting for

dropouts and crossovers with numbers sufficiently low to have minimal potential for bias, and (e) relevant baseline characteristics are presented and substantially equivalent among treatment groups or there is appropriate statistical adjustment for difference. Class II is defined as “prospective matched-group cohort study in a representative population with masked outcome assessment that meets a–e above or a randomized, controlled trial in a representative population that lacks one of the criteria a–e” (Table 1.1).[10] The CPS published a consensus statement on the management of neuropathic pain in 2007.[9] To be recommended in the guidelines, medications had to show efficacy in at least one methodologically sound RCT (Level 1B or better, as defined by Ref.[11]). The guidelines state that they are based on analgesic efficacy, side effect profiles, ease of use, and cost, but describe no criteria for any of these domains except efficacy. To be recommended as first- or second-line, medications had to have high-quality evidence of efficacy and be considered straightforward to prescribe and to monitor. Medications were relegated to third-line if there was good evidence of efficacy but more specialized follow-up and monitoring were required. The fourth guideline, and the only one to specifically address PHN, is the AAN practice parameter published in 2004. The criteria for a level A recommendation were very similar to the Brainin criteria used by the EFNS and required at least one class I study or at least two consistent, convincing class II studies. For class I and class II, the authors also calculated, if possible, absolute risk reduction, number needed to treat (NNT) for adequate pain relief, 95% confidence interval of the NNT, and number needed to harm. Recommendations were then grouped, with Group 1 medications showing medium to high efficacy, good strength of evidence, and low level of side effects, and Group 2 medications showing lower efficacy than those in Group 1 or limited strength of evidence or side effect concerns. (Three other groups with successively lower strength of evidence are also described in the AAN practice parameter.) The criteria for “medium” versus “high” level of efficacy were not defined, nor were the criteria for what constitute a “side effect concern.” The AAN guidelines are somewhat dated but it is interesting to note that, of the four major drug classes currently recommended today as first-, second-, or third-line

3

Chapter 1: Postherpetic neuralgia

therapy in the NeuPSIG, EFNS, or CPS guidelines, all of them are recommended as Group 1 medications (alpha-2 delta ligands, TCAs, opioids, and lidocaine patch) in the AAN guidelines. All drugs listed as Group 2, 3, and 4 options are now considered ineffective or unproven. More recent guidelines downgrade opioids because of the risk of abuse and the added time needed to assess risk, monitor the patient, monitor for adverse effects, and remove patients from therapy if abuse is suspected. The reader must keep in mind that these guidelines have arrived at similar BUT not identical conclusions.

5. Are there any systematic reviews and meta-analysis data on postherpetic neuralgia treatments? In addition to the above-mentioned guidelines, four separate Cochrane reviews have been published, one each on gabapentin,[12] pregabalin,[13] topical lidocaine,[14] or topical capsaicin,[15] as well as a metaanalysis of a broad range of drugs for PHN.[16] Except for the topical lidocaine Cochrane review, which focused exclusively on PHN, the other Cochrane reviews included a range of neuropathic, and at times non-neuropathic, pain conditions. The Cochrane review on gabapentin[12] included PHN studies of immediate-release and gastroretentive gabapentin. It concluded that gabapentin was effective for chronic neuropathic pain but did not draw any conclusions specifically about efficacy in PHN. The Moore review on pregabalin[13] included 5 PHN studies and concluded that pregabalin at both 300 mg/day and 600 mg/day were effective in PHN, with greater responses seen at 600 mg/day. The Khaliq review[14] on topical lidocaine identified nine published trials but excluded seven of them because they did not meet prespecified inclusion criteria. One additional unpublished trial was identified and data were obtained from the FDA and analyzed. According to this review, these three studies demonstrated modest benefit of topical lidocaine in PHN and the authors concluded that there is insufficient evidence to recommend topical lidocaine as first-line therapy in PHN. The Derry[15] review on topical capsaicin analyzed six studies of low-concentration topical capsaicin (0.075%) cream and two studies utilizing the high-concentration topical capsaicin (8%) patch. The authors concluded that repeated daily applications of

4

the cream and a single application of the patch (applied once every 3 months) provided “some degree of improvement” in patients with PHN.[15: p. 14] The meta-analysis conducted by Edelsberg and colleagues[16] analyzed 12 randomized controlled PHN studies involving eight different agents. This analysis demonstrated that gabapentin immediate release (2 studies), pregabalin (3 studies), the TCAs amitriptyline and nortriptyline (1 study each), morphine (1 study), capsaicin (2 studies), tramadol (1 study), and divalproex (1 study) showed statistically significantly greater reductions in pain compared with placebo. In general, the Cochrane reviews and the meta-analysis are all consistent with the recommendations of current guidelines, with the exception of the topical lidocaine Cochrane review, which did not consider sufficient evidence to exist to recommend topical lidocaine as first-line therapy.

6. Are there any gaps in the Postherpetic Neuralgia Treatment Guidelines? High-quality clinical studies have been the foundation of evidence-based medicine and provide a solid foundation for authoritative guidelines, yet interpreting and applying the guidelines to clinical practice must be done with an awareness of the limitations and blind spots of clinical studies and a full understanding of what evidence-based medicine is and what it is not. Evidence-based medicine includes “hard” data but as defined, also allows for the integration of clinical expertise and patients’ values and preferences.[17] As Sackett has stated, evidence-based medicine is “the conscientious, explicit, and judicious use of current best evidence in making decisions about the care of individual patients. In this definition, the practice of evidence-based medicine means integrating individual clinical expertise with a critical appraisal of the best available external clinical evidence from systematic research.”[18] Regrettably, it is our view that this definition is not addressed in the guidelines described above. Clinical trials often select patient populations to minimize intersubject heterogeneity. Specific comorbidities are often excluded, and concomitant medications that many patients would commonly take are excluded. While this approach minimizes variables that confound interpretation by doing so, it also

Chapter 1: Postherpetic neuralgia

excludes the type of patient that is commonly seen in clinical practice. In addition, differences among formulations of the same drug in terms of efficacy, dosing, adherence, and convenience and patient preferences (which may range from dosing convenience to a specific adverse effect that a patient may find problematic) may not be addressed. Also typically not addressed are differences in tolerability in clinically relevant subpopulations; the efficacy at target doses that typically can be achieved in practice (in contrast to those achieved in clinical studies); differences in the various descriptive components of pain; acute exacerbations of pain; and onset of pain relief. Given that head-to-head studies are often lacking, direct comparisons of various pharmacologic options is difficult, and studies used to develop published guidelines, typically do not assess long-term therapy (> 3 months). Although it would be unfair to say that PHN guidelines don’t address these issues at all, if they are addressed it is often done in the context of neuropathic pain in general and lacking in direction regarding how to integrate numerous clinical variables in practice (the “real” world), particularly in complex patients who have significant medical and other comorbidities and who may be taking numerous medications. PHN guidelines, in particular, are further hampered by a lack of inclusion of more recent clinical data that have emerged since the last guidelines were published in 2010.

7. Are there any new clinical data on postherpetic neuralgia treatments? High-concentration (8%) topical capsaicin patch The high-concentration topical capsaicin patch is administered once every 3 months (in contrast to the low-concentration topical capsaicin creams, which are administered several times daily). Since the publication of the last guidelines, multiple publications including two multicenter, randomized, double-blind PHN studies[19,20] and an integrated analysis of four randomized, double-blind PHN studies[21] have become available. The patch was applied for 60 minutes in all studies although in one study the patch was also applied for 30 and 90 minutes.[20] Subjects in the control arms received a 0.04% capsaicin patch to maintain blinding, as a true placebo

would not induce a local site reaction, which occurs with the 8% patch. The primary endpoint was change from baseline in pain intensity level assessed using a Numerical Pain Rating Scale (NPRS). Change from baseline was calculated by comparing baseline scores with the average of daily NPRS scores from weeks 2–8 and weeks 2–12. Data from Week 1 data were not included because subjects received opioid medication in week 1 to alleviate application site pain caused by the patch. The Irving study[19] showed the highconcentration topical capsaicin patch superior to control in change from baseline in NPRS to weeks 2–8, percentage change from baseline in NPRS from weeks 2–8 and weeks 2–12, percentage of patients with a 30% response, and percentage of patients with a 50% response[19: p. 105] (Table 1.2). In the Webster study,[20] a 60-minute application showed significant improvement in percent change from baseline in average pain score (NPRS) over weeks 2–12, but no significant reduction in mean change from baseline over weeks 2–8 or weeks 2–12 or in percent change from baseline over weeks 2–8. The integrated analysis of over 1000 patients in 4 PHN studies likewise demonstrated statistically significant improvements relative to control in percentage change from baseline in NPRS to weeks 2–12, 30% response rate, and 50% response rate as well as patient global impression of change (PGIC). Based on these data, although published guidelines did not address this treatment for the reasons noted above, it is our opinion that the highconcentration topical capsaicin patch should be considered first-line therapy for patients with localized PHN.

Gastroretentive gabapentin Gastroretentive gabapentin is one of two currently available extended-release formulations of gabapentin. When administered with a meal, this tablet swells and resides in the stomach for up to 15 hours, releasing drug gradually for absorption by the proximal small intestine. The starting dose is 300 mg/day once daily and increased over 2 weeks to a target dose of 1800 mg/day. Three multicenter, randomized, controlled double-blind studies have been reported either shortly before publication of the most recent guidelines or after publication. One study of 452 patients randomized to once daily gastroretentive gabapentin or placebo demonstrated a statistically significant

5

Chapter 1: Postherpetic neuralgia

reduction in mean change in NPRS scores from baseline and in percentage change from baseline to the final week of the treatment period (Week 10).[22] A second study of 407 randomized subjects showed statistically significant improvements in a range of secondary endpoints (average pain on the Neuropathic Pain Score; worst pain, average pain, and current pain on the Brief Pain Inventory). Using last observation carried forward (LOCF) imputation method, which was the imputation method used in the high-concentration topical capsaicin studies, once daily gastroretentive gabapentin also showed a statistically significant improvement in average daily pain score (NPRS) over 10 weeks of treatment. However, on the primary endpoint using the prespecified baseline observation carried forward (BOCF) imputation method, once daily gastroretentive gabapentin (1800 mg/day) was no better than placebo over 10 weeks of treatment.[23] A third study[24] also failed to show a statistically significant difference vs. placebo over 4 weeks of treatment, 2 weeks of which were the titration phase and 2 weeks of which were at full dose. All of these studies used a conservative imputation approach to missing data (BOCF versus the less conservative LOCF). The BOCF method will typically underestimate efficacy compared with LOCF; for patients who don’t complete the study, the baseline scores (pretreatment) are carried forward. With LOCF, the last available score before dropout is carried forward (and thus usually includes scores after some interval of treatment). Based on the available evidence, gastroretentive gabapentin meets the standard of first-line therapy in the EFNS guidelines (one rigorous RCT needed). Whether it meets the standard of first-line therapy in the NeuPSIG guidelines (multiple RCTs needed) is a matter of interpretation. Unlike the highconcentration topical capsaicin studies, each of the three gastroretentive gabapentin studies used a conservative imputation method for each primary efficacy analysis, and one of the failed studies showed clear separation from placebo when data were analyzed using the LOCF imputation method. Based on the available evidence and other features of gastroretentive gabapentin (such as dosing convenience, pharmacokinetics), we believe it can be considered a first-line option for PHN in certain clinical situations. When administered with an evening meal, peak dose occurs in the early morning (approximately 3 AM), when patients are sleeping. This may account for the

6

observed improved tolerability of gastroretentive gabapentin (lower rate of dizziness and sedation) relative to published reports of gabapentin IR and pregabalin.

Gabapentin enacarbil Gabapentin enacarbil is a twice daily extended-release formulation of gabapentin, specifically formulated as a prodrug. It is currently FDA approved for restless leg syndrome and PHN. A randomized, double-blind study of 115 patients with PHN showed superior pain relief with gabapentin enacarbil versus placebo as assessed by mean change from baseline in pain scores and 30% response rate.[25] This study consisted of a 4-day titration phase with gabapentin immediate release, a 7 day run-in phase with gabapentin immediate release 1800 mg, followed by randomization to either gabapentin enacarbil 1200 mg BID or placebo, which subjects received for 2 weeks. Data imputation for subjects who did not complete the double-blind treatment consisted of the mean daily pain scores from the preceding 7 days. The primary efficacy endpoint was change in weekly pain score from baseline to the final week on double-blind treatment. Limited published data are available on this product, and the short duration of this trial precludes any assessment of this product’s long-term efficacy. However, a 12-week efficacy study described in the product label showed efficacy at all doses tested (up to 3600 mg/day), but 2400 mg/day and 3600 mg/day showed no greater efficacy than 1200 mg/day, and adverse effects were more pronounced at higher doses. The starting dose of gabapentin enacarbil is 600 mg in the morning for 3 days; on day 4, dose should be increased to 600 mg twice daily. Though early evidence demonstrates efficacy with an increasing dose-dependent side effect profile, twice daily dosing provides a clear disadvantage versus once daily dosing of gastroretentive gabapentin and titration above 1200 mg/day is not indicated. The lack of a published randomized controlled trial of significant duration is a limitation and precludes a full evaluation of this product’s place in treatment.

Pregabalin combination therapeutic approaches Several recent studies have evaluated the use of pregabalin in combination with lidocaine plaster,[26,27]

Chapter 1: Postherpetic neuralgia

oxycodone,[28] or transcutaneous electrical nerve stimulation (TENS).[29] Rehm and colleagues and Baron and colleagues assessed the combination of topical lidocaine and pregabalin but no data on statistical significance of the findings were reported. Zin and colleagues found that the addition of a fixed-dose of oxycodone 10 mg did not add to the efficacy of pregabalin, but given that opioids are typically titrated to effect, the fixed-dose of oxycodone may have been too low.[28] A study comparing the use of pregabalin with TENS showed that the addition of TENS to pregabalin 300 or 600 mg/day resulted, after 4 weeks of treatment, in a statistically significant improvement in pain assessed using a visual analog scale,[29] compared with pregabalin alone.

8. What are key considerations in choosing postherpetic neuralgia treatments? Efficacy is a critical factor in treatment selection, but several other factors must be considered when selecting a treatment for a person with PHN. These include:

Tolerability Common adverse effects associated with first- and second-line options for PHN are shown in Table 1.3.

A key consideration for therapeutic success is the ability of the patient to tolerate the therapy longterm, a parameter that is specifically required of class I evidence only in the PHN-specific guidelines published.[6] For a study to be rated as class I by AAN, at least 80% of subjects must complete the study.[30] Those options associated with the potential for significant drowsiness and somnolence pose a challenge for patients, in particular the elderly. Alpha-2 delta ligands are associated with dizziness and somnolence in 10%–20% of patients and should therefore be used cautiously in patients with gait or balance problems. CNS effects of gabapentin IR, gastroretentive gabapentin, gabapentin enacarbil, and pregabalin are shown in Table 1.4. Given the fact that dizziness and somnolence are common with all first- and secondline systemic medications (except TCAs) for PHN, even an incremental reduction in these adverse effects may be significant. Picking such an agent is difficult in the absence of head-to-head studies but the reader should review Table 1.4 for guidance. Tramadol is associated with seizure risk if given alone or if given with selective serotonin reuptake inhibitors (SSRIs), TCAs, or other opioids. Although a rare side effect, it is also associated with an increased risk of serotonin syndrome if given with SSRIs, selective norepinephrine reuptake inhibitors (SNRIs), TCAs, or monoamine oxidase inhibitors (MAOIs). Anticholinergic effects are common with TCAs, but may be less

Table 1.3. Common adverse effects

Drug class

Key adverse effects

TCAs*

Cardiac toxicity, postural hypotension, urinary retention, angle-closure glaucoma, dry mouth, constipation, sweating

Gabapentin IR

Dizziness, somnolence, ataxia, fatigue, weight gain, dry mouth, peripheral edema

Gastroretentive gabapentin

Dizziness, somnolence, ataxia, fatigue, weight gain, dry mouth, peripheral edema

Gabapentin enacarbil

Dizziness, somnolence, fatigue/asthenia, peripheral edema

Pregabalin

Dizziness, somnolence, ataxia, fatigue, weight gain, dry mouth, peripheral edema

Opioids

Constipation, nausea, somnolence, dizziness, pruritis

Tramadol

Dizziness, nausea, constipation, somnolence, flushing, pruritis, insomnia, asthenia Seizure risk at high doses and when given with SSRIs, TCAs, opioids Serotonin syndrome risk when given with SSRIs, SNRIs, TCAs, MAOIs, and triptans

* Secondary amines (nortriptyline and desipramine) are considered by the NeuPSIG guidelines as better tolerated than tertiary amines (amitriptyline, imitriptyline). Abbreviations: SSRI, selective serotonin reuptake inhibitor; SNRI, selective norepinephrine reuptake inhibitor; MAOI, monoamine oxidase inhibitor; TCA, tricyclic antidepressant.

7

Chapter 1: Postherpetic neuralgia

Table 1.4. CNS effects of alpha2-delta ligands

% of AE with Alpha-2 Delta Ligands (% of AE with Placebo) Gabapentin IR*

Gastroretentive gabapentin†

Gabapentin enacarbil‡

Pregabalin§

Dizziness

28.0 (7.5)

10.9 (2.2)

17.0 (15.0)

21.0[5]

Somnolence

21.4 (5.3)

4.5 (2.7)

10[8]

12.0[3]

Lethargy

NR

1.1 (0.3)

NR

NR

Fatigue/asthenia

NR

NR

6.0[1]

NR

Ataxia

3.3 (0)

NR

NR

3[1]

Vertigo

NR

NR

NR

3[1]

Confusion

NR

NR

NR

2[1]

Thinking abnormal

2.7 (0)

NR

NR

2 (0)

Abnormal gait

1.5 (0)

NR

NR

1 (0)

Incoordination

1.5 (0)

NR

NR

2 (0)

Amnesia

1.2 (0)

NR

NR

1 (0)

Hypesthesia

1.2 (0)

NR

NR

NR

* Neurontin (gabapentin) Package Insert † Gralise (gabapentin) Package Insert ‡ Horizant (gabapentin) Package Insert. Rates are based on 1200 mg/day. At higher doses, dizziness was 26% with 2400 mg/day and 30% with 3600 mg/day. § Lyrica (pregabalin) Package Insert. Abbreviations: AE, adverse event; NR, not reported.

common with the secondary amines (nortriptyline and desipramine) compared to the tertiary amines (amitriptyline and imitriptyline). Opioids’ adverse effects include dizziness, somnolence, constipation, hypogonadism, and nausea and are associated with the risk of misuse and abuse. Although from an analgesic viewpoint, opioids are generally at least as effective as other drugs for PHN, they are typically not recommended as first line mainly because of their adverse effect profile as well as risk of abuse and the need to screen patients for risk of abuse, monitor potential abuse, and intervene if abuse is suspected. Both topical options (capsaicin and lidocaine) have negligible systemic adverse effects and thus can be very useful for patients on multiple medications or who cannot tolerate systemic medications.

Dosing and onset of analgesia Prescriber knowledge of dosing of available drug therapies is critical for success (Table 1.5). To minimize adverse effects, a slow titration phase is required for TCAs, gabapentin IR, and pregabalin. In contrast, gastroretentive gabapentin can be titrated over 2 weeks

8

up to 1800 mg/day, and gabapentin enacarbil over 1 week up to 1200 mg/day. Onset of efficacy for these agents may be delayed, but if the patient is tolerating these drugs well, the provider and patient should make every effort to complete the titration phase and not terminate early. Frequency of dosing is a major contributor to adherence with chronic use. There is no titration required for the 5% lidocaine patch nor the 8% capsaicin patch. Although few studies have assessed dosing frequency and adherence in chronic pain, in several other therapeutic areas adherence increases with decreasing dosing frequency.[31] Ideally, TID medications should be avoided in favor of medications with BID or QD dosing, especially in patients on multiple medications. In this regard, medications such as topical capsaicin (8%) (applied once every 3 months), the topical lidocaine patch (3 patches applied 12 hours daily), gastroretentive gabapentin (once daily), gabapentin enacarbil (twice daily), the TCAs (once daily or given in two divided doses per day), and some extended-release opioid formulations are more attractive. Gabapentin IR is given three times daily, and pregabalin two to three times daily.

Chapter 1: Postherpetic neuralgia

Table 1.5. Dosing and onset considerations*

Drug class

Dosing

Duration of adequate trial

TCAs

Start at 25 mg at bedtime Increase 25 mg/d every 3–7 days

6–8 weeks with at least 2 weeks at maximum tolerated dosage

Gabapentin IR

Start at 100–300 mg at bedtime or 100–300 mg 3 times daily Increase by 100–300 mg 3 times daily every 1–7 d as tolerated

3–8 weeks for titration plus 2 weeks at maximum dose

Gastroretentive gabapentin

Take with evening meal Start at 300 mg/d Increase dose to 600 mg/d on day 2, 900 mg/d on days 3–6, 1200 mg/d on days 7–10, 1500 mg/d on days 11–14, and 1800 mg/d on day 15

Not defined

Gabapentin enacarbil

Start at 600 mg in the morning for 3 days Increase to 600 mg BID beginning on day 4

Not defined

Pregabalin

Start at 50 mg 3 times daily or 75 mg twice daily as tolerated. Increase to 300 mg/d after 3–7 d, then by 150 mg/d every 3–7 d as tolerated

4 weeks

Topical lidocaine

Maximum of three patches daily for a maximum of 12 hours

3 weeks

Topical capsaicin (8%)

1 patch applied for 60 minutes every 3 months

Not defined

Opioids

Start at 10–15 mg morphine or morphine equivalents every 4 hours as needed After 1–2 weeks, convert total daily dosage to long-acting opioid analgesic and continue short-acting medication as needed

4–6 weeks

Tramadol

Start at 50 mg once or twice daily Increase by 50–100 mg/d in divided doses every 3–7 d as tolerated

4 weeks

* Modified from[7] (Table 2).

Are there challenging subsets of patients and guideline gaps in subpopulations of patients with postherpetic neuralgia? The older patient The PHN patient is typically older, has several comorbidities, and takes multiple medications resulting in special considerations and gaps in treatment considerations (Table 1.6). Approximately 20% of people 65 years of age or older are taking 5 or more drugs.[32] Of the 10 most commonly administered medications given to the elderly, 6 of them (hydrochlorothiazide, lisinopril, metoprolol, atenolol, amlodipine, and furosemide) cause drowsiness, dizziness, or somnolence.[33] Thirty percent of hospitalizations are associated with drug-related problems or adverse effects.[34] This population is particularly sensitive to adverse effects of medications, and it is here where treatment selection becomes complicated.

In addition to a standard pain work-up, special attention should be paid to assessing the older patient’s physical function. Range of motion testing, gait, and balance testing should be considered, and if deficits are found, drugs with a higher risk of dizziness and somnolence should be avoided or used with caution. Because some side effects can be minimized or avoided with slow titration, if the patient with gait or balance problems is a candidate for a drug causing significant sedation or drowsiness, a low starting dose and slow titration schedule may alleviate some of these side effects. Older patients have decreased renal and hepatic function, altered drug distribution, and decreased blood volume, which can affect drug metabolism and tolerability. Glomerular filtration rate decreases by about 0.75 to 0.9 ml/min per year after the age of 30–40 years. By the age of 80, glomerular filtration rate may be two-thirds that of a

9

Chapter 1: Postherpetic neuralgia

Table 1.6. Guideline gaps

Special populations Elderly

Depression Anxiety Renal/ hepatic impairment

Cardiovascular comorbidities

History of substance abuse

NA

NA

Dose reduction required for gabapentin and pregabalin in pts with renal insufficiency

Prescribe TCAs with caution in pts with ischemic heart disease or ventricular conduction abnormalities; limit dosages to 100 mg/d when possible; obtain screening ECG

Risk of abuse of tramadol seems considerably less than that with strong opioids Avoid strong opioids as 1st-line therapy due to risk of abuse/misuse If opioids used, monitor for signs of abuse

EFNS

Topical lidocaine, with its NA excellent tolerability, may be considered 1st-line in the elderly

NA

NA

NA

NA

CPS

Topical lidocaine is a good 2nd-line analgesic for elderly

NA

NA

NA

TCAs are “relatively” contraindicated

NA

AAN

NA

NA

NA

NA

NA

NA

AGS

Start with lower doses of most drugs Older pts rarely tolerate TCA doses > 75–100 mg/d Monitor sedation, ataxia, and edema with alpha-2 delta ligands Opioids can be an effective option in properly selected and monitored patients

NeuPSIG Topical lidocaine’s lack of systemic adverse effects and drug interactions make this product advantageous in older patients

NA ¼ not addressed.

healthy 20- to 30-year-old.[35] Elderly patients are also more sensitive to opioids and benzodiazepines.[36] The American Geriatrics Society (AGS) notes that the elderly and patients with multiple comorbidities are rarely studied in randomized controlled trials, so most recommendations are made based on highly selected and younger populations. The AGS recommends a patient-centered approach, which begins with understanding the patient’s primary

10

concern and treatment goals.[37] AGS also provides the following recommendations:  Pain is underreported in the older patient so clinicians must make an effort to assess it, even in patients with cognitive impairment. Special pain assessments for patients with cognitive impairment exist, a summary of which has been described in an expert consensus statement.[30]  Because of age-related decrements in drug metabolism and clearance, starting doses should

Chapter 1: Postherpetic neuralgia

   

be low and titration slow with frequent reassessment for dosage adjustments. Use of TCAs above 75–100 mg/day are rarely tolerated by the older patient and their use in this population is “often contraindicated.”[37] If prescribed gabapentin or pregabalin, patients must be monitored closely for sedation, ataxia, and edema. Gabapentin and pregabalin are considered to have a more “benign” adverse effect profile than TCAs.[37] Short-acting opioids are useful for acute recurrent, episodic, or breakthrough pain but total daily dose combination products containing acetaminophen or non-steroidal anti-inflammatory drugs (NSAIDs) should be used carefully to ensure risk of toxic effects of the non-opioid are minimized. Total daily dose of acetaminophen should not exceed more than 4 g/day (less in patients with impaired hepatic function).

The Beers Criteria for Potentially Inappropriate Medications in Older Adults recommends avoiding tertiary TCAs (amitriptyline, imipramine) in the older patient in general, but also notes that tertiary TCAs are particularly problematic for patients with syncope, delirium, dementia, cognitive impairment, and chronic constipation. Tramadol should be avoided in patients with a history of chronic seizures or epilepsy, and anticonvulsants as a class should be avoided in patients with a history of falls or fractures.[37] Based on these considerations, we recommend topical medications for the frail older patient and avoidance of TCAs. Alpha-2 delta ligands have relatively few drug interactions and are a better first choice, but sedation is a significant adverse effect and should be used with caution in patients with gait problems. Of the alpha-2 delta ligands, gastroretentive gabapentin has a lower risk of dizziness and sedation. Data related to gastroretentive gabapentin indicates increased tolerability with a further reduction in the incidence of dizziness and sedation in patients  65 years of age (dizziness, 9.7% versus 12.9%; sedation, 4.0% versus 5.3%, in patients  65 years of age vs. < 65 years of age, respectively).[19] Given the lack of good options for the older patient, the question arises as to the role of opioids in this population. Opioids are second-line options in most guidelines. The long-term risks of these medications cannot and should not be minimized but the AGS

acknowledges that in properly selected and monitored patients, opioid analgesics are “a potentially effective, and in some patients, indispensable treatment as part of a multimodal treatment strategy.”[37: p. 1338] If the clinician chooses opioids, both the clinician and patient must conform to principles of safe opioid prescribing,[38,39] which requires frequent monitoring for efficacy, adverse effects, and abuse. If abuse is confirmed, the clinician must be comfortable exiting the patient off therapy safely, which will require a frank discussion with the patient, a tapering strategy to minimize risk of withdrawal, and a revised care plan to provide pain relief via some other means. Some clinicians may wish to refer such patients to specialists with experience treating patients who have substance abuse or addiction. Although nausea and somnolence with opioids tend to decrease over time, constipation does not and should be anticipated and treated prophylactically.[40] Methadone is not recommended by the AGS as a first-line treatment for pain.[37] Its halflife is variable and conversion to and from other opioids is complicated. Further, many agents can alter methadone levels by inducing or inhibiting the six P450 enzymes involved in its metabolism. Drug accumulation can occur, with potentially fatal consequences. Conversely, drug-drug interactions can reduce methadone levels and precipitate withdrawal.[41] Of the approximately 14 000 deaths attributed to prescription opioids in 2009, over 5000 of them were attributed to methadone.[42] Methadone should only be used by practitioners knowledgeable in its pharmacology and with experience in its use.[37] Opioid-naïve elderly should be started on immediaterelease opioid first, titrated to an effective dose, and then converted, if warranted, to an extended-release formulation.[43] All patients should be instructed to avoid alcohol, benzodiazepines, and barbiturates, as these are CNS depressants and can exacerbate the CNS depressant effects of opioids.[37] Renal/hepatic impairment Pregabalin and gabapentin dose must be reduced in renally impaired patients.[7] Morphine, hydromorphone, and codeine should be used with extreme caution in patients with renal impairment. Morphine and hydromorphone have active metabolites that can accumulate in renally impaired patients and exacerbate the common risks of opioids, and also can result in neuroexcitatory symptoms (in the case of morphine and hydromorphone) and profound

11

Chapter 1: Postherpetic neuralgia

hypotension and narcolepsy (in the case of codeine).[35] Tramadol extended-release is available in limited dosage strengths, so dose adjustments with tramadol are difficult. Fentanyl and methadone appear to be less affected by renal impairment. For reasons stated above, methadone should be avoided, but fentanyl may be a good option if opioid therapy is indicated.[35] In renally impaired patients, dose of opioid should be lower, the interval between doses longer, and creatinine clearance monitored frequently.[44] In patients with significant hepatic impairment, opioid dose should be reduced. Oxymorphone should not be used in patients with moderate to severe hepatic impairment.[35] Dose reductions are required for TCAs but not alpha-2 delta ligands. Patient with anxiety and depression Many patients with anxiety and depression might be receiving medications such as SSRIs and consequently using additional drugs that also raise serotonin (e.g., TCAs) should be used cautiously. Recent evidence suggests that TCAs may be associated with an increased risk of diabetes. In an analysis of three prospective cohort studies (Health Professional Follow-up Study, Nurses’ Health Study I, and Nurses’ Health Study II) totaling over 250 000 subjects, TCAs were associated with a 26% increased risk of diabetes (HR, 1.26: CI, 1.11, 1.42).[45,46] Patients with depression and anxiety also tend to have a higher risk of substance abuse,[35] so any drug with abuse potential should be avoided or used cautiously. Although a history of substance abuse is not an absolute contraindication for opioid therapy, risk of opioid abuse in this type of patient population is significant. A clinician experienced with this subgroup can assess whether opioid therapy is feasible or create a care plan using non-opioid options. The alpha-2 delta ligands are likely a better first choice in these populations. The lack of drug interactions (relative to TCAs) would make them a good choice for the depressed patient being treated with an SSRI or MAOI and their lack of abuse potential make them a better choice over opioids. Furthermore, gabapentin is effective in the treatment of anxiety/social phobia[47,48] and pregabalin has previously demonstrated efficacy in generalized anxiety disorder,[49] making them good choices in patients with anxiety (perhaps providing additive efficacy to other medications while minimizing the potential for increased adverse effects or drug-drug interactions).

12

The patient with a history of substance abuse Any clinician prescribing opioids should be sure to conduct a risk assessment for every patient. A good risk assessment will include current and past history of drug use and family history of drug abuse for each patient. PHN patients with a significant risk of prescription drug abuse generally should not receive opioids, with alpha-2 delta ligands or topicals being a better first choice. In the absence of other options, opioids can be considered, even for patients with a significant risk of abuse, but the prescriber and the patient must establish clear expectations. Furthermore, the prescriber must make a commitment to monitor the patient routinely with urine drug testing and must be comfortable with taking a patient off opioids if evidence of abuse is suspected. This can be achieved in the primary care setting by clinicians who have made a commitment to learn and to apply the principles of safe opioid prescribing. Implementing such principles does take time, which is at a premium in the typical primary care practice. If the clinician cannot make this commitment to safe opioid prescribing, referral to a pain specialist should be considered. Keep in mind, however, that obtaining an appointment to see a pain specialist may take months and a strategy to manage pain in the interim must be in place.

Miscellaneous considerations Patients with cardiomyopathy, valvular disease, or any condition that reduces cardiac output will have reduced renal and hepatic perfusion, which will decrease the rate of elimination of drugs. Starting doses of drugs should be lower, and titration conservative in these patients. Patients with ischemic heart disease or ventricular conduction abnormalities should avoid TCAs, with doses no higher than 100 mg/day.[7] Methadone should not be used in patients with QT prolongation.[35] Patients with cardiovascular or respiratory compromise who are being considered for opioid therapy should be monitored for respiratory depression. Baseline and periodic studies such as oxygen saturation monitoring and/or blood gas analyses, spirometry, ECG, and chest radiographs should be considered.[35] Alpha-2 delta ligands are established antiepileptic agents, providing potential additional advantages; patients with a history of seizures who are on alpha-2 delta ligands but need to be taken off should have their dose of alpha-2 delta ligands titrated downward prior to discontinuation. Very little

Chapter 1: Postherpetic neuralgia

data are available in treating the patient with dementia or cognitive impairment. Amitriptyline should be avoided as it may impair memory.[50] In the context of central neuropathic pain, some experts[50] recommend pregabalin in patients with moderate to severe dementia.

9. What are the key summarized points for treatment of postherpetic neuralgia? Typically, published PHN guidelines are devoid of practical considerations to aid the prescriber in choosing a

References 1.

Gnann JW, Whitley RJ. Clinical practice. Herpes zoster. NEJM. 2002;347:340.

2.

Weaver BA. Herpes zoster overview: natural history and incidence. J Am Osteopath Assoc. 2009;109(6 Suppl 2):S2–6.

3.

Berger A, Dukes EM, Oster G. Clinical characteristics and economic costs of patients with painful neuropathic disorders. J Pain. 2004;5:143–149.

4.

Harpaz R, Ortega-Sanchez IR, Seward JF. Prevention of herpes zoster: recommendations of the Advisory Committee on Immunization Practices (ACIP). MMWR Recomm Rep. 2008;6(57 RR-5):1–30.

5.

6.

Yawn BP, Itzler RF, Wollan PC, et al. Health care utilization and cost burden of herpes zoster in a community population. Mayo Clin Proc. 2009;84(9):787–794. Dubinsky RM, Kabbani H, ElChami Z, Boutwell C, Ali H. Quality Standards Subcommittee of the American Academy of Neurology. Practice parameter: treatment of postherpetic neuralgia: an evidence-based report of the Quality Standards Subcommittee of the American Academy of Neurology. Neurology. 2004;63(6):959–965. Review. PubMed PMID: 15452284.

7.

particular treatment for an individual patient and thus are not able to truly guide the practitioner. Utilizing key prescribing principles such as considering the efficacy, tolerability, and frequency of administration of a particular PHN treatment can be helpful to the clinician when choosing a treatment option for a given patient. Factors beyond the clinician’s immediate control, including third party reimbursement policies, too often influence the choice of medication prescribed for the treatment of PHN in an individual patient, often exposing the patient to a greater risk of intolerable side effects, subtherapeutic dosing of a particular medication, and treatment failure.

Dworkin RH, O’Connor AB, Audette J, et al. Recommendations for the pharmacological management of neuropathic pain: an overview and literature update. Mayo Clin Proc. 2010;85(3 Suppl): S3–14. Review. PubMed PMID: 20194146; PubMed Central PMCID: PMC2844007.

12. Moore RA, Wiffen PJ, Derry S, McQuay HJ. Gabapentin for chronic neuropathic pain and fibromyalgia in adults. Cochrane Database Syst Rev. 2011;(3): CD007938. Review. PubMed PMID: 21412914.

8.

Attal N, Cruccu G, Baron R, et al. European Federation of Neurological Societies. EFNS guidelines on the pharmacological treatment of neuropathic pain: 2010 revision. Eur J Neurol. 2010;17(9):1113–e88. Epub 2010 Apr 9. Review. PubMed PMID: 20402746.

13. Moore RA, Straube S, Wiffen PJ, Derry S, McQuay HJ. Pregabalin for acute and chronic pain in adults. Cochrane Database Syst Rev. 2009;(3):CD007076. Review. PubMed PMID: 19588419. Spring;12(1):13–21. PubMed PMID: 17372630; PubMed Central PMCID: PMC2670721.

9.

Moulin DE, Clark AJ, Gilron I, et al. & Canadian Pain Society. Pharmacological management of chronic neuropathic pain – consensus statement and guidelines from the Canadian Pain Society. Pain Res Manag. 2007;12(1):13–21.

14. Khaliq W, Alam S, Puri N. Topical lidocaine for the treatment of postherpetic neuralgia. Cochrane Database Syst Rev. 2007;(2):CD004846. Review. PubMed PMID: 17443559.

10. Brainin M, Barnes M, Baron JC, et al. Guideline Standards Subcommittee of the EFNS Scientific Committee. Guidance for the preparation of neurological management guidelines by EFNS scientific task forces–revised recommendations 2004. Eur J Neurol. 2004;11(9):577–581. PubMed PMID: 15379736. 11. MacPherson DW. Evidence-based medicine. Can Commun Dis Rep.

1994;20(17):145–147. English, French. PubMed PMID: 7812231.

15. Derry S, Lloyd R, Moore RA, McQuay HJ. Topical capsaicin for chronic neuropathic pain in adults. Cochrane Database Syst Rev. 2009 Oct 7;(4):CD007393. Review. PubMed PMID: 19821411. 16. Edelsberg JS, Lord C, Oster G. Systematic review and metaanalysis of efficacy, safety, and tolerability data from randomized controlled trials of drugs used to treat postherpetic neuralgia. Ann Pharmacother. 2011;45(12):

13

Chapter 1: Postherpetic neuralgia

1483–1490. Epub 2011 Nov 15. Review. PubMed PMID: 22085778. 17. Panesar SS, Philippon MJ, Bhandari M. Principles of evidence-based medicine. Orthop Clin North Am. 2010;41(2):131– 138. PubMed PMID: 20399352. 18. Sackett DL. Evidence-based medicine. Semin Perinatol. 1997;21(1):3–5. PubMed PMID: 9190027. 19. Irving GA, Backonja MM, Dunteman E, et al. NGX-4010 C117 Study Group. A multicenter, randomized, double-blind, controlled study of NGX-4010, a high-concentration capsaicin patch, for the treatment of postherpetic neuralgia. Pain Med. 2011;12(1):99–109. doi: 10.1111/ j.1526-4637.2010.01004.x. Epub 2010 Nov 18. PubMed PMID: 21087403. 20. Webster LR, Tark M, Rauck R, Tobias JK, Vanhove GF. Effect of duration of postherpetic neuralgia on efficacy analyses in a multicenter, randomized, controlled study of NGX-4010, an 8% capsaicin patch evaluated for the treatment of postherpetic neuralgia. BMC Neurol. 2010 Oct 11;10:92. PubMed PMID: 20937130; PubMed Central PMCID: PMC2958861. 21. Irving G, Backonja M, Rauck R, et al. NGX-4010, a capsaicin 8% dermal patch, administered alone or in combination with systemic neuropathic pain medications, reduces pain in patients with postherpetic neuralgia. Clin J Pain. 2012;28(2):101–107. 22. Sang CN, Sathyanarayana R, Sweeney M; for the DM-1796 Study Investigators. Gastroretentive gabapentin (G-GR) formulation reduces intensity of pain associated with postherpetic neuralgia (PHN). Clin J Pain. 2012 Jul 13. [Epub ahead of print] PubMed PMID: 22801243.

14

23. Wallace MS, Irving G, Cowles VE. Gabapentin extended-release tablets for the treatment of patients with postherpetic neuralgia: a randomized, doubleblind, placebo-controlled, multicentre study. Clin Drug Investig. 2010;30(11):765–776. doi: 10.2165/11539520000000000-00000. PubMed PMID: 20818838. 24. Irving G, Sondag E, Sweeney M. Tolerability and safety of oncedaily gabapentin in the treatment of postherpetic neuralgia. Presented at the American Academy of Pain Medicine, 27th Annual Meeting, March 24–27, 2011. National Harbor, MD. Poster #220. 25. Backonja MM, Canafax DM, Cundy KC. Efficacy of gabapentin enacarbil vs placebo in patients with postherpetic neuralgia and a pharmacokinetic comparison with oral gabapentin. Pain Med. 2011;12(7):1098–1108. doi: 10.1111/j.1526-4637.2011.01139.x. Epub 2011 May 31. PubMed PMID: 21627766. 26. Rehm S, Binder A, Baron R. Postherpetic neuralgia: 5% lidocaine medicated plaster, pregabalin, or a combination of both? A randomized, open, clinical effectiveness study. Curr Med Res Opin. 2010;26(7):1607–1619. PubMed PMID: 20429825. 27. Baron R, Mayoral V, Leijon G, Binder A, Steigerwald I, Serpell M. Efficacy and safety of combination therapy with 5% lidocaine medicated plaster and pregabalin in post-herpetic neuralgia and diabetic polyneuropathy. Curr Med Res Opin. 2009;25 (7):1677–1687. PubMed PMID: 19480610. 28. Zin CS, Nissen LM, O’Callaghan JP, Duffull SB, Smith MT, Moore BJ. A randomized, controlled trial of oxycodone versus placebo in patients with postherpetic neuralgia and painful diabetic neuropathy treated with

pregabalin. J Pain. 2010; 11(5):462–471. Epub 2009 Dec 3. PubMed PMID: 19962354. 29. Barbarisi M, Pace MC, Passavanti MB, et al. Pregabalin and transcutaneous electrical nerve stimulation for postherpetic neuralgia treatment. Clin J Pain. 2010;26 (7):567–572. PubMed PMID: 20639738. 30. Hadjistavropoulos T, Herr K, Turk DC, et al. An interdisciplinary expert consensus statement on assessment of pain in older persons. Clin J Pain. 2007 Jan;23(1 Suppl):S1–43. PubMed PMID: 17179836. 31. Ingersoll KS, Cohen J. The impact of medication regimen factors on adherence to chronic treatment: a review of literature. J Behav Med. 2008;31(3):213–224. Epub 2008 Jan 19. Review. PubMed PMID: 18202907; PubMed Central PMCID: PMC2868342. 32. Kaufman DW, Kelly JP, Rosenberg L, Anderson TE, Mitchell AA. Recent patterns of medication use in the ambulatory adult population of the United States: the Slone survey. JAMA. 2002;287(3):337–344. PubMed PMID: 11790213. 33. Qato DM, Alexander GC, Conti RM, et al. Use of prescription and over-the-counter medications and dietary supplements among older adults in the United States. JAMA. 2008;300(24):2867–2878. PubMed PMID: 19109115; PubMed Central PMCID: PMC2702513. 34. Fick DM, Cooper JW, Wade WE, et al. Updating the Beers criteria for potentially inappropriate medication use in older adults: results of a US consensus panel of experts. Arch Intern Med. 2003;163(22):2716–2724. Erratum in: Arch Intern Med. 2004 Feb 9; 164(3):298. PubMed PMID: 14662625.

Chapter 1: Postherpetic neuralgia

35. Smith H, Bruckenthal P. Implications of opioid analgesia for medically complicated patients. Drugs Aging. 2010; 27(5):417–433. doi: 10.2165/ 11536540-000000000-00000. Review. PubMed PMID: 20450239. 36. Kaye AD, Baluch A, Scott JT. Pain management in the elderly population: a review. Ochsner J. 2010;10(3):179–187. PubMed PMID: 21603375; PubMed Central PMCID: PMC3096211. 37. American Geriatrics Society Panel on the Pharmacological Management of Persistent Pain in Older Persons. Pharmacological management of persistent pain in older persons. Pain Med. 2009; 10(6):1062–1083. Epub 2009 Sep 9. Review. PubMed PMID: 19744205. 38. Gourlay D, Heit H. Universal precautions: a matter of mutual trust and responsibility. Pain Med. 2006;7(2):210–211; author reply 212. PubMed PMID: 16634732. 39. Gourlay DL, Heit HA, Almahrezi A. Universal precautions in pain medicine: a rational approach to the treatment of chronic pain. Pain Med. 2005;6(2):107–112. PubMed PMID: 15773874. 40. van Ojik AL, Jansen PA, Brouwers JR, van Roon EN. Treatment of chronic pain in older people:

evidence-based choice of strongacting opioids. Drugs Aging. 2012;29(8):615–625. doi: 10.2165/ 11632620-000000000-00000. PubMed PMID: 22765848. 41. Ferrari A, Coccia CP, Bertolini A, Sternieri E. Methadone– metabolism, pharmacokinetics and interactions. Pharmacol Res. 2004;50(6):551–559. Review. PubMed PMID: 15501692. 42. Warner M, Chen LH, Makuc DM, Anderson RN, Miniño AM. Drug poisoning deaths in the United States, 1980–2008. NCHS Data Brief. 2011;(81):1–8. PubMed PMID: 22617462. 43. Barber JB, Gibson SJ. Treatment of chronic non-malignant pain in the elderly: safety considerations. Drug Saf. 2009;32(6):457–474. doi: 10.2165/00002018-20093206000003. Review. PubMed PMID: 19459714. 44. Gianni W, Ceci M, Bustacchini S, et al. Opioids for the treatment of chronic non-cancer pain in older people. Drugs Aging. 2009;26 (Suppl 1):63–73. doi: 10.2165/ 11534670-000000000-00000. Review. PubMed PMID: 20136170. 45. Pan A, Sun Q, Okereke OI, et al. Use of antidepressant medication and risk of type 2 diabetes: results from three cohorts of US adults. Diabetologia. 2012;55(1):63–72.

Epub 2011 Aug 3. PubMed PMID: 21811871; PubMed Central PMCID: PMC3229672. 46. Kivimäki M, Batty GD. Antidepressant drug use and future diabetes risk. Diabetologia. 2012;55(1):10–12. Epub 2011 Oct 29. PubMed PMID: 22038524; PubMed Central PMCID: PMC3228947. 47. Chouinard G, Beauclair L, Bélanger MC. Gabapentin: longterm antianxiety and hypnotic effects in psychiatric patients with comorbid anxiety-related disorders. Can J Psychiatry. 1998;43(3):305. PubMed PMID: 9561320. 48. Pande AC, Pollack MH, Crockatt J, et al. Placebo-controlled study of gabapentin treatment of panic disorder. J Clin Psychopharmacol. 2000;20(4):467–471. PubMed PMID: 10917408. 49. Feltner D, Wittchen HU, Kavoussi R, et al. Long-term efficacy of pregabalin in generalized anxiety disorder. Int Clin Psychopharmacol. 2008;23(1): 18–28. PubMed PMID: 18090504 50. Scherder EJ, Plooij B. Assessment and management of pain, with particular emphasis on central neuropathic pain, in moderate to severe dementia. Drugs Aging. 2012;29(9):701–706. PubMed PMID: 23018606.

15

Section 1 Chapter

2

Neurological Disorders

Patient with spinal cord injury pain Daniel Krashin, Natalia Murinova, and Alan David Kaye

Case study A 30-year-old high school history teacher is referred to your practice for evaluation of his chest pain and leg pain. He suffered T8 paraplegia in a diving accident a decade ago, while in college. He functions professionally and socially, and is independent in his activities of daily living (ADL), and travels around using a wheelchair. He reports pain in his chest just above the level where he lost sensation. The pain is exacerbated during the day and when he is particularly physically active. He also has burning and aching pain in his legs, which seems to come and go without a pattern, and which is not alleviated by any measures. He is very puzzled by this, since he has no sensation in his legs otherwise, and wonders if this is like the phantom limb pain he has heard about from friends.

1. What is this patient’s diagnosis? This patient is presenting with neuropathic pain related to a spinal cord injury. He has neuropathic pain which is specifically due to nerve root injury and is centrally mediated below the level for neuropathic pain. A broad distinction can be made between musculoskeletal pain, neuropathic pain, and visceral pain in most patients.

2. How many spinal cord injury (SCI) patients are there, and how common is chronic pain in this population? Spinal cord injury is a common medical problem worldwide. In the USA, SCI prevalence is estimated at 721 per million population, or 176 965 persons

alive with SCI.[1] A report in 2005 estimated the number of survivors of traumatic SCI in the USA to be around 273 000.[2] This report also estimated that about 80% of traumatic SCI survivors are male. It has a bimodal distribution; traumatic SCI is more common in younger patients, and non-traumatic is more common in the older population. The most common causes of traumatic SCI are motor vehicle crashes, falls, and gunshot wounds. SCI is associated with significant morbidity and mortality, both directly related to the neurologic deficits and indirect results, such as increased suicides. Higher spinal levels of injury and greater age at time of injury are negative prognostic factors for health and survival. The most common causes of death include pneumonia, sepsis, and cardiovascular disease. Renal disease used to be a major cause of death in SCI but has faded in significance due to improvements in urologic care.[3] One longitudinal study of pain complaints after traumatic SCI showed that 81% of the subjects reported chronic pain. The most common type of pain was musculoskeletal pain at 59%, less likely to be rated as severe. Neuropathic pain at the level of injury was 41% and below the level of the injury was 34%; they were less common and more severe, and visceral pain was found in only 5% of subjects, but was the most likely to be excruciating.[4] This study demonstrated important facts about pain in SCI: it is common and complex, and there are a number of less common pain conditions, which may be extremely severe. In general, about two-thirds of SCI patients report chronic pain, and about one-third report severe pain that interferes with their quality of life and functioning.[5,6]

Case Studies in Pain Management, ed. Alan David Kaye and Rinoo V. Shah. Published by Cambridge University Press. © Cambridge University Press 2015.

16

Chapter 2: Patient with spinal cord injury pain

Table 2.1. Proposed International Spinal Cord Injury Pain (ISCIP) classification

Pain type

Subtype

Examples

Musculoskeletal

Musculoskeletal pain

Shoulder arthritis

Visceral

Visceral pain

Abdominal pain due to impaction

Other

Other nociceptive pain

Postoperative pain

At-level SCI

At-level SCI pain

Nerve root injury, spinal cord compression

Below-level SCI

Below-level SCI pain

Spinal cord compression, thalamic deafferentation pain

Other

Other neuropathic pain

Carpal tunnel syndrome, polyneuropathy

Nociceptive pain

Neuropathic pain

Other pain syndromes

Fibromyalgia, complex regional pain syndrome (CRPS), etc.

Reproduced from TN Bryce. International Spinal Cord Injury Pain Classification: Part I. Background and Description.

3. What are commonly experienced types and examples of pain conditions in SCI patients? In addition to musculoskeletal pain syndromes, which will be detailed separately, the massive nerve injury represented by SCI gives rise to neuropathic pain in many patients, characterized by certain pain qualities such as burning, tingling, stabbing or shocking. This pain can have a clear pattern of a nerve distribution or be diffuse. See Table 2.1 for proposed International Spinal Cord Injury Pain (ISCIP) classification.

4. What types of neuropathic pain are experienced in SCI patients? Neuropathic pain in SCI takes two characteristic forms, at-level and below-level or distal pain. At-level pain is pain is located in the segments associated with the level of the injury. This type of pain is thought to arise from injuries of the local nerve roots and dorsal horn gray matter. This pain may be provoked or exacerbated by activity or changes of position. It can also be a sign of post-traumatic syringomyelia developing near the level of the injury. It should be evaluated with neuroimaging with MRI to rule out syringomyelia, which might further compromise the injured cord.[7] Below-level, or distal, neuropathic pain is pain perceived below the level of spinal cord injury. It can take many forms clinically. This type of pain is attributed to deafferentation due to injury of the

spinothalamic tract and thalamic deafferentation. Without the continuous nerve tract going from the spinal synapses in the dorsal horn to the thalamus, the thalamic nuclei are thought to become hyperexcitable, developing aberrant activity and nerve pain in the distribution of the interrupted tracts. This pain, since it arises from a malfunctioning central nervous system, is generally independent of position or activity. As a central pain, it is often highly resistant to medical management.[8]

5. What form does visceral pain take in SCI patients? Visceral pain is usually described as burning, cramping, fluctuating, or spasmodic. As in other pain patients, the pain is often perceived as “deep” but vaguely localized, and is sometimes referred to other parts of the body. The pain is not always related to abnormal findings on examination.[9]

6. What other painful conditions are SCI patients prone to? Spasticity and contractures are commonly seen in immobile muscles as sequelae of SCI. The prolonged lack of movement and shortening leads to consolidation and tightening of the collagen matrix in muscles, causing them to become increasingly tight and inflexible. This progressive tightening, exacerbated by immobility and physical deconditioning, may predispose patients to pain and loss of function.[10]

17

Chapter 2: Patient with spinal cord injury pain

Spasticity is related to the loss of inhibition from upper motor neurons, leading to increased activity and excitability in skeletal muscles. While spasticity has some health benefits, such as facilitating transfers and improving venous tone, it can also be associated with uncomfortable positions and painful muscle cramps. Preventive treatments for spasticity include regular stretching and braces. Oral medications for spasticity include baclofen, tizanidine, and dantrolene, but their effectiveness is limited and side effects are common. Diazepam has previously been used extensively together with tizanidine and baclofen, but it is problematic since it is a controlled substance with high abuse potential and carries risks of sedation and withdrawal.[11] Focal areas of spasticity can be addressed with local injections of chemodenervation agents such as onabotulinum, which are effective for 3–4 months. More permanent denervation can be achieved through neurolytic agents such as alcohol or phenol. All of these treatments inhibit lower motor neuron effects, decreasing spasticity, but can have effects including neuropathy, decreased function, and systemic spread of agents. Intrathecal drug pumps allow direct access to the CNS, bypassing the blood–brain barrier and avoiding the first-pass effect. This route of administration makes baclofen much more effective at much lower overall doses. As with intrathecal medications for pain, patients generally have a trial of intrathecal baclofen as a one-time injection, followed by objective assessments of residual spasticity symptoms, before the decision is made for implantation. With either oral or intrathecal baclofen, abrupt discontinuation (as can occur after pump failure or catheter fracture) can result in a severe and life-threatening discontinuation syndrome. Those body parts that retain innervation and full function, typically the upper extremities and shoulders, are prone to overuse injuries and pain. The complexities of post-SCI rehabilitation go beyond the scope of this book, but some observations are worthwhile. Shoulder injuries are most common, as patients “use their shoulders as replacements for hips” with wheelchairs and transfers. Wrists, elbows, and hands are also common sites of injury. Rotator cuff injuries are common, along with a variety of other overuse-related bursitis conditions and tendinopathies, osteoarthritis of the overused joints, and carpal tunnel syndrome. Musculoskeletal pain tends

18

to be sharp, dull, or aching and is related to movement and use.[12–14] Preventive strategies include education and rehabilitation for safe transfer practices, such as the strengthening and optimal movements for painful shoulders (STOMPS) program.[15] Ergonomic assessments may be helpful by specialized PT or occupational therapy (OT) services. In some cases, the use of powered wheelchairs may be helpful to preserve upper extremity function. The loss of regular exercise resulting from the switch to a powered wheelchair should be made up with other suitable physical activities, as SCI patients are already at high risk for metabolic complications and heart disease.

7. What other conditions must the pain provider be aware of? Painful conditions in SCI may be complicated by autonomic dysreflexia in cases where the level of injury is at or above T6.[16] This phenomenon of disinhibited sympathetic reflexes is due to the interruption of descending control by sympathetic fibers. These fibers balance the sympathetic and parasympathetic tone to meet the current needs for blood pressure, heart rate, and other autonomic nervous system functions. Noxious stimuli below the level of injury have the potential to trigger a sympathetic “storm” which, if unopposed, can cause severe hypertension, sweating, anxiety, nausea, bradycardia, or tachycardia. The noxious stimuli include pressure sores, fractures, bladder distention, fecal impaction, or distention of other hollow organs. This condition exists along a clinical spectrum ranging from mild hypertension and headache to seizures, stroke, and even end organ failure.[17] Autonomic dysreflexia often develops within months of injury.[18] When this condition is recognized, it may be treated immediately by placing the patient in the upright position, treating their blood pressure, and instituting a methodical examination for possible etiologies, including bladder catheterization and rectal exam, examination of integument and extremities, and possibly obtaining x-ray images. This condition may be treated preventively with antihypertensives as well as avoidance of triggers. This condition is of relevance to pain providers since they are likely to see SCI patients with pain complaints, and they may also cause unintentional discomfort to the patient in the

Chapter 2: Patient with spinal cord injury pain

process of their examination and interventional treatment.

8. What is the relevance of heterotopic ossification? A frequent complication of SCI is heterotopic ossification. This is a form of abnormal bone deposition in the soft tissues adjacent to the large joints below the level of injury. Stem cells near the joints that are normally inactive are switched on by changes associated with SCI. They differentiate into osteoblasts, and secrete disorganized bone around the joints. These depositions of bone cause pain and decreased movement due to inflammation and stiffening of tissues. This condition can mimic local infection, tumor, arthritis, and other diagnoses. These bone deposits take a month or more to be visible on radiographs. Serum alkaline phosphatase levels may be elevated. The most sensitive and specific test is a three-phase bone scan. Heterotopic ossification may respond to NSAIDs and range of motion exercises.[19] Medications commonly used for osteoporosis such as etidronate and bisphosphonates have also been reported to be effective.[19] In severe cases, surgery and radiotherapy have been used to halt progression and restore joint function.[20]

9. What work-up might be helpful to clarify pain diagnosis in an SCI patient? All SCI patients should have a thorough evaluation determining their level of injury and deficits due to SCI documented, so that new deficits or problems can be recognized. New deficits or development of at-level pain should prompt MRI scan of the affected area to rule out post-traumatic syrinx. Physical examination of the patient, including observation while sitting in a chair and during transfers, helps with identification of musculoskeletal problems and overuse syndromes. Documentation of the nature and severity of pain complaints and accompanying sensory abnormalities assists in longitudinal observation and identifying gradual improvement. A panel of experts has determined that the standard 10-point rating scale is reasonable for assessing pain severity, and the 7-Point Guy/Farrar Patient Global Impression of Change (PGIC) scale is recommended for global ratings of pain, while pain interference ratings are best used in assessing the functional impact of their pain.[21]

10. What are the pharmacologic treatments available for patients with SCI pain? Tricyclic antidepressants and membrane stabilizers are the most commonly used treatments, but often have limited benefit. Pregabalin may be more effective than other membrane stabilizers.[22] A large systematic review found that gabapentin and pregabalin appeared to have the greatest effectiveness in management of post-SCI pain.[11] Tricyclic antidepressants and SNRI antidepressants such as duloxetine may be beneficial, particularly in patients with comorbid depression. IV analgesics such as ketamine, opioids, and lidocaine had short-term benefits. Opioids have been used with limited success in the management of this type of pain.[23]

11. Are there interventional treatments that have been shown to be beneficial for SCI patients? Chemodenervation and neurolysis have been used for specific neuropathies and spasticity. Neurosurgical procedures of nerve root or spinal cord tract destruction have also been used for SCI-related pain, including rhizotomy, dorsal root entry zone (DREZ), myelotomy, cordotomy, and cordectomy.[24]

12. What are the non-pharmacologic treatments effective for SCI pain? Pharmacotherapy has limited benefits in some patients, who are unable to take the optimal dose of medications due to adverse side effects. Other nonpharmacologic approaches should be tried; these include acupuncture and biofeedback, which have been used for treatment-resistant SCI pain.[17] For specific functional or mechanical issues physical therapy can be especially helpful. Transcutaneous nerve stimulations (TENS) and massage can be effective in some patients. Cannabis has been used as a pain reliever in SCI, but has not been studied well. It is reported to benefit central pain and spasticity in SCI. While there are many anecdotal reports of benefit, it is noted to have a narrow therapeutic window and study results have been mixed and not very impressive.[25]

19

Chapter 2: Patient with spinal cord injury pain

13. Are there particular issues or sensitivities that you need to be aware of when working with SCI patients? Depression is extremely common after SCI with rates of more than 40% within the first few months. Suicide rates for the first 5 years after SCI are dramatically higher compared to age-matched uninjured controls.[26,27] Since depression is a risk factor for suicide and is also associated with worsening outcomes in pain treatment, it is important to screen SCI patients for depression regularly even if that is not the focus of treatment.

Summary Pain significantly amplifies suffering in people with spinal cord injury, and diminishes their quality of life. Pain following SCI is very common, and one of the most difficult problems to manage. The pain is likely maintained by a number of different pathophysiologic mechanisms; this suggests that using therapy

References 1.

2.

3.

4.

5.

20

DeVivo MJ, Chen Y. Trends in new injuries, prevalent cases, and aging with spinal cord injury. Arch Phys Med Rehabil. 2011; 92(3):332–338. National Spinal Cord Injury Statistical Center. The 2005 Annual Statistical Report for the Model Spinal Cord Injury Care Systems. Birmingham, Alabama: National Spinal Cord Injury Statistical Center. 2005. Frankel HL, Coll JR, Charlifue SW, et al. Long-term survival in spinal cord injury: a fifty year investigation. Spinal Cord. 1998;36(4):266–274. Siddall PJ, McClelland JM, Rutkowski SB, Cousins MJ. A longitudinal study of the prevalence and characteristics of pain in the first 5 years following spinal cord injury. Pain. 2003; 103(3):249–257. Burchiel KJ, Hsu FPK. Pain and spasticity after spinal cord injury: mechanisms and treatment. Spine. 2001;26(24S):S146–S160.

6.

7.

aimed at many different pain-related targets is more advantageous. The rapid progress of neuroscience research yields ever more potential techniques and targets for modulating pain. At the same time, the gap between laboratory bench and clinical application has grown wider, and the rate at which new treatments become available has been dwindling for some time.[28] Clinicians must therefore make the most of the medications that are currently available for treating this form of pain, particularly using neuropathic pain standbys such as tricyclic antidepressants and anticonvulsants.[29] The best available pharmacologic treatments provide only approximately 30% of people with a 50% reduction in their pain and have unacceptable side effects.[30] Combining pharmacological, interventional, and non-pharmacologic approaches is critical in these complicated pain patients, especially when dealing with their central pain. Pain continues to present a major challenge to those with spinal cord injury and their providers, and more research is urgently needed.

Simpson DM, Gracies J-M, Yablon SA, Barbano R, Brashear A. Botulinum neurotoxin versus tizanidine in upper limb spasticity: a placebo-controlled study. J Neurol Neurosurg Psychiatry. 2009;80(4):380–385. Wasner G, Lee BB, Engel S, McLachlan E. Residual spinothalamic tract pathways predict development of central pain after spinal cord injury. Brain. 2008;131(9): 2387–2400.

8.

Davidoff G, Roth E, Guarracini M, Sliwa J, Yarkony G. Functionlimiting dysesthetic pain syndrome among traumatic spinal cord injury patients: a crosssectional study. Pain. 1987; 29(1):39–48.

9.

Bockenek WL, Stewart PJ. Pain in patients with spinal cord injury. In Spinal Cord Medicine. Lippincott Williams & Wilkins. 2002: pp. 389–408.

10. Adams MM, Hicks AL. Spasticity after spinal cord injury. Spinal Cord. 2005;43(10):577–586.

11. Teasell RW, Mehta S, Aubut J-AL, et al. A systematic review of pharmacologic treatments of pain after spinal cord injury. Arch Phys Med Rehabil. 2010;91(5):816–831. 12. Gellman H, Sib IEN, Waters RL. Late complications of the weightbearing upper extremity in the paraplegic patient. Clin Orthopaed Rel Res. 1988;233:132–135. 13. Hastings J, Goldstein B. Paraplegia and the shoulder. Phys Med Rehabil Clin North Am. 2004;15(3):699–718. 14. Sinnott KA, Milburn P, McNaughton H. Factors associated with thoracic spinal cord injury, lesion level and rotator cuff disorders. Spinal Cord. 2000;38(12):748. 15. Mulroy SJ, Thompson L, Kemp B, et al. Strengthening and optimal movements for painful shoulders (STOMPS) in chronic spinal cord injury: a randomized controlled trial. Physical Therapy. 2011; 91(3):305–324. 16. Bycroft J, Shergill IS, Choong EAL, Arya N, Shah PJR.

Chapter 2: Patient with spinal cord injury pain

Autonomic dysreflexia: a medical emergency. Postgrad Med J. 2005;81(954):232–235. 17. Kirshblum SC, Priebe MM, Ho CH, et al. Spinal cord injury medicine. 3. Rehabilitation phase after acute spinal cord injury. Arch Phys Med Rehabil. 2007; 88(3):S62–S70. 18. Helkowski WM, Ditunno Jr JF, Boninger M. Autonomic dysreflexia: incidence in persons with neurologically complete and incomplete tetraplegia. J Spinal Cord Med. 2003;26(3):244. 19. Teasell RW, Mehta S, Aubut JL, et al. A systematic review of the therapeutic interventions for heterotopic ossification after spinal cord injury. Spinal Cord. 2010;48(7):512–521. 20. Freebourn TM, Barber DB, Able AC. The treatment of immature heterotopic ossification in spinal cord injury with combination surgery, radiation therapy and NSAID. Spinal Cord. 1999; 37(1):50.

21. Bryce TN, Budh CN, Cardenas DD, et al. Pain after spinal cord injury: an evidence-based review for clinical practice and research: report of the National Institute on Disability and Rehabilitation Research Spinal Cord Injury Measures Meeting. J Spinal Cord Med. 2007;30(5):421. 22. Siddall PJ, Cousins MJ, Otte A, Griesing T, Chambers R, Murphy TK. Pregabalin in central neuropathic pain associated with spinal cord injury: A placebocontrolled trial. Neurology. 2006;67(10):1792–1800. 23. Siddall PJ, Molloy AR, Walker S, et al. The efficacy of intrathecal morphine and clonidine in the treatment of pain after spinal cord injury. Anesth Analg. 2000;91(6): 1493–1498. 24. Denkers MR, Biagi HL, O’Brien MA, Jadad AR, Gauld ME. Dorsal root entry zone lesioning used to treat central neuropathic pain in patients with traumatic spinal cord injury: a systematic review. Spine. 2002;27(7):E177–E184.

25. Karst M, Wippermann S, Ahrens J. Role of cannabinoids in the treatment of pain and (painful) spasticity. Drugs. 2010;70(18): 2409–2438. 26. North NT. The psychological effects of spinal cord injury: a review. Spinal Cord. 1999;37(10): 671. 27. DeVivo MJ, Black KJ, Richards JS, Stover SL. Suicide following spinal cord injury. Spinal Cord. 1991;29(9):620–627. 28. Cuatrecasas P. Drug discovery in jeopardy. Journal of Clinical Investigation. 2006;116(11): 2837–2842. 29. Kroenke K, Krebs EE, Bair MJ. Pharmacotherapy of chronic pain: a synthesis of recommendations from systematic reviews. Gen Hosp Psychiatry. 2009;31(3): 206–219. 30. Siddall PJ. Management of neuropathic pain following spinal cord injury: now and in the future. Spinal Cord. 2008;47(5): 352–359.

21

Section 1 Chapter

3

Neurological Disorders

Patient with poststroke pain Natalia Murinova, Claire Creutzfeldt, Daniel Krashin, and Alan David Kaye

Case study A 65-year-old woman presents to the emergency room complaining of severe right face and arm pain. She had been recovering from a minor stroke that she had suffered 3 months previously that had caused only numbness in her right side. This numbness had developed into tingling and then pain, especially to touch. Her arm felt “like it was on fire” almost continuously, to the point that she wanted to cut it off. She denied any other focal neurologic deficits. She has history of obesity, diabetes mellitus, and poorly controlled hypertension. She admitted to forgetting to take her amlodipine and aspirin occasionally. A detailed neurologic exam revealed subtle weakness on motor testing on the right side. Assessing the right side with light touch induced pronounced painful response (allodynia). Light pinprick induced an exaggerated amount of pain (e.g., hyperalgesia). Hyperalgesia was limited to right arm and torso and was not present distal to the beltline. Scratching with a pin along the upper arm produced a persistent “pricking” sensation in the hand (hyperpathia). Once the pain was “stirredup” in the right upper body and extremity, it persisted for over a minute. Her blood pressure was 205 mmHg systolic and her head CT showed an old lacunar infarct in the left medial thalamus.

origin is considered one of the most severe pain syndromes, with severe, burning hemibody pain contralateral to the thalamic lesion.[1] The poststroke pain belongs to a group of central pain syndromes. The first introduction to central pain was by Edinger in 1891.[2] The International Association for the Study of Pain (IASP) defines central pain as “pain initiated or caused by a primary lesion or dysfunction of the central nervous system” at levels of spinal cord, brainstem, or cerebral hemispheres.[3] Central poststroke pain (CPSP) is a type of central pain that occurs after a cerebrovascular accident. Patients with CPSP suffer from constant or intermittent pain, and can have changes of thermal sensation. The pain quality has been depicted as burning, freezing, and some people find it difficult to describe. Misdiagnosis and delay are common, especially if patients have cognitive and expression difficulties poststroke. Patients may also display spontaneous abnormal sensations and stimulus-evoked dysesthesia, allodynia, and hyperalgesia. Other painful syndromes such as headache, painful spasms, contractures, hemiplegic shoulder pain, and other musculoskeletal pain can further complicate the clinical presentation of CPSP.[1]

2. How common is poststroke pain? 1. What is the diagnosis explaining this patient’s pain? This patient has Déjerine-Roussy syndrome, a variant of poststroke pain (CPSP) caused by infarction in the thalamus. More than 100 years ago, Déjerine and Roussy characterized thalamic pain as “among the most spectacular, distressing, and intractable pain syndromes”.[1] Central poststroke pain of thalamic

19–74% of stroke patients suffer pain as a complication of stroke.[4] About 795 000 people suffer a stroke annually in the USA, 600 000 of which are first attacks; this means that at least 150 000 people develop stroke-related pain each year. There are more than 7 million stroke survivors alive in the USA today.[5] Stroke is a leading cause of disability and the third leading cause of death in the USA.[5] Central pain is less common in stroke than in spinal cord

Case Studies in Pain Management, ed. Alan David Kaye and Rinoo V. Shah. Published by Cambridge University Press. © Cambridge University Press 2015.

22

Chapter 3: Patient with poststroke pain

Table 3.1. Number of patients with pain in three different disorders with central pain

Condition

Survivors in USA

Newly diagnosed

All pain

Central pain

Stroke

~7 000 000[5]

795 000[5]

19–74%[4]

1–10%[6]

MS

~211 000[7]

10 400 (estimate)

57.5%[8]

27%[8]

SCI

~273 000[9]

12 000[10]

81%[11]

34–67%[11,12]

~, approximately, as exact numbers are not obtainable.

injury (SCI) or multiple sclerosis (MS); however the incidence of stroke is greater than these disorders, and because of this central pain in stroke is much more commonly encountered than in the other conditions (Table 3.1).

3. What is the incidence of CPSP (epidemiology)? Estimates of the prevalence of CPSP range from 1 to 8%. This fraction of poststroke pain is related directly to the brain area affected by the cerebrovascular accident.[4] It has been estimated that in the USA at least 56 000 cases of CPSP occur each year, if we estimate 700 000 new and recurrent cases of stroke.[6]

Table 3.2. The most common forms of chronic poststroke pain

Poststroke pain type

Percentage of patients with stroke experiencing this kind of pain

Musculoskeletal pain

40%

Shoulder pain

20%

Headache

20%

Central poststroke pain

10%

Spasticity

7%

4. What painful conditions are commonly seen in poststroke patients? See Table 3.2 and Figure 3.1. Shoulder pain is commonly seen in hemiplegia. Studies report the incidence of hemiplegic shoulder pain to be 38% to 84%. The definition of shoulder pain and the time of diagnosis, and period from stroke onset to the development of pain and recognition of pain likely influence the numbers reported.[13–17] When headache types were analyzed poststroke, the most commonly seen types were tension-type headache and migraine; the persistent daily headaches were usually tension-type.[18,19] There is really not much literature to suggest the treatment of headaches in patients poststroke. We recommend using International Headache Society criteria for diagnosing the headache type, and depending on the diagnosis establishing the appropriate treatment.[20] Medication overuse mediated headache, which is commonly seen in headache clinics, especially in people with history of migraine or family history of migraine, was not studied in this population of patients. Further

Figure 3.1. Common types of chronic poststroke pain, showing their relative frequency and overlap in a stroke patient (figure designed by Authors).

research needs to be done if medication overuse mediated headaches are also present in this population of patients, and contribute to their worsening headaches. Some migraine patients, when overusing

23

Chapter 3: Patient with poststroke pain

medications for headaches, develop a cycle of daily or near-daily headaches, which is termed medication overuse headache.[21] Central poststroke pain has a reported incidence of from 2 to 8%; the actual figure is likely to be higher, but it is often misdiagnosed or missed due to difficulty of diagnosis.[22–24] Spino-thalamo-cortical pathway involvement appears to be the major factor in development of CPSP. The lesion level contributes to the associated symptoms and pain features.[22] CPSP can occur after a cerebrovascular accident affecting any level of somatosensory pathways of the brain – this includes medulla, thalamus, and cerebral cortex. The occurrence of CPSP is especially high if the location of the lesion is in the lateral medulla (Wallenberg’s syndrome) or thalamus (ventro-posterior part).[1] When measured 1 year after stroke, painful spasticity is present in 27% to 36% of patients who suffered stroke.[25]

5. What area of the brain injury correlates with the development of central poststroke pain? Sprenger et al tried to identify specific “central painrelated” thalamic nuclei using structural magnetic resonance imaging in poststroke patients. In this study they found that the “ventral posterior nucleus and the pulvinar, coinciding with the ventrocaudalis portae nucleus” lesions correlate with development of thalamic pain. The implication is that structural imaging might be useful for early detection of patients at risk for CPSP, leading to the development of effective pre-emptive treatment.[26]

6. What is the likely mechanism of poststroke pain? What is the likely mechanism of CPSP? Central poststroke pain is likely influenced by a large number of factors with potentially complex interactions. There have been many suggestions as to the causes of central pain; however, they are not well understood. The central pain evolves very likely as a combination of multiple modulating pathophysiologic processes, which include attenuated central inhibition, imbalance of chemical stimuli, and central sensitization.[27] Central sensitization is thought to

24

be the main mechanism that is responsible for the chronic pain associated with the central nervous system. The associated features of central pain include increased activation of N-methyl-D-aspartate (NMDA) and sodium channels.[28] Evidence suggests that chronic pain can alter brain function. The phenomenon of central sensitization is thought to be a prolonged but reversible augmentation in the excitability and synaptic efficacy of the neurons in the central pain pathway.[29] The changes of the brain are seen in reduced neuronal firing threshold, augmented spontaneous firing, and heightened firing during repetitive stimulation.[30] There are many different areas of the central nervous system that have been associated with central sensitization caused by pain. Increased plasticity of the synapses in the CNS, alterations of receptor function, and acquired channelopathies all appear to play a role in this process of central sensitization. Activation of the microglia can cause an inflammatory response which also contributes to amplification of pain-related activity in the nociceptive networks of the CNS.[31]

What is the likely mechanism of shoulder pain? The prevalence of hemiplegic shoulder pain is proportional to the degree of weakness, therefore it is highest in patients presenting with a plegic shoulder.[32] The proposed mechanism underlying hemiplegic shoulder pain (HSP) is a subluxation of the shoulder joint in the setting of both sensory and motor deficits as well as limited passive range of motion. Adhesive capsulitis may be contributory,[33] as is spastic shoulder pain, which shows a pattern of adduction and internal rotation of the shoulder. Although HSP is often thought of as a musculoskeletal pain, a neuropathic component is suggested by the common association with chronic pain throughout the affected side as well as the location of infarction in areas responsible for pain perception and processing.[34]

What is known about spasticity after stroke? Hemihypesthesia is more frequently found in patients with spasticity of the upper and lower limb and is more frequent than in patients without sensory deficits (P ≤ 0.001).[35] In a study by Urban et al spasticity was reported in 42.6% of patients with initial central paresis.[35]

Chapter 3: Patient with poststroke pain

What is known about headache poststroke? There are not many studies addressing headache poststroke. About 10% of patients develop headache poststroke, many of them daily and persistent. Headache poststroke diagnosed using International Headache Society criteria[20] are most commonly tension-type headache and migraine. The mechanism specific to stroke is not addressed in the literature.

7. What is the work-up of poststroke pain? The first step in working up poststroke pain is a high clinical suspicion in patients with stroke. Many stroke survivors are unable to express their pain due to language or cognitive impairment. This group of patients is also less likely to receive adequate pain treatment.[36]

Diagnosis of CPSP Key to the diagnosis of CPSP is (1) the neuroimaging association (CT or MRI) of the pain with an infarct in the corresponding area of the brain and (2) the exclusion of any other causes of neuropathic pain.[1] A careful history and physical exam should be done to look for signs of peripheral nerve or tissue damage, focal neurologic deficits not consistent with the area of the infarction, and other common pain syndromes as discussed below. A depression screen should be considered as depression can exacerbate pain and vice versa, and this information may aid in the choice of pharmacologic treatment (below).

Hemiplegic shoulder pain work-up While shoulder MRI may reveal adhesive capsulitis,[33] or soft tissue injury, a shoulder x-ray will suffice to rule out causes such as fractures or dislocation.

8. What pharmacologic treatments are available for poststroke pain? What treatments are available for CPSP? Our rudimentary understanding of the pathophysiology and neurobiology underlying CPSP makes it challenging to develop novel treatment targets. Large controlled trials to guide the management of CPSP are lacking, in part due to the heterogeneity of the underlying strokes, and the variability in pain quality

and intensity as well as the individual response rate of affected patients.[37] The current drug of choice is the tricyclic antidepressant amitriptyline or the metabolite nortriptyline.[38] This recommendation is based on small studies, and side effects such as dry mouth, drowsiness, and constipation are common. Recommendations for other antidepressants such as venlafaxine and some selective serotonergic receptor inhibitors are based on good evidence supporting their effect on neuropathic pain.[39] Antidepressants should be considered in patients with concurrent depression. Other first-line agents include the antiepileptic drug lamotrigine[40] and gabapentin, whereby the evidence for the latter is extrapolated from its effectiveness in treating neuropathic pain syndromes.[41] There is no conclusive evidence for phenytoin, zonisamide, and topiramate effectiveness in CPSP.[42–44] The effect of opioids on CPSP is questionable (Table 3.3).[45]

What are the treatments available for hemiplegic shoulder pain? Whereas the neuropathic component of HSP may respond to pharmacologic treatment as described above (amitriptyline, lamotrigine, or gabapentin), non-pharmacologic treatments such as ice, heat, and soft tissue massage are commonly recommended. Strengthening of the shoulder girdle with physical therapy may reduce dislocation of the joint, while passive range of motion may reduce the risk of adhesive capsulitis. Many patients wear a shoulder sling at night and/or during ambulation to support the arm and to prevent upper extremity trauma. Overhead movement is best avoided to reduce the risk of subluxation. Non-steroidal anti-inflammatory agents have temporizing pain relief and can be used prior to physical therapy. Intramuscular BOTOX injections and neuromuscular electric stimulation may be helpful.[48] The prognosis for HSP is generally good, with most patients improved at 6 months.[49]

9. Are there any interventional treatments that have been tried for the treatment of patients with poststroke pain? Spinal cord and deep brain stimulation are not effective for CPSP.[50] Motor cortex stimulation fares

25

Chapter 3: Patient with poststroke pain

Table 3.3. Medications studied in central poststroke pain

Medication

Dosage

Number of patients; response to treatment

Notes

Amitriptyline[22]

75 mg[22]

15 patients; 10/15 patients responded[22]

Very effective

Carbamazepine[22]

750 mg[22]

15 patients; 5 of the 14 patients who completed the study responded No statistical significance[22]

Lamotrigine[40]

200 mg[40]

30 patients; 12/30 partial response[40]

Moderately effective

Gabapentin[46]

Up to 2400 mg

23 patients; only two had poststroke pain – not clear if it had effect on these patients

Moderate relief

Phenytoin[42]

150 mg[42]

2 case reports[42]

Significant toxicity

Topiramate[44]

50–200 mg 3 times daily[44]

3 patients; no patient showed CPSP relief[44]

Zonisamide[43]

200 mg[43]

2 case reports[43]

Pregabalin[47]

150–600 mg/day[47]

219 patients, placebo-controlled; no significant pain relief[47]

much better as a treatment for refractory CPSP, a stimulation electrode being placed contralateral to the side of the pain. Cortical stimulation via corticocortical and maybe cortico-thalamic pathways inhibits the perception of pain.[50,51]

10. What are some special concerns in poststroke pain? Chronic poststroke pain is one of the most devastating outcomes of stroke according to patients; however, research into the quality of life (QOL) of these patients is limited. Small studies have shown decreases in health-related QOL.[52] Poststroke shoulder pain in particular is associated with decreased QOL, although this finding is not related to actual functional impairment.[53] A Korean study of patients 6 months or more following stroke found that 42% complained of chronic pain, but their QOL did not differ significantly from that of other patients in the sample.[54] The population of stroke survivors is changing demographically. About half of stroke patients are now under 65 years of age, with a significant number of patients younger than 55, still in economically productive years. Traditionally stroke patients have been discharged to a variety of settings depending on their functional limitations, but expectations are relatively modest. Younger patients with longer life expectancies and milder strokes have greater

26

Well tolerated

expectations regarding restoration of function and quality of life. They have a variety of complex psychosocial needs in addition to the traditional rehabilitation issues, including concerns about independent living, social and occupational fun, functioning, and the ability to fill family roles.[55] It has been shown that the families of stroke survivors also suffer significantly.[56] Comorbid conditions such as poststroke pain and poststroke depression have a large impact on a patient’s ability to face these challenges, but these issues are often neglected, particularly in those patients who do not require extensive rehabilitation and supportive living environments. Many of these patients flounder when they return home and are not able to resume active lives or employment.[57] This is a great loss for the patients, who frequently perceive this as an important marker of recovery and suffer greatly from its absence.[58,59] It is also a large cost to the economy: it has been estimated that of the $65 billion annual cost of stroke in the USA, a third is accounted for by indirect costs related to lost productivity.[60] In the EU, indirect stroke costs make up a similar 31% of the 27 billion euro annual cost of stroke. Despite these large economic and personal impacts, polls of stroke patients suggest that they feel little attention is paid to their concerns about fatigue, employability, and function.[61] Stroke patients also feel significant losses of autonomy after stroke, and appear to benefit from increased support for independence during and after rehabilitation.[62]

Chapter 3: Patient with poststroke pain

Conclusion/summary Stroke is, as we have discussed, an increasingly common condition associated with very significant rates of disability, morbidity, and death among adults. In addition to the obvious neurologic deficits that many are left with, such as aphasia, loss of coordination or function in a limb, and difficulties with activities of daily living, many stroke patients also develop chronic pain conditions.[54] Along with depression, chronic pain is a very common complication of stroke that can have a large impact on a patient’s quality of life. Recognition of poststroke pain can be extremely challenging. Musculoskeletal pain conditions may not be recognized as being related to the stroke by the physician or the patient. Pain quality and distribution can vary among patients, and stroke patients frequently have more than one type of pain.[22] The highest diagnostic yield is probably obtained by careful assessment of the nature of the pain complaints, examination of sensory response, and correlation with neuroimaging data.[1] This should help determine the cause of pain and rule out other causes, but it should be noted that not every encountered CPSP is due to thalamic stroke.[22] Given the large proportion of stroke patients who develop chronic poststroke pain, pain symptoms should be routinely assessed by clinicians. Risk factors for developing poststroke pain are not fully

References 1.

Klit H, Finnerup NB, Jensen TS. Central post-stroke pain: clinical characteristics, pathophysiology, and management. Lancet Neurol. 2009;8(9):857–868.

2.

Edinger L. Giebt es central entstehende Schmerzen? Dtsch Z Nervenheilkd. 1891;1:262–282.

3.

Merskey H, Bogduk N. International Association for the Study of Pain. Task Force on Taxonomy. Classification of Chronic Pain: Descriptions of Chronic Pain Syndromes and Definitions of Pain Terms. Seattle: IASP Press; 1994.

4.

Kim JS. Post-stroke pain. Expert Rev Neurother. 2009;9(5): 711–721.

5.

6.

7.

8.

understood, although it appears to be associated with paresis and sensory changes, and with comorbid depression.[63] It is not known whether CPSP shares any mechanisms or risk factors with other pain conditions attributed to central sensitization. The clinical usefulness of anticonvulsants in both types of pain disorders provides a suggestion that there is some common pathophysiology that is being addressed, possibly increased sensitization and excitability in the nociceptive nerve pathway. If this hypothesis proves correct, it may be possible to identify patients at increased risk for developing chronic pain early, and to institute preventive measures.[29] Since there are no gold-standard diagnostic criteria or treatments for chronic poststroke pain, treatment must be empirical, driven by the diagnosis and guided by patient response. Since an estimated 795 000 people each year in the USA have a new or recurrent stroke, this is a highly significant pain condition.[64] It is important to remember that poststroke pain patients may be affected simultaneously with many other sequelae, including fatigue, motor and sensory abnormalities, emotional lability, strained relationships, disability, and depression.[65] Many of these problems can respond to treatment; the challenge is to address them comprehensively. Improving the evidence base for treating complications and sequelae of stroke is imperative.

Center IS. Stroke Center Statistics. 2006; Statistics about strokes. Available at: http://www. strokecenter.org/patients/stats. htm (accessed 10/08/2013, 2013). Thom T, Haase N, Rosamond W, et al. Heart disease and stroke statistics – 2006 update: a report from the American Heart Association Statistics Committee and Stroke Statistics Subcommittee. Circulation. 2006;113(6):e85–e151. Noonan CW, Kathman SJ, White MC. Prevalence estimates for MS in the United States and evidence of an increasing trend for women. Neurology. 2002;58(1):136–138. Österberg A, Boivie J, Thuomas KÅ. Central pain in multiple sclerosis: prevalence and clinical

9.

characteristics. Eur J Pain. 2005; 9(5):531–542. National Spinal Cord Injury Statistical Center. The 2005 Annual Statistical Report for the Model Spinal Cord Injury Care Systems. Birmingham, Alabama: National Spinal Cord Injury Statistical Center. 2005.

10. DeVivo MJ. Epidemiology of spinal cord injury. In Lin VW, Bono CM, Cardenas DC, eds. Spinal Cord Medicine Principles and Practice. New York, NY: Demos Medical Publishing. 2010: pp. 78–84. 11. Siddall PJ, McClelland JM, Rutkowski SB, Cousins MJ. A longitudinal study of the prevalence and characteristics of pain in the first 5 years following

27

Chapter 3: Patient with poststroke pain

spinal cord injury. Pain. 2003; 103(3):249–257. 12. Finnerup NB, Johannesen IL, Sindrup SH, Bach FW, Jensen TS. Pain and dysesthesia in patients with spinal cord injury: a postal survey. Spinal Cord. 2001;39(5): 256–262. 13. Griffin JW. Hemiplegic shoulder pain. Physical Therapy. 1986; 66(12):1884–1893. 14. Wanklyn P, Forster A, Young J. Hemiplegic shoulder pain (HSP): natural history and investigation of associated features. Disabil Rehabil. 1996; 18(10):497–501.

24. Bowsher D. The management of central post-stroke pain. Postgrad Med J. 1995;71(840):598–604. 25. Watkins CL, Leathley MJ, Gregson JM, et al. Prevalence of spasticity post stroke. Clin Rehabil. 2002;16(5):515–522. 26. Sprenger T, Seifert CL, Valet M, et al. Assessing the risk of central post-stroke pain of thalamic origin by lesion mapping. Brain. 2012;135(8):2536–2545.

15. Roy CW, Sands MR, Hill LD. Shoulder pain in acutely admitted hemiplegics. Clin Rehabil. 1994; 8(4):334–340.

27. Kumar B, Kalita J, Kumar G, Misra UK. Central poststroke pain: a review of pathophysiology and treatment. Anesth & Analg. 2009;108(5):1645–1657.

16. Bohannon RW, Larkin PA, Smith MB, Horton MG. Shoulder pain in hemiplegia: statistical relationship with five variables. Arch Phys Med Rehabil. 1986; 67(8):514–516.

28. Wiesenfeld-Hallin Z, Aldskogius H, Grant G, et al. Central inhibitory dysfunctions: mechanisms and clinical implications. Behav Brain Sci. 1997;20(3):420–425.

17. Teasell RW, McRae M. The painful hemiplegic shoulder. Phys Med Rehabil. 1998;12:489–500.

29. Woolf CJ. Central sensitization: implications for the diagnosis and treatment of pain. Pain. 2011; 152(3):S2–S15.

18. Vestergaard K, Andersen G, Nielsen MI, Jensen TS. Headache in stroke. Stroke. 1993;24(11): 1621–1624. 19. Ferro JM, Melo TP, Guerreiro M. Headaches in intracerebral hemorrhage survivors. Neurology. 1998;50(1):203–207. 20. Headache Classification Committee of the International Headache Society. The International Classification of Headache Disorders, 3rd edition (beta version). Cephalalgia. 2013;33(9):629–808. 21. Tepper SJ, Tepper DE. Breaking the cycle of medication overuse headache. Cleveland Clin J Med. 2010;77(4):236–242. 22. Leijon G, Boivie J. Central poststroke pain: a controlled trial of amitriptyline and carbamazepine. Pain. 1989;36(1):27–36.

28

23. Andersen G, Vestergaard K, Ingeman-Nielsen M, Jensen TS. Incidence of central post-stroke pain. Pain. 1995;61(2):187–193.

30. McMahon SB, Lewin GR, Wall PD. Central hyperexcitability triggered by noxious inputs. Curr Opin Neurobiol. 1993;3(4): 602–610. 31. Saab CY. Pain-related changes in the brain: diagnostic and therapeutic potentials. Trends Neurosci. 2012;35(10):629-637. 32. Lindgren I, Jönsson A-C, Norrving B, Lindgren A. Shoulder pain after stroke: A prospective population-based study. Stroke. 2007;38(2):343–348. 33. Távora DGF, Gama RL, Bomfim RC, Nakayama M, Silva CEP. MRI findings in the painful hemiplegic shoulder. Clin Radiol. 2010; 65(10):789–794. 34. Zeilig G, Rivel M, Weingarden H, Gaidoukov E, Defrin R. Evidence of a neuropathic origin in

hemiplegic shoulder pain. Pain. 2013;154(2):263–271. 35. Urban PP, Wolf T, Uebele M, et al. Occurence and clinical predictors of spasticity after ischemic stroke. Stroke. 2010; 41(9):2016–2020. 36. Kehayia E, Korner-Bitensky N, Singer F, et al. Differences in pain medication use in stroke patients with aphasia and without aphasia. Stroke. 1997;28(10):1867–1870. 37. Gordon A. Best Practice Guidelines for Treatment of Central Pain after Stroke. Central Neuropathic Pain: Focus on Poststroke Pain. Seattle: IASP Press. 2007. 38. Creutzfeldt CJ, Holloway RG, Walker M. Symptomatic and palliative care for stroke survivors. J Gen Intern Med. 2012;27(7): 853–860. 39. Saarto T, Wiffen PJ. Antidepressants for neuropathic pain. Cochrane Database Syst Rev. 2007;4(4). 40. Vestergaard K, Andersen G, Gottrup H, Kristensen BT, Jensen TS. Lamotrigine for central poststroke pain: a randomized controlled trial. Neurology. 2001;56(2):184–190. 41. Serpell MG. Gabapentin in neuropathic pain syndromes: a randomised, double-blind, placebo-controlled trial. Pain. 2002;99(3):557–566. 42. Cantor FK. Phenytoin treatment of thalamic pain. Br Med J. 1972; 4(5840):590. 43. Takahashi Y, Hashimoto K, Tsuji S. Successful use of zonisamide for central poststroke pain. J Pain. 2004;5(3):192–194. 44. Canavero S, Bonicalzi V, Paolotti R. Lack of effect of topiramate for central pain. Neurology. 2002; 58(5):831–832. 45. Frese A, Husstedt IW, Ringelstein EB, Evers S. Pharmacologic treatment of central post-stroke pain. Clin J Pain. 2006;22(3): 252–260.

Chapter 3: Patient with poststroke pain

46. Attal N, Brasseur L, Parker F, Chauvin M, Bouhassira D. Effects of gabapentin on the different components of peripheral and central neuropathic pain syndromes: a pilot study. Eur Neurol. 1998;40 (4):191–200. 47. Kim JS, Bashford G, Murphy TK, et al. Safety and efficacy of pregabalin in patients with central post-stroke pain. Pain. 2011;152 (5):1018–1023. 48. Singh JA, Fitzgerald PM. Botulinum toxin for shoulder pain. Cochrane Database Syst Rev. 2010;9. 49. Gamble GE, Barberan E, Laasch HU, et al. Poststroke shoulder pain: a prospective study of the association and risk factors in 152 patients from a consecutive cohort of 205 patients presenting with stroke. Eur J Pain. 2002; 6(6):467–474. 50. Katayama Y, Yamamoto T, Kobayashi K, et al. Motor cortex stimulation for post-stroke pain: comparison of spinal cord and thalamic stimulation. Stereotact Funct Neurosurg. 2002;77(1–4): 183–186. 51. Rasche D, Ruppolt M, Stippich C, Unterberg A, Tronnier VM. Motor cortex stimulation for long-term relief of chronic neuropathic pain: a 10 year experience. Pain. 2006;121(1): 43–52. 52. Widar M, Ahlström G, Ek AC. Health-related quality of life in persons with long-term pain after

a stroke. J Clin Nurs. 2004;13(4): 497–505. 53. Chae J, Mascarenhas D, Yu DT, et al. Poststroke shoulder pain: its relationship to motor impairment, activity limitation, and quality of life. Arch Phys Med Rehabil. 2007;88(3):298–301. 54. Kong K-H, Woon V-C, Yang S-Y. Prevalence of chronic pain and its impact on health-related quality of life in stroke survivors. Arch Phys Med Rehabil. 2004;85(1): 35–40. 55. Wolf TJ, Baum C, Connor LT. Changing face of stroke: Implications for occupational therapy practice. Am J Occup Ther. 2009;63(5):621–625. 56. Visser-Meily A, Post M, Schepers V, Lindeman E. Spouses’ quality of life 1 year after stroke: prediction at the start of clinical rehabilitation. Cerebrovasc Dis. 2005;20(6):443–448. 57. Banks P, Pearson C. Improving Services for Younger Stroke Survivors and their Families. Edinburgh: Chest Heart and Stroke Scotland. 2003. 58. Alaszewski A, Alaszewski H, Potter J, Penhale B. Working after a stroke: survivors’ experiences and perceptions of barriers to and facilitators of the return to paid employment. Disabil Rehabil. 2007;29(24):1858–1869. 59. Vestling M, Tufvesson B, Iwarsson S. Indicators for return to work after stroke and the importance of work for subjective well-being and life

satisfaction. J Rehabil Med. 2003; 35(3):127–131. 60. Di Carlo A. Human and economic burden of stroke. Age Ageing. 2009;38(1):4–5. 61. Bendz M. The first year of rehabilitation after a stroke: from two perspectives. Scand J Caring Sci. 2003;17(3):215–222. 62. Proot IM, Abu-Saad HH, de Esch-Janssen WP, Crebolder HFJM, ter Meulen RHJ. Patient autonomy during rehabilitation: the experiences of stroke patients in nursing homes. Int J Nurs Stud. 2000;37(3): 267–276. 63. Lundström E, Smits A, Terént A, Borg J. Risk factors for strokerelated pain 1 year after first-ever stroke. Eur J Neurol. 2009;16(2): 188–193. 64. Roger VL, Go AS, Lloyd-Jones DM, et al. Heart Disease and Stroke Statistics – 2012 Update: A Report From the American Heart Association. Circulation. 2012;125(1):e2–e220. 65. Jönsson A-C, Lindgren I, Hallström B, Norrving B, Lindgren A. Prevalence and intensity of pain after stroke: a population based study focusing on patients’ perspectives. J Neurol Neurosurg Psychiatry. 2006;77(5): 590–595. 66. Siniscalchi A, Gallelli L, De Sarro G, Malferrari G, Santangelo E. Antiepileptic drugs for central post-stroke pain management. Pharmacol Res. 2012;65(2): 171–175.

29

Section 1 Chapter

4

Neurological Disorders

Patient with brachial plexopathy Jonathan Chang and Rahul Rastogi

Case study A 25-year-old male football player complains of new right upper extremity numbness and weakness. Symptoms are such that he is unable to catch an American football; however he is able to loosely hold a can of soda. He states the symptoms started after a motorcycle accident 1 week ago and have got worse. The pain from the accident has improved, but the numbness and weakness are unchanged and a little worse. The patient rates his pain at a 3/10 with radiation down his arm from his shoulder to his fingers. Physical exam is remarkable for a well-developed male with prominent upper-body musculature. There is no noted edema, cyanosis, or clubbing of the upper extremity. There are equal strong bilateral radial pulses, 2+ biceps reflexes on the left, 5/5 strength of the left upper extremity, 1+ biceps reflex on the right, 3/5 shoulder abduction, 3/5 biceps flexion, 3/5 wrist extension, and 4/5 strength to the intrinsic muscles of the hand. Tinel’s sign is negative, bilaterally.

The spinal nerve roots C5 to T1 merge to form 3 trunks: the upper trunk from C5 and C6, the middle trunk from C7, and the lower trunk from C8 and T1. Each trunk further divides into anterior and posterior divisions. The anterior divisions of upper and middle trunk form the lateral cord while the anterior division of the lower trunk becomes the medial cord. All three posterior divisions merge to form the posterior cord. The cords are named medial, lateral, and posterior because of their anatomic relationship to the axillary artery. The cords form 5 terminal nerves: the medial cord forms the ulnar nerve, the lateral cord forms the musculo-cutaneous nerve, the medial and lateral cord together form the median nerve, and the posterior cord forms the axillary and radial nerves.

3. Clinical classification of brachial plexopathies

The brachial plexus is a complex web of anterior rami of spinal nerves arising from cervical spine and situated in the neck and shoulder. It provides both sensory and motor nerve supply to the ipsilateral shoulder and upper extremity. Anatomically, the brachial plexus is vulnerable to injury resulting in abnormal function and/or sensation of the ipsilateral shoulder and upper extremity. This constellation of symptoms is termed “brachial plexopathy.”

Brachial plexus lesions are classified into three broad categories in relation to the clavicle: (1) supraclavicular – constitutes mainly roots and trunk; (2) retroclavicular – mainly divisions; and (3) infraclavicular – comprising cords and terminal nerves of the brachial plexus. Supraclavicular plexopathies are the most common type, while retroclavicular plexopathies remain rare. On the basis of trunk involvement, supraclavicular plexopathies are divided into: upper (C5 and 6 root and upper trunk), middle (C7 root and middle trunk), and lower (C8 and T1 root and lower trunk).

2. Describe the anatomy of the brachial plexus

4. What is the epidemiology of plexopathies?

The brachial plexus is composed of 5 nerve roots, 3 trunks, 6 divisions, 3 cords, and 5 terminal nerves.

Brachial plexus injuries occur in ~1% of trauma patients. Young males (average age in 20s) are at higher

1. What is brachial plexopathy?

Case Studies in Pain Management, ed. Alan David Kaye and Rinoo V. Shah. Published by Cambridge University Press. © Cambridge University Press 2015.

30

Chapter 4: Patient with brachial plexopathy

risk for this type of injury. Motor vehicle accidents lead to traumatic etiology, while other types of accidents, e.g., sports injuries, work injuries, and recreational activity injuries, are less common in brachial plexus injuries. Traction/traumatic injuries to the brachial plexus can also result from poor surgical positioning. Supraclavicular injuries are predominant and account for ~60% of injuries. Infraclavicular injuries account for the remaining ~40%. Supraclavicular injuries also tend to be more severe than infraclavicular injuries.

5. What are the causes and mechanisms of brachial plexopathies? Traction, compression, laceration, contusion, ischemia, and inflammation are the predominant mechanisms involved in brachial plexopathies (Table 4.1). Table 4.1. Causes and mechanism of brachial plexopathies

Mechanisms

Causes

Traction

Trauma, iatrogenic, obstetric, surgical positioning, avulsion, sports injuries

Compression

Trauma, metastatic tumor, thoracic outlet syndrome, Pancoast syndrome, use of crutches

Laceration

Trauma, open brachial plexus injury

Contusion

Trauma

Ischemia

Trauma, vascular

Inflammation

Trauma, radiation neuritis

Intraneural factors

Primary nerve neoplasm, i.e., neurofibromatosis

6. What happens to nerves in plexopathy? Injury to nerves was classified by Seddon (1943) (Table 4.2).These injuries are progressively worse: ischemia > demyelination > axonotmesis > neurotmesis.

7. Describe the clinical presentation of brachial plexopathies Brachial plexopathies present with muscle weakness and atrophy, sensory loss or paresthesias, and pain. Clinical presentation varies with the level of the lesion or the type of plexopathy. Careful history and physical examination help make the diagnosis by elucidating the mechanism and possible anatomic level of injury (Table 4.3). Supraclavicular plexopathies present in a segmental distribution. Supraclavicular upper trunk plexopathies are most common and often result from trauma and traction. These plexopathies have the best prognosis. Supraclavicular root plexopathies, i.e., nerve root avulsions, are uncommon and carry a poor prognosis. Tumor spread and compression are a common source of supraclavicular lower trunk plexopathies and may be associated with Horner’s syndrome. In contrast, infraclavicular plexopathies lack segmental distinction, but symptoms present in specific terminal nerve distributions

8. What is the differential diagnosis? a. Nerve root avulsion b. Acute brachial plexus neuritis (Parsonage–Turner syndrome)

Table 4.2. Types of nerve injury

Nerve injury

Neural structural damage

Prognosis

Affects myelin

Incomplete axonal damage

Present

No

AXONOTMESIS (axon damage with preservation of connective tissues)

Yes

Yes

NEUROTMESIS (complete transection of axon including connective tissues)

Yes

Yes

NEUROPRAXIA Ischemic Demyelination

Complete transection of axon

No Conduction block across nerve injury Days to months to recover

Good

No Distal axonal degeneration Recovery depends upon extent of nerve damage

Fair

Yes Distal axonal degeneration Regeneration if axons in aberrant path and forms neuroma

Poor

31

Chapter 4: Patient with brachial plexopathy

Table 4.3. Clinical presentation of brachial plexopathy

Type of plexopathy

Motor

Sensory

Reflexes

Supra/infraspinatus (arm external rotation), deltoid (arm abduction), biceps, and brachioradialis (elbow flexion) Partial weakness in forearm pronation, wrist flexion, and elbow extension Weakness of elbow, wrist, and finger extension, wrist flexion, forearm pronation All ulnar and median innervated muscle + C8 radial innervated muscles Weakness of hand grip, inability to full flexion of fingers, partial weakness of finger & wrist extension

Decreased sensation lateral upper arm (axillary n.), lateral forearm (lateral ante-brachial cutaneous n), and lateral hand and 1–3 digits (post. cutaneous n.) Decreased sensation to posterior forearm, hand over middle finger

Biceps Brachioradialis

Altered sensation in medial arm, medial forearm, medial hand, and 4&5 digits

None

Lateral forearm and digits 1–3 Lateral arm (axillary n.), posterior arm and forearm (posterior cutaneous n.), radial dorsum of hand (superficial radial n.) Altered sensation in medial arm, medial forearm, medial hand, and 4&5 digits

Biceps reflex Triceps Brachioradialis

Widespread sensory loss

All reflexes absent

S U P R A C L A V I C U L A R

Upper trunk (C5, C6) (most common)

I N F R A C L A V I C U L A R

Lateral cord Posterior cord

Wrist flexion, elbow flexion Weakness in finger, arm, wrist extension (wrist drop), weak shoulder ab/ adduction

Medial cord (same as lower trunk except preservation of C8 fibers)

Weakness of grip, weak hand muscles

Middle trunk (C7) (rare) Lower trunk (C8, T1)

PAN PLEXOPATHY

c. d. e. f. g. h. i. j.

Deficit

Weakness of all upper extremities, except rhomboids and serratus anterior function

Thoracic outlet syndrome Cervical radiculopathy Carpel tunnel syndrome Radial neuropathy Ulnar neuropathy Myelopathies Pancoast tumor, syringomyelia, schwannomas Complex regional pain syndrome

Our patient suffers from traumatic nerve root avulsion.

32

Triceps reflex

None

9. How do you diagnose nerve root avulsion/brachial plexopathies? As mentioned previously, a detailed history and physical exam is central to making the correct diagnosis. In particular, the history of upper-body trauma with associated motor deficits (weakness, loss of function, and atrophy of muscles), sensory deficits (paresthesias and numbness), and/or diminished reflexes in the targeted limb strongly suggest nerve root avulsion.

Chapter 4: Patient with brachial plexopathy

Several other diagnostic tools are helpful in making the diagnosis. a. Electromyography/nerve conduction studies. EMG is a complex diagnostic test in the evaluation of brachial plexopathy in the presence of several nerves, roots, trunks, and divisions. Usually this examination includes: i. sensory and motor evaluation of ulnar, median, and radial nerve ii. Needle examination of targeted muscles Sensory exam helps differentiate preganglionic lesions from postganglionic peripheral nerve lesions, because the sensory exam is normal with lesions proximal to dorsal root ganglion. Electroneuromyography (EMGNCV) also helps in differentiation of neuropraxia and axonotmesis/neurotmesis. b. Imaging of cervical spine and shoulder – MRI/CT scan/x-rays. Imaging of cervical spine helps in ruling out any traumatic avulsion, degenerative cervical spine, shoulder, or spinal pathology. It also diagnoses any compressive etiology, i.e., tumor, hematoma, accessory muscles/bands, or ribs. c. Chest and neck x-ray to check for spine, rib cage, or collarbone abnormalities d. Imaging of brachial plexus – MRI/ultrasound. Direct evaluation of brachial plexus rules out any obvious damage and compression causing plexopathy. e. Complete blood count, basic metabolic profile, erythrocyte sedimentation rate, antinuclear antibody assay, biopsy.

10. What other disease processes mimic traumatic nerve root avulsion? There are several disease entities that mimic traumatic nerve root avulsion. Some common diseases are Parsonage–Turner syndrome, cervical radiculopathy, thoracic outlet syndrome (TOS), and complex regional pain syndrome (CRPS) of upper limb (Table 4.4). A detailed history and physical exam is key to differentiating among these entities.

11. How should I treat this patient? Pain and abnormal physical exam findings are common in nerve root avulsion. Depending upon

the level of injury, different care paths should be considered. In a patient with an open injury and neural loss immediate surgical evaluation is recommended. If appropriate, surgical repair should be attempted. In contrast, a patient with a closed injury should initially undergo conservative treatment. Treatment goals include: a. Improving pain relief. Patient should be prescribed analgesics, ranging from OTC to prescription medications, i.e., acetaminophen, NSAIDs, muscle relaxants, opioids, etc. For the neuropathic component of pain, antiepileptics (i.e., gabapentin, pregabalin, etc.) and antidepressants (amitriptyline, nortriptyline, duloxetine, etc.) may be used (Table 4.5). TENS may also be helpful. A spinal cord stimulator should be tried if conservative medical treatments fail to control the pain. b. Improving function. Physical therapy should be routinely utilized to maintain and improve muscle function and strength. Sometimes assist devices, i.e., braces, splints, can help increase upper extremity function. Ergonomics, vocational rehabilitation, and occupational therapy are also helpful in improving function. c. Correcting the underlying etiologies. Nerve damage and/or nerve compression is sometimes amenable to surgical repair and/or surgical decompression. These types of surgery in addition to tendon transfer surgery can be very helpful in regaining a patient’s function. d. Intractable refractory pain (after the failure of the above therapies). Radiofrequency/surgical destruction of brachial plexus roots, i.e., DREZ lesions, amputation of brachial plexus/upper extremity may be an option.

12. Are there any complications to worry about following treatment? Persistent intractable pain, profound numbness, muscle atrophies, development of contractures, and joint deformities are extremely disabling outcomes of brachial plexopathies. Diminishing or loss of mobility results in osteopenia, skin breakdown/infection, depression, and anxiety. These complications often worsen a patient’s pain and slow recovery.

33

34

Table 4.4. Differential diagnosis of brachial plexopathy

Nerve root avulsion

Idiopathic brachial neuritis (Parsonage–Turner syndrome, neuralgic amyotrophy)

Thoracic outlet syndrome (Cervical rib syndrome, scalenus anticus syndrome)

Cervical radiculopathy

Complex regional pain syndrome (CRPS) (Causalgia, reflex sympathetic dystrophy)

Onset

Acute

Acute

Sudden to gradual

Gradual to sudden

Varies

Location

Unilateral shoulder and UE

Unilateral shoulder >> bilateral and UE

Unilateral > bilateral neck, shoulder and UE

Most common roots involved C7 > C6

Unilateral, regional distribution

Etiology

Trauma

Surgery, infection, vaccination

Repetitive UE lifting

Disc herniation or DJD, tumors, infection

Trauma, tumor, stroke, idiopathic

Pain

Immediate sharp, severe pain and numbness in dermatome distribution

Sharp, severe pain usually resolves in a few weeks

Varied pain presentation along with paresthesias and numbness in UE

Varied pain with dermatome distribution

Varied pain with hyperalgesia and allodynia

Weakness

Immediate weakness in myotome distribution, atrophy of denervated muscles over time

As pain resolving, weakness ensues in proximal UE muscles

Varies

+/–

+/–

Motor: sensory deficit

Motor ¼ sensory

Motor >> sensory

Sensory >> motor Usually C8, T1 involvement

+/– Sensory > motor

Sensory > sudo/ vasomotor > motor Motor in late stages

Type of nerve involvement

Variable–complete

Incomplete

Variable–incomplete

Variable

Variable

Epidemiology

♂:♀ 2:1 Age: 20–60 years

♂:♀: 2:1 Age: 30–70 years

nTOS (85–90%)- ♂:♀ 1:3 vTOS (10–12%) – ♂:♀ 1:1 aTOS (2–4%) – ♂:♀ 1:1

Incidence 85:100 000 population

Incidence 1–5% ♂:♀ 1:3 Age: 10–70 years

Pathology

Avulsion

Immunemediated – inflammatory?

Compression of neurovascular bundle in thoracic outlet (brachial plexus (nTOS), subclavian vein (vTOS), subclavian artery (aTOS))

Inflamed nerve root/ compression

Includes: peripheral inflammation, peripheral sensitization, sympathetic– afferent coupling, immune dysfunction, central sensitization & cortical reorganization

Clinical presentation

Variable, in severe cases – numbness and immediate weakness followed by severe pain. Atrophy of affected muscles over time

Intense pain followed by proximal muscle weakness and atrophy of shoulder and UE

nTOS ¼ pain, paresthesias, "" on UE elevation nTOS ¼ swelling & cyanosis of UE, heaviness and pain aTOS ¼ claudicating cramping UE pain

Pain in affected nerve root dermatome with +/– sensory or/and motor deficit, precipitated by cervical spine movements

Regional presentation of pain, allodynia, weakness, atrophy, intermittent color changes, swelling, temperature differences

Risk factors

Age, gender, athlete, trauma

Preceding stress, illness

Age Repetitive heavy use of upper extremity

Heavy manual labor, smoking, operating vibrating equipment

Trauma

Diagnostics

EMG, MRI

EMG, spine imaging

MRI Scalene diagnostic blocks EMG-NCV

Spine imaging shows Cspine abnormalities, SNR diagnostic blocks

No specific diagnostic test

Management

Variable

Symptomatic Conservative – PT +/– OT, analgesics, oral prednisone Late surgical repair

Conservative – PT, medications Surgical repair

Prevention – PT, Conservative – analgesics, PT, cervical epidural steroid Surgical interventions

Symptomatic – analgesics Functional – PT, OT, GMI, psychotherapy Interventional – sympathetic blocks, SCS, sympathectomy, amputation

Outcome

Variable

Usually good prognosis Resolve in 6–18 months

Fair prognosis

Usually good

Varied, usually children have better prognosis

aTOS, arterial thoracic outlet syndrome; C-spine, cervical spine; DJD, degenerative joint disease of spine; EMG-NCV, electroneuromyography; GMI, graded motor imagery; MRI, magnetic resonance imaging; nTOS, neurogenic thoracic outlet syndrome; OT, occupational therapy; PT, physical therapy; SCS, spinal cord stimulation; SNR, selective nerve root; sTOS, symptomatic disputed thoracic outlet syndrome; UE, upper extremities; vTOS, venous thoracic outlet syndrome.

35

Chapter 4: Patient with brachial plexopathy

Table 4.5. Common medications utilized for chronic pain management

Drugs/class

Mechanism

Concern

Dosages

ANALGESICS Unknown

Liver damage

325–4000 mg/d

Non-steroidal antiinflammatory drugs

Ibuprofen Naproxen Meloxicam Celecoxib

Decrease prostaglandins by inhibiting cyclo-oxygenase

GI irritation, renal effects, bleeding

200–2400 mg/d 250–1500 mg/d 7.5–15 mg/d 100–400 mg/d

Opioids

Hydrocodone Fentanyl Morphine Oxycodone Methadone Tramadol Tapentadol

Agonist to opioid receptors Tramadol – additional SSRI and NRI effect Tapentadol – additional NRI action

Nausea/ vomiting, constipation, drowsiness, respiratory depression Methadone-variable t1/2 Tramadol/tapentadol – caution with antidepressants

Variable Variable Variable Variable Variable 50–400 mg/d 50–600 mg/d

Tricyclics (amitriptyline, nortriptyline, desipramine) SNRI (duloxetine)

Modulation of neurotransmission of serotonin and norepinephrine

Sedation, tachycardia, urinary hesitancy, weight gain

Gabapentin Pregabalin

α2δ subunit of voltage-gated N-type Ca2+ channel modulation Na+ channel blockade Na+ channel blockade

Drowsiness, confusion, weight gain, rash LFT monitoring for carbamazepine

GABA-b agonist

Confusion, drowsiness, dizziness

Acetaminophen

ADJUVANT ANALGESICS Antidepressants

Antiepileptics

Lamotrigine Carbamazepine Muscle relaxants

Baclofen Cyclobenzaprine Methocarbamol

Unknown action on CNS

25–150 mg HS

20–90 mg/d 300–3600 mg/d 75–600 mg/d 100–600 mg/d 200–1200 mg/d 10–80 mg/d 10–60 mg/d 500–2000 mg/d

Alpha-2 adrenergics

Clonidine Tizanidine

α2 adrenergic agonism

Drowsiness, #BP, #HR LFT caution – tizanidine

01–0.3 TD patch 2–32 mg/d

Local anesthetic

Mexelitine

Na+ channel blockade

Liver toxicity, #BP

150–900 mg/d

Corticosteroids

Methyl prednisone Dexamethasone

Hyperglycemia, weight gain, edema, agitation

Variable 4–96 mg/d

BP, blood pressure; Ca2+, calcium; CNS, central nervous system; GABA, gamma aminobutyric acid; GI, gastrointestinal; HS, bedtime; HR, heart rate; LFT, liver function test; mg/d, milligram per day; Na+, sodium; NRI, norepinephrine reuptake inhibition; SNRI, serotonin– norepinephrine reuptake inhibitor; SSRI, selective serotonin reuptake inhibition; t1/2, half-life; TD, transdermal.

36

Chapter 4: Patient with brachial plexopathy

13. What is the prognosis of brachial plexopathies? Prognosis is highly variable with spontaneous and surgical recovery within days to months. Recovery may be complete or incomplete dependent on each patient’s particular type and degree of injury.

14. What are the social considerations in brachial plexopathy patients? Patients will need comprehensive support in all aspects of life. Patients will also need to adjust to new limitations in performance of their activities of

References 1.

http://www.aafp.org/afp/2000/ 1101/p2067.html (acute brachial plexus neuritis)

2.

Thompson RW. Challenges in the treatment of thoracic outlet syndrome. Tex Heart Inst J. 2012;39(6):842–843.

3.

4.

https://www.clinicalkey.com/ topics/anesthesiology/thoracicoutlet-syndrome.html Smania N, Berto G, La Marchina E, et al. Rehabilitation of brachial

5. 6.

daily living. Many of these patients are very physically active and are accustomed to independence. They may require psychologic counseling as they adjust to coping with these changes in their physical ability and occupational limitations.

Conclusions Brachial plexopathy is a very distressing syndrome for patients. These patients are often highfunctioning individuals who have a dramatic and rapid change in their functional abilities. There are very limited treatment options with highly variable efficacy.

plexus injuries in adults and children. Eur J Phys Rehabil Med 2012;48;483–506. http://www. minervamedica.it/en/getfreepdf/ qCwJ4qBn6KYLZ0kPMOr ZIH41iynTZ4WNlHUnn WtgkmaWYBozHilflyIK% 252FDlzLlS%252FmB5lTjX% 252BXSUdV43JbSRZAA%253D% 253D/R33Y2012N03A0483.pdf http://emedicine.medscape.com/ article/316888-followup#a2651

population. Neurosurgery. 1997; 40(6):182–188. 7.

Dubuisson AS, Kline DG. Brachial plexus injury: a survey of 100 consecutive cases from a single service. Neurosurgery. 2002; 51(3):673–683.

8.

Thompson RW. Challenges in the treatment of thoracic outlet syndrome. Tex Heart Inst J. 2012; 39(6):842–843.

Midha R. Epidemiology of brachial plexus injuries in a multitrauma

37

Section 1 Chapter

5

Neurological Disorders

Phantom limb pain Jonathan Chang and Rahul Rastogi

Case study

2. What is phantom limb pain?

A 36-year-old male fractured his distal tibia following a motor vehicle accident 5 years prior to presentation and developed foot pain. Surgical fixation did not alleviate the pain which worsened and spread to his leg (below the knee). Treatments including analgesics, physical therapy, sympathetic blocks, and spinal cord stimulation were unsuccessful. Due to persistent pain, an above the knee amputation was performed 4 years following the initial injury. The pain abated for 2 weeks, but painful symptoms developed in the missing left leg and gradually worsened. The pain symptoms were continuous and described as “aching, tightening, and burning,” mainly localized to the distal third of the lower leg and foot. He also reports intermittent twitching and spasms at the stump. He currently takes methadone 20 mg twice a day, cymbalta 60 mg at bedtime, and baclofen 20 mg three times a day with partial benefit.

A range of unpleasant sensations from tingling to pain in the absent postamputation limb; it is a neuropathic type of pain.

1. What is postamputation pain? A variety of unpleasant sensations are experienced after limb amputation, also known as “postamputation pain” (PAP). This was first formally described as a medical problem by Paré in 1551. In 1871 Weir Mitchell described it in Civil War soldiers and termed it “phantom limb pain” (PLP). There are three different sensory experiences described after amputation: (1) non-noxious phantom sensation, (2) residual limb pain (stump pain) (RLP), and (3) phantom pain. Phantom pain commonly involves the limbs, but it can present as “phantom breast,” “phantom tooth,” “phantom testes,” or “phantom (body part)” surgical amputation pain.

3. Demographics/epidemiology of phantom pain? The incidence of PAP is as high as 90%. The phantom sensation occurs in almost all patients undergoing amputation, but the precise incidence of PLP or RLP is difficult to assess due to the overlap of different types of PAP. The incidence of PLP ranges from 45 to 78%. However, 75% of PLP patients report experiencing pain on the first day of amputation and the remaining 25% develop pain within 1–2 weeks. The prevalence of PLP decreases with time. PLP affects upper limbs (≈80%) more often than lower limbs (≈54%). Age, laterality, level of amputation, and gender do not affect prevalence.

4. What are the indications for amputation? Trauma, vascular abnormalities, ischemia, cancer, and intractable pain are common indications for limb amputation. PLP not only follows physical removal of a limb, but can affect a congenitally absent body part or neurologically deficient limb, i.e., palsy, stroke.

5. What are the risk factors for phantom limb pain? Several studies suggest risk of developing PLP after amputation is increased in:

Case Studies in Pain Management, ed. Alan David Kaye and Rinoo V. Shah. Published by Cambridge University Press. © Cambridge University Press 2015.

38

Chapter 5: Phantom limb pain

a. b. c. d. e.

Females Upper limb amputation Presence of pre-amputation pain Persistent residual limb pain Closer to the time since amputation

Table 5.1. Proposed mechanisms of phantom limb pain

Level

Mechanism

Peripheral

Peripheral sensitization: Up-regulation of voltage-gated sodium channel at peripheral neuroma and dorsal root ganglion Sympathetic–afferent coupling Both result in ectopic discharges, and hyperexcitability

Spinal

Lamina reorganization: Peripheral afferent non-noxious neurons (Lamina 3 & 4) forming new crosslinks with other afferent noxious neurons of different lamina (1 & 2) Central sensitization: Barrage of peripheral noxious input results in hyperexcitability and expansion of neuronal receptive field causes “wind-up” (up-regulation) of NMDA receptors

Supraspinal

Cortical reorganization: Amputated limb area taken over by adjacent body area on somatosensory and motor cortex Cortical motor-sensory dissociation Psychogenic

Stress, anxiety, depression, and other emotional factors contribute to triggering, exacerbation, or persistence of PLP.

6. What are the clinical symptoms and signs of phantom limb pain? PLP is a neuropathic pain that presents with a range of descriptors, e.g., tingling, burning, aching, shooting, gripping, knife-like, electrical shock, pricking, and numbing. It presents with a variable intensity on the affected side. Usually pain starts in the distal portion of missing limb and presents intermittently. PLP may be continuously present. Pain develops within the first few weeks. Rarely, it is precipitated in the amputated limb following a spinal or epidural anesthetic block later in life. Usually with time PLP dissipates, but persistence beyond 6 months indicates a poor prognosis. Some PLP patients experience shortening of missing limb (especially shortening of middle portion of missing limb, without affecting size of distal portion), thus the distal portion of limb, i.e., hands or feet feel very close to stump. This phenomenon is called “telescoping.” Telescoping is associated with relatively poor prognosis. RLP may coexist with PLP. RLP affects 5–10% of amputated limbs. It can present with stabbing, throbbing, and/ or aching pain associated with hyperalgesia and/or allodynia of affected stump. RLP can be spontaneous or as a result of a poorly fitted prosthesis. There may be focal neuromata accounting for RLP.

7. Describe the pathophysiology of phantom limb pain The exact mechanism of PLP pain is essentially unclear, but various theories are proposed. Broadly the postulated mechanisms are divided into peripheral, spinal, and supraspinal mechanisms (Table 5.1).

8. How is PLP diagnosed? PLP is diagnosed on the basis of history and examination. No specific diagnostic test is recommended.

9. Howisphantomlimbpainmanaged? PLP is sometimes debilitating and refractory to management. There is a distinct lack of specific treatment modality. A multidisciplinary, multimodal approach should be utilized. Treatment categories include: A. Pharmacological: i. Prevention of PLP: Pre- and perioperative pain control has some advantages in management of PLP and has shown decrease in PLP incidence. This can be effectively achieved by perioperative epidural/perineural analgesia or patient-controlled analgesia. ii. Management of PLP: Pain can be managed with analgesics, i.e., acetaminophen, nonsteroid anti-inflammatory (NSAID) agents. Opioids (tramadol, oxycodone, morphine, methadone, etc.) can be used in intractable pain. Use of adjuvants, i.e., antidepressants (nortriptyline, amitriptyline, mirtazapine, duloxetine, etc.), anticonvulsants (gabapentin, carbamazepine, pregabalin, etc.), NMDA antagonist (i.e., ketamine), and calcitonin have shown some benefit in management of PLP (Table 5.2).

39

Chapter 5: Phantom limb pain

Table 5.2. Common medications utilized for chronic pain management

Drugs/class

Mechanism

Concern

Dosages

ANALGESICS Unknown

Liver damage

325–4000 mg /d

Non-steroidal antiinflammatory drugs

Acetaminophen Ibuprofen Naproxen Meloxicam Celecoxib

Decrease prostaglandins by inhibiting cyclooxygenase

GI irritation Renal effects Bleeding

200–2400 mg/d 250–1500 mg/d 7.5–15 mg/d 100–400 mg/d

Opioids

Hydrocodone Fentanyl Morphine Oxycodone Methadone Tramadol Tapentadol

Agonist to opioid receptors Tramadol – additional SSRI and NRI effect Tapentadol – additional NRI action

Nausea/vomiting, constipation, drowsiness, respiratory depression Methadone – variable long t1/2 Tramadol/tapentadol – caution with antidepressants

Variable Variable Variable Variable Variable 50–400 mg/d 50–600 mg/d

Tricyclics (amitriptyline, nortriptyline, desipramine) SNRI (Duloxetine)

Modulation of neurotransmission of serotonin and norepinephrine

Sedation, tachycardia, urinary hesitancy, weight gain

25–150 mg HS

Gabapentin Pregabalin

α2δ subunit of voltagegated N-type Ca2+ channel modulation Na+ channel blocker Na+ channel blockade

Drowsiness, confusion, weight gain, rash LFT monitoring for carbamazepine

Selective GABA-b agonist Unknown action on CNS

Confusion, drowsiness, dizziness

ADJUVANT ANALGESICS Antidepressants

Antiepileptics

Lamotrigine Carbamazepine Muscle relaxants

Baclofen Cyclobenzaprine Methocarbamol

20–90 mg/d 300–3600 mg/d 75–600 mg/d 100–600 mg/d 200–1200 mg/d 10–80 mg/d 2–32 mg/d 10–60 mg/d 500–2000 mg/d

Alpha-2 adrenergics

Clonidine Tizanidine

α2 adrenergic agonism

Drowsiness, #BP, #HR LFT caution – tizanidine

01–0.3 TD patch 2–32 mg/d

NMDA blockers

Ketamine

NMDA blockade

Delerium, "BP, "HR, cognition, hepatotoxicity, vesicopathy

10–240 mg/d PO 50–600 mg/d SC

Local anesthetic

Mexelitine

Na+ channel blocker

Liver toxicity, #BP

150–900 mg/d

Corticosteroids

Methyl prednisone Dexamethasone

Hyperglycemia, weight gain, edema, agitation

Variable 4–96 mg/d

2+

BP, blood pressure; Ca , calcium; CNS, central nervous system; GABA, gamma aminobutyric acid; GI, gastrointestinal; HS, bedtime; HR, heart rate; LFT, liver function test; mg/d, milligram per day; Na+, sodium; NMDA, N-methyl D-aspartate; NRI, norepinephrine reuptake inhibition; SSRI, selective serotonin reuptake inhibition; t1/2, half-life; TD, transdermal.

A. Non-pharmacological: Various non-pharmacologic modalities have shown mild to moderate benefit in PLP patients. These include:

40

i. Prosthesis: A properly fitted prosthesis has been shown to decrease the intensity of PLP.

Chapter 5: Phantom limb pain

ii. TENS: Use of TENs has shown some efficacy in PLP and RLP pain control. iii. Mirror therapy: Mirror therapy alone or as part of graded motor imagery (GMI) has shown significant benefit in management of PLP. The patient places the normal and amputated limbs into a box with a vertical mirror. The mirror reflects the normal limb and its motion. The patient cannot see the amputated limb since they are looking at the mirror reflection of the normal limb. Due to visuo-proprioceptive dissociation, via activation of mirror neurons in the brain, the PLP symptoms are reduced. iv. Biofeedback/behavioral therapies v. Acupuncture/external heat/cold and massages: Anecdotal evidence of some help. B. Injection therapies: Studies have shown variable benefit from perineural or regional nerve block in RLP, but benefit from injection therapies in PLP is limited. Various injectates including local anesthetics, botulinum toxin, corticosteroids, etc. are used in management of RLP with short-lasting benefit. C. Surgical therapies: Surgical modalities for pain management have variable outcomes; they are reserved for intractable and refractory PLP. Some efficacious modalities include: i. Neuromodulation: a. Peripheral nerve stimulation: It is useful in pain restricted to 1 or 2 peripheral nerve distributions, thus useful in RLP. b. Spinal cord stimulation: Modulation of painful signals at spinal level by use of spinal cord stimulator can provide significant relief and has proven successful in several neuropathic pain conditions. Extrapolation to PLP has proven to be less efficacious. c. Deep brain/motor cortex stimulation: Results for deep brain stimulation for PLP in various studies are suggesting a positive trend. Efficacy is unclear. ii. Dorsal root entry zone lesioning: Surgical lesion at dorsal root entry zone has been successful in management of PAP from brachial plexus avulsions, but studies are limited for lower extremity PAP.

iii. Stump revision: Selective RLP patients with specific neuroma benefit from stump revision or neuroma resection surgery. Outcomes are variable.

10. How to prevent phantom limb pain Not many things have been proven to decrease the risk of PLP. Studies have suggested that effective perioperative pain control decreases the risk of PLP after amputation. Studies recommend achieving effective pain control perioperatively by utilizing local, epidural, and perineural blocks and infusions with local anesthetics or intravenous patient-controlled analgesia 48 hours prior to amputation. These treatments should be continued for 48 hours after surgery.

11. How successful are these treatment modalities? Management of PLP is a challenging proposition, thus various treatment modalities are used for achieving pain control. Studies show limited benefit from pharmacologic agents. Ketamine and opioids (methadone, oxycodone, etc.) trials were statistically beneficial, while results of gabapentin, calcitonin, amitriptyline, and mexiletine are limited and conflicting. Graded motor imagery and mirror box consistently show significant relief in PLP patients. Studies show positive benefit especially for RLP and PLP with the use of prosthesis. In small series, neuromodulation, i.e., spinal cord stimulation and peripheral nerve stimulation, provides significant analgesia. Surgical modality studies are not shown to have any statistically significant benefit, and thus use of this modality is limited to refractory PLP patients.

12. How does this impact a person’s life? Refractory phantom pain is extremely debilitating to a patient’s physical, social, and emotional well being. The pain and loss of limb requires them to limit and/or adjust their activities. This feeling of handicap and dependency on others or aids is sometimes emotionally challenging resulting in anxiety and depression. If unchecked it can lead to suicidal thoughts. The patient

41

Chapter 5: Phantom limb pain

should be encouraged to seek psychotherapy and vocational rehabilitation early.

Conclusions PLP is a challenging life-changing health problem and its exact mechanisms are still unknown and treatments have variable and often poor outcomes. It also affects

References 1.

2.

42

Hsu E, Cohen SP. Postamputation pain: epidemiology, mechanisms, and treatment. J Pain Res. 2013;6:121–136. Knotkova H, Cruciani RA, Tronnier VM, Rasche D. Current and future options for the management of

3.

4.

patients’ physical, emotional, and socioeconomic status negatively. Current management strategies directed toward postulated mechanisms have at best provided limited efficacy. A multimodal approach should be utilized to address multifaceted effects of PLP. Further research on understanding of PLP mechanisms and development of treatments is needed to effectively manage this challenging health problem.

phantom-limb pain. J Pain Res. 2012;5:39–49. Lamont K, Chin M, Kogan M. Mirror Box Therapy – Seeing is believing. Explore. 2011;7:369–372. Weeks SR, Anderson-Barnes VC, Tsao JW. Phantom limb pain: theories and therapies. The Neurologist. 2010;16:277–286.

5.

Subedi B, Grossberg GT. Phantom limb pain: Mechanisms and treatment approaches. Pain Res Treat 2011;2011:864605.

6.

Vishwanathan A, Phan PC, Burton W. Use of spinal cord stimulation in the treatment of phantom limb pain: Case series and review of the literature. Pain Practice. 2010;10:479–484.

Section 1 Chapter

6

Neurological Disorders

Patient with post-thoracotomy pain Rinoo V. Shah

Case study A 44-year-old roofer fell and sustained a T7 burst fracture. He underwent emergent surgical repair requiring an anterolateral approach via a thoracotomy and underwent an anterior surgical stabilization. Two months postoperatively, he is referred for persistent surgical site pain.

1. What is the differential diagnosis? a. b. c. d.

Intercostal neuralgia Thoracic radiculopathy Pleuritis Scar neuromata

2. What is a neuroma? Neuromas are considered “tumors” of neural structures. In this instance, they are non-neoplastic. Neuromas typically form following surgical transection, trauma, or entrapment. Neuromas are considered to be discrete enlargements. If superficial, they may be palpable. If deeper, they may be visualized with noninvasive imaging tools (MRI, ultrasound).

3. Describe the clinical exam and how would one evaluate the differential diagnoses? Neuroma can be stimulated with normal palpation (allodynia). Painful stimuli over a neuroma may lead to an excessive or prolonged pain response, i.e., hyperalgesia and hyperpathia. Due to dysfunction of this neural tissue, there may be impairment in conduction. Motor function and sensory processing may

be dysfunctional. Autonomous and maladaptive reflexes may be present. In this patient’s case, the scar may be well healed. Although spontaneous and evoked pain may spill outside of the zone of the healed scar, neuroma are confined to the location of traumatic injury. Physical examination findings include a palpable and tender swelling. This is painful with light touch (allodynia). Deeper pressure leads to a more protracted (hyperpathic) and heightened (hyperalgesic) pain response. The scar should be healed. Poorly healing scars or ulcers should be addressed, before considering neuroma injections. Some healed surgical scars may demonstrate dystrophic or color changes. There may be a significant amount of allodynia, distributed around the scar. In this situation, there may be a heightened sympathetic response in addition to the presence of neuromas. Passive stretching of the scar or focal neuroma compression should elicit pain. This pain should be eliminated following a neuroma injection. Arguably, a pressure algometer may be useful: “an increase in the pressure pain threshold by 2–3 kg, immediately after the NI will indicate an effective injection.” Trigger points are commonly present in patients who have undergone surgery. This is especially true when the surgical scar injured a peripheral nerve, e.g., limb amputation or rib resection or retraction. Neuromas may be found in the surgical bed, in areas exposed to repetitive trauma, or in areas exposed to overuse. Neuromas may be confused with tender points, as is usually found in patients with fibromyalgia. Unlike fibromyalgia, neuromas are typically isolated and develop secondary to a specific event. Intercostal neuralgia is a peripheral nerve injury and is fairly discrete in its location when associated

Case Studies in Pain Management, ed. Alan David Kaye and Rinoo V. Shah. Published by Cambridge University Press. © Cambridge University Press 2015.

43

Chapter 6: Patient with post-thoracotomy pain

with a stretch injury. Transection or traumatic injury (surgical blade as opposed to surgical retraction for exposure) is a more significant injury. Central and peripheral sensitization is more significant with this type of injury as compared to a stretch. Intercostal neuralgia may overlap with neuroma. The terminal branches that were injured may regrow eccentrically, given the damage to the axonal conduit. These nerves sprout and become enmeshed in scar tissue. Given the peripheral and central sensitization, clinical symptoms may be evoked with breathing, light touch, and palpation. There may be a widened receptor field, so the patient may feel pain outside the surgical site, at one-two dermatomes above and below. There may be altered sensibility, e.g., reduction in light touch but a heightened pain response with a low sensory threshold. Thoracic radiculopathy is plausible. This may occur from the initial burst fracture injury or during internal fixation with hardware and bone graft material. A delayed presentation of pain could be attributed to heterotopic bone growth that encroaches the nerve. Pleuritis may be local and somatic pain due to violation of the parietal pleura; visceral or deeper nagging and suffocating (feeling “stuck” in the chest) may be due to violation of the visceral pleura. Rib fractures may be a consideration, but at 8 weeks they should heal – healing bone pain should improve at this juncture. A cardiopulmonary and abdominal exam is important to distinguish other more urgent findings, specifically viscerally mediated pain. These could include intra-abdominal pathology, cardiac disease, and pulmonary dysfunction. These latter phenomena may be associated with other associated symptoms: dyspnea, palpitations, nausea, vomiting, and referred limb pain. Adjunct diagnostic tests may be necessary: electrocardiogram, upper gastrointestinal endoscopy, chest x-ray, pulmonary function tests, and exercise stress tests.

4. How do you diagnose the conditions listed in the differential diagnosis? Imaging studies would be helpful to evaluate other visceral pathology that is urgent or life threatening as outlined above. However, for the conditions listed in the differential diagnosis, imaging studies except diagnostic ultrasound are generally not as helpful as the physical exam and clinical history.

44

Diagnostic injections may be helpful. Thoracic radiculopathy could be treated with a nerve block or an interlaminar epidural injection with a catheter. The risks of a thoracic spinal nerve block in a patient with prior surgery is discussed in Chapters 59 and 60. A targeted thoracic interlaminar epidural injection with a catheter and administration of local anesthetic may be useful. Intercostal nerve blocks with local anesthetic may be helpful to diagnose intercostal neuralgia. Viscerosomatic pain from pleuritis may be diagnosed with an intrapleural injection which would block visceral pain (thoracic sympathetic chain) and somatic pain (thoracic spinal nerves).

5. How should one treat this condition? Conservative treatment methods are discussed in Chapters 5, 8, and 10. Briefly, they involve multimodal treatment. Physical therapy with alternating passive temperature modalities using hot and cold packs can be used. Iontophoresis of steroids or local anesthetics or transcutaneous electrical stimulation may be used. Acupuncture and massage are useful holistic methods. Cognitive behavioral methods to minimize fear avoidance and anxiety associated with movement, spontaneous pain, and breathing should be advised. Analgesics from different drug classes, such as antineuropathic, antiseizure, antidepressant, antiinflammatory, and anti-spasmodic, should be considered. Opioids are usually a last resort, but severe neuropathic pain may not be controlled with nonopioid adjuvant medications. The “diagnostic” methods described above may serve a dual treatment role. An intercostal nerve block may be performed blindly, with ultrasound, or fluoroscopic guidance. In the case of a thoracic spine surgery, a catheter directed thoracic epidural steroid injection may be the best option for thoracic radiculitis. In this author’s experience, a thoracic transforaminal epidural steroid injection should be attempted in this patient if all other options are not feasible – given the legitimate concern about paraplegia. An intrapleural injection or thoracic sympathetic block may be an option to evaluate thoracic visceral pain. A consultation with an interventional pain specialist is advised for these procedures. In a practical sense, however, a neuroma injection should be the first consideration. Neuroma injections are commonly used as a treatment option in patients

Chapter 6: Patient with post-thoracotomy pain

with acute and chronic pain. The primary goal is to inactivate the neuromas by anesthetizing the primary area of pain through needling and infiltration with an injectable solution. Perineural infiltration of neuroma by direct feel and palpation or with ultrasound guidance is useful to diagnose neuroma pain. One should avoid direct intraneural injections since permanent nerve damage could result and paradoxically lead to a deafferentation pain syndrome. These should be conducted in a sterile fashion with the use of betadine, chlorhexidine, or an ethyl chloride spray. A local anesthetic solution (1% lidocaine and/or 0.25% or 0.5% marcaine) mixed with or without a steroid (40 mg depo-medrol, 3–6 mg betamethasone, or 2–4 mg dexamethasone) may be used. Small gauge needles with variable lengths should be used: 1. 25 G 1½ inch needle for superficial neuromas; and 2. 22 or 25 G 3½ inch needle for deeper neuromas. The patient is positioned in the prone or side lying position unless the neuroma is located on the anterior aspect of the body (chest wall, abdomen, inguinal region, limbs). One must make sure that the patient is comfortable and breathing appropriately. Noninvasive monitors may be advised for higher risk patient due to the risk of vasovagal reaction. Area to be injected is cleaned with an antiseptic solution of choice or ethyl chloride used until there is a slight frost point over the skin. The needle should be advanced past skin, subcutaneous tissue, and normal

References 1:

2:

Atluri S, Glaser SE, Shah RV, Sudarshan G. Needle position analysis in cases of paralysis from transforaminal epidurals: consider alternative approaches to traditional technique. Pain Physician. 2013;16(4):321–334. Shah RV. Paraplegia following thoracic and lumbar

muscle until it contacts the neuroma or fibrotic tissue. Pain should be elicited. The needle trajectory should be at a 45 degree angle to the skin. This is especially important near the chest wall. After negative aspiration of blood, fluid, or air a total of 1–5 ml of solution per neuroma location should be injected.

6. Are there potential complications from injections? Infection, bleeding, reaction to the medications used (keep total lidocaine used to no more than 20 ml), vasovagal reaction, injection site pain (temporary flare up), or more serious complications such as pneumothorax are possible complications. If using steroid be aware of potential skin depigmentation changes and possible skin atrophy particularly in thin patients and with superficial muscles.

7. What are the outcomes? Unfortunately, many of these patients will continue to have pain. They may require maintenance analgesic therapy for the rest of their lives. Physical therapy, psychologic counseling, and injections may have to be used periodically to help with pain. In this author’s estimation, patients that comply with multimodal treatment are likely to have better functional outcomes as opposed to those just seeking a passive route: analgesics and staying at home.

transforaminal epidural steroid injections: how relevant are particulate steroids? Pain Pract. 2013 Oct 24. doi: 10.1111/ papr.12110 [Epub ahead of print]. 3:

Shah RV. Paraplegia following thoracic and lumbar transforaminal epidural steroid injections: how relevant is physician negligence?

J Neurointerv Surg. 2013 Aug 28. doi: 10.1136/neurintsurg-2013010903 [Epub ahead of print]. 4:

Shah RV. The problem with diagnostic selective nerve root blocks. Spine (Phila Pa 1976). 2012;37(24):1991–1993.

5:

Shah RV. Spine pain classification: the problem. Spine (Phila Pa 1976). 2012;37(22):1853–1855.

45

Section 1 Chapter

7

Neurological Disorders

Complex regional pain syndrome Gaurav Jain and Nashaat N. Rizk

Case study A 50-year-old woman sustained an injury to her right wrist after a computer fell on it. A few weeks later she had pain and swelling in her right wrist. All wrist movements were painful. Due to the possibility of tendon injury, a plastic surgeon operated on her wrist and found no abnormalities. After surgery, her hand was swollen and pain worsened. It was mostly aching and burning, but sometimes sharp in nature. Gradually, she was unable to use her right hand due to pain, swelling, discomfort, tightness, and weakness. Gradually, she started to notice that the right hand felt colder and looked paler than the other hand. She had poor nail growth and the skin on her affected hand became dry. She was crippled in her personal and occupational life due to the above condition.

1. What are the differential diagnoses in this case?  Cellulitis  Lymphedema  Soft tissue or bone injury, including occult or stress fracture  Compartment syndrome  Arthritis or arthrosis  Tenosynovitis  Upper or lower limb venous thrombosis  Arterial insufficiency such as thromboangiitis obliterans or severe atherosclerosis  Scleroderma  Plexitis, peripheral neuropathy  Erythromelalgia

2. What is complex regional pain syndrome? Complex regional pain syndrome (CRPS) is a chronic regional (not in a specific nerve territory or dermatome) pain syndrome that occurs most often in an extremity in association with abnormal autonomic nervous system activity and trophic changes. The pain is seemingly disproportionate in time or degree to the usual course of any known trauma or other lesion. The disorder has both nociceptive and neuropathic features and is characterized by disabling persistent pain, hyperalgesia or allodynia, swelling, vasomotor instability, sudomotor abnormality, and impairment of motor function. In many cases, the syndrome is preceded by an inciting noxious event, surgery, trauma, or immobilization, while in some cases (9%) there is no precipitating trauma at all. However, the condition is not related to trauma severity. The syndrome shows variable progression over time. Transient features of CRPS are much more common than most clinicians realize, occurring in up to 25% of minor limb injuries. Approximately 15% of sufferers will have unrelenting pain and physical impairment up to 5 years after CRPS onset, although more patients will have a lesser degree of ongoing pain and dysfunction impacting their ability to work and function normally. The incidence per person-years at risk of CRPS based on the results of two epidemiologic studies ranged from 5.46 to 26.6/100000 personyears at risk. It is commoner in females than males, at a ratio of 2–3:1, and frequently occurs in the 5th– 7th decade of life.

Case Studies in Pain Management, ed. Alan David Kaye and Rinoo V. Shah. Published by Cambridge University Press. © Cambridge University Press 2015.

46

Chapter 7: Complex regional pain syndrome

Table 7.1. The 2007 Budapest Consensus Dignostic Criteria for CRPS*

Category

Symptom

Sign (evidence needed on exam)

Sensory

Hyperesthesia and/or allodynia

Hyperalgesia (to pinprick) and/or allodynia (to light touch and/or temperature sensation and/or deep somatic pressure and/or joint movement)

Vasomotor

Temperature asymmetry and/or skin color changes and/or skin color asymmetry

Temperature asymmetry (> 1°C) and/or skin color changes and/or asymmetry

Sudomotor/ edema

Edema and/or sweating changes and/or sweating asymmetry

Edema and/or sweating changes and/or sweating asymmetry

Motor/ trophic

Decreased range of motion and/or motor dysfunction (weakness, tremor, dystonia) and/or trophic changes (hair, nail, skin)

Decreased range of motion and/or motor dysfunction (weakness, tremor, dystonia) and/or trophic changes (hair, nail, skin)

* One symptom in at least three categories and one sign in at least two categories are required for diagnosis.

3. What are the classification and diagnostic criteria of CRPS? CRPS is classified into two types based on the absence (type I) or presence (type II) of a definable nerve injury. In 1998, the International Association for the Study of Pain (IASP) established the following four criteria that must be present for a clinical diagnosis of CRPS to be made: 1. Preceding noxious event without (CRPS I) or with obvious nerve lesion (CRPS II). 2. Spontaneous pain or hyperalgesia-hyperesthesia not limited to a single nerve territory and disproportionate to the inciting event. 3. Edema, skin blood flow (temperature), or sudomotor abnormalities, motor symptoms, or trophic changes present in the affected limb, in particular at distal sites. 4. Other diagnoses are excluded. Although the IASP diagnostic criteria had a high sensitivity, their specificity was only around 40%. In 2007, the Budapest Consensus refined the diagnostic criteria to include stricter conditions for clinical diagnosis, which increased the specificity to about 70% while maintaining high sensitivity. The criteria were as follows: 1. Continuing pain disproportionate to any inciting event 2. Must report at least one symptom in three of the four categories listed in Table 7.1

3. Must display at least one sign at time of evaluation in two or more of the categories listed in Table 7.1 4. No other diagnosis better explains the signs and symptoms (see Table 7.1) Schwartzman et al divided CRPS into three clinical stages, which are useful descriptively. The syndrome may not always follow this stepwise evolution. The stages of CRPS are described as follows: i. Stage 1: severe pain; pitting edema; redness; warmth; increased hair and nail growth; hyperhidrosis may begin; osteoporosis may begin. ii. Stage 2: continued pain; brawny edema; periarticular thickening; cyanosis or pallor; livedo reticularis; coolness; hyperhidrosis; increased osteoporosis; ridged nails. iii. Stage 3: pallor; dry, cool skin; atrophic soft tissue (dystrophy); contracture; extensive osteoporosis.

4. How does one make the diagnosis of CRPS? CPRS is primarily a clinical diagnosis The pathophysiology of CRPS is poorly understood. Based on current literature, several hypothesized mechanisms appear to play roles: autonomic dysfunction, neurogenic inflammation, and neuroplastic changes within the central nervous system (central/peripheral sensitization and progressive small-fiber degeneration).

47

Chapter 7: Complex regional pain syndrome

Currently no diagnostic test is considered a gold standard and no objective test is specific for CRPS. However, several diagnostic studies may be helpful in its evaluation and to rule out other pathologic processes. Autonomic function can be assessed by following tests: infrared thermometry and thermography, quantitative sudomotor axon reflex test (QSART), thermoregulatory sweat test (TST), and laser Doppler flowmetry. The skin temperature can be measured by Doppler flowmeter and infrared thermography; cutaneous blood flow can be measured by vital capillaroscopy (the affected extremity may demonstrate higher perfusion); sweat output can be assessed by quantitative sudomotor axon reflex testing; and coexisting nerve injury and muscle fiber loss can be quantified by electromyography and nerve conduction studies. The limitations of these tests are non-specific to this condition and most require special equipment and setup that make clinical applications less viable. Imaging is useful to exclude other diagnoses. Plain films are usually normal except in extreme cases, in which demineralization can occur (Sudeck’s atrophy). Trophic changes can be assessed by threephase bone scintigraphy, which detects pathologic delayed uptake in the distal bones such as the metacarpophalangeal or metacarpal bones. The sensitivity and specificity of three-phase bone scintigraphy are variable. Although an abnormal bone scan finding can confirm the clinical diagnosis of CRPS, the condition cannot be ruled out by a normal study. Magnetic resonance imaging may demonstrate marrow edema, soft tissue swelling, and joint effusion. Although clinically unavailable, central nervous system functional imaging studies may provide clues to reorganization in central somatosensory and motor networks, which lead to an altered central processing of tactile and nociceptive stimuli, as well as to an altered cerebral organization of movement.

5. What is the treatment approach for CRPS? Prompt diagnosis and early treatment is the cornerstone in management. It helps to avoid secondary physical problems associated with disuse of the affected limb and the psychologic consequences of living with undiagnosed chronic pain. Early referral to physiotherapy and encouraging gentle movement as

48

early as possible may potentially prevent progression of symptoms. Except in mild cases, patients with CRPS are generally best managed in specialist pain management or rehabilitation programs. An integrated and interdisciplinary pain rehabilitation treatment approach that includes the following four components is required: a. Patient information and education b. Pain relief with medications and procedures c. Physical and vocational rehabilitation d. Psychologic interventions (pain-coping skills, biofeedback, relaxation training, and cognitive behavior therapy) Treatment with medications and procedures can be individualized according to the symptoms, signs, and degree of severity. Tricyclic antidepressants are traditional choices in neuropathic pain disorders with good evidence to support their use for neuropathic pain. Antiepileptic agents are some of the best-studied agents for neuropathic pain, and strong evidence demonstrates their effectiveness. Non-steroidal antiinflammatory drugs may be effective in the acute phase with symptoms of swelling, erythema, or warmth. Oral corticosteroid agents can be particularly effective early in the disease when significant inflammation is present, and their use is substantially supported by randomized controlled clinical trials. A short course of steroids in the acute stage of the disease may be indicated. The lidocaine patch is used topically to deliver medication locally to the area of allodynia. Because of the suspected role of increased sympathetic nervous system activity in CRPS, alphaadrenergic antagonists such as phenoxybenzamine and phentolamine have also been used and may be beneficial in cases of sympathetically maintained pain. Opioids may be useful in the acute stages of CRPS for pain control. However, their use in chronic pain conditions and conditions with neuropathic features remains controversial. Methadone may be a choice in cases of severe neuropathic pain because of its NMDA receptor antagonist activity. Bisphosphonates have been tested in randomized controlled trials with some demonstrated efficacy, with the assumption that antinociceptive effect is primarily due to their capacity to inactivate osteoclasts and inhibit prostaglandin E2, proteolytic enzymes, and lactic acid. Calcitonin is another recent addition to the CRPS drug therapy armamentarium. However, results of randomized trials have been equivocal.

Chapter 7: Complex regional pain syndrome

6. What interventional methods are available to treat CRPS? Local anesthetic sympathetic blockade is the conventional and most common early intervention. However, patients can be divided by those with sympathetically maintained pain and those with sympathetically independent pain based on positive or negative response to selective sympathetic blockade or blockade of the alpha-adrenergic receptors. Stellate ganglion blocks for upper limb and lumbar chain blocks for lower limb symptoms can be offered. Alternatively, intrapleural infusion of local anesthetic can be used to block the sympathetic chain from T1 to L2.

Bier block procedures, involving the intravenous infusion of pharmacologic substances into a limb after gravitational drainage of the venous bed, may also be used. Depending on the substance infused, this can accomplish regional sympathetic blockade with guanethidine, sensorimotor blockade with lidocaine, or a combination of the two. For those patients with sympathetically independent pain, regional sensorimotor blockade with lidocaine should be the early intervention of choice. Such procedures have the possibility of achieving rapid and effective pain relief, allowing more timely progression in rehabilitation. In addition to interventional pain control procedures, which should be used aggressively early in the Figure 7.1. Lumbar sympathetic block. From personal files of Rinoo V. Shah, MD, MBA.

Figure 7.2. Stellate ganglion block. From personal files of Rinoo V. Shah, MD, MBA.

49

Chapter 7: Complex regional pain syndrome

Figure 7.3. Spinal cord stimulation. From personal files of Rinoo V. Shah, MD, MBA.

Figure 7.5. Same patient as Figure 7.4: cervical spinal cord stimulation midline (dorsal columns) and dorsal root/entry zone stimulation. From personal files of Rinoo V. Shah, MD, MBA.

50

Figure 7.4. Cervical spinal cord stimulation midline/dorsal columns. From personal files of Rinoo V. Shah, MD, MBA.

disease course, spinal cord stimulation (SCS) can be a beneficial treatment modality for those who do not have a satisfactory response to the above treatment in 12–16 weeks. SCS has been shown to be effective for treatment of both CRPS I and CRPS II when other less invasive treatment strategies have failed. Neuromodulation may act to restore normal gamma-aminobutyric acid levels in the dorsal horn and affect release of adenosine, thus reducing neuropathic pain. SCS has proven effective in supporting functional restoration in the affected limb. Peripheral nerve stimulation uses a similar technique to SCS. However, due to a new modality, available data is limited. A spinal cord stimulator lead can sometimes be placed in the dorsolateral epidural space to target the dorsal roots or dorsal root entry zone. Patients who have a good response to sympathetic blocks can be offered sympathetic denervation through radiofrequency ablation or surgical sympathectomy. However, the quality of evidence for these treatments is poor and several complications can occur, which include postsympathectomy sympathalgia, compensatory hyperhidrosis, Horner’s syndrome, infection, and spinal cord injury.

Chapter 7: Complex regional pain syndrome

Sometimes CRPS may spread to the contralateral limb or to involve a different region of the body. Surgeons operating on patients with resolved or dormant CRPS must be aware of reactivation and spread of this disease, even if the surgery is remote to the original CRPS involved limb. If recurrence and spread occur, blocks and infusions targeting the sympathetically independent and maintained pain generators should be pursued, according to Shah and Day.[15]

7. What is the course of CRPS? The outcome of CRPS varies from person to person. Younger patients, especially children and teenagers, tend to have good recovery. Occasionally patients are left with unremitting pain and crippling, irreversible

References 1.

2.

3.

4.

5.

Janig W, Stanton-Hicks M (eds). Reflex sympathetic dystrophy: a reappraisal. In Progress In Pain Research and Management, vol. 6. Seattle, Washington: IASP Press; 1996. Harden RN, Bruehl S, StantonHicks M et al. Proposed new diagnostic criteria for complex regional pain syndrome. Pain Med. 2007;8:326–331. de Mos M, de Bruijn AGJ, Huygen FJPM, et al. The incidence of complex regional pain syndrome: a populationbased study. Pain. 2007; 129:12–20. Raja SN, Grabow TS. Complex regional pain syndrome I (reflex sympathetic dystrophy). Anesthesiology. 2002; 96:1254–1260. Veldman PHJM, Reynen HM, Arntz IE, et al. Signs and symptoms of reflex sympathetic dystrophy: prospective study of 829 patients. Lancet. 1993;342:1012–1016.

6.

7.

8.

changes despite treatment. There is some evidence to suggest early treatment, particularly rehabilitation, is helfpul in limiting the disorder, but this benefit has not yet been proven in large randomized clinical studies. More research is needed to understand the causes of CRPS, how it progresses, and the role of early treatment. In a sub-group of patients with unremitting chronic pain, it may give rise to physical deconditioning, anxiety, depression, weight gain, and sleep disturbance. Also, with inadequate treatment, all aspects of the patient’s life can be affected, often with negative social, vocational, financial, and recreational consequences. In addition, limb contracture and loss of strength may lead to difficulty in ambulation and activities of daily living.

Schwartzman RJ, McLellan TL. Reflex sympathetic dystrophy: a review. Arch Neurol. 1987;44:555–561. Atkins RM, Duckworth T, Kanis JA. Features of algodystrophy after Colles’ fracture. J Bone Joint Surg. 1990;72:105–110.

Schasfoort FC, Bussmann JB, Stam HJ. Impairments and activity limitations in subjects with chronic upper-limb complex regional pain syndrome type I. Arch Phys Med Rehabil. 2004;85:557–566. 9. Janig W, Baron R. Complex regional pain syndrome: mystery explained? Lancet Neurol. 2003;2:687–697. 10. Cepeda MS, Carr DB, Lau J. Local anesthetic sympathetic blockade for complex regional pain syndrome. Cochrane Database Syst Rev. 2005;4:CD004598. 11. Taylor RS, Van Buyten JP, Buchser E. Spinal cord stimulation for complex regional pain syndrome: a systematic review of the clinical and costeffectiveness literature and

assessment of prognostic factors. Eur J Pain. 2006;10:91–101. 12. Turner-Stokes L, Goebel A. Complex regional pain syndrome in adults: concise guidance. Clin Med. 2011; 11: 596–600. 13. Albazaz R, Wong Y, HomerVanniasinkam S. Complex regional pain syndrome: a review. Ann Vasc Surg. 2008;22: 297–306. 14. Sharma A, Williams K, Raja S. Advances in treatment of complex regional pain syndrome: recent insights on a perplexing disease. Curr Opin Anaesthesiol, 2006;19:566–572. 15. Shah RV, Day MR. Recurrence and spread of complex regional pain syndrome caused by remotesite surgery: a case report. Am J Orthop (Belle Mead NJ). 2006;35 (11):523–526. 16. Bailey A, Auclette JF. Complex regional pain syndrome. In Frontera WR, Silver JK, Rizzo TD, (eds). Essentials of Physical Medicine and Rehabilitation, 2nd edn. Philadelphia, PA: Saunders/ Elsevier. 2008: pp. 511–517.

51

Section 1 Chapter

8

Neurological Disorders

Diabetic neuropathy Gulshan Doulatram and Tilak Raj

Case study A 55-year-old female presents to your clinic with pain in the back and associated tingling, numbness, and pain in both her legs for the past year. The symptoms are worse at night. She has an antalgic broad-based gait and has been using a walker to get around. She was working as a school teacher, but has been on short-term disability for the last 4 months. She is depressed about her prognosis and is asking you to fix her so she can return to work.

1. How prevalent is this disease presentation?Couldyouexplainsomeof the epidemiologic features of this disease? Are there any cost concerns? The 2011 Diabetes fact sheet published by the Centers for Disease Control confirmed that 25 million Americans have diabetes, a disease now affecting one in every four patients. Five percent of these individuals have type 1 diabetes mellitus (DM) and 95% have type 2 DM. Diabetic patients have a life time prevalence of 60% of developing diabetic neuropathy.[1] There are currently 8 million people in the USA who have symptomatic diabetic polyneuropathy. Diabetic polyneuropathy is a length-dependent disorder of peripheral nerve fibers, characterized by a distal-to-proximal loss of peripheral nerve axons and function. The progression of disease from painful neuropathy to loss of sensation and development of foot ulcers and amputations causes a significant burden to society both in social and financial ways. The cost of diabetic neuropathy was estimated to be $50 billion in 2007, which is 25% of total costs related to DM. This number is expected to rise exponentially.[2] Patients with painful diabetic peripheral neuropathy (PDPN) are more likely

to have foot ulcers and amputations, further increasing the burden of the disease and decreasing quality of life for those affected. The prevalence of diabetic polyneuropathy (DPN) and PDPN increases with age, duration of diabetes, and worsening of glucose tolerance. The overall prevalence of PDPN in the diabetic population is 15%.[3]

2. What are some of the other conditions that could have the same presentation? In up to 25% of diabetic patients with neuropathy, the neuropathy could have another cause. Hence the diagnosis of diabetic neuropathy requires careful evaluation.[4,5] Other conditions with a similar presentation include:  Posterior disc protrusion  Spinal cord tumors  Malignant nerve root infiltrations  Inflammatory neuropathies  Pernicious anemia  Vitamin B6 intoxication  Alcoholism  Uremia  Chemical toxins  Nerve entrapment and compression of benign etiology  Hepatitis  Idiopathic  Congenital (various hereditary sensory motor neuropathies)  Paraneoplastic syndrome  Syphilis  HIV/AIDS

Case Studies in Pain Management, ed. Alan David Kaye and Rinoo V. Shah. Published by Cambridge University Press. © Cambridge University Press 2015.

52

Chapter 8: Diabetic neuropathy

 Medication (e.g., chemotherapy, isoniazid, radiation induced)  Spine disease (e.g., radiculopathy, stenosis)  Vitamin B12 deficiency  Hypothyroidism

3. How would you differentiate diabetic neuropathy from some of these other disorders? Diabetic neuropathy is usually diagnosed presumptively by the presentation including the symptoms, medical history, and physical exam. Fasting plasma glucose and hemoglobin A1c are important laboratory screening tests for diabetic neuropathy. While imaging of the spine rarely helps diagnose or manage diabetic neuropathy, it may exclude other causes mimicking diabetic neuropathy. A subset of patients may have abnormal glucose tolerance (AGT) and still have severe neuropathic symptoms.

4. What diagnostic studies would you obtain?  Fasting plasma glucose and hemoglobin A1c – screening tests for diabetes.  Urine analysis to screen for nephropathy and proteinuria.  Complete blood cell count and complete metabolic panel (electrolytes, renal function, and liver function panel).  Vitamin B12 and folate levels.  Thyroid function tests.  Erythrocyte sedimentation rate and C-reactive protein.  Serum protein electrophoresis with immunofixation electrophoresis.  Antinuclear antibody, Anti-SSA and SSB antibodies, rheumatoid factor.  Paraneoplastic antibodies.  Rapid plasma regain.  Genetic screens.  Hematology screen to check for anemia.  Nerve conduction studies (NCS) and electromyography (EMG).  Imaging of the spine to exclude other causes.  Quantitative sensory testing (QST) – QST[6] measures sensory thresholds for pain, touch,

vibration, and hot and cold temperature sensation. It is increasingly used, especially in clinical therapeutic trials. A number of devices are commercially available and range from handheld tools to sophisticated computerized equipment with complicated testing algorithms. Specific fiber functions can be assessed: Aδ-fibers with cold and cold-pain detection thresholds, C-fibers with heat and heat-pain detection thresholds, and large fiber (Aαβ-) functions with vibration detection thresholds. Elevated sensory thresholds correlate with sensory loss and lowered thresholds occur in allodynia and hyperalgesia. In asymptomatic patients, abnormal QST thresholds suggest subclinical nerve damage. QST is a psychophysical test and therefore is dependent upon patient motivation, alertness, and concentration.  Autonomic function testing. Autonomic testing is valuable in patients with neuropathic pain disorder in which patients had normal or mildly abnormal electrophysiologic (NCV/EMG) findings (27% of patients). The most useful tests are the QSART, thermoregulatory sweat test, heart rate responses to deep breathing, Valsalva ratio, and surface skin temperature. In a recent study of patients with diabetic polyneuropathy, discordance was noted between efferent C-fiber responses in sudomotor tests (QSART and sweat imprint), and primary afferent (nociceptor) C-fiber axon reflex flare responses. These findings indicate that these two C-fiber subclasses can be differentially affected in diabetic small-fiber polyneuropathy. Autonomic functions can also be abnormal in peripheral neuropathies not associated with pain.  Skin biopsy. Epidermal nerve fiber density and morphology, e.g., tortuosity, complex ramifications, clustering, and axon swellings, can be quantified and compared with control values with a 3-cm skin biopsy and immunohistochemistry. A reduced density of epidermal nerve fibers is seen in small-fiber neuropathies, diabetic neuropathy, and impaired glucose tolerance neuropathy, each of which is associated with neuropathic pain. In a subgroup analysis of one such study, the skin biopsy was found to be a more sensitive measure than QSART or QST in diagnosing neuropathy in patients with burning feet and normal NCVs.

53

Chapter 8: Diabetic neuropathy

 Functional brain imaging. Positron emission tomography (PET) and functional magnetic resonance imaging (fMRI) may not be practically used but are currently useful in study outcomes and have potential in the future.  Microneurography. This looks at individual nerve fibers and is very sensitive in detecting early neuropathy, but can be too cumbersome to be used routinely in clinical practice.

5. How would you make a diagnosis of diabetic neuropathy? Screening test for DPN. The Center for Medicare and Medicaid Services, the International Diabetes Federation, and the World Health Organization recommend that this testing on the feet should be done by The Semmes Weinstein monofilament examination (SWME). The optimal method is to use the 5.07/10 g monofilament to test the plantar aspects of the great toe, third, and fifth metatarsal heads. Patients unable to detect one or more sites should be classified as at risk in order to maximize sensitivity.[7] Neuropathic Pain Scale (NPS). This is the first scale developed specifically to assess neuropathic pain. The NPS includes characteristics of pain intensity and unpleasantness that assess the global dimensions of pain. The NPS also includes eight classifications of pain that assess specific qualities of neuropathic pain: sharp (like a knife), hot (on fire), dull (aching), cold (freezing), sensitive (raw skin), itchy (like poison oak), deep, and surface. Leeds Assessment of Neuropathic Symptoms and Signs Pain Scale (LANSS). This consists of a 7-item pain scale, including both sensory descriptors and items for sensory examination. Neuropathic Pain Questionnaire (NPQ). This instrument provides a general assessment of neuropathic pain symptoms and is useful in discriminating between neuropathic and non-neuropathic pain. Neuropathic Pain Symptom Inventory (NPSI). NPSI includes 10 descriptors that allow discrimination and quantification of five distinct clinically relevant dimensions of neuropathic pain syndromes. The psychometric properties of the NPSI make it extremely useful in assessing and quantifying the response to various pharmacologic and non-pharmacologic interventions. All of these scales are extremely useful in studies to assess outcomes of specific modalities but are not commonly used in clinical practice.

54

Table 8.1. Defining citeria for diabetic polyneuropathy according to The Toronto Expert Panel on Diabetic Neuropathy[5]

Classification

Characteristics

Possible clinical DN

Symptoms or signs of DN. Symptoms can be positive (pain) or negative (loss of sensation) in the feet Signs can include symmetrical decreased sensory loss in the feet or decreased or absent ankle reflexes

Probable clinical DN

A combination of symptoms and signs of DN, as described above, with any two or more of the following: neuropathic symptoms, decreased sensation, or decreased or absent ankle reflexes

Confirmed clinical DN

An abnormal nerve conduction study and a symptom or sign of DN, as described above

Subclinical DN (stage 1a)

An abnormal nerve conduction study with no signs or symptoms of DN

Diagnostic criteria for diabetic polyneuropathy were developed 25 years ago by a panel of neurologists and diabetologists but proved too costly and time-consuming. A second group of experts (Toronto Expert Panel on Diabetic Neuropathy) published revised criteria in 2011. These criteria are presented in Table 8.1.[1]

6. What are some of the other presentations in diabetic neuropathy? Diabetic neuropathies are heterogeneous diseases and can have diverse clinical manifestations. They can be classified as follows:[8]  Generalized or symmetrical neuropathies: 

Sensory neuropathies  



Acute sensorimotor neuropathy Chronic sensorimotor distal polyneuropathy

Autonomic neuropathies   

Cardiovascular Gastrointestinal Genitourinary

Chapter 8: Diabetic neuropathy

 Focal and multifocal or asymmetrical neuropathies:     

Cranial Truncal Focal limb Proximal motor (amyotropy) Chronic inflammatory demyelinating polyneuropathies (CIDP)

Generalized, symmetrical polyneuropathy is the most common type and may have sensory, motor, and autonomic manifestations. It typically has a chronic presentation with features of small-fiber dysfunction, i.e., pain with loss of pain, temperature and vibration sensory perception, absent ankle reflexes, and formation of foot ulcers. Symptoms start in the toes and feet and ascend in the lower limb; upper limb involvement is rare and occurs in long-standing disease. Some neuropathic features include:  Paresthesias – abnormal sensations that are not painful; some examples include tingling and burning  Dysesthesia – abnormal sensations that are painful  Mechanical allodynia – abnormal perception of pain from usually non-painful mechanical stimulation  Thermal allodynia – abnormal sensation of pain from usually non-painful thermal stimulation such as cold or warmth  Summation – abnormally increasing painful sensation to a repeated stimulus although the actual stimulus remains constant  Hyperalgesia – exaggerated pain response from a usually painful stimulation  Hyperpathia – abnormally painful and exaggerated reaction to a stimulus, especially to repetitive stimuli  Aftersensation – abnormal persistence of a sensory perception provoked by a stimulus even though the stimulus has ceased Autonomic neuropathy is common and underreported and may affect many organ systems but most commonly involves cardiovascular, gastrointestinal, and genitourinary systems. Features include resting tachycardia, orthostatic hypotension, distal anhidrosis, sexual dysfunction, and gastrointestinal features including severe constipation and diarrhea. Among the multifocal neuropathies, the mononeuropathies commonly involve median, ulnar, and

common peroneal nerves. Cranial neuropathies involving the oculomotor and abducens nerve are rare. Diabetic amyotropy generally occurs in type 2 diabetes, and is subacute with pain, asymmetric weakness, and atrophy of proximal limb muscles. Rarely distal lower limb and upper limb muscles may be involved. Sensory deficit is minimal but pain is usually severe with loss of patellar reflex. Other subacute presentations include:  Development of acute sensory neuropathy when blood sugar levels are high  Development of acute sensory neuropathy when treated with insulin  Acute painful neuropathy associated with weight loss (diabetic neuropathic cachexia)

7. What are the EMG findings in diabetic neuropathy? In patients with diabetes, abnormalities may be found on nerve conduction study, even in the absence of clinical symptoms of polyneuropathy. Nerve conduction studies and electromyography (EMG) can provide objective evidence of dysfunction of large myelinated (Aβ) nerves, characterize the neuropathy (e.g., axonal, demyelinating), localize it (e.g., mononeuropathy versus radiculopathy or distal neuropathy), and possibly assess the severity and even prognosis. Findings on nerve conduction studies depend on the pattern of nerve damage.[9] Axon loss results in loss of amplitude of nerve action potentials, and evidence of denervation is found on needle examination of affected muscles. Myelin loss results in slowed conduction velocities, prolonged distal latencies, conduction block, temporal dispersion, and prolonged minimum F-wave latencies. NCS/EMG does not provide information about the function of small myelinated (Aδ) or unmyelinated (C) fibers which is a major limitation. Polyneuropathies with only small-fiber involvement can have normal NCVs and EMG despite significant nerve damage and neuropathic pain. The most common presentation is distal symmetric sensorimotor neuropathy, which is associated with predominant axonal loss and causes reduced or absent sensory nerve action potentials. The lower limbs are affected first and more severely. With progression of the neuropathy, compound motor action potential amplitudes may also be reduced and abnormalities may then be observed in the hands.[10] These changes

55

Chapter 8: Diabetic neuropathy

reflect length-dependent degeneration of large-diameter myelinated nerve fibers. Conduction velocities are generally within the normal range (or only mildly slowed) in distal symmetrical polyneuropathy. Conduction velocities less than 70% or a conduction block may suggest demyelination, and, if generalized, should prompt further evaluation for CIDP. EMG may be normal in mild or asymptomatic subjects, but demonstrates denervation in more severe cases. Denervation changes include positive sharp waves and fibrillation potentials (spontaneous discharges). Chronicity is indicated by reinnervation changes such as large-amplitude, long-duration, and polyphasic motor unit potentials. Focal slowing of conduction velocity at common entrapment sites may indicate one of the mononeuropathies. Paraspinal muscle abnormalities indicate spinal nerve root disease.

8. What is Seddon’s and Sunderland’s classification for peripheral nerve injuries? The most widely accepted classifications of peripheral nerve injuries are those by Seddon (neuropraxia, axonotmesis, and neurotmesis) and Sunderland (Grade 1–5 nerve injury)[10] (see Table 8.2).

Table 8.2. Classification of peripheral nerve injuries

Seddon

Sunderland

Pathophysiology

Neuropraxia (compression)

Type 1

Local myelin damage with the nerve still intact

Axonotmesis (crush)

Type 2

The continuity of axons is lost. The endoneurium, perineurium, and epineurium remain intact. Loss of continuity of axons with wallerian degeneration due to disruption of axoplasmic flow Type 2 with endoneurial injury Type 2 with endoneurial and perineurial injury but an intact epineurium

Type 3 Type 4

Neurotmesis (transection)

Type 5

Complete physiologic disruption of the entire nerve trunk. Early surgical intervention necessary. Prognosis guarded

9. What is the pathophysiology of PDPN? The pathophysiology of PDPN is multifactorial and involves both the toxic effect of glucose on nerve cells and ischemia of peripheral nerves.[11] High blood glucose activates the polyol pathway, generates reactive oxygen species (ROS), and causes accumulation of advanced glycation end products (AGE). Accumulation of sorbitol produced by activation of the polyol pathway causes reduction in nerve myo-inositol and disruption of Na+/K+-ATPase membrane activity, leading to intracellular sodium accumulation, impaired axonal transport, and finally structural damage to the nerves. Hyperglycemia causes glycation of the free amino group on proteins, lipids, and nucleic acids with alteration in their molecular structure and function. The basement membrane of endothelial cells becomes glycosylated which causes vasoconstriction and contributes further to ischemia. AGEs bind to receptors of AGE on macrophages, causing the release of a cascade of proinflammatory cytokines (interleukin-1,

56

tumor necrosis factor-α), growth factors (insulin-like growth factor, platelet-derived growth factor, tissue growth factor-β), and adhesion molecules (vascular cell adhesion molecules-1) (VCAM-1). Elevated intracellular glucose causes activation of protein kinase C (PKC), which produces direct neuronal damage by affecting endothelial function. Several treatment modalities have targeted these pathophysiologic pathways with varying success in PDPN.

10. What are some treatment modalities you would offer to this patient? The treatment of the PDPN is multi-pronged to achieve optimal results. The treatment modalities include a vast array of options, including physical therapy, psychology, injections, implantable devices

Chapter 8: Diabetic neuropathy

Figure 8.1. Diabetic neuropathy.

DIABETIC NEUROPATHY

HISTORY / EXAM

BLOOD GLUCOSE, HgA1C

RULE OUT OTHER CAUSES

POSSIBLE QST, EMG, NCV, SKIN BIOPSY IF DIAGNOSIS IS IN DOUBT

MEDICAL MANAGEMENT

TRICYCLIC ANTIDEPRESSANT

SEROTONIN NOREPINEPHRINE REUPTAKE INHIBITORS

VOLTAGE-GATED CALCIUM CHANNEL BLOCKERS

COMBINATION OF THREE

NO RELIEF

OPIOIDS

TRAMADOL

NO RELIEF

SPINAL CORD STIMULATION

or neuromodulation, surgery, medications, holistic treatments, and herbal medications. Overall, strict blood sugar control is necessary to prevent some of the diabetic complications. A definite correlation has been found between the degree of blood sugar control and development of painful neuropathy. Lifestyle modification with diet, exercise, and correction of metabolic derangements and associated morbidities are essential for optimal outcomes. Most of the treatment modalities available to a diabetic patient are often only symptomatic and do not change the course of the disease. Treatment can often be frustrating, both for the patient and pain practitioner, especially when medications fail to provide the desired relief;

however a stepwise logical approach must be utilized in all patients with PDPN. The treatment algorithm presented here is a modification from guidelines formulated by the Toronto Expert Panel on Diabetic Neuropathy.[12] (Figure 8.1)

11. Disease-modifying medications α-Lipoic acid D-L-α-lipoic acid (ALA), a potent antioxidant, has been extensively evaluated in prospective, placebocontrolled studies in subjects with PDPN. Several large-scale trials have shown an improvement in both

57

Chapter 8: Diabetic neuropathy

the neuropathic pain scores and electrophysiologic parameters.[11] A dose of 600 mg/day appears to offer the best balance between efficacy and side effect profile.

Protein kinase C inhibitors Activation of PKC is thought to play an important role in the microvascular complications of diabetes. A multinational, randomized, Phase II double-blind placebo-controlled trial using ruboxistaurin (a PKC-β inhibitor) showed improvement in neuropathy and vibration detection thresholds (VDT).

Polyol pathway Aldose reductase inhibitors (ARIs) reduce the flux of glucose through the polyol or sorbitol pathway, hence decreasing the levels of intracellular sorbitol and fructose. Fidarestat was associated with significant improvement in electrophysiologic and symptomatic measures in patients with type 1 and 2 diabetes. The ARI epalrestat, approved in Japan for clinical use, also showed an improvement in patients’ symptoms and prevented the deterioration of median motor nerve conduction velocity. Another ARI that has been evaluated in a Phase III study is ranirestat, which has been shown to slow the progression of neuropathy.

Advanced glycation end products The accumulation of AGEs causes release of inflammatory mediators and leads to microvascular damage. However, the identification and testing of a safe AGE inhibitor has proved problematic. Amino-guanidine was discontinued because of toxicity in humans. Benfotiamine, a derivative of thiamine (vitamin B1), has been shown to reduce tissue AGEs. Studies with benfotiamine have been reported to show some effectiveness compared to placebo. Benfotiamine has also been studied in combination with pyridoxamine (vitamin B6) and cyanocobalamin (vitamin B12). These studies reported a significant improvement in vibration perception threshold, motor function, and symptom scores.

12. Symptomatic treatment Anticonvulsants These include the traditional agents, such as carbamazepine and valproate, and newer agents, such as gabapentin and pregabalin.

58

Carbamazepine, phenytoin (Dilantin), and valproate are some of the older anticonvulsants that have been used to treat neuropathic pain. There are not too many studies specifically testing these drugs for PDPN. Patients should have detailed laboratory tests prior to initiation of therapy, including: blood urea nitrogen, creatinine, transaminase, iron levels, a complete blood count (including platelets), reticulocyte count, and liver function test. Carbamazepine can also cause dermatologic reactions, such as toxic epidermal necrolysis and Stevens–Johnson syndrome. In light of the array of adverse effects, newer anticonvulsants are preferred.[12,13] Gabapentin is used in the treatment of neuropathic pain and PDPN. The major side effects reported from gabapentin include sedation and dizziness. The major drawbacks are the requirement of high doses and poor bioavailability.[12] Pregabalin, an analog of the neurotransmitter gamma-aminobutyric acid (GABA), binds the alpha2delta (a2-d) unit of the calcium channels, reducing calcium influx at nerve terminals. This reduces the release of several neurotransmitters, including glutamate, noradrenaline, and substance P. Pregabalin does not bind GABA-A and GABA-B receptors, and it is not converted metabolically into GABA.[13] A 2008 metaanalysis of seven trials showed pregabalin was effective in treating diabetic peripheral neuropathic pain in 1510 patients.[14] Pregabalin is one of the only two drugs approved by the FDA in the treatment of PDPN. Pregabalin is usually well tolerated, and has a good safety profile. Common side effects include somnolence, dizziness, weight gain, and peripheral edema, which rarely require stopping the medication. Rare but serious adverse events include rhabdomyolysis, acute renal failure, hyperthermia, and secondary acute-angle glaucoma. The dose of pregabalin requires careful titration in patients with chronic kidney disease. Pregabalin has the advantage of improving mood and sleep, and thus addresses the interaction of chronic pain, sleep loss, and mood disturbance in diabetic neuropathy.

Antidepressants TCA antidepressants, such as amitryptiline and nortryptiline, are effective in the treatment of diabetic neuropathy because of their central modulation of inhibitory pathways.[15] They are not tolerated well by patients due to their effects on alpha-adrenergic,

Chapter 8: Diabetic neuropathy

H1-histamine, muscarinic cholinergic, and N-methylD-aspartate receptors. Some of the adverse effects reported with TCAs include orthostatic hypotension, cardiac arrhythmias, dizziness, and sedation. TCAs are contraindicated in the presence of heart failure, arrhythmias, or recent myocardial infarction. Because of the anticholinergic effects of TCAs, physicians should be cautious when prescribing them for patients with narrow-angle glaucoma, benign prostatic hypertrophy, orthostasis, urinary retention, impaired liver function, or thyroid disease. QTc interval should be assessed because of the risk of torsades de pointes. Serotonin-norepinephrine reuptake inhibitors (SNRIs), including venlafaxine (Effexor) and duloxetine (Cymbalta), are also used in the treatment of diabetic peripheral neuropathic pain.[2,16] They are better tolerated and have fewer drug interactions than TCAs. Several trials have shown promising results with venlafaxine. Duloxetine hydrochloride is a dual-reuptake inhibitor of both 5-HT and NE (SNRI) transporters, and is the only other agent (apart from pregabalin) that has been approved by the FDA in the treatment of diabetic neuropathy.[17] Duloxetine has been shown to be both effective and well tolerated. Some of the side effects include somnolence, nausea, dizziness, decreased appetite, and constipation. When compared to TCA, duloxetine can be safely prescribed to diabetic patients with concomitant cardiovascular problems. It has also been shown that if either pregabalin or duloxetine is not effective, combination therapy can be tried in the treatment of diabetic neuropathy. Duloxetine was found to be more cost-effective than pregabalin. All three classes of drugs are found to be fairly effective in the treatment of PDPN. Pregabalin has been shown to improve sleep function as an added advantage compared to duloxetine and tricyclic antidepressants.[18,19]

Local anesthetics Intravenous lidocaine has been shown to be effective in diabetic neuropathy; however, the duration and need for monitoring do not make this practical in a long-term setting. Oral mexilitine is used if there is a positive response to lidocaine. However, its use is extremely limited by its side effects.

Topical agents 5% lidocaine, a sodium channel blocker, is used for patients with painful sensory neuropathy, and is a useful adjunct to the use of antidepressants and anticonvulsants.[20] A multicenter randomized, openlabel, parallel-group study of lidocaine patch versus pregabalin with a drug washout phase of up to 2 weeks and a comparative phase of 4-week treatment period showed that lidocaine was as effective as pregabalin in reducing pain and was free of side effects. Capsaicin (0.075%) is a topically applied alkaloid that acts peripherally by depleting the neurotransmitter substance P from sensory nerves. The most common adverse effects are stinging and burning related to the brief release of substance P. Recently, an 8% capsaicin patch has shown to provide long-lasting relief in twothirds of the study population.[20] Topical clonidine gel has also shown promise in a few patients with minimal side effects. Several compounding creams containing a mixture of different medications including gabapentin, non-steroidal anti-inflammatory drugs (NSAIDs), and clonidine are now available. The effectiveness of these creams has not yet been established. Tramadol acts at both the opioid receptor and serotonin/norepinephrine receptor, and has been shown to be effective in treating pain, quality of life, and physical functioning in diabetic patients.[16] However, tramadol should be used as a second-line drug only after firstline treatments either alone or in combination have been found to be ineffective. The side effects of tramadol are related to both its opioid and serotonergic effects. Constipation, respiratory depression, lowered seizure threshold, somnolence, and serotonin syndrome (especially in patients taking concomitant antidepressants) can occur. Opioids, including short-acting and long-acting opioids, are used as the last line of medications if all other medications have failed and the pain is associated with other musculoskeletal abnormalities.[2] In that case, opioids should be used along with other neuropathic medications. Monotherapy with opiates should be reserved for patients who do not achieve pain relief goals with other therapies. Relying on opioids as sole agents in the treatment carries the risk of tolerance and opioid-induced hyperalgesia. A 2006 Cochrane review evaluated the use of opiates including methadone, levorphanol, morphine, and controlled-release oxycodone (Oxycontin) and demonstrated the superiority of opiates over placebo.

59

Chapter 8: Diabetic neuropathy

Combination therapies

Intrathecal medication devices

Traditionally, single agents are tried prior to starting combination therapies. Some limited evidence exists to support combination therapies including adding opioids to gabapentin. Similarly, a combination of TCA and gabapentin has been shown to be effective. Current consensus guidelines specify that the treatment of DN should include first-line drugs including antidepressants such as tricyclics, venlafaxine, duloxetine, and pregabalin. If these medications are not successful, then second-line drugs such as tramadol and opioids can be used in conjunction with the first-line drugs.[12] However, the concomitant use of tramadol and SSRIs carries the risk of serotonin syndrome, a potentially serious condition.

There are no clinical trials supporting intrathecal medication devices (IMD) in the treatment of PDPN. The use of ziconitide has been found to be promising in other neuropathic conditions, but no studies have been done to demonstrate its efficacy in PDPN.

Interventional therapy Spinal cord stimulation SCS can be an option if the patient has failed conventional treatment or if such treatment is limited due to side effects. There are four studies which look at the efficacy of SCS in PDPN.[21,22] These studies showed long-term clinically relevant pain relief ranging from 2 to 5 years (SCS). The mechanism of pain relief with SCS for diabetic neuropathy is by a direct effect on spinothalamic tracts, segmental inhibition via coarse fiber activation, effects on the central sympathetic system, and brain stem loops to thalamocortical mechanisms. Patients who have severe autonomic neuropathy did not seem to get long-term benefit from pain and ischemia with SCS. Improvement in transcutaneous oxygen tension (TCPO2) in the trial period was a good prognostic sign for long-term benefits from SCS.[23,24] Diabetic patients with peripheral arterial occlusive disease and severe autonomic neuropathy, and those without an increase of TCPO2 in the test period should be excluded from permanent device implantation on the basis of poor long-term results and cost. One must keep in mind that all the trials supporting the use of SCS for PDPN are small and in the absence of RCTs, a blanket recommendation for the use of SCS cannot be made. Decision to use SCS must be made on an individual basis after all other modalities have failed. Further studies should also be able to predict if a certain subgroup of PDPN will respond to SCS, or if SCS should be used early or late in the disease process.

60

Deep brain stimulation A review by the European Federation of Neurologic Societies found weak but positive results for deep brain stimulation (DBS) in peripheral neuropathic pain, with 70% of the small number of patients showing long-term benefit.

Surgery A review of 11 case series has shown improvement in pain scores in patients undergoing surgical decompression. Good outcomes were predicted by the presence of a positive Tinel sign preoperatively in both diabetic and non-diabetic patients.[25] However, surgery is reserved only for those patients where there is both clinical and diagnostic evidence (NCV) of nerve compression.

Physical therapy Patients with PDPN have increased risks of falls, pressure ulcers, and limited mobility due to pain, foot deformities, and sensory neuropathy. Weight-bearing exercise has the advantage of improving several parameters, including glucose levels, deconditioning, and overall quality of life, but must be balanced against constant plantar pressure and predisposition to developing ulcers in insensate feet.[26] In an independent study, Tai Chi improved glucose control, balance, neuropathic symptoms, and some dimensions of quality of life in diabetic patients with neuropathy.[27]

Psychologic treatment Diabetic patients suffer from a significantly higher rate of depressive symptoms due to the chronicity of the disease, pain, limited mobility, and complications caused either by the disease itself or its treatments. Psychotherapy treatments in diabetes mellitus can have a positive influence on anxiety and depressive symptoms, but more importantly also improve somatic complaints and blood glucose levels.[28,29] Specifically, biofeedback can reduce foot pressure to a safe level in patients with PDPN.[28]

Chapter 8: Diabetic neuropathy

Complementary and alternative medicine Complementary and alternative medicine (CAM) modalities can be used as adjuncts along with traditional therapies. Some of the CAM modalities found to be effective include biofeedback, hypnosis, and progressive muscle relaxation. A study also showed that thermal biofeedback improved healing of foot ulcers by increasing nutritive blood flow. In addition, biofeedback decreases blood pressure, probably by reducing sympathetic outflow and by enhancing the descending inhibitory systems. L-carnitine, alphalipoic acid, and primrose oil, all of which are available over-the-counter, have shown positive results, but more long-term data are needed. TENS therapy has shown good results and can be used as an alternative non-pharmacologic therapy. TENS works by the production of endogenous opioids and gate control mechanisms.[30] It has been initially postulated that low-frequency TENS works by the release of endogenous opioids, and high-frequency TENS works by stimulation of large-diameter afferent fibers, inhibiting second-order neurons in the dorsal horn and preventing impulses carried by smalldiameter C and A delta fibers from being transmitted (Gate Control theory). Low-frequency TENS activates μ opioid receptors and high-frequency TENS activates delta opioid receptors. Frequency-modulated electromagnet neural stimulation therapy includes the use of pulsed electromagnetic fields, static magnetic field therapy, lowfrequency pulsed magnetic field, high-frequency external muscle stimulation (HF), frequency-modulated electromagnetic neural stimulation (FREMS), and percutaneous electrical nerve stimulation (PENS). Each of these modalities has been shown to be effective in individual studies, but a comparative meta-analysis did not confirm this. In the absence of this data, a recommendation for or against use of these modalities cannot be made and must be made at the individual level.

New developing drugs Several new and emerging drugs are currently in the testing phase for the treatment of PDPN. These drugs target specific receptors known to be upregulated in neuropathic pain.[31] Transient receptor potential channels–vanilloid receptors (TRP V1): These agents are promising

in the treatment of diabetic neuropathy. The only agent that is approved currently is capsaicin, TRPV1 agonist, used both in lower concentrations and the higher (8%) patch. Studies are currently underway evaluating the long-term effects of capsaicin for diabetic neuropathy. Some concerns about the patch reflect its ability to cause small-fiber sensory and autonomic denervation, both of which are concerning in patients who may already have peripheral denervation. Oral TRP antagonists are still at experimental stages, but are showing some promise in Phase 1 and 2 trials for neuropathic pain. Selective sodium channel blockers (Na (v) 1.3–1.9) are also currently in the planning phase for the treatment of diabetic neuropathic pain. Opioid agonist/norepinephrine reuptake inhibitor tapentadol. This is a novel, centrally acting analgesic which acts at the μ receptor and noradrenaline receptor and has been approved for the treatment of painful diabetic neuropathy. Several studies have suggested that the drug has an efficacy comparable to other opioids but a lower incidence of constipation. Gene therapy: Currently, the only open label uncontrolled study that has been shown to be effective is intramuscular injection of plasmid DNA containing hepatocyte growth factor (HGF) given 2 weeks apart in three doses. A randomized double-blinded placebo-controlled trial is underway to objectively assess the role of HGF in diabetic neuropathy. Tanezumab, an antibody that inhibits nerve growth factor, has been studied in patients with diabetic neuropathy and shown to have a positive response, although there was a higher incidence of joint-related complications.

13. What are some of the complications from the treatment modalities? Complications associated with SCS[32] include lead migration (14%), lead breakage (7%), implanted pulse generator migration (1%), loss of therapeutic effect/ paresthesia, infection or wound breakdown (10%), pain at pulse generator implantation site (12%), and fluid collection at pulse generator implantation site (5%). Overall, diabetic patients are at higher risks from SCS complications due to their underlying disease and poor wound healing.

61

Chapter 8: Diabetic neuropathy

14. The patient has been unable to return to work. How would you improve this? Diabetic neuropathy has a negative impact on all areas of a patient’s life, including activities of daily living, mood, sleep, self-worth, independence, ability to work, and enjoyment of life. A careful assessment at this time by a psychologist may help differentiate the cause of disability. Pain, motor weakness, low tolerance to activity (including walking), complications related to medications or any interventions, and poor healing foot ulcers may all contribute to disability. At this time, we would utilize all the modalities, including pharmacologic and interventional, physical therapy, and psychotherapy to address the somatic and psychologic derangements common in this disease.

15. Describe the conclusions you would draw from this case? This patient is suffering from PDNP that is possibly associated with motor abnormalities. This confirms both small and large fiber involvement as is seen in the more chronic form of the disease.[11,33] After a complete assessment including detailed history, physical examination, metabolic profile, and panel of neuropathic diagnostic studies, treatment should promptly address pain, physical impairment, and psychosocial issues. Good communication between the patient and provider is essential for both early

References 1.

2.

3.

62

Callaghan BC, Hur J, Feldman EL. Diabetic neuropathy: one disease or two? Curr Opin Neurol. 2012; 25(5):536–541. Callaghan BC, Cheng HT, Stables CL, Smith AL, Feldman EL. Diabetic neuropathy: clinical manifestations and current treatments. Lancet Neurol. 2012;11(6):521–534. Centers for Disease Control and Prevention. National diabetes fact sheet: National estimates and general information on diabetes and prediabetes in the United States.

diagnosis and therapeutic decisions. Efficacy, side effects, and cost of different treatment modalities need to be discussed, as well as realistic expectations. She should be reassured that strict glucose control and compliance with therapy will increase the chance of successfully managing this chronic condition. Painful DPN is a significant clinical problem affecting up to a quarter of all diabetic patients and resulting in loss of quality of life. The minimum requirements for diagnosis of painful DPN are assessment of symptoms and a comprehensive neurologic examination. Other diagnostic tests are helpful in confirming the diagnosis and assessing the severity of nerve damage. Other reversible causes of neuropathy should be considered and ruled out. Current treatment guidelines recommend first-line therapies be considered. These include TCA, the SNRI duloxetine, and anticonvulsants such as pregabalin or gabapentin. Monotherapy should be instituted before combining them. The choice will depend on costs, sleep patterns, and presence of other coexisting diseases. If these are ineffective, medications such as opioids, lidocaine patches, and/or capsaicin patches can be used. Glucose levels should be optimized. There is emerging evidence that several nonpharmacologic therapies are also effective in treating both pain and disability associated with diabetic neuropathy. Invasive therapies such as SCS and IMD can be considered, but we still lack extensive data to support the routine use of these modalities. Finally, there is a promise of newer drugs, including gene therapy, that are currently being developed.

Atlanta, GA: Centers for Disease Control and Prevention. 2011. 4.

Rutkove SB. A 52-year-old woman with disabling peripheral neuropathy: review of diabetic polyneuropathy. JAMA. 2009; 302(13):1451–1458.

5.

Waldman SD. Diabetic neuropathy: diagnosis and treatment for the pain management specialist. Curr Rev Pain. 2000;4(5):383–387.

6.

Horowitz SH. The diagnostic workup of patients with neuropathic pain. Med Clin North Am. 2007;91(1):21–30.

7.

Feng Y, Schlosser FJ, Sumpio BE. The Semmes Weinstein monofilament examination as a screening tool for diabetic peripheral neuropathy. J Vasc Surg. 2009;50(3):675–682.

8.

Williams KA, Hurley RW, Lin EA, Wu CL. Neuropathic pain syndromes. In Benzon H, Rathmell JP, Wu CL, Turk DC, Argoff CE, eds. Raj’s Practical Management of Pain, 4th ed. Philadelphia: Mosby Elsevier. 2013: pp. 427–444.

9.

Bril V. Electrophysiologic testing. In Gries FA, Cameron NE, Low

Chapter 8: Diabetic neuropathy

PA, Ziegler D, eds. Diabetic Neuropathy. New York: Thieme. 2003: pp. 177–184. 10. Lalkhen AG, Bhatia K. Perioperative peripheral nerve injuries. Cont Edu Anaesth Crit Care Pain. 2012;12(1):38–42. 11. Shakher J, Stevens MJ. Update on the management of diabetic polyneuropathies. Diabetes Metab Syndr Obes. 2011;4:289–305. 12. Tesfaye S, Vileikyte L, Rayman G, et al. Painful diabetic peripheral neuropathy: consensus recommendations on diagnosis, assessment and management. Diabetes Metab Res Rev. 2011; Jun 21. 13. Wiffen PJ, Derry S, Moore RA, McQuay HJ. Carbamazepine for acute and chronic pain in adults. Cochrane Database Syst Rev. 2011;(1):CD005451. 14. Freeman R, Durso-Decruz E, Emir B. Efficacy, safety, and tolerability of pregabalin treatment for painful diabetic peripheral neuropathy: findings from seven randomized, controlled trials across a range of doses. Diabetes Care. 2008; 31(7):1448–1454. 15. Saarto T, Wiffen PJ. Antidepressants for neuropathic pain. Cochrane Database Syst Rev. 2007;(4):CD005454. 16. Ziegler D. Painful diabetic neuropathy: advantage of novel drugs over old drugs? Diabetes Care. 2009;32(Suppl 2): S414–S419. 17. Wernicke JF, Pritchett YL, D’Souza DN, et al. A randomized controlled trial of duloxetine in diabetic peripheral neuropathic pain. Neurology. 2006;67(8): 1411–1420. 18. Boyle J, Eriksson ME, Gribble L, et al. Randomized, placebocontrolled comparison of amitriptyline, duloxetine, and pregabalin in patients with chronic diabetic peripheral

neuropathic pain: impact on pain, polysomnographic sleep, daytime functioning, and quality of life. Diabetes Care. 2012;35(12): 2451–2458. 19. Tanenberg RJ, Irving GA, Risser RC, et al. Duloxetine, pregabalin, and duloxetine plus gabapentin for diabetic peripheral neuropathic pain management in patients with inadequate pain response to gabapentin: an openlabel, randomized, noninferiority comparison. Mayo Clin Proc. 2011;86(7):615–626. 20. Martini C, Yassen A, Olofsen E, et al. Pharmacodynamic analysis of the analgesic effect of capsaicin 8% patch (Qutenza) in diabetic neuropathic pain patients: detection of distinct response groups. J Pain Res. 2012;5:51–59. 21. de Vos CC, Rajan V, Steenbergen W, van der Aa HE, Buschman HP. Effect and safety of spinal cord stimulation for treatment of chronic pain caused by diabetic neuropathy. J Diabetes Complications. 2009;23(1):40–45. 22. Dworkin RH, O’Connor AB, Kent J, et al. Interventional management of neuropathic pain: NeuPSIG recommendations. Pain. 2013;154(11):2249–2261. 23. Petrakis IE, Sciacca V. Does autonomic neuropathy influence spinal cord stimulation therapy success in diabetic patients with critical lower limb ischemia? Surg Neurol. 2000;53(2):182–188. 24. Petrakis IE, Sciacca V. Spinal cord stimulation in diabetic lower limb critical ischaemia: transcutaneous oxygen measurement as predictor for treatment success. Eur J Vasc Endovasc Surg. 2000;19(6): 587–592. 25. Tesfaye S, Vileikyte L, Rayman G, et al. Diabetic neuropathies: update on definitions, diagnostic criteria, estimation of severity, and treatments. Diabetes Metab

Res Rev. 2011 Jun 21 [Epub ahead of print]. 26. Mueller MJ, Tuttle LJ, Lemaster JW, et al. Weight-bearing versus nonweight-bearing exercise for persons with diabetes and peripheral neuropathy: a randomized controlled trial. Arch Phys Med Rehabil. 2013; 94(5):829–838. 27. Ahn S, Song R. Effects of Tai Chi Exercise on glucose control, neuropathy scores, balance, and quality of life in patients with type 2 diabetes and neuropathy. J Altern Complement Med. 2012;18(12):1172–1178. 28. De Leon Rodriguez D, Allet L, Golay A, et al. Biofeedback can reduce foot pressure to a safe level and without causing new at-risk zones in patients with diabetes and peripheral neuropathy. Diabetes Metab Res Rev. 2013;29(2):139–144. 29. Simson U, Nawarotzky U, Friese G, et al. Psychotherapy intervention to reduce depressive symptoms in patients with diabetic foot syndrome. Diabet Med. 2008;25(2):206–212. 30. Stein C, Eibel B, Sbruzzi G, Lago PD, Plentz RD. Electrical stimulation and electromagnetic field use in patients with diabetic neuropathy: systematic review and meta-analysis. Braz J Phys Ther. 2013;17(2):93–104. 31. Freeman R. New and developing drugs for the treatment of neuropathic pain in diabetes. Curr Diab Rep. 2013;13(4):500–508. 32. Pluijms W, Huygen F, Cheng J, et al. Evidence-based interventional pain medicine according to clinical diagnoses. 18. Painful diabetic polyneuropathy. Pain Pract. 2011;11(2):191–198. 33. Khalil H. Painful diabetic neuropathy management. Int J Evid Based Healthcare. 2013; 11(1):77–79.

63

Section 1 Chapter

9

Neurological Disorders

Alcohol-induced neuropathy Gulshan Doulatram, Tilak Raj, and Ankur Khosla

Case study A 60-year-old man presents with pain characterized as pins and needles on his soles extending up to his knees. He also complains of weakness, and has had two falls in the last 2 months. The symptoms have been gradually getting worse over the last 6 months. He appears very cachectic and reports a very heavy alcohol use for the last 30 years.

1. What are some of the epidemiologic considerations for this disease? What is the financial burden it imposes on society? Alcohol is one of the most commonly used substances in the world, and the abuse of this toxin closely mirrors in incidence. Consequently, the myriad detrimental effects to the body result in significant morbidity and mortality. From the notable increase in fat deposition, to pains of alcohol-induced gout, alcoholinduced neuropathy (AIN) is the most common. The true incidence of this condition is difficult to ascertain due to the varying definitions employed for AIN in the different studies. Definitions set forth by the Diagnostic and Statistical Manual of Mental Disorders, Fourth Edition (DSM-IV) estimate that neuropathy is present in 25–66% of defined “chronic alcoholics” when using clinical and electrodiagnostic criteria.[1] When evaluating the risk factors for this condition, there appears to be some form of genetic component. Pessione et al demonstrated an increased risk in people with a parental history of alcoholism.[2] AIN is more common in women compared to men.[3] Data investigating this phenomenon suggests that peripheral nerves among females are

at increased sensitivity to the toxic effects of this chemical.[4] Given that the risk of developing alcoholic neuropathy is associated with the duration and extent of total lifetime consumption of alcohol, it is not surprising that it is more prevalent in elderly patients. Most people present with symptoms of the disease between the ages of 40–60. There is a dearth of information on the economic toll imposed upon society with specific regards to alcoholic neuropathy. Hence, the economic constraints of diabetic neuropathy can be used as a surrogate measure. It is reported that in the USA, the total annual cost of DPN and its complications was estimated to be between $4.6 and $13.7 billion, of which up to 27% of the costs of diabetes may be attributed to DPN.[5] It is evident that the burden carried by the individual and by society is of serious consideration during this era of chronic healthcare.

2. What is the pathophysiology of the disease? Alcoholic neuropathy is a primary axonal neuropathy characterized by wallerian degeneration of the axons and secondary demyelination of sensory and small motor fibers. Acetaldehyde, a metabolite of alcohol, has a direct neurotoxic effect by impairing axonal transport. The pathophysiology of alcohol-related nerve damage is complex and multifactorial, and includes: activation of spinal cord microglia, oxidative stress leading to free radical damage to nerves, activation of metabotropic glutamate receptors (mGlu5) in the spinal cord, and activation of the sympathoadrenal and hypothalamo– pituitary–adrenal (HPA) axis.[6] Ethanol promotes oxidative stress by decreasing the concentration of endogenous antioxidants, by generating reactive oxygen species, and increasing lipid peroxidation.

Case Studies in Pain Management, ed. Alan David Kaye and Rinoo V. Shah. Published by Cambridge University Press. © Cambridge University Press 2015.

64

Chapter 9: Alcohol-induced neuropathy

Thiamine, an essential vitamin in the metabolism of pyruvate, has a role in the health of the peripheral nervous system. Thiamine deficiency is commonly found in alcoholic patients due to decreased absorption and hepatic depletion. Other studies have linked the direct toxic effects of alcohol on peripheral nerves with development of neuropathy. A combination of nutritional deficiency and direct toxicity is likely involved in the pathogenesis of alcoholic neuropathy, and these effects may be additive. Thiamine affects the larger motor fibers, compared to alcoholic neuropathy which affects small sensory fibers primarily by causing axonal damage.[7]

3. What are some other clinical conditions that may present in a similar way? Signs and symptoms of alcoholic neuropathy are not specific to this disorder. The range of neuropathic symptoms (including allodynia and paresthesia) are shared among a few other comorbidities. A detailed history and physical exam is important in helping to navigate through the possibilities. A non-exhaustive list includes:  Nutritional causes: Beriberi (thiamine deficiency) Folate deficiency Vitamin B12 deficiency Infectious causes:  Lyme disease Postpolio syndrome HIV-1-associated acute/chronic inflammatory demyelinating polyneuropathy HIV-1-associated distal painful sensorimotor polyneuropathy HIV-1-associated neuromuscular complications HIV-1-associated vacuolar myelopathy Leptomeningeal carcinomatosis Neuropathy of leprosy Tropical myeloneuropathies  Toxic/metabolic causes: Diabetic neuropathy Diabetic lumbosacral plexopathy Hypothyroid Disulfiram-induced polyneuropathy

Organophosphates Chemotherapeutical agents Radiation therapy  Immune system causes: Amyotrophic lateral sclerosis Charcot–Marie–Tooth disease Mononeuritis multiplex Chronic inflammatory demyelinating polyradiculoneuropathy Lambert–Eaton myasthenic syndrome Paraneoplastic autonomic neuropathy Paraneoplastic encephalomyelitis Primary lateral sclerosis Sarcoidosis Syringomyelia Compressive causes:  Femoral mononeuropathy Meralgia paresthetica Peroneal mononeuropathy

4. What are some of the clinical features of alcohol-induced neuropathy? Given that alcohol abuse comes with comorbid conditions such as vitamin deficiencies, the presentation of alcoholic neuropathy can be concurrent with a vitamin B1 deficiency-induced neuropathy. The former is a progressive sensory-dominant symptomatology, and the latter has an inconsistent presentation. In alcohol-induced neuropathy, paresthesias of the feet and toes are commonly reported. With disease progression, the paresthesias migrate in a proximal and symmetric distribution. Consequently, difficulty walking with propensity for falls secondary to decreased afferent neuronal input may be reported. The paresthesias follow a “stocking” distribution, and objectively will demonstrate diminished vibratory/pinprick sensation, as well as thermal/ proprioceptive sensation. In advanced cases, reflex arches can be disrupted. The clinician should examine the patellar, Achilles, and gastrocnemius– soleus muscle complex reflexes. This will help evaluate progression of symptoms. Visual inspection with palpation for muscle body definition/ tone of the lower leg and foot will provide useful information.

65

Chapter 9: Alcohol-induced neuropathy

5. What are other tests including lab tests and imaging that may aid in your diagnostic work-up? How would you make a diagnosis of alcohol neuropathy?  Complete metabolic panel (electrolytes, renal function, and liver function panel – chronic alcohol consumption causes an increase in liver enzymes, e.g., gamma glutamyltransferase, aspartate aminotransferase, alanine aminotransferase).  Thiamine, vitamin B12, and folate levels – these vitamins are essential for the proper functioning of the nervous system and should be checked early in a patient with polyneuropathy. Alcoholics may have nutritional deficiencies which may contribute to the development of neuropathy.[8] Other vitamin levels that need checking include pyridoxine (B6), pantothenic acid and biotin, niacin, and vitamin A.  Hematology screen to check for anemia.  Diabetes testing – plasma glucose and HbA1c – peripheral neuropathy may be the presenting symptom in diabetes; however diabetic polyneuropathy usually occurs in patients who have had diabetes for several years.  Urine analysis and serum creatinine to screen for renal insufficiency.  Thyroid function tests.  Erythrocyte sedimentation rate and C-reactive protein.  Serum protein electrophoresis with immunofixation electrophoresis.  Antinuclear antibody, Anti-SSA and SSB antibodies, rheumatoid factor.  Paraneoplastic antibodies.  Rapid plasma regain.  Esophagogastroduodenoscopy and lower GI series may be considered only if symptoms are present. After the more common diagnoses have been excluded then the following are required:  Screening for heavy metal toxicity, e.g., lead.  Tests for HIV and venereal disease – Distal symmetrical polyneuropathy can be a common and early manifestation of HIV infection.[9] HIV-

66

 







 



infected patients who use illicit substances like alcohol are at a higher risk for developing distal symmetrical polyneuropathy.[10] Imaging studies – There may be radiographic evidence of distal neuropathic arthropathy from long-standing sensory deficits. Nerve conduction studies – Although not specific for alcoholic neuropathy, NCS can help with the diagnosis of neuropathy and quantify the extent. NCS may be normal if only small fibers are involved, although more typically they show a pattern of sensory axon loss with abnormal sural sensory action potentials and conduction velocity as the most sensitive markers.[11] Tibial H-reflex – There may be absent response or symmetrically reduced amplitude or increased latency. This is thought to be the most sensitive test to measure nerve conduction velocity in alcoholic polyneuropathy with some studies quoting rates as high as 60%.[1] T-wave – Like the H-reflex the T-wave response is also a sensitive test for latent alcoholic neuropathy[12] showing a delay in 60% of patients with subclinical alcoholic neuropathy. It is simple and painless to perform. The T-wave is elicited by a tap from the reflex hammer. The smallest latency as well as the maximum reflex amplitude is recorded from eight taps in succession. Needle electromyography (EMG) – A typical neuropathy screen will involve a proximal and a distal muscle in the upper and lower extremity. Significant abnormalities seen in patients with ETOH neuropathy include the presence of positive sharp waves and/or fibrillation potentials indicating denervation. Complex, repetitive discharges also may be observed. Quantitative sensory testing (QST) – This measures sensory thresholds for pain, touch, vibration, and hot and cold temperature sensation. Skin biopsy – Epidermal nerve fiber density and morphology, e.g., tortuosity, complex ramifications, clustering, and axon swellings, can be quantified and compared with control values with a 3-cm skin biopsy and immunohistochemistry. Sural nerve biopsy – Small-fiber-predominant axonal loss is characteristic of alcoholic neuropathy. Myelin irregularities and segmental demyelination due to widening of consecutive

Chapter 9: Alcohol-induced neuropathy

nodes of Ranvier most likely caused by axonal atrophy were conspicuous.[13]  Diagnosis is based on establishing the presence of a slowly progressive sensory-dominant neuropathy and chronic alcohol abuse, and ruling out other causes of neuropathy. Unhealthy drinking is defined as more than three or four drinks per day or more than 10 g of alcohol per day.

6. Briefly describe the therapeutic options available to treat and manage alcohol neuropathy The treatment of AIN starts by preventing any further damage to the nerves. This includes complete abstinence from alcohol by rehabilitation and correction of any coexisting nutritional deficiencies. Although nutritional deficiencies often coexist with alcoholism and contribute toward the overall symptoms and pathophysiology, correcting them alone does not cause significant improvement. Hence, a comprehensive approach aimed toward correction of all associated nutritional deficiencies (especially vitamin B1 and B12) and cessation of alcohol provides the most realistic chances of partial recovery from nerve damage and the associated neuropathy. After achieving these two primary goals, a wide array of neuropathic medications can be utilized to treat the symptoms of alcoholic neuropathy.[6] Unfortunately, there are no randomized controlled trials assessing the effectiveness of the different neuropathic medications in AIN. Hence, the medications described below can be tried, keeping in mind that the evidence for their use has been extrapolated from other known, well-studied neuropathic conditions such as painful diabetic peripheral neuropathy.

Alpha-lipoic acid Alpha-lipoic acid is a nutrient that has been shown to increase glucose uptake, glutathione concentrations, and blood flow in neurons. Though most studies have established the effectiveness of alpha-lipoic acid in the treatment of diabetic neuropathy, this medication has not been studied in alcohol-induced neuropathy.[14]

Acetyl-L-carnitine Acetyl-L-carnitine (ALC) is a molecule derived from acetylation of carnitine in the mitochondria. ALC

supplementation can potentially induce neuroprotective and analgesic effects in the peripheral nervous system. Several studies (including double-blind, placebo-controlled, parallel-group studies and few open studies) have shown an effect of ALC in various neuropathies, such as diabetic neuropathy, HIV and antiretroviral therapy-induced neuropathies, and neuropathies due to compression and chemotherapeutic agents. ALC is known to work even after neuropathic pain has been established.[15] ALC can also improve the function of peripheral nerves by increasing nerve conduction velocity, reducing sensory neuronal loss, and promoting nerve regeneration. ALC regulates processes in energy metabolism, as well as activation of muscarinic cholinergic receptors in the forebrain. Though this drug has never been studied in alcoholic neuropathy, it shows potential primarily due to its mechanisms of actions.

Vitamin E Studies with vitamin E have shown that both alphatocopherol and tocotrienol are effective in rat models of alcohol-induced neuropathy by virtue of their antioxidant properties.[16] Studies in humans, unfortunately, do not exist to confirm this. In another recent study, curcumin, an alkaloid isolated from Curcuma longa, was shown to improve motor nerve conduction velocity and decrease pain in rats.[17] This protective effect of curcumin may be accounted for by a reduction in oxidative stress, inhibition of cytokines, and a decrease in DNA fragmentation.

Anticonvulsants These include the traditional agents, such as carbamazepine and valproate, and newer agents, such as gabapentin and pregabalin. Carbamazepine, phenytoin (Dilantin), and valproate are some of the older anticonvulsants that have been used to treat neuropathic pain.[18] Patients should have detailed laboratory tests, including blood urea nitrogen, creatinine, transaminase, iron levels, a complete blood count (including platelets), reticulocyte count, and liver function test prior to initiation of therapy. Carbamazepine also can cause dermatologic reactions such as toxic epidermal necrolysis and Stevens–Johnson syndrome. In light of the array of adverse effects, newer anticonvulsants are preferred. Gabapentin is used widely in the treatment of neuropathic pain. The major side effects reported from

67

Chapter 9: Alcohol-induced neuropathy

gabapentin include sedation and dizziness. The major drawback is the poor bioavailability of the drug requiring high doses.[19] Pregabalin, an analog of the neurotransmitter GABA, binds the alpha2-delta (a2-d) unit of the calcium channels reducing calcium influx at nerve terminals. This reduces the release of several neurotransmitters, including glutamate, noradrenaline, and substance P. Pregabalin does not bind GABA-A and GABA-B receptors and it is not converted metabolically into GABA.[2] Pregabalin is usually well tolerated, and has a good safety profile. Common side effects include somnolence, dizziness, weight gain, and peripheral edema, which rarely require discontinuation of therapy. Rare but serious adverse events include rhabdomyolysis, acute renal failure, hyperthermia, and secondary acute-angle glaucoma. The dose of pregabalin requires careful titration in patients with chronic kidney disease.

Antidepressants TCA antidepressants, such as amitryptiline and nortryptiline, are effective in treatment of various neuropathies by their central modulation of inhibitory pathways.[20] They are not tolerated well by patients due to their effects on alpha-adrenergic, H1-histamine, muscarinic, cholinergic, and N-methyl-D-aspartate receptors. Some of the adverse effects reported with TCAs include orthostatic hypotension, cardiac arrhythmias, dizziness, and sedation. TCA are contraindicated in the presence of heart failure, arrhythmias, or recent myocardial infarction. Anticholinergic effects of TCAs warrant caution in patients with narrow-angle glaucoma, benign prostatic hypertrophy, orthostatic hypotension, urinary retention, impaired liver function, or thyroid disease. QTc interval should be assessed because of the risk of torsades de pointes. SNRIs, including venlafaxine (Effexor) and duloxetine (Cymbalta) are used in the treatment of neuropathic pain. They are better tolerated and have fewer drug interactions than TCAs. Duloxetine hydrochloride, a dual-reuptake inhibitor of 5-HT and NE (SNRI), is both effective and well tolerated by patients with neuropathic pain. Some of the side effects include somnolence, nausea, dizziness, decreased appetite, and constipation.

Topical agents 5% lidocaine, a sodium channel blocker, is a useful adjunct to antidepressants and anticonvulsants in patients with painful sensory neuropathy.

68

Capsaicin (0.075%) is a topically applied alkaloid that acts peripherally by depleting the neurotransmitter substance P from sensory nerves. The most common adverse effects are stinging and burning related to the brief release of substance P. In a large, multicenter, double-blinded placebo-controlled trial, it was shown that patients who received 0.075% capsaicin had reduced intensity and improved pain relief.[19] However, this study was done in patients with diabetic neuropathy and the results can be extended to alcoholic patients with similar neuropathic symptoms. It can be speculated that the higher concentration 8% capsaicin patch, an agonist at transient receptor potential channels–vanilloid receptors (TRP V1) could also be effective in these patients. Topical clonidine gel has also shown promise in a few patients with minimal side effects. Several compounding creams containing a mixture of different medications including gabapentin, NSAIDs, and clonidine are now available. The effectiveness of these creams has not yet been established.

Opioids Tramadol acts at both the opioid and serotonin/norepinephrine receptors, and is effective in treating pain and improving quality of life and physical functioning, specifically in patients with neuropathic pain.[21] However, tramadol should be used only as a secondline drug after first-line treatments either alone or in combination are found to be ineffective. The side effects of tramadol are related to both its opioid and serotonergic effects. Constipation, respiratory depression, lowered seizure threshold, somnolence, and serotonin syndrome, especially in patients taking concomitant antidepressants, can occur. Opioids, including short-acting and long-acting opioids are used in combination with other neuropathic medications as a last resort if all other medications have failed. Relying solely on opioids in the treatment of AIN carries the risk of tolerance and opioid-induced hyperalgesia. This is especially true in patients who may have a potential for opioid abuse in addition to primary alcohol dependence. Opioid therapy should be implemented very cautiously, titrating doses slowly and instituting objective measures of compliance, including regular urinary drug screening modalities. Methylcobalamin: Patients with clinical vitamin B12 deficiency can be tested for serum levels of metabolites

Chapter 9: Alcohol-induced neuropathy

such as methylmalonic acid and homocysteine. Hypomethylation at the myelin sheath contributes to neuropathy associated with this deficiency.[22] Supplementation with methylcobalamin may improve these clinical symptoms. Further studies are required to see if correction of these changes will improve the clinical course in patients with alcoholic neuropathy. Benfotiamine: S-benzoylthiamine O-monophosphate, a synthetic derivative of thiamine has been shown to reverse neuropathic symptoms and electrophysiologic changes in some small studies.[23] Deficiency of vitamin B6 has been shown to occur commonly in alcoholics due to inadequate dietary intake, and decreased absorption and depletion of thiamine diphosphate (the active coenzyme of thiamine). Myo-inositol: Myo-inositol is an important constituent of the phospholipids that make up nerve cell membranes. Supplementation with myo-inositol completely prevented a reduction in nerve conduction velocity in diabetic rats.[24] Further studies are required with myo-inositol to determine its effectiveness in alleviating symptoms associated with alcoholic neuropathy. N-acetylcysteine: N-acetylcysteine, an amino acid, is a potent antioxidant, and helps to enhance glutathione concentrations.[25] It has been used in animal models of diabetic and cisplatin-induced neuropathy. Further preclinical and clinical studies are required to assess this molecule in alcoholic neuropathy.

Physical therapy Patients with alcoholic neuropathy need a comprehensive physical therapy plan to improve gait and balance, which are frequently affected in these patients. These include strengthening of weakened lower extremity muscles, range of motion (ROM) exercises, and stretching to prevent contracture and maintain normal gait mechanics. An ankle-foot orthosis (AFO) may be needed to assist patients with weak ankle dorsiflexion, eversion, and/or plantar flexion. This device can also help with ankle proprioception and improve gait and prevent ankle sprains. Vigilant foot care and the use of shoes with an enlarged toe box are useful in preventing foot ulcers.

Occupational therapy Occupational therapy is also a key component of the rehabilitation process in patients with alcoholic neuropathy. The occupational therapist can (1) assist

with several aspects to improve function including ADL, with adaptive equipment, and (2) focus on compensatory mechanics to overcome both gait and strength abnormalities.

Psychotherapy Consultation with a psychologist may be indicated to help patients with chronic alcoholism recover from the physical and emotional withdrawal associated with cessation of alcohol consumption. Consultation with a nutritionist may help formulate strategies for replacement of essential nutrients in malnourished alcoholic patients. Referral to a substance abuse support group, such as Alcoholics Anonymous (AA), may help patients cope with alcohol cessation. Complete cessation of alcohol consumption is necessary to improve or reverse the symptoms associated with alcoholic neuropathy.

6. Describe any treatment-related complications The complications related to treatment of alcoholic neuropathy are primarily from the medications commonly used for alcoholism or neuropathic pain. One specific mention is disulfiram associated neuropathy. In fact, 1 in 15 000 patients taking disulfiram develops peripheral neuropathy every year due to disulfiram toxicity. These patients are often misdiagnosed as having alcoholic neuropathy and pose a challenge to patients attempting to abstain from alcohol.

7. Patient refuses to stop drinking and using marijuana. Does that impose any ethical issues? The presence of ongoing neuropathic pain in a patient with alcoholic neuropathy who refuses to stop drinking poses a unique dilemma to the treating physician. The physician will need to counsel the patient about the effects of alcohol on the nervous system, while offering a comprehensive treatment plan. The patient should also be provided complete support from a psychologist, social services, and rehabilitation services. Potential conflicts with opioid therapy may arise in patients with continued alcohol or drug use. A decision to withdraw opioid therapy must be made at an individual level, and after extensive counseling has been offered to the patient. The patient may still, however, require extensive

69

Chapter 9: Alcohol-induced neuropathy

and sustained psychologic support to avoid relapses. Close communication with the primary care physician and regular follow-up care for the patient is essential to monitor the effectiveness of therapy.

8. What conclusions could you draw from this case? Alcohol neuropathy is associated with increased morbidity and a definite decrease in the quality of life. These patients are often not diagnosed early, and usually present to pain consultants after the involvement of sensory and long motor fibers. The disease is commonly associated with several nutritional deficiencies (especially thiamine and folate) which worsen the neuropathy. Initially, neuropathy from alcohol was thought to be only due to thiamine deficiency. More recent studies have elucidated the direct toxic

References 1.

2.

3.

4.

5.

70

Laker SR. Alcoholic neuropathy. Medscape com 2013 June 1. Available from: URL: http:// emedicine.medscape.com/article/ 315159-overview (cited Aug 23, 2013).

6.

7.

effect of alcohol and its metabolites on nerves. The treatment involves abstinence of alcohol to halt the progressive nature of nerve damage, correction of nutritional deficiencies, and an array of medications. Consultation with a nutritionist may help formulate strategies for replacement of essential nutrients in malnourished alcoholic patients. None of the medications have been studied extensively in humans. Of the medications, benfotiamine, alpha-lipoic acid, ALC, and methylcobalamin are among the well-researched options for the treatment of peripheral neuropathy. Other potential nutrient or botanical therapies include vitamin E, myo-inositol, N-acetylcysteine, and topical capsaicin. Understanding the basic pathophysiologic mechanisms involved in alcohol-induced neuropathic pain will pave the way in the development of new therapeutic modalities which can target disrupted cellular signaling machinery.

Chopra K, Tiwari V. Alcoholic neuropathy: possible mechanisms and future treatment possibilities. Br J Clin Pharmacol. 2012; 73(3):348–362. Koike H, Iijima M, Sugiura M, et al. Alcoholic neuropathy is clinicopathologically distinct from thiamine-deficiency neuropathy. Ann Neurol. 2003;54(1):19–29.

subjects. J Peripher Nerv Syst. 2005;10(4):375–381. 12. Schott K, Schafer G, Gunthner A, Bartels M, Mann K. T-wave response: a sensitive test for latent alcoholic polyneuropathy. Addict Biol. 2002;7(3):315–319. 13. Koike H, Sobue G. Alcoholic neuropathy. Curr Opin Neurol. 2006;19(5):481–486.

Pessione F, Gerchstein JL, Rueff B. Parental history of alcoholism: a risk factor for alcohol-related peripheral neuropathies. Alcohol Alcohol. 1995;30(6):749–754.

8.

Monforte R, Estruch R, Valls-Sole J, et al. Autonomic and peripheral neuropathies in patients with chronic alcoholism. A dose-related toxic effect of alcohol. Arch Neurol. 1995;52(1):45–51.

Peters TJ, Kotowicz J, Nyka W, et al. Treatment of alcoholic polyneuropathy with vitamin B complex: a randomised controlled trial. Alcohol Alcohol. 2006;41(6):636–642.

14. Kishi Y, Schmelzer JD, Yao JK, et al. Alpha-lipoic acid: effect on glucose uptake, sorbitol pathway, and energy metabolism in experimental diabetic neuropathy. Diabetes. 1999;48(10):2045–2051.

9.

Fama R, Eisen JC, Rosenbloom MJ, et al. Upper and lower limb motor impairments in alcoholism, HIV infection, and their comorbidity. Alcohol Clin Exp Res. 2007;31(6): 1038–1044.

15. Pisano C, Pratesi G, Laccabue D, et al. Paclitaxel and Cisplatininduced neurotoxicity: a protective role of acetyl-Lcarnitine. Clin Cancer Res. 2003; 9(V):5756–5767.

Ammendola A, Gemini D, Iannaccone S, et al. Gender and peripheral neuropathy in chronic alcoholism: a clinicalelectroneurographic study. Alcohol Alcohol. 2000;35 (4):368–371. Gordois A, Scuffham P, Shearer A, Oglesby A, Tobian JA. The health care costs of diabetic peripheral neuropathy in the US. Diabetes Care. 2003;26(6): 1790–1795.

10. Robinson-Papp J, Gelman BB, Grant I, et al. Substance abuse increases the risk of neuropathy in an HIV-infected cohort. Muscle Nerve. 2012;45(4):471–476. 11. Zambelis T, Karandreas N, Tzavellas E, Kokotis P, Liappas J. Large and small fiber neuropathy in chronic alcohol-dependent

16. Tutuncu NB, Bayraktar M, Varli K. Reversal of defective nerve conduction with vitamin E supplementation in type 2 diabetes: a preliminary study. Diabetes Care. 1998;21(11): 1915–1918. 17. Kandhare AD, Raygude KS, Ghosh P, Ghule AE, Bodhankar SL. Therapeutic role of curcumin

Chapter 9: Alcohol-induced neuropathy

in prevention of biochemical and behavioral aberration induced by alcoholic neuropathy in laboratory animals. Neurosci Lett. 2012;511(1):18–22. 18. Finnerup NB, Otto M, McQuay HJ, Jensen TS, Sindrup SH. Algorithm for neuropathic pain treatment: an evidence based proposal. Pain. 2005;118(3):289–305. 19. Rains C, Bryson HM. Topical capsaicin. A review of its pharmacological properties and therapeutic potential in postherpetic neuralgia, diabetic neuropathy and osteoarthritis. Drugs Aging. 1995;7(4):317–328. 20. Sandercock D, Cramer M, Wu J, et al. Gabapentin extended release for the treatment of painful

diabetic peripheral neuropathy: efficacy and tolerability in a double-blind, randomized, controlled clinical trial. Diabetes Care. 2009;32(2):e20. 21. Callaghan BC, Cheng HT, Stables CL, Smith AL, Feldman EL. Diabetic neuropathy: clinical manifestations and current treatments. Lancet Neurol. 2012;11(6):521–534. 22. Saperstein DS, Barohn RJ. Peripheral neuropathy due to cobalamin deficiency. Curr Treat Options Neurol. 2002; 4(3):197–201. 23. Woelk H, Lehrl S, Bitsch R, Kopcke W. Benfotiamine in treatment of alcoholic polyneuropathy: an 8-week

randomized controlled study (BAP I Study). Alcohol Alcohol. 1998;33(6):631–638. 24. Sundkvist G, Dahlin LB, Nilsson H, et al. Sorbitol and myo-inositol levels and morphology of sural nerve in relation to peripheral nerve function and clinical neuropathy in men with diabetic, impaired, and normal glucose tolerance. Diabet Med. 2000; 17(4):259–268. 25. Love A, Cotter MA, Cameron NE. Effects of the sulphydryl donor N-acetyl-L-cysteine on nerve conduction, perfusion, maturation and regeneration following freeze damage in diabetic rats. Eur J Clin Invest. 1996;26(8):698–706.

71

Section 1 Chapter

10

Neurological Disorders

HIV neuropathy Gulshan Doulatram, Tilak Raj, and William Yancey

Case study A 44-year-old male presents with burning in both legs for the past 8 months which has become severe in the last 2 months. He has been diagnosed with HIV for the past 3 years. His current CD4+ cell count is 250 cells/µl and viral load 50 copies/ml. He was started on highly active antiretroviral therapy (HAART) about 3 months ago. He smokes marijuana and insists that it helps with his pain.

1. How prevalent is this disease presentation?Couldyouexplainsomeof the epidemiologic features of this disease? Are there any cost concerns? In its 2012 HIV Surveillance Supplemental Report, the Centers for Disease Control estimates that over 1.1 million persons in the USA are infected with HIV, and predicts an additional 50000 new infections each year.[1,2] The global prevalence of HIV is 33 million. These individuals are at risk for multiple neurologic complications due to their disease. The most common of these complications is distal symmetric polyneuropathy (DSP).[3,4] Its high prevalence ranks it as one of the more frequently encountered etiologies of neuropathic pain in the general population. As such, DSP is a substantial contributor to the healthcare cost of neuropathic pain as a whole. Berger et al examined this and demonstrated an average yearly healthcare cost of $17355 for a patient with a painful neuropathic disorder versus $5715 for a matched control patient.[5] The prevalence of DSP among HIV-infected individuals has been reported to be between 34 and 63%.[6–8] Data is limited for pediatric populations, but DSP appears to be similarly common in children with HIV with a prevalence of 34%.[9] Differences

among these estimates can be attributed in part to the evolution of HIV into a chronic disease with the advent of effective antiretroviral therapy (ART).[10] ART has resulted in a decrease in the incidence of DSP as well as a shift in its associated risk factors.[7,10,11] Prior to effective ART, the risk of DSP had been linked to decreased CD4 lymphocyte counts and increased HIV viral loads.[12,13] More recent studies have not confirmed these risk factors[7,11] and have, instead, identified factors such as substance abuse,[14] diabetes, nutritional deficiencies,[15] and aging.[11]

2. What is the differential diagnosis of HIV neuropathy? The patient with HIV is susceptible to multiple painful neurologic complications, including those most commonly associated with HIV infection and those of the general population. A complete differential should include both categories and allow for the coexistence of two or more diagnoses to explain a patient’s pain. HIV neuropathy is often difficult to differentiate from antiretroviral neuropathy.[16,17] Differentiating the two requires identification of a potentially offending agent, such as a dideoxynucleoside, among the patient’s medications and determining whether onset of therapy with this drug was associated with the onset of neuropathy (usually 1 week to 6 months later).[16] Additional painful neuropathies associated with HIV infection include mononeuropathy multiplex (MM) and progressive polyradiculopathy (PP). These conditions may be most easily differentiated from DSP based on their clinical presentations. For example, in contrast to the classic stocking-glove distribution and primarily sensory effects of DSP, MM presents with asymmetric, multifocal findings that may involve both peripheral and cranial nerves. Further,

Case Studies in Pain Management, ed. Alan David Kaye and Rinoo V. Shah. Published by Cambridge University Press. © Cambridge University Press 2015.

72

Chapter 10: HIV neuropathy

MM is more likely to result in motor deficits.[17] PP can similarly be discerned by its motor effects, which may include a rapidly progressive flaccid paraparesis.[18] Additionally, PP is associated with lumbosacral pain and radicular symptoms.[16] Lastly, a complete differential should include diagnoses not necessarily related to HIV infection. Depending on clinical suspicion, these could include the following:  Nutritional causes: Beriberi (thiamine deficiency) Folate deficiency Vitamin B12 deficiency  Infectious causes: Lyme disease Postpolio syndrome Leptomeningeal carcinomatosis Neuropathy of leprosy Tropical myeloneuropathies Syphilis  Toxic/metabolic: Diabetic neuropathy Diabetic lumbosacral plexopathy Hypothyroid Disulfiram-induced polyneuropathy Organophosphates Chemotherapeutical agents Radiation therapy Alcohol Uremic neuropathy  Immune: Amyotrophic lateral sclerosis Charcot–Marie–Tooth disease Mononeuritis multiplex Chronic inflammatory demyelinating polyradiculoneuropathy Lambert–Eaton myasthenic syndrome Paraneoplastic autonomic neuropathy Paraneoplastic encephalomyelitis Primary lateral sclerosis Sarcoidosis Syringomyelia  Compressive: Femoral mononeuropathy Meralgia paresthetica Peroneal mononeuropathy

Most of these diagnoses can be evaluated by simple laboratory studies.

3. Describe the clinical presentations of HIV neuropathy Distal symmetric polyneuropathy (DSP) This is the most common form of neuropathy and usually affects the lower extremities in the toes and soles similar to diabetic neuropathy. The upper extremities usually are involved much later. Sensory symptoms are the hallmarks of DSP.[17] Significant muscle weakness and loss of proprioception are usually not seen and should prompt an evaluation for other causes of neuropathy or a different presentation. Visual analog scale (VAS) and Gracely (a 20 point scale with verbal descriptors of pain) pain scales are commonly used in DSP. Some of the signs and symptoms include:  Paresthesia  Dysesthesia  Numbness  Diminished ankle reflexes  Reduction of pinprick  Reduction of temperature  Antalgic gait  Loss of vibration

ARV-associated neuropathy This painful neuropathy is triggered by the antiretroviral therapy including HAART. Thirty-six percent of HIV patients treated with HAART will develop this neuropathy.[17,20] Risk factors include age, severe immunosuppression, and combined use of dideoxynucleoside analogs stavudine (D4T), zalcitabine (ddC), and didanosine (ddI). These drugs inhibit mitochondrial DNA synthesis which leads to mitochondrial dysfunction and reduced energy availability. There is predominantly small-fiber involvement with prominent pain and paresthesias very similar to DSP.

Progressive polyradiculopathy PP involving the lumbar nerve roots is fortunately rare and only seen in the very advanced forms of the disease. Opportunistic infection of the lumbar nerve roots is usually caused by cytomegalovirus (CMV), varicella-zoster, herpes simplex, syphilis, tuberculosis, and lymphoma. Clinically, patients present with

73

Chapter 10: HIV neuropathy

radicular pain, profound weakness, and bladder and bowel incontinence. Signs include flaccid paralysis and absent reflexes and sensation.[16] Treatment includes early detection and aggressive treatment of the infectious causes, which in most cases is CMV related. This can potentially prevent advanced and permanent motor and sensory sequelae.

Mononeuropathy multiplex Mononeuropathy multiplex involves cranial and peripheral nerves and is associated with pain and paresthesias in the affected dermatomes with motor deficits.[17] Steroids, plasmapheresis and intravenous immunoglobulin can be used in severe cases.

4. How would you make a diagnosis of HIV neuropathy? Apart from a history of HIV infection, clinical evaluation, and laboratory testing to rule out other potential causes for neuropathy, there are a few screening measures which can diagnose and quantify the severity of neuropathy.[17]

Brief peripheral neuropathy screen A brief peripheral neuropathy screen (BPNS) is a simple way to assess neuropathic pain symptoms in HIV patients and has the advantage that it relies only on the history and physical exam. The test involves three questions in the history and two examination measures. BPNS has a high specificity and low sensitivity and has the advantage that it can be used clinically and does not rely on other tests. The signs and symptoms include: 1. Pain, aching, or burning in the feet 2. Pins and needles in feet/legs 3. Numbness in legs 4. Lower extremity vibration 5. Lower extremity reflexes

Total neuropathy score Total neuropathy score (TNS) is based on clinical tests and functional nerve studies. These include sensory and motor symptoms, sensation to pin and vibration, tendon reflexes, motor function, QST, and NCS. All these components are summed up to give a TNS which can highlight the severity of the disease and assess response to various treatment regimes.

74

5. Describesomeofthelaboratorytests that would aid in your diagnosis Quantitative sensory testing QST measures sensory thresholds for pain, touch, vibration, and hot and cold temperature sensation. It is increasingly used, especially in clinical therapeutic trials. A number of devices are commercially available and range from handheld tools to sophisticated computerized equipment with complicated testing algorithms. Specific fiber functions can be assessed: Aδ fibers with cold and cold-pain detection thresholds, C-fibers with heat and heat-pain detection thresholds, and large fiber (Aαβ-) functions with vibration detection thresholds. Elevated sensory thresholds correlate with sensory loss and lowered thresholds occur in allodynia and hyperalgesia. In asymptomatic patients, abnormal QST thresholds suggest subclinical nerve damage. QST is a psychophysical test and therefore is dependent upon patient motivation, alertness, and concentration.

Nerve conduction studies Sural nerve amplitude is absent or reduced in DSP with occasional involvement of median and ulnar nerves. In MM, there is a dramatic decrease in compound motor and sensory action potentials.

EMG EMG shows signs of chronic denervation with reinnervation in distal muscles of lower extremities. In a patient with PP, EMG findings include severe and widespread axonal pathology of the lumbar nerve roots.

Nerve biopsy Sural nerve biopsy shows degeneration of myelinated and unmyelinated axons of the sural nerve with associated moderate inflammatory infiltrates. These changes are more marked in MM with detection of CMV inclusions in peripheral nerves.

Skin biopsy Decrease in distal epidermal nerve fiber density (ENFD) is associated with worsening of distal sensory polyneuropathy and is a very sensitive test to detect early disease.

Chapter 10: HIV neuropathy

Measures of small-fiber neuropathy Most of the tests mentioned above measure large fiber involvement. ART therapy for HIV is usually associated with small-fiber neuropathy and hence may go undetected with the standard tests.[18] Utah Early Neuropathy Scale (UENS) and QSART can be used in this subset of population to detect neuropathy in the early stages of therapy.

Utah Early Neuropathy Scale The Utah Early Neuropathy Scale (UENS) has a sensitivity of 92% for early sensory loss. UENS emphasizes severity and spatial distribution of pin (sharp) sensation loss in the foot and leg and focuses less on motor weakness.

Quantitative Sudomotor Axon Reflex Test QSART is sensitive (80% sensitivity) and non-invasive and has been studied in other well-known neuropathies including diabetes. It is generally shown to correlate very well with fiber loss as measured by intraepidermal nerve fiber density (IENFD). The test assesses autonomic C-fibers by measuring sweat volume, and is a measure of autonomic mediated cutaneous sweat production in response to iontophoresed acetylcholine.

Autonomic function testing Autonomic testing is valuable in patients with neuropathic pain disorder who have normal or mildly abnormal electrophysiologic (NCV/EMG) findings. The most useful tests are the QSART, thermoregulatory sweat test, heart rate responses to deep breathing, Valsalva ratio, and surface skin temperature. It has been clearly shown that DSP is closely associated with autonomic neuropathy in HIV patients.[19]

6. Are there any imaging tests that would help in the diagnosis? Imaging modalities are rarely required to diagnose distal sensory neuropathy. However, MRI with contrast may be useful in PP because it typically shows enhancement of spinal nerve roots.

7. What is the pathophysiology of this disease? The envelope glycoprotein of HIV, gp-120, is primarily responsible for the development of DSP.[20] The

interaction between gp120 and chemokine receptors CXCR4 and CXCR5 on the spinal cord microglia promotes DSP in HIV patients. This interaction is further potentiated by nitric oxide (NO) by causing the release of proinflammatory cytokines, such as tumor necrosis factor (TNF) and interleukins. Nitric oxide inhibitors and TNF and interleukin antagonists have been shown to reduce allodynia in animal models. Proinflammatory cytokines such as TNF, IL-1, and IL-6 released by the microglia and dorsal root ganglia are nociceptor mediators and play an important role in the development and maintenance of HIV-associated neuropathy. Mitochondrial dysfunction and energy failure in the distal axon are also thought to be responsible in the pathophysiology of DSP.

8. What treatment modalities exist currently for this condition? As with other neuropathies, once the diagnosis is established, treatment should promptly begin utilizing a stepwise approach. Patients with HIV-related neuropathy are usually undertreated. This is due to inadequate evidence for the effectiveness of therapeutic options and resistance on the part of physicians due to fear of addiction in patients with concomitant substance abuse. There are no FDA-approved medications for the use of HIV-associated neuropathy. The main goals of therapy include effective analgesia and improvement in the quality of life. Mild analgesics such as acetaminophen, aspirin, and other non-steroidal agents could be effective initially.[20] Anticonvulsants, antidepressants, lidocaine patch, and opioids have also been used. If DSP is related to antiretroviral therapy, then a careful consideration needs to be made to stop therapy if ineffective or to continue therapy with concomitant analgesics. In addition to pharmacologic options, non-pharmacologic remedies such as biofeedback, meditation, acupuncture, and physical therapy have been shown to be equally effective in the treatment of distal sensory polyneuropathy.

Pharmacologic treatment Anticonvulsants Most of the data that support the use of anticonvulsants stem from studies in other well-known neuropathic pain states. However, according to existing guidelines in the treatment of neuropathic pain, all first-line agents, including anticonvulsants, have been found to be

75

Chapter 10: HIV neuropathy

ineffective in DSP.[20] Some of the anticonvulsants commonly used in other neuropathic pain states include gabapentin, carbamazepine, lamotrigine, phenytoin, topiramate, and clonazepam. Several multicenter double-blinded randomized trials showed no improvement in the VAS scores despite using high doses of gabapentin and pregabalin. There is also some evidence for lamotrigine in the treatment of DSP. Apart from a modest effect in HAART-associated neuropathy, lamotrigine was also found to be ineffective and hence does not have a significant place in the analgesic therapy.

Antidepressants Two trials that have looked at the efficacy of amitryptiline and mexilitine in HIV-associated neuropathy have shown no benefits compared to placebo treatment. In the absence of evidence, and in light of anticholinergic side effects such as arrhythmias, orthostatic hypotension, dry mouth, and sedation, it may be prudent not to consider this class of medications as a first choice in the treatment of DSP.

Topical agents Topical preparations are useful adjuncts to oral agents since they are generally well tolerated and safe. 5% lidocaine and 0.75% clonidine can be used in patients exhibiting classic neuropathic symptoms. Four trials have examined the role of 0.075% capsaicin and found it to be ineffective.[21] The 8% capsaicin patch, however, has shown promise in patients with distal sensory neuropathy.[22] Capsaicin binds to vanilloid receptors causing depolarization of C-fiber nociceptors, increased permeability to sodium and calcium ions, and release of Substance P. This explains the transient burning seen with the application of capsaicin. Subsequently, depletion of Substance P contributes toward its analgesic effect with eventual loss of C-fiber neurons causing degeneration of epidermal nerve fibers. This effect is controversial in HIV neuropathy where there is already loss of epidermal nerve fibers.

Opioids In 2007, recommendations from the Neuropathic Pain Special Interest Group moved opioids to second-line therapy in the treatment of neuropathic pain.[23] Exceptions to this rule include acute severe pain, neuropathic cancer pain, and pain not controlled with usual first-line medications. Opioids are not routinely recommended for long-term use because their long-term safety has not been established. Other reasons include

76

potential for immunosuppression, hypogonadism, opioid-induced hyperalgesia, and addiction. Longacting opioids are preferred due to consistent plasma levels. Potential interactions of opioids and other HIV medications need to be considered. Antiretrovirals such as ritonavir increase plasma concentrations of oxycodone. Compliance to therapy should be monitored very carefully, especially if there are other risk factors such as prior substance abuse. More recent evidence has shown that opioid therapy may actually be nociceptive by upregulating specific chemokine receptors that promote pain in HIV patients. μ-Opioids can increase toxicity either by their direct action on neurons or by potentiating HIV replication in infected cells of the CNS, which, in turn, increases the production of neurotoxic proteins, cytokines, glutamate, reactive oxygen species, and nitric oxide.

Smoked cannabis Smoked cannabis has been shown to be statistically superior to placebo in two separate studies by causing a greater than 30% improvement in VAS scores.[24] Cannabis exerts its effects by binding to cannabinoid receptors in the central and peripheral nervous system, which interact with noradrenergic and kappa-opioid receptors to promote attenuation of pain.

Antiretroviral therapy The effects of antiretroviral therapy on DSP are usually positive, with studies demonstrating that patients improved both symptomatically as well as scored well in quantitative thermal testing. However, in one study of a small group of patients, the neurotoxic dideoxynucleoside may have worsened or caused neuropathic pain.[25] The diagnosis is usually made by decreasing or stopping the medication for 4 weeks to see if symptoms improve. Often, the therapy can be switched to non-toxic antiretroviral agents, but, if that is not possible, then a consideration should be made to continue therapy and manage symptoms of neuropathy with other medications. One must keep in mind that DSP is clinically indistinguishable from ART associated neuropathy, except for the temporal association. Hence, stopping therapy and worsening the outcome of HIV or continuing therapy and managing the side effects is often a therapeutic dilemma for the patient and practitioner.

Nerve growth factor One study has looked at recombinant human nerve growth factor (rhNGF) in the treatment of

Chapter 10: HIV neuropathy

HIV-associated neuropathy.[24] Patients who were given two doses for 18 weeks demonstrated improvement in the Gracely Pain Scores, but there was no evidence of nerve regeneration.

Prosapeptide Prosapeptide is a peptide known to reverse nerve regeneration and can improve allodynia and hyperalgesia in a rat model. A very small study showed promising results in humans, and the drug is currently being evaluated on a larger scale.

What conclusions can be made for the treatment of HIV-associated neuropathy? The evidence for the treatment of HIV-associated neuropathy is not conclusive. In fact, most studies showed a negative effect for gabapentin, pregabalin, amitryptiline, mexilitine, peptide-T, acetyl-carnitine, and lamotrigine. Evidence for efficacy currently exists only for rhNGF (which is clinically unavailable) and smoked cannabis, which cannot be recommended for routine therapy.[24]

Complementary and alternative medicine CAM has been used and studied well in HIV neuropathy, and may improve quality of life for those living with HIV/AIDS.[26] Some of these include:  Acupuncture: Acupuncture stimulates the release of endogenous opioids, thereby altering pain perception. Acupuncture has been used to treat HIV-related symptoms such as peripheral neuropathy, diarrhea, nausea, vomiting, insomnia, and muscle pains.[26] It has shown a modest decrease in symptoms of DSP, especially in ARTinduced neuropathy. It also demonstrated an improvement in sleep quality, an indirect measure of quality of life. However, the studies with acupuncture are small, hence some large-scale studies are needed to see if these positive effects can be replicated.  Massage therapy: HIV patients use manual healing techniques including massage therapy, Shiatsu, Reiki, therapeutic touch, acupressure, and chiropractic manipulation. Touch can increase blood flow, decrease pain, cause relaxation, and stimulate the immune system and release of endorphins. Massage therapy has

been shown to decrease stress and anxiety in HIV patients.  Nutritional support: Patients with HIV are often nutritionally compromised and would benefit from vitamin and nutritional supplementation. No studies exist to support or disprove this theory. A small study using L-glutamine antioxidant showed weight gain in these patients.  Herbal remedies: Marijuana plant-based therapy has been shown to be effective in HIV neuropathy. Cannabinoid-based drugs, such as dronabinol (Marinol), have been reported to stimulate appetite, weight gain, and provide relief from nausea, especially in anorexic HIV patients.[26] Alternatively, other plant-based therapies, such as St John’s Wort and garlic, interact with antiretroviral medications and can cause resistance to treatment. The psychologic benefits of various CAM modalities should not be underestimated. There is evidence suggesting that decreasing depression can decrease HIV-related somatic complaints. Currently, many patients do not self-report the use of CAM, and physicians are unlikely to recommend these modalities. This will likely change as CAM becomes more widely recognized as a legitimate medical intervention. Larger studies are needed to accurately assess the safety of such interventions.

Psychotherapy/cognitive behavioral therapy Cognitive behavior intervention can reduce pain and suffering in some patients with HIV neuropathic pain.[27] Cognitive behavioral therapy (CBT) was compared to supportive psychotherapy in reducing pain, pain-related interference with functioning, and distress. The cognitive behavior group improved in most domains of functional activity and distress compared to the supportive psychotherapy group. The high dropout rate suggests that psychotherapeutic treatments for HIV-related pain, though important, may not be practically feasible.

Interventional techniques Currently, there are no studies that have looked at spinal cord stimulation, deep brain stimulation, and implantable medication devices in HIV neuropathy. A careful assessment should be done before any of these modalities are considered. Rationale behind the use of these

77

Chapter 10: HIV neuropathy

interventions has to be made from their use in other known neuropathic conditions, such as diabetic neuropathy and complex regional pain syndrome.

9. Are there any special concerns that you might have in the care of patients with HIV neuropathy? Patients with HIV neuropathy often have other sequelae of the disease, including AIDS. These patients often have poor support systems and nutritional status, which further compound the pain associated with neuropathy. Patients may not be compliant with therapy and have other concomitant issues with illicit drugs and alcohol. Smoked cannabis, which can cause cognitive and motor dysfunction, has been shown to be effective in the treatment of DSP. This causes a therapeutic dilemma for the pain physician, especially if the patient is also using chronic opioids.

10. What conclusions would you draw from this case? HIV neuropathy is commonly seen in patients with HIV, and causes a distal involvement of sensory fibers

References 1.

2.

3.

4.

78

Centers for Disease Control and Prevention. Monitoring selected national HIV prevention and care objectives by using HIV surveillance data – United States and 6 U.S. dependent areas – 2010. HIV Surveillance Supplemental Report. 2012;17(Number 3 Part A). Centers for Disease Control and Prevention. Estimated HIV incidence among adults and adolescents in the United States, 2007–2010. HIV Surveillance Supplemental Report. 2012;17 (No. 4).

especially of the lower extremity. Its clinical presentation is often very similar to other causes of neuropathy. Diagnosis can be confirmed by a battery of tests to look for small sensory and autonomic fiber abnormalities. These tests can be pivotal in the early diagnosis of HIV neuropathy. The treatment of neuropathic pain usually relies on known modalities such as antidepressants and anticonvulsants initially. However, these modalities have not been shown to work in HIV neuropathy. Opioid therapy is also very controversial in this subset of patients but may become necessary in advanced disease. Antiretroviral therapy must be continued unless there is a suspicion of HAART-induced neuropathy.[23,28] There are currently an estimated 33 million people living with HIV, and this number is expected to rise with the use of highly effective antiretroviral therapy. Furthermore, treatment of this chronic condition does not follow the algorithmic approach used to treat other neuropathic conditions. Of all therapies, only recombinant nerve growth factor, smoked cannabis, and high-concentration capsaicin patch have shown positive results. Evidence-based treatment approaches will be required in the future to effectively treat this chronic disease. neuropathy associated with acquired immunodeficiency syndrome: Prevalence and clinical features from a population-based survey. Arch Neurol. 1988;45(9): 945–948.

virus-associated distal sensory polyneuropathy: still common after many successes. Arch Neurol. 2010;67(5):534–535. 5.

6.

Bacellar H, Munoz A, Miller EN, et al. Temporal trends in the incidence of HIV-1-related neurologic diseases: Multicenter AIDS Cohort Study, 1985–1992. Neurology. 1994;44(10):1892–1900.

7.

Kolson DL, Gonzalez-Scarano F. Human immunodeficiency

8.

Berger A, Dukes EM, Oster G. Clinical characteristics and economic costs of patients with painful neuropathic disorders. J Pain. 2004;5(3):143–149. Hall CD, Snyder CR, Messenheimer JA, et al. Peripheral neuropathy in a cohort of human immunodeficiency virus-infected patients: Incidence and relationship to other nervous system dysfunction. Arch Neurol. 1991;48(12):1273–1274. Simpson DM, Kitch D, Evans SR, et al. HIV neuropathy natural history cohort study: assessment measures and risk factors. Neurology. 2006;66(11): 1679–1687. So YT, Holtzman DM, Abrams DI, Olney RK. Peripheral

9.

Araujo AP, Nascimento OJ, Garcia OS. Distal sensory polyneuropathy in a cohort of HIV-infected children over five years of age. Pediatrics. 2000; 106(3):E35.

10. Lichtenstein KA, Armon C, Baron A, et al. Modification of the incidence of drugassociated symmetrical peripheral neuropathy by host and disease factors in the HIV outpatient study cohort. Clin Infect Dis. 2005;40(1): 148–157. 11. Evans SR, Ellis RJ, Chen H, et al. Peripheral neuropathy in HIV: prevalence and risk factors. AIDS 2011;25(7):919–928.

Chapter 10: HIV neuropathy

12. Childs EA, Lyles RH, Selnes OA, et al. Plasma viral load and CD4 lymphocytes predict HIV associated dementia and sensory neuropathy. Neurology. 1999; 52(3):607–613. 13. Tagliati M, Grinnell J, Godbold J, Simpson DM. Peripheral nerve function in HIV infection: clinical, electrophysiologic, and laboratory findings. Arch Neurol. 1999;56(1): 84–89. 14. Robinson-Papp J, Gelman BB, Grant I, et al. Substance abuse increases the risk of neuropathy in an HIV-infected cohort. Muscle Nerve. 2012;45(4):471–476. 15. Kieburtz KD, Giang DW, Schiffer RB, Vakil N. Abnormal vitamin B12 metabolism in human immunodeficiency virus infection: Association with neurological dysfunction. Arch Neurol. 1991; 48(3):312–314. 16. Keswani SC, Pardo CA, Cherry CL, Hoke A, McArthur JC. HIV-associated sensory neuropathies. AIDS. 2002; 16(16):2105–2117. 17. Verma S, Estanislao L, Simpson D. HIV-associated neuropathic pain: epidemiology, pathophysiology and

management. CNS Drugs. 2005; 19(4):325–334. 18. Boger MS, Hulgan T, Haas DW, et al. Measures of small-fiber neuropathy in HIV infection. Auton Neurosci. 2012;169(1): 56–61. 19. Robinson-Papp J, Sharma S, Simpson DM, Morgello S. Autonomic dysfunction is common in HIV and associated with distal symmetric polyneuropathy. J Neurovirol. 2013;19(2):172–180. 20. Smith HS. Treatment considerations in painful HIVrelated neuropathy. Pain Physician. 2011;14(6):E505–E524. 21. Paice JA, Ferrans CE, Lashley FR, et al. Topical capsaicin in the management of HIV-associated peripheral neuropathy. J Pain Symptom Manage. 2000;19(1): 45–52. 22. Simpson DM, Estanislao L, Brown SJ, Sampson J. An open-label pilot study of high-concentration capsaicin patch in painful HIV neuropathy. J Pain Symptom Manage. 2008;35(3):299–306. 23. Dorsey SG, Morton PG. HIV peripheral neuropathy:

pathophysiology and clinical implications. AACN Clin Issues. 2006;17(1):30–36. 24. Phillips TJ, Cherry CL, Cox S, Marshall SJ, Rice AS. Pharmacological treatment of painful HIV-associated sensory neuropathy: a systematic review and meta-analysis of randomised controlled trials. PLoS One. 2010; 5(12):e14433. 25. Capers KN, Turnacioglu S, Leshner RT, Crawford JR. Antiretroviral therapy-associated acute motor and sensory axonal neuropathy. Case Rep Neurol. 2011;3(1):1–6. 26. Power R, Gore-Felton C, Vosvick M, Israelski DM, Spiegel D. HIV: effectiveness of complementary and alternative medicine. Prim Care. 2002;29(2):361–378. 27. Evans S, Fishman B, Spielman L, Haley A. Randomized trial of cognitive behavior therapy versus supportive psychotherapy for HIV-related peripheral neuropathic pain. Psychosomatics. 2003;44(1):44–50. 28. Wiebe LA, Phillips TJ, Li JM, Allen JA, Shetty K. Pain in HIV: an evolving epidemic. J Pain. 2011;12(6):619–624.

79

Section 2 Chapter

11

Spinal Disorders

Cervicogenic headache Eric R. Helm and Nashaat N. Rizk

Case study A 30-year-old male presents to your clinic with a 2month history of base of neck and posterior scalp pain after a motor vehicle accident. He complains of headaches, poor sleep duration, and depressed mood. He has returned to work as an attorney, but poor concentration and cognitive fatigue is affecting his performance.

1. What is the differential diagnosis? a. b. c. d. e. f.

Cervicogenic headache Cervical facet arthropathy Occipital neuralgia Concussion with post-concussive symptoms Occipital-atlantoaxial joint instability Cervical myofascial pain syndrome

Cervicogenic headache is a relatively common secondary source of headache that is seen in 4–6% of the population. It is defined as pain referred to the head from a source in the cervical spine. There is a strong correlation between motor vehicle collision and development of cervicogenic headache. Cervical “whiplash” is a common injury sustained in rear-end motor vehicle collisions. Injury typically occurs in the lower cervical spine components, including zygoapophyseal joint capsular ligament tears, annular disruption, and ligamentum flavum stretch. The suboccipital muscle group can be involved as well. These consist of the rectus capitus posterior major and minor and the superior and inferior oblique. Elliott et al[1] found significantly greater fatty infiltration in the cervical extensors, especially in the deeper muscles of the upper cervical spine, in patients with whiplash-associated disorders compared to healthy controls. The deep suboccipital muscle group produces functional movement

and provides proprioception and stabilization to the craniocervical junction. The pathophysiology behind cervical “whiplash” injuries involves the generation of peak horizontal extension forces transmitted to the cervical spine. During the course of rear-ended impact, the neck is subjected to shear forces parallel to the direction of impact, as well as compression, tension, flexion, and extension at different cervical levels and at different stages of the event.[2] The initial impact results in forward acceleration of both truck and shoulders, forcing the lower cervical spine into extension, as the head moves posterior to the T1 vertebral body. The head is then thrust into extension, followed by forward acceleration, which forces the entire neck into flexion. The discrepancy between the upper and lower cervical spinal segments in flexion and extension, respectively, in combination with the velocity of the impacting vehicle, result in horizontal force transmission. The proposed threshold for a cervical strain is approximately 4–5 g, which is seen with velocities of 6–8 km/hour.[3] At velocities as low as 8 km/hour, 38% of subjects exposed to controlled rear-end collisions experienced cervical “whiplash” symptoms.[4] Whiplash is a multifaceted clinical syndrome that includes neck pain and stiffness, upper limb pain and paresthesias, headaches, visual disturbances, memory and concentration problems, and emotional disturbances. The annual incidence of neck pain associated with whiplash varies greatly. More importantly, although 50% of whiplash victims recover in 3 to 6 months, 30% to 40% have persisting mild to moderate pain and 10% to 20% retain more severe pain.[5] Controversy exists regarding the role psychologic factors play in the injured patient. These patients may also be experiencing neuropsychologic disturbances from an acquired traumatic

Case Studies in Pain Management, ed. Alan David Kaye and Rinoo V. Shah. Published by Cambridge University Press. © Cambridge University Press 2015.

81

Chapter 11: Cervicogenic headache

brain injury. The most common problems seen in brain injuries of this magnitude are inattention, working memory difficulties, and cognitive fatigue. Currently, there is not a lot of strong data to support the incidence of cervical discogenic disease, cervical radiculopathy, and temporomandibular joint dysfunction associated with cervical whiplash. However there is considerable overlap between whiplash-associated disorders and the cervical anatomic abnormalities that are discussed below.

2. What are the differentiating features of the cervical spine pain syndromes? Cervicogenic headache is pain perceived in the head referred from a primary source in the cervical spine. When clinical criteria have been used, the prevalence of cervicogenic headache has been estimated to be up to 4.1% in the general population and as high as 17.5% among patients with severe headaches.[6] The most reliable features are: pain that starts in the neck and radiates to the fronto-temporal region; pain that radiates to the ipsilateral shoulder and arm; and provocation of pain by neck movement.[7] Sometimes it is tough to distinguish cervicogenic headache from migraine. Clinical features that point more toward cervicogenic headache are the absence of an aura, history of neck trauma, fluctuating non-throbbing pain quality, and exacerbation with provocative neck maneuvers (e.g., supine cervical facet glide examination). Given the mechanism discussed above for cervical “whiplash,” the cervical zygapophysial joints (Z-joints, facet joints) are at a substantial risk for overload and injury. The prevalence of cervical facet joint-mediated pain in patients with complaints of neck pain ranges from 36% to 60%.[8] Cervical zygapophysial joints extend from C2-C3 to C7-T1. The distribution of axial cervical pain associated with these joints can encompass the suboccipital to mid-scapular regions, depending on the joint involved. Upper cervical zygapophysial joint pain (e.g., C2-C3, C3-C4) may present as neck pain with associated headaches, whereas lower cervical joint involvement (e.g., C5-C6, C6-C7) may present as neck pain with associated shoulder or mid-scapular pain.[9] The majority of patients with cervical facet joint pain has only one symptomatic joint; however traumatic etiologies are more likely to have multiple joint involvements. Traumatically induced lower cervical pain attributable

82

to a facet joint most commonly involves the C5-C6 level.[10] Non-traumatic involvement is likely caused by either improper biomechanics or cervical spondylosis. Key features of cervical facet joint-mediated pain include: axial neck pain with potential radiation into the periscapular region, focal tenderness to palpation posterolaterally over symptomatic joint, and increased focal suboccipital pain exacerbated with 45 degrees of cervical flexion followed by axial rotation.[11] Myofascial pain syndrome is a local and regional pain disorder characterized by the presence of trigger points, which can be active or latent. Active trigger points convey painful information in the absence of palpation and are more likely present in patients with regional conditions. Latent trigger points require palpation to produce local and referred pain. These trigger points are discrete areas of deep muscle tenderness located in a taut band in the muscle. Palpation results in distant zones of perceived pain, local “twitch response,” or muscle contraction. Trigger points are a distinct entity from the tender points that comprise the clinical diagnosis of fibromyalgia. Myofascial pain syndrome is more common in certain patient populations, such as female sex, whiplash-associated disorders, mood disorders (e.g., depression, anxiety), and tension-type headaches. Cervical joint subluxation is a serious injury that can be sustained during a motor vehicle accident. Abrupt deceleration typically results in a flexiontype injury, while abrupt acceleration results in an extension-type injury. The mechanism of injury for cervical joint subluxation or dislocations is the application of an axial load in conjunction with excessive cervical flexion or extension. The most common affected level is C5-C6 because of the increased movement in this area. Excessive flexion in the cervical spine, especially C3-C7 levels, can cause an anterior subluxation or unilateral or bilateral facet subluxations. The most devastating complication of bilateral cervical facet joint subluxations is when they result in “jumped locked” facets. This typically results in a neurologically complete spinal cord injury.

3. What are some of the legal ramifications encountered with workplace and motor vehicle injuries? The most expensive source of injuries is work and transportation-related musculoskeletal disorders. A National Academy of Sciences study found that

Chapter 11: Cervicogenic headache

musculoskeletal disorders of the neck and arm cause more than 1 million workers to miss time from their job each year, at an annual cost of more than $50 billion.[12] Workers’ compensation involves interaction between medical and legal systems, and physicians that treat spine-related conditions need to understand some basics of the legal system. When a medico-legal case is contested, the treating physician can be required to give sworn testimony regarding their patient. This can be in the form of a courtroom testimony or deposition. Prior to this deposition, it is prudent to review all available medical records. Law professionals always stress the five Ps: “Proper preparation prevents poor performance.” The typical issues in question during deposition involve the mechanism of injury, pre-existing conditions, contributing factors, and whether the resulting impairments or disability is consistent with the injury.[13] In dealing with medico-legal cases, important terms to understand are malingering, causation, and Daubert. Malingering is the intentional misrepresentation of signs or symptoms with the intent to receive secondary gain.[13] It should not be confused with symptom exaggeration or magnification. Workplace and traffic camera use of surveillance to document potential malingering is becoming more commonplace in today’s society. Hospital-based outpatient practices can use video evidence, especially in determining functional ability and return to work status. In a medico-legal case, causation provides a connection between the mechanism of injury and the resultant functional ability. A physician’s opinion during a deposition must be within a “reasonable degree of medical certainty.” This reasonable certainty means that, based on available evidence, the truth of the statements is more likely than not.[14] The principle of Daubert is important to understand if you are asked to give expert testimony in a lawsuit. It states that expert testimony must be generally well accepted in the medical community, published in peer-reviewed literature, have a scientific basis, and have a known error rate.[13] These four criteria must be met for an expert witness to appropriately testify.

4. What are the differing roles of clinical assessment between a clinical practitioner and independent medical assessment? The in-depth clinical assessment of a patient with neck pain and headache involves an extensive history and

physical examination with provocative testing, radiologic viewing, medical record review, diagnosis, treatment plan, and follow-up care. A patient-physician relationship has been established and the treatment plan is consistently being updated and reviewed at subsequent office visits. Based on response to treatment and return to work goals, formal rehabilitation programs such as work hardening can be utilized. Functional capacity evaluations (FCE) can also be beneficial in return to work status. This is an extensive evaluation with formalized physical and occupational testing. They are performed by physical and occupational therapists. The FCE often can guide or clarify what category job the injured worker can perform and is helpful in documenting inconsistencies, decreased effort, and lack of validity on repeat testing.[13] Vocational rehabilitation is helpful when a patient is unable to return to their previous employment. A clinical assessment differs from an independent medical assessment. An independent medical assessment (IME) involves an independent review of a medical case. An IME can help to clarify controversies regarding causation, maximal medical improvement, work restrictions, impairments, and disability.[13] This encounter does not establish a doctor-patient relationship and no treatments are ordered by the examining physician. The physician performing the IME thoroughly reviews all available medical records, radiologic imaging, laboratory testing, and performs an extensive history of physical examination. Similar to a clinical assessment, diagnoses are stated and recommendations are given. A commendable IME involves a thorough assessment, unbiased thought process, clear recommendations, and is legally defensible in court. Important definitions to understand when conducting an IME are maximal medical improvement, impairment, and disability. Maximal medical improvement is attained once the medical condition has resolved or has become fixed and stable. At this point, further diagnostic testing and intervention are not recommended. At maximum medical improvement (MMI), the injured worker is not expected to significantly change in pain level or functional ability in the near future.[14] The American Medical Association (AMA) defines impairment as a significant deviation, loss, or loss of use of any body structure or body function in an individual with a health condition, disorder, or disease.[15] The impairment rating scales are used to calculate the range of

83

Chapter 11: Cervicogenic headache

whole-person impairment in certain disease states. Disability is the inability to engage in any substantial gainful activity, by reason of any medically determinable physical or mental impairment, that can be expected to result in death or that has lasted or can be expected to last for a continuous period of not less than 12 months.[16]

5. How do clinical assessments differ among practitioners in the diagnosis of cervicogenic headache? The clinical assessment of cervicogenic headache starts with an in-depth history of physical examination. It is important to determine the mechanism of injury, chronicity of symptoms, radiologic studies, pre-morbid diagnosis that may confer increased risk, completed treatments, previous and current functional abilities, and return to work status. Then there is the formulation of a “working” differential diagnosis. This is followed by a formal physical examination with emphasis on the upper and lower cervical spine. Clinical evaluation of the upper cervical spine is the most important part of the examination. This evaluation is focused on the assessment of dynamic movements that occur in the craniocervical junction. Hypermobility of the craniocervical junction can lead to mechanical instability. Severe instability can lead to spinal cord compression, resulting in bowel and bladder incontinence, gait ataxia, hemiparesis, and paresthesias. Clinicians need to consider the possibility of craniocervical junction hypermobility and instability when evaluating a patient with axial neck pain and headaches. Physical therapists, chiropractors, and interventionalists each have their own specialty training and approach to the assessment and treatment of cervicogenic headache. Physical therapists are trained to assess body position and functional movements. Their overall goals are to functionally assess, stretch, stabilize, strengthen, and return patient to sport. Appropriate modifications in manual therapy, such as cervical facet mobilization and manipulation, should be made in underlying craniocervical junction hypermobility. Two specific physical examination procedures can be used diagnostically to test hypermobility at these joints: the Sharp-Purser and lateral shear test. The Sharp-Purser test evaluates flexion of the atlas while the lateral shear test evaluates lateral translation of the atlas. The Sharp-Purser test is

84

performed by placing one hand over the patient’s forehead, while the thumb of the opposite hand is placed over the C2 spinous process for stabilization.[17] The patient is asked to flex the head on the neck, while you apply a posteriorly directed force on the forehead. A positive test is indicated if there is backward movement of the head, which may be accompanied by a “clunk.” The backward movement indicates that the subluxation of C1, produced by forward flexion of the neck, has been reduced.[18] The associated sensitivity is 0.69, and specificity is 0.96, when compared with a radiographic reference standard in patients with rheumatoid arthritis.[19] The lateral shear test is performed with the patient lying supine. The metacarpophalangeal (MCP) joint of digit 2 is placed against the transverse process of the atlas on one side and the MCP of digit 2 from the contralateral hand is placed on the transverse process of axis on the opposite side. The two MCP joints are pushed toward one another, creating a shear force of C1 on C2. The test is considered positive if you feel an increased transitory “shift” between the two bones or patient symptoms are provoked by the maneuver.[18] Chiropractors also perform a clinical functional assessment. However, more time is spent in determining specific areas that have restricted motion and evaluating for frank instability. The chiropractor’s emphasis is on treatment, not diagnostic work-up, which differs significantly from both physical therapy and physicians. Both chiropractors and physical therapists employ techniques such as high velocity low amplitude (HVLA) manipulation, cervical facet mobilization, myofascial release techniques, and cervical spine traction. However, typically physical therapists also focus on postural mechanics, vestibular therapy exercises, and return to work/sport, all of which are important in treating cervicogenic headache in active individuals. Clinicians should consider interventional treatment when patients have not improved with noninterventional strategies, including physical therapy with manual techniques, chiropractor care, medication trials, and activity modification. At this point an interventionalist will perform diagnostic cervical medial branch blocks, therapeutic intra-articular facet joint injections, and radiofrequency neurotomy. It is important to remember that the success of all radiofrequency neurotomy procedures for managing cervicogenic headache depends on the patient’s previous response to diagnostic challenge with cervical medial

Chapter 11: Cervicogenic headache

Table 11.1. Criteria for cervicogenic headache per the International Headache Society

Criteria

Description

A

Pain, referred from a source in the neck and perceived in one or more regions of the head and/or face, fulfilling criteria C and D

B

Clinical, laboratory, and/or imaging evidence of a disorder or lesion within the cervical spine or soft tissues of the neck known to be or generally accepted as a valid cause of headache

C

Evidence that the pain can be attributed to the neck disorder or lesion based on at least one of the following:  Demonstrations of clinical signs that implicate a source of pain in the neck; or  Abolition of headache after diagnostic blockade of a cervical structure or its nerve supply by the use of placebo or other adequate controls

D

Pain resolves within 3 months after successful treatment of the causative disorder or lesion

branch block.[20] The most recent criteria of the International Headache Society (see Table 11.1) list diagnostic blockade of a cervical structure or of its nerve supply as mandatory for the diagnosis of cervicogenic headache.[21] Despite discussions and controversy among many practitioners about the diagnosis of cervicogenic headache or for that matter, most benign spinal pain syndromes, these discussions have focused on a structural basis for pain. There has been a gap, unfortunately, between the scientific understanding of chronic pain and our clinical methods to assess pain. Neurobiologists increasingly view chronic pain as a disease of the nervous system and less of a structural problem, per se. Clinicians, likewise, should begin embracing these concepts rather than sticking to an oversimplified structural approach. In fact the opposite is true, wherein ad hoc classification systems and treatment approaches abound. Does the patient have cervicogenic headache, muscle spasms, Qi blood stagnation syndrome, subluxed joints, entrapped/ inflammed upper cervical and occipital nerves, or facet synovitis? Several articles have raised concern that these disparate classification systems are integral

to how practitioners treat pain and how these treatments are not coordinated. This is unique to chronic pain. One would not use multiple classification systems and disparate treatments for cardiac disease, diabetes mellitus, or cancer – but we do for chronic pain syndromes. The use of ad hoc classification systems in chronic pain, such as cervicogenic headache, highlights our lack of understanding of this disease. Consequences include patient and practitioner frustration with treatment, rising expenditures, payor denial of care, and societal costs.[22–25]

6. What symptoms can be associated with cervicogenic headaches? Cervicogenic headache is a multifaceted clinical syndrome that includes neck pain and stiffness, upper limb pain and paresthesias, headaches, tinnitus, visual disturbances, memory and concentration problems, and emotional disturbances. The most commonly seen cognitive problems are cognitive fatigue, poor concentration, and inattention, especially divided attention. Functional MRI studies have shown the frontal and parieto-occipital lobes as common areas of hypoperfusion in cervicogenic headache patients. Memory loss can also be seen, especially anterograde memory loss. The Galveston Orientation and Amnesia Test (GOAT) and the Orientation Log (O-Log) are clinical tests that can be used in assessment and to track progress of memory loss associated with posttraumatic amnesia. Deficits in working memory can be seen in higher functioning individuals. Mood disorders, such as depression and anxiety, are typically seen late in the disease course. These are seen more commonly in “chronic” pain states. Cortical neurotransmitter reorganization, with deficits in serotonin and norepinephrine, has been seen in basic science research. Patients may warrant psychologic assessment and the addition of a SSRI or SNRI as part of the treatment plan. The diagnosis of cervicogenic headache relies on establishing that the pain generator lies in the neck, using reliable and validated diagnostic techniques. Nikolai Bogduk’s convergence trigeminocervical nucleus convergence theory (Figure 11.1) is the most commonly accepted model. In this theory, nociceptive afferents from the C1, C2, and C3 spinal nerves converge onto second-order neurons that also receive afferents from adjacent cervical nerves and from the first division of the trigeminal nerve, via the

85

Chapter 11: Cervicogenic headache

Figure 11.1. Convergence between cervical and trigeminal afferents in trigeminocervical nucleus.[35]

Midbrain

Pons

Trigeminal nerve (V)

Trigeminothalamic tract Spinal tract of trigeminal nerve

C1 spinal nerve

C2 spinal nerve Trigeminocervical nucleus C3 spinal nerve

trigeminal nerve spinal tract.[26] This convergence results in referral of upper cervical pain to the head, specifically the occipital and auricular region and parietal, frontal, and orbital area. The cervical spine has multiple possible pain generators and referral patterns (Table 11.2) that must be considered in the work-up of a patient with cervicogenic headache. Common areas of pain referral are toward the vertex of the scalp, ipsilateral anterolateral temple, forehead, midface, or ipsilateral shoulder girdle. Cyriax[27] showed that stimulation of the suboccipital muscles with injections of hypertonic saline could produce referred pain in the head. The more cephalad the site of stimulation, the closer to the forehead the pain projected. Stimulation of upper cervical spine structures elicits pain in the occipital, frontal, and orbital regions, while more caudal stimulation elicits pain in the base of the neck and periscapular region. The referred pain from cervical facet

86

joints, especially the C2-C3 zygapophysial joint, is a common cause of secondary headache. Among patients with neck pain and headache in which headache was the dominant symptom, the prevalence of C2-C3 zygapophysial joint pain was 50%.[28] Lord et al[29] performed 44 double-blind, placebocontrolled blocks of the third occipital nerve; 53% of patients whose major complaint was headache obtained relief from the block that was of appropriate length based on whether a short- or longer-acting anesthetic was used.

7. What are some of the different subsets in cervicogenic headache? We will discuss the common subsets that encompass cervicogenic headache. These are the occipitalatlantoaxial joint complex and greater, lesser, and least occipital neuralgia. The occipital-atlantoaxial

Chapter 11: Cervicogenic headache

Table 11.2. Cervicogenic headache pain generators and their referral patterns[30]

Pain generator

Innervation

Referral pattern

Atlanto-occipital joint

Ventral ramus of C1

Occiput, subocciput

Atlantoaxial joint

Medial AA joint: recurrent meningeal branch (ventral ramus) of C1, C2, C3 Lateral AA joint: ventral ramus of C2

Occiput, subocciput, vertex, orbit, and ear

C2-C3 zygapophysial joint

Third occipital nerve (superficial medial branch of C3 dorsal primary ramus)

Head, upper, and lateral cervical region

C3-C4 zygapophysial joint

C3 and C4 medial branch of dorsal primary ramus

Head, upper, and lower cervical region

C2-C3 disc

Sinuvertebral branch of superficial medial branch of C3 dorsal ramus

Occiput

Greater occipital nerve

C2 medial branch of dorsal primary ramus

Occiput, C2 dermatome

Lesser occipital nerve

C3 dorsal primary ramus

Occiput (lateral to GON)

Greater auricular nerve

C2 and C3 dorsal ramus contributions

Posterior scalp behind the ear, posterior auricular region, and skin overlying the parotid gland

joint is the most complex joint in the craniocervical junction. These are two distinct joint complexes: occipital-atlanto (C0-C1) and atlantoaxial (C1-C2) joint complexes. The C0-C1 joint is responsible for flexion, extension, and lateral flexion. The C1-C2 joint is designed for rotation, with 50% of the total cervical rotation taking place at this level.[31] Previously discussed above, the suboccipital muscle group provides both active movement and proprioception to these joints. The C1-C2 articulation has two primary ligamentous stabilizers: the cruciform ligament and the paired alar ligaments. The cruciform ligament’s horizontally oriented fibers are the primary passive restraint of C1 displacement in the sagittal plane.[32] The alar ligaments are important passive restraints to axial rotation of the atlas on the axis.[33] The alar ligaments restrain contralateral axial rotation (e.g., the left alar ligament restrains right axial rotation) and lateral flexion. Patients with mild to moderate instability of these joint complexes present with symptoms of suboccipital pain, dizziness, unilateral headache, and upper limb paresthesia.[34] Severe instability can lead to catastrophic consequences, including disturbances of bowel/bladder control, impaired gait, motor incoordination, sensory loss, and the extreme instance of death from spinal cord compression.[36] Clinicians must be cautious in the application of a manual force to the craniocervical

junction and prior diagnostic work-up should include anterior-to-posterior open mouth radiographs, with the head in neutral, right, and left lateral flexion positions (Figures 11.2 and 11.3). The typical radiology order for these views are: three open mouth AP views of C1-C2 (one as a straight AP, one maximally laterally flexed to the right, one maximally laterally flexed to the left) to assess for lateral deviation of the C1–2 joint. The International Classification of Headache Disorders II has three diagnostic criteria for the classification of occipital neuralgia[21]: A. Paroxysmal stabbing pain, with or without persistent ache between paroxysms, in the distribution(s) of the greater, lesser, and/or third occipital nerves. B. Tenderness over the affected nerve. C. Pain eased temporarily by local anesthetic block of the nerve. Occipital neuralgia must be distinguished from occipital referral of pain from the atlantoaxial joints, upper cervical zygapophysial joints, or active trigger points in neck muscles or their insertions.[37] Each peripheral occipital nerve has a different distinct level of innervation. The greater occipital nerve is a continuation of the C2 medial branch of dorsal primary ramus, while the lesser occipital nerve is a

87

Chapter 11: Cervicogenic headache

L

R

Figure 11.2. Abnormal right translation of the right lateral mass over the body of C2, due to the failure of the left alar ligament. The right alar ligament appears intact.

3 mm

A

B

C

Figure 11.3. Normal C1-C2 joint translation in right lateral flexion, straight AP, and left lateral flexion (A–C).

continuation of the C3 dorsal primary ramus. The 3rd occipital nerve (also known as the least occipital nerve) originates from the superficial medial branch of C3 dorsal primary ramus. There is considerable

88

overlap in the referral pattern of these pain generators (Table 11.2). The 3rd occipital nerve is most commonly implicated in the interventional treatment of cervicogenic headache. However, it is

Chapter 11: Cervicogenic headache

important to recognize the more peripherally located pain generators.

8. What are the available treatments for cervicogenic headache and occipital neuralgia? Cervicogenic headache treatment options are based on detailed history and physical examination findings, radiologic studies, and formal clinical assessment. In the evaluation of a patient with cervicogenic headache, the clinician must determine if there is any sign of hypermobility in the craniocervical junction and rule out abnormal neurologic signs. In the presence of either hyper- or hypo-mobility of the C0-C1, C1-C2, or C2-C3 zygapophysial joints, an initial treatment plan should include referral to physical therapy. The manual treatment plan must include cervical stabilization exercises, and proprioceptive training with eye and head coordination exercises in dynamic positions. Postural exercises focusing on cervicoscapular stabilizers, scapular mechanics, and upper-body ergometry are important to emphasize.[38] Examples of these include prone scapular depression and elevation, upright rows, and seated pull-downs. Dynamic strengthening exercises are incorporated later on in the plan of care. At the completion of the outpatient physical therapy program, a home exercise program of trunk stabilization is provided, consisting of bridging, prone planks, and side planks.[17] In the presence of hypomobility of the cervical and thoracic spine, both the physical therapist and chiropractor should employ a manual therapy protocol. This should include: mobilization of the occiput, cervical and thoracic facet, and costo-transverse joints, traction manipulation of the occiput, and occipital glide mobilization.[39] The primary rational for these components is to inhibit increased activity in the suboccipital and levator scapulae muscles and relieve stiffness in the upper thoracic region.[40] Thoracic mobilization is important in this patient population because restoring normal scapulothoracic movement will assist with reducing postural stiffness. The C2-C3 zygapophysial joint is believed to be responsible for 70% of cervicogenic headaches, while the atlantoaxial joint is thought to be the 2nd most common source.[41] Interventional procedures targeting the 3rd occipital nerve, craniocervical junction, cervical medial branch dorsal primary ramus

anesthetic blocks, and upper cervical intra-articular facet joints are common treatments after poor symptom relief with less invasive modalities. All diagnostic and therapeutic cervical spine procedures require imaging guidance with live and static fluoroscopy to ensure correct placement of the needle/probe and injectate at the targeted structure. The use of nonionic contrast media should also be used for target localization and exclude intravascular uptake. Current literature supports the role for diagnostic medial branch blocks and not intra-articular injections as the criterion standard for diagnosis of pain emanating from or mediated by the cervical zygapophysial joint.[42] The 3rd occipital nerve is thicker than the cervical medial branches of the dorsal primary ramus and usually embedded in the periscapular fascia of the C2-C3 joint. Three target points have been defined by Bogduk and Dreyfuss to ensure correct needle placement for 3rd occipital nerve blocks. The target points lie on a vertical line that bisects the C2-C3 joint. The cranial target point lies opposite the level of the apex of the C3 superior articular process. The caudal target point lies opposite the inferior aspect of C2-C3 intervertebral foramen. The middle point lies midway between these two points, typically on the subchondral plate of the C3 superior articular process. See Figure 11.4. Positive comparative blocks with two different local anesthetics will generally allow the operator to be 81% certain that a confirmatory response represents a “true” positive. This statistic is true regardless of whether the result is “concordant” positive or “discordant” positive, in which the duration of pain relief correlates or does not correlate with the local anesthetic used.[42] The International Headache Society criteria for cervicogenic pain accept  90% reduction in pain to a level of < 5 on a 100-point visual analog scale.[21] The most researched interventional treatment for cervicogenic headache is percutaneous radiofrequency neurotomy (radiofrequency ablation). The goal of this procedure is destruction of the afferent nerve supply from the 3rd occipital nerve and C2 and C3 medial branches. The target radiofrequency temperatures range from 80 to 90°C and duration from 80 to 90 seconds. It is important to remember that the coagulation occurs in a radial direction, perpendicular to the long axis of the electrode,[43] so the active radiofrequency needle tip must be placed parallel to the targeted nerve. Data shows that 3rd occipital

89

Chapter 11: Cervicogenic headache

A

B

C

Figure 11.4. Lateral views of the upper cervical spine showing the target points for third occipital nerve blocks (A–C).

nerve radiofrequency neurotomy is effective in 88% of patients that have confirmatory controlled diagnostic blocks with a 297-day median duration of relief.[44] Pulsed radiofrequency neurotomy is a technique in which energy is applied intermittently in short bursts, allowing dispersion of heat between each cycle and temperature is held at a non-neuronal destruction level. One study evaluated its use for cervicogenic headache and was performed at the lateral atlantoaxial joint. Fifty percent of patients had > 50% pain relief at 2 and 6 months, and 44% at 1 year.[45] Intra-articular corticosteroid injections, especially targeting the lateral atlantoaxial joint, have been

90

described as effective treatment for cervicogenic headache. Narouze et al investigated 32 patients that received lateral atlantoaxial joint injections for C1-C2 pain. More than 80% of patients had 50% pain relief immediately after the injection. However at the 6th month time-point, they were no longer experiencing any significant symptom improvement.[46] Slipman et al retrospectively evaluated C2-C3 facet joint injections for cervicogenic headache after a whiplash event. Residual improvement was demonstrated at an average of 12 months.[47] Patients with whiplash-associated cervicogenic headache typically suffer from headaches, and ultimately may develop chronic daily headaches. A greater

Chapter 11: Cervicogenic headache

A

B

C

Figure 11.5. Lateral and AP views of the upper cervical spine showing the target points for third occipital nerve radiofrequency neurotomy (A–C).

occipital nerve block can be a beneficial treatment for both the suboccipital pain and headaches. The injection site is localized 1/3 of the distance from the greater occipital protuberance to the mastoid process. A newer, more novel treatment for chronic daily headaches that have a cervicogenic component is Botulinum toxin. The FDA has approved Botulinum

toxin for treatment of chronic migraine headaches, with 15 or more headache days a month, each lasting 4 hours or more. Botulinum toxin (BOTOX) is not specifically indicated for treatment of cervical spine pain generators, but may prove beneficial for these patients suffering from chronic daily headaches. Botulinum toxin is a neurotoxin and biologic product

91

Chapter 11: Cervicogenic headache

A

B

Figure 11.6. AP and lateral views of the upper cervical spine showing the target points for lateral atlantoaxial joint injection (A–B).

of bacteria, Clostridium botulinum. All seven serotypes (A–G) act at the presynaptic neuromuscular junction to inhibit the release of acetylcholine. The mechanism of action involves uptake by the synaptic vesicle, followed by cleavage of one or more of the SNAPE proteins. This destruction results in the inability of the vesicle to bind and release the acetylcholine into the synaptic cleft, resulting in transient motor weakness. It is also FDA approved for the treatment of conditions such as strabismus, blepharospasm, upper limb spasticity, and cervical dystonia. The recommended dose for treating chronic migraine is 155 Units administered intramuscularly using a sterile 30-gauge, 0.5-inch needle. Two industry-sponsored randomized, multicenter, placebo-controlled doubleblind studies have shown that treated patients had a greater decrease in headache frequency, especially after 4 weeks. Line et al[48] analyzed the response to Onabotulinum toxin A in 28 cervicogenic headache patients. The randomized, placebo-controlled, crossover study found no significant difference between Onabotulinum toxin A and placebo in the reduction of days with moderate to severe headache. Peripheral nerve stimulation does offer a novel approach in the treatment of occipital neuralgia. Peripheral nerve stimulation is an important treatment

92

algorithm for neuropathic pain. Peripheral nerve stimulation is defined as the direct electrical stimulation of named peripheral nerves that lie outside the neuroaxis. Peripheral nerve field stimulation is the stimulation of unnamed nerves that lie in the vicinity of painful, subcutaneous structures. FDA defines both uses as “off-label” in the treatment of neuropathic pain syndromes. Percutaneous occipital nerve stimulation involves a trial placement of subcutaneous electrodes placed superficial to the cervical muscular fascia in the suboccipital area.[37] If the patient has an effective trial, a permanent implant may be carried out using the same percutaneous electrode lead or paddle-type surgical lead, which is attached to a pulse generator implanted in the infraclavicular area, flank, upper buttock, or abdomen.[37] Indications for occipital stimulation include patients diagnosed with cervicogenic headache and occipital neuralgia. There is clinical discrepancy regarding diagnostic confirmation of symptom improvement to occipital nerve blocks and/or interventional treatment in the upper cervical spine. Contraindications to the procedure include untreated psychiatric disorder, bleeding disorder, and systemic or local infections. Several small, uncontrolled studies have investigated the use in occipital neuralgia. Johnstone and Sunderaj found that 70% of patients

Chapter 11: Cervicogenic headache

that underwent permanent implantation had at least > 50% reduction in VAS pain score.[49] Slavin et al. performed occipital nerve stimulation on 14 patients with occipital neuralgia. This was a retrospective study

References 1.

2.

3.

4.

5.

6.

7.

8.

Elliott J, Jull G, Noteboom JT, et al. Fatty infiltration in the cervical extensor muscles in persistent whiplash-associated disorders: a magnetic resonance imaging analysis. Spine (Phila Pa 1976). 2006;31:E847–855. Tencer AF, Mirza S, Bensel K. Internal loads in the cervical spine during motor vehicle rear-end impacts: the effect of acceleration and head-to-head restraint proximity. Spine. 2002;27: 34–42. Allen ME, Weir-Jones I, Motiuk DR, et al. Acceleration perturbations of daily living: a comparison to “whiplash.” Spine. 1994;19:1285–1290. Brault JR, Wheeler JB, Siegmund GP, Brault EJ. Clinical response of human subjects to rear-end automobile collisions. Arch Phys Med Rehabil. 1998;79:72–80. Carroll LJ, Holm, LW, HoggJohnson S, et al. Course and prognostic factors for neck pain in whiplash-associated disorders (WAD): results of the Bone and Joint Decade 2000–2010 task force on Neck Pain and Its Associated Disorders. J Manip Physiol Therapeut. 2009;32(Suppl 2): S97–S107. Evers S. Comparison of cervicogenic headache with migraine. Cephalalgia. 2008; 28(Suppl 1):16–17. Van Suijlekom JA, de Vet HCW, van den Berg SGM, Weber WEJ. Interobserver reliability of diagnostic criteria for cervicogenic headache. Cephalalgia. 1999;19:817–823. Barnsley L, Lord SM, Wallis BJ, et al. The prevalence of chronic cervical

9.

with a mean follow-up of 22 months. Seventy percent of the patients had a successful trial and underwent permanent implantation, with reduction in VAS pain scores that ranged between 60 and 90%.[50]

zygapophyseal joint pain after whiplash. Spine. 1995;20:20–26.

PA: WB Saunders Company. 2008.

Cooper G, Baily B, Bogduk N. Cervical zygapophysial joint pan maps. Pain Med. 2007; 8(4):344–353.

19. Utivlugt G, Indenbaum S. Clinical assessment of atlantoaxial instability using Sharp-Purser test. Arthritis Rheum. 1988;31:321–329.

10. Bogduk N, Aprill C. On the nature of neck pain, discography and cervical zygapophysial joint blocks. Pain. 1993;54:213–217.

20. Blume HG. Cervicogenic headaches: radiofrequency neurotomy and the cervical disc and fusion. Clin Exp Rheumatol. 2000;18(Suppl 19):S53–S58.

11. Dreyfuss P, Michaelsen M, Fletcher D. Atlanto-occipital and lateral atlanto-axial joint pain patterns. Spine. 1994;19:1125–1131.

21. International Headache Society. International Headache Society Classification (ICHD-II). 2001, Nov 2.

12. National Academy of Sciences. Musculoskeletal Disorders and the Workplace: Low Back and Upper Extremities. Washington, DC, National Academy of Sciences. 2001. 13. Braddom L. Physical Medicine & Rehabilitation. Philadelphia: Saunders Elsevier. 2012. 14. Melhorn JM. Impairment and disability evaluations: understanding the process. J Bone Joint Surg Am. 2001;83(12): 1905–1911. 15. Cocchiarella L, Andersson GBJ, eds. Guides to the Evaluation of Permanent Impairment, 5th ed. Chicago: American Medical Association Press. 2001. 16. SSA. Disability Evaluation Under Social Security. Washington, DC: US Government Printing Office. 1994. 17. Mathers SK, Schneider M, Timko M. Occult hypermobility of the craniocervical junction: a case report and review. J Orthop Sports Phys Ther 2011;41(6):444–457. 18. Magee DJ. Orthopedic Physical Assessment, 5th edn. Philadelphia,

22. Shah RV. Spine pain classification: a solution. Pain Physician. 2013;16 (2):E51–59. PubMed PMID: 23511691. 23. Shah RV. The problem with diagnostic selective nerve root blocks. Spine (Phil Pa 1976). 2012;37(24):1991–1993. doi: 10.1097/BRS.0b013e318270a7ba. PubMed PMID: 22941096. 24. Shah RV. Spine pain classification: the problem. Spine (Phila Pa 1976). 2012;37(22):1853–1855. doi: 10.1097/BRS.0b013e3 182652a86. PubMed PMID: 22732822. 25. Shah RV, Kaye AD. Evolving concepts in the understanding of cervical facet joint pain. Pain Physician. 2004;7(3):295–299. PubMed PMID: 16858465. 26. Bogduk N. Cervicogenic headache: anatomic basis and pathophysiologic mechanisms. Curr Pain Headache Rep. 2001;5: 382–386. 27. Cyriax J. Rheumatic headache. BMJ. 1938;2:1367–1368. 28. Lord SM, Barnsley L, Wallis BJ, Bogduk N. Chronic cervical zygoapophyseal joint pain after

93

Chapter 11: Cervicogenic headache

whiplash: a placebo-controlled prevalence study. Spine. 1996; 21:1737–1744. 29. Lord SM, Barnsley L, Wallis BJ, Bogduk N. Third occipital nerve headache: a prevalence study. J Neurol Neurosurg Psychiatry. 1994;57:1187–1190. 30. Mehnert MJ, Freedman MK. Update on the role of Z-joint injection and radiofrequency neurotomy for cervicogenic headache. PM R. 2013;5:221–227. 31. Vernon H, Minor S. The Neck Disability Index: a study of reliability and validity. J Manipulative Physiol Ther. 1991;14:409–415. 32. Dvorak J, Panjabi MM. Functional anatomy of the alar ligaments. Spine. 1987;12:183–189. 33. Haldeman S, Kohlbeck FJ, McGregor M. Unpredictability of cerebral vascular ischemia associated with cervical spine manipulation therapy: a review of sixty-four cases after cervical spine manipulation. Spine. 2002;27:49–55. 34. Bitterling H, Stabler A, Bruckmann H. [Mystery of alar ligament rupture: value of MRI in whiplash injuries – biomechanical, anatomical and clinical studies]. Rofo. 2007;179:1127–1136. 35. Bogduk N. The neck and headaches. Neurol Clin N Am. 2004;22:151–171. 36. Aspinall W. Clinical testing for the craniovertebral hypermobility

94

syndrome. J Orthop Sports Phys Ther. 1990;12:47–54. 37. Narouze S. Cervicogenic headache. In Benzon HT, ed. Essentials of Pain Medicine. Philadelphia, PA: Elsevier/ Saunders. 2011: pp. 278-283. 38. O’Leary S, Falla D, Elliott JM, Jull G. Muscle dysfunction in cervical spine pain: implications for assessment and management. J Orthop Sports Phys Ther. 2009;39:324–333. 39. Grieve G. Modern Manual Therapy of the Vertebral Column. London, UK: Churchill Livingstone. 1986. 40. Erhard R. The Spinal Exercise Handbook. A Home Exercise Manual for a Managed Care Environment. Pittsburgh, PA: Laurel Concepts. 1998. 41. Dwyer A, Aprill C, Bogduk N. Cervical zygapophyseal joint pain patterns I: A study in normal volunteers. Spine. 1990;15:453–457. 42. Bogduk N. On the rationale use of diagnostic blocks for spinal pain. Neurosurg Q. 2009;19:88–100. 43. Lord SM, McDonald GJ, Bogduk N. Percutaneous radiofrequency neurotomy of the cervical medial branches: a validated treatment for cervical zygapophyseal joint pain. Neurosurg Q. 1998;8:288–308. 44. Govind J, King W, Bailey B, Bogduk N. Radiofrequency neurotomy for the treatment of third occipital headache. J Neurol

Neurosurg Psychiatry. 2003;74:88–93. 45. Halim W, Chua NHL, Vissers KC. Long-term pain relief in patients with cervicogenic headaches after pulsed radiofrequency application into the lateral atlantoaxial (C1–2) joint using an anterolateral approach. Pain Practice. 2010;10:267–271. 46. Narouze SN, Casanova J, Mekhail N. The longitudinal effectiveness of lateral atlantoaxial intraarticular steroid injection in the treatment of cervicogenic headache. Pain Med. 2007;8:184–188. 47. Slipman CW, Lipetz JS, Plastaras CT, et al. Therapeutic zygapophyseal joint injections for headaches emanating from the C2–3 joint. Am J Phys Med Rehabil. 2001;80:182–188. 48. Linde M, Hagen K. Onabotulinum toxin A treatment of cervicogenic headache: a randomized, double-blind, placebo-controlled crossover study. Cephalalgia. 2011;31(7): 797–807. 49. Johnstone CS, Sunderaj R. Occipital nerve stimulation for the treatment of occipital neuralgia: eight case studies. Neuromodulation. 2006;9:41–47. 50. Slavin KV, Nersesyan H, Wess C. Peripheral neurostimulation for treatment of intractable occipital neuralgia. Neurosurgery. 2006;58:112–119.

Section 2 Chapter

12

Spinal Disorders

Cervical stenosis and myelopathy Santhosh A. Thomas and Garett J. Helber

Case study

2. Assessment

A 68-year-old right-hand dominant female presents with resolving neck and left upper extremity pain. She reports pain developed 8 weeks ago following exercise. She reports her neck pain was constant, sharp, and achy in quality with stabbing pain radiating down her left arm. She noted associated left hand numbness which has resolved. She denies any weakness of her upper or lower extremities. Pain was made worse with nothing specific and improved with stretching. She underwent a course of physical therapy and steroid taper with resulting resolution of all symptoms. On physical examination her tandem gait is impaired with preserved ability to toe and heel walk. Cervical range of motion is full and without pain. Sensation is preserved to light touch in the bilateral upper and lower limb dermatomes. Muscle stretch reflexes noted to be 3+/4 in the right triceps and biceps, bilateral patella, and bilateral Achilles. Sustained clonus is present at the right ankle and Hoffmann’s sign is grossly positive on the left hand. Muscle strength is preserved in all muscles tested of the bilateral upper and lower extremities.

A 68-year-old female with resolved left cervical radiculopathy with concern for cervical myelopathy.

1. Cervical x-rays taken 6 weeks earlier reveal A grade 1 anterior subluxation of C3, minimal posterior subluxation of C5, and grade 1 anterior subluxation of C7. The C3–4 through C6–7 interspaces are severely narrowed with opposing endplate sclerosis, anteroposterior spurring, and uncovertebral joint osteophytes.

3. Plan 1. MRI cervical spine. 2. Call after results obtained to review and formulate treatment plan which may include surgical consultation. 3. Continue home exercise program and activity as tolerated.

4. MRI cervical spine following encounter reveals C2–C3: Canal and foramina are patent. C3–C4: The interspace is severely narrowed and appears fused. Uncovertebral change moderately narrows the neural foramina. The central osseous canal is patent. C4–C5: The interspace is severely narrowed. Disc/ osteophyte changes result in moderate right and severe left foraminal encroachment. There is cord contact with mild ventral cord compression. C5–C6: The interspace is severely narrowed. Disk osteophyte change results in mild ventral cord compression with mild right and severe left foraminal encroachment. C6–C7: The interspace is severely narrowed. Uncovertebral change mildly narrows the neural foramina. Central canal is patent.

Case Studies in Pain Management, ed. Alan David Kaye and Rinoo V. Shah. Published by Cambridge University Press. © Cambridge University Press 2015.

95

Chapter 12: Cervical stenosis and myelopathy

C7–T1: Canal and foramina are patent. Mild facet hypertrophic changes are present bilaterally.

5. What is the differential diagnosis? The differential diagnosis to consider is quite broad and includes:  Cervical spondylotic myelopathy (CSM)  Cervical plexopathy/radiculopathy and/or peripheral neuropathy  Polyradiculitis (Guillain-Barré syndrome)  Multiple sclerosis  Spinal cord injury (central cord syndrome)  Cerebrovascular disease  Syringomyelia  Tabes dorsalis  Atrophic lateral sclerosis  Rheumatoid arthritis  Subacute combined degeneration (vitamin B12)  Intraspinal or intracranial tumor  Spinal arteriovenous malformation  Epidural abscess or hemorrhage  Chiari malformation or other congenital malformation of the brain stem  Ossification of the posterior longitudinal ligament  Normal pressure hydrocephalus  Hereditary spastic paraplegia  Vascular ischemia of the spinal cord

6. What is the definition of cervical spondylotic myelopathy? Cervical myelopathy is present when there is clinically symptomatic dysfunction affecting the cervical spinal cord. When this dysfunction is due to cord compromise resulting from degenerative changes (spondylosis) of the cervical spine it is referred to as CSM. It is important that cervical myelopathy be distinguished from other disorders including cervical radiculopathy and axial neck pain as these conditions may coexist.

7. What is the epidemiology of CSM? CSM is the leading cause of spinal cord dysfunction in older patients and predominantly affects men in their seventh decade of life. Due to its subtle presentation that often involves elderly patients, the diagnosis can be overlooked and thus may still be under-recognized.

96

In one study 23.6% of 585 patients admitted with tetraparesis or paraparesis to a UK regional neuroscience center were diagnosed with CSM.

8. What are the clinical manifestations of CSM? Clinical presentations vary from patient to patient. The presenting symptoms depend on the stage of myelopathy and impaired signals from the spinal cord based on intrinsic or extrinsic neural conditions which include:  Difficulty walking with a wide-based, sometimes jerky or spastic gait is the most common presentation.  Balance difficulties with unsteadiness while on their feet.  Weakness of the upper extremities often with loss of fine motor coordination involving the hands.  Associated numbness and paresthesias of the upper extremities may be present.  Positive Lhermitte sign: neck flexion results in an electrical shock sensation that extends throughout the body (thought to be due to stimulation of the dorsal columns).  Lower extremity dysfunction, including sensory and motor changes, typically occurs after the upper extremities are involved.  Dysfunction of bladder and/or bowel is noted infrequently and is often a late finding. Most common finding in early CSM is urinary frequency and urgency. Urinary retention is mostly associated in patients with CSM over the age of 65.

9. What are the physical examination findings in a patient with CSM?  Hyperreflexia below level of compression.  Inverted radial reflex.  Decreased sensation of any or all sensory modalities depending on the anatomic location of the lesion, but pain and temperature sensation are most commonly affected (due to compression of the spinothalamic tracts).  Weakness due to upper motor neuron damage may be present at the level of the lesion.  Increased muscle tone and spasticity may be found below the level of the lesion.

Chapter 12: Cervical stenosis and myelopathy

 Clonus, Babinski, and Hoffmann’s sign.  Hands may demonstrate intrinsic muscle atrophy.  Evaluation of gait typically reveals a broad-based, hesitant, stiff or spastic gait, secondary to upper motor neuron disease and proprioceptive loss.

10. What are the imaging studies available to evaluate for CSM? A diagnosis of CSM requires a patient who is both symptomatic and has radiographic evidence of spinal cord impingement or compression, thus making radiographic assessment essential. Magnetic resonance imaging is the gold standard as it provides the best view of spinal cord, exiting nerve roots, and CSF signal. CT myelography however may be useful in instances of previous surgery in viewing residual bony anatomy while minimizing artifact from residual hardware. It is important to remember that the appearance of cord compression does not indicate the presence of myelopathy. Further, the correlation between radiographic spondylotic cord damage and clinically significant CSM is not well established. Xray studies, including anteroposterior, oblique, and lateral views with or without flexion and extension of the spine, may reveal narrowing of the disc space, osteophyte formation, spondylolisthesis, and/or instability. These factors have all been identified and implicated in the development of CSM.

11. What radiologic criteria are used to diagnose stenosis? Many investigations have been made in an attempt to diagnose cervical stenosis and correlate it to the development of CSM. Of paramount importance is the fact that a congenitally narrowed canal will lower the threshold at which the cumulative effects of these various structures encroaching on the spinal cord will result in signs and symptoms of myelopathy. The normal cervical canal diameter from C3 to C7 is 17–18 mm in White and 15–17 mm in Japanese individuals. It has been shown that an absolute AP diameter of the canal that is less than 11 mm correlates with a high risk of CSM and that a Pavlov ratio (anteroposterior (AP) diameter of the spinal canal to the anteroposterior diameter of vertebral body at the same level) of 0.8 or less also places a patient at greater risk for development. (A Torg ratio is the

same as a Pavlov ratio.) A normal ratio is 1.0. Symptoms are also believed to develop when the spinal cord has been reduced by at least 30%. Despite numerous studies no definitive criteria currently exist to quantify stenosis, as substantial stenosis has been reported in association with mid-sagittal diameters of < 10 mm,[1] < 12 mm,[2] and < 14 mm.[3]

12. What additional studies may be of benefit in the evaluation of CSM? Bednarik et al found that electromyography and sensory-evoked potential abnormalities, in association with clinical radiculopathy, when present initially predicted the development of CSM.[4]

13. What is the natural history of patients with CSM? The natural history of CSM is mixed and variable and therefore it is very difficult to predict the course of disease in any given patient. Some patients will remain neurologically stable, some will even improve (though significant improvement is rare), while others may experience additional neurologic deficits. A 2002 Cochrane review concluded there is no clear evidence to support the idea that CSM patients experience inevitable neurologic deterioration. Numerous studies demonstrate that longer symptom duration (12–24 months) portends worse neurologic recovery. Subtle progression is characteristic of the disease process, with findings of urinary urgency or incontinence, difficulties with balance and gait, and loss of fine motor control concerning for disease progression.

14. What are the risk factors associated with the development of CSM?  Underlying structural kyphosis.  Abnormal or excessive cervical motion.  Almost all patients with CSM from strictly degenerative changes, excluding those with ossification of posterior longitudinal ligament (OPLL), have congenital stenosis.  Risk factors for the development of spondylosis include advanced age, heavy labor, posture, and genetic predisposition.

97

Chapter 12: Cervical stenosis and myelopathy

15. What is the pathology behind the development of CSM? Any disease leading to loss of space available for the spinal cord, with resulting compromise and dysfunction, may be implicated in the development of CSM. Despite involvement of the spinal cord, this condition begins extrinsic to it with involvement of surrounding osseous and soft tissue structures. The initial lesion is the deterioration of the intervertebral disc, which is often insidious and without symptoms. Repeated stress and aging leads to several changes including disc collapse or deterioration, loss of elasticity, and unequal distribution of hydrostatic pressure on the annulus with compressive forces. As the disc loses its strength and integrity surrounding structures are required to bear a greater burden of the applied load. As these structures continue to bear more weight they undergo secondary changes. End plates, uncovertebral joints, and facet joints remodel and form osteophytes to increase the weight-bearing surface area. Reactive hyperostosis occurs, thus increasing the diameter of the vertebral body at the level. As a result the spondylotic bars/osteophytes can project posteriorly into the spinal canal and reduce the space available for both the spinal cord and its blood supply. Osteophytes that arise from the joints of Luschka and facet joints further compromise the areas of both the spinal canal and neuroforamina. In addition the ligamentum flavum may invaginate into the canal as the disc collapses and additionally compresses the spinal cord. Involvement is most commonly seen at C5–6, followed by C6–7, where most of flexion and extension in the subaxial spine occurs, creating greater reactive changes. Symptoms may be worsened owing to the fact that C5–7 is a watershed area of the cervical cord, thus increasing the risk for spinal cord ischemia at these levels. The resulting neurologic deficit is thus likely due to a combination of neuronal compression and alterations of local neuronal blood flow resulting in ischemia.

16. How is CSM managed nonoperatively? In mild CSM a conservative course appears appropriate but those with significant or progressive neurologic deficits should be considered candidates for surgical intervention. All treatments should attempt

98

to decrease pain and restore function with consideration given for patient safety and if needed home alterations. Patients should first be instructed to avoid those activities which precipitate their symptoms. Immobilization in a firm cervicothoracic orthosis serves to decrease the motion of the vertebral bodies although there is little evidence for efficacy. Those with significant pain may be managed with nonsteroidal anti-inflammatory agents as with other degenerative diseases, with analgesics reserved for more acute and intense periods of pain. The use of epidural steroids has not been shown to result in any lasting benefit.

17. What are the surgical options for management of CSM? Decompression is indicated for significant or progressive neurologic deterioration. The goal of treatment is to relieve the compression on the cord while maintaining spinal stability. The best results may be obtained when decompression is achieved early (6–12 months) after the onset of symptoms and in those who develop early, mild myelopathic findings. Numerous techniques exist to achieve decompression which can be completed via an anterior or posterior approach. Anterior techniques include anterior discectomy with or without fusion and anterior corpectomy with fusion which may also include plate fixation for more extensive disease. Posterior techniques include laminectomy with or without fusion and canal expansive laminoplasty. Anterior cervical discectomy and fusion remains the standard in cases of CSM arising from a single level disc herniation. The procedure allows relief of spinal cord compression with a low rate of postoperative axial neck pain. Multiple discectomies are not favored for multilevel disease as there exists an increased likelihood of symptomatic pseudoarthrosis formation due to the increased number of surfaces across which fusion is expected to occur. Anterior corpectomy and strut grafting is reserved for those cases of multilevel disease where there exists compression of the cord across more than just the disc space or in a patient with pronounced kyphosis. This procedure allows for complete decompression of the cord and the restoration of a more normal cervical alignment. Posterior decompression via laminectomy has historically been the mainstay of treatment in relieving

Chapter 12: Cervical stenosis and myelopathy

compression and restoring neurologic function in patients with neutral or lordotic alignment. However, in those with some degree of kyphosis there is a significant risk of progressive postoperative kyphotic deformity with recurrence of CSM. Long-term studies after isolated laminectomy range from 14% to 47% for postoperative kyphosis. As a result of the potential for kyphotic deformity fusion may be recommended. While accompanying fusion is effective in preventing deformity it often results in loss of a significant degree of motion and thus the risks and benefits of its use must be carefully weighed. Posterior decompression via laminoplasty allows for decompression of the cord without leading to increasing kyphotic deformity. However, the procedure does not ensure that the spinal canal will be completely open for the cord when the posterior elements are hinged open while the lamina remains preserved. However, as there is no fusion less loss of cervical motion occurs and lessens the need for postoperative immobilization. Despite these general indications there is currently no class I or II evidence to suggest superiority among laminoplasty, laminectomy with arthrodesis, anterior cervical corpectomy and fusion (ACCF), or anterior cervical discectomy and fusion (ACDF) with plate fixation.

18. How does ossification of posterior longitudinal ligament cause CSM? The spinal ligamentous tissue is replaced by ectopic new bone formation, leading to narrowing of the spinal canal. The etiologies of OPLL remain ambiguous; nevertheless, genetic background is a contributing factor. This disease is more prevalent among Japanese and other Asians compared to Whites. OPLL progression has been noted in long-term follow-up after cervical laminoplasty and should be given consideration as a cause of recurrent myelopathy.

Cited References 1.

Adams CBT, Logue V. Studies in cervical spondylotic myelopathy. II. The movement and contour of the spine in relation to the neural complications of cervical spondylosis. Brain. 1971;94: 569–586.

19. How does rheumatoid arthritis cause CSM? Cervical spine involvement is well established in patients with rheumatoid arthritis (RA). Upper cervical lesions recognized as atlantoaxial subluxation are well known in patients with RA. Synovial pannus formation is also well recognized in patients with RA. Indirect compression of the spinal cord by cervical subluxation and/or direct compression of the spinal cord by the synovial pannus can present as CSM. The atlas dental interval (ADI) is a useful marker for evaluating atlantoaxial subluxation. It is generally accepted that patients with ADI exceeding 5 mm have a greater risk of CSM but this can be unreliable. Cranial settling can also be seen in about 5–8% of patients with RA where C1 settles on top of C2 leading to dens of C2 to move upwards. Depending on severity of compression, patients may present with neck pain or CSM.

20. What is the expected neurologic outcome after decompression surgery? Numerous studies reveal that the majority of patients either improve or remain neurologically stable following surgical decompression. A study by Lesoin reports only 10% with continued deterioration who were managed operatively and followed for 20 years.[5] Poorer neurologic recovery has been demonstrated in those with greater radiographic canal stenosis (canal area of 30 to 45 mm2). It has also been concluded that age (> 60) and abnormal cervical curvature (lack of normal cervical lordosis) predict less postoperative neurologic improvement. The presence of preoperative high signal intensity within the spinal cord may also reflect less neurologic improvement (T2 hyperintensity at multiple levels or T2 hyperintensity in combination with T1 hypointensity).

2.

Epstein BS, Epstein JA, Jones MD. Cervical spinal stenosis. Radiol Clin North Am. 1977;15:215–226.

3.

Countee RW, Vijayanathan T. Congenital stenosis of the cervical spine: diagnosis and management. J Nat Med Assn. 1979;71:257–264.

4.

Bednarik J, Kadanka Z, Dusek L, et al. Presymptomatic spondylotic cervical cord compression. Spine (Phila Pa 1976). 2004;29(20): 2260–2269.

5.

Lesoin F, Bouasakao N, Clarisse J, Rousseaux M, Jomin M. Results of surgical treatment of

99

Chapter 12: Cervical stenosis and myelopathy

radiculomyelopathy caused by cervical arthrosis based on 1000 operations. Surg Neurol. 1985;23: 350–355.

References 1.

2.

3.

4.

Alexander JT. Natural history and nonoperative management of cervical spondylosis. In Menezes AH, Sonntag VK, eds. Principles of Spinal Surgery. New York: McGraw-Hill Companies, Health Professions Division. 1996: pp. 547–557. Al-Mefty O, Harkey LH, Middleton TH, et al. Myelopathic cervical spondylotic lesions demonstrated by magnetic resonance imaging. J Neurosurg. 1988;68(2):217–222. Arnold JG, Jr. The clinical manifestations of spondylochondrosis (spondylosis) of the cervical spine. Ann Surg. 1955;141:872–889. Baron EM, Young WF. Cervical spondylotic myelopathy: a brief review of its pathophysiology, clinical course, and diagnosis. Neurosurgery. 2007;60(1 Supp1 1): S35–S41.

5.

Bohlman HH. Cervical spondylosis and myelopathy. Instr Course Lect. 1995;44:81–98.

6.

Bohlman HH, Emery SE. The pathophysiology of cervical spondylosis and myelopathy. Spine (Phila Pa 1976). 1988;13(7): 843–846.

7.

Chiles 3rd BW, Leonard MA, Choudhri HF, Cooper PR. Cervical spondylotic myelopathy: patterns of neurological deficit and recovery after anterior cervical decompression. Neurosurgery. 1999;44(4): 762–769.discussion 769–770.

8.

100

Clark C. Degenerative conditions of the cervical spine: differential diagnosis and nonoperative management. In Frymoyer JW, ed. The Adult Spine: Principles and Practice, 2nd edn. Philadelphia:

Lippincott-Raven. 1997: pp. 1323–1348. 9.

Clark CR. Cervical spondylotic myelopathy: history and physical findings. Spine. 1988;13:847–849.

10. Collins DN, Barnes CL, FitzRandolph RL. Cervical spine instability in rheumatoid patients having total hip or knee arthroplasty. Clin Orthop. 1991;272:127–135. 11. Connell MD, Wiesel SW. Natural history and pathogenesis of cervical disk disease. Orthop Clin North Am. 1992;23(3):369–380. 12. El-Khoury GY, Wener MH, Menezes AH, Dolan KD. Cranial settling in rheumatoid arthritis. Radiology. 1980;137(3):637–642. 13. Emery SE. Cervical spondylotic myelopathy: diagnosis and treatment. J Am Acad Orthop Surg. 2001;9(6):376–388. 14. Emery SE, Bohlman HH, Bolesta MJ, Jones PK. Anterior cervical decompression and arthrodesis for the treatment of cervical spondylotic myelopathy: two to seventeen-year follow-up. J Bone Joint Surg [Am]. 1998;80(7): 941–951. 15. Epstein JA, Epstein NA. The surgical management of cervical spinal stenosis, spondylosis, and myeloradiculopathy by means of the posterior approach. In The Cervical Spine Research Society Editorial Committee, ed. The Cervical Spine, 2nd edn. Philadelphia, J.B. Lippincott. 1989: pp. 625–643. 16. Epstein N, Epstein J, Carras R. Cervical spondylostenosis and related disorders in patients over 65: current management and diagnostic techniques. Orthotransactions. 1987;11:15. 17. Fouyas IP, Statham PF, Sandercock PA. Cochrane review on the role of surgery in cervical spondylotic radiculomyelopathy. Spine (Phila Pa 1976). 2002; 27(7):736–747.

18. Fujiwara K, Yonenobu K, Ebara S, et al. The prognosis of surgery for cervical compression myelopathy: an analysis of the factors involved. J Bone Joint Surg [Br]. 1989;71(3): 393–398. 19. Hayashi H, Okada K, Hamada M, et al. Etiologic factors of myelopathy. A radiographic evaluation of the aging changes in the cervical spine. Clin Orthop. 1987;214:200–209. 20. Heller JG, Edwards 2nd CC, Murakami H., Rodts GE. Laminoplasty versus laminectomy and fusion for multilevel cervical myelopathy: an independent matched cohort analysis. Spine (Phila Pa 1976). 2001; 26(12):1330–1336. 21. Herkowitz HN. The surgical management of cervical spondylotic radiculopathy and myelopathy. Clin Orthop. 1989;239:94–108. 22. Hirabayashi K, Satomi K. Operative procedure and results of expansive open-door laminoplasty. Spine. 1988;13: 870–876. 23. Hirabyashi K, Bohlman HH. Multilevel cervical spondylosis: Laminoplasty versus anterior decompression. Spine. 1995;20:1732–1734. 24. Hiroshima K, Ono K, Fujiwara K. Pathology of cervical spondylosis, spondylotic myelopathy, and similar disorders: is clinicopathological correlation verified? In Ono K, Dvorak J, Dunn E, eds. Cervical Spondylosis and Similar Disorders. Singapore, New Jersey, London, Hong Kong: World Scientific. 1998: pp. 89–139. 25. Hududa S, Ogata M, Katsura A. Experimental study on acute aggravating factors of cervical spondylotic myelopathy. Spine. 1988;13:15–20. 26. Kaptain GJ, Simmons NE, Replogle RE, Pobereskin L. Incidence and outcome of

Chapter 12: Cervical stenosis and myelopathy

kyphotic deformity following laminectomy for cervical spondylotic myelopathy. J Neurosurg. 2000;93(Suppl 2): 199–204. 27. Kasai Y, Uchida A. New evaluation method using preoperative magnetic resonance imaging for cervical spondylotic myelopathy. Arch Orthop Trauma Surg. 2001;121(9):508–510. 28. Kawaguchi Y, Kanamori M, Ishihara H, et al. Progression of ossification of the posterior longitudinal ligament following en bloc laminoplasty. J Bone Joint Surg. 2001;83A:1798–1802. 29. Law M, Bernhardt M, White AA. Evaluation and management of cervical spondylotic myelopathy. Inst Course Lect. 1995;44:99–110. 30. Lestini WF, Wiesel SW. The pathogenesis of cervical spondylosis. Clin Orthop. 1989;239:69–93. 31. Lipson SJ. Rheumatoid arthritis in the cervical spine. Clin Orthop. 1989;239:121–127. 32. Macdonald RL, Rehlings MG, Tator CH, et al. Multilevel anterior cervical corpectomy and fibular allograft fusion for cervical myelopathy. J Neurosurg. 1997;86:990–997. 33. Matsunaga S, Sakou T. Epidemiology of ossification of the posterior longitudinal ligament. In Yonenobu K, Sakou T, Ono K, eds. Ossification of the Posterior Longitudinal Ligament. Tokyo, Berlin, Heidelberg, New York: Springer. 1997: pp. 11–17. 34. Moore AP, Blumhardt LD. A prospective survey of the causes of non-traumatic spastic paraparesis and tetraparesis in 585

patients. Spinal Cord. 1997;35(6): 361–367. 35. Morio Y, Teshima R, Nagashima H, et al. Correlation between operative outcomes of cervical compression myelopathy and MRI of the spinal cord. Spine (Phila Pa 1976). 2001;26(11): 1238–1245. 36. Morio Y, Yamamoto K, Kuranobu K, et al. Does increased signal intensity of the spinal cord on MR images due to cervical myelopathy predict prognosis? Arch Orthop Trauma Surg 1994;113(5): 254–259. 37. Northover JR, Wild JB, Braybrooke J, Blanco J. The epidemiology of cervical spondylotic myelopathy. Skeletal Radiol. 2012;41:1543–1546 38. Ono K, Ebara S, Fuji T, et al. Myelopathy hand: new clinical signs of cervical cord damage. J Bone Joint Surg [Br]. 1987;69(2): 215–219. 39. Orr RD, Zdeblick TA. Cervical spondylotic myelopathy: approaches to surgical treatment. Clin Ortho Relat Res. 1999;359: 58–66. 40. Ota K, Ikata T, Katoh, S et al. Implications of signal intensity on T1 weighted MR Image on the prognosis of cervical spondylotic myelopathy. Orthop Trans. 1996;20:443. 41. Parke WW. Correlative anatomy of cervical spondylotic myelopathy. Spine. 1988;13: 831–837. 42. Pavlov H, Torg JS, Robie B, Jahre C. Cervical spinal stenosis: determination with vertebral body ratio method. Radiology, 1987;164:771–1775.

43. Penning L, Wilmink JT, van Woerden HH, Knole E. CT myelographic findings in degenerative disorders of the cervical spine: clinical significance. AJR Am J Roentgenol. 1986;146:793–801. 44. Penning L. Some aspects of plain radiography of the cervical spine in chronic myelopathy. Neurology. 1962;12:513–519. 45. Roberts A. Myelopathy due to cervical spondylosis treated by collar immobilization. Neurology. 1966;16:951–954. 46. Sadasivan KK, Reddy RP, Albright JA. The natural history of cervical spondylotic myelopathy. Yale J Biol Med. 1993;66(3):235–242. 47. Satomi K, Hirabayahi K. Ossification of posterior longitudinal ligament. In Herkowitz HN, Eismont FJ, Garvin SR, et al., eds. RothmanSimeone, The spine, 4th edn. Philadelphia: WB Saunders. 1999: pp. 565–580. 48. Snow RB, Weiner H. Cervical laminectomy and foraminotomy as surgical treatment of cervical spondylosis: a follow-up study with analysis of failures. J Spinal Disord. 1993;6:245–251. 49. Uchida K, Nakajima H, Sato R, et al Cervical spondylotic myelopathy associated with kyphosis or sagittal sigmoid alignment: clinical article. J Neurosurg Spine 2009;11(5): 521–528. 50. Yonezawa T, Tsuji H, Matsui H, et al. Subaxial lesions in rheumatoid arthritis: radiological factors suggestive of lower cervical myelopathy. Spine. 1995;20: 208–215.

101

Section 2 Chapter

13

Spinal Disorders

Thoracic outlet syndrome (TOS): an enigma in pain medicine Narendren Narayanasamy and Rahul Rastogi

Case study A 26-year-old female started having strange tingling and dull ache in her left arm. Her symptoms started at 17 years of age during her training to become a beauty therapist. She also noticed numbness in her upper extremities with elevation of arms above the shoulders. She put off these symptoms for a long time attributing them to physical exertion. Upon evaluation, her primary care physician incidentally found significant wasting of the hand muscles with prominent tendons, and decreased sensation of the arm and hand to pin prick.

1. What is thoracic outlet syndrome? Over the last few decades, a constellation of symptoms comprising pain, numbness/tingling, and weakness in the shoulder and upper extremity were addressed with different names such as cervical rib syndrome, scalenus anticus syndrome, costoclavicular syndrome, arm hyperabduction syndrome, etc. It was in 1956 that Pette et al collectively called all these different clinical entities “thoracic outlet syndrome” (TOS). TOS is a myriad of symptoms secondary to compression of the neurovascular bundle consisting of the subclavian vein, subclavian artery, and/or brachial plexus, during their course through the cervicothoracobrachial region into the axilla.

2. What are the types of thoracic outlet syndrome? Due to lack of specific diagnostic criteria, thoracic outlet syndrome remains a controversial and complex clinical diagnosis. It is broadly divided into three categories on the basis of the presence of predominant symptoms from compression of neurovascular bundle in the thoracic outlet; namely arterial (aTOS),

venous (vTOS), and neurogenic (nTOS) TOS. Isolated pure TOS of any type is rare. Most of them are mixed presentation of the above types. There is a group of patients who do not fall under any of the above categories. They are currently classified as disputed or symptomatic TOS (sTOS) with predominant neurogenic symptoms.

3. What is the epidemiology of TOS? Incidence of TOS is essentially unknown, although nTOS is the common (95%) type followed by venous vTOS (2–4%) and aTOS (1–2%) types. TOS usually affects middle aged adults (20–60 years), with the exception of aTOS, which affects both the young and older adults. Gender ratio favors females in nTOS (3:1), and males in vTOS (2:1), while aTOS is gender neutral.

4. What is thoracic outlet? Anatomically the “thoracic outlet” is a narrow passage in the neck, which extends up to axilla that house muscles, subclavian vessels, and the brachial plexus. It is divided into three narrow passages/spaces, namely scalene triangle, costoclavicular space, and subcoracoid/subpectoral space (see Table 13.1, Figure 13.1). Besides functional and traumatic causes, anomalous anatomy of the components of thoracic outlet can exert compressive effects on neurovascular bundle along these tight passages to produce TOS symptoms.

5. What is the clinical classification of the brachial plexus? The brachial plexus is classified on the basis of its relationship to the clavicle: supraclavicular plexus constitutes roots and trunks;

Case Studies in Pain Management, ed. Alan David Kaye and Rinoo V. Shah. Published by Cambridge University Press. © Cambridge University Press 2015.

102

Chapter 13: Thoracic outlet syndrome (TOS): an enigma in pain medicine

Table 13.1. Thoracic outlet spaces

Spaces

Boundaries

Structures

Interscalene triangle

Anterior, middle scalene muscles, and first rib

Subclavian vein runs anterior to anterior scalene muscle Subclavian artery and supraclavicular upper brachial plexus run between 2 scalene muscles

Costoclavicular space

Subcoracoid space

Between clavicle and first rib

Below coracoid process and pectoralis minor tendon

Vessels and supraclavicular upper plexus then runs under subclavius muscle and costoclavicular ligament, and finally under clavicle to enter subcoracoid space Neurovascular bundle passes under tendon of pectoralis minor through this tight space to enter axilla

retroclavicular plexus constitutes divisions; and infraclavicuar plexus constitutes cords and terminal nerve plexi. The supraclavicular plexus is further subdivided into: (a) upper plexus that includes the upper trunk and C5 and C6 nerve root; (b) middle plexus that includes the middle trunk and C7 nerve root; and (c) lower plexus that includes the lower trunk and C8 and T1 nerve roots.

6. Discuss the etiology of TOS There are several causes that lead to the development of TOS symptoms. They can be divided into four categories: 1. Congenital: This includes the presence of accessory cervical rib, prolonged C7 transverse process, muscular anomalies of scalene muscles, anomalous fibrous bands, altered course of neurovascular bundle, etc. 2. Post-traumatic: Bone overgrowth/callus in clavicle or first rib, fibrosis of scalene muscle

C4

C5 MS

C6 AS C7

1 2

T1

3 T2

M

C

V

A

R1 PM R2

Figure 13.1. Thoracic outlet spaces: 1 ¼ scalene triangle, 2 ¼ costoclavicular space, 3 ¼ subpectoral space. A – subclavian artery, AS – anterior scalene, C – clavicle, C4 to C7 – cervical vertebrae, M – manubrium, MS – middle scalene muscle, PM – pectoralis minor, R1 to R2 – first and second ribs, T1 to T2 – thoracic vertebrae, V – subclavian vein.

following soft tissue trauma, whiplash injury, etc. 3. Functional acquired: This is a common cause resulting from hypertrophied muscles due to repetitive movements of outstretched hands and elevation of arm above the shoulder, drooping shoulders from poor posture, prolonged downward shoulder girdle pressure, etc. The patient in this case indulged in repetetive movements with outstretched hands as a beauty therapist that predisposes her to the development of TOS. 4. Other acquired: Pancoast tumor, atherosclerotic plaques formation, inflammatory, and infective fibrosis. Trauma remains the most common cause of TOS.

7. Discuss and differentiate clinical presentations by the type of TOS Isolated TOS with symptoms pertaining to neural, venous, or arterial etiology is rare. True nTOS is

103

Chapter 13: Thoracic outlet syndrome (TOS): an enigma in pain medicine

Table 13.2. Clinical presentation of thoracic outlet syndrome

nTOS

sTOS

aTOS

vTOS

Etiology/compression

Brachial plexus lower trunk, T1 > C8 nerve root

Unknown

Subclavian artery

Subclavian vein

Pain (neck, shoulder, upper extremity, chest, periscapular)

+ Late Early minimal pain

+ Early and late Predominant pain symptoms

Intermittent claudication of shoulder and upper extremity

Fullness of upper extremity Cramping of UE

Sensory – numbness, paresthesia, in medial aspect of arm, forearm, and ulnar distribution of finger

+

+, Along other segmental distribution

–/+

–/+

Motor deficit (lower trunk), thenar weakness, intrinsic hand muscle weakness

+, Presents with chronic progressive UE weakness

+/–

Weakness in UE upon elevation and resolution upon dependency of UE

+/–

Radial artery pulse

+

+



+

UE color changes





Pale

Cyanotic

Swelling







++

Supraclavicular tenderness and Tinel’s sign

+

+

+/–

+/–

Elevation of UE precipitating symptoms

+

+





Occipital headache, vison/hearing changes, facial/ jaw pain

Supraclavicular pulsatile swelling, limb ischemia

Cyanotic shoulder, venous HTN

Other

aTOS, arterial thoracic outlet syndrome; HTN, hypertension; nTOS, neurogenic thoracic outlet syndrome; sTOS, symptomatic disputed thoracic outlet syndrome; UE, upper extremities; vTOS, venous thoracic outlet syndrome.

relatively uncommon (4–5%). Disputed sTOS constitutes the majority of the nTOS (95–96%). In these cases, pain and clinical presentation mimics the established TOS presentation, but lacks a specific identifiable cause for the pain. Lower brachial plexus trunk (C8 and T1) is affected in the majority of TOS patients. However, T2 is rarely involved and presents as retrosternal pain. Upper trunk involvement presents only as shoulder and periscapular symptoms. See Table 13.2 for clinical presentations of TOS.

8. Discuss the differential diagnosis of TOS There are no defined diagnostic criteria for TOS. Due to its variable presentation, other clinical entities

104

could mimic TOS presentation. Hence, it is advisable to be cautious in reaching a diagnosis. Syndromes that present as TOS: 1. Brachial plexopathies (BPs): Usually involve upper plexus C5-C6, which can usually be ruled out with EMG study. 2. Cervical radiculopathies: These arise as a result of cervical spondylosis, metastatic spine disease, or nerve tumors. The symptoms follow specific root distribution. These can be differentiated from TOS using provocation tests, EMG, and imaging of cervical spine. 3. Complex regional pain syndrome of limb: History of injury and presence of sudomotor and vasomotor symptoms with normal spine imaging helps in distinguishing this entity from TOS.

Chapter 13: Thoracic outlet syndrome (TOS): an enigma in pain medicine

Table 13.3. Provocative diagnostic tests for thoracic outlet syndrome

Test

Maneuver

Result

VASCULAR TESTS (positive in 50% of the general population) Adson test

Upon deep breath, ipsilateral upper extremity is extended & supinated with lateral rotation of neck toward test side

Decrease or absence of radial pulse

Allen test

Upon lateral rotation of neck toward opposite side with test side shoulder abducted at 90° with 90° flexion of elbow

Decrease or absence of radial pulse

NON-VASCULAR TESTS Elevated arm stress test of Roos (EAST)

In 90° flexion of elbow and abduction of shoulder (surrender position), patient is asked to repetitively close and open the fist

Precipitates paresthesia and symptoms within 3 minutes

Upper limb tension test of Elvey (ULTT)

In sitting position, abduction and external rotation of shoulder and extended elbow Position 1: then active dorsiflexion of wrist Position 2: then further lateral flexion of neck

Triggering symptoms on ipsilateral upper extremity with position 1 and on contralateral upper extremity with position 2

Halstead test

Elevation of chin with pulling the shoulder joint behind in an extreme “attention” position (military position)

Precipitates symptoms

Morley test

Asymmetrical suprascapular tenderness on palpation

Triggers radiating symptoms in UE

4. Peripheral nerve etiology (nerve lesion and/or entrapment syndromes – cubital/carpal tunnel, neuropathy): Location of pain and neural symptoms, provocative tests, and EMG helps in reaching a precise diagnosis. 5. Apical lung etiology: Infection, pancoast tumor, etc.

9. What is double crush syndrome in TOS? Upton et al suggested that proximal nerve lesion makes the distal nerve more vulnerable to compression, resulting in coexistent proximal and distal nerve symptoms called “double crush syndrome” (DCS). This results in a complex clinical presentation with the development of carpal tunnel syndrome in the presence of pre-existing TOS. Double crush syndrome has poor outcomes from isolated peripheral nerve release surgery like carpal/cubital tunnel release. Thus, simultaneous surgical release of both the compressive etiology of thoracic outlet and peripheral nerve compartments are required to achieve an optimal outcome. In DCS there is a sudden decompensation of TOS symptomatology, and/or rapid occurrence of symptoms

of peripheral nerve entrapment neuropathies. DCS is thought to affect mainly C8 fibers; however the median nerve sensory fibers (in carpal tunnel syndrome) do not follow this path. Therefore other possible etiologies should be evaluated to explain DCS, i.e., central mechanisms or more proximal compression of these fibers as in scalene syndrome.

10. Discuss the relevant physical examination findings in TOS The general physical exam findings in patients with TOS include decreased function of upper extremity, lowered and protracted position of the shoulder, contracted scalene and scapular muscles, and wasting and weakness of the intrinsic muscles of the hand with prominent tendons. Additionally, these signs could accompany supraclavicular fullness due to cervical rib and/or aneurismal pulsations. Vascular TOS in particular may also present with cyanosis, edema, and collaterals of the upper extremity. Several provocative tests were suggested for TOS, and are broadly divided into vascular and non-vascular tests (Table 13.3). Vascular tests lack specificity. Among

105

Chapter 13: Thoracic outlet syndrome (TOS): an enigma in pain medicine

non-vascular provocative tests, the upper limb tension test (ULTT) of Elvey is pathognomic for TOS.

11. What are the diagnostic modalities of TOS? There is no specific diagnostic test used for diagnosis of symptomatic TOS. These tests are mainly used to rule out differential diagnosis, and to evaluate anatomical variations for possible surgery. Thus, comprehensive history and physical exam are the basis of diagnosis of TOS. Tests commonly in use are: A. Chest, shoulder, and cervical spine imaging (plain radiographs, CT scan, and/or MRI): These imaging techniques help to identify anatomical defects and variations, i.e., hypertrophic callus from old fracture, accessory ribs, fibrous scalene bands, compressive tumors, spinal disc, or canal pathologies, etc. B. Vascular imaging (CT or MR angiography, duplex ultrasound): These imaging techniques are helpful in the presence of vascular clinical signs and symptoms to delineate vascular pathologies and their differential. These diagnostic tests are sometimes essential in surgical decision making. C. Neural imaging (MR neurography): Direct visualization of neural structures under MRI including anatomical relationship of nerves, nerve injury, and inflammation aid in diagnosis of TOS. D. Nerve conduction and electromyographic study (EMG): sTOS, the commonest form of TOS, comprises more than 90% of total TOS. In these patients EMG findings are normal or non-specific, but presence of partial denervation of intrinsic muscles of hand and absent or decreased amplitude of action potential in the ulnar and medial antebrachial cutaneous sensory nerve and decreased amplitude of the median nerve compound motor action potential suggest neurogenic TOS. The above findings along with normal sensory median nerve action potential essentially rules out cervical radiculopathy and myelopathy, and are highly suggestive of lower trunk (C8-T1) compression of the brachial plexus. E. Anterior scalene muscle (ASM) block: In patients with suspected nTOS caused by ASM pathology, a local anesthetic injected into the ASM produces temporary relaxation of the muscle relieving

106

tension on the involved nerve resulting in improvement of symptoms. Ninety-four percent of patients who have a positive block and who subsequently undergo surgery were shown to have a positive outcome compared to only 50% who underwent surgical correction following a failed block.

12. What are the conservative management strategies of TOS? Vascular TOS usually needs a surgical intervention. Symptom monitoring is rarely suitable in lieu of active surgical options in vascular TOS. However, neurogenic TOS treatment strategies remain controversial. A multimodal treatment approach should be incorporated in a patient’s regimen. These include: A. Physiotherapeutic and occupational rehabilitation. Rehabilitative approach should be recommended to all neurogenic TOS patients with the goal to improve function, ergonomical changes at workplace, and correction of cervicobrachial structural musculoskeletal imbalance resulting in dysfunction. Modalities include physical therapy (active stretching exercises, mobilization techniques, muscular tapping and postural correction exercises, etc.), weight loss, life style changes, and improving work place ergonomics. B. Pharmacologic therapies. Medical management is essentially directed toward symptom management. Neuropathic pain is the primary and debilitating symptom, which is managed by use of analgesics (non-steroidal anti-inflammatory agents and/ or opioids). Use of adjuvants to manage pain is routinely utilized like muscle relaxants (i.e., methocarbamol, baclofen, tizanidine) and/or anticonvulsants (i.e., gabapentin, pregabalin, etc.), and/or antidepressants (i.e., amitriptyline, nortriptyline, cymbalta, etc.). In vascular TOS anticoagulant medications, i.e., warfarin, clopidogrel, etc., are part of the treatment regimen to prevent clot formation and patency of vessels. See Table 13.4. C. Injection therapies. Use of early thromboembolic therapy is considered in vascular TOS when thrombi are considered an etiology. For relieving painful myofascial symptoms in TOS patients,

Chapter 13: Thoracic outlet syndrome (TOS): an enigma in pain medicine

Table 13.4. Common medications utilized for chronic pain management

Drugs/class

Mechanism

Concern

Dosages

ANALGESICS Acetaminophen

Unknown

Liver damage

325–4000 mg/d

Non-steroidal antiinflammatory drugs

Ibuprofen Naproxen Meloxicam Celecoxib

Decrease prostaglandins by inhibiting cyclooxygenase

GI irritation Renal effects Bleeding

200–2400 mg/d 250–1500 mg/d 7.5–15 mg/d 100–400 mg/d

Opioids

Hydrocodone Fentanyl Morphine Oxycodone Methadone Tramadol Tapentadol

Agonist to opioid receptors Tramadol – additional SSRI and NRI effect Tapentadol – additional NRI action

Nausea/vomiting, constipation, drowsiness, respiratory depression Methadone – variable long t1/2 Tramadol/tapentadol – caution with antidepressants

Variable Variable Variable Variable Variable 50–400 mg/d 50–600 mg/d

Tricyclics (amitriptyline, nortriptyline, desipramine) SNRI (duloxetine)

Modulation of neurotransmission of serotonin and norepinephrine

Sedation, tachycardia, urinary hesitancy Weight gain

25–150 mg HS

Gabapentin Pregabalin Lamotrigine Carbamazepine

α2δ subunit of voltagegated N-type Ca2+ channel modulation Na+ channel blocker Na+ channel blockade

Drowsiness, confusion, weight gain, rash LFT monitoring for carbamazepine

Baclofen

Selective GABA-b agonist

Confusion, drowsiness, dizziness

ADJUVANT ANALGESICS Antidepressants

Antiepileptics

Muscle relaxants

Cyclobenzaprine Methocarbamol

20–90 mg/d

Unknown action on CNS

300–3600 mg/d 75–600 mg/d 100–600 mg/d 200–1200 mg/d 10–80 mg/d 2–32 mg/d 10–60 mg/d 500–2000 mg/d

Alpha-2 adrenergics

Clonidine Tizanidine

α2 adrenergic agonism

Drowsiness, #BP, #HR LFT caution – tizanidine

01–0.3 TD patch 2–32 mg/d

Local anesthetic

Mexelitine

Na+ channel blocker

Liver toxicity, #BP

150–900 mg/d

Corticosteroids

Methyl prednisone Dexamethasone

Hyperglycemia, weight gain, edema, agitation

Variable 4–96 mg/d

BP, blood pressure; Ca2+, calcium; CNS, central nervous system; GABA, gamma aminobutyric acid; GI, gastrointestinal; HS, bedtime; HR, heart rate; LFT, liver function test; mg/d, milligram per day; Na+, sodium; NRI, norepinephrine reuptake inhibition; SSRI, selective serotonin reuptake inhibition; t1/2, half-life; TD, transdermal.

injection of local anesthetic and/or botulinum toxin into anterior scalene and/or pectoralis muscle can produce the desired relief, although it is only short lasting. Trigger point injections to the surrounding musculature involved are utilized for breakthrough pain control.

13. Discuss surgical management strategies of TOS Patients with a definitive etiology and who have failed conservative management are ideal candidates for surgical correction. The three surgical approaches

107

Chapter 13: Thoracic outlet syndrome (TOS): an enigma in pain medicine

include a supraclavicular, transaxillary, and posterior surgical approach. A supraclavicular approach is preferred for resection of a cervical rib along with scalenectomy, while some surgeons choose a transaxillary approach for first rib and scalene muscle resection. The posterior approach is technically challenging and utilized when the above two methods have failed to relieve symptoms. A dorsal sympathectomy is also considered in these patients. A vascular surgeon is usually involved to surgically correct vascular deformities and/or compromise to restore circulation to the affected extremity. Regardless of the surgical technique used, a major factor that is shown to prevent patients from going back to work is a pre-existing poor psychosocial working condition.

14. What are the treatment outcomes for TOS? Vascular TOS operated early has been shown to produce excellent results. By the disability of the shoulder, arm, and hand (DASH) questionnaire, 80% of patients reported minimal disability for up to 10 years. nTOS is challenging to diagnose and treat. Patient selection, comorbidities, baseline functional status, and surgical expertise and the type of surgical approach seem to play a role in the outcome of this pathology. Generally, demonstrating a definitive etiology during surgery is correlated with 80% success

References

15. What could be the impact of TOS on a person’s life? TOS can be a life-altering debilitating disease upon failure of standard treatment strategies. Many of the patients with TOS are physically active prior to the onset of symptoms, and are accustomed to an independent life style. Because of the decline in their functional status, they frequently need comprehensive support in all aspects of life. They may require psychologic counseling and vocational rehabilitation as they adjust to cope with the changes that result from their physical inability and occupational limitation.

Conclusion This case serves to emphasize that thorough clinical examination and appropriate clinical testing is critical in identifying TOS. More importantly, raising awareness about this entity could help practitioners consider this pathology in their differential diagnosis for shoulder and upper extremity pain and/or paresthesia symptoms so that their patients are directed appropriately to get timely interventions. documentation of brachial plexus/ thoracic outlet compression during elevated arm stress testing. Hand, American Association of Hand Surgery, online version published 4 May 2013.

definition, aetiological factors, diagnosis, management and occupational impact. J Occup Rehabil. 2011;21(3): 366–373.

1.

Klaassen Z, Sorenson E, Tubbs RS, et al. Thoracic outlet syndrome: a neurological and vascular disorder. Clin Anat. 2014;27(5):724–732.

2.

Ozoa G, Alves D, Fish DE. Thoracic outlet syndrome. Phy Med Rehabil Clin N Am. 2011;22(3):473–483.

4.

3.

Laulan J, Fouquet B, Rodaix C, et al. Thoracic outlet syndrome:

5.

108

rate. However, recurrence due to scar tissue is commonly seen within 6 months of surgery. Further, late recurrence due to a different pathology makes this type of TOS extremely difficult to manage and treat.

Thompson JF. Thoracic outlet syndrome (Review). Surgery, 2013;31(5):256–260. Fried S, Nazarian LN. Dynamic neuromusculoskeletal ultrasound

6.

Winterton R, Farnell R. Peripheral nerve entrapment syndromes of the upper limb. Surgery. 2013; 31(4):172–176.

Section 2 Chapter

14

Spinal Disorders

Patient with cervical radiculopathy Robert B. Bolash and Jianguo Cheng

Case description A 52-year-old male presented with neck, peri-scapular, and left upper extremity shooting pain associated with paresthesias in the forearm, hand, and third-fifth fingers without any clear provocative event. His complaints were associated with subjective weakness in the wrist flexors.

1. Provide a differential diagnosis for the patient’s complaint      

Cervical radiculopathy Zygapophysial (facet) joint syndrome Brachial plexopathy Thoracic outlet syndrome Pancoast tumor Shoulder pathology including tendonitis, impingement syndromes, rotator cuff tears, adhesive capsulitis, gleno-humeral arthritis  Peripheral nerve entrapment  Sympathetically mediated pain syndromes

2. Who is affected by cervical radiculopathy? Cervical radiculopathy refers to a set of conditions in which one or more nerve roots in the cervical region are affected and do not function properly. The emphasis is on the nerve root involvement that results in pain (radicular pain), weakness, and numbness in specific areas of the arms and hands. It has an approximate annual incidence of 0.1% and affects men nearly twice as often as women. The peak age of onset is in the fifth decade of life. While patients often cite a motor vehicle accident as the cause of their cervical radicular symptoms, this association is

not supported by large epidemiologic studies. Trauma is present in only 15% of patients suffering from radiculopathy, and the absence of an inciting event is consistent with the pathophysiologic mechanism; namely that radicular symptoms are caused by progressive degenerative changes more often than an acute disc herniation.

3. Describe the likely anatomical causes of cervical radiculopathy and the pathophysiologic mechanisms for the observed symptoms There are seven cervical vertebrae and eight pairs of cervical spinal nerves. The cervical nerve roots are numbered to correspond with the vertebral body caudal to the level where the nerve exits the foramen. The C1 nerve root exits above the vertebral body of C1 while the C8 nerve root exits the foramen created by the posterior elements of the vertebral body of C7 and T1. Most cases of cervical radiculopathy are caused by encroachment upon the intervertebral foramen by hypertrophic zygapophysial and uncovertebral joints. Stress on these posterior joints can be accelerated by progressive loss of disc height, though disc herniation is not thought to be the predominant pathophysiologic entity. In contrast to the proposed mechanisms for the development of lumbar radiculopathy, only 22% of the cases of radiculopathies in the cervical region result from herniation of the cervical disc. Hypertrophy of the facet (zygapophysial) joints narrow the intervertebral foramen and cause compressive and inflammatory changes of the nerve root. These changes have a propensity to involve the lower cervical spine with the C7 nerve root most often affected (70% of cases), followed by the C6 (20%) nerve root.

Case Studies in Pain Management, ed. Alan David Kaye and Rinoo V. Shah. Published by Cambridge University Press. © Cambridge University Press 2015.

109

Chapter 14: Patient with cervical radiculopathy

Deficits on clinical examination can assist in predicting the affected cervical nerve root (Table 14.1). Compression of the nerve root by the overgrowth of the bony elements is thought to cause compression and hypoxia of both the nerve root and dorsal root Table 14.1. Deficits on clinical examination can assist in predicting the affected cervical nerve root

Effected root

Motor group deficit

Sensory deficit

Reflex decrement

C5

Shoulder abductors

Lateral upper arm

Supinator reflex

C6

Elbow flexion

Thumb

Biceps reflex

C7

Wrist flexion

Third digit

Triceps reflex

C8

Thumb flexors

Fifth digit

None

Hypertrophy of uncovertebral joint

Spinal ganglion

Hypertrophy of zygapophysial joint

Herniation of nucleus pulposus

Spinal nerve

Superior articular process

Spinal cord

Cervical vertebra

110

ganglion which can result in pain in the corresponding dermatome. Because the cervical nerve roots emerging from the foramen are mixed, both sensory symptoms such as pain or paresthesia, as well as motor symptoms such as hyporeflexia or weakness can occur. Herniation of the nucleus pulposus, if present, occurs when the tough outer disc annulus fractures resulting in protrusion or extrusion of the gelatinous contents of the nucleus pulposus through the annular defect. The posterior longitudinal ligament does not extend very far laterally in the cervical region providing a path of low resistance for the nuclear contents to travel posterolateral toward the neural structures. Radicular symptoms develop if the disc contents mechanically compress or chemically irritate the nerve root, resulting in inflammatory changes in and around the nerve roots. Inflammatory mediators including interleukins and prostaglandins result in Figure 14.1. Cervical vertebrae and associated neural elements. Cervical radiculopathy can develop when the nerve root is compressed by the development of an osteophyte at the uncovertebral or zygapophysial joint, or when a disc hernation compresses the nerve as it exits the intervertebral foramen. From Carette S, Fehlings MG. N Engl J Med. 2005;353:392–399, with permission.

Chapter 14: Patient with cervical radiculopathy

edema and worsen the local compression. In the presence of inflammatory mediators, the nerves of nociception are more easily activated due to peripheral sensitization. As a general rule, a history consistent with acute worsening of symptoms is more consistent with disc herniation, while a gradual progression suggests a bony overgrowth as the etiology. A minority of cervical radiculopathies are attributable to noncompressive causes such as infection, granulomatous infiltrate, or demyelinating disorders. These etiologies tend not to be confined to a single nerve root, instead presenting with symptoms involving multiple levels. The natural history of non-compressive etiologies tends to be dictated by the response to treatment of the underlying condition.

4. What factors from the history and physical examination support the diagnosis? Cervical radiculopathy presents as neck and arm pain described as burning or tingling and distributed in a myotomal or dermatomal pattern. The radicular syndrome is sometimes associated with the loss of stretch reflexes in the distribution of the affected nerve root. Subjective weakness is seen in the minority of cases. At times, clinically isolating the affected nerve root by history or physical exam maneuvers is often difficult because of overlap of the cervical dermatomes. Myotomal deficits may have greater diagnostic value than sensory dermatomes. Clinical diagnosis can be further complicated by the observation that there is some anatomic variation in the innervation of the upper extremity. Frame shift variants of the cervical innervation by one vertebral level can be present in some individuals complicating the clinical exam. Physical exam maneuvers for the diagnosis of cervical radiculopathy are centered on assessing the response to either provoking or alleviating tension on the neural elements at the level of the nerve root, or opening or narrowing the neural foramen. At least four tests have been described and have some evidence to support their use: (1) Spurling’s maneuver, (2) shoulder abduction, (3) Valsalva maneuver, and (4) traction/neck distraction. The Spurling’s maneuver is performed in the sitting position. The patient extends the neck and rotates and laterally bends the head toward the symptomatic

side; an axial compression force of approximately 7 kg is then applied by the examiner through the top of the patient’s head; the test is considered positive when the maneuver elicits the typical radicular arm pain. Pain in the neck without radicular symptoms is a negative result. Rotation narrows the intervertebral foramen while neck extension worsens disc bulge. To perform the shoulder abduction relief test, the practitioner asks the patient to place the hand of the affected side atop their head in the seated position and assess whether there is any change in the radicular symptoms. A positive test results in decrease or elimination of the radicular symptoms with the maneuver. Some patients may have discovered this on their own and assume a hand-atop-head position when reclining and provide this information on history. The Valsalva maneuver requires that the patient take a deep breath and exhale forcefully against a closed upper airway. The resultant increase in intrathecal pressure worsens nerve root compression when space-occupying lesions such as an osteophyte or herniated disc are present. The traction/neck distraction test is performed with the patient in a comfortable supine position. The physician then grasps the head by placing one hand under the occiput and another beneath the chin and applying 15 kg of axial traction force. A positive test will result in a decrease or elimination of the radicular symptoms. The Spurling’s, Valsalva, and traction/neck distraction tests all demonstrated high specificity and low sensitivity when reviewed with objective testing. Overall, a positive test is helpful in establishing the diagnosis of cervical radiculopathy, but a negative test has low utility in narrowing the differential diagnosis.

5. What is the value of diagnostic testing and imaging in evaluating a patient with cervical radiculopathy? Given that cervical radiculopathy is a clinical diagnosis, a careful history and physical exam is the most important diagnostic tool. The ability to make the diagnosis based on history alone has been reported as high as 75%. The prevalence of asymptomatic disc bulge can be seen in over half of all patients without complaints of neck pain undergoing imaging studies for other indications. Therefore, radiographic imaging

111

Chapter 14: Patient with cervical radiculopathy

is a poor screening test. Despite these observations, imaging or electrodiagnostic modalities can support the clinical diagnosis and remains useful in differentiating cervical radiculopathy from other disorders and in planning a surgical approach. Plain radiographs are of limited utility in the assessment of cervical radiculopathy. The inability to detect tumor, infection, disc herniation, or cord compression limits the utility of plain radiographs. They are reserved for patients with traumatic injury and are often obtained as part of a flexion/extension series to evaluate patients with spondylolisthesis. Cervical radiculopathy due to compressive etiologies is best evaluated with non-contrast MRI. Though soft tissue structures are well visualized, bony abnormalities may be underestimated when compared to CT. In the presence of clinical concern for non-compressive etiologies, MRI with gadolinium contrast is also indicated. There are no well-established guidelines dictating when to order an MRI, but most would agree that clinical suspicion for tumor or infection, as well as progressive symptoms are indicators for obtaining imaging. Neuroimaging is recommended when persistent symptoms do not resolve after 6 weeks of conservative treatment or when significant neurologic symptoms such as weakness or myelopathy are present. CT can be useful in detecting osteophytes or assessing the bony architecture in clinical situations where an MRI is not easily obtainable. While easily able to visualize bony structures, CT alone has limited ability to visualize soft tissue structures. CT myelography combines traditional CT with the administration of intrathecal contrast. Myelography provides assessments similar in quality to those obtained with MRI, but requires a dural puncture and carries the risks associated with interventional procedures. For these reasons, it has fallen out of favor as an initial imaging modality and is instead reserved for those individuals who are unable to undergo MRI. CT myelography may also offer better sensitivity for foraminal and bony abnormalities when clinical signs are discordant with MRI findings. Non-contrast cervical MRI can be normal in noncompressive etiologies of cervical radiculopathy and may prompt investigation with electrodiagnostic studies. Upper extremity electrodiagnostic studies can also be performed when differentiating cervical radiculopathy from other neurologic conditions is difficult. Estimated sensitivity of electrodiagnostic testing is thought to be as high as 50–71%.

112

Nerve conduction studies (NCS) are accompanied by needle electromyography (EMG) in both the muscles of the arm and posterior neck. NCS are most useful to rule out peripheral nerve entrapment syndromes or diffuse peripheral neuropathies while EMG is useful to characterize abnormalities that occur in a myotomal distribution. Spontaneous single muscle fiber action potentials are seen in the presence of ongoing axonal loss of motor neurons. Fibrillation potentials and positive sharp wave potentials develop subacutely after muscle denervation occurs, and are considered to be most sensitive for establishing the diagnosis of cervical radiculopathy. Nerve fibers innervating the unaffected muscles slowly sprout to supply the denervated muscle groups and demonstrate longer and larger motor units on needle EMG. In order to capture this phenomenon, EMG is therefore often delayed until the symptoms are present for more than 3 weeks. Laboratory testing is non-specific for mechanical causes of radicular pain. C-reactive protein and erythrocyte sedimentation rate can be obtained when considering infectious and neoplastic processes, though these tests are non-specific and nearly universally followed by imaging modalities.

6. What is the efficacy of non-surgical treatment strategies? Analgesics, traction, manipulation, physical therapy, cervical collars, and massage are among the most commonly employed conservative strategies despite variable evidence supporting their superiority over a wait-and-see approach. Oral analgesics including non-steroidal anti-inflammatories, membrane stabilizers, muscle relaxants, corticosteroid tapers, and opioids have all been utilized without consistent evidence supporting one agent over another. Despite the lack of a significant body of supportive evidence, these agents are typically well accepted by patients and are often employed as the first-line treatment approach. Cervical traction applies a distracting force to the neck to relieve the pressure on the cervical nerve roots by increasing the joint spaces and relaxing the cervical paraspinous musculature. Despite its widespread use, several studies and a systematic review have failed to consistently demonstrate efficacy of this modality, and some have shown the therapy may actually worsen symptoms.

Chapter 14: Patient with cervical radiculopathy

Both low velocity low amplitude and high velocity low amplitude manipulation have been shown to reduce radicular symptoms and improve pain scores in cervical radiculopathy patients. MRI obtained after a course of chiropractic therapy demonstrated a reduction in disc herniation in 63% of treated patients, the majority of whom were able to return to work. Risks with chiropractic manipulation include worsening of neurologic symptoms and rare catastrophic vascular events. In considering patients to exclude from chiropractic therapy, pre-manipulation imaging can be obtained to exclude those at increased risk. In a randomized control trial, both a short-term semi-hard cervical collar and physical therapy program have been shown to be more effective than a wait-and-see approach at 6-week follow-up. This leads the clinician to question whether it is best to counsel patients to increase activity or decrease mobility. In another study, manual therapy combined with exercise physical therapy (PT) is superior to either modality alone. The effectiveness of PT is seen at 6 weeks but is no better than a wait-and-see intervention at 6 months follow-up. Similarly the outcomes with a cervical collar are no better at 6-month follow-up than those that employed a strategy promoting mobility. Despite the conflicting evidence, it is typically recommended that patients with cervical radiculopathy commence a formal physical therapy program shortly after presentation given the minimal risk, though a cervical collar seems to be equally effective and is likely less costly.

7. Describe the benefits and risks of interventional strategies to treat cervical radiculopathy Prospective studies have demonstrated the effectiveness of epidural steroid injections for the treatment of cervical radiculopathy resulting in resolution of upper extremity complaints in 40–75% of patients. Intramuscular placement was compared to epidural injection and the effectiveness of the corticosteroid was site specific. Both interlaminar and transforaminal approaches have been employed, but there has been no head-to-head study of the superiority of one approach over the other in the cervical region. However, cervical transforaminal injection of particulate steroids has particularly been linked to catastrophic

complications and is therefore generally not recommended. Although the effects of epidural steroid injection are usually short lived, complete or near complete pain relief can be sustained for more than 3 years at long-term follow-up in some cases. Diagnostic selective nerve root blocks can be performed to isolate a single nerve root when clinical and radiographic diagnosis remains uncertain. Using fluoroscopy and radiographic contrast material, 0.5 ml of local anesthetic is injected along the suspected nerve root and pain is assessed postprocedurally. If pain persists, the intervention can be carried out at another level until the affected nerve root is identified. This strategy will offer only shortterm relief, but may be useful in planning a surgical intervention, especially when multilevel or discordant pathology is present on neuroimaging. Reported complications of cervical epidural injections include brainstem or spinal cord ischemia due to vasospasm, infarction due to the inadvertent intravascular injection of particulate steroid, and unrecognized intrathecal injection of local anesthetic. At least 15 fatal case reports with a transforaminal approach have been noted. To mitigate these risks, the following are generally recommended: the use of skilled practitioners, fluoroscopic guidance, an interlaminar approach, and the avoidance of general anesthesia. Though not in widespread use, pulsed radiofrequency neurotomy of the dorsal root ganglion has been shown to result in a sustained decrease in pain scores at 3-month follow-up. Neuritis, paresthesia, and motor weakness have all been reported following radiofrequency ablation. Despite its clinical application, spinal cord stimulation has not been well studied for cervical radiculopathy.

8. Discuss the role for a surgical consultation Symptoms of gait disturbances, lower extremity symptoms, sphincter incontinence, hyperreflexia, hypertonia, clonus, spasticity, and clumsiness suggest myelopathy and should prompt a surgical referral. Myelopathy can often be progressive and irreversible and therefore warrants early consideration for surgical decompression. Fever, chills, or weight loss suggest non-compressive etiologies including neoplastic or infectious causes and require prompt evaluation and treatment of the underlying etiology.

113

Chapter 14: Patient with cervical radiculopathy

Surgical consult is recommended if the patients with MRI evidence of nerve root compression fail to respond to multiple conservative and interventional approaches in 3 months. Three-quarters of patients typically obtain significant relief of their radicular symptoms following surgical decompression and this is sustained at 2-year follow-up. The presence of preoperative EMG abnormalities correlates with a higher likelihood of a successful surgical outcome. Discectomy and corpectomy can be performed via an anterior approach and can employ insertion of a synthetic disc spacer or bone graft. The anterior approach minimizes manipulation of the spinal cord and permits removal of both disc material and osteophytes. Laminectomy, foraminotomy or laminoplasty require posterior surgical approaches and are typically employed when a single paramedian disc herniation is present. Intervertebral fusion can be performed via either anterior or posterior approaches. Specific surgical complications include non-union, dysphagia, nerve root or spinal cord injury, and increased symptoms including worsening pain.

9. Compare the anticipated outcomes with conservative, interventional, and surgical approaches Between 40 and 90% of patients will experience resolution of cervical radicular symptoms with conservative

References 1.

2.

3.

114

Arnasson O, Carlsson CA, Pellettieri L. Surgical and conservative treatment of cervical spondylotic radiculopathy and myelopathy. Acta Neurochir (Wien). 1987;84:48–53. BenEliyahu DJ. Magnetic resonance imaging and clinical follow-up: study of 27 patients receiving chiropractic care for cervical and lumbar disc herniations. J Manipulative Physiol Ther. 1996;19: 597–606. Boden SD, McCowin PR, Davis DO, et al. Abnormal magneticresonance scans of the cervical spine in asymptomatic subjects: a prospective investigation. J Bone Joint Surg Am. 1990;72(8): 1178–1184.

4.

5.

6.

7.

measures alone advocating the importance of patient counseling and reassurance. Larger and more lateral disc herniations, when they occur, tend to exhibit a greater degree of regression. Five predictors of poor outcome have been defined: (1) multiple episodes of radicular symptoms for more than 5 years, (2) more than three episodes of radiculopathy, (3) bilateral symptoms, (4) women over age 50, and (5) symptoms which are worsening at the time of presentation. There are few head-to-head trials comparing outcomes with various treatment modalities. When randomized to treatments with a hard collar, physical therapy, or surgical intervention, the surgical group obtained the most significant improvement in pain reduction at 3-month follow-up. When again assessed at 1-year or 2-year follow-up, there were no differences between the surgery group and the conservative treatment groups. Conservative treatment strategies are preferred since most patients will be asymptomatic or only suffer minor limitations at long-term follow-up. It is reasonable to anticipate that one-third of patients will suffer a reoccurrence of radicular symptoms following initial resolution. We counsel patients to again follow a conservative stepwise approach and employ those strategies they found useful in the past if their symptoms return.

CM Bono CM, Ghiselli G, Gilbert TJ, et al. An evidence-based clinical guideline for the diagnosis and treatment of cervical radiculopathy from degenerative disorders. Spine J. 2011;11:64–72. British Association of Physical Medicine. Pain in the neck and arm: a multicentre trial of the effects of physiotherapy, arranged by the British Association of Physical Medicine. Br Med J. 1966;1:253–258. Bush K, Hillier S. Outcome of cervical radiculopathy treated with periradicular/epidural corticosteroid injections: a prospective study with independent clinical review. Eur Spine J. 1996;5:319–325. Carette S, Fehlings MG. Clinical practice: Cervical radiculopathy. N Engl J Med. 2005;353:392–399.

8.

Cicala RS, Thoni K, Angel JJ. Long-term results of cervical epidural steroid injections. Clin J Pain. 1989;5:143–145.

9.

De Hertogh WJ, Vaes PH, Vijverman V, et al. The clinical examination of neck pain patients: the validity of a group of tests. Man Ther. 2007;12:50–55.

10. Graham N, Gross AR, Goldsmith C. Mechanical traction for mechanical neck disorders: a systematic review. J Rehabil Med. 2006;38:145–152. 11. Kuijper B, Tans JT, Beelen A, et al. Cervical collar or physiotherapy versus wait and see policy for recent onset cervical radiculopathy: randomised trial. BMJ. 2009;339:b3883. 12. Martin GM, Corbin KB. An evaluation of conservative

Chapter 14: Patient with cervical radiculopathy

treatment for patients with cervical disk syndrome. Arch Phys Med Rehabil. 1954;35:87–92. 13. Persson L, Karlberg M, Magnusson M. Effects of different treatments on postural performance in patients with cervical root compression: A randomized prospective study assessing the importance of the neck in postural control. J Vestib Res. 1996;6:439–453. 14. Persson LC, Carlsson CA, Carlsson JY. Long-lasting cervical radicular pain managed with surgery, physiotherapy, or a cervical collar: A prospective, randomized study. Spine. 1997;22:751–758. 15. Radhakrishnan K, Litchy WJ, O’Fallon WM, et al. Epidemiology of cervical radiculopathy: A population-based study from Rochester, Minnesota, 1976 through 1990. Brain. 1994;117:325–335. 16. Rodine RJ, Vernon H. Cervical radiculopathy: a systematic review

on treatment by spinal manipulation and measurement with the Neck Disability Index. J Can Chiropr Assoc. 2012;56: 18–28.

21. Valtonen EJ, Kiuru E. Cervical traction as a therapeutic tool: A clinical analysis based on 212 patients. Scand J Rehabil Med. 1970;2:29–36.

17. Rubinstein SM, Pool JJ, van Tulder MW, et al. Systematic review of the diagnostic accuracy of provocative tests of the neck for diagnosing cervical radiculopathy. Eur Spine J. 2007;16:307–319.

22. van der Heijden GJ, Beurskens AJ, Koes BW, et al. The efficacy of traction for back and neck pain: a systematic, blinded review of randomized clinical trial methods. Phys Ther. 1995;75:93–104.

18. Shabat S, Leitner Y, David R, et al. The correlation between Spurling test and imaging studies in detecting cervical radiculopathy. J Neuroimaging. 2012;22:375–378.

23. van Kleef M, Liem L, Lousberg R. Radiofrequency lesion adjacent to the dorsal root ganglion for cervicobrachial pain: a prospective double blind randomized study. Neurosurgery. 1996;38:1127–1131.

19. Tong HC, Haig AJ, Yamakawa K. The Spurling test and cervical radiculopathy. Spine. 2002;27:156–159. 20. Vallée JN, Feydy A, Carlier RY, et al. Chronic cervical radiculopathy: lateral-approach periradicular corticosteroid injection. Radiology. 2001;218:886–892.

24. Van Zundert J, Huntoon M, Patijn J, et al. Cervical radicular pain. Pain Pract. 2010;10:1–17. 25. Young IA, Michener LA, Cleland JA, et al. Manual therapy, exercise, and traction for patients with cervical radiculopathy: a randomized clinical trial. Phys Ther. 2009;89(7):632–642.

115

Section 2 Chapter

15

Spinal Disorders

Patient with axial neck pain Vikram B. Patel

Case study A 38-year-old healthy female was in a car accident. She was behind another car at a stop light when a pick-up truck rear-ended her vehicle at about 20 mph. She was taken to an emergency room and a cervical fracture was ruled out with a plain x-ray. She continued to have neck pain radiating to the left shoulder and also had severe muscle spasms. Pain does not radiate beyond the shoulder on the left side and she does not have any tingling or numbness in the arm or hand. MRI of the cervical spine after 1 month shows a bulging disc at C5–6 with no neural compromise and no spinal cord compression.

1. What is the differential diagnosis? a. Cervical facet joint injury causing cervical facet syndrome secondary to whiplash type of injury b. Cervical discogenic pain due to annular disruption of the disc secondary to flexion injury c. Myofascial pain syndrome Cervical pain after a whiplash type of injury is a very common symptom, and was initially defined by the Quebec Task Force (QTF) in 1995.[1] Transition to chronicity is also prevalent. About 50% of patients reported neck pain after a 1-year follow-up.[2] As this woman was rear-ended, the impact has likely caused an extension type of injury leading to the facet joints being inflamed and generating pain. Cervical spine undergoes a sigmoid deformation very early after impact. During this deformation, lower cervical segments undergo posterior rotation around an abnormally high axis of rotation, resulting in abnormal separation of the anterior elements of the cervical spine, and impaction of the facet joints. The demonstration of a mechanism for injury of the facet joints complements postmortem studies that reveal lesions

in these joints, and clinical studies that have demonstrated that facet joint pain is the single most common basis for chronic neck pain after injury.[3] As she was behind another car, she would also have suffered a frontal collision causing somewhat milder flexion type of injury. Flexion injuries are the main cause of cervical disc damage and hence a disc bulge or even a rupture is very common based on the speed and severity of the impact. Disc strains are highest in the C4-C5-C6 disc segments, and ligament strains are greatest in these ligaments.[4]

2. What is the mechanism of injury in this patient? a. Whiplash type of cervical spine injury b. Extension type of injury during a rear-ended impact c. Flexion type of injury due to a frontal collision d. Spinal pain leading to secondary myofascial pain e. Shoulder joint may be involved during the initial injury f. Joint mobility may be reduced due to muscle pain, immobility, and dysfunction i. This can lead to secondary joint pain because of decreased range of motion ii. Shoulder adhesions may cause additional reduced range of motion and pain

3. Why is this condition occasionally misdiagnosed? Cervical axial pain is often misdiagnosed as radicular pain or shoulder pain (because of the radiation pattern). This may be secondary to an incidental finding of a

Case Studies in Pain Management, ed. Alan David Kaye and Rinoo V. Shah. Published by Cambridge University Press. © Cambridge University Press 2015.

116

Chapter 15: Patient with axial neck pain

Figure 15.2. Anteroposterior x-ray of the cervical spine with “0pen mouth” view showing the joints at the upper level of the cervical spine. Figure 15.1. Cervical facet joint pain radiation pattern.

damaged intervertebral disc or the radiation pattern of the pain. Nerve root compression leading to radicular pain usually presents with paresthesiae in the distribution of that particular nerve root and may be accompanied by pain, numbness, and/or weakness. Axial pain is more commonly presented without any neuropathic symptoms. It is localized in case of discogenic pain and has a specific radiation pattern if the facet joints are the pain generators (Figure 15.1). The facet joint radiation pattern from adjacent joints often overlap but it rarely radiates beyond the mid-upper arm.

4. Describe the anatomy and pathophysiology of the cervical facet (zygopophysial) joints The cervical spine consists of seven cervical vertebral bodies and intervertebral discs between them. The first vertebra is a ring-shaped structure called the atlas and articulates with the skull at the atlanto-occipital joint and with the second vertebral body at the atlantoaxial joint. The second vertebra has a vertical process called the odontoid process which provides rotational movement with its articulation along the anterior aspect of the 1st cervical vertebra (Figure 15.2). The cervical spine may be arbitrarily divided into anterior, middle, and posterior segments (Figure 15.3).

The anterior segment is made up of the vertebral body, intervertebral disc, and the anterior longitudinal ligament. The middle segment contains the intervertebral foramen, the posterior longitudinal ligament, the intervertebral foramen, the exiting nerve roots plus the blood vessels to and from the spinal cord. The posterior segment is made up of the articular elements and in the cervical spine these elements are labeled articular pillars. Each vertebra articulates with the superior as well as inferior vertebral body. These articular joints are called the facet or zygopophysial joints (Figures 15.3 and 15.4). In the cervical spine, these segments overlap and the middle + posterior segments present in the same plane (Figure 15.3). The anterior segment containing the disc is subjected to excessive pressure during a forceful forward flexion as well as rotational movements and may rupture. Subluxation of the cervical vertebral bodies is also likely and may cause damage to the vertebral arteries.

5. How to diagnose cervical axial pain? Diagnosis of axial neck pain is based on the imaging studies (e.g., MRI of the neck, plain x-ray) as well as the history and physical examination. a. MRI findings: i. A bulging disc in the cervical spine often denotes an internally disrupted disc. A visibly

117

Chapter 15: Patient with axial neck pain

Figure 15.3. Cervical spine cross-section at C7.

Figure 15.4. Articular pillars and the facet joints in the cervical spine.

herniated disc may cause axial pain as well as radicular pain if a nerve root is compressed and also be due to the inflammatory response from the herniated disc. Narrowing of the cervical spinal canal is usually present and may compress the spinal cord itself. ii. The facet joints may show narrowing, inflammation, and swelling. b. Plain x-ray of the cervical spine may show any anatomical displacement in the form of subluxation or a fracture. It may also reveal reduction of cervical lordosis and reduction in the disc height. c. History of injury or accident and the nature of the injury would help determine the pain generator: i. Facet joint pain is usually increased with extension and rotation on the ipsilateral side of

118

the cervical spine and may also radiate in a specific pattern but rarely extends beyond the mid-humeral level. This is an extension type of injury. ii. Increased pain with flexion with or without any paresthesiae may indicate a disc as the pain generator, and is usually the result of a flexion type of injury. iii. Superimposed myofascial pain is usually secondary to the spine pain and may exacerbate overall pain during flexion as well as extension and cause stiffness of the neck, restricting the movements. iv. Complaints of radiating pain vs. non-radiating (axial) pain would help in differentiating the facet vs. nerve root related pain.

Chapter 15: Patient with axial neck pain

Figure 15.5. Cervical medial branches AP and lateral views. The centroid of the lateral masses represents on a lateral view represents the target point for a diagnostic cervical medial branch block.

6. What are the treatment options for axial neck pain? a. Conservative approaches. Conservative treatment for axial neck pain due to the disc or the facets is largely based on the intensity of pain, radiologic findings, and the patient’s ability to perform dayto-day activities. In most patients, cervical discs heal without any interventions if further damage is prevented. PT, anti-inflammatory agents, and mild oral analgesics are the mainstay of the conservative treatment options. b. Interventional treatments: i. Minimally invasive treatment options are reserved for patients who fail to improve despite adequate trials of physical therapy, inability to perform physical therapy due to increased pain, inadequate response to medications, or inability to tolerate medications due to side effects. ii. Facet joint procedures: (1) Intra-articular facet joint injections of steroid have recently lost their appeal after several studies revealed poor evidence for the efficacy of such injections.[5] (2) Facet joints are supplied by the medial branches of the cervical nerve roots. Each joint is supplied by two medial branches, one from the same level and one from the

Figure 15.6. Cervical medial branch block needle placement.

level below, thus C3–4 facet joint is supplied by the C3 as well as C4 medial branches (Figure 15.5). Hence to denervate these joints one has to lyse two medial branches per level. (3) A carefully performed diagnostic medial branch block (Figure 15.6) on two different occasions using two different local anesthetics provides the most reliable diagnostic criteria to identify the facets

119

Chapter 15: Patient with axial neck pain

Figure 15.7. Radiofrequency ablation of the cervical medial branch in a patient with post laminectomy syndrome. Note the needle tip at the “waist” of the cervical articular pillar in AP view.

as the pain generators.[5] Radiofrequency ablation is then performed to denervate these joints for the long term (Figure 15.7). iii. Disc procedures: (1) Minimally invasive intradiscal procedures are aimed at reducing the bulk of the intervertebral disc by removing part of the nucleus percutaneously. A properly performed discogram may help determine if the disc has an intact annulus and the herniation is contained.

7. Procedural description a. Facet joint medial branch block: i. The block is ideally performed under fluoroscopic guidance, although recently it has been performed by some practitioners with the use of ultrasound guidance. ii. Lateral or posterior approaches have been described. The needle tip is fluoroscopically positioned at the midpoint of the articular pillar at the given level. A small amount of local anesthetic is then injected (usually < 0.25 ml) at each level (Figure 15.6).

120

iii. The level of pain relief is assessed soon after the block and the duration of the block depends on the type of local anesthetic injected. At least 75% of pain reduction lasting for the duration of the injected local anesthetic is a reliable indicator of facet joint mediated pain,[6] and the evidence for this procedure is a level II-1.[7] However, the patient may report longer duration of pain relief as the blocked facets may also relieve the superimposed muscular spasm and pain. b. Medial branch radiofrequency ablation: i. After a successful diagnostic set of medial branch blocks, radiofrequency ablation of the same medial branches is performed for longer pain relief. ii. The needles (22-G, 5 cm needle with 4 or 5 mm active tip) may be placed via posterior or lateral approach. Lateral approach may be preferable in patients who cannot tolerate prone position due to other comorbidities. Mild sedation may be provided for this procedure. iii. Once the needle tip is at the right placement, i.e., at the midpoint of the articular pillar on lateral view and at the “waist” of the articular pillar in anteroposterior view, sensory as well as motor stimulation is carried out. Sensory

Chapter 15: Patient with axial neck pain

stimulation may reveal pressure-like sensation in the neck and motor stimulation would elicit paraspinal muscle stimulation. These stimuli should not radiate to the arm. iv. After satisfactory stimulation pattern is established, local anesthetic with or without a minute amount of corticosteroid is injected (not to exceed 0.25 ml) and radiofrequency ablation is carried out. v. Preferences for the duration as well as temperature vary among practitioners. The author prefers a 60-second lesion set at 70°C.

8. What are the possible complications from these procedures? Risk of infection, bleeding, no pain relief, or nerve damage is similar to any cervical spine procedure but perhaps less with the medial branch block as it only involves local anesthetic injection. The radiofrequency lesioning may cause nerve damage if proper protocols are not observed and the needle tip is too close to the nerve roots. Sedation may also lead to complications if the patient is heavily sedated and cannot respond in an appropriate manner. A short period of increased pain may be due to muscular spasms and myofascial pain.

9. What are the outcomes with facet joint procedures? a. Radiofrequency ablation provides sustained longterm pain relief and may also help reduce the muscular pain which is secondary to the facet joint pain. Combined with proper physical therapy, it may provide 6–9 months of good pain relief. b. The evidence for diagnosis of cervical facet joint pain with controlled comparative local anesthetic blocks is Level I or II-1. The indicated evidence for therapeutic facet joint interventions is Level II-1 for medial branch blocks, and Level II-1 or II-2 for radiofrequency neurotomy.[7],[5]

10. Intradiscal procedures a. Diagnostic discogram i. Indications: disc bulge on an MRI ii. Increased pain with flexion of the neck iii. Failed conservative treatments

iv. It is a diagnostic tool rather than a prognostic tool and should be interpreted as such. However, a combination of diagnostic discogram and a pathologic disc pattern on imaging studies can essentially confirm the diagnosis of discogenic pain.[8] b. Procedure i. The procedure is performed under fluoroscopic guidance with strict aseptic precautions in supine position. Patient receives preoperative IV antibiotics and may also have antibiotics in the discography injectate. ii. A small caliber needle is placed within the nucleus of the target disc via right anterolateral approach.[9] This approach is preferable to mainly avoid penetration of the esophagus which can lead to a higher chance of infection (discitis). iii. A small amount of contrast material is injected (not to exceed 0.5 ml) and subsequent imaging is obtained to evaluate the disc morphology. A post-discogram CT scan provides the best diagnostic value. Reproduction of concordant pain is another important parameter that should be noted.

11. What are the possible complications from a cervical intradiscal procedure? a. Bleeding, nerve root damage, intraspinal penetration with possible spinal cord trauma. b. Infection is a very real risk[9] with subsequent discitis and possible epidural abscess, especially due to the possibility of the needle accidently traversing the esophagus on its way to the nucleus. Appropriate precautions such as strict aseptic technique and preoperative IV antibiotics as well as intradiscal antibiotics are required to lower such a risk. c. Excessive pressurization during a discogram may lead to the rupture of an already weakened annulus. Soft bulge of the disc with possible cord compression has also been reported. d. Damage to the endplates from the percutaneous discectomy procedures are likely if the probe is in contact with the endplate of the vertebral body.

121

Chapter 15: Patient with axial neck pain

References 1.

2.

122

Spitzer WO, Skovron ML, Salmi LR, et al. Scientific monograph of the Quebec Task Force on Whiplash-Associated Disorders: redefining “whiplash” and its management. Soine. 1995; 20(8 Suppl):1S–73S. Carroll LJ, Holm LW, HoggJohnson S, et al. Course and prognostic factors for neck pain in whiplash-associated disorders (WAD): results of the Bone and Joint Decade 2000–2010 Task Force on Neck Pain and Its Associated Disorders. Spine. 2008;33(4 Suppl): S83–92.

3.

Bogduk N, Yoganandan N. Biomechanics of the cervical spine Part 3: minor injuries. Clin Biomech (Bristol, Avon). 2001; 16(4):267–275.

joint injections. Pain Physician. 2012;15(6):E807–838. 7.

Falco FJ, Erhart S, Wargo BW, et al. Systematic review of diagnostic utility and therapeutic effectiveness of cervical facet joint interventions. Pain Physician. 2009;12(2):323–344.

4.

Panzer MB, Fice JB, Cronin DS. Cervical spine response in frontal crash. Med Eng Phys. 2011;33(9): 1147–1159.

8.

5.

Falco FJ, Manchikanti L, Datta S, et al. Systematic review of the therapeutic effectiveness of cervical facet joint interventions: an update. Pain Physician. 2012;15 (6):E839–868.

Siebenrock KA, Aebi M. The value of diskography in disk-related pain syndrome of the cervical spine for evaluation of indications for spondylodesis. Z Orthop Ihre Grenzgeb. 1993;131(3):220–224.

9.

6.

Falco FJ, Datta S, Manchikanti L, et al. An updated review of the diagnostic utility of cervical facet

Singh, V. The role of cervical discography in interventional pain management. Pain Physician. 2004;7(2):249–255.

Section 2 Chapter

16

Spinal Disorders

Patient with thoracic spine pain Ankit Maheshwari and Jianguo Cheng

Case study

Table 16.1. Differential diagnosis of a patient with thoracic pain

A 27-year-old male presents with pain in the back between the shoulder blades for 10 years. The pain is described as aching, heavy, and continuous. It is worse in the morning and improves as the day progresses. It has progressively deteriorated over the last 10 years. On physical exam, the patient has poor posture, reduced range of motion of the spine, paraspinal muscle spasm, and bilateral sacroiliac pain as well.

Nociceptive pain Compression fracture of the vertebrae Rib fracture/malignancy Facet arthropathy Axial Spondyloarthropathies: costochondritis, costoclavicular joint pain, costosternal joint pain Teitze’s syndrome DISH Myofascial pain syndrome

1. What are the anatomical structures that can produce pain in the thoracic region? Thoracic pain is not an uncommon presentation in the pain clinic affecting about 5% to 15% of patients.[1,2] The chest wall is made up of the vertebrae posteriorly, the sternum anteriorly, and the ribs laterally. The musculature of the chest wall may also be a source of pain. In addition, the chest viscera may produce pain in the thoracic region.[3] The differential diagnosis of thoracic pain is listed in Table 16.1. A thorough history and physical exam are essential to diagnosis. Interventional pain procedures have diagnostic and therapeutic value. We will focus on chest pain of spinal and musculoskeletal origin here. However, it is imperative not to overlook visceral pain when evaluating a patient because these conditions may be life threatening (cardiovascular and pulmonary diseases). Pain from abdominal viscera and the diaphragm can also be referred to the thoracic region.

Neuropathic pain Spinal cord disease: multiple sclerosis, tumor, syringomyelia Epidural cord compression: vertebral pathology, abscess, hematoma, facet hypertrophy Disc prolapse with cord/root compression Intercostal neuralgia - Infectious: herpes zoster and postherpetic neuralgia, syphilis - Traumatic: rib fracture, post-thoracotomy, postmastectomy - Tumor: schwannoma, neurofibroma

2. What are the key features in the differential diagnosis of thoracic pain? Thoracic pain can be nociceptive, neuropathic, mixed, or idiopathic. It could originate from the bony structures (rib fracture, sternum fracture, and compression fracture of the vertebral body), the intervertebral discs, the facet joints, the costovertebral joints, and the muscles and fascia. It can also be neuropathic, affecting the peripheral nervous system in such conditions as nerve root compression, herpes zoster and

Case Studies in Pain Management, ed. Alan David Kaye and Rinoo V. Shah. Published by Cambridge University Press. © Cambridge University Press 2015.

123

Chapter 16: Patient with thoracic spine pain

postherpetic neuralgia, painful diabetic neuropathy, post-thoracotomy syndrome, postmastectomy syndrome, postradiation neuritis, and Maigne’s disease (posterior rami syndrome). Less frequently, neuropathic pain can originate from pathologies of the central nervous system such as syringomyelia, multiple sclerosis, and lesions compressing the spinal cord. Neuropathic pain is often described as burning, tingling, lancinating, and “electric like” while nociceptive pain is usually described as aching, heavy, tight, stiff, and sometimes sharp. Acute pain can typically be linked to specific inciting events and pathology

while chronic pain can be challenging to determine the cause and pathology. The key differential factors on history and physical exam are listed in Table 16.2. Conditions that must be kept in the differential are ankylosing spondylitis, diffuse idiopathic skeletal hyperostosis (DISH), Scheuermann’s kyphosis, rheumatoid arthritis, osteoarthritis (compression fractures), costochondritis, Tietze syndrome, apophyseal facet syndrome, and primary cancer or metastatic disease of the spine. In addition, muscular pain with discrete trigger points is suggestive of myofascial pain syndrome.

Table 16.2. Key findings of various clinical syndromes affecting the thoracic spine

124

Disease

History

Physical and investigation

1. Ankylosing spondylitis

Adolescent to young adult Morning stiffness Pain improves with exercise Shifting low back (SI joint) pain

Restricted range of motion SI joint involvement Anterior chest wall involvement Lab: High ESR, C-reactive protein (CRP) HLA B-27 may be positive Imaging: sacroiliitis, spinal fusion “bamboo spine”

2. Vertebral compression

Elderly person History of osteoarthritis/trauma Initially sharp and localized pain but can later become radiating (neuropathic) due to compression of nerve roots/spinal cord

Vertebral and paravertebral tenderness Imaging: x-ray shows compression fracture MRI needed of planning vertebroplasty

3. DISH

Middle aged or older patient Back and spinal stiffness causing aching pain

Spinal immobility Tenderness to palpation spread across affected vertebrae Reduced ROM Imaging is necessary to make the diagnosis, findings as listed in Table 3

4. Costovertebral joint arthritis

Patient with history of manual labor or known history of inflammatory arthritis Localized aching pain of mild to moderate intensity

Tenderness to deep palpation of affected joints Diagnostic local anesthetic injection of the joint relieves pain

5. Facet arthropathy

Patient with history of deep aching pain, sometimes sharp and worse with extension of the spine. Leaning forward makes the pain better

Tenderness of deep palpation of the affected joints Facet loading elicits pain Paraspinal muscle spasm is commonly associated Diagnostic medial branch block of the affected levels relieves the pain

6. Costochondritis

More common in younger people Focal area of pain in the lower anterior rib cage Exacerbation of pain with coughing, sneezing, and deep breathing

Focal tenderness to palpation over the costochondral joints Horizontal flexion test positive Diagnostic local anesthetic injection of the joint relieves pain

Chapter 16: Patient with thoracic spine pain

Table 16.2. (cont.)

Disease

History

Physical and investigation

7. Tietze syndrome

Commoner in people older than 50 years of age which confuses this with a coronary event Localized pain and swelling of the upper anterior chest wall Aggravating factors same as costochondritis

Tenderness to palpation and localized swelling over the 2nd and 3rd costochondral joints Imaging is normal

8. Myofascial pain

Patient will usually give a history of heavy lifting, repeated and heavy use of the affected musculature Mild to moderate aching pain on pressing and deep breathing

Discrete trigger points present Injection of local anesthetic in trigger points relieves pain

9. Dercum’s disease (adiposis dolorosa)

Patient may be an obese woman Presence of subcutaneous nodules, most commonly in the chest and arm area causing shooting/stabbing pain

Painful subcutaneous nodules with shooting pain on palpation

10. Spinal cord disease (tumor, syrinx, MS)

Diffuse and poorly localized pain, which is burning or tingling in nature Sensory or motor loss of varying degrees

Diffuse pain, no focal areas of tenderness Detailed neurologic examination reveals abnormal findings based on the location and extent of disease Imaging: MRI of the spine

11. Epidural lesion causing spinal compression (hematoma, abscess, tumor)

History of infectious/traumatic/iatrogenic event Localized back pain Associated neurologic impairment

Tenderness to deep palpation or fist thumping on the affected segment Detailed neurologic examination reveals abnormal findings based on the location and extent of disease Imaging: MRI of the spine with contrast

12. Disc herniation causing spinal compression

Generally older patient Localized moderate to severe pain in a segmental nerve distribution Described as shooting or throbbing aggravated by coughing, sneezing, straining

Discogenic pain is infrequent in the thoracic region. Pain is primarily radicular due to compression of the spine/nerve roots from disc herniation Imaging: MRI spine

13. Intercostal neuralgia (herpes zoster, PHN, tabes dorsalis)

History of herpes or syphilis Severe burning or lancinating pain in a segmental/dermatomal distribution

Hyperalgesia, Hyperesthesia in the affected dermatome Rash +/ Lab confirmation for herpes or syphilis Imaging of spine/thorax normal

14. Maigne’s syndrome (posterior rami syndrome)

Lower thoracic, upper lumbar back pain preceded by sudden twisting of the spine Concomitant pain in the gluteal or groin region

Tenderness over the thoracolumbar facets with possible limitation of range of motion and positive facet loading with pain reproduction on the ipsilateral side Low back pain that has not responded to treatments aimed at the lumbosacral levels

3. What is ankylosing spondylitis? Ankylosing spondylitis (AS) is a form of spondyloarthropathy which refers to any joint disease of the vertebral column. AS results in stiff spine due to

arthritic changes in the intervertebral joints including the costovertebral, costotransverse, and apophyseal facet joints.[4] History of morning stiffness, waking up in the second half of the night, and improvement of pain with movement and sacroiliitis is highly

125

Chapter 16: Patient with thoracic spine pain

predictive of AS. This history associated with key radiologic and laboratory findings are highly sensitive for the diagnosis of AS.[5] Inflammatory markers such as erythrocyte sedimentation rate (ESR) and C-reactive protein are elevated. The HLA-B27 antigen system is commonly positive although negative HLA-B27 does not rule out the diagnosis of AS. Pain from costovertebral joints is common in AS (Bechterew’s disease). The 1st, 11th, and 12th ribs (ribs with just one facet plane) and the 6th through 8th ribs (longest) are the most frequently affected.[6] The prevalence of thoracic facet joint pain in patients with localized thoracic pain is about 42%.[7] Pain in the anterior chest wall (ACW) is often due to involvement of the costochondral and costosternal joints. The presentation and subsequent tests in this case are consistent with the diagnosis of AS. The general approach to treatment of thoracic pain is discussed later.

4. What is Tietze syndrome? Tietze syndrome is a benign but painful swelling of the 2nd or 3rd costal cartilage.[8] Straining, severe cough, heavy manual work, and arthritic conditions have been implicated as the causes. There is localized swelling and tenderness over the involved cartilage.[9] The condition is usually self-limited with occasional exacerbations and remissions of unclear cause. Tietze syndrome is not the same as costochondritis and involves swelling of the costal cartilages, which does not appear in costochondritis. Costochondritis is the most common cause of anterior chest wall syndrome.[10] The pain is described as aching to sharp. Unlike Teitze syndrome, it involves multiple sites, typically the 3rd to 6th costal cartilages. There is no swelling but tenderness to palpation over the involved cartilage is present. The horizontal flexion test is indicative of costochondritis. It consists of flexing the arm at the shoulder and crossing across the anterior chest wall while applying steady traction in a horizontal direction. Rotating the patient’s head on the ipsilateral side produces pain by pressuring the costochondral joints.

5. What is DISH? DISH is a common disease with prevalence as high as 15% in women and 25% in men over age 50, and 26% in women and 28% in men over age 80.[11,12] The usual presentation is a middle aged or older patient with chronic middle to lower back pain and spinal stiffness. The diagnosis is predominantly radiologic as described

126

Table 16.3. Radiologic features of DISH

1. Flowing ossification along the anterolateral aspect of at least four contiguous vertebrae 2. Preservation of disc height in the involved vertebral segment; the relative absence of significant degenerative changes, such as marginal sclerosis in vertebral bodies or vacuum phenomenon 3. Absence of facet joint ankylosis; absence of sacroiliac erosion, sclerosis, or intra-articular osseous fusion Figure 16.1. Radiograph of the thoracic spine shows flowing new bone formation along the anterior aspects of at least four vertebral bodies. The disc spaces are maintained.

by Resnick et al (Table 16.3, Figure 16.1).[13] The disease may be associated with extra-articular manifestations which are usually bilateral and symmetric. The absence of sacroiliitis, true syndesmophytes, and ankylosing apophyseal joints distinguish this syndrome from AS.[14]

6. What is Scheuermann’s kyphosis? Describe the end plate changes of this disease Scheuermann’s kyphosis is the most common cause of angular, progressive, structural thoracic, or

Chapter 16: Patient with thoracic spine pain

neurologic complications secondary to degenerative spondylosis and disc herniation.[17] The treatment depends on the severity of the deformity and development of neurologic and cosmetic symptoms. Physiotherapy includes hamstrings stretching and strengthening of the core and trunk extensors. PT does not correct the progression of the deformity. Bracing may be a solution for patients with flexibility of the spine at the levels with kyphosis, kyphosis < 65 degrees, and skeletal immaturity.[18] Surgical correction may be needed for severe deformity and neurologic and pulmonary complications and after failure of conservative therapy.[19]

7. What is posterior rami syndrome (dorsal ramus syndrome, Maigne’s syndrome)?

Figure 16.2. Characteristic radiographic findings in a patient with Scheuermann’s disease showing irregular vertebral endplates with Schmorl’s nodes narrowing of the intervening intervertebral disc space.

thoracolumbar hyperkyphosis with associated back pain in adolescence. It has an incidence of 4–8% with no gender predominance.[15] Patients complain of a dull non-radiating pain at the apex of the “gibbus.” This is associated with a compensatory lumbar hyperlordosis, tightness in the hamstrings and iliopsoas, and stiffness of the anterior shoulder girdle.[15] The radiologic diagnosis can be established in the presence of kyphosis > 45 degrees with anterior wedging of three or more consecutive vertebrae by 5 degrees, the presence of irregular vertebral endplates with Schmorl’s nodes narrowing of the intervening intervertebral disc space (Figure 16.2). The angle of kyphosis is measured using the Cobb method.[16] The classic definition of involvement of three or more continuous vertebrae has been challenged by some authors and practically the presence of the above findings even in one vertebra has been used to make this diagnosis earlier. The condition is expected to have a benign course after completion of spinal growth. However, a patient with severe curves can develop progressive deformities with potential for

Posterior ramus syndrome, also referred to as thoracolumbar junction syndrome, is caused by the unexplained activation of the primary division of a dorsal ramus of spinal nerve. This causes neuropathic pain distributed in a tri-branched pattern. One branch sets off anteriorly to the groin or pubic region; a second branch remains posterior, innervating the lower back and upper gluteal region; and a third branch passes down the anterolateral thigh or trochanter region. While any (or all) of these branches may be involved, their constancy of location is what allows this to be defined as a distinct syndrome. Although the pain is distributed in areas of the lumbosacral region, the facet joints in the thoracic spine are implicated in the pathophysiology of this syndrome. The orientation of the thoracic facet joints between T9 and T12 vertebrae may change abruptly to that of the lumbar area.[20] The stress of this transition can produce facet arthropathy over time and result in irritation of the dorsal rami, causing dorsal ramus syndrome.[21] This pain is conducted through the lateral branches of the posterior primary rami of the lower thoracic and upper lumbar nerve roots, specifically the cluneal nerves from T12, L1, and L2.[22] The diagnosis of thoracolumbar syndrome is clinical with the variable presence of four criteria. First, the patient usually relates the onset of pain to a rotational twisting movement. The affected posterior ramus ends cutaneously causing trophic changes of the skin. Neuropathic pain in three well-described

127

Chapter 16: Patient with thoracic spine pain

regions serves as the principal clinical component of the syndrome. Typical neuropathic skin changes are a thickening or nodularity of the skin, hair loss, or even a swollen puffy appearance. Second, the patients usually do not have spontaneous pain at the offending spinal level. Pain can be provoked by palpation of the facet joints and spinous processes which may help to determine the level of origin. Additional key physical findings include: (a) pain and deep tenderness to palpation over the iliac crest at the point where the distal cluneal nerve branches cross the iliac crest, (b) hypersensitivity of the skin and subcutaneous tissues of the gluteal and iliac crest areas, noted when a fold of skin is rolled between the fingers, (c) localized tenderness over the affected thoracolumbar segment when posterior-to-anterior or lateral pressure is applied to the spinous process at the affected level, and (d) tenderness and restricted motion. Pressure at these points will reproduce local discomfort and the patient’s referred pain.[23] Typically, patients do not have pain radiating below the knee, which is more typical of anterior ramus involvement. Third, radiographic evidence is non-contributory. Fourth, the diagnosis is confirmed by injection of local anesthetic into the correct facet joint that results in pain relief. This diagnostic procedure can also be therapeutic; the steroid injection or radiofrequency ablation of the medial branch can be applied for longer pain relief. Other treatment includes anti-inflammatory medications, spinal manipulation, and physical therapy.

8. How would you manage patients with thoracic pain? In the last decades, significant progress has been made in the management of patients with immunemediated diseases such as AS.[24] Early diagnosis can prevent or delay future anatomical abnormalities and painful spinal immobility. Generally, the strategy to treat patients with thoracic pain should include the following.

Conservative approaches Inflammatory arthritis and small joint arthritic pain such as costochondritis usually respond well to NSAIDs. Disease-modifying anti-rheumatologic drugs (DMARDs) and TNF-alpha inhibitors are the treatment of choice for most axial spondyloarthopathies and AS, typically in conjunction with NSAIDs,

128

physical therapy, and interventional treatments as needed. For neuropathic pain, the use of gabapentin or pregabalin has been shown to be effective for postherpetic neuralgia.[21–23] Antidepressants, such as TCAs, may also be effective.[25] Opioid medications should be reserved as a last resort in the treatment of these patients and used only for a short period of time with clear goals and close monitoring. TENS units have been shown to improve pain of myofascial origin as well as neuropathic pain. This may be a useful adjunct to medications and physical therapy in the treatment of thoracic spinal pain. Physical therapy for mobilization and strengthening and occupational therapy for posture correction is beneficial for somatic pain conditions of the thoracic spine. Braces may be indicated for spinal deformities.[18]

Interventional approaches Epidural or paravertebral injection of local anesthetics with steroid, with or without additives such as clonidine, has been shown to provide pain relief for thoracic radiculopathy, zoster and postherpetic neuralgia, intercostal neuralgia, and post-thoracotomy syndrome. Epidural injections for managing chronic thoracic pain showed fair evidence with one randomized controlled trial in patients with various causes whereas the evidence for post-thoracotomy syndrome was poor.[26] Chest wall pain from rib fractures after trauma is amenable to treatment with continuous epidural infusion through a catheter placed in the epidural space. This reduces splinting due to pain and improves ventilation.[27,28] Intercostal neuralgia may be treated with intercostal nerve block and paravertebral block.[29] Facet medial branch block can be diagnostic and therapeutic. There is strong evidence for diagnostic facet joint block for the diagnosis of facet joint pain. Studies have shown sustained pain relief of 1 year with repeated thoracic facet medial branch block.[30] Radiofrequency ablation of the medial branches after two successful medial branch blocks may be considered in patients with good but transient pain relief. Facet joint intra-articular injections can be done but have not been extensively studied for the thoracic spine.[31] Costochondral and costovertebral joint injections may have both diagnostic and therapeutic values.

Chapter 16: Patient with thoracic spine pain

These injections may facilitate participation in physical therapy. Trigger point injections with local anesthetic with or without steroid are indicated for myofascial pain where discrete trigger points can be identified. Spinal cord stimulation may be considered for neuropathic chest wall pain in refractory cases.[32,33] Vertebroplasty or kyphoplasty may be indicated for the treatment of compression fractures. The best time to treat patients with this modality is

References 1.

Linton SJ, Hallden K. Can we screen for problematic back pain? A screening questionnaire for predicting outcome in acute and subacute back pain. Clin J Pain. 1998;14(3):209–215.

2.

Lou L, Gauci CA. Radiofrequency treatment in thoracic pain. Pain Pract. 2002;2(3):224–225.

3.

Snell RS. Clinical Anatomy by Regions, 8th ed. Philadelphia: Lippincott Williams & Wilkins; 2007.

4.

Reuler JB, Girard DE, Nardone DA. Sternoclavicular joint involvement in ankylosing spondylitis. South Med J. 1978; 71(12):1480–1481.

5.

Walker BF, Williamson OD. Mechanical or inflammatory low back pain: what are the potential signs and symptoms? Man Ther. 2009;14(3):314–320.

6.

Sidiropoulos PI, Hatemi G, Song IH, et al. Evidence-based recommendations for the management of ankylosing spondylitis: systematic literature search of the 3E Initiative in Rheumatology involving a broad panel of experts and practising rheumatologists. Rheumatology (Oxford). 2008;47(3):355–361.

7.

8.

Manchikanti L, Boswell MV, Singh V, et al. Prevalence of facet joint pain in chronic spinal pain of cervical, thoracic, and lumbar regions. BMC Musculoskelet Disord. 2004;5:15. Motulsky A, Rohn RJ. Tietze’s syndrome; cause of chest pain and

within 6 weeks after the fracture.[34,35] Alternative therapies such as acupuncture and chiropractic manipulation may have a role in facet arthropathy, costovertebral joint pain, costochondritis, and myofascial pain syndrome. Surgical interventions, such as discectomy and spine deformity correction or fusion, may be required in advanced stages of disease or advanced anatomical abnormalities that are not amenable to conservative treatment.

chest wall swelling. J Am Med Assoc. 1953;152(6):504–506. 9.

Gill G. Epidemic of Teitze’s Syndrome. BMJ. 1977;(2):499.

10. Scobie BA. Costochondral pain in gastroenterologic practice. N Engl J Med. 1976;295(22):1261. 11. Meyer PR Jr. Diffuse idiopathic skeletal hyperostosis in the cervical spine. Clin Orthop Relat Res. 1999;(359):49–57. 12. Weinfeld RM, Olson PN, Maki DD, Griffiths HJ. The prevalence of diffuse idiopathic skeletal hyperostosis (DISH) in two large American Midwest metropolitan hospital populations. Skeletal Radiol. 1997;26(4):222–225. 13. Resnick D, Shapiro RF, Wiesner KB, et al. Diffuse idiopathic skeletal hyperostosis (DISH) [ankylosing hyperostosis of Forestier and Rotes-Querol]. Semin Arthritis Rheum. 1978; 7(3):153–187. 14. Resnick D, Niwayama G. Radiographic and pathologic features of spinal involvement in diffuse idiopathic skeletal hyperostosis (DISH). Radiology. 1976;119(3):559–568. 15. Ali RM, Green DW, Patel TC. Scheuermann’s kyphosis. Curr Opin Pediatr. 1999;11(1):70–75. 16. Voutsinas SA, MacEwen GD. Sagittal profiles of the spine. Clin Orthop Relat Res. 1986;210:235–242. 17. Tsirikos AI, Jain AK. Scheuermann’s kyphosis: current controversies. J Bone Joint Surg Br. 2011;93(7):857–864.

18. Weiss HR, Turnbull D, Bohr S. Brace treatment for patients with Scheuermann’s disease: a review of the literature and first experiences with a new brace design. Scoliosis. 2009;4:22. 19. Arlet V, Schlenzka D. Scheuermann’s kyphosis: surgical management. Eur Spine J. 2005; 14(9):817–827. 20. White AA, Panjabi MM. Clinical Biomechanics of the Spine. Philadelphia: JB Lippincott. 1978. 21. Grieve GP. Common Vertebral Joint Problems. New York: Churchill Livingstone. 1981. 22. Maigne R. Low back pain of thoracolumbar origin. Arch Phys Med Rehabil. 1980;61(9):389–395. 23. Maigne R. [The thoraco-lumbar junction syndrome. Low back pain, pseudo-visceral pain, pseudo-hip pain and pseudopubic pain (author’s transl)]. Sem Hop. 1981;57(11–12):545–554. 24. Haroon N, Inman RD, Learch TJ, et al. The impact of TNFinhibitors on radiographic progression in Ankylosing Spondylitis. Arthritis Rheum. 2013;65(10):2645–2654. 25. Saarto T, Wiffen PJ. Antidepressants for neuropathic pain: a Cochrane review. J Neurol Neurosurg Psychiatry. 2010;81(12):1372–1373. 26. Benyamin RM, Wang VC, Vallejo R, Singh V, Helm Ii S. A systematic evaluation of thoracic interlaminar epidural injections. Pain Physician. 2012;15(4):E497–514.

129

Chapter 16: Patient with thoracic spine pain

27. Mackersie RC, Karagianes TG, Hoyt DB, Davis JW. Prospective evaluation of epidural and intravenous administration of fentanyl for pain control and restoration of ventilatory function following multiple rib fractures. J Trauma. 1991;31(4):443–449; discussion 9–51. 28. Ullman DA, Fortune JB, Greenhouse BB, Wimpy RE, Kennedy TM. The treatment of patients with multiple rib fractures using continuous thoracic epidural narcotic infusion. Reg Anesth. 1989; 14(1):43–47. 29. Cheng J, Cata J. Interpleural analgesia. In Smith H, ed. Current Therapy in Pain. Philadelphia,

130

PA: Elsevier (Saunders). 2008: pp. 92–94. 30. Manchikanti KN, Atluri S, Singh V, et al. An update of evaluation of therapeutic thoracic facet joint interventions. Pain Physician. 2012;15(4):E463–481. 31. Pope J, Cheng J. Facet joint injections: cervical, lumbar and thoracic. In Benzon H, Huntoon M, Narouze S, eds. Spinal Injections and Peripheral Nerve Blocks, 1st edn. Philadelphia, PA: Elsevier. 2010: pp. 129–135. 32. Wininger KL, Bester ML, Deshpande KK. Spinal cord stimulation to treat postthoracotomy neuralgia: nonsmall-cell lung cancer: a case

report. Pain Manag Nurs. 2012; 13(1):52–59. 33. Yakovlev AE, Resch BE, Karasev SA. Treatment of chronic chest wall pain in a patient with LoeysDietz syndrome using spinal cord stimulation. Neuromodulation. 2011;14(1):27–29; discussion 9. 34. Barr JD, Barr MS, Lemley TJ, McCann RM. Percutaneous vertebroplasty for pain relief and spinal stabilization. Spine (Phila Pa 1976). 2000;25(8):923–928. 35. Mathis JM, Barr JD, Belkoff SM, et al. Percutaneous vertebroplasty: a developing standard of care for vertebral compression fractures. AJNR Am J Neuroradiol. 2001; 22(2):373–381.

Section 2 Chapter

17

Spinal Disorders

Patient with lumbar disc herniation Julian Sosner

Case study A 23-year-old, medical student presents with a 3-day history of severe posterolateral leg pain that was preceded by 2 weeks of ipsilateral back and buttock pain. The character of the pain changed during this interval from a dull and spasmodic back ache to a sharp, lancinating pain going down the leg to the calf. He has difficulty with push off during normal ambulation and sprints. Sitting and squatting worsen pain. Pain is severe in the morning but never lasts more than half an hour. Physical exam demonstrates weakness with ipsilateral plantar flexion, reduced ankle jerk (muscle stretch reflex – or the misnomer, deep tendon reflex), reduced light touch in S1 dermatome, hypersensitivity to pinprick in S1 dermatome, and ipsilateral straight leg raise test positive.

1. What is the differential diagnosis? The patient’s age and presentation with no past medical history suggests a benign condition: 1. Lumbar facet pain 2. Lumbar discogenic pain 3. Lumbar synovial cyst 4. Lumbar spinal stenosis – congenital 5. Muscle spasm 6. Acute inflammatory demyelinating polyneuropathy 7. Mononeuritis multiplex 8. Traumatic spinal cord syrinx 9. Lumbar disc herniation The most likely etiology based on the history (age, acuity, progression from back to leg, weakness with

push off, pain worse with sitting) and physical exam (myotomal weakness, dermatomal radiation, reduced muscle stretch reflex, dural tension signs) suggests a lumbar radiculopathy. The most common etiology for this patient would be a lumbar disc herniation (LDH).

2. What is a lumbar disc herniation? There are multiple ways to classify an LDH: (1) radiologic (MRI or CT); and (2) anatomical (for surgical planning and surgical outcomes). A widely used classification, as recommended by the combined task forces of the North American Spine Society, American Society of Spine Radiology, and American Society of Neuroradiology, is based on CT or MRI imaging. A herniation is defined as a localized displacement of disc material beyond the limits of the intervertebral disc space contour. It can be “focal” or “broad based” depending on the size of the base of this displacement relative to the disc circumference: (1) Focal is < 25%; and (2) broad based is 25–50%. If the broad based is > 50%, this is considered to be a “bulging” disc and not an LDH. The second set of criteria for an LDH refers to the size of the displaced material relative to the base. If the base is larger in terms of width as compared to the width of the displaced material, this is a disc protrusion. If the base is smaller in terms of width as compared to the width of the displaced material, this is a disc extrusion. If the displaced disc material is detached from the rest of the disc then it is called a “sequestered” disc herniation. If there is a layer of annulus around the herniation it is called “contained” and if not, it is called

Case Studies in Pain Management, ed. Alan David Kaye and Rinoo V. Shah. Published by Cambridge University Press. © Cambridge University Press 2015.

131

Chapter 17: Patient with lumbar disc herniation

Figure 17.1. Lumbar disc herniation classification on vertebral model.

“uncontained.” Dye injected into the disc will leak out of the disc. A displacement into the vertebral endplate is called “intravertebral herniation” or a “Schmorl’s node.” A “migrated” herniated disc is disc material displaced beyond the opening in the annulus. It can be continuous or detached from the disc. With respect to the posterior longitudinal ligament (PLL), a herniation may be “subligamentous,” “extraligamentous,” “transligamentous,” or “perforated.” Based on the location within the spinal canal, an LDH may also be classified as central, paracentral, lateral (foraminal), or far lateral (extraforaminal) (Figure 17.1). The bony landmarks are the medial borders of the facets and the medial and lateral border of the pedicles. The central canal zone is the area between the medial borders of the facets. The subarticular zone is the lateral recess area. The pedicle area is the foraminal zone. The area beyond the lateral pedicle border is the extraforaminal or far lateral zone. This obviously has clinical significance in the clinical presentation and surgical treatment of an LDH. A central or paracentral herniation will affect nerve roots that exit at lower levels, while a foraminal or extraforaminal herniation will affect the nerve root at the same numerical level as the disc level affected. There is more “space” in the central portion of the vertebral canal, i.e., the central and paracentral herniation zone is larger than the lateral or far lateral zone. So, a larger volume of herniated disc can be tolerated centrally or paracentrally and be less

132

symptomatic as compared to a similar sized herniation in the lateral or far lateral zone. The location of the disc herniation can be further described in the caudocranial direction based on its relationship to the pedicle and disc: supra-pedicular, pedicular, infrapedicular, and discal.

3. Describe the pathogenesis of lumbar radiculopathy secondary to an LDH There are many other causes of radiculopathy besides that from an LDH, which are beyond the scope of this chapter. The radiculopathy from a herniated disc can be caused by direct and indirect mechanical compression, by an inflammatory reaction, or by a combination of the two. Direct mechanical compression or vascular compromise (venous) can block nerve transmission (neuropraxia) or cause axonal damage (axonotmesis). True neurotmesis is rare and will occur only in prolonged, severe compression of the nerve. A multitude of clinical scenarios may present depending on the degree of sensory, motor, and sympathetic nerve fiber compromise. Differentional nerve fiber damage occurs due to size and location – the larger the diameter of the nerve fiber, the worse the damage. A profound inflammatory reaction occurs when the nucleus pulposus extrudes beyond the annulus and onto the nerve. This is mediated by phospholipase A2, nitric oxide, prostaglandin E, tumor necrosis factor alpha (TNF-α), and various cytokines.

Chapter 17: Patient with lumbar disc herniation

4. Discuss clinical symptoms, signs, imaging, and diagnostic testing for an LDH Clinical symptoms and signs of lumbar radiculopathy can be thought as being grouped in positive and negative signs groups. The positive (irritation) signs are: (1) presence of pain in a dermatomal distribution; (2) areas of hypesthesia; (3) presence of lumbar paraspinal spasm; (4) signs of peripheral nerve irritation such as sciatic or Valeix tender points (tenderness to deep localized pressure in the gluteal sciatic notch and posterior thigh, respectively); and (5) signs of nerve root irritation with dural tension testing. Dural tension tests involve passively stretching the limb, and hence, the nerve, against an LDH. These tests have been validated in cadavers. The straight leg sign is positive if pain is evoked by straightening of the knee and flexion of the hip while the patient is seated or supine. The Lasegue sign is positive if pain is evoked in the posterior thigh and leg upon passive extension of the flexed knee while raising the lower limb with the patient supine. Lazarevic sign is evoked pain in the posterior thigh and leg when bending toward one foot or the other. The femoral stretch sign is evoked pain in the anterior thigh upon passive flexion of the knee and extension of the hip. The negative (deficit) signs are motor and sensory deficits in corresponding myotomes and dermatomes. There may be decreased or absent muscle stretch reflexes (deep tendon reflexes). The knee reflex (quadriceps) involves the L2, L3, and L4 spinal nerves. The hamstring reflex involves L5 and S1. The ankle reflex involves S1. This patient had evidence of an S1 radiculopathy.

Imaging of the LDH includes computerized tomography (CT), MRI, myelograpy, and discography. The “gold standard” is the MRI of the lumbar spine (Figure 17.2). It shows the relevant soft tissues: disc, ligaments, epidural space, nerves, and bone marrow. Intravenous contrast can help differentiate between a herniated disc and scar tissue (prior back surgery), infection, or neoplastic processes. Synovial (usually facet joint) or discal cysts (another etiology of radiculopathy) and their origin can be well visualized. The advantage of MRI is the richness of information it can provide and the absence of ionizing radiation to the body. The disadvantages are: (1) cost; (2) length of time for scanning; (3) claustrophobic patients; and (4) contraindication in patients with metallic implants or shrapnel. A CT of the lumbar spine is valuable in diagnosing the disc herniation location, but is less accurate in detailing the configuration of the herniated fragments or extrusion. CT is useful in delineating bony details. CT is particularly useful in combination with myelography or discography – these modalities provide detailed information about the relationships between the disc herniation, the nerve root, the dural sleeve, and the vertebral and foraminal canals. The advantages of CT are: (1) relatively short scan times; (2) reduced risk of claustrophobia; and (3) lower costs. The main disadvantages are the high dose of ionizing radiation that the patient receives and a less detailed image of the soft tissues as compared to an MRI. Myelography details the contour of the dural sac and dural sleeves. CT myelography can identify neural compromise by the LDH and relative stenosis of the vertebral and foraminal canals. The disadvantages are high costs and risk of dural puncture complications:

Figure 17.2. Left L5-S1 disc herniation T2 axial, T1 axial, T2 sagittal (left to right).

133

Chapter 17: Patient with lumbar disc herniation

infection; hematoma; positional headache; intracranial hypotension; seizures; and allergic reactions. A discography test gives detailed information about the disc architecture and the pathologies involved in producing LDHs. Annular tears, endplate herniations, multi-directional herniations, contained herniations, and internal disc disruption may be identified. In addition to anatomic detail, physiologic information may be identified with discography. Intradiscal pressures can be measured. The degree of “sensitization” of the disc depends on the evoked pain pressure threshold also known as discogenic pain. A postdiscogram CT can better delineate the LDH or internal disc disruption. Disadvantages of the discogram are high cost and the potential for complications (infection, hematoma, and nerve damage). There is some discussion in the literature that performing a discogram can accelerate the disc degeneration. The relevant diagnostic tests in case of radiculopathy are mainly nerve conduction and electromyography testing. The test is performed on the muscles and nerves of both the lower extremities and lumbar spine. In a positive test for radiculopathy there are usually normal nerve conduction findings (latency times, conduction velocities, and muscle action potentials). Needle electromyography demonstrates abnormal findings such as “denervation potentials” (positive sharp waves and fibrillations) in the paraspinal and peripheral muscles that are innervated by the compromised nerve roots. These findings become positive about 3 weeks after the occurrence of the acute radicular compression. During the first 3 weeks there might be only increased “needle insertional activity” which means an initial muscle membrane instability that can lead later to the denervation potentials mentioned above. This means that often the test is normal in the first 3 weeks.

5. Discuss conservative treatment options The pain is unbearable. Hence, analgesia is paramount before addressing rehabilitation. Oral steroids, muscle relaxants, antiseizure medication, NSAIDs, and opioids may be considered. Modified, active rest within the first 2 days may be advised. Patients may need an assistive device, back brace, ankle-foot orthotic, or knee brace given the type of myotomal weakness. Eventually, physical therapy may be commenced. Usually, spinal extension, core strengthening, and neural flossing exercises

134

are used. Passive modalities include heat, cold packs, transcutaneous electrical stimulation. Non-traditional therapies such as acupuncture may be beneficial for pain control, but the reality is that this pain is extremely severe. Interventional procedures include epidural steroid injections, via the interlaminar, transforaminal, or caudal routes. Conservative measures have demonstrated comparable long term success to surgery, but for more immediate relief, surgery has been successful.

6. Discuss surgical approaches to the lumbar disc herniation The surgical approaches to the LDH can be grossly divided into discectomies that are percutaneous, minimally invasive endoscopic, minimally dissecting using a surgical microscope, and conventional discectomies. Percutaneous disc procedures are usually performed with the aid of fluoroscopy and rarely with CT. A conventional discogram approach is performed with a needle (14 to 18 Gauge) introduced percutaneously into the disc. Then a “device” is introduced through the needle to decompress the nucleus polposus. There are several such devices: DeKompressor (Stryker), enSpire (Spine View), and Nucleotome (Clarus). Most use a mechanical or heating element to decompress the disc centrally; the hope is that intradiscal volume reduction, even by a small amount, will allow the LDH to shrink inwards and take pressure off the nerve root. The principal indication for these procedures is broad-based, contained protrusions of the disc. The advantages of these procedures are that they are the least invasive, relatively simple to perform, require minimal specialized equipment, and may be done with local anesthesia or mild sedation. The disadvantages are that the amount of disc material removed is small and the decompression is expected to indirectly relieve pressure on the herniated part. There is no direct removal of the herniated disc material. Minimally invasive endoscopic discectomy allows endoscopic visualization of the anatomic structures and a controlled direct discectomy. It can be performed via a modified “discogram approach” entering the disc first and then the herniation from inside the disc (“inside–out”) (Figure 17.3). It can also be performed via a transforaminal approach – first approaching the herniation and then the disc (“outside–in”). In both techniques the approach to the disc occurs through the Kambin’s triangle at the foramen (see Chapter 59). The

Chapter 17: Patient with lumbar disc herniation

Figure 17.3. Lumbar endoscopic discectomy In-Out approach.

confines of this virtual “triangle” are delimited ventrally by the exiting root, caudally by the pedicle and the posterior vertebral wall, dorsally by the traversing root and the articular processes, and the floor by the intervertebral disc. Some of the instruments mentioned below permit a partial facetectomy to enlarge the bony foramen (foraminotomy/foraminoplasty) in addition to a discectomy. In these procedures a needle is introduced percutaneously and advanced to the desired target point in the Kambin’s triangle. A guide wire is passed through the needle and kept in place while the needle is removed. Progressively larger dilators are passed over the guide wire to form a channel of 6 to 9 millimeters through which a cannula is introduced. The cannula serves as a conduit and working channel for the scope, the circulating fluid, and the working instruments. A discogram is then performed with a mixture of contrast, Carmine blue, and antibiotics. This allows visualizing the disc and the herniated part with both the fluoroscopy and the scope. Hemostasis and a partial discectomy can be performed utilizing specialized bipolar cautery. Discectomy is done with a range of different endoscopic grasping instruments. In the “inside–out” technique, contained disc herniations can be targeted. With the “outside–in” technique, both contained and extruded herniations as well as some migrated fragments can be removed. There are several endoscopic spinal discectomy systems including the JoiMax TESSYS (Transforaminal Endoscopic System) (Joimax Inc.), MaxMore (MaxMore spine), the EnSpire MIS (Spine View Inc.), Disc-Fx (Elliquence), YESS (Yeung Endoscopic Spine Surgery), and Vertebris (Richard Wolf). These systems are very versatile and can treat most of the types of herniated discs independently of their location. Indeed, a recent study showed that the surgical outcomes are similar to conventional microdiscectomy. For very large herniated discs a bilateral approach is sometimes necessary. The principal

advantages of these surgical modalities are that there is no muscle, fascia, or ligament cutting and no periosteal detachment of muscles which allows for the patient’s quick recovery; he or she can return home on the day of surgery. There is a reduced infection rate due to the continuous irrigation during the procedure. Neural injury may be reduced due to the use of local anesthetics and mild sedation, which allows patient feedback. The disadvantages are that there is a steeper learning curve for the pain specialist or surgeon to use these techniques and instruments and to master the fluoroscopic views and endoscopic images. Minimal dissection microscopic discectomy is performed via a dorsal lumbar approach using only a small incision of 2 to 5 cm. The surgical technique mimics the traditional discectomy. There is dissection of the paravertebral muscles down to the lamina and incision of the fascia and ligaments. Than a partial laminectomy and ligamentum flavum removal is performed to gain access to the dorsal epidural space. A surgical microscope is used to visualize the deep structures. The dural sac and traversing nerve roots have to be manipulated and moved in order to gain access to the disc and to the herniated part in the anterior epidural space. The procedure is usually done with sedation or general anesthesia. A traditional discectomy is done similarly to the microscopic discectomy but the incision, the disruption of tissue, and also the size of the laminectomy are larger than in all of the procedures discussed above. The disadvantages are that the surgery is usually done with general anesthesia and the patient frequently stays in the hospital for a few days postsurgically. The larger tissue dissection leads to slower recovery. Most importantly, the manipulation of the dura and nerves combined with the laminectomy and flavectomy discussed above, can cause formation of scar tissue and neuropathic pain. This is true also for the microscopic

135

Chapter 17: Patient with lumbar disc herniation

discectomy discussed above. The advantages are that the epidural space can be easily inspected and extruded and uncontained fragments can be removed even if they are lodged in multiple locations. Also, a large or complete laminectomy can be done to decompress the canal in case of concomitant canal stenosis.

References

Clearly, a less invasive procedure with less tissue and muscle disruption and manipulation leads to less complications, less postoperative pain, faster patient mobilization, and return to normal daily activities and less expensive global surgical and rehabilitation treatment.

1.

Tarulli AW, Raynor EM. Lumbosacral radiculopathy. Neurol Clin. 2007;25(2):387–405.

5.

2.

Parke WW, Whalen JL., The vascular pattern of the dorsal root ganglion and its probable bearing on a compartment syndrome. Spine. 2002;27:347–352.

Kambin P. Arthroscopic and Endoscopic Spinal Surgery: Text and Atlas. New Jersey: Humana Press. 2005.

6.

McMahon SB, Koltzenburg M. Wall and Melzack’s Textbook of Pain, 5th edn. Elsevier Churchill Livingstone. 2006.

3.

4.

136

Fardon DF, Milette PC: Nomenclature and classification of lumbar disc pathology. Recommendations of the Combined task forces of the North American Spine Society, American Society of Spine Radiology, and American Society of Neuroradiology. Spine. 2001;26(5). Lewandrowski K-U, Lee S-H, Iprenburg M. Endoscopic Spine

Award Winner: lumbosacral nerve root displacement and strain: part 1. A novel measurement technique during straight leg raise in unembalmed cadavers. Spine (Phila Pa 1976). 2007;32(14):1513–1520. PubMed PMID: 17572621.

Surgery. JP Medical Publishers. 2013.

7.

8.

Gilbert KK, Brismée JM, Collins DL, et al. Young Investigator Award Winner: lumbosacral nerve root displacement and strain: part 2. A comparison of 2 straight leg raise conditions in unembalmed cadavers. Spine (Phila Pa 1976). 2007;32(14):1521– 1525. PubMed PMID: 17572622. Gilbert KK, Brismée JM, Collins DL, et al. 2006 Young Investigator

9.

Shah RV, Lutz GE. Lumbar intraspinal synovial cysts: conservative management and review of the world’s literature. Spine J. 2003;3(6):479–488. Review. PubMed PMID: 14609693.

10. Shah RV, Everett CR, McKenzieBrown AM, Sehgal N. Discography as a diagnostic test for spinal pain: a systematic and narrative review. Pain Physician. 2005;8(2):187–209. PubMed PMID: 16850074

Section 2 Chapter

18

Spinal Disorders

Patient with lumbar facet-mediated pain Vikram B. Patel

Case study A 72-year-old male was suffering from low back pain for the last 5 years. He worked as a baggage handler at JFK airport in New York. He had retired when he was 65 years old and had no medical problems until then. Lately, he had been having low back pain which has gradually increased in severity. He had seen his primary care physician who had prescribed an anti-inflammatory agent for pain. However, his recent cardiac stent placement mandated clopidogrel which he has to take every day and hence he is unable to take the anti-inflammatory medicine. He was referred to a pain specialist who obtained an MRI of his lumbar spine. The findings suggested hypertrophy of the lumbar facets at multiple levels, more severe on the left side. His intervertebral discs were also degenerated at almost all the lumbar levels with significant disc space narrowing. There was mild spinal canal central stenosis but no foraminal compromise and no nerve root compression. He has occasional tingling in the left leg but no numbness or weakness. He feels pressure-like sensation in both thighs after walking for about 10 min and has to sit down leaning forward to relieve his symptoms. On physical examination he had increased pain with extension of his lumbar spine and tended to walk and sit leaning forward. He had no sensory motor deficits in the legs and the straight leg raising test was negative in both legs. There was moderate tenderness on palpation in the lower back on both sides. There were no other positive findings.

1. What is the differential diagnosis? a. Lumbar facet joint mediated pain causing low back pain b. Lumbar spinal stenosis with possible neurogenic claudication

c. Myofascial pain syndrome secondary to the spinal pain d. Referred pain from abdominal organs e. Lumbar discogenic pain Lumbar facet joint pain is common in older age due to degeneration of the discs, especially combined with a life style that constantly creates loading of the lumbar spine. The most frequent causes of LS facet syndrome are functional disorders (functional blockade or dysfunction of facet joint¼reversible restriction of facet joint movements caused by meniscoid entrapment) and degenerative changes of facet joints while the others, such as spondyloarthropathies, infection, tuberculosis, synovial cyst, and injury, are less frequent.[1] Most of these patients have a gradual onset of pain with increasing severity. They complain of primarily lower back pain which increases with upright position and is partially relieved after sitting down and flexing the spine. Patients also have increased pain while descending stairs with extended back, walking, sitting straight for prolonged periods, etc. They tend to lean forward or take support while walking. Neurogenic claudication is a symptom of spinal stenosis caused by multiple factors. The increased narrowing of the spinal canal secondary to ligamentum flavum hypertrophy as well as facet joint hypertrophy leads to spinal canal stenosis, which is symptomatic after a variable length of spinal extension such as while walking. The buckling of the ligamentum flavum is partially reduced after sitting down with a flexed spine, thus relieving the symptoms of neurogenic claudication. The facet joint syndromes are also common after lumbar spine surgery, especially a fusion where the facet joints above and below the fusion suffer accelerated degeneration causing facet joint syndrome. Some of the precipitating factors for patients to develop such pain are advanced

Case Studies in Pain Management, ed. Alan David Kaye and Rinoo V. Shah. Published by Cambridge University Press. © Cambridge University Press 2015.

137

Chapter 18: Patient with lumbar facet-mediated pain

age, long operative time, intraoperative complications, history of recurrent disc prolapse, discectomy, and lack of rehabilitation.[2]

2. What is the mechanism of pain generation in this patient? a. Inflammation of the lumbar facet joints secondary to degeneration and arthritis b. Facet joint loading during extension of the lumbar spine c. Spinal stenosis causing neurogenic claudication d. Spinal pain leading to secondary myofascial pain e. Forward flexion leading to increased pressure on the already degenerated discs i. Further narrowing of the facet joint space ii. Increased degeneration of the facet joints

3. What is the difference between the axial back pain and radicular pain? a. Spinal axial pain is basically secondary to the musculo-skeletal elements of the spine. b. Discogenic pain is also axial in nature. c. There is no nerve root compromise; hence there is no radicular pain secondary to radiculopathy. d. Pain is usually localized but may radiate in a certain pattern in the lower extremities if

e.

f.

g. h. i.

the facet joints are the pain generators (Figure 18.1). The radiation pattern significantly overlaps between the involved joints. There is also anterior thigh and groin radiation from the L3–S1 facets. The pain usually does not radiate beyond the posterior thigh but may occasionally radiate up to the calf from the L5–S1 facet joint. Pain does not radiate to the foot as opposed to a radicular pattern of pain radiation. There are no paresthesiae associated with axial spine pain. Pain is usually felt as a deep dull ache, which can sometimes be described as sharp, especially if the muscular pain is also present.

4. Describe the anatomy and pathophysiology of the lumbar facet (zygapophysial) joint pain? The lumbar spine consists of: a. Five and sometimes six lumbar vertebral bodies and intervertebral discs between them. b. The L5 vertebral body articulates with the sacrum to form the L5–S1 facet joints. i. Each vertebra consists of (Figure 18.2): (1) Body (2) Transverse process (TP) (3) Laminar arches – joining in the middle to form the spinous process (4) Superior articular process (SAP) (5) Inferior articular process (IAP) (6) The pedicles join the body with the articular arches c. The intervertebral discs (IVD) join the vertebrae above and below and provide the articulation and cushioning to the spine (Figure 18.3):

Figure 18.1. Lumbar facet joint pain radiation pattern.

138

i. The IVD has two distinct parts: the tough fibrous outer rim called the annulus fibrosus and a softer inner core called the nucleus pulposus. ii. Disc herniation is a result of the nucleus bulging out or extruding through partial or complete tear of the annulus. iii. The disc derives its blood supply from the vertebral end plates above and below.

Chapter 18: Patient with lumbar facet-mediated pain

L1 Transverse process Superior articular process Lamina

Inferior articular process

L5 Sacrum

Figure 18.2. Lumbar spine anatomy.

Figure 18.4. Lumbar spine zones.

the posterior longitudinal ligament, the intervertebral foramen, the exiting nerve roots, plus the blood vessels to and from the spinal cord. The posterior segment is made up of the articular elements comprising the superior and inferior articular processes. Each vertebra articulates with the superior as well as inferior vertebral body. These articular joints are called the facet or zygapophysial joints (Figure 18.4).

5. How to diagnose lumbar axial pain? Diagnosis of axial lower back pain is based on the imaging studies (e.g., MRI of the lumbar spine, plain x-ray) as well as the history and physical examination. There are no unique identifying features in the history, physical examination, and radiologic imaging of patients with pain of lumbar zygapophysial (facet) joint origin.[3,4,5] a. MRI findings:

Figure 18.3. The intervertebral disc (IVD). NP – nucleus pulposus, AF – annulus fibrosus, SC – spinal cord, NR – nerve root, G.R. – gray ramus, SV.N. – sinuvertebral nerve, S.C. – sympathetic chain.

iv. Only the outer third of the annulus possesses the nerve endings responsible for pain generation. Clinically, the lumbar spine elements may be divided into three separate zones. The anterior segment is made up of the vertebral body, intervertebral disc, and the anterior longitudinal ligament. The middle segment contains the intervertebral foramen,

i. MRI of the lumbar spine may reveal disc degeneration, facet joint hypertrophy, and osteophyte formations which are the signs of lumbar spondylosis. ii. The facet joints may show narrowing, inflammation, and swelling. iii. CT scan is not a reliable test for facet joint syndrome.[6] b. Plain x-ray of the lumbar spine may show any anatomical displacement in the form of anterior or posterior displacement of the vertebral body (spondylolisthesis) or a fracture. It may also reveal any congenital defects such as a pars articularis defect. c. Work history is important in a chronically worsening low back pain. Any history of injury with an extension type of impact may lead to facet joint pain and dysfunction.

139

Chapter 18: Patient with lumbar facet-mediated pain

i. Facet joint pain is usually increased with extension and rotation on the ipsilateral side of the lumbar spine and may also radiate in a specific pattern but rarely extends beyond thigh or calf level. It is usually not felt when rising from a sitting position as opposed to SI joint pain.[7] ii. Increased pain with flexion with or without any paresthesiae may indicate a disc as the pain generator. iii. Superimposed myofascial pain is usually secondary to the spine pain and may exacerbate overall pain during flexion as well as extension and cause stiffness of the lower back, restricting all the movements. iv. Complaints of radiating pain vs. non-radiating (axial) pain with paresthesiae and/or numbness and weakness would help in differentiating the facet vs. nerve root related pain. d. Diagnostic block of the medial branches is the only reliable diagnostic approach for this pain syndrome.[8]

6. What are the treatment options for axial low back pain due to the facets? Conservative approaches Conservative treatment for axial low back due to the facets is largely based on the intensity of pain,

radiologic findings, and the patient’s ability to perform day-to-day activities. Physical therapy, antiinflammatory agents, and mild oral analgesics are the mainstay of the conservative treatment options. TENS unit may also be helpful particularly if the myofascial pain is the main source of a patient’s pain.

Interventional treatments i. Minimally invasive treatment options are required for patients who fail to improve despite adequate trials of physical therapy, inability to perform physical therapy due to increased pain, inadequate response to medications, or inability to tolerate medications due to side effects. Certain degenerative states do not respond adequately to conservative measures and require interventional treatment options. ii. Facet joint procedures: (1) Intra-articular steroid injection for the facet joints has moderate evidence for efficacy for lumbar facet joints.[9,10] Patients who do not respond to the intra-articular injections or who have advanced degenerative changes, require ablation of the facet joint nerves. (2) Facet joints are supplied by the medial branches of the lumbar nerve roots. Each joint is supplied by two medial branches, one from the same level and one from the level above, thus L3–4 facet joint is supplied by the L3 and L2 medial branches (Figure 18.5). Hence to

Figure 18.5. Lumbar facet joint intra-articular injection under oblique fluoroscopic view.

140

Chapter 18: Patient with lumbar facet-mediated pain

B

A

Figure 18.6. (A,B) Lumbar facet medial branch blocks. Images demonstrate a left sided L4 medial branch which lies over the left L5 transverse process and a right sided L5 medial branch block which lies in the groove between the right sacral superior articular process and the ala of the sacrum.

denervate these joints one has to lyse two medial branches per level. The evidence for radiofrequency ablation of the lumbar medial branches is moderate to strong.[10] (3) A carefully performed diagnostic medial branch block (Figure 18.6) on two different occasions using two different local anesthetics provides the most reliable diagnostic criteria to identify the facets as the pain generators.[11,12] The evidence for diagnosis of lumbar facet joint pain with controlled local anesthetic blocks is Level I or II-1. False-positive rate for a single diagnostic block is 27–47%.[12] Radiofrequency ablation is then performed to denervate these joints for long term (Figure 18.7). Level II-2 or II-3 evidence for radiofrequency neurotomy.[12]

7. Procedural description a. The approach for intra-articular facet joint injection is from a postero-lateral approach under fluoroscopic guidance (Figure 18.5). More recently, ultrasound-guided injections have also

been published and found to be as effective as fluoroscopically guided injection.[13] i. During an oblique view it is important to obtain a proper angle as the joint is most visible on fluoroscopy at its middle portion rather than the posterior joint border and hence the fluoroscopic view should be halted just as the joint becomes visible while going from an AP to oblique view. ii. A small amount of radiologic contrast medium to confirm intra-articular placement followed by a small amount of injectate consisting of local anesthetic and steroid is preformed. The total amount should not exceed 1 ml as the normal joint space is limited in a degenerate joint. b. Facet joint medial branch block: i. The block is ideally performed under fluoroscopic guidance, although recently it has been performed by some practitioners with the use of ultrasound guidance. ii. Posterolateral (oblique) view (Figures 18.5 and 18.6) is ideal for performing this block. However, the angle depends on the level to be injected to almost AP at higher levels.

141

Chapter 18: Patient with lumbar facet-mediated pain

B

A

Figure 18.7. (A,B) Note the tangential approach to the left L4 medial branch on an oblique view and the corresponding image demonstrating the needle position in a lateral view.

A

B

Figure 18.8. (A,B) Radiofrequency ablation of the right L5 medial branch at the junction of the ala of the sacrum and its corresponding superior articular process on AP and lateral fluoroscopic views.

c. Radiofrequency ablation: i. After a successful double diagnostic block of the related medial branches to diagnose the pain generator, radiofrequency ablation can be performed. ii. The approach is slightly different than an intra-articular injection in that the needle should be placed parallel to the medial branch

142

with a tangential approach from one level below to obtain an optimal lesion (Figures 18.7 and 18.8). iii. An AP and a lateral view is a must to confirm the needle tip posterior to the neural foramen. iv. A 10 or 15 cm radiofrequency needle with a 10 or 15 mm active tip is preferred. Needle gauge of 20 or 22 is commonly utilized.

Chapter 18: Patient with lumbar facet-mediated pain

v. The sensory and motor stimulation is then carried out to assure proper proximity to the medial branch and also to confirm its distance from the exiting nerve root. vi. After a successful placement, local anesthetic (and sometimes combined with a small amount of steroid) is injected to prevent pain from the lesion generation. Usually about 0.5 ml is injected. A radiofrequency lesion is generated at 80°C for 90 seconds.

8. What are the possible complications from these procedures? Risk of infection, bleeding, failure to relieve pain, or nerve damage is similar to any lumbar spine procedure but perhaps less with the medial branch block as it only involves local anesthetic injection and is much more posterior to the neural foramen. The radiofrequency lesioning may cause nerve damage if proper protocols are not observed and the

References 1.

2.

3.

4.

5.

Grgić V. Lumbosacral facet syndrome: functional and organic disorders of lumbosacral facet joints. Lijec Vjesn. 2011; 133(9–10):330–336. Steib K, Proescholdt M, Brawanski A, et al. Predictors of facet joint syndrome after lumbar disc surgery. J Clin Neurosci. 2012;19(3):418–422. Dreyer SJ, Dreyfuss PH. Low back pain and the zygapophysial (facet) joints. Arch Phys Med Rehabil. 1996;77(3):290–300. Schwarzer AC, Aprill CN, Derby R, et al. Clinical features of patients with pain stemming from the lumbar zygapophysial joints: is the lumbar facet syndrome a clinical entity? Spine. 1994; 19(10):1132–1137. Schwarzer AC, Wang SC, Bogduk N, et al. Prevalence and clinical features of lumbar zygapophysial

needle tip is too close to the nerve roots. Sedation may also lead to complication if the patient is heavily sedated and cannot respond in an appropriate manner. A short period of increased pain may be due to muscular spasms and myofascial pain. The majority of complications are short lived.

9. What are the outcomes with facet joint procedures? a. Diagnostic lumbar facet joint blocks are safe and reliable modalities to diagnose the facet joint related pain. A double block technique is preferred to minimize false-positive rates. The strength of evidence for this procedure is Level I or II-1 based on multiple controlled trials.[12] b. The level of evidence for radiofrequency ablation of the medial branches to treat the facet joint related pain is Level II to II-3 based on studies with a recommendation of 1B or 1C for this procedure.[12]

joint pain: a study in an Australian population with chronic low back pain. Ann Rheum Dis. 1995;54(2):100–106. 6.

7.

8.

9.

Schwarzer AC, Wang SC, O’Driscoll D, et al. The ability of computed tomography to identify a painful zygapophysial joint in patients with chronic low back pain. Spine. 1995;20(8): 907–912. Young S, Aprill C, Laslett M. Correlation of clinical examination characteristics with three sources of chronic low back pain. Spine. 2003;3(6): 460–465. Cohen SP, Huang JH, Brummett C. Facet joint pain: advances in patient selection and treatment. Nat Rev Rheumatol. 2013; 9(2):101–116. Boswell MV, Colson JD, Sehgal N, et al. A systematic review of therapeutic facet joint interventions in chronic spinal

pain. Pain Physician. 2007;10(1): 229–253. 10. Boswell MV, Colson JD, Spillane WF. Therapeutic facet joint interventions in chronic spinal pain: a systematic review of effectiveness and complications. Pain Physician. 2005;8(1):101–114. 11. Atluri S, Datta S, Falco FJ, et al. Systematic review of diagnostic utility and therapeutic effectiveness of thoracic facet joint interventions. Pain Physician. 2008;11(5):611–629. 12. Datta S, Lee M, Falco FJ, et al. Systematic assessment of diagnostic accuracy and therapeutic utility of lumbar facet joint interventions. Pain Physician. 2009;12(2):437–460. 13. Yun DH, Kim HS, Yoo SD, et al. Efficacy of ultrasonographyguided injections in patients with facet syndrome of the low lumbar spine. Ann Rehabil Med. 2012;36(1):66–71.

143

Section 2 Chapter

19

Spinal Disorders

Discogenic pain in the setting of lumbar spondylosis James Kelly and Jianguo Cheng

Case study A 45-year-old construction worker presents complaining of low back pain with radiation into the buttock and posterior thigh. He is otherwise healthy and his work consists of jack hammering and shoveling. He notes a gradual onset of his pain over the last 2 to 3 months and describes it as aching in nature. It is worsened by sitting or standing for long periods of time and also by transitioning from sitting to standing. He is unsure whether his pain is worsened by sneezing and denies numbness in his lower extremities or the loss of bowel or bladder control. He has taken over-thecounter naproxen sodium for the last week with little relief. He feels that the only thing that helps to alleviate his pain is lying down.

1. What is the differential diagnosis? a. b. c. d. e. f. g.

Muscle strain or sprain Lumbar facet disease Sacro-iliac joint pain Discogenic pain Vertebral compression fracture Spinal stenosis Aceto-femoral osteoarthritis

Lumbar degenerative disc disease is a leading cause of low back pain. It often occurs in younger, relatively healthy individuals. The onset is gradual in nature. It is often described as a deep pressure or ache that becomes intolerant upon activities that load the axial spine such as prolonged sitting or standing or transitioning from sitting to standing. Pain is also often worsened with maneuvers that raise intradiscal pressure (coughing, sneezing, bearing down, etc.). Positions that unload the spine, such as lying down,

often reduce or eliminate pain. The pain experienced is normally axial but may radiate/refer into the buttock or posterior thigh in a non-dermatomal fashion. Discogenic pain alone should not cause overt weakness or radiate below the knee. Straight leg raise testing is often negative and deep palpation over the spinous process of the affected disc may reproduce pain. Normal lumbar range of motion may be limited and symptoms are often worsened upon flexion. Reflexes and sensation are within normal limits and gait is most often normal. The diagnosis is usually established after excluding the other conditions listed above. It is noteworthy though that discogenic pain may coexist with one or more of these listed conditions.

2. What is the anatomy of a healthy intervertebral disc? A normal intervertebral disc consists of an inner nucleus pulposus and an outer annular fibrosus. The nucleus pulposus is sparsely populated with chondrocyte-like cells while cells in the annular fibrosus have fibrocyte-like features. The nucleus pulposus is primarily made up of water and proteoglycans, giving it a gelatin-like consistency. It is contained within the annulus fibrosis’ layered collagen, forming a high-pressured cushion that is well suited to withstand repetitive, constant stress. The intervertebral disc is the largest avascular structure in the body: its nerves are mostly mechanoreceptors and the blood vessels are typically found only in the outer third of the annulus fibrosus. This relatively anaerobic environment predisposes the disc, especially the inner nucleus pulposus, to degeneration in the face of injury or metabolic derangement. For

Case Studies in Pain Management, ed. Alan David Kaye and Rinoo V. Shah. Published by Cambridge University Press. © Cambridge University Press 2015.

144

Chapter 19: Discogenic pain in the setting of lumbar spondylosis

this same reason, discs are also prone to spontaneous or iatrogenic infection. Intervertebral discs derive their innervation from plexuses along the anterior and posterior intervertebral ligaments. The anterior and posterior plexuses both receive their input from the gray rami communicans. The posterior plexus also receives contributions from the sinuvertebral nerve that arises from each spinal level.

3. What risk factors predispose patients to develop degenerative disc disease? a. b. c. d. e. f. g.

Advanced age Trauma Certain athletic activity Obesity Vascular disease Family history Increased mechanical stress

4. What is the pathophysiology of degenerative disc disease? The intervertebral disc was first presumed to be a pain generator in 1947 by Inman. Mechanically, KirkaldyWillis in 1978 described a three phased cascade of degeneration marked by dysfunction, instability, and finally, stabilization. The cascade involves the progression of an acute injury to a chronic disarrangement due to poor body mechanics, immobility, and lack of activity with resultant ligamentous alterations (“my back gives out”) and eventual spondylytic changes (“my back is always stiff”). Biologically, dysfunction and decline in the viable cells of the nucleus pulposus, coupled with an increase in cytokines and proinflammatory mediators within the disc, start a vicious cycle that results in the reduction of the proteoglycan content. This change in matrix content increases the compressibility of the nucleus pulposus and, in turn, increases the pressure applied to the annulus fibrosus. Buckling of the annular lamellae results in microfractures and fissures. This serves as a substrate for neovascularity and migration of annular nociceptors inward toward the nucleus pulposus. In the setting of increased proinflammatory mediators, the nociceptors are most likely sensitized and prone to hyperesthesia and hyperalgesia.

5. What is the role of radiographic imaging in discogenic pain? Plain radiographs are of limited value for discogenic pain evaluation. Because loss of disc height and sclerotic bone changes are late findings, plain films lack in sensitivity for diagnosing degenerative disc disease. Computed tomography provides better bony detail and inferences as to the soft tissue than radiographs but MRI has quickly become the modality of choice for disc investigation. MRI protocols consist of T1- and T2-weighted sagittal and axial imaging. Contrast-enhanced studies are reserved for imaging of postoperative changes or in patients who are suspected of having an infection or tumor. Many practitioners find T2-weighted sagittal images to be most useful in analyzing the intervertebral discs as the overall shape, its hydration, and inferences as to the neural elements in the spinal canal and foramen are best appreciated. A grading system of five grades was developed by Pfirrmann in 2001 to assess and standardize disc degeneration as appreciated on MRI. Grade I discs, as seen in normal adolescents, have signal equal to that of the cerebrospinal fluid, a homogeneous contour, no loss of height, and a clear distinction between the annulus fibrosus and the nucleus pulposus. Grade II discs, as seen in normal adults, are similar to Grade I discs but may contain an inhomogeneous contour and gray horizontal bands. Grade III discs, classified as being mildly degenerated, show a gray signal, decreased body height, and an indistinct border between the nucleus pulposus and the annulus fibrosus. Grade IV discs, classified as being moderately degenerated, are gray or black, show normal to moderate loss of height, and have complete loss of the border between the nucleus pulposus and annulus fibrosus. Grade V discs, severely degenerated, are black with a collapsed disc space. These disc variations are often compared to the three types of endplate and subchondral bone marrow changes classified by Modic in 1988 (Figure 19.1). Modic Type 1 involves disruption and fissuring of the endplate with regions of degeneration, regeneration, reactive bone formation, endplate edema, and infiltrative vascular granulation tissue. This is reflected on MRI as an edema pattern of hypointense T1-weighted imaging and hyperintense T2-weighted imaging. Modic Type 2 changes are hyperintense on T1-weighted imagine and isointense or slightly hyperintense on

145

Chapter 19: Discogenic pain in the setting of lumbar spondylosis

A

B

C

Figure 19.1. Type 1(A), type 2(B), and type 3(C) modic endplate changes. With permission from reference[9].

T2-weighted imaging and are associated with conversion of normal red hemopoietic bone marrow into yellow fatty marrow as a result of marrow ischemia. Modic Type 3 changes are described as hypointense on both T1- and T2-weighted imaging and are thought to

146

represent subchondral bone sclerosis. Mixed-type 1/2 and 2/3 Modic changes have also been reported, suggesting that these changes can convert from one type to another and that they all present different stages of the same pathologic process. The absence of Modic changes, a normal anatomic appearance, has often been designated Modic type 0. Modic Type 1 changes intuitively represent inflammation and are believed to best correlate to segmental instability and discogenic low back pain. Most recently, Modic Type 1 change has been closely linked to bacterial infection of the herniated disc in several studies. The most commonly isolated pathogen from disc tissues obtained from spine surgeries is Proprionibacterium acnes. These studies convincingly linked chronic low back pain to bacterial infection of the herniated discs and the subsequent changes of subchondral bone marrow changes. A recent doubleblind randomized, placebo-controlled clinical trial clearly demonstrated the efficacy and safety of antibiotic treatment in patients with chronic low back pain and Modic Type 1 changes. Taken together, this line of studies signifies a new era of etiologic treatment of certain types of chronic low back pain. There has been an increased interest in highintensity zones (HIZ) on MRI. HIZ are best viewed on T1-weighted images and represent a tear in the posterior annulus. They have been found to correlate closely with the pain of discography and represent yet another tool to diagnose discogenic pain. However, it remains controversial on the correlation between HIZ and discogenic pain and some authors argue that psychologic components are better predictors. Lumbar imaging is not without its drawbacks. It is not only that grading of lesions varies greatly between observers, but more importantly, disc abnormalities are appreciated in 64% of asymptomatic patients. Thus, the high sensitivity and low specificity of the imaging studies emphasize the need that the imaging results must be carefully correlated to findings from the history and physical examinations.

6. Is there a gold standard for the diagnosis of discogenic pain? Provocative discography has long been considered the gold standard for the diagnosis of discogenic pain. It was first introduced by Schmorl and Junghanns and was first performed in the US at the Cleveland Clinic by Wise and Weiford in 1951. The key diagnostic

Chapter 19: Discogenic pain in the setting of lumbar spondylosis

feature is the reproduction of the patient’s low back pain and this process requires the cooperation of the patient. It is important to note that discogenic pain is a different diagnosis from disc herniation. The North American Spine Society Diagnostic and Therapeutic Committee suggest that discography should only be used when a patient has failed an adequate course of non-operative treatment and non-interventional tests such as MRI have failed to provide sufficient diagnostic information. Further, the group asserts that discography should not be performed in the setting of mild to moderate, acute low back pain. Accordingly, many physicians feel that, given the invasive nature of discography, patients should be trialed with less invasive procedures such as medial branch blocks and trigger point injections prior to the procedure. The American Association of Neurologic Surgeons/Congress of Neurologic Surgeons asserts that positive discography in the setting of normal MRI findings should be considered a contraindication to surgical or other invasive interventions. Discography involves accessing the disc that is the suspected pain generator and two to three control discs. The patient is placed in the prone position and the lower back prepped and draped. Anterior– posterior fluoroscopic images with alignment of the end plates of the adjacent vertebral bodies are first obtained to provide good visualization of the disc. The C-arm is then oblique until the superior articular process reaches the midline of the corresponding vertebral endplate. The entry point is marked just lateral to the superior articular process and local anesthetic infiltrated. A two-needle technique has been shown to reduce the rate of discitis to 0.7%, thus an 18-gauge needle is advanced roughly two inches. A 6-inch, 22gauge needle is inserted through the 18-gauge needle. Care is taken not to touch the tip of the 22-gauge needle. The disc is found to have a rubbery texture when encountered. At this point, AP and lateral images should be obtained to assure that the needle is in the center of the disc. After placement of the needles in the control discs, the opening pressure is noted and non-ionic contrast administered into the disc. Concordant pain is sought with an increase of less than 30 psi above the opening pressure. Concordant is most often defined as significant pain in the distribution of the patient’s normal pain. Most practitioners start with a control disc and

pressure should not be raised above 90 psi to avoid false-positive results. Many practitioners consider pain in two or more control discs to suggest that the study is invalid. Furthermore, the subjective nature of the test adds an additional confounding factor. Moreover, patients with elevated scores on the hysteria and hypochondriasis scale of the Minnesota Multiphasic Personality Inventory were significantly more likely to report pain during injection than controls. See Figure 19.2.

7. Does the spread of the contrast on a discogram tell us anything? Efforts were made to standardize visual results obtained from discograms. In 1986 Adams et al described fluoroscopic classification of discs on cadavers. He reported 87% reproduction of his results by repeating the procedure 6 months after the initial discogram. Following the procedure, the morphology can be further evaluated by CT imaging. The Adams classification for discogram morphology describes five types of results. Type I is described as a cotton ball. The dye is well contained within the nucleus and the density is uniform. Type II, termed lobular, results when the dye is distributed within the nucleus pulposus in two distinct lobes. Type III is an irregular dye pattern with evidence of penetration of the dye into the inner annulus fibrosus. This is appreciated by a speckled dye pattern. Type IV morphology shows evidence of fissures. The contrast reaches the outer annulus fibrosus and may even extend beyond the edge of the vertebral body if the disc is herniated. A Type V contrast pattern shows rupture of the disc with spread of the dye into the epidural space. Adams and colleagues’ initial study suggested that discogram morphologic results were reproducible. Later studies have looked at the interobserver and intraobserver agreement and the reliability of the scale. In the clinical setting the absolute interobserver and intraobserver agreement occurred in 82 levels or 62%, regardless of level of experience (Agorastides et al 2002).

8. What are the complications of discography? Potential complications of discography include pain at the entry site, increased disc pain, discitis, epidural

147

Chapter 19: Discogenic pain in the setting of lumbar spondylosis

A

B

Figure 19.2. (A) Anteroposterior view of lumbar discogram L2–3, L3–4, L4–5, L5-S1; with left L5-S1 contrast extravasation through high-grade annular tear. (B) Post discogram CT scan with thin cross-sectional slices. From personal files of Rinoo V. Shah, MD, MBA.

hematoma, transient radiculopathy, spinal headache, arachnoiditis, and pulmonary embolism. Lumbar discography is associated with complications in 0% to 2.7% of patients. There is no evidence to support that lumbar discography causes damage or causes lumbar discs to herniate. The most common complication of discography is increased pain. This occurs in up to 81% of individuals undergoing the procedure. There is a correlation between patients who had significant back pain 1-year post discography and significant emotional, psychologic, and chronic pain problems. Discitis remains the most common serious complication. Aside from standard prep, drape, and scrub, many practitioners employ a broad spectrum, parenteral antibiotic prior to discography. A one-time dose of cafazolin roughly 30 minutes prior to the procedure is often the choice. However, Willem found the infections rate to be 0.25% in 4891 patients and 0.094% in 12 770 discs in which preprocedural antibiotics were not used. A twoneedle technique, as described earlier, has been touted as a more effective means by which to reduce infection.

148

9. What treatment modalities are available for discogenic pain? As difficult a disease as it is to diagnose, discogenic pain is an even more difficult entity to treat. As is the case when targeting other pain generators, it is recommended for practitioners to start with the least invasive options and mold the treatment protocol based on response. As with other chronic pain syndromes, psychiatric illness remains a barrier to substantial symptoms reduction. A recent exciting development is the isolation of Proprionibacterium acnes bacteria as a cause of low back pain. The results of the recent randomized controlled trial (RCT), based on the infected herniated disc hypothesis, have provided strong evidence that antibiotic therapy can lead to dramatic reduction in disability and pain in patients with Modic Type 1 change. It is quite hopeful that etiologic treatment for certain types of chronic discogenic pain may be within reach in the near future if confirmatory studies reproduce the reported results. Conservative measures such as alternating ice and heat and oral anti-inflammatories, neuromodulators, and analgesics provide only modest relief of symptoms.

Chapter 19: Discogenic pain in the setting of lumbar spondylosis

A

B

Figure 19.3 Lateral (A) and anteroposterior (B) fluoroscopic view of L4–5 IDET (intradiscal electrothermal therapy). From personal files of Rinoo V. Shah, MD, MBA.

Exercise and physical therapy remains a mainstay of treatment with a focus on core strengthening, traction, and flexibility. Many patients suffer from worsening deconditioning due to their fear that exercise will worsen their symptoms. In fact, exercise was related to increased flexibility and strength, reduced pain, and decreased negative behavior and beliefs about pain. When such a workout program was coupled with cognitive therapy results were even more significant in terms of reduced healthcare visits, work absenteeism, and taking long-term sick leave disability. Minimally invasive treatments have aimed to denature sensitized nociceptors. One such agent, methylene blue, has been postulated to denervate the small fibers that grow into the annulus fibrosus. One study reported a success rate of 89% following one treatment with intradiscal methylene blue. However, confirmatory studies remain to be seen to reproduce the reported stellar results. Intradiscal steroids and ozone have shown equally equivocal results.

Figure 19.4. IDET L3–4 and L4–5. From personal files of Rinoo V. Shah, MD, MBA.

149

Chapter 19: Discogenic pain in the setting of lumbar spondylosis

Patients with discogenic pain of duration greater than 6 months may be candidates for thermal ablation. Procedures such as intradiscal electrothermal therapy (IDET) and, more recently, biacuplasty, aim to not only ablate the sensitized nociceptors but also thermally modify collagen fibers located in the annulus. It is of note that thermal lesioning of the disc of a prone patient renders the nerve roots in a more vulnerable, anterior position. IDET was shown to be effective in select patients but the procedure has fallen out of favor due to a lack of consistency in terms of outcomes from randomized controlled trials. Biaculoplasty creates a lesion using bipolar electrodes that are easily placed as compared to IDET. Initial studies

References 1.

Buckwalter JA, Mow VC, Boden SD, et al. Intervertebral disk structure, composition and mechanical function. In Buckwalter JA, Einhorn TA, Simon SR, eds. Orthopaedic Basic Science: Biology and Biomechanics of the Musculoskeletal System. Rosemont, IL: American Academy of Orthopaedics Surgeons. 2000: pp. 548–555.

2.

Bogduk N, Tynan W, Wilson AS. The nerve supply to the human lumbar intervertebral discs. J Anat. 1981;132:39–56.

3.

Kirkaldy-Willis WH, Wedge JH, Yong-Hing K, Reilly J. Pathology and pathogenesis of lumbar spondylosis and stenosis. Spine. 1978;3:319–328.

4.

Buckwalter JA. Aging and degeneration of the human intervertebral disc. Spine. 1995;20:1307–1314.

5.

Kang JD, Georgescu HI, McIntyre-Larkin L. Herniated lumbar intervertebral discs spontaneously produce matrix metalloproteinases, nitric oxide, interleukin-6 and prostaglandin E2. Spine. 1996;21:271–277.

6.

Ledermann HP, Schweitzer ME, Morrison WB, Carrino JA. MR imaging findings in spinal infections: rules or myths? Radiology. 2003;228(2):506–514.

150

7.

8.

9.

involving few patients show improvements in functional capacity, pain scores, and opiate use at 1 and 6 months of observation. However, a recent RCT did not provide convincing evidence of efficacy. Theoretically removal of the diseased disc and stabilization through fusion should relieve the discogenic pain. Artificial disc replacement has also been trialed. However, results have been mixed at best. In fact, despite advances in surgical technology, the rates of failed back surgery syndrome have not declined. This speaks to the multifactorial nature of lumbar discogenic pain and the need for future research and innovation. See Figures 19.3 and 19.4.

Pfirrmann CW, Metzdorf A, Zanetti M, Hodler J, Boos N. Magnetic resonance classification of lumbar intervertebral disc degeneration. Spine. 2001;26(17): 1873–1878. Modic MT, Steinberg PM, Ross JS, Masaryk TJ, Carter JR. Degenerative disk disease: assessment of changes in vertebral body marrow with MR imaging. Radiology. 1988; 166:193–199. Rahme R, Moussa R. The Modic vertebral endplate and marrow changes: pathologic significance and relation to low back pain and segmental instability of the lumbar spine. Am J Neuroradiol. 2008;29(5):838–842.

10. Jensen MC, Brant-Zawadzki MN, Obuchowski N, et al. Magnetic resonance imaging of the lumbar spine in people without back pain. N Engl J Med. 1994;331(2): 69–73. 11. Peng B, Hou S, Wu W, et al. The pathogenesis and clinical significance of a high-intensity zone (HIZ) of lumbar intervertebral disc on MR imaging in the patients with discogenic low back pain. Eur Spine J. 2006;15: 583–587. 12. Carragee EJ, Paragioudakis SJ, Khurana S. Volvo award winner in clinical studies. Lumbar highintensity zone and discography in

subjects without low back problems. Spine. 2000;25(23): 2987–2992. 13. Wise RE, Weiford EC. X-ray visualization of the intervertebral disk: report of a case. Cleve Clin Q. 1951;18:127–130. 14. Willems PC, Jacobs W, Duinkerke ES, De Kleuver M. Lumbar discography: should we use prophylactic antibiotics? A study of 435 consecutive discograms and a systematic review of the literature. J Spinal Disord Tech. 2004;17:243–247. 15. Block AR, Vanharanta H, Ohnmeiss DD, Guyer RD. Discographic pain report: influence of psychological factors. Spine. 1996;21:334–338. 16. Adams MA, Dolan P, Hutton WC. The stages of disc degeneration as revealed by discograms. J Bone Joint Surgn Br. 1986;68-B:36–41. 17. Agorastides ID, Lam KS, Freeman BJ, Mulholland RC. The Adams classification for cadaveric discograms: inter- and intraobserver error in the clinical setting. Eur Spine J. 2002;11: 76–79. 18. Schreck RI, Manion WL, Kambin P, et al. Nucleus pulposus pulmonary embolism: a case report. Spine. 1995;22: 2463–2466

Chapter 19: Discogenic pain in the setting of lumbar spondylosis

19. Walker J, Omar EA, Zacharia I, Stefan M. Discography in practice: a clinical and historical review. Curr Rev Musculoskelet Med. 2008;1(2):69–83. 20. Johnson RG. Does discography injure normal discs? An analysis of repeat discograms. Spine. 1989;14(4):424–426 21. Kahanovitz N, Arnoczky SP, Sissons HA, Steiner GC, Schwarez P. The effect of discography on the canine intervertebral disc. Spine. 1986;11(1):26–27. 22. Tallroth K, Soini J, Antti-Poika I, et al. Premedication and short term complications in iohexol discography. Ann Chir Gynaecol. 1991;80:49–53. 23. Carragee EJ, Chen Y, Tanner C, Hayward C, Rossi M, Hagle C. Can discography cause long term back symptoms in previously asymptomatic subjects? Spine. 2000;25(14):1803–1808. 24. Buenaventura RM, Shah RV, Patel V, Benyamin R, Singh V. Systematic review of discography as a diagnostic test for spinal pain: an update. Pain Physician. 2007;10(1):147–64. Review. PubMed PMID: 17256028. 25. Shah RV, Everett CR, McKenzieBrown AM, Sehgal N. Discography as a diagnostic test for spinal pain: a systematic and narrative review. Pain Physician. 2005;8(2):187–209.

26. Rainville J, Hartigan C, Martinez E, et al. Exercise as a treatment for chronic low back pain. Spine J. 2004;4:106–115.

electrothermal therapy: a preliminary histologic study. Arch Phys Med Rehabil. 2001;82 (9):1230–1237.

27. Linton SJ, Boersma K, Jansson M, Svard L, Botvalde M. The effects of cognitive-behavioral and physical therapy preventive interventions on pain-related sick leave: a randomized controlled trial. Clin J Pain. 2005;21:109–119.

34. Kapural L, Ng A, Dalton J, Mascha E, et al. Intervertebral disc biacuplasty for the treatment of lumbar discogenic pain: results of a six-month follow-up. Pain Med. 2008;9(1):60–67.

28. Peng B, Zhang Y, Hou S, Wu W, Fu X. Intradiscal methylene blue injection for the treatment of chronic discogenic low back pain. Eur Spine J. 2007;16:33–38. 29. Gupta G, Radhakrishna M, Chankowsky J, Francisco Asenjo J. Methylene blue in the treatment of discogenic low back pain. Pain Physician. 2012;15:333–338. 30. Simmons JW, McMillin JN, Emery SF, Kimmich SJ. Intradiscal steroids: a prospective double-blind clinical trial. Spine. 1992;17:S172–S175. 31. Muto M, Ambrosanio G, Guarnieri G, et al. Low back pain and sciatica: treatment with intradiscal-intraforaminal O(2)O(3) injection. Our experience. Radiol Med. 2008;113:695–706. 32. Mekhail N, Kapural L. Intradiscal thermal annuloplasty for discogenic pain: an outcome study. Pain Pract. 2004;4(2):84–90. 33. Shah RV, Lutz GE, Lee J, Doty SB, Rodeo S. Intradiskal

35. Gibson JN, Waddell G, Grant IC. Surgery for degenerative lumbar spondylosis. Cochrane Database Syst Rev. 2000;2:CD001352. 36. Chan CW, Peng P. Failed back surgery syndrome. Pain Med. 2011;12(4):577–606. 37. Rahme R, Moussa R. The Modic vertebral endplate and marrow changes: pathologic significance and relation to low back pain and segmental instability of the lumbar spine AJNR Am J Neuroradiol. 2008;29: 838–842. 38. Kapural L. Vrooman B, Sarwar S, et al. A randomized, placebocontrolled trial of transdiscal radiofrequency, biacuplasty for treatment of discogenic lower back pain. Pain Med. 2013;14; 362–373. 32. Agorastides ID, Lam KS, Freeman BJ, Mulholland RC. The Adams classification for cadaveric discograms: inter- and intraobserver error in the clinical setting. Eur Spine J. 2002;11(1): 76–79.

151

Section 2 Chapter

20

Spinal Disorders

Unusual pain syndromes: epidural lipomatosis Vikram B. Patel

Case study A 58-year-old male with chronic low back pain had been receiving epidural steroids on a regular basis for several years for his symptoms. He has concurrent diagnoses of obesity, hypertension, and type 2 diabetes mellitus. He recently had a repeat epidural injection as the symptoms had gradually worsened over the last few months with increased low back pain which was now accompanied by tingling and deep ache in both legs as well as poor gait and balance.

1. What is the differential diagnosis? a. Worsening spinal degeneration b. Increasing lumbar spinal stenosis with possible neurogenic claudication c. Centrally herniated intervertebral disc causing compression of the nerve roots d. Cauda equine syndrome i. Loss of bowel control, urinary retention, bilateral. Lower extremity weakness and numbness, low back pain e. Thoracic or cervical disc herniation causing spinal cord compression

f. g. h. i.

i. May have accompanying cervical and upper extremity symptoms Epidural abscess Epidural hematoma Idiopathic spinal epidural lipomatosis secondary to corticosteroid administrations Other rare conditions such as multiple sclerosis, syringomyelia, transverse myelitis, etc.

Lumbar spine degeneration is a common occurrence and is frequently treated with epidural steroid injections. These injections are sometimes performed

on a regular basis for chronic conditions that do not respond to short-term treatments. Worsening of symptoms following an epidural injection should prompt a practitioner to evaluate the possible etiology. Although worsening symptoms may just be a progression of the patient’s usual pathology, additional symptoms such as lower extremity weakness, neurogenic claudication, paresthesiae, fever, etc. should be properly evaluated to rule out any serious condition.

2. What are the most common symptoms of idiopathic epidural lipomatosis? a. b. c. d. e. f.

Low back pain Radicular pain Sensory loss Burning dysesthesiae Motor weakness Bladder dysfunction

3. What is a proper approach to these symptoms? a. A thorough history and physical examination: i. History of injury, infections, fever, duration of symptoms, bowel and bladder dysfunction ii. Physical examination of the neurologic system is a must and should include gait and balance assessment b. Vital signs including temperature to rule out any infectious process c. Blood tests including WBC count, erythrocyte sedimentation rate, and C-reactive protein d. MRI of the lumbar spine

Case Studies in Pain Management, ed. Alan David Kaye and Rinoo V. Shah. Published by Cambridge University Press. © Cambridge University Press 2015.

152

Chapter 20: Unusual pain syndromes: epidural lipomatosis

Diagnosis of the condition requires an MRI of the lumbar spine. Several conditions causing these symptoms can be either confirmed or ruled out based on this study. Soft tissue and fluid (blood, pus, seroma) can only be properly assessed with an MRI. A bone scan may be required in some patients. In patients who cannot have an MRI (e.g., pace maker, spinal cord stimulator), a CT scan may be performed along with a myelogram to properly evaluate the spinal canal.

4. What are the findings during a physical examination in epidural lipomatosis?[1,2] a. On observation, one can sometimes observe unsteady and unbalanced gait b. Patient may complain of neurogenic claudication after walking only a short distance c. Sensory deficit can be elicited in the lumbar dermatomes (or higher dermatomes depending on the level of lipomatosis) d. Motor weakness in the lower extremities e. Decreased deep tendon reflexes f. Positive straight leg raising test

5. What is the pathophysiology of idiopathic epidural lipomatosis? a. Excessive deposits of adipose tissue in the epidural space b. It may be caused by endogenous elevation of steroids c. It is also known to occur in exogenous steroid therapy such as in immunosuppressive therapy after organ transplantation, steroid therapy for asthma, steroid therapy for rheumatoid arthritis, etc.[1–5] d. In most cases, the etiology is idiopathic e. Majority of these cases have been reported in morbidly obese patients, receiving steroid therapy or suffering from endocrinopathies such as Cushing’s syndrome, hypothyroidism, etc.[2–6]

6. What is the most reliable diagnostic modality? a. Magnetic resonance imaging is the most reliable imaging study[6]

7. How should this patient be treated? a. Patients with minimal or no spinal cord compression symptoms may be treated with conservative modalities such as reduction of steroid dosage, weight reduction, correction of underlying causes such as hypothyroidism b. Aggressive treatment is recommended in patients representing cord compression symptomatology to avoid long term and irreversible damage to the spinal cord and the nerve roots c. Surgical decompression is the most effective therapy d. Extensive laminectomies and subsequent fusion may be required for effective debulking of the epidural adipose tissue

8. What is the long-term outcome after treatment? a. In most cases recurrence is not seen after 2 years follow-up.[2] b. Early intervention can help prevent permanent spinal cord or nerve root injury

Summary Epidural lipomatosis is a rare condition that requires aggressive treatments especially if the spinal cord or the nerve roots are compressed. It may affect any age group but is more common in middle aged patients. Exogenous steroid therapy is the most common cause for this condition. The condition arises when there is excessive deposition of adipose tissue in the epidural space which is in most cases circumferential. Likely risk factors such as morbid obesity, hypothyroidism, Cushing’s syndrome, and immunosuppressive therapy should be considered in patients receiving steroid therapy on a chronic basis. Magnetic resonance imaging is the gold standard for evaluating such symptoms due to its higher reliability to differentiate the soft tissues. Aggressive therapy with extensive laminectomies and fusion are required in patients exhibiting signs and symptoms of spinal cord or nerve root compression. Milder symptoms may be managed conservatively.

153

Chapter 20: Unusual pain syndromes: epidural lipomatosis

References 1.

Fessler R, Johnson D, Brown F, et al. Epidural lipomatosis in steroid-treated patients. Spine. 1992;17(2):183–188.

2.

Lisai P, Doria C, Crissantu L, et al. Cauda equina syndrome secondary to idiopathic spinal epidural lipomatosis. Spine. 2001;26(3):307–309.

154

3.

Chapman PH, Martuza RI, Poletti CE, et al. Symptomatic spinal epidural lipomatosis associated with Cushing’s syndrome. Neurosurgery. 1981;8: 724–727.

4.

Kumar K, Nath RK, Nair CPV, et al. Symptomatic epidural lipomatosis secondary to obesity. J Neurosurg. 1996;85: 348–350.

5.

Russell NA, Belanger G, Benoit BG, et al. Spinal epidural lipomatosis: a complication of glucocorticoid therapy. Can J Neurol Sci. 1984;11:383–386.

6.

Toshniwal PK, Glic RP. Spinal epidural lipomatosis: Report of a case secondary to Hypothyroidism and review of literature. J Neurol. 1987;234(3): 172–176.

Section 2 Chapter

21

Spinal Disorders

Unusual pain syndromes: Bertolotti’s syndrome Jiang Wu and Jianguo Cheng

Case study A 34-year-old woman presents with a chief complaint of left low back pain for 1 year. She describes the pain as deep aching, and progressively getting worse in the last few months. The pain radiates into her left hip and posterior thigh above the knee. It is better with rest and worse with physical activity particularly with extension or e.g. bending to the left side. The use of NSAIDs and core strengthening exercise failed to relieve the pain. She has tenderness over the low back left to the lumbar spine. She was suspected to have spondylosis and facet arthropathy. On the day of a scheduled diagnostic facet medial branch block, she was found under fluoroscopy to have a lumbosacral transitional vertebra that articulated with the left ilium through an enlarged transverse process. A diagnosis of Bertolotti’s syndrome was thus established.

1. What is Bertolotti’s syndrome? Lumbosacral transitional vertebra is an anatomical variation of the most caudal lumbar vertebra in which an enlarged transverse process can articulate or fuse with the sacrum or ilium. The association of this congenital variant with chronic low back pain and the change in the biomechanical properties of the lumbar spine is called Bertolotti’s syndrome.[1] Although Bertolotti’s syndrome is a congenital abnormality, it is often clinically manifested only after the second decade of life.[2] It is estimated this syndrome accounts for 4.6%[2] to 7%[1] of cases of low back pain in adults, and for more than 11% of patients with low back pain who are under 30 years old.[2] The differential diagnosis of Bertolotti’s syndrome includes sacroiliac joint pain, myofascial pain, lumbar disc herniation (DDD), lumbar facet pain, and stress fracture.

2. Describe the pathophysiology of Bertolotti’s syndrome The incidence of lumbosacral transitional vertebra is 4–8% in the general population.[1] Although it was stated as early as in 1917 that these abnormal vertebrae may produce low back pain[3] and an association has been found between lumbosacral transitional vertebra and disc herniation as well as facet joint degeneration,[4,5] little is yet known about the biomechanical and pathophysiologic effects of such abnormal vertebra. It has been hypothesized that the primary effect of lumbosacral transitional vertebra is the reduced and asymmetrical motion between the transitional vertebra and the sacrum. This asymmetry can result in early arthritic changes occurring at “pseudoarticulation,” which is similar in mechanism to a surgical pseudarthrosis in which repeated motion over an unstable bony bridge of fibrous mass results in local limitation and inflammation.[6] The impingement of enlarged transverse process on the nerve root extra-foraminally could result in sciatica.[7] The secondary effects of such abnormal vertebra are the progressively compensatory modifications in the biomechanics of the mobile vertebral segments superior to the transitional vertebra due to the restriction of rotation and bending motion at the lumbosacral articulation.[8] The disc above a transitional vertebra appears to predispose to degenerative changes, whereas the disc below appears to be protected.[5,9–11] Secondary degeneration and strain of the supra-adjacent disc cause discogenic pain or inflame the adjacent lumber nerve root resulting in “sciatic” or radicular pain patterns.[5] Generating abnormal weight overload in the opposite articular facets results in ongoing lumbar facet pain.[12]

Case Studies in Pain Management, ed. Alan David Kaye and Rinoo V. Shah. Published by Cambridge University Press. © Cambridge University Press 2015.

155

Chapter 21: Unusual pain syndromes: Bertolotti’s syndrome

In addition to the progressive modifications, the instability above the transitional vertebrae because of a weak iliolumbar ligament also leads to dysfunctional motion and muscle strain pain.[9,13–15]

3. What is Castellvi’s classification? The Castellvi classification was proposed to morphologically characterize four types of lumbosacral transitional vertebrae:[4] Type I – dysplastic transverse process with height > 19 mm Type II – incomplete lumbarization/sacralization (diarthrosis) Type III – complete lumbarization/sacralization with complete fusion with the neighboring sacral basis Type IV – mixed The type II transitional vertebra has been associated with an increased number of disc prolapses, and related to discogenic and/or contralateral facetogenic low back pain.[4,16]

4. How does Bertolotti’s syndrome manifest clinically? Patients with Bertolotti’s syndrome usually present with chronic, progressive midline, or paramedian low back pain that is deep, sharp, or dull in nature or a sensation of pulled muscle or unilateral upper buttock pain. The severity of pain is moderate to severe, worse with physical exertion, and better with rest. The provocative factors of pain include heavy lifting, forward flexion, excessive extension or lateralization of the back to the same side of the megaapophysis. It may be accompanied with sciatica, medial thigh cramping, leg radicular pain, weakness, or numbness. Patients may have significant ambulatory and functional limitations. Physical examination demonstrates focal tenderness along the base of the lumbosacral spine and near the posterior-superior iliac spine, provoked by superficial and deep palpation. Patients may have normal and symmetric muscle bulk and tone in their paraspinal muscles and in all extremities. Laséque’s sign, or straight leg raise test, may be positive; range of motion may be impaired. Pronator drift, Romberg signs, patellar and Achilles deep tendon reflexes, and sensation test usually are intact.

156

5. How do you diagnose a Bertolotti’s syndrome? Because of unapproved association of this congenital variant with chronic low back pain and poor understanding of the biomechanics of such abnormal vertebrae, Bertolotti’s syndrome is very difficult to recognize. The correct diagnosis is made based on imaging studies which included lumbosacral CT scans, plain x-rays, and MRI scans in the context of typical history of low back pain and physical exam. The extension-flexion lumbosacral radiographs in anteroposterior, lateral, and oblique views demonstrate lumbosacral transitional vertebra, with an enlarged unilateral or bilateral transverse processes of the most distal lumbar vertebra, abnormally articulating with the ala of the sacrum and degenerative changes of the pseudarthrosis. In addition, plain radiographs can reveal lumbar lordosis and/or disc space narrowing. In addition to showing sacralization of the most distal lumbar vertebra with pseudoarticulation of the enlarged transverse process and the ala, CT scan of the lumbosacral spine may help identify associated stenosis, osteophytes, and areas of sclerosis surrounding the pseudoarticulation. MRI provides detailed anatomic information regarding degenerative disc disease and possibly associated disc herniation. It shows degeneration and desiccation of disc spaces, any central canal or foraminal stenosis, and the degree of compression of the dural sac or spinal nerves. A selective radiculogram of the spinal nerve may demonstrate entrapment of the spinal nerve in the extra-foraminal zone. Bone scintigraphy may reveal an inflammatory process within the articular facets, specifically at the level of the mega-apophysis. Singlephoton emission CT may be useful in the identification of possible candidates for local anesthetic infiltration and future radiofrequency ablation.[17]

6. How should I treat this patient? A course of conservative management including activity modification, medication management with NSAIDs, muscle relaxants, opioids, and rehabilitative physical therapy should be offered initially, with the recognition that these therapies are less likely to result in satisfactory pain control.

Chapter 21: Unusual pain syndromes: Bertolotti’s syndrome

Due to the multifactorial etiology of low back pain in patients with Bertolotti’s syndrome, the identification of the primary and secondary origin of the pain become paramount to choose the most appropriate treatment for each case. If the primary origin of the pain is at the pseudoarticulation between the transverse process and ilium due to the arthritic changes, a local anesthetic and corticosteroid can be injected into this pseudoarticulation after contrast dye confirmation of correct spread in or around the pseudoarticulation under fluoroscopic guidance.[6,15,18] These blocks should be performed with a minimal amount of anesthetic delivered precisely to the point of interest to achieve temporary pain relief. If rapid pain relief doesn’t occur almost instantaneously, alternative pain generators should be sought. The anesthetic block can be repeated if in doubt. If the patient experiences temporary pain relief and the pain is truly localized in the transitional joint without evidence of disc pathology, a minimally invasive approach may be taken to resect the anomalous transverse process with the accompanying pseudoarticulation.[12] If still unsuccessful, based on the hypothesis that the rigidity of the L5–S1 portion of the spine puts extra stress on the L4–5 level, further surgical intervention either to free the rigid level or to further stabilize the spine by resection or fusion of the anomalous transverse process to protect L4–5 could be considered.[3] Due to progressively compensative modifications in the biomechanics of the mobile vertebral segments superior to the transitional vertebra, multiple secondary origins of pain may exist. If the degenerative changes occur at the contralateral facet joints as a possible source of pain, then diagnostic medial branch blocks should be performed and, if positive, followed by radiofrequency sensory ablation of these

References 1.

2.

Elster AD. Bertolotti’s syndrome revisited: transitional vertebrae of the lumbar spine. Spine. 1989; 14(12):1373–1377. Quinlan JF, Duke D, Eustace, S. Bertolotti’s syndrome: a cause of back pain in young people. J Bone Joint Surg Br. 2006; 88(9):1183–1186.

3.

4.

joints.[18] If the disc above the transitional vertebra is thought to be the source of pain, then discography may be useful in diagnosis. The therapeutic armamentarium for degenerative disc disease includes surgical microdiscectomy, nucleolysis, and arthrodesis.[2] The latter involves pedicular screws, transforaminal lumbar interbody fusion (TLIF), posterior lumbar interbody fusion (PLIF) and, more recently, anterior lumbar interbody fusion (ALIF).[8] If a spinal nerve is impinged in the extraforaminal zone by disc herniation or transverse process, surgical options including fenestration, posterolateral fusion, and transforaminal interbody fusion may be considered.[7] Microendoscopic decompression may be particularly useful in this location.[19]

7. What are the outcomes? There is a paucity of literature regarding interventional outcomes. Conventional radiofrequency neurolysis may be used for facetogenic pain, providing significant pain relief and aiding future physical rehabilitation programs.[20] However, the relief may be temporary and repeated procedures may be necessary.[8] Minimally invasive techniques have been developed to limit iatrogenic soft tissue injury during exposures for spine surgery. These techniques provide adequate exposure of the involved spinal segment with limited tissue destruction and retraction. It has been hypothesized that limiting iatrogenic tissue injury may reduce chronic back pain years after surgery.[21–24] Surgical treatment of Bertolotti’s syndrome was only slightly better than conservative treatment and should only be used in very selective patients with disc pathology.[15] In order to achieve long-term improvement by any therapeutic options, a continuing physical rehabilitation program is often needed.

Bertolotti M. Contributo alia conoscenza dei vizi di differenzazione regionale del rachide con speciale riguardo all assimilazione sacrale della V. lombare. Radiologique Medica 1917;4:113–144. Castellvi AE, Goldstein LA, Chan DPK. Lumbosacral transitional vertebra and their relationship

with lumbar extadural defects. Spine. 1983;9:493–495. 5.

Vergauwen S, Parizel PM, Van Breusegem L, et al. Distribution and incidence of degenerative spine changes in patients with a lumbosacral transitional vertebra. Eur Spine J. 1997;6:168–172.

6.

Ugokwe KT, Chen TL, Klineberg E, Steinmetz MP. Minimally

157

Chapter 21: Unusual pain syndromes: Bertolotti’s syndrome

7.

8.

9.

invasive surgical treatment of Bertolotti’s Syndrome: case report. Neurosurgery. 2008;62(5 Suppl 2): ONSE454–5; discussion ONSE456.

12. Brault JS, Smith J, Currier BL. Partial lumbosacral transitional vertebra resection for contralateral facetogenic pain. Spine. 2001;26(2):226–229.

Shibayama M, Ito F, Miura,Y, et al. Unsuspected reason for sciatica in Bertolotti’s syndrome. J Bone Joint Surg Br. 2011;93(5): 705–707.

13. Jonsson B, Stromqvist B, Egund N. Anomalous lumbosacral articulations and low back pain: evaluation and treatment. Spine. 1989;14:831–834.

Almeida DB, Mattei TA, Soria MG, et al. Transitional lumbosacral vertebrae and low back pain: diagnostic pitfalls and management of Bertolotti’s syndrome. Arquivos de NeuroPsiquiatria. 2009;67(2A): 268–272.

14. Magora A, Schwartz A. Relationship between the low back pain syndrome and x-ray findings. 2. Transitional vertebra (mainly sacralization). Scan J Rehabil Med. 1978;10:135–145.

Aihara T, Takahashi K, Ogasawara A, et al. Intervertebral disc degeneration associated with lumbosacral transitional vertebrae: a clinical and anatomical study. J Bone Joint Surg Br. 2005;87:687–691.

10. Luoma K, Vehmas T, Raininko R, Luukkonen R, Riihimaki H. Lumbosacral transitional vertebra: relation to disc degeneration and low back pain. Spine. 2004;29:200–205. 11. Brown MF, Rockall AG, Hallam P, Hall-Craggs MA, Edgar MA. Transitional lumbosacral vertebra: incidence of disc degeneration above and below. J Bone Joint Surg Br 82-B 2000;(Suppl II):180.

158

15. Santavirta S, Tallroth K, Ylinen P, Suoranta H. Surgical treatment of Bertolotti’s syndrome: follow-up of 16 patients. Arch Orthopaed Trauma Surg. 1993;112(2):82–87. 16. Dai L. Lumbosacral transitional vertebrae and low back pain. Bull Hosp Jt Dis. 1999;58:191–193. 17. Scharf S. SPECT/CT imaging in general orthopedic practice. Sem Nuclear Med. 2009;39(5):293–307. 18. Burnham R. Radiofrequency sensory ablation as a treatment for symptomatic unilateral lumbosacral junction pseudarticulation (Bertolotti’s syndrome): a case report. Pain Med. 2010;11(6):853–855. 19. Matsumoto M, Chiba K, Ishii K, et al. Microendoscopic partial

resection of the sacral ala to relieve extraforaminal entrapment of the L5 spinal nerve at the lumbosacral tunnel: technical note. J Neurosurg Spine. 2006;4:342–346. 20. Endo K, Ito K, Ichimaru K, et al. A case of severe low back pain associated with Richard disease (lumbosacral transitional vertebra). Minim Invasive Neurosurg. 2004;47:253–255. 21. Airaksinen O, Herno A, Kaukanen E, et al. Density of lumbar muscles 4 years after decompressive spinal surgery. Eur Spine J. 1996;5:193–197. 22. Kawaguchi Y, Yabuki S, Styf J, et al. Back muscle injury after posterior lumbar spine surgery: topographic evaluation of intramuscular pressure and blood flow in the porcine back muscle during surgery. Spine. 1996;21:2683–2688. 23. Sihvonen T, Paljarvi L. Point of view: preventive measures of back muscle injury after posterior lumbar spine surgery in rats. Spine. 1998;23:2288. 24. Styf JR, Willén J. The effects of external compression by three different retractors on pressure in the erector spine muscles during and after posterior lumbar spine surgery in humans. Spine. 1998;23:354–358.

Section 2 Chapter

22

Spinal Disorders

Unusual pain syndromes: Baastrup’s disease/interspinous bursitis Jijun Xu and Jianguo Cheng

Case study A 70-year-old male presents with chronic low back pain for more than 10 years. He reports tenderness to deep palpation along the midline at the lumbar region and over the facet joints. The pain is worse with back extension and is better with forward flexion. Plain radiographs on lateral view show enlarged L3–5 posterior spinous processes that are closely approximated and appear to be in direct contact. On computed tomography (CT) image, small bone geodes are present between the spinous processes of L3–5 that are in close proximity (“kissing”). The sagittal contrastenhanced T1-weighted fat-saturated MRI shows that the L3 and L4 spinous processes are “touching” each other and the residual interspinous space is filled with enhanced tissue with surrounding edema and inflammatory changes in the interspinous ligaments. The intervertebral disc height and neuroforamen are unremarkable.

1. What is Baastrup’s disease? Baastrup’s disease (BD), a back pain syndrome named after Danish radiologist Christian Ingerslev Baastrup, is characterized by several pathologic changes in adjacent spinous processes and the soft tissues between them. BD is also known as “kissing spine” because of the close approximation and contact of adjacent spinous processes on sagittal plane; and “interspinous bursitis (ISB)” because the interspinous region can form an adventitious bursa with the creation of a synovial articulation.[1] For this reason, BD and ISB are used interchangeably in this chapter.

2. Describe the epidemiology of Baastrup’s disease BD was reported in many patients with heavy work such as miners and in athletes. It is now generally accepted that it is an age-related disease and tends to be more common in elderly patients.[2] Kissing spine was diagnosed clinically in 6.3% of college athletes, most commonly gymnasts.[3] In a cross-sectional retrospective review of 539 patients with lumbar spine MRI, lumbar interspinous bursitis was present in 8.2% of patients.[4] The majority (47.7%) of lumbar interspinous bursitis was seen at multiple levels, most commonly at the L4–L5 level. There was a statistically significant association between the presence of lumbar ISB and age, disc bulging, central canal stenosis, and anterolisthesis. On the other hand, gender, disc degeneration, disc herniation, endplate marrow signal alteration, facet osteoarthritis, retrolisthesis, lordosis, or scoliosis was not significantly associated with lumber ISB. In another large cohort study of 1008 patients,[5] evidence of BD (close contact between adjacent spinous processes and the opposing ends were sclerotic) was found in 413 patients (41.0%). The frequency increases in elderly patient populations with a peak of 81.3% among patients older than 80 years. Again, BD occurred most commonly at the L4–L5 level but as many as five levels could be affected. Associated degenerative changes were found at almost all affected levels (899/901).

3. Describe the anatomy and pathophysiology of Baastrup’s disease The space between adjacent lumbar spinous processes is occupied by interspinous ligament, bilateral

Case Studies in Pain Management, ed. Alan David Kaye and Rinoo V. Shah. Published by Cambridge University Press. © Cambridge University Press 2015.

159

Chapter 22: Unusual pain syndromes: Baastrup’s disease/interspinous bursitis

paraligamentous bursae, and bilateral paired interspinous lumborum muscles. The etiology of BD is not precisely known, but has been attributed to translational movement or abutting of the posterior spinous processes resulting from substantial disc space loss or excessive lordosis. BD may also develop secondary to chronic inflammatory facet arthropathy. Conversely, facet synovitis may occur secondary to active interspinous bursitis.[6] Chronic inflammation of the interspinous ligaments may gradually lead to abutting of the spinous processes, and eventually BD characterized by small cystic erosions or geodes occurring where the spinous processes contact (Figure 22.1). This process may lead to interspinous adventitial bursa formation and eventually formation of a synovial-lined articulation between the spinous processes. Calcium pyrophosphate dehydrate (CPPD) and hydroxyapatite crystal deposition may be present in the bursa. The adjacent posterior paraspinal musculature and nearby facet capsules may also be inflamed. A communication between the interspinous bursa and the facet joint has been demonstrated in some patients by injecting the interspinous bursa. As the interspinous structures degenerate, the “kissing spines” may rub against each other. This process of agitation may gradually lead to an overgrowth of the hard bone tissue, resulting in interspinous osteophytes which may subsequently cause considerable back pain particularly on spine extension. BD may cause neurogenic claudication according to a case report.[7]

4. How to diagnose Baastrup’s disease? The prominent feature of BD is the extravagant osteophytosis and approximation of the adjacent spinous processes. Often the condition is asymptomatic but it may present with reduced mobility of the spine associated with back pain. The midline localized spine tenderness and back pain can be aggravated on back extension and relieved on flexion. In some patients, increase in lordosis may bring the spinous processes into contact, leading to reactive sclerosis. These patients have axial low back pain on assuming a lordotic posture. Kwong et al[5] suggested that, because of the nearly universal association with other degenerative changes, caution should be taken before diagnosing BD as the cause of back pain. On the other hand, many clinicians fail to consider spinous process as a possible cause of back pain and many radiologists do not routinely use fat suppression MRI sequences to

160

Figure 22.1. Baastrup disease. Lateral radiograph of the lumbar spine shows enlarged spinous processes that are flattened and sclerotic in their inferior and superior portions (arrow). http://flylib. com/books/en/4.48.1.15/1/

uncover the edematous/inflammatory changes of the spinous process and interspinous ligament. Lamer et al[8] suggested that BD should be considered in the differential diagnosis and work-up of back pain when the following clinical and radiographic criteria are present: 1. Midline back pain reproduced with palpation of the spinous process and exacerbated by extension

Chapter 22: Unusual pain syndromes: Baastrup’s disease/interspinous bursitis

the bursitis as well as surrounding edema and inflammation as mentioned above (Figure 22.2). Increased fluorodeoxyglucose (FDG) uptake may also be observed in inflamed interspinous ligaments using positron emission tomography-computed tomography (PET-CT imaging) in patients with BD[10–12] whereas CT alone may fail to identify the definite abnormality.[12]

5. What are the differential diagnoses of Baastrup’s disease?

Figure 22.2. Baastrup disease. Contrast-enhanced T1-weighted fat-saturated sagittal image showed narrowing and enhancing of the interspinous associated with small erosion of the adjacent spinous process (arrows). Adapted from Czervionke[6] with permission.

of the spine. Pain relief with local anesthetic injection around the affected spinous processes. 2. Lateral view x-ray reveals spinous processes that appear to be in direct contact (Figure 22.1). MRI is more sensitive in detecting interspinous inflammation, bursa, and new bone formation.[9] BD may precede the changes on x-rays. MRI demonstrates edema and/or inflammation in and surrounding the spinous process. Regional inflammation evidenced by contrast-enhanced, fat-saturated, T1-weighted sequences, and edema by non-enhancing increased fat-suppressed T2 signal intensity (Figure 22.2). MRI is useful to provide insight into the soft tissue with regards to depicting interspinous fluid representing

The differential diagnosis of BD includes: 1. Proliferative hyperostosis of the lumbar spinous processes: Usually seen in diffuse idiopathic skeletal hyperostosis (DISH) with formation of pseudoarthrosis between the bases of spinous processes. DISH is a non-inflammatory disease, with the principal manifestation being calcification and ossification of spinal ligaments in regions where tendons and ligaments attach to bone. The most common and characteristic radiographic findings involve the thoracic spine, but abnormalities may also be present in the cervical and lumbosacral spine. There may be extraspinal involvement with hyperostosis in the olecranon, patella, calcaneus, shoulder, and acetabulum. 2. Degenerative disease of the spine: Degeneration of the intervertebral disc and/or facet joints can present with joint space narrowing, bone eburnation, and osteophytosis. These disorders may present along with BD. Radiographic evidence of degenerative changes is nearly ubiquitous in patients over 65 year of age. BD is often identified with close approximation of spinous process along with degenerative changes of the spine in the differential diagnosis and workup of back pain. 3. Sclerotic bone metastases to spine: Can arise from a number of different primary malignancies and lesions are often located in vertebra. Knowledge of increased FDG uptake in the interspinous space on PET-CT scan is important to differentiate BD from a spinous process metastatic lesion. 4. Ankylosing spondylosis: May cause erosion of the spinous processes and the interspinous ligament may calcify and eventually ossify, resulting in fusion of the spinous processes.[1] In ankylosing

161

Chapter 22: Unusual pain syndromes: Baastrup’s disease/interspinous bursitis

spondylitis, however, the bony bridges are slender, vertical bony bridges that involve the outer margin of the annulus fibrosis; erosions and bony ankylosis of the sacroiliac and apophyseal joints are not seen in BD. 5. Ossification of the posterior longitudinal ligament: It occurs more commonly in East Asian patients and predominantly in the cervical spine. Most symptomatic patients present with neurologic deficits such as myelopathy or radiculopathy, and surgery is frequently required. 6. Cysts: a. Aneurysmal bone cyst (ABC) of the spine: ABC mainly affects children and young adults and is commonly located at the metaphysis of long bones.[13] Vertebral lesions tend to start posteriorly and may spread through the pedicle into the vertebral body and epidural space.[14] b. Posterior epidural cysts: Chen et al reported that BD can be associated with posterior intraspinal epidural cysts leading to compression of the thecal sac and, in some cases, central canal stenosis.[15]

Figure 22.3. Left fluoroscopic oblique view, left intra-articular L4–5 facet injection demonstrates communication with interspinous bursa and contralateral L4–5 facet joint. From personal files of Rinoo V. Shah, MD, MBA.

6. How should you treat a patient with BD/ISB? Both conservative and surgical options are available for the treatment of BD/ISB. However, there is a lack of evidence from controlled clinical trials largely because this condition appears to have a low prevalence rate. The current therapeutic approaches are, therefore, mainly empirical. 1. Traditional management: a. Pharmacologic therapy: Non-steroidal antiinflammatory drugs (NSAIDs) are typically prescribed although they may or may not be effective. Short-term steroid dose pack may be effective in treating patients with BD. b. Physical therapy: Manual mobilization can be tried once the local tenderness is improving. Contrast heat and hot fomentation can also be effective. Ergonomic corrective methods and postural awareness are of great importance. 2. Interventional management: Targeted injections can be tried if traditional management is not

162

Figure 22.4. Anteroposterior view, left L4–5 intra-articular facet injection demonstrates communication with interspinous bursa and contralateral L4–5 facet joint. From personal files of Rinoo V. Shah, MD, MBA.

effective. Lamer et al[8] reported effectiveness of fluoroscopically guided injection to treat BD in a 3-case series. A 22-gauge needle is advanced approximately midway along the dorsal-ventral axis of the affected spinous processes. Two to three milliliters of 0.25% bupivacaine and 3 mg of betamethasone was injected after confirmed contrast spread between the targeted spinous

Chapter 22: Unusual pain syndromes: Baastrup’s disease/interspinous bursitis

processes. Two patients had a long-term response to the injection while the third one, who had more prominent degenerative changes, responded only temporarily to the injection. They concluded that, if BD has not responded to traditional treatments, local anesthetic injection of the inflamed spinous process and associated interspinous ligaments may be diagnostic and local anesthetic/ corticosteroid injection may be therapeutic. Others reported that interspinous ligament injections with 0.5 ml of 1% lidocaine and 0.5 ml of 40 mg/ml of triamcinolone acetate achieved pain free for 3 months after the injection.[16] In patients with associated inflammatory facet arthropathy, the facet joints should be injected initially because the interspinous bursae may fill during facet injection.[6] See Figures 22.3 and 22.4.

References 1.

2.

3.

4.

5.

Sartoris DJ, Resnick D, Tyson R, Haghighi P. Age-related alterations in the vertebral spinous processes and intervening soft tissues: radiologic-pathologic correlation. AJR Am J Roentgenol. 1985;145(5): 1025–1030. Epub 1985/11/01.

6.

Czervionke LF. Interspinous Bursitis (Baastrup’s Disease). In Czervionke LF, Fenton DS, eds. Imaging Painful Spine Disorders, 1st ed. Saunders. 2011: pp. 302–131.

7.

Rajasekaran S, Pithwa YK. Baastrup’s disease as a cause of neurogenic claudication: a case report. Spine. 2003;28(14): E273–275. Epub 2003/07/17.

Bywaters EG, Evans S. The lumbar interspinous bursae and Baastrup’s syndrome: an autopsy study. Rheumatol Int. 1982;2(2): 87–96. Epub 1982/01/01.

8.

Keene JS, Albert MJ, Springer SL, Drummond DS, Clancy WG, Jr. Back injuries in college athletes. J Spinal Disord. 1989;2(3): 190–195. Epub 1989/09/01.

9.

Maes R, Morrison WB, Parker L, Schweitzer ME, Carrino JA. Lumbar interspinous bursitis (Baastrup disease) in a symptomatic population: prevalence on magnetic resonance imaging. Spine. 2008;33(7): E211–215. Epub 2008/04/02. Kwong Y, Rao N, Latief K. MDCT findings in Baastrup disease: disease or normal feature of the aging spine? AJR Am J Roentgenol. 2011;196(5):1156–1159. Epub 2011/04/23.

3. Surgical management: Surgery is reserved for refractory BD patients who fail to respond to the above therapies. Surgical options include interspinous process decompression devices (e.g., Wallis system, X-STOP) and excision of the affected spinous processes.[8] If there is associated vertebral instability, surgical fusion procedure may also be considered.[6] The outcome of surgical excision varied. Franck reported improvement of pain and symptoms in 10 patients[17] whereas Beks demonstrated that only 11 out of 64 patients were free of complaints after the operation and stayed asymptomatic. The pain remained or returned in the other 53 patients, in whom other spine pathologies were found to be more evident.[18]

Lamer TJ, Tiede JM, Fenton DS. Fluoroscopically-guided injections to treat “kissing spine” disease. Pain Physician. 2008;11(4): 549–554. Epub 2008/08/12. Clifford PD. Baastrup disease. Am J Orthop (Belle Mead NJ). 2007;36 (10):560–561. Epub 2007/11/23.

10. Gorospe L, Jover R, Vicente-Bartulos A, et al. FDG-PET/CT demonstration of Baastrup disease (“Kissing” Spine). Clin Nucl Med. 2008;33(2):133–134. Epub 2008/01/23. 11. Lin E. Baastrup’s disease (kissing spine) demonstrated by FDG PET/CT. Skeletal Radiol. 2008;37 (2):173–175. Epub 2007/12/19. 12. Ho L, Wassef H, Seto J, Henderson R. Multi-level lumbar Baastrup disease on F-18 FDG PET-CT. Clin Nucl Med. 2009; 34(12):896–897. Epub 2010/02/09.

13. Vergel De Dios AM, Bond JR, Shives TC, McLeod RA, Unni KK. Aneurysmal bone cyst: a clinicopathologic study of 238 cases. Cancer. 1992;69(12): 2921–2931. Epub 1992/06/15. 14. Hay MC, Paterson D, Taylor TK. Aneurysmal bone cysts of the spine. J Bone Joint Surg Br. 1978;60-B(3):406–411. Epub 1978/08/01. 15. Chen CK, Yeh L, Resnick D, et al. Intraspinal posterior epidural cysts associated with Baastrup’s disease: report of 10 patients. AJR American J Roentgenol. 2004;182 (1):191–194. Epub 2003/12/20. 16. Mitra R, Ghazi U, Kirpalani D, Cheng I. Interspinous ligament steroid injections for the management of Baastrup’s disease: a case report. Arch Phys Med Rehabil. 2007;88(10): 1353–1356. Epub 2007/10/03. 17. Franck S. Surgical treatment of intraspinal osteoarthrosis (kissing spine). Acta Orthop Scand. 1944;14:127–152. 18. Beks JW. Kissing spines: fact or fancy? Acta Neurochirurgica. 1989;100(3–4):134–135. Epub 1989/01/01.

163

Section 2 Chapter

23

Spinal Disorders

Lumbar spinal stenosis and neurogenic claudication Ike Eriator and Zachariah Chambers

Case study A 72-year-old female reports pain in the low back during periods of standing or ambulation for the past 2 years. This is associated with numbness and tingling in the posterior thighs after walking for about half a block. The symptoms are relieved with leaning forward or sitting. Physical examination revealed a broadbased gait, negative sensory, motor, reflex testing or provocative maneuvers.

1. What is spinal stenosis? Lumbar spinal stenosis (LSS) is defined as buttock or lower extremity pain which may occur with or without low back pain, associated with diminished space available for neural and vascular elements in the lumbar spine. Neurogenic claudication refers to pain or discomfort that radiates to the lower extremity which occurs with walking or prolonged standing, and is relieved by rest or bending forward. Today, there are more treatment options for spinal stenosis than any other spinal pathology. It is the commonest indication for spine surgery in people over the age of 65 years.[1] About 75% of the cases of spinal stenosis occur in the lumbar spine. In 2007, about 38 000 operations were performed on patients with a primary diagnosis of lumbar spinal stenosis at a cost of $1.7 billion.[2]

2. What is the Verbiest syndrome? Spinal stenosis started appearing in the medical literature in the early 19th century. In 1893 a laminectomy was successfully performed to relieve a woman of the symptoms of cauda equina syndrome caused by spinal stenosis.[3] A description of lumbar spinal

stenosis was published by Sachs and Frankel[4] in 1900, but it is the classic description by the Dutch neurosurgeon, Henk Verbiest[5] in 1954 in a paper titled “A radicular syndrome from developmental narrowing of the lumbar vertebral canal” that is widely accepted as the initial description of the clinical syndrome of lumbar spinal stenosis. Neurogenic claudication or intermittent spinal claudication is called the Verbiest syndrome. Verbiest defined the clinical syndrome of lumbar stenosis in seven middle aged and older men who had back pain, bilateral radicular pain, and motor and sensory disturbances in the legs caused by standing, walking, and hyperextension. He described a myelographic block in the lumbar spine in every case, and at surgery a shallow canal with a compressed dural sac was observed. He postulated that the encroachment upon the canal by an enlarged articular process was a possible cause. Until the description, intermittent claudication was usually attributed to peripheral vascular disease of the aorto-iliac system. In the next two decades, the syndrome began to be more recognized and diagnosed. Kirkaldy-Willis and colleagues in 1978 described the pathology and pathogenesis of lumbar spondylosis and stenosis and described the three-joint complex composed of the facet joints and the intervertebral disc. They postulated that rotation and compression injuries led to degenerative changes of the three-joint complex. Subsequent to such injuries, the intervertebral discs can develop circumferential or radial annular tears, internal disruption, loss of disc height, and protrusion. The facet joints can then undergo synovitis, cartilage destruction, osteophyte formation, capsular laxity, ligamentum hypertrophy or buckling, and joint

Case Studies in Pain Management, ed. Alan David Kaye and Rinoo V. Shah. Published by Cambridge University Press. © Cambridge University Press 2015.

164

Chapter 23: Lumbar spinal stenosis & neurogenic claudication

instability or subluxation. The results of these changes to the three-joint complex create degenerative spondylolisthesis, retrolisthesis, degenerative scoliosis, and rotational deformities.[3]

3. What is the natural history of spinal stenosis? With age, progressive changes occur in the composition of the intervertebral disc similar to changes in other aging tissues in the body. This change affects the mechanical properties of the disc leading to decreased stiffness and strength, as well as accumulation of degraded matrix material. Some of these changes may be seen on MRI as “dark disc disease.” However, there is no direct relationship between such age-related changes and back pain.[6] Loss of disc integrity often leads to anterior instability, causing ligaments to buckle and hypertrophy from exposure to excessive forces, including new torsion forces. This may further lead to facet degeneration and hypertrophy. Bony encroachment into foramen of the exiting nerve root may lead to radicular features. If the anterior structures including the discs and ligaments fail at the same rate as the posterior structures, anterior subluxation (partial dislocation) of one vertebra on another may occur, leading to spondylolisthesis (anterior or posterior displacement). This may lead to a decrease in canal or foraminal space and spinal stenosis. Spondylosis refers to the agerelated changes with disc collapse and bony spur formation seen on radiologic investigations. The chronic compression of the nerve roots of the cauda equina contributes to the pain. The mechanical deformation of the cauda equina leads to increase in the intraspinal pressure, venous congestion, dilatation of the epidural veins, ischemia, and axonal injury.[7] The onset of pain is usually insidious; the natural history is characterized by fluctuations in the severity of symptoms and a tendency toward modest improvement in patients who choose not to have surgery. Degenerative lumbar stenosis is 3–5 times more common in women than men, and more commonly affects the L4–5 segment, followed by L3–4. L5–S1 segments rarely have degenerative slips because their facet joints have a coronal orientation, unlike the sagittal orientation in that of L4–5. Although lumbar spinal stenosis is typically degenerative in etiology, it is not necessarily progressive. Over time, the spine could undergo physiologic

arthrodesis, leading to long-term relief. In a prospective randomized trial (conservative versus surgical) that included 100 patients who were followed up for 10 years, it was noted that after 3 months, pain relief had occurred in most patients. After 4 years, 50% of patients in the conservative group had excellent or fair results compared to 80% in the surgically managed group. Delaying surgery did not appear to worsen the outcome.

4. What is the anatomical basis of spinal stenosis? The vertebral canal is formed with the combination of each successive vertebral foramen. The anterior boundary of the canal is made up of the posterior longitudinal ligament, the posterior surface of the vertebral body, and the intervertebral discs. The lateral border of the canal is formed by the superior and inferior pedicles of each vertebral segment. The posterior canal is bound by the vertebral lamina and the ligamentum flavum. The exiting nerve roots follow a course running laterally through the neural foramen which is an opening just inferior to each pedicle. The amount of room that is available for nerves in the vertebral canal is determined by the shape as well as the size of the vertebral canal. Spinal stenosis or canal stenosis occurs when the space is decreased due to encroachment of the boundaries of the canal. In congenital or developmental spinal stenosis, the shape and size of the canal is abnormally small due to abnormal development of the neural arch. For instance, the articular process may be too large or the pedicle too thick in relation to the size of the canal. This renders the canal relatively small for the contained neural elements.[8] It predisposes the patient to compression should minor changes occur in the boundaries of the canal. However, age-related degeneration associated with the upright position required for bipedal motion is the most common cause of spinal stenosis. Lumbar spinal stenosis occurs due to changes in the three-joint complex formed by the disc and the two facet joints. These three-joint complexes define the spine as a tripod with the disc as one leg and the facet joints as the other two legs forming the posterior support.[7] Dysfunction in any of these joints causes abnormal biomechanical stresses leading to abnormal degeneration in the other joints, thus creating a cycle of degenerative changes. Acquired

165

Chapter 23: Lumbar spinal stenosis & neurogenic claudication

Table 23.1. Etiologic classification of lumbar spinal stenosis

Acquired

Congenital

Degenerative Spondylolisthesis/ spondylosis Scoliosis Intervertebral disc bulge/herniation Facet hypertrophy Ligamentum flavum hypertrophy Synovial cysts

Idiopathic Dwarfism Achondroplasia Mucopolysaccharidosis

Degenerative/congenital Spondylolysis Iatrogenic Postlaminectomy Postfusion Post-traumatic Vertebral body compression fracture Osteoporosis Trauma Metastatic disease Tumors Metabolic Paget’s disease Fluorosis Miscellaneous Diffuse idiopathic skeletal hyperostosis Epidural hematoma Epidural abscess

spinal stenosis occurs when the structural boundaries of the canal are affected by diseases or degeneration leading to enlargement of the structure and encroachment into the canal. Spinal stenosis usually has a gradual onset and progresses as a person ages; as the intervertebral discs desiccate, degenerate, and bulge, the ligamentum flavum begins to buckle inwards. Hypertrophic osteophytes form at the facet joints. Such changes may affect one or multiple vertebral levels. Acquired spinal stenosis is the most common type of anatomic lumbar spinal stenosis and involves a combination of these factors: disc bulge or herniation, facet joint hypertrophy, and ligamentum hypertrophy/buckling. Spinal stenosis may also be caused by bone diseases, such as Paget’s

166

disease, achondroplasia, tumors, or intrinsic bone pathology that encroaches on the nerve roots or the spinal cord (Table 23.1). Spinal canal stenosis is often multifactorial in origin.

5. How is lumbar spinal stenosis classified radiologically? Radiologic classification of lumbar stenosis refers to spinal canal narrowing on cross-sectional imaging. Stenosis may be: (1) central; (2) subarticular; or (3) foraminal. Central stenosis refers to the space between the medial edges of the two facet joints. Lateral recess (subarticular) stenosis refers to the area between the medial edge of the facet joint and the medial border of the pedicle. Neuroforaminal stenosis refers to the zone underneath the pedicle, i.e., between the medial and lateral borders of the pedicle. Anatomical lumbar canal stenosis refers to the intraoperative finding of a narrow spinal canal. The anteroposterior diameter of the lumbar spinal canal varies with race and gender. In general, a midsagittal diameter greater than 12 mm is considered normal. Relative stenosis exists when the diameter is between 10 and 12 mm. LSS can be defined as a spinal canal with an anterior-posterior diameter of less than 12 mm, while absolute LSS is 10 mm or less.[9] In a review that included 25 studies and four systematic reviews, Steurer and his colleagues noted that in general, in radiologic quantification of lumbar canal stenosis, an anteroposterior diameter of less than 10 mm and a cross-sectional area of the canal that is less than 70 mm2 were the criteria most often used for central canal stenosis.[10] For lateral stenosis, height and depth of the lateral recess was typically used and the foraminal diameter was used for foraminal stenosis. A lateral recess height of 5 mm or more is normal. A height of 3–4 mm suggests lateral stenosis, while a height of 2 mm or less is pathologic. The lumbar intervertebral foramen is shaped like an inverted tear drop. The normal width is 8–10 mm. A foramen height that is less than 5 mm and a width less than 4 mm has been found to be associated with nerve root compression 80% of the time. These measures are only guidelines. The patient’s symptoms are more important than the canal diameter.

Chapter 23: Lumbar spinal stenosis & neurogenic claudication

6. How is the diagnosis of spinal stenosis made? There are no widely accepted set of diagnostic criteria. The gold standard for the diagnosis is still the impression of an expert clinician, confirmed with radiologic or anatomical finding of the narrowed spinal canal. Spinal stenosis is characterized by non-specific limb symptoms that interfere with the duration of comfortable standing or walking (neuroclaudication or pseudoclaudication). Lumbar spinal stenosis causes numbness and pain in the legs with the patient walking or/and standing up for a moderate amount of time. The distress usually subsides when the patient bends forward or sits, and recurs when standing up and walking. One typical example of this disorder in daily life is the patient feeling pain and leg numbness while shopping at the supermarket. When the patient leans against the shopping cart (shopping cart sign) slightly bending forward, the symptoms are alleviated and the patient manages to finish the shopping. The patients often find that the symptoms improve when walking upstairs, and get worse when walking down. Sleeping postures result in lumbar extension and the patient finds that the symptom is worse in the middle of the night or early in the morning. This type of pain is characteristic of spinal stenosis, as opposed to intermittent claudication of vascular etiology that does not subside with sitting and bending forward. The symptoms of vascular insufficiency can be relieved by simply standing still. Neurogenic claudication may present with more subtle symptoms including a feeling of weakness, abnormal sensations, and fatigue affecting the lower extremity. Physical examination may reveal weakness, sensory loss, or gait changes.[11] However, symptoms are rarely associated with strong focal findings on examination. Often, symptoms will get worse slowly over time. Most often, symptoms will be on one side of the body or the other, but may involve both legs. Most people with spinal stenosis cannot walk for a long period of time. More serious symptoms include: difficulty or poor balance when walking and problems controlling urine or bowel movements. In severe cases of spinal stenosis, the patient is not able to take even one step and has urinary or defecation problems, due to intense pressure on the cauda equina, which may ultimately lead to incontinence.

The findings that most strongly suggest lumbar spinal stenosis are symptoms that improve with bending forward and absence of pain when seated.[11] Other strongly suggestive features include unexplained urinary disturbance (retention or incontinence), intermittent claudication, and presence of bilateral buttock or leg pain. The finding of a widebased gait is also important in ruling in the diagnosis. The absence of neurogenic claudication decreases the likelihood of lumbar spinal stenosis. In patients with history and physical examination findings consistent with degenerative lumbar spinal stenosis, MRI is the most appropriate, non-invasive test to confirm the presence of anatomic narrowing of the spinal canal or the presence of nerve root impingement. Where MRI is contraindicated or inconclusive, CT myelography is recommended. MRI or CT with axial loading is useful as an adjunct to routine imaging. Electrodiagnostic studies can help to rule out other causes that mimic the symptoms of LSS. LSS requires the presence of characteristic clinical findings (intermittent claudication, radicular pain, or their combination) and radiographic or anatomic confirmation. Many individuals with radiographic or anatomic lumbar spinal canal stenosis may not demonstrate the symptoms and signs of LSS. Radiographic or anatomic stenosis by themselves are not sufficient to diagnose this clinical syndrome.

7. What are the differential diagnoses of lumbar spinal stenosis? a. Lumbosacral radicular pain secondary to nerve root impingement This is pain induced by irritation, inflammation, pressure, or ectopic activation of nociceptive afferent fibers in a lumbar spinal nerve or its roots. The pain is perceived as being located in the ipsilateral lower extremity and is often characterized as a sharp, stabbing, electric shock sensation.

b. Referred pain from adjacent anatomic structures The facet or zygapophysial joints are paired diarthrodial articulations between adjacent vertebrae that are innervated by the medial branches of the dorsal rami. The referral pain patterns from the facet joint include the lumbar spinal and gluteal areas as well as the

167

Chapter 23: Lumbar spinal stenosis & neurogenic claudication

trochanter, lateral thigh, posterior thigh, and groin regions. Although not typical, the referral pain may also extend below the knee. The intervertebral disc is composed of the nucleus pulposus, the annulus fibrosis, and the vertebral endplates. Chemical and mechanical factors can explain pain emanating from the discs. The patterns of intervertebral disc pain include a sharp bilateral pain located at the posterior belt line which is usually preceded by multiple episodes of less severe low back pain. The pain is localized to the lower back and gluteal area and increases with flexion, rotation, or prolonged sitting or standing. Pain can be relieved in a recumbent position. Buttock pain is the most common pain referral area from a symptomatic hip joint. Groin and thigh referral areas can also occur but are less common. Hip joint pain can occasionally refer distally to the foot. Lower lumbar spine referral does not usually occur. Symptoms of greater trochanteric bursitis consist of persistent pain in the lateral hip radiating along the lateral aspect of the thigh to the knee and occasionally below the knee and/or buttock. Physical examination reveals point tenderness in the posterolateral area of the greater trochanter.

c. Lumbar vertebral compression fracture The osteoporotic vertebral fracture can cause a sudden, acute, intense pain that is aggravated even with the slightest movement and is located at the center of the spine, approximately at the level of the fracture, potentially radiating along the sides of the body.

d. Intermittent claudication secondary to peripheral vascular disease This is an intermittent cramping pain which is severe and usually arises after fixed and consistent amounts of exercise. Pain is usually relieved with rest or placing the limb in a dependent position. Low back pain is usually absent in vascular claudication. Vascular pain begins in the calves and progresses proximally, unlike neurogenic claudication which begins proximally and progresses distally. The stationary bicycle test can help with diagnosis. Patients with neurogenic claudication are able to tolerate the exercise since the lumbar is flexed, unlike the vascular claudication patients who should become symptomatic. This diagnosis can generally be ruled out if the ankle brachial indices are

168

normal, unlike vascular claudication where the indices are typically closer to 0.7. Sometimes, these two conditions can coexist!

e. Peripheral neuropathy Peripheral neuropathy is a type of neuropathic pain attributed to dysfunction of peripheral nerves. It is manifested with functional nerve decline affecting various sensations such as touch, pain, vibration leading to numbness and dysesthesia. Longer nerves are more susceptible and, as a result, peripheral neuropathy symptoms initially occur in hands and feet, following the so-called “glove and stocking” distribution. Peripheral neuropathy usually affects both sides of the body symmetrically.

f. Visceral referred pain Visceral structures are highly sensitive to distention, ischemia, and inflammation while they are relatively insensate to cutting or burning. They present with vague pain symptoms with poor localization due to a low density of sensory innervation of the viscera and extensive divergence of the visceral input within the central nervous system. Multiple visceral structures including the kidneys, prostate, urethra, bowel, bladder, and pancreas can refer pain to the low back, buttocks, and upper legs.

g. Other differential diagnoses These include pain related to the sacroiliac joint, piriformis syndrome, myofascial pain, and compartment syndrome of the leg. In pathology due to the sacroiliac joint, the pain is in the lower back, overlying the posterior-superior iliac spine and may radiate to the buttocks and lower extremity. In piriformis syndrome, the pain is localized over the piriformis muscle in the buttocks, and may radiate down to the posterior buttock and lower extremity. In myofascial pain, trigger points can be felt over the muscle. In compartment syndrome, patients have tightness in the calf induced by strenuous exercise and are relieved slowly with elevation of the limb.[11]

8. Are conservative treatments effective in lumbar spinal stenosis? Conservative management is geared toward reducing inflammation, decreasing pain, strengthening the

Chapter 23: Lumbar spinal stenosis & neurogenic claudication

paravertebral muscles, and increasing the range of motion. Surgery is not devoid of complications and therefore many physicians will begin treatment with a conservative management regimen including physical therapy, pharmacotherapy, manipulation, bracing, traction, CBT, psychologic counseling, and electrical stimulation. Cold packs and heat therapy may also help with pain flare-ups. Although physical therapy is a popular treatment option, passive physical therapy shows only minimal benefits in the patient with LSS when used as sole therapy.[12] There has been no optimal regimen designed for active physiotherapy in patients with LSS. However, a regime that combines manual therapy/exercise and body weight supported ambulation has shown higher rates of perceived recovery in the short term.[13] Other modalities often used in physical therapy including transcutaneous electrical nerve stimulation and ultrasonography also have limited benefit in the treatment of back pain. Although back braces have been a popular method of treating back pain there has been no evidence that braces correct the cause of the pain generator. Braces may, however, be a physical reminder that the patient needs to use correct bending and lifting techniques while at work or home. A number of different medications have been trialed for pain secondary to lumbar spinal stenosis including gabapentin, limaprost, and methylcobalamin. Non-steroidal anti-inflammatory drugs (NSAIDs), muscle relaxants, and opioid analgesics are often used to treat low back pain based on their mechanism of action. Calcitonin has analgesic properties and can decrease the blood supply to bones by decreasing the metabolic activity. Parenteral calcitonin (but not intranasal administration) can transiently decrease pain.[12] Gabapentin by binding to the alpha 2 subunit of the calcium channel, modulates neural transmission and provides analgesia. In lumbar spinal stenosis trials, patients on gabapentin had a greater walking distance, significantly lower pain score at 3–4 months, and decreased sensory deficits.[14] Limaprost (alprostadil) is an oral prostaglandin E1 analog with vasodilatory and antiplatelet properties. In a shortterm RCT, patients with LSS who were on limaprost in comparison to those on etodolac, had better pain scores, walking distance, improvement in leg numbness, and satisfaction after 8 weeks.[15] NSAIDs are effective for both anti-inflammatory and analgesic effects; however they should be used

with caution in patients due to the risk of gastritis, gastrointestinal bleeding, and renal dysfunction. NSAIDs should be used at the lowest dose for the desired effect and should not be used long-term. Their cardiovascular risk profile calls for care in this group of patients who are usually elderly. Acetaminophen is an effective medication for mild to moderate pain but has no effect on inflammation or muscle relaxation. Opioids can help in moderate to severe pain in general, but their use in chronic non-cancer pain remains controversial.

9. What is the next step if simple conservative options are not effective? Epidural corticosteroid injections have been shown to be effective in the short-term treatment of acute and subacute lumbar radicular pain. The transforaminal epidural injection may be more effective than the classical interlaminar approach since the infusion of the theraupeutic drug solution is selectively targeted to the affected nerve root.[16] Interlaminar epidural steroid injections can also provide short-term (weeks) improvement in function. The addition of steroid to the local anesthetic may not significantly increase the duration of relief.[12] A multiple injection regimen of radiographically guided transforaminal epidural steroid injection or caudal injections can provide medium-term (3–36 months) relief of pain.

10. Neuroplasty therapy (adhesiolysis) There is strong evidence supporting the efficacy of neuroplasty with corticosteroids in the short and long-term control of pain in refractory radiculopathy and neuropathic spinal pain. Such epidural adhesiolysis can produce reduction in pain and improvement in disability index over 12 months.[17] In this procedure, the sacral hiatus is located after sterile cleaning and draping. Lidocaine is infiltrated through the skin and subcutaneous tissue. An epidural/introducer needle is inserted until the tip of the needle is positioned within the sacral hiatus and well below a horizontal line connecting the inferior border of the S3 foramina (to avoid the dural sac). Contrast is used to confirm placement. The epidurogram typically shows a Christmas tree pattern. A radiopaque, styletted, and steerable catheter is inserted through the needle and guided until the tip is below the stenotic

169

Chapter 23: Lumbar spinal stenosis & neurogenic claudication

level. Contrast is used to confirm placement and to ensure absence of subarachnoid, subdural or vascular spread and to ensure adequate run off (no loculation). Local anesthetic and steroid are then injected. The needle is retracted with the catheter left in place. Fluoroscopy is used to confirm that the catheter is still in position. The catheter is then secured to the back. After confirming that the catheter is not in the intrathecal space, hypertonic saline is injected in aliquots. Hypertonic saline solution has historically been 10%. Hyaluronidase may be used prior to the hypertonic saline. The catheter is removed with the tip intact. Sedation (and analgesia) is tailored to the patient’s comfort during the procedure.

11. Minimally invasive lumbar decompression Minimally invasive lumbar decompression (MILD) is focused on debulking the ligamentum flavum, thereby widening the spinal canal without disruption of the biomechanical support. It can be performed in an ambulatory setting and may be an option for patients who are not suitable candidates for surgical decompression due to comorbidity or other reasons. It is usually done under local anesthetic with sedation. In this procedure, contrast material is injected in the epidural space under fluoroscopic guidance. A cannula is inserted about one and a half vertebral bodies below the level of the epidural needle and slightly medial to the pedicle and clamped in place. The edges of the lamina and the thickened ligamentum flavum are resected using special tools under fluoroscopic guidance.[18] The process is repeated on the opposite side for bilateral decompression of the canal stenosis. In a randomized trial that compared the effectiveness of MILD with that of epidural steroid injection, LSS patients in the MILD group showed statistically significant greater pain reduction and improvement in mobility at 12 weeks follow-up.[19] Complications including nerve transection and dural tear and high failure rates have been reported. Practitioner’s training and careful patient selection are very important for success. Patients who are chosen for the MILD procedure must be differentiated from those with pure radicular pain. Patients who have ligamentum flavum hypertrophy may also have co-factors that worsen LSS symptoms, including facet hypertrophy, spondylolisthesis,

170

disc protrusion, epidural lipomatosis, and foraminal stenosis. Although patients may have multiple comorbidities related to the LSS it is not necessary to treat all of the causes of LSS to see symptom relief.[20] Studies have indicated that most patients show significant early improvement following treatment and have considerable stability and durability 2 years after the MILD procedure.[21]

12. Interspinous spacers In some patients with lumbar spinal stenosis in whom conservative approaches have not been successful, interspinous spacers are surgical alternatives that can be performed under local anesthetic with sedation in an outpatient setting. It is a less invasive procedure compared to a decompressive laminectomy. A titanium implant is fixated to the interspinous ligament between the two symptomatic vertebrae, thus decreasing extension of the spine at that level. Flexion can still occur. Considering that lumbar spinal stenosis occurs in the latter decades of life when patients have multiple medical problems, some of which may be debilitating, the risk profile associated with the interspinous spacers makes them a viable alternative to traditional laminectomy with or without fusion. The X-STOP Interspinous Process Decompression (IPD) System was approved by the FDA in 2005 for implantation at one or two lumbar levels. The X-STOP consists of two titanium flanking “wings” connected by a central bar. During the procedure the surgeon removes one of the wings, inserts the bar between two adjacent spinous processes, and then locks the second wing down. The contraindications include allergy to titanium, significant lumbar spine instability, ankylosis at the levels to be treated, significant scoliosis, acute fracture of the spinous process or pars interarticularis, severe osteoporosis, cauda equina syndrome, or systemic or local infections. Patients can still have surgical decompression in the future if their symptoms persist or recur. The X-STOP PEEK (polyetherketone) is a modified version of the X-STOP and was approved in 2006. It includes a PEEK spacer and additional 16 mm spacer size. The Coflex Interspinous Spacer is a U-shaped titanium alloy with pairs of wings that surround the superior and inferior spinous processes and is designed to improve the cross-sectional diameter of the spinal canal. It was approved by the FDA in 2012.

Chapter 23: Lumbar spinal stenosis & neurogenic claudication

In a retrospective cohort analysis of medicare claims for 2006–2009, Dayo et al (2013)[22] compared the complications and repeat operations in patients who had surgical decompression or fusion to those who had interspinous spacers for lumbar spinal stenosis. Patients who had placement of spacers were older (though there was little evidence of greater comorbidity). Patients who received the spacer alone had fewer complications than those who had decompression or fusion (1.2% vs. 1.8% vs. 3.3%). However, at the 2-year line, patients who were treated with the spacers had higher rates of further inpatient lumbar surgery (16.7% vs. 8.5% for decompression and 9.8% for fusion). Hospital payments for spacer surgery were higher than those for decompression alone, but less than that for fusion.

13. How effective is decompression for the treatment of LSS? The purpose or goal of surgical intervention is to correct the underlying process that is causing the symptoms of lumbar spinal stenosis. It is usually performed for the relief of pain in the lower extremity, not low back pain. Surgical treatment can provide longterm improvement in patients with degenerative lumbar spinal stenosis and has been shown to improve outcomes in a large percentage of patients.[23] Elderly patients with spinal stenosis who tolerate their daily activities well usually do not need surgery unless they develop new signs of bowel and bladder dysfunction. Patient’s preferences, the presence of other medical conditions, and the risks of surgery should be considered. Surgery for spinal stenosis is rarely considered in the first 3 months of symptoms. Surgery is beneficial in appropriately selected patients, and it may be done through several options. Decompressive laminectomies are the most common surgeries performed on the lumbar spine and involve removal of the laminae (roof) of the vertebrae to create more space for the neural elements. However, conventional decompressive laminectomy disrupts several supportive elements including the spinous process, lamina, and inter-spinal ligaments and leads to potential instability and increased stress on the adjacent discs and facets. Decompression alone is suggested for patients with leg predominant symptoms without instability. It involves removal of ligamentum and lamina. The dural sac is then retracted to access the disc material causing the neural

compression. Due to variations in the skills, knowledge, and experience of the surgeon, the outcome also varies.[6] Foraminotomy takes the pressure off the nerve root and allows the spine to move more easily. Laminectomy with or without fusion has been the surgical treatment favored by most surgeons for patients with lumbar spinal stenosis and associated poor quality of life. Complications include worsening pain, disability, neurologic deficit, poor wound healing, and even death. About 30% to 70% of patients report significant improvement in symptoms. Gross total laminectomies are rarely done nowadays due to the complicating post laminectomy kyphosis. Rather, midline sparing procedures like laminotomies or hemilaminectomies are preferable, and these can be done through minimally invasive approaches. Microdiscectomy by any method is now the operation of choice, as it is less invasive and is associated with decreased postoperative pain, hospital costs, and the number of missed work days. The prevalence of recurrent disc herniation varies and it may relate more to the surgical skill rather than to the choice of microdiscectomy or open discectomy. Minimal access procedures like percutaneous discectomy, microscopic discectomy, or energy-assisted discectomy minimizes surgical exposure, but they require greater knowledge of spinal anatomy, as separating the nerves from bony structures can be difficult with such minimally invasive exposures.[6] In certain conditions like spondylolisthesis, spinal fusion is the standard procedure to ensure spinal stabilization and this provides a better outcome.

14. How would you manage this patient? The diagnosis of lumbar spinal stenosis should be considered in older patients presenting with a history of gluteal or lower extremity symptoms exacerbated by walking or standing, which improves or resolves with sitting or bending forward. Patients whose pain is not made worse with walking have a low likelihood of stenosis. The most common symptom associated with lumbar spinal stenosis is neurogenic claudication. The likelihood of clinical lumbar spinal stenosis increases with age, especially in individuals above 70 years of age. A diagnosis of spinal stenosis is made by history and physical examination and confirmed by diagnostic testing.

171

Chapter 23: Lumbar spinal stenosis & neurogenic claudication

The history and physical examination findings in this patient are suggestive of lumbar spinal stenosis. The bicycle test can help to differentiate neurogenic and vascular claudication. An MRI would be helpful for radiologic confirmation and quantification. Plain x-rays (with flexion/extension) can provide an estimate of the degree of instability. It can show fractures, osteophytes spinous process settling (kissing spine), and metastatic disease. Depending on comorbidities, other special tests like ankle brachial indices and EMG may be needed to rule out the differential diagnoses. If the MRI confirms the spinal stenosis, it will also help with planning in terms of the location and classification of the stenosis. The treatment options, associated side effects, and expected outcomes should be discussed. The symptoms may not deteriorate if she chooses palliative relief and simple follow-up. But about 15% of patients with lumbar spinal stenosis will deteriorate,

References 1.

2.

7.

Lurie JD, Tosteson AN, Tosteson, TD, et al. Reliability of reading of magnetic resonance imaging features of lumbar spinal stenosis. Spine (Phila Pa 1976), 2008; 33(14):1605–1610.

Storm PB, Chou D, Tamargo RJ. Lumbar spinal stenosis, cauda equine syndrome and multiple lumbo-sacral radiculopathies. Phys Med Rehabil Clin N Am. 2002;13:713–733.

8.

Deyo RA, Mirza SK, Martin BI, et al. Trends, major medical complications, and charges associated with surgery for lumbar spinal stenosis in older adults. JAMA. 2010;303:1259–1265.

Bogduk N. The lumbar lordosis and vertebral canal. In Clinical Anatomy of the Lumbar Spine and Sacrum, 3rd ed. London, UK: Churchill Livingstone. 1997: chapter 5, pp. 55–62.

9.

Kalichman L, Cole R, Kim DH, et al. Spinal stenosis prevalence and association with symptoms: The Framingham study. Spine J. 2009;9(7): 545–550.

3.

Botwin K, Gruber R. Lumbar spinal stenosis: anatomy and pathogenesis. Phys Med Rehabil Clin N Am. 2003;14:1–15.

4.

Sachs B, Frankel V. Progressive and kyphotic rigidity of the spine. J Nerv Ment Dis. 1900;27:1–15.

5.

Verbiest H. A radicular syndrome from developmental narrowing of the lumbar vertebral canal. J Bone Joint Surg Br. 1954;36B(2): 230–237.

6.

172

and about 15% will have improvement. Nonconservative approaches with analgesics, coanalgesics, and physical therapy (manual with body weight support) may help. But considering the limited ambulation and the gait disturbance, our advice will be to consider further treatment options. Lumbar epidural injection, the technical approach of which can be tailored to the MRI findings of the location of stenosis, can help in the short term. Epidurolysis may also be helpful in this case. If no long-term relief is obtained, and the patient chooses to have something more done, the options of MILD, inter spinous spacer, microdiscectomy, and lumbar laminectomy with or without fusion should be discussed guided by the MRI findings. The most efficacious treatment for patients with lumbar stenosis remains elusive. A multidisciplinary approach to treatment is the recommended course at this time.

Lavelle W, Carl A, Lavelle ED. Invasive and minimally invasive surgical techniques for back pain conditions. Med Clin N Am. 2007;91:287–298.

10. Steurer J, Roner S, Gnannt R, Hodler J. Quantitative radiologic criteria for the diagnosis of lumbar spinal stenosis: a systematic literature review. BMC Musculoskeletal Disorders. 2011;12:175. 11. Suri P, Rainville J, Kalichman L, Katz J. Does this older adult with lower extremity pain have the clinical syndrome of lumbar spinal stenosis? JAMA. 2010; 304(23):2628–2636.

12. Tran DQH, Doung S, Finlayson RJ. Lumbar spinal stenosis: a brief review of the nonsurgical management. Can J Anesth. 2010;57:694–703. 13. Whitman JM, Flynn TW, Childs JD, et al. A comparison between two physical therapy treatment programs for patients with lumbar spinal stenosis: a randomized clinical trial. Spine (Phila Pa 1976). 2006;31: 2541–2549. 14. Yaksi A, Ozgonenel L, Ozgonenel B. The efficiency of gabapentin therapy in patients with lumbar spinal stenosis. Spine. 2007;32:939–942. 15. Matsudaira K, Seichi A, Kunogi J, et al. The efficacy of prostaglandin E1 derivative in patients with lumbar spinal stenosis. Spine (Phila Pa 1976). 2009;34:115–120. 16. Lee JH, An JH, Lee SH. Comparison of the effectiveness of interlaminar and bilateral transforaminal epidural steroid injections in treatment of patients with lumbosacral disc herniation and spinal stenosis. Clin J Pain. 2009;25(3):206–210.

Chapter 23: Lumbar spinal stenosis & neurogenic claudication

17. Manchikanti L, Cash KA, McManus C, et al. The preliminary results of a comparative effectiveness evaluation of adhesiolysis and caudal epidural injections in managing chronic low back pain secondary to spinal stenosis: a randomized equivalence control trial. Pain Physician. 2009;34:E342–351. 18. Vallejo R, Benyamin R. Novel options for patients with lumbar spinal stenosis: minimally invasive lumbar decompression and other strategies. Tech Reg Anesth Pain Manag. 2013;16:84–88.

19. Brown L. A double blind, randomized prospective study of epidural steroid injection vs the MILD procedure in patients with symptomatic lumbar spinal stenosis. Pain Pract. 2012;12: 333–341. 20. Mekhai N, Costandi S, Abraham B, Samuel S. Functional and patient-reported outcomes in symptomatic lumbar spinal stenosis following percutaneous decompression. Pain Practice. 2012;12(6):417–425. 21. Chopko B. Long-term results of percutaneous lumbar

decompression of LSS: two-year outcomes. Clin J Pain. 2013; 29(11):939–943. 22. Dayo RA, Martin, BI, Ching A, et al. Interspinous spacers compared with decompression or fusion for lumbar stenosis: complications and repeat operations in the medicare population. Spine (Phila Pa 1976). 2013;38(10):1865–1872. 23. Airaksinen O, Herno A, Turunen V, et al. Surgical outcome of 438 patients treated surgically for lumbar spinal stenosis. Spine. 1997;22:2278–2282.

173

Section 2 Chapter

24

Spinal Disorders

Management of the patient with postlaminectomy pain syndrome Jay S. Grider

Case study The patient is a 64-year-old male who underwent microdiscectomy 10 years prior. Eighteen months ago the same spine surgeon diagnosed spinal stenosis and performed a two level, decompressive laminectomy. This was successful in relieving his pain for approximately 1 year; however the patient reports that pain has worsened over the last 6 months and is now intolerable with conservative measures such as physical therapy, NSAIDs, and opioids having been tried. He describes low back pain across the center of his lumbar region with a dull ache to the posterior aspect of both hips and thighs. Additionally, he reports pain down the left leg to the top of his left foot. He presents to the Pain Center today inquiring about treatment options as the oxycodone 7.5/325 tablets he was taking 3 times per day as prescribed by his primary care physician have become less efficacious with time.

1. What is postlaminectomy pain syndrome? There are several conditions which fall under the heading of postlaminectomy pain syndrome (PLPS). While the name implies an etiology that is confined to patients who have had laminectomy, the syndrome may be more accurately defined as postlumbar surgery syndrome. This “catch all” term as used in clinical vernacular also often includes patients who have had discectomy or fusion. The term failed back surgery syndrome (FBSS) is also used, but for obvious reasons carries connotations that are distasteful to the spine surgeon. The concept of pain persisting despite appropriate execution of the surgical intervention and anatomical correction (to the degree possible) is the

principle which underpins the concept of PLPS regardless of nomenclature.[1,2] It has been estimated that as many as 30% of patients will have ongoing pain after decompression.[2] Likewise approximately 75% of patients have persistent low back pain 10–20 years following discectomy. While the surgical procedures mentioned can be performed at any spinal level, the concept of postlaminectomy pain syndrome as a clinical entity is usually confined to the lumbar spine.[1–3]

2. What is the etiology of postlaminectomy pain syndrome? There are established sources of PLPS that can be attributed to the pre-, intra- and postoperative period. Table 24.1 outlines these etiologies. Though beyond the control of the interventional pain physician, the preoperative selection process and the psychosocial aspects around patient selection for initial surgery are often overlooked. It stands to reason that the outcome of spinal surgery in an individual with major tendencies toward somatization will frequently yield mixed results. It is clear that patients with preoperative legal or workers’ compensation issues surrounding their care have inferior outcomes;[2,4] the reasons for this are multifactorial. Intraoperative issues include surgical intervention on an anatomically aberrant structure that, despite the poor appearance on imaging (and occasionally corroborated with neurodiagnostic studies), is not the primary pain generator.[4] An example of this concept would be the patient with disc herniation and dermatomal radicular pain in which the leg pain improves after surgery while the low back pain persists into the postoperative period. If low back pain

Case Studies in Pain Management, ed. Alan David Kaye and Rinoo V. Shah. Published by Cambridge University Press. © Cambridge University Press 2015.

174

Chapter 24: Management of the patient with postlaminectomy pain syndrome

Table 24.1. Factors associated with postlaminectomy pain syndrome

Preoperative factors

Intraoperative factors

Postoperative factors

Ignoring psychologic factors complicating symptoms

Inaccessible pathology

Restenosis/ herniation

Litigation

Misdiagnosis

Fibrosis

Poor surgeonpatient communication

Wrong level/ inadequate decompression

Continued deterioration of lumbar spine

Unrealistic expectation (patient)

Fusion instability

was the primary presenting complaint to the surgeon while the radicular lower extremity pain was secondary, the patient may be dissatisfied with the outcome when radicular pain improves and back pain persists. Anecdotally, This appears to be relatively common anecdotally. Common postoperative causes of persistent pain include residual anatomic pathology that is not accessible via the employed surgical approach, reoccurrence of the pathology, internal disc disruption, and fibrosis of the surgical site.[3] Recent studies have suggested a link between the severity of pain after lumbar spine surgery and the degree of fibrosis that occurs.[5] These concepts have led to several studies designed to limit fibrosis and concomitant adhesions after spine surgery.[5] While beyond the scope of the current discussion it is important to note that these intraoperative/postoperative applications of lidocaine, primecrolimus, heparin extracts, and allantoin have had variable success in preventing fibrosis.[3]

3. How is postlaminectomy pain syndrome diagnosed? Diagnosis often begins with imaging. Standing flexion/extension x-rays give basic clues to alignment and stability of the lumbar spine. MRI imaging without contrast enhancement can evaluate the disc and the diameter of the central and lateral spinal canal with regard to stenosis.[6,7] Reevaluation for patients who have had discectomy may require contrast

enhancement to determine the presence of discrelated pathology. The presence of pedicle screws may cause artifact on MRI that may obscure the desired view. In these cases CT-myelogram may be a superior imaging choice to evaluate stenosis; however this modality has the downside of requiring intrathecal access. If postdural puncture headache occurs in the laminectomy/fusion patient, treatment with an epidural blood patch may be difficult.[7–9] While the formal designation of what constitutes critical canal stenosis has not been established it has been suggested that a canal diameter in the 10–12 mm range may become symptomatic.[9] Anecdotally the development of moderate-severe canal stenosis above or below the level of previous decompression or fusion must be considered. Discogram is a method to identify symptomatic intradiscal pathology. The usefulness of this modality in diagnosing the source of discogenic pain has many advocates and detractors. The interested reader is directed to several excellent references on this topic.[10,11] While useful in the diagnosis of discogenic pain and disrupted disc morphology, the risks (disc infection, false-positive) and benefits must be carefully considered. The systematic use of injective therapy as a diagnostic tool is often underappreciated as these modalities are primarily thought of as therapeutic interventions. As the clinician uses image-guided injections to identify the pain generator a more precise treatment plan may be formulated. Diagnostic image-guided injections of the sacroiliac joint, facet joints, at the neuroforamen/pedicle screw interface, and myofascial structures surrounding hardware are often sources of pain that can be evaluated. The use of selective nerve root/transforaminal injections can identify the level of pathology.[12] The use of ultrasound as a modality to evaluate musculoskeletal issues and to guide diagnostic intervention is currently a rapidly evolving area of interest. It is important to underscore to the patient who may have been prescribed injective therapy prior to their surgery that the use of diagnostic blocks is to determine if there is new or treatable pathology. Underscoring the diagnostic application of these blocks ensures that the patient understands that there may not be long-term therapeutic benefit to the procedure they are undergoing. The inability to identify a pain generator may provide important information for the clinician. The treatment paradigm may change

175

Chapter 24: Management of the patient with postlaminectomy pain syndrome

from an interventional or rehabilitative model to a more palliative model that assists with coping and support mechanisms for the patient with pain that may not be easily treated.

4. What interventional treatments may be utilized? The modalities available to treat PLPS include the following: 1. Epidural steroid injection – interlaminar and transforaminal 2. Lumbar facet interventions – steroid facet injections and radiofrequency ablation 3. Adhesiolysis 4. Spinal cord stimulation 5. Intrathecal drug delivery 6. Medication management a. Adjunctive medications b. Opioid analgesics

Figure 24.1. Lumbar transforaminal. Note extensive lumbar laminectomy and prior removal of hardware at L4, L5, and S1 pedicles. From personal files of Rinoo V. Shah, MD, MBA.

Lumbar epidural steroid injections and PLPS There is evidence to suggest that lumbar epidural steroid injections certainly provide short-term (less than 3 months) and possibly long-term (greater than 6 months) decrease in pain-related symptoms and dysfunction. This point is however controversial as recent studies have called the long-term efficacy of interlaminar injections into question.[13–15] It is possible that transforaminal epidural injections may provide greater long-term efficacy as at least one report has suggested that this approach may provide enough pain relief to prevent consideration of reoperation.[14] It is imperative that these injections be image-guided rather than blind in nature as the transforaminal data would suggest that specific anatomic localization improves outcome.[14] Since many of the lumbar epidural steroid articles range over several years (with a wide variety of techniques and approaches used) conclusions on efficacy are somewhat difficult with many of the recent studies suggesting less benefit than previously thought from midline transforaminal lumbar epidural steroid injections. Recently an interlaminar approach to the lateral recess has been described and may hold promise as a therapeutic approach.[15] Clinically epidural steroid injections can be uncomfortable for the patient if increasing volumes are used. This could be due to

176

the volume of injectate breaking down scar tissue in a rudimentary lysis of adhesions fashion.[3] See Figure 24.1.

Caudal epidural injection and PLPS Special consideration to caudal epidural injections is warranted as there is evidence to support this approach in long-term treatment of pain associated with patients who have had spine surgery with residual pain.[16] At least one well-designed randomized controlled trial has shown benefit superior to the caudal approach vs. the traditional lumbar interlaminar treatment approach.[17] While not limited to laminectomy, this study demonstrated that benefit was obtained with either lidocaine alone or with steroid. Use of other medications such as hyaluronidase have had mixed results when utilized in the lumbar/ caudal epidural space.[15,17,18]

Facet and medial branch interventions The contribution of facetogenic pain to PLPS is currently unknown but diagnosis and treatment follows the same algorithms as non-PLPS facet-mediated pain.[16] There is evidence to suggest that patients

Chapter 24: Management of the patient with postlaminectomy pain syndrome

Figure 24.2. Lumbar epidural adhesiolysis, note epidural scar impedance of contrast cephalad to L5–S1. From personal files of Rinoo V. Shah, MD, MBA.

with axial back pain of facet origin respond similarly despite previous surgical intervention.[19,20]

Adhesiolysis When lumbar/caudal injections do not prove effective in treating PLPS pain, adhesiolysis has been shown to be effective in improving pain and increasing function.[16] Lidocaine, hypertonic saline, and nonparticulate betamethasone solutions have been used and shown to provide greater benefit than epidural steroid injections.[16] Adjuvant drugs such as hyaluronidase may be utilized as well. Conversely, adhesiolysis may be more uncomfortable for the patient than epidural steroid injection. Evidence concerning the safety and efficacy of the procedure in patients with PLPS has been supported by several clinical trials.[16,21–24] See Figures 24.2 and 24.3.

Spinal cord stimulation The use of spinal neuromodulation in the treatment of PLPS has been studied in one of the better interventional pain studies to date: the PROCESS trial (Prospective Randomized Controlled Trial of Effectiveness of Spinal Cord Stimulation).[25–27] In this study and two other related studies, 100 patients were randomized to both spinal neuromodulation and conventional medical management versus medical management. The results suggest that spinal cord

Figure 24.3. Lumbar transforaminal-far lateral approach given posterolateral bone graft on transverse processes.[36] From personal files of Rinoo V. Shah, MD, MBA.

stimulation coupled with conventional medical management significantly improves analgesia over medical management alone in patients with PLPS. Similarly North and colleagues[28] found that spinal cord stimulation provides better outcomes with regard to function and analgesia than a second spinal surgery. These studies have led many thought leaders to suggest that spinal neuromodulation should be considered earlier in the treatment algorithm of lumbar radicular pain and certainly before reoperation unless there is a clear anatomic cause for surgical intervention. As neuromodulation techniques have matured, the use of stimulation in the periphery for PLPS has been reported with success. While in the early stages of description several reports suggest that true peripheral nerve stimulation and peripheral field stimulation, both with and without epidural lead placement, may have benefit in this patient population.[29] These concepts require further investigation but appear to have a very favorable risk–benefit profile.[16] See Figures 24.4 and 24.5.

Pharmacologic management There is reasonable evidence for analgesic efficacy with NSAIDs; however this class of medications has side effect profiles that limit their usefulness in several patient populations. Anticonvulsant and selected

177

Chapter 24: Management of the patient with postlaminectomy pain syndrome

Figure 24.5. Lateral view cervical spinal cord stimulation in patient with cervical postlaminectomy syndrome; left lead cannot pass further cephalad due to epidural scarring-whereas right lead can pass. From personal files of Rinoo V. Shah, MD, MBA.

Figure 24.4. Anteroposterior view cervical spinal cord stimulation in patient with cervical postlaminectomy syndrome; left lead cannot pass further cephalad due to epidural scarring-whereas right lead can pass. From personal files of Rinoo V. Shah, MD, MBA.

antidepressant medications have also been utilized with success for neuropathic pain associated with PLPS but have proven less effective for isolated chronic low back pain.[3] The use of opioids in the treatment of patients with previous spine surgery, while common, has limited evidence of success in the literature and as such is a topic of much debate currently.[30,31] The significant risks to be considered from an opioid habituation and aberrancy standpoint must be carefully weighed in an assessment of improved function.[30]

Intrathecal drug delivery Patients who have not received benefit from less invasive interventional treatment options, such as injections, medications and even spinal cord stimulation, may be candidates for intrathecal drug delivery (IDD). In many cases patients become tolerant to

178

the oral opioid effects and/or have side effects of oral opioids that preclude their continued use. It has been estimated that oral opioid therapy is discontinued in over 50% of subjects in whom it is started because of poor efficacy or side effects.[32,33] Patients with PLPS are often excellent candidates for IDD because they often have axial spine pain that is a greater symptomatic issue than radicular lower extremity pain. For patients with predominantly low back pain, IDD has been suggested as an excellent source of analgesia without systemic side effects.[32] Currently only preservative-free morphine, baclofen, and ziconotide are approved by the FDA for intrathecal use; the Polyanalgesic Consensus Guidelines prepared by the North American Neuromodulation Society lists other commonly accepted agents for which there is reasonable safety data for intrathecal use.[32] These medications include the opioids hydromorphone and fentanyl as well as the local anesthetic bupivacaine and antihypertensive medication clonidine.[31] The opioids are often used for axial spinal pain while the other adjunctive medications and ziconotide are considered useful for neuropathic pain.[32] The balance between clinical experience (which for many of these medications is vast) and regulatory approval via FDA has created a dilemma for clinicians as they attempt to treat patients with IDD.[33] Aggressive use of medications not approved for intrathecal use has

Chapter 24: Management of the patient with postlaminectomy pain syndrome

drawn stern warnings from many professional societies; however the Polyanalgesic Guidelines represent evidence-based, reasonable pharmacologic applications of drugs with long histories of safety and efficacy some of which would represent an off-label usage.[33] A recent patient alert bulletin by the Medtronic Corporation suggests that changing between medications (morphine to hydromorphone, for example) may result in a short-term aberration in drug delivery (hours) that may result in slightly higher or lower doses of medication than intended and as such careful attention to the medication bridge-bolus with patient education as to possible effects is important.[33] Additionally, recent bulletins concerning improper filling of the devices (known as “pocket fills”) has led to renewed emphasis on safety during all phases of care with IDD.[33,34] The trialing and dosing regimens used by clinicians for IDD have been based largely on anecdotal and expert opinion for over the past three decades. Recently there have been attempts by several major pain societies to examine the evidence surrounding IDD patient selection and maintenance dosing.[31] While the 1990s saw aggressive dosing of IDD, recently there have been calls for reexamination of higher doses of medications delivered intrathecally with the concomitant risk of granuloma formation from these more aggressive dosing regimens. Recent studies have suggested the dose-response relationship in humans may be toward the lower end of the currently used dose spectrum for humans and as such aggressive dose titration may be, at the least, ineffective and at the most, harmful.[35]

Interdisciplinary management The role of physical therapy and behavioral therapy in treatment of PLPS cannot be discounted. Many patients suggest that physical therapy has not benefitted them in the past. The reasons for this are multiple but often include (1) a therapist with strong bent toward aggressive rehabilitation which is difficult for the deconditioned PLPS patient or (2) the deconditioning itself concomitant with the PLPS discourages many patients giving a sense of hopelessness with regards to rehabilitation. A trusted therapist who understands that the PLPS patient did not become dysfunctional suddenly and that it

will take time and patience to regain function is key to improvement. The services of an experienced behavioral therapist with pain management experience helps the patient to understand that their plight is not unusual and that the mental approach to chronic disease management often is a major determinant in return to function.[3] This is especially true for the patient who may have become opioid habituated along the way but is now deriving less benefit from this therapy. Soberly addressing the true reasons behind usage of medications commonly prescribed by practitioners such as opioids and benzodiazepines is of vital importance. Determining on a consistent basis that aberrant usage patterns are not developing is also vital to ensuring that the PLPS patient is moving from a less functional state to improved function.

5. Complications/Conclusions Postlaminectomy pain syndrome is a very common clinical entity encountered in contemporary pain medicine due to the explosion in spinal surgery with its estimated 20–40% failure rate with regard to pain relief.[2] The complications of PLPS are the summation of the risks of all the treatment modalities discussed and therefore are difficult to estimate.[3] One common refrain heard anecdotally from patients is that pain is worsening while repeat imaging shows little disease progression. Perhaps the biggest complication in PLPS is the tendency for diligence in clinical work-up to fatigue after years of treating a patient with marginal improvement in symptoms or satisfaction. This diligence fatigue on the part of the clinician could prevent detection of new pathologies, which must always be in the differential diagnosis. Continuously monitoring the patient in a cost-efficient manner (i.e., not ordering expensive imaging routinely) while maintaining a watchful eye for new or clinically worsening pathology is a clinical challenge. The constellation of symptoms with PLPS usually requires a multimodal and multidisciplinary approach to management. Providing the patient with a stable clinical environment often is key to (1) limiting unnecessary expenditures to the health system in the form of emergency department visits and (2) providing reassurance that easily accessible medical attention for this chronic ongoing problem is available.

179

Chapter 24: Management of the patient with postlaminectomy pain syndrome

References 1.

Vleggeert-Lankamp C, Arts M, Jacobs W, Peul W. Failed back (surgery) syndrome: time for a paradigm shift. J Pain. 2013;7:48–55.

2.

Deyo R, Gray D, Kreuter W, Mirza S, Martin B. United States trends in lumbar fusion surgery for degenerative conditions. Spine. 2005;30:141–145.

3.

Hussain A, Erdek M. Interventional pain management for failed back surgery syndrome. Pain Pract. 2013;3. doi: 10.1111/ papr.12035.

4.

5.

Slipman CW, Shin CH, Patel RK, et al. Etiologies of failed back surgery syndrome. Pain Med. 2002;3:200–214; discussion 214–217. Ross J, Frederickson R, et al. Association between peridural scar and recurrent radicular pain after lumber discectomy: magnetic resonance evaluation. ADCONL European study group. Neurosurgery. 1996:38:855–861.

6.

Herzog RJ, Marcotte PJ. Assessment of spinal fusion. Critical evaluation of imaging techniques. Spine (Phila Pa 1976). 1996;21:1114–1118.

7.

Bokov A, Isrelov A, Skorodumov A, et al. An analysis of reasons for failed back surgery syndrome and partial results after different types of surgical lumbar nerve root decompression. Pain Physician. 2011;14:545–557.

8.

9.

Guyer RD, Patterson M, Ohnmeiss DD. Failed back surgery syndrome: diagnostic evaluation. J Am Acad Orthop Surg. 2006;14:534–543. Marnish N, Brumann, M, Hodler J, et al. Radiologic criteria for the diagnosis of spinal stenosis: results of the Delphi survey. Radiology. 2012;264(1):174–179.

10. Buenaventura RM, Shah RV, Patel V, Benyamin R, Singh V. Systematic review of discography as a diagnostic test for spinal pain:

180

an update. Pain Physician. 2007; 10(1):147–164. Review. PubMed PMID: 17256028. 11. Shah RV, Everett CR, McKenzieBrown AM, Sehgal N. Discography as a diagnostic test for spinal pain: a systematic and narrative review. Pain Physician. 2005;8(2):187–209. PubMed PMID: 16850074. 12. Shah RV, Merritt W, Collins D, Racz GB. Targeting the spinal nerve via a double-needle, transforaminal approach in failed back surgery syndrome: demonstration of a technique. Pain Physician. 2004;7(1):93–97. PubMed PMID:16868618. 13. Gharibo CG, Varlotta GP, Rhame EE, et al. Interlaminar versus transforaminal epidural steroids for the treatment of subacute lumbar radicular pain: a randomized, blinded, prospective outcome study. Pain Physician. 2011;14:499–511. 14. Deer T. An update on the medical pain management of the multiply operate lumbar spine including neuroablative and other minimally invasive techniques. Semin Spine Surg. 2008; 20:248–256. 15. Candido K, Raghawendra M, Chinthagada M, et al. A prospective evaluation of iodinated contrast flow patterns with fluoroscopically guided lumbar epidural steroid injections: the lateral parasagittal interlaminar epidural approach versus the transforaminal epidural approach. Anesth Anal. 2008;106 (2);638–644. 16. Manchikanti L, Abdi S, Alturi S, et al. An update of comprehensive evidence-based guidelines for interventional techniques in chronic spinal pain. Part II: Guidance and recommendations. Pain Physician. 2013;16:S49–S283. 17. Manchikanti L, Singh V, Cash KA, Pampati V, Datta S. Management of pain of post lumbar surgery

syndrome: one-year results of a randomized, double-blind, active controlled trial of fluoroscopic caudal epidural injections. Pain Physician. 2010;13:509–521. 18. Yousef AA, EL-Deen AS, Al-Deeb AE. The role of adding hyaluronidase to fluoroscopically guided caudal steroid and hypertonic saline injection in patients with failed back surgery syndrome: a prospective, doubleblinded, randomized study. Pain Pract. 2010;10:548–553. 19. Cohen SP, Hurley RW, Christo PJ, et al. Clinical predictors of success and failure for lumbar facet radiofrequency denervation. Clin J Pain. 2007;23:45–52. 20. Leclaire R, Fortin L, Lambert R, Bergeron YM, Rossignol M. Radiofrequency facet joint denervation in the treatment of low back pain: a placebocontrolled clinical trial to assess efficacy. Spine (Phila Pa 1976). 2001;26:1411–1416; discussion 1417. 21. Manchikanti L, Pampati V, Bakhit CE, Pakanati RR. Non-endoscopic and endoscopic adhesiolysis in post-lumbar laminectomy syndrome: a one-year outcome study and cost effectiveness analysis. Pain Physician. 1999;2:52–58. 22. Manchikanti L, Pampati V, Fellows B, et al. Role of one day epidural adhesiolysis in management of chronic low back pain: a randomized clinical trial. Pain Physician. 2001;4:153–166. 23. Manchikanti L, Rivera JJ, Pampati V, et al. Spinal endoscopic adhesiolysis in the management of chronic low back pain: a preliminary report of a randomized, double-blind trial. Pain Physician. 2003;6:259–267. 24. Manchikanti L, Singh V, Cash KA, Pampati V, Datta S. A comparative effectiveness evaluation of percutaneous adhesiolysis and epidural steroid

Chapter 24: Management of the patient with postlaminectomy pain syndrome

injections in managing lumbar post-surgery syndrome: a randomized, equivalence controlled trial. Pain Physician. 2009;12:E355–E368. 25. Kumar K, Taylor RS, Jacques L, et al. Spinal cord stimulation versus conventional medical management for neuropathic pain: a multicenter randomized controlled trial in patients with failed back surgery syndrome. Pain. 2007;132:179–188. 26. Kumar K, Taylor RS, Jacques L, et al. The effects of spinal cord stimulation in neuropathic pain are sustained: a 24-month followup of the prospective randomized controlled multicenter trial of the effectiveness of spinal cord stimulation. Neurosurgery. 2008;63:762–770; discussion 770. 27. Manca A, Kumar K, Taylor RS, et al. Quality of life, resource consumption and costs of spinal cord stimulation versus conventional medical management in neuropathic pain patients with failed back surgery syndrome (PROCESS trial). Eur J Pain. 2008;12:1047–1058.

28. North RB, Kidd D, Shipley J, Taylor RS. Spinal cord stimulation versus reoperation for failed back surgery syndrome: a cost effectiveness and cost utility analysis based on a randomized, controlled trial. Neurosurgery. 2007;61:361–368; discussion 368–369.

Conference 2012: Recommendations for management of pain by Intrathecal (intraspinal) drug delivery: Report of an interdisciplinary expert panel. Neuromodulation. 2012;15: 436–466.

29. Reverberi C, Dario A, Barolat G. Spinal cord stimulation (SCS) in conjunction with peripheral nerve field stimulation (PNFS) for the treatment of complex failed back surgery syndrome. Neuromodulation. 2013;16:78–83.

33. Falco F, Patel V, Hayek S et al. Intrathecal infusion systems for long-term management for chronic non-cancer pain: an update of the evidence. Pain Physician. 2013;16: SE185–216.

30. White AP, Arnold PM, Norvell DC, Ecker E, Fehlings MG. Pharmacologic management of chronic low back pain: synthesis of the evidence. Spine (Phila Pa 1976). 2011;36(21 Suppl): S131–S143.

34. Medtronic pump refill safety bulletin June 2013. http:// professional.medtronic.com.

31. Dworkin RH, O’Connor AB, Audette J, et al. Recommendations for the pharmacological management of neuropathic pain: an overview and literature update. Mayo Clin Proc. 2010;85(3 Suppl): S3–S14. 32. Deer T, Prager J, Levy R, Rathmell J, et al. Polyanalgesic Consensus

35. Grider J, Harned M, Etscheidt M. Patient selection and outcomes using a low-dose intrathecal opioid trialing method for chronic nonmalignant pain. Pain Physician. 2011;14:1533–1559. 36. Shah RV, Merritt W, Collins D, Racz GB. Targeting the spinal nerve via a double-needle, transforaminal approach in failed back surgery syndrome: demonstration of a technique. Pain Physician. 2004;7(1):93–97.

181

Section 2 Chapter

25

Spinal Disorders

A patient with a lumbar compression fracture Nihir Waghela and Magdalena Anitescu

Case study A 73-year-old woman with a long history of osteoporosis had sudden onset of unrelenting and severe mid-back pain after a strong sneeze. Her pain was unresponsive to a 6-week course of conservative therapy with NSAIDs and opiates. She has been bed bound since her symptoms started and was being referred for consultation to a pain specialist for her treatment.

1. What is the differential diagnosis? a. b. c. d. e.

Vertebral compression fracture Acute intervertebral disc herniation Degenerative disc disease Muscle spasm of the paravertebral muscles Degenerative joint disease and facet arthropathy

In a patient with severe osteoporosis vertebral compression fracture is suspected especially after sudden increased axial pressure from a forceful cough. The main symptom of a compression fracture is unrelenting pain even with small movements, similar to that described in the case above. In a compressed vertebral body, tapping over the mid-thoracic area elicits pain. In most cases, the pain does not radiate from the localized area of discomfort, thus the practitioner must differentiate between radicular pain from disc herniation or back pain from facet arthropathy and a newly developing compression fracture. Vertebral compression fractures can occur spontaneously in osteoporosis or after minimal pro-dromal injury. Postmenopausal women with osteoporosis are most likely to be affected. Depending on the magnitude of force applied to the osteoporotic bone, the fracture can be a simple or a burst compression fracture.

2. What is the difference between a simple vertebral compression fracture and a burst fracture? In 1983 Denis described a three column model for the spine: anterior, middle, and posterior. The anterior column consists of the anterior longitudinal ligament, anterior half of the vertebral body, and its adjacent intervertebral disc and annulus. The middle column consists of the posterior half of the vertebral body, disc, the posterior annulus, and the posterior longitudinal ligament. The posterior column consists of the facet joints, transverse process, spinous process, pedicles, ligamentum flavum, and inter- and supraspinous ligaments.[1–3]

Figure 25.1. The anterior, middle, and posterior column as described by Denis.[1]

Case Studies in Pain Management, ed. Alan David Kaye and Rinoo V. Shah. Published by Cambridge University Press. © Cambridge University Press 2015.

182

Chapter 25: A patient with a lumbar compression fracture

The middle column is in the neutral axis of the spine during flexion and extension. It provides the greatest mechanical stability and bears the greatest axial load of the spine. A compression fracture affects only the anterior column; a burst fracture, both the anterior and the middle columns.[1–3] In a compression fracture the anterior column fails under compression while the middle and posterior columns remain intact. The intact middle column acts as a hinge. The fracture is usually stable and rarely associated with neurologic compromise because bone fragments are not retropulsed into the vertebral canal. The column becomes unstable if the ligamentous complex in the posterior column is disrupted.[1–3] A burst fracture results from failure of the vertebral body under an axial load. The anterior and middle columns fail under compression; the posterior column remains intact. Posterior fragments may be driven into the spinal cord or cauda equina, making this injury more dangerous than a simple compression fracture. A burst fracture can be stable or unstable depending on the integrity of the posterior ligamentous complex.[1–3]

Figure 25.2. Forces acting from above and below will increase flexion. Most thoracolumbar injuries are hyperflexion injuries.[1,2,7]

3. What is the pathogenesis and pathophysiology of a vertebral compression fracture? Is osteoporosis a factor? Each year about 700 000 vertebral compression fractures occur in the USA, with a prevalence of up to 25% in women over the age of 50. Only about onethird of those fractures are symptomatic but almost all increase in morbidity and mortality from functional and psychologic impairment.[4,5] The most common site of injury to the spine is the thoracolumbar junction where the spine transitions from the more rigid thoracic spine to the more mobile lumbar spine.[6] Injuries to the vertebral bodies tend to occur from compression, flexion, or twisting forces. The posterior elements are usually damaged by direct trauma. Most injuries occur as a consequence of hyperflexion since the predominant natural force of the spine is that of flexion.[1,2,7] Osteoporosis is a skeletal disease characterized by compromise in bone strength predisposing an individual to fracture. Two types of osteoporosis have

been described (Table 25.1).[9] Osteoporosis is the main cause of vertebral compression fractures. Secondary causes of osteoporosis must be excluded before diagnosing a patient with primary, idiopathic, or iatrogenic osteoporosis. Secondary causes of osteoporosis include Paget’s disease, malabsorption syndrome, hyperparathyroidism, multiple myeloma, hyperthyroidism, prolonged corticosteroid therapy, and osteomalacia hypogonadism.[10] Osteoporosis is manifested by reduction in bone mass per unit volume. Normal individuals have a bone mineral density (BMD) of one standard deviation (SD) from the mean of young adults.

183

Chapter 25: A patient with a lumbar compression fracture

In osteopenia BMD is between 1 and 2.5 SDs below the mean of the young adult population. In osteoporosis BMD is more than 2.5 SDs from the young adult mean.[11] There is a direct association between each Table 25.1 Osteoporosis clinical manifestation

Type 1

Type 2

Age

51–75

> 70

Sex ratio (F:M)

6:1

2:1

Type of bone loss

Trabecular

Cortical

Rate of bone loss

Accelerated

Not accelerated

Fracture site

Vertebrae (crush) and Colles’ fractures

Vertebral (multiple wedge), pelvic, proximal tibia, and proximal humerus

PTH function

Decreased

Increased

Calcium deficiency

Absent

Present

Metabolism of 25-OH-D to 1, 25(OH)2D

Secondary decrease

Primary decrease

Main causes

Estrogen deficiency, low calcium intake, low weight-bearing routine, cigarette smoking, excessive alcohol intake

Low calcium intake, no estrogen deficiency

SD decrease in bone mineral density and rate of bone turnover and the risk of fracture. Bone strength is based on bone density, turnover, size, area, microarchitecture, and degree of mineralization. The vertebral body is composed of hard cortical bone on the outside and less dense, cancellous trabecular bone inside. The inner, trabecular bone is sensitive to high bone turnover. Trabecular bone is largely responsible for axial and extra-axial stress. With osteoporosis, as trabecular bone density decreases structural strength is lost. Rapid bone turnover leads to an imbalance in bone renewal and to loss of connectivity within the trabeculae to irreversibly weaken the structural integrity of the bone.[12–15]

4. How is vertebral compression fracture diagnosed? The mainstay of diagnosis of vertebral compression fracture is a thorough history and physical examination. Only about one-third of vertebral fractures are diagnosed because patients do not report back pain serious enough to warrant imaging studies.[16–19] Patients often have acute or subacute back pain with no associated provoking event. If the supine position alleviates the pain and standing or walking exacerbates it deep and midline, the pain usually improves with rest even though patients have difficulty sleeping with pain exacerbation on movement.[20–22] A physical examination reveals point tenderness over the area of acute fracture. It may also reveal an increased degree of kyphosis.[23] Kyphosis is an important sign of vertebral compression fracture in the elderly. Figure 25.3. Construction crane analogy as described by Whiteside. The crane falls forward after the cable holding it vertical breaks. Injuries to the spine which compromise the PLC produce this type of motion.[8]

184

Chapter 25: A patient with a lumbar compression fracture

Figure 25.4. Lateral plain film of thoracic spine showing severe wedge fracture of T7 body.[28]

A height loss of > 4 cm is associated with kyphosis of 15 degrees.[24] Gait is usually normal. The simplest and the most cost-effective way to confirm diagnosis is a plain radiograph. Vertebral fractures are commonly observed on radiographs obtained for other reasons in patients who may not show signs or symptoms of fracture. Loss of vertebral height (> 4 mm or > 20% compared to baseline), disruption of alignment along anterior and posterior vertebral body lines, facet dislocation, and an increase in interpedicular and interspinous distance (> 7 mm) are indicators of vertebral column disorder on a plain radiograph.[25,26] Plain film does not detect ligamentous injury.[27] MRI is indicated if there is suspicion of vertebral compression fracture. MRI helps detect the age of the

fracture, spinal column compromise, or tumor presence. Acute injuries are identified by a T2 signal because of increased signal intensity from vertebral body edema.[25] A hypointense T1 image also suggests edema.[29] Short tau inversion recovery (STIR) is an MRI image that causes loss of fat signal (bone marrow) from the relaxation properties of fat protons. STIR imaging is the most sensitive modality for visualization of edema and thus of acute fractures.[29,30] Lack of edema on MRI or lack of radiopharmaceutical uptake on a bone scan suggests chronic fracture.[28] MRI is also useful for differentiating osteoporotic fractures from pathologic fractures by showing bone marrow and contrast enhancement in adjacent tissue for pathologic fractures.[28] The “fluid sign” (presence of a fat-fluid level, or lipohemarthrosis) on MRI can be useful to distinguish osteoporosis from malignancy as the cause for pathologic fracture.[31] A CT scan is used if an MRI is contraindicated. CT can be helpful for identifying a fracture not well visualized on plain films. CT can reveal complex fractures, spinal canal narrowing, and compression or a burst fracture. It can also reveal whether the fracture line has extended through the posterior wall of a vertebral body. It cannot detect horizontal fractures of the vertebral body or the pedicles.[20] A nuclear bone scan is also useful. Acute or unhealed fractures will take up the injected 99mTcMDP tracer in higher concentration than does normal bone. It is particularly helpful in a variation of vertebral compression fracture, the sacral insufficiency fractures, which are difficult to identify on plain films. Sacral injury usually appears as an “H” or “butterfly” pattern across the sacrum.[20,28,32]

5. How are vertebral compression fractures treated? Describe a conservative and an interventional treatment The mainstay and gold standard of treatment for vertebral compression fractures is conservative medical management with or without immobility.[33,34] Conservative management includes a short period of bed rest followed by gradual mobilization with external orthoses.[35] A cruciform anterior spinal hyperextension brace is used since these injuries are usually flexion injuries. Older patients require longer bed rest than younger patients since they do not tolerate

185

Chapter 25: A patient with a lumbar compression fracture

Figure 25.5. Sagittal STIR image shows compression deformity of L2 vertebral body.[29]

braces well.[36] Non-steroidal and opiate medications are prescribed with bed rest and orthoses. Most of these medications have unwanted adverse effects including GI ulcers, hemorrhage, somnolence, physical dependence, tolerance, nausea, vomiting, and constipation. Calcitonin when administered subcutaneously, rectally, or intranasally has analgesic properties from increased endorphin levels.[37–40] Most patients have spontaneous resolution of pain even without medications within 4–6 weeks of initial fracture.[34,41,42] Interventional management is the second line of treatment if conservative management has failed. If pain is refractory to oral medications over a 6- to 12-week period or if oral medications or hospital admission for parenteral narcotics is contraindicated,

186

Figure 25.6. T1-weighted image showing acutely fractured T12 and L1 body with hypointense signal compared to chronically fractured L2 body.[28]

intervention is considered.[33,43–46] Given the possible benefit of effective restoration of vertebral height, vertebral augmentation (VBA) procedures may be indicated in fractures < 3 months old.[47,48] Although fractures can heal spontaneously in 4–6 months, they can also cause symptoms beyond this time frame from chronic non-union or avascular necrosis of the vertebral body (Kummel’s disease).[49] In those instances, vertebral augmentation procedures may help the elderly who are completely disabled by severe back pain.

Chapter 25: A patient with a lumbar compression fracture

Figure 25.7. Lack of tracer uptake on bone scan (A) indicating old fracture. Corresponding plain film (B) showing L2 fracture.[28]

Figure 25.8. Bone scan (A) shows tracer uptake at L2. Corresponding plain film (B) shows burst fracture at L2.[28]

187

Chapter 25: A patient with a lumbar compression fracture

Not all symptomatic patients are suitable for VBA. Common criteria for consideration are as follows: a. Severe incidental pain with movement, reproducible with vigorous tapping over the affected area b. MRI showing edema visible in T2 and STIR images c. Absence of neurologic compromise The best candidates for VBA are patients with fractures of < 50% collapse. Burst fractures with > 60% loss of height may be difficult to access. They may be treated better medically because of the risk of cement leak, embolus, or nerve injury. If medical treatment is ineffective or quality of life rapidly deteriorates, VBA may be considered only after detailed explanation of risks and benefits to the patient and the family.[50–52] There are several surgical options for the management of painful osteoporotic fractures. VBA through minimally invasive techniques such as kyphoplasty and percutaneous vertebroplasty (VP) are among the most popular. Percutaneous vertebroplasty has been performed in the USA since the 1990s. Polymethylmethacrylate (PMMA) is injected into the fractured vertebral body with unipedical or bipedical needles. Most patients report reduced pain and pain-related morbidity with this procedure. Almost half of patients have complete resolution of symptoms.[53–55] Women and patients < 75 years appear to experience the most benefit. The best results have been obtained in fractures < 6 months old. The main objective of VP is not height restoration but stabilization of the fracture and prevention of any further collapse.[26,55,56] The advantages of VP include low cost, a short procedure, and stabilization of the vertebral body. Disadvantages include cement extravasation in more than one-third of patients, limited correction of lost vertebral body height and failure to correct sagittal imbalance.[57] Balloon kyphoplasty, introduced in 1998, is another option for treatment of stable compression fracture. In this procedure a needle with an inflatable vertebral balloon is inserted into the fractured vertebra. The inflated balloon creates a cavity and reexpands the compressed vertebra which is then filled with a thick PMMA mixture under low pressure with a unipedical or bipedical needle.[58] Kyphoplasty may restore height and improve kyphosis.[53,54,59]

188

Kyphoplasty usually works best with acute fractures of < 3 months.[26,47,48] The advantages of kyphoplasty are low cement extravasation, restored vertebral body height, corrected sagittal imbalance, and low complication rate. Disadvantages include increased cost and procedure time, the need for general anesthesia, and an overnight hospital stay.[57] Complications of all VBA procedures include cement leak (41% for vertebroplasty vs. 9% for kyphoplasty), radiculopathy, infection, cord compression, cement embolism, bleeding, hematoma, and neurologic deficit. Cement can leak into the epidural space, blood stream, and surrounding muscles producing a toxic reaction. Delayed complications include fracture at other vertebral levels.[26,54,57,60,61] VBA procedures are contraindicated with active site infection, uncorrectable coagulopathy, young age, pain unrelated to fracture, solid tissue or osteoblastic tumor, spinal instability, pregnancy, myelopathy, burst fractures, fractured pedicles, bone fragment retropulsion, or allergy to PMMA cement or contrast agent.[57] The most common injectable bone cement used in all VBA procedures is PMMA. It reinforces and hardens the fractured vertebral body. In VP a fluid PMMA with a longer liquid phase is injected under high pressure. In balloon kyphoplasty partially cured, “doughy” cement with a shorter liquid phase is injected under lower pressure to fill the cavity left in place by the “tamp” balloon. Cement can be ready for injection in as little as 5 minutes (in partially cured products) to 20 minutes. The polymerization time is increased by refrigerating the product or placing the syringe in a surgical glove filled with ice. All the cement injectable materials contain barium sulfate in various concentrations for direct visualization of the product during injection.[62–65] PMMA is a bioinert material considered ideal for reinforcing frail bone. It does not, however, remodel or biologically integrate into the surrounding bone. It results in a high polymerization temperature of up to 70°C in the center of the vertebrae during cement setting producing discomfort for patients and potential monomer toxicity. PMMA within the fractured bone may increase stiffness of the vertebrae to 174% of baseline, thus predisposing adjacent vertebrae to early post-procedure fractures within 3 months to 1 year. In osteoporotic fractures treated with VBA procedures, other vertebrae fractured in up to 28% of

Chapter 25: A patient with a lumbar compression fracture

patients after VP and up to 33% after balloon kyphoplasty, higher than the 19% incidence reported in the general osteoporotic population. Since 66% of the new fractures after VBA were adjacent to cemented vertebrae, special attention should be paid to the amount of the cement placed within the vertebrae. It is recommended that no more than 4–6 cc per vertebral body and no more than six vertebrae be treated per session (or a maximum of 25 cc cement injected). The most feared complication of PMMA injection is leakage of the material into the nearby structures, entrapping nerve roots and compressing of the spinal cord. Liquid PMMA can cause pulmonary emboli. Cement leakage after VP was 41% and after balloon kyphoplasty, 9%. PMMA can be highly arrythmogenic and cardiotoxic if taken up in the systemic circulation with an estimate of a fatality risk of 1 in 3000–5000 hip arthroplasty surgeries. Assuming similar risk with the VBA procedures, cement use limits should apply. In view of increased morbidity or mortality risk with PMMA vascular uptake, it is currently advisable to limit the amount of cement placed in the vertebral body to a maximum of 6 ml per vertebrae with a maximum of two vertebrae (or 12 ml cement total) treated per surgical intervention.[66,67] Patients undergoing VBA procedures are usually frail. Care is taken in positioning the patient for the procedure, and neurologic status is evaluated immediately after the procedure.[68–71] There is no consensus on the type of anesthesia to be used for the VBA procedures. The number of vertebrae to be treated and the associated patient comorbidities affect the decision. Active distention with the balloon in kyphoplasty can be painful, and general anesthesia may be an option; however, if the number of vertebrae treated exceeds 3, a long operating time in an uncomfortable position may make general anesthesia preferable. Patient comorbidities and age favor local anesthesia and conscious sedation. In patients with advanced COPD and CO2 retention, a deep sedation can aggravate respiratory depression, worsen oxygenation, and increase the right ventricular afterload because of hypercapnia and pulmonary vasoconstriction. Therefore, regional anesthesia can be used for these cases. Using neuraxial short-acting local anesthetic agents and small doses of neuraxial opioids, the VBA can be performed with light systemic sedation and maximum patient comfort.[72–75] Two fluoroscopic image intensifiers are used for anteroposterior and lateral monitoring. If one

fluoroscopic machine is used, it must be adjusted between the two views, lengthening the operative time. Perfect alignment and squared vertebral body in both anteroposterior and lateral positions is ideal for easy pass of working cannulas within the vertebral pedicles. Cement is injected under a direct lateral view and injection is stopped at the least sign of extravasations. “Paste-like” cement has less chance of leakage. Nonetheless, expansion of cement close to the posterior wall can be associated with complications. To prevent PMMA placement close to the posterior wall, introducers are pushed as far forward in the vertebral body as safely possible. Two working cannulas are placed in each vertebral body. Once the desired amount of cement is deposited in the first one, the filler is removed and the original stylet is placed within the working cannula to prevent tracking the cement toward the posterior vertebral wall or in the pedicle. Evaluation of the effect of the VBA takes place within 30 min–1 hour after cementing. Patients report significant pain relief immediately upon hardening of the cement. This procedure is performed in an ambulatory care center, and the patients are routinely discharged home on the same day.[76–78] Response to treatment of osteoporotic fracture is better than to treatment of metastatic disease (75–95% improvement in osteoporosis vs. 56–85% in neoplastic vertebral collapse). With meticulous technique, careful selection of patients, a controlled OR procedure, and frequent post-procedure followup excellent patient outcomes are possible.

6. What is the controversy surrounding vertebral augmentation? Vertebral augmentation is a common procedure now given the rapidity of clinical response and low procedural risk. Yet there has been debate about its utility and societal cost-effectiveness. Some clinicians have described prophylactic vertebroplasty.[61] There is no consensus about when to use VP or balloon kyphoplasty. When 69 peer-reviewed reports of clinical trials of VP and balloon kyphoplasty were studied, pain relief was similar (87% and 92% for VP and balloon kyphoplasty, respectively), and pain scores were comparable (8.2 to 3 for VP and 7.2 to 3.4 for balloon kyphoplasty).[79] Two multicenter, randomized, double-blind placebo-controlled trials have assessed the effectiveness

189

Chapter 25: A patient with a lumbar compression fracture

Figure 25.9. Lumbar kyphoplasty-balloon insufflations – lateral fluoroscopy view. From personal files of Rinoo V. Shah, MD, MBA.

Figure 25.11. Lumbar kyphoplasty-PMMA cement – anteroposterior view. From personal files of Rinoo V. Shah, MD, MBA.

of VP in the treatment of painful osteoporotic compression fractures.[41,80] Both studies had a similar number of patients in both arms, and pain was lessened after VP. Both studies concluded that the VP groups did not show a statistical advantage over

190

Figure 25.10. Lumbar kyphoplasty-PMMA cement – lateral fluoroscopy view; note small cement tail in pedicle-not a complication. From personal files of Rinoo V. Shah, MD, MBA.

Figure 25.12. Vertebroplasty–primary bone tumor. From personal files of Rinoo V. Shah, MD, MBA. 70f with recurrent chondrosarcoma. Bone scan increased activity T8, T9 and CT scan increased radiopacity with some trabecular infiltration of T8 and T9.

the placebo group. These results have stimulated a debate in the medical community. Criticism was mainly about the scientific validity of the trials, possible placebo effect, the high crossover rate between the two arms, and the small sample size.

Chapter 25: A patient with a lumbar compression fracture

Figure 25.13. Staxx kyphoplasty with PEEK wafers. From personal files of Rinoo V. Shah, MD, MBA. Figure 25.14. Staxx kyphoplasty with PEEK wafers and PMMA cement. From personal files of Rinoo V. Shah, MD, MBA.

A randomized controlled trial demonstrated efficacy and safety of kyphoplasty over conventional non-surgical care with improvement in pain and function.[76] Overall, kyphoplasty and vertebroplasty

References 1.

2.

Denis F. The three column spine and its significance in the classification of acute thoracolumbar spinal injuries. Spine. 1983;8(8):817–831. Magerl F, Aebi M, Gertzbein SD, Harms J, Nazarian S. A comprehensive classification of thoracic and lumbar injuries. Eur Spine J. 1994;3(4):184–201.

have a good safety profile with multiple series demonstrating improvement in quality of life, function, and pain control.

osteoporosis: insights afforded by epidemiology. Bone. 1995;17(5): S505–S511. 6.

Patel U, Skingle S, Campbell GA, Crisp AJ, Boyle IT. Clinical profile of acute vertebral compression fractures in osteoporosis. Rheumatology. 1991;30(6): 418–421.

7.

Dennis HH, Tak HH. A review of thoracolumbar spine fracture classifications. J Orthos Trauma. 2011:1–5.

Bohlman HH, Freehafer A, Dejak J. The results of treatment of acute injuries of the upper thoracic spine with paralysis. J Bone Joint Surg Am. 1985;67(3):360–369.

8.

4.

Cummings SR, Melton LJ. Epidemiology and outcomes of osteoporotic fractures. Lancet. 2002;359(9319):1761–1767.

Whitesides TE Jr. Traumatic kyphosis of the thoracolumbar spine. Clin Orthop. 1977;128: 78–92.

9.

5.

Riggs BL, Melton LJ III. The worldwide problem of

Riggs BL, Melton LJ III. Involutional osteoporosis. N Engl J Med. 1986;314(26):1676.

3.

10. Kenny AM, Prestwood KM. Osteoporosis: pathogenesis, diagnosis, and treatment in older adults. Rheum Dis Clin N Am 2000;26(3):569–591. 11. WHO Study Group on Assessment of Fracture Risk and its Application to Screening for Postmenopausal Osteoporosis. Assessment of Fracture Risk and its Application to Screening for Postmenopausal Osteoporosis (No. 843). Geneva: World Health Organization. 1994. 12. Snyder BD, Piazza S, Edwards WT, Hayes WC. Role of trabecular morphology in the etiology of age-related vertebral fractures. Calcif Tissue Int. 1993;53(1):S14–S22. 13. Preteux F, Bergot C, LavalJeantet AM. Automatic quantification of

191

Chapter 25: A patient with a lumbar compression fracture

vertebral cancellous bone remodeling during aging. Anat Clin. 1985;7(3):203–208. 14. Riggs BL, Melton LJ III, O’Fallon WM. Drug therapy for vertebral fractures in osteoporosis: evidence that decreases in bone turnover and increases in bone mass both determine antifracture efficacy. Bone. 1996;18(3):S197–S201. 15. Mosekilde L. Age-related changes in vertebral trabecular bone architecture: assessed by a new method. Bone. 1988;9(4):247–250. 16. Riggs BL, Melton LJ III. The prevention and treatment of osteoporosis. N Engl J Med. 1992;327(9):620–627. 17. Cooper C, Melton LJ III. Vertebral fractures, how large is the silent epidemic? BMJ. 1992;304:793–794. 18. Ross PD. Clinical consequences of vertebral fractures. Am J Med. 1997;103(2):S30–S43. 19. Cooper C, Atkinson EJ, O’Fallon WM, Melton JL. Incidence of clinically diagnosed vertebral fractures: a population-based study in Rochester, Minnesota, 1985-1989. J Bone Miner Res. 1992;7(2):221–227. 20. Old JL, Calvert M. Vertebral compression fractures in the elderly. Am Fam Phys. 2004; 69(1):111. 21. Asenjo JF, Rossel F. Vertebroplasty and kyphoplasty: new evidence adds heat to the debate. Curr Opin Anaesthesiol. 2012;25(5):577–583. 22. Hulme PA, Krebs J, Ferguson SJ, Berlemann U. Vertebroplasty and kyphoplasty: a systematic review of 69 clinical studies. Spine 2006;31(17):1983–2001. 23. Bratton RL. Assessment and management of acute low back pain. Am Fam Phys. 1999; 60(8):2299–2308. 24. Papaioannou A, Watts NB, Kendler DL, et al. Diagnosis and management of vertebral fractures

192

in elderly adults. Am J Med. 2002;113(3):220–228. 25. Alexandru D, So W (2012). Evaluation and management of vertebral compression fractures. The Permanente Journal. 16(4):46. 26. Nevitt MC, Ettinger B, Black DM, et al. The association of radiographically detected vertebral fractures with back pain and function: a prospective study. Ann Intern Med. 1998;128 (10):793–800. 27. Epstein O, Ludwig S, Gelb D, Poelstra K, O’Brien J. Comparison of computed tomography and plain radiography in assessing traumatic spinal deformity. J Spinal Disord Tech. 2009; 22(3):197–201. 28. Lenchik L, Rogers LF, Delmas PD, Genant H K. Diagnosis of osteoporotic vertebral fractures: importance of recognition and description by radiologists. Am J Roentgen. 2004;183(4):949–958. 29. Long SS, Yablon CM, Eisenberg RL. Bone marrow signal alteration in the spine and sacrum. Am J Roentgen. 2010;195(3):W178– W200. 30. Gelderen WF, Al-Hindawi M, Gale RS, Steward AH, Archibald CG. Significance of short tau inversion recovery magnetic resonance sequence in the management of skeletal injuries. Australas Radiol. 1997;41(1): 13–15. 31. Baur A, Stäbler A, Arbogast S, et al. Acute osteoporotic and neoplastic vertebral compression fractures: fluid sign at MR imaging. Radiology. 2012; 225(3):730–735.

Vertebral Compression Fractures. Reston, Virginia: American College of Radiology. 2010. 34. Rousing R, Andersen MO, Jespersen SM, Thomsen K, Lauritsen J. Percutaneous vertebroplasty compared to conservative treatment in patients with painful acute or subacute osteoporotic vertebral fractures: three-months follow-up in a clinical randomized study. Spine. 2009;34(13):1349–1354. 35. Gardner MJ, Demetrakopoulos D, Shindle MK, Griffith MH, Lane JM. Osteoporosis and skeletal fractures. HSS. 2006; 2(1):62–69. 36. Truumees E, Hilibrand A, Vaccaro AR. Percutaneous vertebral augmentation. Spine. 2004;4(2):218–229. 37. Lyritis GP, Tsakalakos N, Magiasis B, et al. Analgesic effect of salmon calcitonin in osteoporotic vertebral fractures: a double-blind placebo-controlled clinical study. Calcif Tissue Int. 1991;49(6):369–372. 38. Pun KK, Chan LW. Analgesic effect of intranasal salmon calcitonin in the treatment of osteoporotic vertebral fractures. Clin Ther. 1988;11(2):205–209. 39. Lyritis GP, Ioannidis GV, Karachalios T, Roidis N, Kataxaki E. Analgesic effect of salmon calcitonin suppositories in patients with acute pain due to recent osteoporotic vertebral crush fractures: a prospective double-blind, randomized, placebo-controlled clinical study. Clin J Pain. 1999;15(4):284–289.

32. Gates GF. SPECT bone scanning of the spine. Semin Nucl Med. 1998;28(1):78–94.

40. Laurian L, Oberman Z, Graf E, et al. Calcitonin-induced increase in ACTH, β-endorphin and cortisol secretion. Horm Metab Res. 1986;18(04):268–271.

33. Saad WE, Funaki BS, Ray CE Jr, et al. Expert Panel on Interventional Radiology: ACR Appropriateness Criteria. Radiologic Management of

41. Kallmes DF, Comstock BA, Heagerty PJ, et al. A randomized trial of vertebroplasty for osteoporotic spinal fractures. N Engl J Med. 2009;361(6):569–579.

Chapter 25: A patient with a lumbar compression fracture

42. Silverman SL. The clinical consequences of vertebral compression fracture. Bone. 1992;13:S27–S31. 43. Cortet B, Cotton A, Boutry N, et al. Percutaneous vertebroplasty in the treatment of osteoporotic vertebral compression fractures: osteoporotic vertebral compression fractures. An open prospective study. J Rheumatol. 1999;26(10):2222–2228. 44. Cotten A, Boutry N, Cortet B, et al. Percutaneous vertebroplasty: state of the art. Radiographics. 1998;18(2):311–320. 45. Jensen ME, Dion JE. Percutaneous vertebroplasty in the treatment of osteoporotic compression fractures. Neuroimaging Clin N Am. 2000; 10(3):547–568. 46. Jensen ME, Evans AJ, Mathis JM, et al. Percutaneous polymethylmethacrylate vertebroplasty in the treatment of osteoporotic vertebral body compression fractures: technical aspects. Am J Neuroradiol. 1997;18(10):1897–1904. 47. Shen MS, Kim YH. Vertebroplasty and kyphoplasty: treatment techniques for managing osteoporotic vertebral compression fractures. Bulletin NYU Hospital Joint Diseases. 2006;64(3–4):106. 48. Garfin SR, Buckley RA, Ledlie J. Balloon Kyphoplasty Outcomes Group: Balloon kyphoplasty for symptomatic vertebral body compression fractures results in rapid, significant, and sustained improvements in back pain, function, and quality of life for elderly patients. Spine. 2006; 31(19):2213–2220. 49. Van Eenenaam DP, Georges Y. Delayed post-traumatic vertebral collapse (Kummell’s disease): case report with serial radiographs, computed tomographic scans, and bone scans. Spine. 1993; 18(9):1236–1241.

50. Heini PF, Wälchli B, Berlemann U. Percutaneous transpedicular vertebroplasty with PMMA: operative technique and early results. Eur Spine J. 2000; 9(5):445–450. 51. Mathis JM, Deramond H, Belkoff SM, eds. Percutaneous Vertebroplasty. New York: Springer. 2002. 52. Lieberman IH, Togawa D, Kayanja MM. Vertebroplasty and kyphoplasty: filler materials. Spine. 2005;5(6):S305–S316. 53. Blattert TR, Jestaedt L, Weckbach A. Suitability of calcium phosphate cement in osteoporotic vertebral body fracture augmentation: a controlled, randomized, clinical trial of balloon kyphoplasty comparing calcium phosphate versus polymethylmethacrylate. Spine. 2009;34(2):108–114. 54. Muto M, Perrotta V, Guarnieri G, et al. Vertebroplasty and kyphoplasty: friends or foes? Radiol Med (Torino). 2008; 113(8):1171–1184. 55. Masala S, Mammucari M, Angelopoulos G, et al. Percutaneous vertebroplasty in the management of vertebral osteoporotic fractures. Shortterm, mid-term and long-term follow-up of 285 patients. Skeletal Radiol. 2009;38(9):863–869. 56. Jha RM, Yoo AJ, Hirsch AE, Growney M, Hirsch JA. Predictors of successful palliation of compression fractures with vertebral augmentation: singlecenter experience of 525 cases. J Vasc Interv Radiol. 2009; 20(6):760–768. 57. Halpin RJ, Bendok BR, Liu JC. Minimally invasive treatments for spinal metastases: vertebroplasty, kyphoplasty, and radiofrequency ablation. J Support Oncol. 2004; 2(4):339–351. 58. Papadopoulos EC, Edobor-Osula F, Gardner MJ, Shindle MK, Lane JM. Unipedicular balloon

kyphoplasty for the treatment of osteoporotic vertebral compression fractures: early results. J Spinal Disord Tech. 2008;21(8):589–596. 59. Carbognin G, Sandri A, Girardi V, et al. Treatment of type-A3 amyelic thoracolumbar fractures (burst fractures) with kyphoplasty: initial experience. Radiol Med. 2009;114(1):133–140. 60. Eskey CJ, Hirsch JA, Manchikanti L. Vertebroplasty and kyphoplasty. In Manchikanti L, Singh V, eds. Interventional Techniques in Chronic Spinal Pain. Paducah, KY: ASIPP Publishing. 2007: pp. 633–652. 61. Kobayashi N, Numaguchi Y, Fuwa S, et al. Prophylactic vertebroplasty: cement injection into non-fractured vertebral bodies during percutaneous vertebroplasty. Acad Radiol. 2009;16(2):136–143. 62. Deramond H, Wright NT, Belkoff SM. Temperature elevation caused by bone cement polymerization during vertebroplasty. Bone. 1999; 25(2):17S–21S. 63. Belkoff SM, Molloy S. Temperature measurement during polymerization of polymethylmethacrylate cement used for vertebroplasty. Spine. 2003;28(14):1555–1559. 64. Webb JCJ, Spencer RF. The role of polymethylmethacrylate bone cement in modern orthopaedic surgery. J Bone Joint Surg Br. 2007;89(7):851–857. 65. Jaeblon T. Polymethylmethacrylate: properties and contemporary uses in orthopaedics. J Am Acad Orthop Surg. 2010;18(5): 297–305. 66. Coventry MB, Beckenbaugh RD, Nolan DR, Ilstrup DM. 2,012 total hip arthroplasties: a study of postoperative course and early complications. J Bone Joint Surg. 1974;56(2):273–284.

193

Chapter 25: A patient with a lumbar compression fracture

67. Charnley J. Systemic Effects of Monomer. Acrylic Cement in Orthopaedic Surgery. Baltimore: Williams and Wilkins Livingstone. 1970: pp. 72–78. 68. Kasperk C, Grafe IA, Schmitt S, et al. Three-year outcomes after kyphoplasty in patients with osteoporosis with painful vertebral fractures. J Vasc Interv Radiol. 2010;21(5): 701–709. 69. McGirt MJ, Parker SL, Wolinsky JP, et al. Vertebroplasty and kyphoplasty for the treatment of vertebral compression fractures: an evidenced-based review of the literature. Spin. 2009;9(6): 501–508. 70. Liu JT, Liao WJ, Tan WC, et al. Balloon kyphoplasty versus vertebroplasty for treatment of osteoporotic vertebral compression fracture: a prospective, comparative, and randomized clinical study. Osteoporos Int. 2010;21(2): 359–364. 71. Pflugmacher R, Taylor R, Agarwal A, et al. Balloon kyphoplasty in the treatment of metastatic disease of the spine: a 2-year prospective

194

evaluation. Eur Spine J. 2008; 17(8):1042–1048.

trial. Lancet. 2009;373 (9668):1016–1024.

72. Krueger A, Bliemel C, Zettl R, Ruchholtz S. Management of pulmonary cement embolism after percutaneous vertebroplasty and kyphoplasty: a systematic review of the literature. Eur Spine J. 2009;18(9):1257–1265.

77. Yang HL, Wang GL, Niu GQ, et al. Using MRI to determine painful vertebrae to be treated by kyphoplasty in multiple-level vertebral compression fractures: a prospective study. J Int Med Res. 2008;36(5):1056–1063.

73. Elshaug AG, Garber AM. How CER could pay for itself: insights from vertebral fracture treatments. N Engl J Med. 2011;364(15):1390–1393.

78. Berenson J, Pflugmacher R, Jarzem P, et al. Cancer Patient Fracture Evaluation (CAFE) Investigators. Balloon kyphoplasty versus non-surgical fracture management for treatment of painful vertebral body compression fractures in patients with cancer: a multicenter, randomised controlled trial. Lancet Oncol. 2011;12(3): 225–235.

74. Burton AW, Hamid B. Kyphoplasty and vertebroplasty. Curr Pain Headache Repr. 2008; 12(1):22–27. 75. Schofer MD, Efe T, Timmesfeld N, Kortmann HR, Quante M. Comparison of kyphoplasty and vertebroplasty in the treatment of fresh vertebral compression fractures. Arch Orthop Trauma Surger. 2009;129(10):1391–1399. 76. Wardlaw D, Cummings SR, Van Meirhaeghe J, et al. Efficacy and safety of balloon kyphoplasty compared with non-surgical care for vertebral compression fracture (FREE): a randomised controlled

79. Hulme PA, Krebs J, Ferguson SJ, Berlemann U. Vertebroplasty and kyphoplasty: a systematic review of 69 clinical studies. Spine. 2006;31(17):1983–2001. 80. Buchbinder R, Osborne RH, Ebeling PR, et al. A randomized trial of vertebroplasty for painful osteoporotic vertebral fractures. N Engl J Med. 2009;361(6): 557–568.

Section 2 Chapter

26

Spinal Disorders

Sacroiliac joint pain and arthritis Garrett LaSalle and Jianguo Cheng

Case study A 45-year-old female presents with persistent low back and right buttock pain for 13 months. The pain started after a motor vehicle accident, in which her automobile was struck from behind while the patient was stepping on the brake pedal of her car with her right foot. The pain is described as sharp and aching, with the most intense pain being located in the right buttock, and radiation of the pain down the posterolateral aspect of the right lower extremity to the ankle. Straight leg raising test is negative. A trial of oral NSAIDs, duloxetine, and physical therapy for 3 months failed to provide significant improvement. MRI of the lumbosacral spine demonstrated mild multilevel degenerative disc and facet arthropathy without any significant neuroforaminal or central canal stenosis. She has undergone two separate transforaminal epidural steroid injections by a local pain physician (at L5 and subsequently at S1) without significant improvement.

1. What are the differential diagnoses in this patient? The differential diagnosis of this patient includes:  Discogenic pain  Lumbosacral radiculitis  Myofascial pain/trigger points of gluteal musculature  Piriformis syndrome  Sacroiliac joint (SIJ) pathology  Lumbar facet arthropathy SI joint pain accounts for approximately 15–30% patients complaining of axial low back pain.[1] The

clinical history of patients presenting with SI joint pain can be quite diverse. Pain in the SI joint is commonly associated with an inciting event. The inciting events can be classified as a single traumatic event (44%), repetitive injury (21%), or idiopathic (35%).[2] The most common single traumatic events include motor vehicle accidents, falls onto the buttock, pregnancy/parturition, and pelvic fracture. Repetitive injury can be secondary to repeated lifting, running, and altered gait secondary to disorders of the lower extremities. The association of SI joint pain with altered gait deserves special attention. Unbeknownst to the patient, a minor leg length discrepancy, prior lower extremity injury resulting in altered gait, or even a slight lower extremity asymmetry that may be introduced by lower extremity total joint arthroplasties (hip or knee) can lead to altered biomechanics that may ultimately manifest as SI joint arthritis and pain. Low back pain is a very common complaint among pregnant women. One study[3] that followed 855 pregnant women starting at 12 weeks gestation found the 9-month prevalence of all types of back pain to be 49%, with pain localized in the SI joint region in approximately 50% of the patients. In addition, of the three distinct patterns of back pain reported (high back pain above the lumbar region, low back pain, and SI pain), the SI pain pattern was the only pattern associated with increasing intensity as the pregnancy progressed. A major limitation of this study is that the diagnosis of SI joint pain was made solely on the basis of pain location, and not on the basis of provocative physical exam testing or the gold-standard diagnostic test (fluoroscopically guided local anesthetic injection into the SI joint).

Case Studies in Pain Management, ed. Alan David Kaye and Rinoo V. Shah. Published by Cambridge University Press. © Cambridge University Press 2015.

195

Chapter 26: Sacroiliac joint pain and arthritis

A class of rheumatologic conditions known as the seronegative spondyloarthropathies is associated with increased risk of SI joint pain. There are usually five distinct entities in this group: ankylosing spondylitis, enteropathic arthritis (arthritis associated with inflammatory bowel disease), psoriatic arthritis, reactive arthritis, and idiopathic/undifferentiated spondyloarthropathy. A key feature of SI joint pain in this population is the inflammatory nature of the pain. This is often manifested as a dull pain with prominent morning stiffness and pain that lasts at least 30 minutes, but often several hours, with gradual improvement as the day progresses. This is in contrast with degenerative SI joint pain, which is often worse at the end of a day of activity. The classic description of inflammatory back pain is: “worse with rest, improved with light activity.” The pain may initially present in the buttocks, but as the systemic inflammatory process progresses the lower lumbar and other areas of the spine may become symptomatic. Additionally, SI joint pain in the spondyloarthropathies usually has an earlier age of onset (usually adolescence or early adulthood). Other features that can signal SI joint pain related to a systemic inflammatory condition include the following: positive family history of SI joint pain, HLA B27 positivity, and loss of spinal and chest wall mobility (evaluated with the modified Schober test and chest wall expansion evaluation).[4] If any of these features are present in the history of a patient with SI joint pain, further evaluation may need to be done. These features may indicate an underlying inflammatory systemic condition that requires, not simply pain management, but also prompt rheumatologic evaluation and systemic treatment.

Another group of patients at increased risk for SI joint pathology/pain include those with prior lumbar or lumbosacral fusion. Studies have shown increased rates of degenerative disease that occur following spinal fusion in those spinal segments adjacent to the fused segments. This likely results, at least partially, from altered biomechanics at those segments above and below the fusion mass. The SI joint constitutes an essential articulation within the spine axis and, thus can functionally be classified as a spinal segment, subject to the same biomechanical and degenerative processes that affect other spinal segments after fusion.[5]

2. What are the referred pain patterns that are important in the differential diagnosis? Patients with SI joint pain tend to present with variable pain distribution patterns that are not confined to the anatomic region of the SI joint (Figure 26.1). A retrospective study[6] evaluated the pain referral patterns of 50 patients with injection-confirmed SI joint pain. The vast majority of patients (> 90%) reported buttock pain, with a large majority (72%) reporting lower lumbar pain. Half of all patients had some form of lower extremity pain, with more than 50% reporting pain radiating distal to the knee and 12% reporting pain as far distal as the foot. This finding suggests that not all low back pain with radiation to the lower extremity (even as far distally as the foot) is secondary to lumbosacral radiculitis. The SI joint innervation is very complex and variable between patients, resulting in multiple, complex referral zones. In addition, there are a large number of structures intimately associated with the SI joint that Figure 26.1 Sacroiliac joint injection. From personal files of Rinoo V. Shah, MD, MBA.

196

Chapter 26: Sacroiliac joint pain and arthritis

can be affected by inflammatory or structural pathology, including the piriformis muscle, sciatic nerve, and lumbosacral spinal nerves. Even though the SI joint is a paramedian structure, axial lower lumbar pain is common. It often responds to unilateral SI joint injection. While the majority of pain referral patterns included the low back, buttocks, and lower extremities (as would be expected), atypical pain referral patterns do occur, such as referred pain in the groin and abdomen.

3. What anatomic considerations help to explain the variable presentation of SI joint pain? The sacroiliac joint is the articulating surface between the sacrum and the iliac bones, thus providing the functional unit connecting the spine to the lower extremities. The SI joint is classified as a true diarthroidial joint–with opposing articular surfaces separated by a synovial fluid-filled space and covered by a fibrous capsule. Despite this general characteristic, the SI joint has several unique features. It has a large surface area covered with both hyaline cartilage and fibrocartilage and it has much less mobility among its surfaces (most of the motion at this joint is not perceived during the majority of physiologic activities). Unlike most joints that have smooth articulating surfaces such as the knee or hip, the lateral surface of the sacrum is characterized by multiple elevations and depressions that articulate with complementary depressions and elevations on the medial surface of the iliac bones, thus providing a loose, multiplyrepeated interlocking contact surface which serves to significantly reduce the mobility of the joint.[7] The innervation of the sacroiliac joint is complex and variable. The posterior innervation of the SI joint most commonly arises from the dorsal ramus of L5 and the lateral branches of the dorsal rami of S1-S3, with contributions in some patients from the L4 medial branch and S4 lateral branch.[1] The ventral innervation of the joint appears to arise from the ventral rami of L5-S2 with some potential contributions from L4. The clinical finding that the SI joint is a source of pain in many patients is supported by histologic examination of joint structures and surrounding tissues. Positive immunohistochemical staining for calcitonin gene-related peptide (CGRP) and substance P has been identified

in both iliac and sacral cartilage and adjacent ligamentous structures.[8]

4. What are the physical exam findings that would suggest SI joint pain? In addition to history, physical examination findings may suggest the SI joint as a pain generator. However, no historical or physical examination findings are either sufficiently sensitive or specific for SI joint pain. As a result, a combination of several tests have commonly been used to increase sensitivity and specificity. The majority of the tests are used to reproduce the pain complaints in patients suffering from SI joint pathology (provocation tests). Five of the most commonly used tests are listed below. How these tests are performed is demonstrated in one of several education videos[9] (the authors have no financial connection to this video). A provocation test is considered positive if the test reproduces the patient’s original pain complaint. 1. Thigh thrust test: The patient lies supine with the hip and knee of the affected side both flexed to approximately 90°. The unaffected hemipelvis is stabilized as dorsal pressure is applied along the axis of the femur of the affected side. 2. Compression test: The patient is placed in a lateral decubitus position with the affected side up, the hips flexed at approximately 45°, the knees flexed at approximately 90°, and a pillow placed between the knees. The examiner exerts medial (downward) pressure over the anterior portion of the iliac crest on the affected side. 3. Distraction test: The patient lies supine with a pillow under the knees and one arm placed under their lumbar spine. The examiner stands at the patient’s side and uses a crossed arm technique to apply dorso-lateral pressure in a sustained manner to the bilateral anterior superior iliac spines. 4. Gaenslen test: The patient lies supine at the edge of the examination table in a position such that the lower extremity of the affected side is allowed to drop toward the floor in an extension motion past the horizontal plane of the examination table. The contralateral (unaffected) hip is flexed maximally with the patient actively pulling that knee toward the chest. The examiner exerts downward/dorsal pressure on the anterior aspects of bilateral knees in an attempt to facilitate a rotation motion.

197

Chapter 26: Sacroiliac joint pain and arthritis

5. Patrick test (FABER test): The patient lies supine. The femur on the affected side is flexed, abducted, and externally rotated (FABER) so as to place the lateral malleolus on the anterior aspect of the contralateral knee. The unaffected hemipelvis is stabilized at the anterior superior iliac spine and dorsal (downward) pressure is applied to the knee of the affected side. Each of these tests in isolation has very poor sensitivity and specificity for diagnosing SI joint pain. However, when several of these tests are positive in the same patient, sensitivity and specificity increase significantly.[10] A combination of three or more positive tests was found to have high sensitivity and specificity (85% and 79%, respectively) as well as high positive and negative predictive values (77% and 87%, respectively) for injection-confirmed SI joint pain. Therefore, multiple provocation tests need to be positive in a single patient in order to have a high suspicion of pain emanating from the SI joint complex. In addition, unilateral pain and worsening symptoms when arising from a sitting position are also contributory.

5. Does imaging play a large role in the diagnostic work-up of this patient? The specificity of imaging findings has traditionally been poor for SI joint pain. Imaging is typically used in an attempt to rule out other types of pathology when the diagnosis is in question.[11] CT imaging of painful SI joints was found to be only 57.5% sensitive and 69% specific.[12] Radionuclide bone scan with Tc-99m only has 12.9% specificity and 100% sensitivity for SI joint pain.[13] MRI appears to be more beneficial in evaluating patients with SI joint pain and concurrent spondyloarthropathy, but does not have good test characteristics for degenerative conditions. Thus, imaging has limited value in the diagnosis of SI joint pathology and pain.

6. How is the diagnosis of SI joint pain made? While the history and physical examination may suggest the SI joint as a primary pain generator, a diagnosis should not be based solely on these findings. The currently accepted gold standard for diagnosis of SI joint pain is a positive response to local anesthetic

198

instillation within the SI joint.[2] Many practitioners advocate a double block paradigm (improvement of pain after each of two separate blocks) because a single intra-articular SI joint injection has a high false-positive rate, which is arbitrarily defined as > 50% pain relief following the first injection but with lack of pain relief following a second (confirmatory) SI joint injection.[1] However, a false-negative SI joint “confirmatory” block is also possible secondary to technical or patient factors. The response to SI joint injections is often used to predict success with sacral lateral branch radiofrequency ablation (RFA). The reliability of this approach is an area of active research.[14]

7. What are the conservative management options in patients with SI joint pain? A trial of conservative management is usually appropriate in most patients. Topical therapy such as lidocaine or NSAIDs can be attempted. Some patients may respond to a TENS unit applied to the painful region. Oral non-opioid analgesics such as NSAIDs and other adjuvant therapies, such as duloxetine and pregabalin, can be considered, as in other chronic musculoskeletal complaints. Physical therapy and manipulative therapy can have significant positive impact with low risk. Attempts to restore proper body mechanics with gluteal, abdominal, and lower extremity strengthening/therapy are often employed. SI joint belts can be used in patients with muscle weakness and pain in order to provide compression, which can improve proprioceptive feedback to gluteal musculature.[15] In those with mild or moderate pain, the belt can be worn only with activity; however, those with severe pain/weakness frequently note improvement with more regular use, even during sedentary activities. Orthotics and shoe modifications such as shoe lifts can be used in an attempt to compensate for lower extremity pathology resulting in altered biomechanics.

8. When considering SI joint injection, what factors should be kept in mind? Many practitioners have advocated the use of contrast injection into the SI joint prior to local

Chapter 26: Sacroiliac joint pain and arthritis

anesthetic injection, in order to obtain an arthrogram under fluoroscopy and verify needle placement into the SI joint. However, considering the fact that both the intra-articular and extra-articular structures of the SI joint such as the capsules and ligaments are innervated by nociceptive fibers, it is conceivable that blocking all these structures would lead to better pain reduction. Elderly patients tend to have pain originating from bilateral SI joints within the SI joint itself (intra-articular pathology such as degenerative arthritic changes). In contrast, young people often have unilateral SI joint pain arising from the periarticular structures such as the muscles, fascia, and ligaments.[1] It is conceivable that the entity “sacroiliac joint pain” is, in reality, a spectrum of disorders: including (at one end of the spectrum) those pain states arising from within the SI joint, those arising from structures outside the joint proper, and those arising from both of these structures. All states result in pain complaints that localize to the SI joint region but likely respond differently to interventional therapies. The differential response of structures within the SI complex was elegantly demonstrated in a double-blind, randomized, placebo-controlled fashion.[16] In this study, 70% of normal subjects were found to have anesthesia of posterior extraarticular structures (interosseous ligament and dorsal sacroiliac ligament) upon L5 dorsal ramus and sacral lateral branch (S1-S3) local anesthetic infiltration in a multi-site and multi-depth technique. However, it was found that 86% of these patients (with excellent posterior SI complex/ extra-articular anesthesia) did not have anesthesia to capsular distention. This finding seems to indicate that extra-articular/posterior SI complex anesthesia is not synonymous with intra-articular anesthesia and that the SI joint is not solely innervated by the sacral dorsal rami. It appears that intra-articular pathology is well-addressed with intra-articular injections, but is not well-addressed with sacral lateral branch blockade. These two diagnostic procedures may be complementary and be used in conjunction with each other to diagnose pain arising from the two separate regions of the SI joint complex. The optimal volume of intraarticular injection is unknown. Rupture of the capsule due to injection of a large volume is a concern. A total volume of 2–4 ml is commonly used in clinical practice.[7,15]

9. Is there a long-term therapeutic solution to SI joint pain? A patient is usually diagnosed as having SI joint pain based on history, physical exam, and one or more diagnostic intra-articular SI joint injections. Once a diagnosis of SI joint pain is made, a more long-term treatment is usually sought. For those who do not have long-lasting pain relief after SI joint steroid injection, RFA is usually employed. The current RFA treatment of chronic SI joint pain targets only the lateral branches of the S1–3 sacral dorsal rami and L4 medial branch/L5 dorsal ramus. This approach is not able to access ventral neural structures, and thus leaves some intra-articular structures untreated. In traditional thermal RFA (tRFA), the region of significant heating surrounding the active tip of a traditional RFA needle is confined to a small area around the active tip of the probe. Because of the complexity and wide variability of the lateral branches, this approach may lead to partial denervation of the SI joint. In an attempt to increase the size of the lesion and modify the geometry of the lesions, two other forms of RFA have been developed, cooled RFA (cRFA) and bipolar RFA (bRFA). Cooled RFA is introduced to create larger lesions lateral to the sacral foramen in hope to denervate as many sub-branches of the lateral branches to overcome the anatomical variations. This technique uses an RF probe that is internally cooled with water/ saline. This cooling prevents tissues surrounding the RF probe from reaching temperatures sufficient to result in tissue charring that may impede further heat conductance from the RF probe tip. When this is avoided, greater energy is transmitted to tissue sites distant to the RF probe tip and larger lesion is created. cRFA is theoretically more amenable to neural ablation. More recently, a bipolar RFA approach has been introduced. It is a modification of the tRFA method.[18] Instead of using a grounding electrode the electric circuitry is completed between two adjacent RFA probes. The probes are placed in close proximity to each other at about 1 cm intervals. Using this method, a “strip lesion” can be created between two such bipolar RFA needles. In a chain of such “strip lesions” barricading the neural foramen and the SI joint, the neural tissue is ablated and a more complete denervation of the SI joint may be achieved.

199

Chapter 26: Sacroiliac joint pain and arthritis

Pulsed RF treatment is a further modification and the electrical current is delivered over very short time frames, with intervening periods of no energy delivery. This pulsed delivery of energy allows for tissues to dissipate heat in between the delivery periods, thus leading to lower maximum temperatures.[17] In fact the temperatures are low enough that neuronal destruction is prevented. The exact mechanisms by which pulsed RF treatment appears to exert its effects are unclear, but may involve the alteration of nociceptive transmission without the destruction of the neural structure.[1] Pulsed RF treatment has been evaluated in a small prospective case series.[17] This study demonstrated a statistically significant improvement in VAS scores in 73% of patients who underwent pulsed RF of the medial branch of L4, dorsal ramus of L5, and S1 and S2 lateral branches. The mean duration of pain relief was 20 weeks which is comparable to the outcomes of tRFA and cRFA. Comparative outcomes of these various modalities of treatment is an active area of research.

 SI joint pain is likely to result from either a single inciting event (motor vehicle accident, fall onto the buttock, parturition, etc.) or repetitive inciting events (running, repetitive lifting, lower extremity pathology altering the biomechanics of ambulation, etc.).  Pain emanating from the SI joint can have variable radiation patterns, with a large

1.

2.

3.

200

Cohen SP, Chen Y, Neufeld NJ. Sacroiliac joint pain: a comprehensive review of epidemiology, diagnosis and treatment. Expert Rev Neurother. 2013;13(1):99–116. Chou LH, Slipman CW, Bhagia SM, et al. Inciting events initiating injection-proven sacroiliac joint syndrome. Pain Med. 2004; 5(1):26–32. Ostgaard HC, Andersson GBJ, Karlsson K. Prevalence of back pain in pregnancy. Spine. 1991; 16(5):549–552.





Key points

References





proportion of patients experiencing pain in the lower extremities, as far distal as the foot. Not all low back pain that radiates to the distal lower extremities is related to lumbosacral radiculitis. Single physical examination/provocation tests for SI joint pain are unreliable, but when three or more of these tests are positive in the same patient, probability of SI joint pathology is greatly increased. Multiple structures of the SI joint complex can result in clinical symptoms of pain. These include intra-articular structures (degenerative arthritis, and inflammatory conditions) as well as extra-articular structures (ligaments, muscles, etc.). It appears that intra-articular injections address intra-articular pathology while targeting the sacral lateral branches, in addition to L5 dorsal ramus and maybe L4 medial branch, addresses a broader pathology of the SI joint. There is no universally accepted method of long-term pain control in chronic SI joint pain. Radiofrequency denervation of the SI joint has emerged as one of the best options. There are multiple types of RF treatment. Comparative outcomes of these modalities remain to be determined. The optimal treatment for chronic SI joint pain involves a multidisciplinary, personalized approach.

4.

Lianne G. Clinical features of ankylosing spondylitis. In Hochberg M, Silman A, Smolen J, Weinblatt M, Weismann M, eds. Rheumatology. Philadelphia, PA: Elsevier. 2011: pp. 1129–1133.

5.

Kee-Yong H, Jun-Seok L, Ki-Won K. Degeneration of sacroiliac joint after instrumented lumbar or lumbosacral fusion. Spine. 2008;33(11):1192–1198.

6.

Slipman CW, Jackson HB, Lipetz JS, et al. Sacroiliac joint pain referral zones. Arch Phys Med Rehabil. 2000;81:334–338.

7.

Forst SL, Wheeler MT, Fortin JD, Vilensky JA. The sacroiliac joint: anatomy, physiology and clinical significance. Pain Physician. 2006;9:61–68.

8.

Szadek KM, Hoogland PVJM, Zuurmond WWA, De Lange JJ, Perez RSGM. Possible nociceptive structures in the sacroiliac joint cartilage: an immunohistochemical study. Clin Anat. 2010;23(2): 192–198.

9.

Mazza, B. Diagnosing SI joint disorders: provocative testing. YouTube® video 2011;6:50.

Chapter 26: Sacroiliac joint pain and arthritis

http://www.youtube.com/watch? v=ukDJ_OxOuBY. 10. van der Wurff P, Buijs EJ, Groen GJ. A multitest regimen of pain provocation tests as an aid to reduce unnecessary minimally invasive sacroiliac joint procedures. Arch Phys Med Rehabil. 2006;87:10–14. 11. Vanelderen P, Szadek K, Cohen SP, et al. Sacroiliac joint pain. Pain Practice. 2010;10(5): 470–478. 12. Elgafy H, Semaan HB, Ebraheim NA, Coombs RJ. Computed tomography findings in patients with sacroiliac pain. Clin Orthop Relat Res. 2001;382:112–118. 13. Slipman CW, Sterenfeld EB, Chou LH, Herzog R, Vresilovic E. The value of radionuclide imaging in the diagnosis of sacroiliac joint syndrome. Spine. 1996;21 (19):2251–2254. 14. Cohen SP, Strassels SA, Kurihara C, et al. Outcome predictors for

sacroiliac joint (lateral branch) radiofrequency denervation. Reg Anesth Pain Med. 2009;34: 206–214.

Uncited References 1.

15. Prather H, Hunt D. Conservative management of low back pain, part I. Sacroiliac joint pain. Dis Mon. 2004;50(12):670–683.

Fortin, JD, Washington WJ, Falco FJE. Three pathways between the sacroiliac joint and neural structures. Am J Neuroradiol. 1999;20:1429–1434.

2.

16. Dreyfuss P, Henning T, Malladi N, Goldstein B, Bogduk N. The ability of multi-site, multi-depth sacral lateral branch blocks to anesthetize the sacroiliac joint complex. Pain Med. 2009; 10(4):679–688.

Cheng J, Pope JE, Dalton JE, Cheng O, Bensitel A. Comparative outcomes of cooled versus traditional radiofrequency ablation of the lateral branches for sacroiliac joint pain. Clin J Pain. 2013;29:132–137.

3

Hagiwara S, Iwasaka H, Takeshima N, Noguchi T. Mechanisms of analgesic action of pulsed radiofrequency on the adjuvant-induced pain in the rat: roles of descending adrenergic and serotonergic systems. Eur J Pain. 2009;13(3):249–252.

4.

Srejic U, Calvillo O, Kabakibou K. Viscosupplementation: a new concept in the treatment of sacroiliac joint syndrome: a preliminary report of four cases. Reg Anesth Pain Med. 1999;24(1):84–88.

17. Vallejo R, Benyamin RM, Kramer J, Stanton G, Joseph NJ. Pulsed radiofrequency denervation for the treatment of sacroiliac joint syndrome. Pain Med. 2006; 7(5):429–434. 18. Cosman ER Jr, Gonzalez CD. Bipolar radiofrequency lesion geometry: implications for palisade treatment of sacroiliac joint pain. Pain Pract. 2011; 11(1):3–22.

201

Section 2 Chapter

27

Spinal Disorders

Sacral insufficiency fracture and treatment options Rinoo V. Shah

Case study A 83-year-old lady slipped on black ice and fell. She developed severe pain in the buttock area and could not get up. She was transferred to the hospital for treatment.

1. What is the differential diagnosis? a. b. c. d. e. f.

Vertebral compression fracture Sacroiliac joint pain Inflammatory sacroiliitis Sacral insufficiency fracture Stress fracture Hip fracture

Sacral insufficiency fracture (SIF) was described by Lourie as “spontaneous osteoporotic fracture of the sacrum.” This is essentially a “stress fracture” of the sacrum in patients with osteoporosis. Postmenopausal women with osteoporosis are the most likely to be affected. Although this can develop with trauma, many patients report a negligible pro-dromal injury or no injury whatsoever. The mean age afflicted is 71–81 years old. In a meta-analysis conducted by Finiels et al the typical patient was over 60 years old and twothirds reported no trauma.

2. What risk factors predispose patients to develop sacral insufficiency fractures? a. b. c. d.

Osteoporosis to osteopenia continuum Rheumatoid arthritis Local pelvic irradiation Corticosteroid use

e. Less common conditions: i. ii. iii. iv.

Hyperparathyroidism Lumbar fusion Paget’s disease Pregnancy (stress fracture versus sacral insufficiency fracture) v. Eating disorders (stress fracture versus sacral insufficiency fracture)

3. Why is this condition overlooked? SIFs are overlooked. There is a low background incidence ranging from 0.14% to 2%. The higher rates were reported in a spine specialty clinic that is dedicated to the diagnostic evaluation of spinal pathology. With advanced imaging options and increased awareness of SIFs, reporting has increased over the past few decades. In Finland, the prevalence of SIFs increased between 1970 and 2012 in patients over 60. Nonetheless, SIFs are still missed due to a lack of knowledge by practitioners, prevalence of other spinal pathologies in the elderly, and the low fidelity of radiographs for detecting SIFs.

4. Describe the anatomy and pathophysiology of a SIF The sacrum looks like a curved spade. It is convex posteriorly and concave anteriorly. The sacrum is composed of a body, sacral ala, rudimentary fused articular processes, superior sacral facets (articulates with inferior articular processes of L5), central spinal canal, sacral foramina (dorsal and ventral), median sacral crest (roof of sacral canal), lateral auricular surface (articulates with ilium to create sacroiliac joint), sacral cornua, and sacral hiatus.

Case Studies in Pain Management, ed. Alan David Kaye and Rinoo V. Shah. Published by Cambridge University Press. © Cambridge University Press 2015.

202

Chapter 27: Sacral insufficiency fracture and treatment options

The Denis classification for traumatic sacral fractures is useful for sacral insufficiency fractures. There are three vertical zones separated by imaginary lines. Zone 1 is lateral to the sacral foramina and extends to the lateral auricular surface. This zone is roughly parallel to the sacroiliac joint. Neurologic symptoms are rare when traumatic fractures occur in this zone. SIFs most commonly occur in Zone 1. Zone 2 is the foraminal zone. Traumatic injuries typically cause radiculopathy. SIFs however primarily affect the bone margins and spare the exiting nerve roots. Zone 3 is defined by the median sacral crest, the sacral canal, and body. Zone 3 traumatic injuries are typically horizontal and may cause cauda equina syndrome. However, SIFs in this region rarely cause cauda equina syndrome. SIFs occur because of normal daily stress and forces transmitted toward weakened bone. Normal axial stresses are transferred from the spine. These pass through the sacrum en route through the lower limbs. The sacrum however has reduced mineralization, reduced elastic resistance, and reduced trabecular bone density. Since compressive strength is proportional to density squared, osteoporosis significantly reduces compressive strength. The ventral corpus or sacral body is better able to withstand forces, as compared to the ala. Furthermore, degenerative disc disease reduces the capacity of the spine to withstand axial forces and transfers forces to the sacrum. Pelvic ring failure results in increased instability and stress across the sacroiliac joints. Repetitive cyclical loading leads to a fracture and regular movement contributes to a persistent nonunion.

5. How do you diagnose a SIF? The pain could be acute as in the above described patient. Other patients may present with insidious, intractable pain radiating to the lower back, buttocks, and pelvis. Pain could refer to the legs and groin. Patients may have significant ambulatory and functional limitations. Sometimes, they are confined to the bed. Pain worsens with weight bearing and improves with rest. There may be tenderness to palpation at the SIJ. SIJ provocation tests may be positive (see Chapter 26), e.g., Patrick test: the hip is flexed, abducted, and externally rotated. Gait is antalgic and apprehensive. Patients may feel they need to hang onto the walls or a walker to move. Neurologic assessment may

be normal. The straight leg raise may be negative for radicular pain but passive movement of the limb may reproduce pain. Laboratory studies are usually not helpful. The alkaline phosphatase may be slightly elevated. However, laboratory studies may be helpful in assessing the etiology of secondary causes of osteoporosis. Radiographs have low fidelity for SIFs. Chronic SIFs may show sclerotic bands. Advanced imaging studies play an important role in diagnosing SIFs. Bone scintigraphy with technetium 99 is very sensitive for acute fractures. An SIF could be diagnosed within 48–72 hours of an injury. The classic H or “Honda” sign may be present in 43% of patients. False-negatives may appear because of uptake in the SIJ due to sacroiliitis or osteoarthritis. False positives occur in the presence of sacral metastases. Hence, false positives are still useful. Magnetic resonance imaging provides detailed anatomic and physiologic information. This test is very sensitive but, again, not specific, i.e., low falsenegative rate but high false-positive rate. Again, false positives may be sacral metastases. The SIF appears as low signal on T1-weighted images, high on T2-weighting, and high on STIR sequences. The latter is particularly useful for detecting bone edema or fractures. The CT scan may be used in patients with contraindications to MRI, e.g., pacemaker implants. The CT scan is also useful for percutaneous cement augmentation procedures, such as sacral kyphoplasty. Bone detail is excellent with sclerotic healing indicating a healing fracture. Fracture lines indicate a recent fracture. Multiplanar reformatting enables 3-dimensional rendering of the fracture. This level of detail is important for percutaneous sacral kyphoplasty.

6. Is there any other diagnostic testing that should be done? A sacral insufficiency fracture, along with vertebral compression fractures, is a fragility fracture. Some consider these pathognomonic for osteoporosis. Low energy compression fractures were rarely referred by orthopedic surgeons for additional diagnostic testing. Only 60% of orthopedic surgeons checked a DEXA (dual energy x-ray absorptiometry) scan. Thirty-nine percent checked serum chemistries. Among those specialists that did not order the above tests, only 63% referred the patient to the primary care physician

203

Chapter 27: Sacral insufficiency fracture and treatment options

or endocrinologist for further management. Pain specialists may be consulted and should ensure appropriate referral for medical management of osteoporosis. DEXA scans evaluate bone density and compare patient results to a nomogram. A patient’s bone density is compared to the mean density of young, healthy adult men or women (T-score). This population represents healthy patients at the peak of bone density. If the T-score is more than 2.5 standard deviations less than this mean, the patient has osteoporosis. A T-score between 1 and 2.5 standard deviations is osteopenia. A T-score less than one standard deviation is considered normal. The Z-score is the mean bone density for agematched controls, not young adult healthy controls. If the patient’s bone density is between 1 and 2.5 standard deviations from the Z-score, this patient has primary or senile osteoporosis. If the bone density is more than 2.5 standard deviations from the Z-score, the patient has a secondary cause of osteoporosis. This requires the input of an endocrinologist.

7. How should I treat this patient? Conservative approaches This is a controversial topic. Patients improve within 2 weeks and then slowly improve over the next 6–12 months. Early rehabilitation is imperative. Patients should avoid immobility and associated deconditioning. Patients should be reminded of the deleterious effects of immobilization: venous thromboembolic disease, urinary retention and infection, reduced cardiac output, postural hypotension, pressure ulcers, depression, catabolic state, and pneumonia. In the USA, some of these complications will not be compensated by insurance carriers. Prevention of these complications necessitates more intensive nursing and rehabilitation care. So, an active approach to conservative care is needed with appropriate use of analgesics and physical therapy. Typical analgesic therapy consists of non-steroidal anti-inflammatories or opioids. These analgesics carry greater risks in the elderly. If a procedure is not planned, venous thromboprophylaxis is advised. A sacroiliac joint belt may help reduce pain and reduce apprehension with standing or transfers. Osteoporosis treatment includes calcium and vitamin D supplementation, antiresorptive agents to slow bone loss, teriparitide (recombinant parathyroid

204

hormone), and selective estrogen receptor modulators. Teriparitide increases bone density, trabecular bone, and cortical thickness, while reducing osteocyte apoptosis.

Procedural Given limitations of conservative care, percutaneous cement augmentation of sacral insufficiency fractures has gained popularity: sacroplasty or sacral kyphoplasty. Open surgical approaches or radiotherapy, although feasible for sacral metastases, are not utilized for SIFs. Surgical fixation with pedicle screws, pins, or similar devices is not used given the osteoporosis and potential migration risks. Sacral kyphoplasty or sacroplasty are minimally invasive procedures with low morbidity and good efficacy rates. Preoperative planning is essential. Medical clearance is advised since the patient may need monitored anesthetic care or general anesthesia. Patients are placed in a prone position so eye, peripheral joint, and skin assessments are useful, to ensure adequate pressure relief. Preoperative antibiotics should be administered within 30–60 minutes of the incision. Review the CT scan preoperatively. Imaging software permits making distance measurements from the median sacral crest to the lateral margin of the foramen. An indelible pen is used to mark the midline on the skin surface. The triangulation tool on the CT scan allows one to trace the sacroplasty trajectory from the skin to the sacral ala; one can then plan the correct trajectory in the patient. Depth, cephalocaudad angulation, and medio-lateral distance may be estimated. Fracture lines can be identified to help predict cement spread. A short axis approach, which is roughly perpendicular to the sacroiliac joint, is commonly used. Lidocaine 1% with 1:100 000 epinephrine is used to anesthetize the skin. Stab incisions are made along a vertical line connecting the 9 o’clock or 3 o’clock positions of the S1-S3 sacral foramina, on the left and right respectively. In the horizontal plane, the incisions are made between the foramina, which are remnants of “fused articular processes”: between the superior articular process of the sacrum and above the S1 foramen; between S1-S2 foramen; and between the S2-S3 foramen. The styletted cannula is directed in a medio-lateral direction, about 45 degrees to the skin surface. Bony

Chapter 27: Sacral insufficiency fracture and treatment options

Figure 27.2. Anteroposterior fluoroscopic view; sacral kyphoplasty post cement delivery. From personal files of Rinoo V. Shah, MD, MBA. Figure 27.1. Sagittal fluoroscopic view; sacral kyphoplasty post cement delivery. From personal files of Rinoo V. Shah, MD, MBA.

contact should ideally be made lateral to the aforementioned vertical line, well within Denis Zone 1. The cannulas are advanced to the auricular surface of the sacrum. The sacroiliac joint and ventral cortex of the sacrum should not be violated. Fluoroscopy confirms final cannula placement. Stylets are removed and a balloon may or may not be inserted. The balloon is insufflated to a volume of 2–4 ml. The balloon creates a cavity, which delimits the space available for cement delivery. Polymethylmethacrylate is mixed with barium powder (for radiopacity). After sufficient viscosity is reached, 1–3 ml of cement is delivered through each cannula. Fluoroscopy is essential to monitor cement spread. Typically, four cannulas (two per side) are sufficient. After the cannulas are removed, steri-strips are applied to the stab incisions and a pressure dressing is applied. Patients report rapid pain relief within hours of the procedure. If relief doesn’t occur almost instantaneously, alternative pain generators should be sought.

8. Are there any complications to worry about following sacroplasty or sacral kyphoplasty? The major risks are infection, bleeding, no pain relief, or nerve damage. A cement leak onto the

sacral foramina or sacral canal can cause permanent damage. Cement can also leak into the soft tissues. This is often asymptomatic; however, the cement hardens and causes an exothermic reaction which could cause soft tissue necrosis and severe pain. The cannulas similarly could violate the cortical bone and cause nerve or soft tissue damage, which is one reason why the cannulas are placed lateral to the fused articular pillars. Cement extravasation into these structures typically will not cause pain or nerve damage. The cement leak is siphoned off, so to speak, by these fused articular pillars.

9. What are the outcomes? According to Frey et al, mean visual analog scores dropped from 8.2 to 3.4 in the immediate postoperative period, in a prospective observational cohort study.[1] The mean visual analog score continued to drop to 0.8 at 52 weeks. Shah confirmed similar findings in a retrospective study of sacral kyphoplasty.[2] There is an ethical dilemma when extrapolating observational and retrospective studies to clinical practice; on the other hand, randomized clinical trials are more difficult to conduct and may have negative outcomes. Practitioners must exercise due diligence, sound judgment, and provide informed consent.

205

Chapter 27: Sacral insufficiency fracture and treatment options

Figure 27.3. Coronal oblique CT scan; sacral kyphoplasty post cement delivery. From personal files of Rinoo V. Shah, MD, MBA.

References 1.

206

Frey ME, Depalma MJ, Cifu DX, et al. Percutaneous sacroplasty for osteoporotic sacral insufficiency fractures: a prospective,

Figure 27.4. Coronal CT scan; sacral kyphoplasty post cement delivery. From personal files of Rinoo V. Shah, MD, MBA.

12(2):113–120. doi: 10.1016/j. spinee.2012.01.019.

multicenter, observational pilot study. Spine J. 2008;8(2):367–373. 2.

Shah RV. Sacral kyphoplasty for the treatment of painful sacral insufficiency fractures and metastases. Spine J. 2012;

3.

Depalma MJ, Ketchum JM, Saullo T. What is the source of chronic low back pain and does age play a role? Pain Med. 2011;12:224–233.

Section 2 Chapter

28

Spinal Disorders

Skeletal metastases and treatment options Rinoo V. Shah

Case study A 58-year-old man underwent a right lower lobectomy, for squamous cell carcinoma. He received adjuvant chemotherapy, but could not complete the entire course due to side effects. Nonetheless, he remained disease free as evidenced by negative postoperative CT scans. During a routine referral for smoking cessation and obstructive airway disease - the patient continued to smoke postoperatively for almost 2 years - the pulmonologist advised bronchoscopy. Cancer recurrence was identified at the bronchial stump. PET scanning demonstrated multicentric disease and the patient was not a candidate for pneumonectomy. Over the next few months, the patient was frequently hospitalized for pulmonary and pain complaints. One painful area was localized over the anterior sternum.

1. What is the differential diagnosis? a. b. c. d. e. f. g.

Costochondritis or Tietze’s syndrome Myofascial pain Pleuritis Visceral pain (angina) Mediastinis Aortic dissection Metastasis

A sternal CT demonstrates a circumscribed osteolytic and cavitating lesion.

2. What is this? This is a skeletal metastasis. Bone lesions are common in advanced cancer, particularly lung, prostate, breast, and multiple myeloma. Metastatic non-small cell lung cancer is the leading cause of cancer death among men and women in the USA and worldwide. Among

patients with metastatic breast and prostate cancer, 65–75% will have bone metastases. Bone metastases in breast cancer have a high predilection for causing skeletal-related events (SRE): pathologic fractures, spinal cord compression, severe bone pain, structural impairment requiring prophylactic stabilization, and hypercalcemia. Life expectancy in patients with metastatic bone disease is significantly shortened. Patients with metastatic non-small cell lung cancer have a median survival of 8.9 months and with palliative care, this increases to 11.6 months. Breast cancer patients with bone metastases have a survival of about 2 years, but are at risk for SREs. Typically, survival is less than 1 year for metastatic lung cancer and a few years for metastatic breast and prostate cancer. Quality of life is poor, however, due to the risk of 3–4 SREs per year.[1–6]

3. How do you diagnose this? Typically, the history suggests an acute presentation. Patients may have other systemic signs, such as weight loss, cachexia, progressive debilitation, and fevers. The metastasis may cause additional SREs. Patients may report neurologic loss, weakness, and non-focal pain that radiates to the limbs. Other non-skeletal metastases may cause visceral/organ dysfunction or cognitive impairment. Patients may have significant ambulatory and functional limitations. Sometimes, they are confined to bed. Pain worsens with weight bearing, but only slightly improves with rest. Patients may have night pain and poor sleep. There may be tenderness to palpation over the bony prominences, e.g., spine, hips, knees, shoulders, arms, and skull. Laboratory studies are usually not helpful. The alkaline phosphatase and calcium may be elevated. Radiographs may demonstrate osteolytic lesions, but

Case Studies in Pain Management, ed. Alan David Kaye and Rinoo V. Shah. Published by Cambridge University Press. © Cambridge University Press 2015.

207

Chapter 28: Skeletal metastases and treatment options

may be missed in patients with osteoporosis. Advanced imaging studies are important in diagnosing skeletal metastases. Bone scintigraphy with technetium 99 is very sensitive for acute osteolytic lesions but may be confused with fractures. A sacral metastases for instance may not present with the classic “Honda” sign since lesions could be unilateral. Magnetic resonance imaging provides detailed anatomic and physiologic information. This test has high diagnostic accuracy. Lesions appear high on T2-weighting and high on STIR (fat suppression) sequences, due to bone edema. Due to tumor vascularity, contrast preferentially enhances tumor and advanced imaging, such as an MRI or CT, should be ordered with this modality, as long as there are no contraindications such as renal insufficiency or allergies. The CT scan may be used in patients with contraindications to MRI, e.g., pacemaker implants. The CT scan is also useful for percutaneous cement augmentation procedures, such as sacral kyphoplasty (Figure 28.1). Bone detail is excellent and multiplanar reformatting enables 3-dimensional rendering of the metastasis.[1,2]

4. Why is treatment or palliation of this lesion important? Early palliative care of skeletal metastases, within 8 weeks of diagnosis, will lead to improvements in quality of life and mood.[1–6]

5. The patient presented with skeletal pain shortly after being diagnosed with recurrent bronchial stump cancer. Why is this? Early stage metastatic bone disease may remain undetected. Patients often present with pain with latestage metastases. Skeletal-related events (SREs), such as spinal cord compression or vertebral fractures, are devastating.[3] Metastases alone, however, can be painful and disabling due to direct invasion and osteolysis. This patient’s pain was due to osteolysis.[1–6]

6. How would you treat this pain? Multimodal care is imperative. This includes analgesics, bracing (for support), protected weight bearing if the metastasis involves the lower limbs, activity modification balanced with mobilization, and counseling given

208

the prevalence of fear and apprehension. Antiresorptive agents may be necessary given the risk of hypercalcemia. Polymodal analgesic care may include nonsteroidal anti-inflammatory agents, muscle relaxants, antiepileptic drugs, anxiolytics, and opioids. Fear, anxiety, and apprehension are common in this population. Not only is fear of pain paramount, but so is apprehension about additional procedures and mobility given the severity of pain. An extensive discussion of these approaches is covered in Chapters 48, 49, and 50.

7. Is there a role for targeted therapy in this population? These lesions are very focal, painful, and disabling. The doses of opioids necessary may be very high; these doses carry attendant risks of opioid hyperalgesia (Chapter 29) and respiratory sedation. Furthermore, at the end of life, comfort afforded by high-dose opioids may impair cognition, alertness, and sleep-wake cycles. This interferes with the patient’s ability to coordinate end of life plans and communication with family members. So, targeted palliative care is an important consideration and should be broached with the oncologist, oncology team, and palliative care team. These specialists may have the perception that treatment of these lesions is invasive, disruptive to patient care, and expensive. So, targeted therapy plays a role in this population.

8. What types of targeted therapy for skeletal metastases are available? Historically, radiation therapy and open surgical excision were the most common treatments. Open surgical approaches or radiotherapy carry high morbidity. Radiotherapy may damage soft tissues around the tumor bed. Surgery carries intraoperative and perioperative risks. Increasingly, percutaneous ablative procedures are used to target painful soft tissue tumors. These techniques are being used, with increasing frequency, in primary and metastatic bone tumors.[4] These procedures offer a lower risk of morbidity compared to radiotherapy and surgery. These methods utilize imaging and specialized access devices. Tumor destruction is afforded by chemical agents (ethyl alcohol or acetic acid) or thermal energy (laser, microwave, ultrasound, cryotherapy, radiofrequency).[4] Radiofrequency ablation produces a discrete thermal lesion and has been efficacious in painful skeletal metastases.[5] Bipolar

Chapter 28: Skeletal metastases and treatment options

A

Figure 28.1. (A) CT reconstruction, focal lytic sternal lesion. (B) Sternal kyphoplasty, cannula and balloon displacing anterior and posterior cortical walls of sternum note cavitation. (C) PMMA cement in sternal metastasis; some anterior extravasation, but none posterior into mediastinum. From personal files of Rinoo V. Shah, MD, MBA.

B

C

209

Chapter 28: Skeletal metastases and treatment options

radiofrequency or monopolar radiofrequency lesioning affords tumor necrosis and involution to improve focal pain. Discrete vertebral column metastases have been successfully treated with percutaneous spine stabilization.[6] Pain relief following vertebroplasty and kyphoplasty, in pathologic and non-pathologic vertebral fractures, has been attributed to spinal stabilization.[6,7] Radiofrequency ablation, osteoplasty, and kyphoplasty are relatively safe, target specific, and efficacious. Most can be conducted with monitored anesthetic care or conscious sedation. In those patients requiring general anesthesia, the procedure time is short, blood loss is minimal, and surgical stimulation is mild. Preoperative planning is essential. Medical clearance is advised. Patients are placed in a prone position with attention to eye, peripheral joint, and skin protection. Preoperative antibiotics should be administered within 30–60 minutes of the incision. Review the CT scan preoperatively. Imaging software permits making distance measurements from the skin to bone metastasis and trajectory planning. A transpedicular or extrapedicular approach is used for spinal metastases in the thoracic and lumbar spine. An anterolateral approach to the vertebral body is used for cervical spine tumors. A short axis or modified short axis approach is advised for sacral metastases. Other flat bones may be targeted under fluoroscopic or CT guidance. A recent novel approach using fluoroscopy and sonography, coupled with pre-op CT imaging software was used to treat a sternal metastasis. Under fluoroscopy, a styletted cannula is advanced using the aforementioned approaches into the metastasis. For kyphoplasty or radiofrequency ablation of thoracolumbar metastases, the technique is modified. The cannula is advanced to the posterior third of the vertebral body. The stylet is removed from the cannula and replaced with a curette. This curette advances past the tip of the cannula toward the anterior vertebral body wall. A bone biopsy may then be obtained. Fluoroscopy is used to confirm placement. The curette is removed and replaced by a balloon. The balloon is inflated and the margins should be within the confines of the vertebral body. The pressure rating is typically less than 250 pounds per square inch. Some vendors of kyphoplasty equipment have developed higher balloon pressure ratings for very dense, compact, and sclerotic bone tumors. Bone density or tumor density will affect balloon pressure and volume. In the case of vertebroplasty,

210

the cannula is advanced to the anterior third and no balloon is used. Adjunctive equipment is available to create a cavity, if the balloon doesn’t suffice. Polymethylmethacrylate (PMMA) is mixed with barium powder (for radiopacity). High viscosity PMMA with a thickened consistency is delivered in small aliquots under low pressure. Fluoroscopy is imperative to ensure no leak occurs. PMMA may sometimes be obscured by the cannulas and due to patient habitus. Working time is 10–20 minutes allowing one enough time for slow controlled delivery. In the case of spinal tumors, cement may leak unpredictably and rapidly. For instance, a midline and anterior cannula may access the vertebrobasilar venous system. The veins are low resistance as compared to bone or tumor. PMMA could then rapidly extravasate into the epidural space. Another issue is target specificity. Bone tumors may be well circumscribed and careful placement under fluoroscopy is necessary: in some cases, CT guidance may be necessary. Older versions of PMMA had to be delivered in a liquid form – which increased the risk of extravasation and leakage: working times were shorter and PMMA was not deliverable once hardened (see Chapter 25 for additional details). Current versions of PMMA have a thicker consistency and longer working times, which enhances safety and allows more complete fills. PMMA is preferable to newer biointegrative cements in metastatic disease; adjacent segment fractures are less of a concern as compared to effective palliation of metastases. PMMA causes tumor necrosis and neurolysis via an exothermic reaction, while providing structural stabilization. Polyetheretherketone (PEEK) wafers may provide additional fracture reduction/tumor stabilization while minimizing cement volume requirements (Figure 28.2). In patients with complex metastases that pose a high risk/contraindication to vertebroplasty/ kyphoplasty, e.g., retropulsion of the vertebral body, radiofrequency thermocoagulation may be considered. A bipolar radiofrequency needle connected to a radiofrequency generator, while cooled by continuous saline irrigation, is a recent advance for this patient population. Saline irrigation permits needle tip temperature stabilization while augmenting lesion size. Radiofrequency ablation technology has been discussed in Chapters 15, 16, and 26. In essence, a high-frequency electrical current is sent to the uninsulated portion of the needle. This causes oscillation of surrounding molecules and places them at a higher energy state.

Chapter 28: Skeletal metastases and treatment options

A B

C

D

Figure 28.2. (A–D) Peek wafer lumbar kyphoplasty (Staxx FX Kyphoplasty Spine Wave, Shelton, Connecticut); sequential placement of PEEK wafers (A, B, C) and PMMA instillation (D). From personal files of Rinoo V. Shah, MD, MBA.

As these molecules return to their baseline energy state, heat is generated. RF energy causes protein denaturation in a very circumscribed spheroid configuration allowing it to be very target specific. The bone metastasis and surrounding edema have lower impedance compared to surrounding intact bone. Ablation preferentially occurs within the metastasis as opposed to bone. This is opposed to electrocautery or an open flame which can char intact bone in an uncontrolled manner. Bipolar versus monopolar refer to the location of the electric sink or ground. In bipolar RF, the grounding pad/electrical sink is contained within the

needle itself and is proximal to the active tip. Lesion size is larger in an axial/radial dimension as compared to monopolar lesions. Continuous saline irrigation further enables a larger radial lesion; it promotes temperature stabilization and a rapid temperature drop off to protect surrounding tissues. Patients report rapid pain relief within hours of the procedure. Recently, clinical trials have demonstrated the success of combining kyphoplasty with radiopharmaceutical agents: 125-I seeds and samarium-153. These target-specific options are being explored as options to external beam radiotherapy and intravenous radiopharmaceutical agents.[1–2,7–25]

211

Chapter 28: Skeletal metastases and treatment options

9. What is the mechanism of rapid pain relief? Cementoplasty of metastases in non-weight-bearing bones suggests a mechanism of pain relief, unrelated to bone stabilization.[8] Polymethylmethacrylate cement causes an exothermic reaction, with curing and solidification. Temperatures ranging from 50 to 57°C have been reported at the bone-cement interface, during polymerization. Average peak temperatures ranging from 45 to 100°C, depending on the cement, have been reported.[9] Neurolysis occurs at 45°C;[10] these temperatures result in destruction of tumor cells and vascular supply.[8] So, pain relief may occur due to several mechanisms: (1) bone stabilization; (2) direct tissue toxicity; (3) neurolysis; and (4) thermal injury. There have been several reports of successful cementoplasty of skeletal metastases.[1–2,7–25]

10. Are there any complications to worry about following vertebroplasty or kyphoplasty? The major risks are infection, bleeding, no pain relief, or nerve damage. A cement leak into the spinal canal or neuroforamina can cause permanent neurologic damage; a cement leak into the soft tissues is often asymptomatic. Sometimes, the cement hardens and

References 1.

Shah RV. Sacral kyphoplasty for the treatment of painful sacral insufficiency fractures and metastases. Spine J. 2012;12(2): 113–120. doi: 10.1016/j. spinee.2012.01.019.

2.

Shah RV. Sternal kyphoplasty for metastatic lung cancer: imageguided palliative care, utilizing fluoroscopy and sonography. Pain Med. 2012;13(2):198–203. doi: 10.1111/j.1526-4637.2011.01299.x. Epub 2012 Jan 13. PubMed PMID: 22239702.

3.

Jemal A, Siegel R, Xu J, Ward E. Cancer statistics, 2010. CA Cancer J Clin. 2010;60(5):277–300. Epub 2010 Jul 7. PubMed PMID: 20610543.

212

causes an exothermic reaction that leads to soft tissue necrosis and severe pain. However, leaks into the pleural cavity or vascular supply can be catastrophic. Cement emboli into the lungs may cause rapid patient demise. The cannulas similarly could violate the cortical bone and cause nerve or soft tissue damage.[1,2]

11. What can one conclude? These patients require multimodal therapy. Unfortunately, focal tumor metastases are difficult to treat with systemic methods. Often high doses of opioids are required with associated complications of sedation, respiratory depression, and opioid hyperalgesia. Patients may become nauseated, constipated, or pruritic, which further complicates management. Radiotherapy and surgery may lead to morbidity. Focal treatment with kyphoplasty, radiofrequency ablation, and targeted radiopharmaceuticals promote the principles of tumor debulking and necrosis for pain control. They enhance safety by minimizing collateral tissue damage. Shah et al demonstrated effective palliation of a non-weightbearing bone (sternum - metastasis) with kyphoplasty.[2] There is an ethical dilemma in extrapolating observational and retrospective studies to clinical practice. Practitioners must exercise due diligence, sound judgment, and provide informed consent, while cognizant of the desperate nature of spinal metastatic disease.

4.

Temel JS, Greer JA, Muzikansky A, et al. Early palliative care for patients with metastatic nonsmall-cell lung cancer. N Engl J Med. 2010;363(8):733–742.

5.

Major PP, Cook RJ, Lipton A, et al. Natural history of malignant bone disease in breast cancer and the use of cumulative mean functions to measure skeletal morbidity. BMC Cancer. 2009;9:272. doi: 10.1186/14712407-9-272. PubMed PMID: 19660124; PubMed Central PMCID: PMC2739221.

6.

Hirsh V. Skeletal disease contributes substantially to morbidity and mortality in patients with lung cancer. Clin Lung Cancer. 2009;10(4):223–229 [Review].

7.

Kurup AN, Callstrom MR. Ablation of skeletal metastases: current status. J Vasc Interv Radiol. 2010;21(8 Suppl): S242–250. 8. Thanos L, Mylona S, Galani P, et al. Radiofrequency ablation of osseous metastases for the palliation of pain. Skeletal Radiol. 2008;37(3):189–194. Epub 2007 Nov 21. 9. Yang Z, Yang D, Xie L, et al. Treatment of metastatic spinal tumors by percutaneous vertebroplasty versus percutaneous vertebroplasty combined with interstitial implantation of 125I seeds. Acta Radiol. 2009;50(10):1142–1148. 10. Dalbayrak S, Onen MR, Yilmaz M, Naderi S. Clinical and

Chapter 28: Skeletal metastases and treatment options

radiographic results of balloon kyphoplasty for treatment of vertebral body metastases and multiple myelomas. J Clin Neurosci. 2010 Feb;17(2):219–224. 11. Choi HR, Lee PB, Kim KH. Scapuloplasty alleviates scapular pain resulting from lung cancer metastasis. Pain Physician. 2010;13(5):485–491. 12. Anselmetti GC, Manca A, Kanika K, et al. Temperature measurement during polymerization of bone cement in percutaneous vertebroplasty: an in vivo study in humans. Cardiovasc Intervent Radiol. 2009;32(3):491– 498. 13. Shah RV, Lutz GE, Lee J, Doty SB, Rodeo S. Intradiskal electrothermal therapy: a preliminary histologic study. Arch Phys Med Rehabil. 2001;82(9): 1230–1237. 14. Zhou B, Wu CG, Li MH, Gu YF, Cheng YD. Percutaneous osteoplasty for painful sternal lesion from multiple myeloma. Skeletal Radiol. 2009;38(3): 281–285. 15. Masala S, Manenti G, Roselli M, et al. Percutaneous combined therapy for painful sternal metastases: a radiofrequency thermal ablation (RFTA) and cementoplasty protocol.

Anticancer Res. 2007;27(6C): 4259–4262. 16. Gianfelice D, Gupta C, Kucharczyk W, et al. Palliative treatment of painful bone metastases with MR imaging– guided focused ultrasound. Radiology. 2008;249(1):355–363. 17. Tang D, Peng EW, Giri D, Chowdhary M, Sarkar P. Mediastinal irradiation and its effect on the cardiovascular system. Br J Hosp Med (Lond). 2009;70(4):222–224. 18. Adams MJ, Hardenbergh PH, Constine LS, Lipshultz SE. Radiation-associated cardiovascular disease. Crit Rev Oncol Hematol. 2003;45(1):55–75. 19. Dalbayrak S, Onen MR, Yilmaz M, Naderi S. Clinical and radiographic results of balloon kyphoplasty for treatment of vertebral body metastases and multiple myelomas. J Clin Neurosci. 2010;17(2): 219–224. 20. Mendel E, Bourekas E, Gerszten P, Golan JD. Percutaneous techniques in the treatment of spine tumors: what are the diagnostic and therapeutic indications and outcomes. Spine (Phila Pa 1976). 2009;34(22 Suppl):S93–100.

21. Belfiore G, Tedeschi E, Ronza FM, et al. Radiofrequency ablation of bone metastases induces longlasting palliation in patients with untreatable cancer. Singapore Med J. 2008;49(7):565–570. 22. Lane MD, Le HB, Lee S, et al. Combination radiofrequency ablation and cementoplasty for palliative treatment of painful neoplastic bone metastasis: experience with 53 treated lesions in 36 patients. Skeletal Radiol. 2011;40(1):25–32. 23. Amdur RJ, Bennett J, Olivier K, et al. A prospective, phase II study demonstrating the potential value and limitation of radiosurgery for spine metastases. Am J Clin Oncol. 2009;32(5):515–520. 24. Cardoso ER, Ashamalla H, Weng L, et al. Percutaneous tumor curettage and interstitial delivery of samarium-153 coupled with kyphoplasty for treatment of vertebral metastases. J Neurosurg Spine. 2009;10(4):336–342. 25. Yang Z, Yang D, Xie L, et al. Treatment of metastatic spinal tumors by percutaneous vertebroplasty versus percutaneous vertebroplasty combined with interstitial implantation of 125I seeds. Acta Radiol. 2009;50(10):1142–1148.

213

Section 2 Chapter

29

Spinal Disorders

Fibromyalgia and opioid-induced hyperalgesia Grace Chen and Elliot Palmer

Case study A 45-year-old woman was referred to a pain management center from her primary care doctor’s office for evaluation and treatment of her fibromyalgia. Per history, patient complains of worsening fatigue, headache, and widespread and migrating pain all over her body. She relates that the pain is most bothersome in her shoulders and lower back but is also present in her thoracic spine, neck, right elbow, and bilaterally in her knees. She remembers having diffuse body pain since age 25 when she got into a motor vehicle accident in which her car was wrecked even though she did not have any fractures or hospitalization from that accident. On further evaluation, she has also been dealing with depression and insomnia. She does not think that she has fibromyalgia and she would like to have a different diagnosis and be cured of her pain. She also would like to increase her opioid dosage as her pain has worsened since her son had gone to college.

1. What is the differential diagnosis? Some differential diagnoses for widespread body pain include: a. Fibromyalgia b. Polymyalgia rheumatica c. Myositis/myopathies d. Myofascial pain syndrome e. Rheumatoid arthritis f. Systemic lupus erythematosis g. Sjogren’s syndrome h. Ankylosing spondylitis i. Hypothyroidism j. Somatoform disorder

k. Cervical spinal stenosis l. Systemic vasculitis

2. What is fibromyalgia? How do you diagnose it? Are there diagnostic and/or clinical criteria? According to the 2010 American College of Rheumatology preliminary revised diagnostic criteria, fibromyalgia is characterized by widespread body pain, cognitive symptoms, unrefreshed sleep, fatigue, and a number of somatic symptoms.[1] These relatively new diagnostic criteria mark a change from the 1990 diagnostic criteria espoused by the ACR that included physical exam findings of 11/18 manual tender points. For 20 years, practitioners used American College of Rheumatology’s “Manual Tender Point Survey” along with history and physical exam as mainstays in the diagnosis of fibromyalgia (Figure 29.1). This survey involves palpation of 18 specified muscle insertion sites throughout the body with approximately 10 pounds of weight and a simple “yes” or “no” answer from the patient regarding the presence or absence of tenderness. There are three control points built in to the survey, and the diagnosis requires that 11/18 noncontrol points be scored as positive.[2] However, this survey was originally developed as a research tool and was not intended for use in clinical practice. Additionally, many clinicians thought that the criteria were too restrictive and failed to account for the heterogeneity in the population thought to have fibromyalgia. Born from this and other factors, the ACR has more recently published a set of not yet validated preliminary diagnostic criteria for fibromyalgia that does not include the tender point exam. These

Case Studies in Pain Management, ed. Alan David Kaye and Rinoo V. Shah. Published by Cambridge University Press. © Cambridge University Press 2015.

214

Chapter 29: Fibromyalgia and opioid-induced hyperalgesia

Table 29.1 2010 ACR fibromyalgia diagnostic criteria

A patient satisfies diagnostic criteria for fibromyalgia if the following three conditions are met: 1. Widespread Pain Index (WPI)  7 and Symptom Severity (SS) Scale Score  5 OR WPI 3–6 and SS  9 2. Symptoms have been present at a similar level for at least 3 months 3. The patient does not have a disorder that would otherwise explain the pain WPI: Answer the question, “in how many areas has the patient had pain over the last week?” The score will be from 0–19. Areas: 1. Shoulder girdle, left 2. Shoulder girdle, right 3. Upper arm, left 4. Upper arm, right 5. Lower arm, left 6. Lower arm, right 7. Hip (buttock/trochanter), right 8. Hip (buttock/trochanter), left 9. Upper leg, left 10. Upper leg, right 11. Lower leg, left 12. Lower leg, right 13. Jaw, left 14. Jaw, right 15. Chest 16. Abdomen 17. Upper back 18. Lower back 19. Neck SS Scale Score: The sum of the severity of three specified symptoms + the severity of somatic symptoms in general 1. For each of the 3 (i, ii, iii) symptoms below, indicate the level of severity over the past week (0 ¼ no problem, 1 ¼ slight or mild problems/intermittent, 2 ¼ moderate problems/often present, 3 ¼ severe, pervasive, continuous problems): i. Fatigue ii. Waking unrefreshed iii. Cognitive symptoms 2. Considering somatic symptoms in general*, indicate whether the patient has: 0 ¼ no symptoms 1 ¼ few symptoms 2 ¼ a moderate number of symptoms 3 ¼ a great deal of symptoms * Somatic symptoms might include: muscle pain, irritable bowel syndrome (IBS), fatigue, cognitive problems, weakness, headache, abdominal pain/cramps, numbness, tingling, dizziness, insomnia, depression, constipation, chest pain, blurred vision, dry mouth, itching, tinnitus, nausea, vomiting, etc.

1 3

2

5

10

4 6

12

7

11 13

14 9

15

8 16 17

19

18

20

21

Figure 29.1 Manual Tender Point Survey.

criteria are proposed as a parallel method of diagnosis and represent the most current thinking (Table 29.1). The foremost feature of fibromyalgia is chronic, widespread pain that is not explained by an injury or another rheumatic or systemic disorder. There are a number of other salient features, aside from pain, that are present in many patients with fibromyalgia – common among them are fatigue, mood disturbance, and insomnia. There is a strong association between fibromyalgia and other autoimmune disorders. Also, it is clear that many patients with fibromyalgia demonstrate similar neuroendocrine changes, patterns of central nervous system response to pain, and even distinct features on functional neuroimaging. Fibromyalgia was far more common in 1st degree relatives of others with the diagnosis than in matched controls with rheumatoid arthritis.[3] It also commonly coexists with other systemic disorders like temporomandibular joint disorders, headaches, and irritable bowel syndrome.[4]

215

Chapter 29: Fibromyalgia and opioid-induced hyperalgesia

Stressors from the environment also seem to play a role. The development of fibromyalgia has been associated with certain infections[5] as well as with physical abuse and trauma.[6] Furthermore, patients with fibromyalgia show alterations in CSF levels of endogenous opioids,[7] norepinephrine, dopamine, and serotonin[8] and show abnormal patterns of cerebral blood flow and neural activity when compared to controls.[9,10] Fibromyalgia was once thought of as an unknown entity or a label for patients without any identifiable cause for their pain. However, it is now clear that fybromyalgia is a complex condition with multiple objective identifying features. Patients present clinically with signs of central sensitization such as allodynia, hyperpathia, and hyperalgesia. One clinical hypothesis for fibromyalgia pathophysiology is a combination of spinal cord amplification of pain that may be triggered by multiple causes, combined with autonomic arousal such as the fight-or-flight response, leading to loss of slow-wave sleep. Autonomic arousal can be due to psychiatric causes such as post-traumatic stress, anxiety, or even to sleep apnea and/or cervical cord impingement. Loss of slow-wave sleep correlates with the loss of pain inhibition. Peripheral pain generators, such as conditions like osteoarthritis, are amplified in the dorsal horn of the spinal cord. Pain signals travel to the brain, causing another set of disturbances, including neuro-hormonal dysfunction, especially of the hypothalamic–pituitary– adrenal axis that adversely affect cortisol and growth hormone release. Fibromyalgia patients have blunted morning cortisol release, contributing to their intense morning fatigue. Growth hormone release is poor in these patients and is needed for a sense of well-being and for repair of muscle microtrauma. This may explain the post-exertion pain and fatigue seen in these patients. It is also clear that dopaminergic neurotransmission is impaired to areas of the brain related to pain inhibition. Restless leg syndrome, a manifestation of dopamine depletion, can occur in severe fibromyalgia. It reflects poor dopamine activity in the somatosensory cortex. It also disrupts sleep. Small hippocampi are seen in patients with posttraumatic stress disorder (PTSD) and severe depression. The hippocampus is needed for short-term memory storage and moderates autonomic arousal. Mood issues, such as anxiety, depression, and cognitive dysfunction, are often comorbidities with fibromyalgia and share neurotransmitter pathways in the brain, possibly contributing to the “fibrofog,” a complaint

216

voiced by many patients. Poor cognition in fibromyalgia patients may be aggravated by compromised BDNF (brain derived neurotrophic factor) activity which is needed to make new synapses. Interestingly, aerobic exercise increases BDNF and may improve cognition. Changes in corticotrophin-releasing factor (CRF) receptor sensitivity also increase anxiety and high CRF activity causes hippocampal atrophy. Thus, fibromyalgia is a constellation of pain, fatigue, and poor sleep. Fibromyalgia may have a genetic predisposition and may be triggered by life events. The use of manual tender points in the previous diagnosis of fibromyalgia highlights one of the most prominent features of this condition. In addition to widespread pain, patients with fibromyalgia are very sensitive to painful stimuli. They also demonstrate both allodynia and hyperalgesia. These aspects are thought to be due to neurochemical changes as well as increased wind-up and diminished descending inhibitory pathways. Patients with fibromyalgia also have a high incidence of sleep disorders[11] and psychiatric illness.[12] However fibromyalgia is defined, it is clear that it is a relatively common disorder with clear ramifications for the quality of a patient’s life. The estimated prevalence of fibromyalgia is 2–4% of the population, thus affecting over 6 million people in the USA alone, with a clear female predominance.[13–15] In addition, fibromyalgia imposes significant financial burdens on both the patients and medical system, and these increase with the severity of the disease.[16]

3. How do you differentially diagnose fibromyalgia from similar problems? According to the 2010 preliminary criteria, fibromyalgia is a diagnosis of exclusion. Thus, all other diagnoses causing widespread pain and fatigue must be excluded before giving the diagnosis of fibromyalgia. According to the 1990 criteria, fibromyalgia diagnosis is an independent one that may coexist with other diagnoses caused by similar symptoms, as long as the patient has chronic pain and more than 11 of 18 manual tender points. The following are some features of other rheumatologic diagnoses that may also cause widespread pain and fatigue.  Polymyalgia rheumatica (PMR)

:

Pain and fatigue are common to both PMR and fibromyalgia; however, the pain in PMR is

Chapter 29: Fibromyalgia and opioid-induced hyperalgesia

typically limited to the neck, shoulders, and hips whereas the pain in fibromyalgia is classically described as affecting all four quadrants of the body (left and right sides, above and below the waist). Additionally, PMR typically affects adults over the age of 60 whereas fibromyalgia is most commonly seen between the ages of 20 and 50.  Myositis/myopathies

:

Myositis, a general term for inflammation of muscles, encompasses a wide variety of conditions including myositis ossificans, dermatomyositis, polymyositis, pyomyositis, and drug-induced (i.e., statins) myositis, among others. Myopathy, or muscle disease, is a similarly broad term with many variations. It is beyond the scope of this chapter to delineate these, but the etiologies range from inflammatory to infectious to metabolic, and diagnostic criteria vary widely as well. Fibromyalgia has been described as a disorder of “central sensitization,” whereas a myositis or myopathy is most often a clearly peripheral phenomenon.  Myofascial pain syndrome (MPS)

:

Myofascial pain syndrome and fibromyalgia share some features and may even coexist, but they are often distinguishable. Myofascial pain involves irritable foci of pain termed “trigger points.” The pain is often described as a deep aching sensation, and muscle stiffness is typically present.[17]

Additionally, trigger points in MPS are most commonly found in the erector spinae, gluteal fascia, and presacral fascia as opposed to the generalized and widespread tenderness seen in the majority of fibromyalgia patients.  Somatoform disorders

:

There are different varieties of somatoform disorders, though the one commonly confused with fibromyalgia is somatization disorder. The DSM IV-TR defines somatization disorder as a history of many physical complaints beginning before age 30 that occur over several years and result in treatment being sought or significant functional impairment. Each of the following criteria must be met, with individual symptoms

occurring at any time during the course of the disturbance: – 4 pain symptoms: a history of pain related to at least four different sites or functions; – 2 gastrointestinal symptoms: a history of at least two gastrointestinal symptoms other than pain; – 1 sexual symptom: a history of at least one sexual or reproductive symptom other than pain; – 1 pseudoneurologic symptom: a history of at least one symptom or deficit that suggests a neurologic condition not limited to pain. Additionally, either each of the symptoms cannot be fully explained by a known general medical condition or the direct effects of a substance after appropriate investigation, or when there is a related general medical condition, the physical complaints or resulting social or occupational impairment are in excess of what would be expected from the history, physical examination, or laboratory findings. Importantly, the symptoms in this disorder are not intentionally produced or feigned.[18]

4. What is the treatment for fibromyalgia? Is it ethical to prescribe opioids in the treatment of fibromyalgia? Pharmacologic treatment is the primary approach to management for the majority of patients. However, most physicians recognize that patients with fibromyalgia benefit from a multidisciplinary approach including physical therapy, CBT, and pain psychology.[19] Many medications have been used to treat fibromyalgia with varying degrees of success. For a long time, none of the commonly used medications were recognized or approved. However, in 2007 Lyrica (pregabalin) became the first FDA-approved drug for specifically treating fibromyalgia. One year later, Cymbalta (duloxetine hydrochloride) became the second approved drug. In 2009, the FDA approved a third drug, milnacipran. Several classes of drugs have been used to treat fibromyalgia: tricyclic antidepressants (TCAs), SSRIs, selective serotonin/norepinephrine reuptake inhibitors (SNRIs), monoamine oxidase

217

Chapter 29: Fibromyalgia and opioid-induced hyperalgesia

inhibitors (MAOI), 5-HT3 receptor antagonists, anticonvulsive or antiseizure medications (AEDs), muscle relaxants, NMDA receptor antagonists, dopamine agonists, NSAIDs, and opioids.[20] There are altered levels of norepinephrine and serotonin in fibromyalgia. Drugs that augment these neurotransmitter levels within the central nervous system (TCAs, SNRIs, tramadol) are thus useful in the treatment of fibromyalgia.  Tricyclic antidepressants

:

These are the most well-studied medications for fibromyalgia, in part because they have been around the longest. These medications work by blocking reuptake and thus increasing the concentrations of serotonin and norepinephrine. They appear to effectively improve sleep, stiffness, and tenderness associated with fibromyalgia.[21]  Selective serotonin reuptake inhibitors

:

SSRIs have also been used in the treatment of fibromyalgia, largely due to their superior side effect profile when compared to TCAs. Trials of efficacy have had conflicting results, but in one trial, when compared head-to-head, SSRIs were found to be effective but not quite as efficacious as TCAs.[22]  Selective serotonin and norepinephrine reuptake inhibitors

:

Fibromyalgia, similar to other chronic pain states, seems to respond better to serotonergic–noradrenergic antidepressants than to purely serotonergic antidepressants.[23] The similarity of these drugs to TCAs makes their efficacy not surprising. As mentioned above, both duloxetine and milnacipran have now been FDA approved for the treatment of fibromyalgia.  Other drugs

:

218

Pregabalin is an anticonvulsant drug also used for chronic pain, designed as a more potent successor of gabapentin. Pregabalin works by binding to the alpha-2 delta subunit of voltage gated calcium channels in the central nervous system and reducing their activity. Thus, it decreases neurotransmitter release, such as glutamate, norepinephrine, and substance P. In a randomized controlled trial, beneficial effects on pain, sleep disturbance, and fatigue

were shown.[24] Until recently, pregabalin was the only pharmacologic agent approved for the treatment of fibromyalgia. – Gabapentin has similar pharmacology to pregabalin. It is commonly used in the treatment of other neuropathic pain states and demonstrates efficacy in fibromyalgia,[25] though is not a commonly selected medication. – Sedative hypnotics including benzodiazepines and zolpidem are widely used for fibromyalgia and may be of benefit in improving sleep and reducing fatigue, though these medications are of limited benefit in other arenas and carry with them both side effects and the potential for abuse. – Opioids have been widely used for the treatment of fibromyalgia, despite a lack of evidence for their efficacy. Opioid use in the treatment of fibromyalgia is a particularly contentious topic and deserves further discussion. Appropriately, their use is becoming more and more limited due to concerns about addictive potential, opioid-induced hyperalgesia, and lack of efficacy. Many guidelines have been published delineating the clear lack of efficacy of opioids, though their widespread use continues.[26] Interestingly, patients have largely reported the most effective medications are those that had no evidence of efficacy, not to mention a high potential for abuse – hydrocodone, alprazolam, oxycodone, zolpidem, and clonazepam.[27] This clearly provides a dilemma for physicians. It is most likely that the perceived efficacy of these medications reflects interaction of these medications with the pleasure and reward systems rather than relief of the pain or other symptoms of fibromyalgia. Prescribing physicians should review the evidence and keep in mind the many deleterious effects and possible harm associated with chronic opioid therapy when deciding on treatment options. Non-pharmacologic treatments include CBT, exercise, physical therapy, sleep hygiene, and a wide variety of other CAM treatments. Among these, the best evidence exists for CBT and exercise.[28,29] Though high-quality evidence does not exist to support their use, many alternative therapies including

Chapter 29: Fibromyalgia and opioid-induced hyperalgesia

acupuncture, chiropractic care, and massage are commonly used by patients suffering from fibromyalgia, and anecdotal evidence suggests further, controlled studies may be indicated.

Case follow-up After a few office visits, you give the patient a diagnosis of fibromyalgia and recommend initial treatment with a combination of pregabalin, pain psychology, and light exercise. Your patient tells you that this treatment would take too much time and effort, and she seeks a second opinion. One year later she returns to you with similar complaints, but worsening of her pain. In the interim she has been taking opioids prescribed to her by another physician.

5. Does the worsening of her pain despite opioid therapy make you question your diagnosis? Are there any other diagnoses to consider at this point? Patients with fibromyalgia have altered opioid response. Harris et al observed that patients with fibromyalgia have decreased mu-opioid receptor availability.[30] Fibromyalgia patients were found to have more met-enkephalin-Arg6-Phe7 (MEAP) in their cerebrospinal fluid and concomitant lower pain threshold compared to chronic low back pain groups.[31] In a study of more than 8000 patients with chronic non-cancer pain, 7% of whom had fibromyalgia, potent opioids are more effective in pain control and functional improvement compared to non-

References 1.

2.

3.

opioids.[32] Clinically, many fibromyalgia patients do indeed find benefit with opioids. However, escalating doses of opioids has multiple side effects and complications. In this case, her complaint of worsening pain should prompt further investigation. One entity that should be specifically considered in this case is opioidinduced hyperalgesia (OIH). Opioid-induced hyperalgesia is defined as a state of nociceptive sensitization caused by exposure to opioids.[33] This causes a paradoxical response in which a patient receiving opioids actually experiences a worsening of pain or an increase in sensitivity to painful stimuli.[34] At least three types of evidence support the notion that patient exposure to opioids lowers their pain threshold.[35] First, several studies found that patients on methadone maintenance have lower pain threshold to cold pressors than patients not on methadone maintenance.[36] Second, some studies involving patients undergoing surgery suggest that acute high-dose infusions of opioids seem to increase postoperative opioid consumption.[37] Third, in healthy volunteers, remifentanyl infusion was found to aggravate pre-existing mechanical hyperalgesia. This hyperalgesia is attenuated by NMDA antagonist ketamine and clonidine infusion.[38] Clinically, opioidinduced hyperalgesia is suspected when patients are taking increasing doses of opioids but present with rapid tachyphylaxis or diffuse hyperalgesia to stimuli. It can often be difficult to distinguish OIH from other painful conditions, like fibromyalgia, but the diagnosis must be considered in someone using escalating doses of opioids with worsening diffuse pain. Treatment includes tapering off and discontinuing opioid therapy and can occasionally include supplementation with NMDA receptor antagonists.

Arnold LM, Hudson JI, Hess EV, et al. Family study of fibromyalgia. Arthritis Rheum. 2004;50(3):944–952.

Wolfe F, Clauw DJ, Fitzcharles MA, et al. The American College of Rheumatology preliminary diagnostic criteria for fibromyalgia and measurement of symptom severity. Arthritis Care Res. 2010;62(5):600–610.

4.

The American College of Rheumatology. 1990 Criteria for the Classification of Fibromyalgia. Arthritis Rheum. 1990;33(2): 160–172.

Sperber AD, Atzmon Y, Neumann L, et al. Fibromyalgia in the irritable bowel syndrome: studies of prevalence and clinical implications. Am J Gastroenterol. 1999;94(12):3541–3546.

5.

Endresen GK. Mycoplasma blood infection in chronic fatigue and fibromyalgia syndromes.

Rheumatol Int. 2003;23(5): 211–215. 6.

Walker EA, Keegan D, Gardner G, et al. Psychosocial factors in fibromyalgia compared with rheumatoid arthritis: II. Sexual, physical, and emotional abuse and neglect. Psychosom Med. 1997; 59(6):572–577.

7.

Baraniuk JN, Whalen G, Cunningham J, Clauw DJ. Cerebrospinal fluid levels of opioid peptides in fibromyalgia

219

Chapter 29: Fibromyalgia and opioid-induced hyperalgesia

and chronic low back pain. BMC Musculoskel Disord. 2004;5:48. 8.

9.

Best Pract Res Clin Rheumatol. 2007;21(3):427–445.

Russell IJ, Vaeroy H, Javors M, Nyberg F. Cerebrospinal fluid biogenic amine metabolites in fibromyalgia/fibrositis syndrome and rheumatoid arthritis. Arthritis Rheum. 1992;35(5):550–556.

18. American Psychiatric Association. Diagnostic and Statistical Manual of Mental Disorders Fourth Edition Text Revision (DSM-IV-TR). American Psychiatric Association. 2000.

Guedj E, Taieb D, Cammilleri S, et al. 99mTc-ECD brain perfusion SPECT in hyperalgesic fibromyalgia. Eur J Nucl Med Mol Imaging. 2007;34(1):130–134.

19. Barkhuizen A. Rational and targeted pharmacologic treatment of fibromyalgia. Rheum Dis Clin North Am. 2002;28:261–290.

10. Cook DB, Lange G, Ciccone DS, et al. Functional imaging of pain in patients with primary fibromyalgia. J Rheumatol. 2004;31(2):364–378.

20. Mease P. Fibromyalgia syndrome: review of clinical presentation, pathogenesis, outcome measures, and treatment. J Rheumatol Suppl. 2005;75:6–21.

11. Roizenblatt S, Neto NS, Tufik S. Sleep disorders and fibromyalgia. Curr Pain Headache Rep. 2011; 15(5):347–357.

21. Arnold LM, Keck PE, Welge JA. Antidepressant treatment of fibromyalgia: a meta-analysis and review. Psychosomatics. 2000; 41(2):104–113.

12. Arnold LM, Hudson JI, Keck PE, et al. Comorbidity of fibromyalgia and psychiatric disorders. J Clin Psychiatry. 2006;67(8):1219–1225. 13. Buskila D, Cohen H. Comorbidity of fibromyalgia and psychiatric disorders. Curr Pain Headache Rep. 2007;11(5):333–338. 14. Bartels EM, Dreyer L, Jacobsen S, et al. Fibromyalgia, diagnosis and prevalence. Are gender differences explainable? Ugeskr Laeger. 2009;171(49):3588–3592. 15. Wolfe F, Ross K, Anderson J, Russell IJ, Hebert L. The prevalence and characteristics of fibromyalgia in the general population. Arthritis Rheum. 1995;38(1):19–28. 16. Changran A, Shaefer C, Ryan K, McNett M, Zlateva G. The comparative economic burden of mild, moderate, and severe fibromyalgia: results from a retrospective chart review and cross-sectional survey of workingage U.S. adults. J Manag Care Pharm. 2012;18(6):415–426. 17. Bennett R. Myofascial pain syndromes and their evaluation.

220

22. Capaci K, Hepguler S. Comparison of the effects of amitriptyline and paroxetine in the treatment of fibromyalgia syndrome. The Pain Clinic. 2002;14(3):223–228.

27. Bennett RM, Jones J, Turk DC, Russell IJ, Matallana L. An internet survery of 2,596 people with fibromyalgia. BMC Muscoloskelet Disord. 2007;8:27. 28. Goldengerg DL, Burckhardt C, Crofford L. Management of fibromyalgia syndrome. JAMA. 2004;292(19):2388–2395. 29. Falcao D, Sales L, Leite J, et al. Cognitive Behavioral Therapy for the treatment of fibromyalgia syndrome: a randomized controlled trial. J Muscoloskel Pain. 2008;16(3):133–140. 30. Harris RE, Clauw DJ, Scott DJ, et al. Decreased central μ-opioid receptor availability in fibromyalgia. J Neurosci. 2007; 27(37):10000–10006. 31. Baraniuk JN, Whalen G, Cunningham J, Clauw DJ. Cerebrospinal fluid levels of opioid peptides in fibromyalgia and chronic low back pain. BMC Musculoskel Disord. 2004;5(1):48.

23. Fishbain D. Evidence-based data on pain relief with antidepressants. Ann Med. 2000;32(5):305–316.

32. Furlan A, Sandoval JA, MailisGagnon A, Tunks E. Opioids for chronic noncancer pain: a metaanalysis of effectiveness and side effects. Can Med Assoc J. 2006; 174(11):1589–1594.

24. Crofford LJ, Rowbotham MC, Mease PJ, et al. Pregabalin for the treatment of fibromyalgia syndrome: results of a randomized, double-blind, placebo-controlled trial. Arthritis Rheum. 2005;52(4):1264–1273.

33. Sørensen J, Sjøgren P. Opioidinduced hyperalgesia. In Hanna M, Zylicz Z, eds. Cancer Pain. London: Springer. 2013: pp. 131–142.

25. Arnold LM, Goldenberg DL, Stanford SB, et al. Gabapentin in the treatment of fibromyalgia: a randomized, double-blind, placebo-controlled, multicenter trial. Arthritis Rheum. 2007; 56(4):1336–1344. 26. Hauser W, Eich W, Herrmann M, et al. Fibromyalgia syndrome: classification, diagnosis, and treatment. Dtsch Arztebl Int. 2009;106(23):383–391.

34. Lee M, Silverman S, Hansen H, Paterl A, Manchikanti L. A comprehensive review of opioid-induced hyperalgesia. Pain Physician. 2011;14:145–161. 35. Angst A, Martin S, Clark JD. Opioid-induced hyperalgesia: a qualitative systematic review. Anesthesiology 2006;104(3): 570–587. 36. Compton MA. Cold-pressor pain tolerance in opiate and cocaine abusers: correlates of drug type

Chapter 29: Fibromyalgia and opioid-induced hyperalgesia

and use status. J Pain Symptom Manage. 1994;9:462–473. 37. Guignard B, Bossard AE, Coste C, et al. Acute opioid tolerance: intraoperative remifentanil

increases postoperative pain and morphine requirement. Anesthesiology. 2000;93:409–417. 38. Angst MS, Koppert W, Pahl I, Clark DJ, Schmelz M. Short-term

infusion of the mu-opioid agonist remifentanil in humans causes hyperalgesia during withdrawal. Pain. 2003;106: 49–57.

221

Section 3 Chapter

30

Musculoskeletal Pain

Patient with myofascial pain syndrome: focus on functional restoration Tracy P. Jackson

Case study A 45-year-old male truck driver reported acute pain in his back after a collision with another motor vehicle 6 months ago, and filed a workers’ compensation claim at that time with his employer. He is an obese smoker, but otherwise healthy. He has been taking hydrocodone, cyclobenzaprine, and meloxicam and reports persistent 8/10 daily pain, sometimes up to 11–12/10. His MRI demonstrates degenerative disc disease with a broad-based central protrusion at L5/S1 without central canal or neuroforaminal stenosis. He has trialed a series of three epidural steroid injections, and a course of physical therapy which he could not complete because of pain. His physical exam reveals no focal neurologic findings, although he does report significant tenderness to palpation of his lumbar paraspinal musculature, and reports bending or twisting in any direction is painful. He has not been able to return to work and is applying for permanent disability benefits as a result of his inability to drive secondary to pain, limited range of motion, and difficulty with prolonged sitting. He is at your office with his case manager to request a functional capacity evaluation and is asking for pain medication to get to his next appointment with his primary treating physician; he ran out of pills from his last prescription a few days earlier secondary to “a really bad night” a few days ago. He says “nothing else works for my pain except the hydrocodone.” He has retained an attorney, as he is so frustrated with “getting the runaround from workers’ comp,” and is thinking of suing the driver who hit his truck.

1. What happens when an employee files a claim through workers’ compensation? The workers’ compensation (WC) system was developed by individual states in the USA in the early

twentieth century in order to strike a bargain between employer and employee rights. Employers agreed to be liable for payment for injuries incurred on the job regardless of fault in exchange for a limit to their liability; employees gave up the right to sue their employers in exchange for prompt and guaranteed payment. Although WC laws still vary by state, there are commonalities with regard to the process by which a claim is filed. The initial step is for the employee to notify the employer about the injury, whether acute (as in the case of the truck driver in a motor vehicle collision) or chronic (like wrist pain from repetitive keyboard usage). The employee may seek initial medical treatment from a physician of his choosing. The employer will immediately notify the insurer who then assigns a claims adjuster to the case. The claims adjuster will then review the insurance policy, request medical records for review, and take a report of the circumstances surrounding the injury from the employee, employer, and any witnesses. The adjuster will then make the decision to approve or deny the claim. Once a claim is approved, all further billing claims for healthcare related to the injury must go through the adjuster. Many times, the insurance carrier may hire a registered nurse case manager (NCM). The role of the NCM is to assist the injured worker (IW) in the planning and coordination of healthcare services and to be a liaison between the adjuster, the employer, the treating providers, and the patient. Although hired by the employer, the NCM is meant to be an advocate for the patient, helping the IW understand and navigate the complexities of the system and avail himself or herself of treatment options with informed consent.

Case Studies in Pain Management, ed. Alan David Kaye and Rinoo V. Shah. Published by Cambridge University Press. © Cambridge University Press 2015.

223

Chapter 30: Myofascial pain syndrome: focus on functional restoration

2. Why do some patients have an attorney? If the adjuster determines that injury is not workrelated, or that the injury is not caused by factors in the employer’s control, the claim is denied. The IW may file an appeal, and may even sue the employer if the IW can demonstrate intentional or egregious harm on the part of the employer. An involved third party (like the driver of the motor vehicle that struck the truck) may be subject to a personal injury lawsuit by the IW. If there is a dispute at any time between the IW and the employer/insurer, then an attorney is often retained by the IW to represent him. Many attorneys will accept these claims free of charge to the IW, or a state WC board may appoint an attorney to represent him upon request. Some IWs choose to have legal representation for the purposes of personal injury litigation; others choose to retain counsel in order to contest an adjuster’s denials of recommended medical care during the course of treatment. Attorneys for both sides may request a deposition with a physician to clarify that the patient’s symptoms (for example, pain) are related to the work injury, and/or to clarify the need for recommended services under dispute. The insurers or attorneys may also request a utilization review (UR) periodically, often involving an independent medical opinion from a non-treating physician to assist in determining medical necessity for proposed treatment plans. Of note, IWs seeking compensation via the legal system have higher mental health complaints at baseline compared to the non-compensation group, and these psychologic outcomes improve to a lesser degree by the time of settlement when compared to those not compensated with the aid of an attorney.[1] Claimants who retain an attorney secondary to dissatisfaction with the WC system also have longer times to settlement, higher cost, greater disability, and higher levels of post-settlement socioeconomic stress and catastrophizing at long-term follow-up.[2]

3. What are the different types of disability benefits and how is candidacy for these benefits determined? The level of disability is assessed in association with any WC claim in order to determine the amount of

224

financial restitution for which the IW qualifies. A physician makes this determination through performance of a functional capacity evaluation (FCE). There are several published metrics for FCE, all of which are intended to provide an objective measure of the IW’s functional ability compared to the physical tasks required for employment, and to track outcomes of treatment and rehabilitation programs. Using FCEs, physicians recommend duty or time restrictions when IWs return to work. FCEs may also aid in designation of a degree of impairment, which is typically used to describe anatomic or functional loss in a particular organ or body part (eye, spine, lung, finger). Assignment of the degree of disability takes into account the effect of this impairment on the patient’s ability to function in work or society. There are four types of disability recognized by state WC programs, although the schedules for length and amount of reimbursement vary. These are: 1. Temporary total disability (TTD). Wage-earning capacity is lost, but just temporarily. 2. Temporary partial disability (TPD). Wage-earning capacity is only partially lost, on a temporary basis. 3. Permanent total disability (PTD). Wage-earning capacity is permanently and totally lost. 4. Permanent partial disability (PPD). A portion of wage-earning capacity is permanently lost. The determination of permanent benefits is predicated on the patient reaching maximum medical improvement (MMI) as deemed by a physician. This implies that no further healing or improvement is possible despite continuing medical care or rehabilitation. If there is any dispute as to the impairment rating, assignation of MMI, or disability recommendations by the primary treating physician, the insurer my request an independent medical evaluation (IME) by a qualified medical evaluator (QME). QMEs are trained and licensed specifically to evaluate IWs and must participate in continuing medical education surrounding the changing regulatory guidelines in the state in which they practice. WC disability benefits only cover impairments sustained on the job. However, IWs may be eligible for Social Security Disability benefits (SDI) if they have permanent impairments that preclude any gainful work in any capacity, regardless of the location of the disabling event. Receipt of SDI requires that the IW has paid social security taxes prior to the disabling event. Supplemental Security Income (SSI) is another

Chapter 30: Myofascial pain syndrome: focus on functional restoration

federal subsidy program available to any disabled citizen with limited income or resources.[3–5]

4. What are pharmacologic options for our patient going forward? Medications This patient has received multimodal analgesic therapy with NSAIDs, muscle relaxants, and opioids and still reports 8/10 pain at baseline. The use of opioids is a controversial topic in management of chronic nonmalignant pain in both IWs and the population at large. Despite extensive investigation, there is still a lack of robust evidence that pain or functionality is improved in chronic non-malignant pain with opioid use longer than 12 weeks, and in fact, disability may be higher.[6] While opioids are clearly indicated for the management of acute pain secondary to injury, persistent opioid therapy in this patient, and all patients, should be carefully evaluated with individualized contextual assessment of the risks and benefits of ongoing opioid therapy. At a minimum, published clinical guidelines support opioids should provide both analgesia and increased activity as a result of this analgesia. Compared with IWs not prescribed opioids, odds of chronic work loss are estimated to be six times greater for IWs using schedule II opioids and 11–14 times greater for IWs with opioid prescriptions of any type for longer than 3 months, suggesting that opioid therapy may not arrest the cycle of pain and work loss.[7] Additionally, for those IWs who receive opioids long-term, opioid doses and cost increase substantially without clinically important improvement in pain and function, suggesting the benefit of chronic opioid therapy may not be effective in improving pain or reducing disability in the WC population.[8,9] Opioid therapy is also not without risk: The nationwide epidemic of opioid misuse and diversion is well documented.[10] For ongoing opioid therapy, adverse side effects and aberrant behavior should be absent, and screening and surveillance for opioid abuse is of paramount importance.[11,12] Risk factors for opioid misuse, abuse, or diversion include the following: 1. Past or current history of any substance abuse 2. Patterns: early refills, lost or stolen medication 3. Multiple ER visits, multiple doctors 4. Focus on opioids, won’t consider other options

5. Increasing dose with decreasing function 6. Patient’s belief of potential for addiction 7. “Excessive” opioid needs relative to other patients with same diagnosis[13] This patient has been on opioids > 12 weeks, abuses tobacco, and is requesting more opioids; he notes no demonstrable increase in function, no apparent sustained analgesia, and focuses on hydrocodone as the only medication that works. Given his risk factors, further surveillance and monitoring (controlled substance database query to check for multiple prescribers, urine drug screening, pill counts, close follow-up, etc.) is indicated if opioids are to be continued. Other options include opioid rotation, inpatient or outpatient detoxification from opioids, and trials of pharmacotherapy for chronic pain with antineuropathics or antidepressants. Medication management should be individualized and based on ongoing risk/benefit assessment in the broader context of multidisciplinary pain management strategies for patients refractory to initial treatment.[14–19]

5. Are there interventional options for our patient? As with medications, clinical guidelines for low back pain management recommend inclusion of interventional options within a multimodal plan. Options for interventional management of axial low back pain can target musculature (trigger point injections), facet joints (intra-articular steroid injections and radiofrequency ablation of medial branch nerves), epidural steroid injections (to which this patient has not responded), and sacroiliac injections, among others. However, IW status is relevant to consider when deciding upon interventional therapy, especially if initial or recurrent targeted interventional therapy is ineffective. This has been addressed in groups of IWs undergoing back surgery and spinal cord stimulation (SCS) for failed back surgery syndrome (FBSS). In one sample of IWs with FBSS, the high procedure cost of spinal cord stimulation (SCS) was not offset by lower costs of subsequent care.[20] In another trial of IWs randomized to either SCS, evaluation by a pain specialist without SCS, or neither intervention, < 10% of IWs in any group had reduced opioid use or improvement in any outcome measuring pain or function at 24 months.[21] Additionally, when combined with non-operative care, surgical treatment of

225

Chapter 30: Myofascial pain syndrome: focus on functional restoration

lumbar disc herniation appears to result in no meaningful clinical change in pain or functional outcomes for IWs, as opposed to the improvements attributable to surgery in a non-compensated group.[22] These data encourage caution in applying traditional interventional approaches to IWs whose pain is not durably relieved in a timely fashion with such measures. In these IWs, pain may be exacerbated by a variety of confounding psychosocial factors specific to the WC system, and further treatment strategies should take these into account.

6. What type of physical therapy is best? Physical therapy has long been a mainstay of treatment for the management of low back pain. However, the type and degree of physical rehabilitation can vary widely. Limitations of the literature are a result of very little consistency in the described components of the therapy, the type or amount of therapist involvement, the duration of therapy, the intensity of therapy, or the outcome measures used to assess efficacy. The general consensus is that exercise therapy that consists of individually designed programs (including stretching or strengthening) delivered with supervision may improve pain and function in chronic non-specific low back pain.[23] Specifically with regard to IWs, physical therapy modalities are the main components of rehabilitation. Cohesive outpatient physical therapy programming (which may or may not involve some vocational counseling or psychologic support) are often referred to as “back school,” “work simulation programs,” and “work hardening.” The focus is generally on spinal mobility, trunk strength, endurance, coordination, lifting capacity, positional tolerance, cardiovascular fitness, and ergonomics, although this too can vary widely. “Intense” physical conditioning programs that involve some sort of workplace focus or simulation are likely to have a more significant effect on the amount of work loss than “light” physical conditioning.[24] In any case, it is clear that non-compliance with physical therapy in this patient predicts a poorer functional prognosis.

7. Is psychologic therapy necessary for this patient? The interface between IWs and the WC system results in high patient-reported frustration, financial strain,

226

and mental distress independent of the injury or pain. This is attributed to limited healthcare access, conflicting medical opinions, poor understanding of the system, and confusion about decision-making authority.[25] Since psychosocial function is a key determinant of the capacity to rehabilitation in IWs, the ability to work and amount of work loss is mediated to a large extent by undiagnosed mental health comorbidities, and not purely somatic symptoms.[26] The fear avoidance model of pain (FAM) describes the cycle in which many patients with chronic pain, particularly in a WC setting, become progressively disabled. Pain catastrophizing is a psychologic feature describing those who have significant disabling anxiety regarding their ability to function in any context given the pain they experience. These patients often describe pain as > 10/10 on a numerical rating scale, as exhibited by this patient. As pain catastrophizing is the cognitive antecedent of pain-related fear, and pain-related fear is the emotional antecedent of depression and disability, catastrophizing and kinesiophobia are independent predictors of poor long-term pain-related outcomes. Furthermore, catastrophizing predicts long-term pain intensity and kinesiophobia predicts long-term work disability.[27] Workers who stay at work despite chronic musculoskeletal pain have lower levels of fear avoidance behavior, catastrophizing, and perceived workload. They also are more likely to accept their pain, and feel they have some control over both the pain and their life in general.[28] IWs who do not continue working tend to exhibit risk factors prior to injury. Occupational risk factors include:  Jobs that require effort beyond perceived physical capabilities  Low levels of job satisfaction  Poor working conditions  Poor rating by superiors  History of compensation for spinal condition  Litigation regarding compensation  Past receipt of work-related sickness payments[29]     

Psychologic risk factors include: Low level of schooling Low income Significant family dysfunction Depression, anxiety, anger, somatization High self-reported pain and disability levels

Chapter 30: Myofascial pain syndrome: focus on functional restoration

 Low self-reported expectation of improvement  Catastrophizing[29–31] As many of these risk factors are psychosocial, and often preceded the injury, meaningful reduction in disability and/or return to work may require a comprehensive multimodal approach to pain management and rehabilitation. While over 70% of IWs are reported to recover and return to work within a few months of injury, only 50% of IWs out of work for 6 months will return to work, and virtually none return after 2 years.[32] Therefore, in the presence of occupational or psychological risk factors, particularly catastrophizing and kinesiophobia, early multidisciplinary rehabilitation is recommended. With little psychologic history, this patient already has been out of work for 6 months and exhibits catastrophizing, kinesiophobia, high self-reported pain and disability, and possible litigation regarding his compensation. It is critical that psychological support be initiated if there is a chance for meaningful functionality in the future. This may be best accomplished with a functional restoration program.

8. What is a functional restoration program? Functional restoration programs (FRPs) are also variously described as functional rehabilitation programs or biopsychosocial/comprehensive multidisciplinary rehabilitation programs. The fundamental key to efficacy is that any program designated an FRP must have components to simultaneously address physical, social, psychological, and occupational deficits in a team setting.[24] The crux of therapy uses daily graded activity with application of operant behavioral principles toward movement aversion. Greater than 100 hours of continuous outpatient therapy is needed for improved functionality and reduction in pain and work loss when compared to shorter outpatient or inpatient programming.[33] Many FRPs also include opioid detoxification in the curricula, and those IWs who undergo withdrawal of opioids during the program still experience significant and durable improvement in pain severity and functioning, with no difference compared in outcomes when compared to those who did not undergo detoxification.[34] Goals of FRPs are primarily functional and include:  Improvement in physical function

     

Improvement in general function (social) Increase in self-management Improvement in vocational disability Reduction in opioid/sedative medication Reduction in healthcare utilization Reduction in pain level[35]

The physical components must involve “progression by contract” and education regarding “hurt vs. harm” so that IWs may not stop participating secondary to pain. In order to be effective at reducing work loss compared to usual care, a therapist or team using cognitive-based behavioral coaching must directly supervise intensive physical training, which must be active (as opposed to passive hands-on work by therapist) and should address aerobic capacity, muscle strength, endurance, and coordination in a way that is relevant to working. The key psychologic interventions often include a tailored combination of the following:  Behavioral pain management  Muscle relaxation  Guided imagery for stress reduction  EMG/temperature-guided biofeedback  Cognitive behavioral skills  Assertiveness training  Crisis intervention and family counseling  Education into meaning of disability and unemployment Overall, when FRPs use these parameters, patients return to work faster, have fewer sick leaves, and report less subjective disability.[24,36] This also results in significant reduction in annual healthcare costs.[37] Furthermore, comparable outcomes have been reported in different states and countries, with different economic and social systems.[38] Readiness to self-manage pain is a critical factor predicting completion.[39] In the 20% who do not complete the programs, risk factors include older age, female gender, opioid dependence, antisocial or cluster B personality disorders, longer duration of pain, extreme disability or receipt of SDI/SSI at admission, and non-working status at discharge.[40,41] In this patient, a FRP coupled with opioid detoxification, initiated as soon as possible, likely offers the best chance for meaningful functional recovery.

227

Chapter 30: Myofascial pain syndrome: focus on functional restoration

References 1.

Elbers NA, Hulst L, Cuijpers P, Akkermans AJ, Bruinvels DJ. Do compensation processes impair mental health? A meta-analysis. Injury. 2013;44:674–683.

2.

Chibnall J, Tait R. Legal Representation and dissatisfaction with workers’ compensation: implications for claimant adjustment. Psychological Injury and Law. 2010;3:230–240.

3.

http://www.tn.gov/labor-wfd/ wcomp.shtml.

4.

https://http://www.wcfgroup.com/ glossary-terms.

5.

http://www.ssa.gov/policy/docs/ ssb/v65n4/v65n4p3.html.

6.

Deshpande A, Furlan A, MailisGagnon A, Atlas S, Turk D. Opioids for chronic low back pain. Cochrane Database Syst Rev. 2007:CD004959.

7.

8.

9.

Volinn E, Fargo JD, Fine PG. Opioid therapy for nonspecific low back pain and the outcome of chronic work loss. Pain. 2009;142:194–201. Franklin GM, Rahman EA, Turner JA, Daniell WE, FultonKehoe D. Opioid use for chronic low back pain: a prospective, population-based study among injured workers in Washington state, 2002–2005. Clin J Pain. 2009;25:743–751. Bernacki EJ, Yuspeh L, Lavin R, Tao XG. Increases in the use and cost of opioids to treat acute and chronic pain in injured workers, 1999 to 2009. J Occup Environ Med. 2012;54:216–223.

10. Manchikanti L, Fellows B, Ailinani H, Pampati V. Therapeutic use, abuse, and nonmedical use of opioids: a tenyear perspective. Pain Physician. 2010;13:401–435. 11. Manchikanti L, Abdi S, Atluri S, et al. American Society of Interventional Pain Physicians (ASIPP) guidelines for responsible opioid prescribing in chronic

228

non-cancer pain: Part 2 – guidance. Pain Physician. 2012;15: S67–116. 12. Manchikanti L, Abdi S, Atluri S, et al. American Society of Interventional Pain Physicians (ASIPP) guidelines for responsible opioid prescribing in chronic non-cancer pain: Part I – evidence assessment. Pain Physician. 2012;15:S1–65. 13. Manchikanti L, Atluri S, Trescot AM, Giordano J. Monitoring opioid adherence in chronic pain patients: tools, techniques, and utility. Pain Physician. 2008;11: S155–180. 14. Chou R, Huffman LH; American Pain Society; American College of Physicians. Nonpharmacologic therapies for acute and chronic low back pain: a review of the evidence for an American Pain Society/American College of Physicians clinical practice guideline. Ann Intern Med. 2007;147:492–504. 15. Chou R, Huffman LH. Medications for acute and chronic low back pain: a review of the evidence for an American Pain Society/American College of Physicians clinical practice guideline. Ann Intern Med. 2007;147:505–514. 16. Chou R, Huffman LH. Nonpharmacologic therapies for acute and chronic low back pain: a review of the evidence for an American Pain Society/American College of Physicians clinical practice guideline. Ann Intern Med. 2007;147:492–504. 17. Bouton C, Roche G, Roquelaure Y, et al. Management of low back pain in primary care prior to multidisciplinary functional restoration: a retrospective study of 72 patients. Ann Readapt Med Phys. 2008;51: 650–656, 56–62. 18. Kroenke K, Krebs EE, Bair MJ. Pharmacotherapy of chronic pain: a synthesis of recommendations

from systematic reviews. Gen Hosp Psychiatry. 2009;31:206–219. 19. Morlion B. Chronic low back pain: pharmacological, interventional and surgical strategies. Nat Rev Neurol. 2013. 20. Hollingworth W, Turner JA, Welton NJ, Comstock BA, Deyo RA. Costs and cost-effectiveness of spinal cord stimulation (SCS) for failed back surgery syndrome: an observational study in a workers’ compensation population. Spine (Phila Pa 1976). 2011;36:2076–2083. 21. Turner JA, Hollingworth W, Comstock BA, Deyo RA. Spinal cord stimulation for failed back surgery syndrome: outcomes in a workers’ compensation setting. Pain. 2010;148:14–25. 22. Atlas SJ, Tosteson TD, Blood EA, et al. The impact of workers’ compensation on outcomes of surgical and nonoperative therapy for patients with a lumbar disc herniation: SPORT. Spine (Phila Pa 1976). 2010;35:89–97. 23. Hayden JA, van Tulder MW, Tomlinson G. Systematic review: strategies for using exercise therapy to improve outcomes in chronic low back pain. Ann Intern Med. 2005;142:776–785. 24. Schaafsma F, Schonstein E, Whelan KM, et al. Physical conditioning programs for improving work outcomes in workers with back pain. Cochrane Database Syst Rev. 2010: CD001822. 25. Kosny A, MacEachen E, Ferrier S, Chambers L. The role of health care providers in long term and complicated workers’ compensation claims. J Occup Rehabil. 2011;21:582–590. 26. Olaya-Contreras P, Styf J. Biopsychosocial function analyses changes the assessment of the ability to work in patients on long-term sick-leave due to chronic musculoskeletal pain: the role of undiagnosed mental health

Chapter 30: Myofascial pain syndrome: focus on functional restoration

comorbidity. Scand J Pub Health. 2013;41:247–255. 27. Sullivan MJ, Adams H, Martel MO, Scott W, Wideman T. Catastrophizing and perceived injustice: risk factors for the transition to chronicity after whiplash injury. Spine (Phila Pa 1976). 2011;36:S244–249. 28. de Vries HJ, Reneman MF, Groothoff JW, Geertzen JH, Brouwer S. Workers who stay at work despite chronic nonspecific musculoskeletal pain: do they differ from workers with sick leave? J Occup Rehabil. 2012;22:489–502. 29. Valat JP, Goupille P, Vedere V. Low back pain: risk factors for chronicity. Rev Rhum Engl Ed. 1997;64:189–194. 30. Gatchel RJ, Polatin PB, Mayer TG. The dominant role of psychosocial risk factors in the development of chronic low back pain disability. Spine (Phila Pa 1976). 1995;20:2702–2709. 31. Feuerstein M, Callan-Harris S, Hickey P, et al. Multidisciplinary rehabilitation of chronic workrelated upper extremity disorders: long-term effects. J Occup Med. 1993;35:396–403.

32. Poiraudeau S, Rannou F, Revel M. Functional restoration programs for low back pain: a systematic review. Ann Readapt Med Phys. 2007;50:425–429, 19–24. 33. Snodgrass J. Effective occupational therapy interventions in the rehabilitation of individuals with work-related low back injuries and illnesses: a systematic review. Am J Occup Ther. 2011;65:37–43. 34. Townsend CO, Kerkvliet JL, Bruce BK. A longitudinal study of the efficacy of a comprehensive pain rehabilitation program with opioid withdrawal: comparison of treatment outcomes based on opioid use status at admission. Pain. 2008;140:177–189. doi: 10.1016/j.pain.2008.08.005. Epub 08 Sep 19. 35. Sanders SH, Harden RN, Vicente PJ. Evidence-based clinical practice guidelines for interdisciplinary rehabilitation of chronic nonmalignant pain syndrome patients. Pain Pract. 2005;5:303–315. 36. Guzman J. Multidisciplinary bio-psycho-social rehabilitation for chronic low back pain. Cochrane Database Syst Rev. 2008;23:CD002213.

37. Gatchel RJ, Okifuji A. Evidencebased scientific data documenting the treatment and costeffectiveness of comprehensive pain programs for chronic nonmalignant pain. J Pain. 2006;7:779–793. 38. Gatchel RJ, Mayer TG. Evidenceinformed management of chronic low back pain with functional restoration. Spine J. 2008;8:65–69. 39. Tkachuk GA, Marshall JK, Mercado AC, McMurtry B, Stockdale-Winder F. Readiness for change predicts outcomes of functional rehabilitation following motor vehicle accident. J Occup Rehabil. 2012;22:97–104. 40. Howard KJ, Mayer TG, Theodore BR, Gatchel RJ. Patients with chronic disabling occupational musculoskeletal disorder failing to complete functional restoration: analysis of treatment-resistant personality characteristics. Arch Phys Med Rehabil. 2009;90: 778–785. 41. Brede E, Mayer TG, Gatchel RJ. Prediction of failure to retain work 1 year after interdisciplinary functional restoration in occupational injuries. Arch Phys Med Rehabil. 2012;93:268–274.

229

Section 3 Chapter

31

Musculoskeletal Pain

Spinal manipulation, osteopathic manipulative treatment, and spasticity Monika A. Krzyzek, John P. McCallin, Justin B. Boge, Dean Hommer, Prasad Lakshminarasimhiah, Rebekah L. Nilson, and Brandon J. Goff

Case study A 31-year-old male was referred to your office for spinal manipulation with a diagnosis of myofascial pain syndrome. He states that he has chronic low back pain and is asking for spinal manipulative treatment for symptomatic relief of his low back pain. On examination you notice that he has left-sided muscle spasm and somatic dysfunction of the lumbar spine at L5–S1 (rotated left side bent right). He also has mild spasticity, confined to the lower extremities, which upon further questioning you find out is from a benign but compressive meningioma that was neurosurgically removed from his thoracic spine when he was younger. His gait is altered due to lower extremity spasticity, and he has chronic low back pain that is worse when he ambulates or stands for a prolonged period of time. MRI shows chronic postsurgical changes, and mild facet hypertrophy, but is otherwise unremarkable.

1. Introduction to spinal manipulation Spinal manipulation is a technique that is employed by osteopathic physicians, chiropractors, physical therapists, and occupational therapists in the treatment of somatic dysfunction and pain that arises from misalignment of the vertebral segments. It is only one aspect of a broader field of manual therapy which includes a variety of interventions to address both joint and soft tissue dysfunctions. Specifically, manipulation of the spine involves passive, high, and low velocity thrust maneuvers that are performed before or at the physiologic end range of motion of synovial joints. Chiropractors refer to spine mobilization as an “adjustment,” while physical therapists call it a Grade V mobilization. Osteopathic physicians call spinal mobilization osteopathic manipulation therapy (OMT)

or osteopathic manipulative medicine (OMM). Spinal mobilization is a passive, high-velocity motion performed within the available physiologic range of motion at a dysfunctional segment. Somatic dysfunction may limit the motion of that vertebral segment, causing pain. Thus spinal manipulation, also known as HVLA (high velocity, low amplitude), attempts to restore proper motion and joint biomechanics to a spinal segment which has limited range of motion due to somatic dysfunction restriction. During spinal manipulation, the practitioner applies directed manual impulse, or thrust, to a joint, at or near the end of the physiologic tissue range of motion. This is usually accompanied by an audible “crack” or “pop” as the joint is moved through its restrictive/pathologic barrier. The reason for this audible “crack” is controversial; however it is widely thought to be a result of cavitation (implosion of gas bubbles) of the facet joint. Normal physiologic range of motion of the spinal segment is thus re-established by moving it through the restrictive barrier. The HVLA thrust is also thought to forcefully stretch contracted muscles. This sends afferent impulses from the muscle spindles to the central nervous system, causing an inhibitory response and relaxing the muscle. It is also thought that the stretch of the muscle causes stretching of the Golgi tendon apparatus which causes further relaxation of the tight muscle.[1–3] The basic principles of manipulation center around the premises that the body as a whole is a unit that is self regulating and self healing and that structure and function are reciprocally related. Somatic dysfunction is an alteration of function of the skeletal and myofascial structures, as well as associated vasculature, lympatics, and nerves. Somatic dysfunction is diagnosed on physical examination through tissue texture changes, asymmetry, restriction of motion,

Case Studies in Pain Management, ed. Alan David Kaye and Rinoo V. Shah. Published by Cambridge University Press. © Cambridge University Press 2015.

230

Chapter 31: Spinal manipulation, osteopathic manipulative treatment, and spasticity

Table 31.1. Fryette’s Laws of Spinal Motion

Principle I

Seen when the spine is in NEUTRAL position. Sidebending and rotation which occur to the OPPOSITE side

Seen in TYPE I SOMATIC DYSFUNCTIONS (aka. Group curves, where more than one vertebral segment is out of alignment). The entire group curve will be rotated to the side of the convexity

Principle II

Seen when the spine is in a NON-NEUTRAL position (either flexed or extended). Sidebending and rotation which occur to the OPPOSITE side in the restricted segment

Seen in TYPE II SOMATIC DYSFUNCTIONS (when only one segment is out of alignment). Segment is restricted in motion and becomes worse with either flexion or extension

Principle III

When motion is introduced in one plane it will reduce motion in the other planes

A dysfunction of motion in one physiologic plane will negatively affect all the other planes of motion of the spine

and tenderness on palpation. Acute somatic dysfunction of a spinal segment will show restricted range of motion with tenderness around the segment. Skin temperature around the area tends to be warm and there can be increased skin moisture around the area, as apparent with increased skin drag. The muscle and soft tissue is boggy, edematous, erythematous, and there is segmental muscle hypertonicity. Chronic somatic dysfunction also shows restricted range of motion and tenderness, however the tenderness to palpation tends to be more dull than sharp (as in the acute phase) and the patient may experience paresthesias in the area. The skin is cool and dry and feels ropy, fibrotic, and decreased muscle tone can be appreciated with palpation.[2,3] Multiple reflexes or arcs can produce stimuli which can generate a dysfunction in an area other than the primary problem. These include the somato-somatic reflex, somato-visceral reflex, viscera-visceral reflex, and the viscera-somatic reflex. Facilitation, a lowered threshold of neuronal excitation, can further lead to dysfunctions in body regions that are innervated by the same pool of neurons as the original dysfunction.[1,2] At the atlanto-occipital joint sidebending and rotation occur in opposite directions. At the atlantoaxial joint, mainly rotation occurs, with very little flexion or extension. In most of the cervical spine (C2–C7), coupling is seen where sidebending and rotation occur in the same direction.[1–3] In the thoracic and lumbar spine, normal motion of the vertebral segments with regard to sidebending and rotation occurs in the opposite direction. When the motion of individual spinal segments becomes restricted, somatic dysfunction occurs. In other words, when a vertebral level is significantly flexed or extended, as compared to the others around it, sidebending and rotation occur in

the same direction, which goes against normal spinal segmental motion. Spinal motion is summarized in Fryette’s Laws in Table 31.1.[1,2]

2. Contraindications to spinal manipulation Absolute contraindications to spinal manipulation include: osteoporosis, osteomyelitis, vertebral fracture, dislocation, skeletal neoplasm in the area of treatment, rheumatoid arthritis (especially of the cervical spine due to risk of atlantoaxial subluxation due to ligamentous laxity), Down’s syndrome (due to high incidence of atlantoaxial instability), abdominal aortic aneurism, bleeding diatheses, spondylolysis, spondylolisthesis, myelopathy, caudal equina compression, cord compression, and nerve root compression with increasing neurologic deficit. Relative contraindications include: pneumonia, coagulopathy, use of anticoagulants, acute muscle injury, acute joint inflammation, acute whiplash, neurologic pathologies such as a radiculopathy, herniated intervertebral disc, vertebral artery stenosis, cerebrovascular accident, pregnancy, open wounds, recent surgery, and joint hypermobility.[1,3–5] Risks of spinal adjustments are rare, provided the patients are appropriately screened and treated by experienced manipulative practitioners. These risks include: vertebrobasillar accidents (CVA and TIA), vertebral artery dissection, disc herniations or worsening of an existent disc herniation, vertebral fractures, epidural hematoma, rib fractures, and spinal cord injury (e.g., quadriplegia and cauda equina syndrome). Serious risks, although rare, can cause significant morbidity and mortality and should be included in any spinal manipulation informed consent.[1,2,5,6]

231

Chapter 31: Spinal manipulation, osteopathic manipulative treatment, and spasticity

3. Current literature considerations Strong randomized controlled trials for the use of spinal manipulation are lacking and more study into this area is needed. Some studies suggest that early spinal manipulation is effective for low back pain. Others show benefit for both short- and long-term outcome measures with spinal manipulation being better than placebo in treatment of acute low back pain.[7] Some studies point to spinal manipulation being more effective than general exercise but not specific home exercise programs or physical therapy but with spinal manipulation being more costeffective than formal therapy programs.[8] However, at this point, it is unclear whether spinal manipulation is more effective than physical therapy, massage, and other modalities.[7–9] There seems to be a difference between treating chronic versus acute pain and the part of the spine which is painful (cervical, thoracic, or lumbar). Overall there seems to be insufficient evidence to either support or refute the effectiveness of spinal manipulation on acute back pain, discogenic pain, or chronic back pain. Methodologic flaws in many of the studies make drawing conclusions and the development of clear guidelines difficult. Mechanism of injury, acuity, normal course of the disease process, or injury and cost-effectiveness, as well as side effects and risks and benefits all need to be weighted in the decision whether spinal manipulation is the correct course of treatment. We need to take into consideration that much of the acute spine (especially low back) pain improves on its own, therefore it is difficult to measure efficacy of therapies. Further studies are needed in the area of spinal manipulation as compared to other commonly used treatments such as structured physical therapy programs, specific home exercise programs, “back schools,” traction, bracing, acupuncture, biofeedback, and TENS (and other modalities) as well as medication management and interventional therapies.[10,11]

4. Introduction to spasticity Spasticity has been defined as an increase in muscle tone due to hyperexcitability of the stretch reflex and is characterized by a velocity-dependent increase in tonic stretch reflexes.[12] Spasticity is a component of upper motor neuron syndrome. Upper motor neuron syndrome can be seen in many conditions including stroke, spinal cord injury, traumatic

232

Table 31.2. Modified Ashworth Scale

0

No increase in tone

1

Slight increase in muscle tone, manifested by a catch and release or by minimal resistance at the end of the range of motion when the affected part(s) is moved in flexion or extension

1+

Slight increase in muscle tone, manifested by a catch, followed by minimal resistance throughout the remainder (less than half) of the ROM

2

More marked increase in muscle tone through most of the ROM, but affected part(s) easily moved

3

Considerable increase in muscle tone, passive movement difficult

4

Affected part(s) rigid in flexion or extension

brain injury, cerebral palsy, multiple sclerosis, and motor neuron disease, such as amyotrophic lateral sclerosis (ALS). Assessment of spasticity should include identifying which muscles or muscle groups are affected and how spasticity affects patient function, including mobility and ADLs. There are many ways to quantify spasticity. One of the most widely used scales is the Modified Ashworth Scale (Table 31.2).[13] An increase in spasticity over a patient’s baseline should raise suspicion for a noxious stimulus causing an aggravation of the patient’s spasticity. These factors can include urinary tract or other infections, urinary retention, constipation, a painful stimulus such as ingrown toenail, skin ulcer, or deep venous thrombosis (DVT). A cornerstone for spasticity management is prevention of these factors and early recognition and treatment.[14,15]

5. Spasticity management Spasticity can be managed non-pharmacologically, pharmacologically, and surgically. Non-pharmacologic treatments should be considered before moving to pharmacologic or other more invasive treatments. These interventions include daily ROM exercise programs, splinting and serial casting, cold and heat application, electrical stimulation, and vibration. When splinting and serial casting, special attention must be paid to skin integrity. Overlying skin must be frequently inspected for early evidence of skin breakdown.[14,15]

Chapter 31: Spinal manipulation, osteopathic manipulative treatment, and spasticity

There are many pharmacologic treatments that are mainstays in the treatment of spasticity. These treatments can include oral and IV medications as well as injectable neurolytics or neurotoxins.[14,15] GABA (gamma aminobutyric acid) is one of the main inhibitory neurotransmitters in the central nervous system. Some of the pharmacologic treatments have their effects on the GABA receptors. Baclofen is a GABA analog that binds to the GABA-B receptor decreasing calcium influx, ultimately reducing excitatory neurotransmitter release. Baclofen is often the drug of choice in treating spasticity related to spinal cord injury or multiple sclerosis. Sedation is one of the most common side effects. Patients and providers must be aware that abrupt discontinuation of baclofen can lower the seizure threshold. Another class of medications that acts on the GABA receptors is the benzodiazepines such as diazepam and clonazepam. These medications act on the GABA-A receptors and inhibit muscle contraction.[14,15] The alpha-2 agonists tizanidine and clonidine are centrally acting agents that decrease reflex activity. Common side effects of alpha-2 agonists are related to the alpha-2 adrenergic effects. These include hypotension, bradycardia, dizziness, and sedation. Although present with each, these side effects are generally less severe with tizanidine than clonidine.[14,15] While most of the oral pharmacologic treatments for spasticity are centrally acting, there is one agent that acts peripherally and therefore does not have the central side effects of the other medications. Dantrolene sodium, widely known for its use in malignant hyperthermia, acts peripherally on skeletal muscle decreasing calcium release from the sarcoplasmic reticulum, thereby inhibiting muscle contractions. This medication can be hepatotoxic and long-term use should be accompanied by routine monitoring of the liver function tests.[14,15] All of these oral and IV agents can affect spasticity in the entire body. For local treatment of spasticity, local injection of neurolytic agents or neurotoxins can be used.[14,15] The use of neurostimulation techniques, electromyography (EMG), or ultrasound for guidance can help localize the target site of injections for these local treatments and minimize the amount of medication required. The two most common neurolytic agents used are phenol and ethyl alcohol. Phenol causes protein denaturation and subsequent axonal necrosis. Ethyl alcohol is thought to cause dehydration of the nerve

tissues. The most common side effect is a dysesthetic pain when injected into a sensory or mixed nerve that can last for a significant amount of time. Because of this side effect these injections are usually only done on primarily motor nerves or in motor point blocks. For example, in a patient with a scissoring gait from adductor spasticity, a phenol injection of the obturator nerve can be performed with minimal side effects as the obturator nerve has only a small area of cutaneous sensory innervations.[14,15] Another common injectable treatment for spasticity is botulinum toxin. Botulinum toxin acts at the neuromuscular junction, presynaptically; it prevents release of acetylcholine by damaging the SNARE protein (Soluble N-ethyl-maleimide sensitive factor Attachment REceptor) and prevents contraction of the muscle. There are many different formulations of botulinum toxin and each has a different recommended dosing. In addition, it should be noted that there is no reliable conversion factor between the different types of botulinum toxin. Typically the peak effects of botulinum toxin are seen in 4–6 weeks after treatment and usually last 2–4 months.[14,15] A less common but effective pharmacologic treatment of spasticity involves intrathecal delivery of medication. Most commonly the medication delivered by this route is baclofen. The advantage this delivery system has is a significant increase in the potency of the medication with less systemic side effects. The primary disadvantage to this delivery method is the need for an invasive procedure for the implantation of the intrathecal pump. Additionally, intrathecal delivery of baclofen is generally more effective for lower extremity spasticity making it less ideal for the treatment of upper extremity spasticity. Complications of pump implantation include infection, pump failure, and tube dysfunction. In addition, the medication reservoir must be refilled periodically. Failure to refill the reservoir will result in the abrupt discontinuation of baclofen which, as described earlier, can decrease the seizure threshold. All of these factors must be taken into account when selecting patients for intrathecal baclofen. Prior to implantation, a trial must be performed. Typically this involves a lumbar puncture with delivery of 25 μg of baclofen into the intrathecal space. The level of spasticity is then monitored and compared to the preinjection level for the ensuing 4–6 hours. A decrease in one to two grades on the Modified Ashworth Scale in the desired muscle or muscle group would be

233

Chapter 31: Spinal manipulation, osteopathic manipulative treatment, and spasticity

considered a successful trial and pump implantation can then be considered. If an appropriate response is not obtained subsequent trials of increasing doses of 50, 75, and 100 μg may be considered.[14,15] If non-pharmacologic and pharmacologic treatments have failed to achieve satisfactory spasticity management and function, there are orthopedic and neurosurgical procedures that can be considered to assist with meeting these goals. These procedures include tendon lengthenings, tendon transfers, such as a split anterior tibial tendon transfer for equinovarus deformity, and neuroablative procedures, such as a dorsal rhizotomy or a myelotomy.[14,15] Finally it is well known that spasticity can result in multiple complications to include chronic pain, skin breakdown, altered gait, and difficulty with hygiene and other ADLs. An association has been identified

References 1.

2.

3.

4.

5.

6.

234

Modi RG, Shah N. Clinical Anatomy and Osteopathic Manipulative Medicine: COMLEX Review 2006, 1st ed. Malden, MA: Blackwell Publishing Inc. 2006. Savarese RG, Capobianco JD, Cox JJ. OMT Review: A Comprehensive Review in Osteopathic Medicine, 3rd ed. Jacksonville: OMT Review. 2003. Greenman PE. Principles of Manual Medicine, 3rd edn. Philadelphia, PA: Lippincott Williams & Wilkins. 2003. Cook C, Hegedus E, Showalter C, Sizer PS Jr. Coupling behavior of the cervical spine: a systematic review of the literature. J Manipulative Physiol Ther. 2006;29(7):570–575. Stevinson C, Ernst E. Risks associated with spinal manipulation. Am J Med. 2002;112(7):566–571. Di Fabio RP. Manipulation of the cervical spine: risks and benefits. Phys Ther. 1999;79(1):50–65.

between spasticity and heterotopic ossification, especially in patients with spinal cord injury and traumatic brain injury. However, before reducing spasticity, it is important to determine if the spasticity is benefitting the patient’s function. For example, a patient with lower extremity weakness may be using the spasticity to assist with standing or transfers, thereby achieving a higher level of function.[14,15]

Disclosure The view(s) expressed herein are those of the author(s) and do not reflect the official policy or position of Brooke Army Medical Center, the US Army Medical Department, the US Army Office of the Surgeon General, the Department of the Army, Department of Defense or the US Government.

7.

Jonsson ALNE. Neck and Back Pain: The Scientific Evidence of Causes, Diagnosis, and Treatment. Philadelphia: Lippincott Williams & Wilkins. 2000.

8.

Bronfort G, Evans R, Anderson AV, et al. Spinal manipulation, medication, or home exercise with advice for acute and subacute neck pain: a randomized trial. Ann Intern Med. 2012;156(1 Pt 1): 1–10. doi: 10.1059/0003-4819156-1-201201030-00002.

9.

Chou R, Huffman LH. Nonpharmacologic therapies for acute and chronic low back pain: a review of the evidence for an American Pain Society/American College of Physicians clinical practice guideline. Ann Intern Med. 2007;147(7):492–504.

10. Rubinstein SM, van Middelkoop M, Assendelft WJ, de Boer MR, van Tulder MW. Spinal manipulative therapy for chronic low back pain: an update of a Cochrane review. Spine (Phila Pa 1976). 2011;36(13):E825–846. doi: 10.1097/BRS.0b013e3182197fe1.

11. Rubinstein SM, Terwee CB, Assendelft WJ, de Boer MR, van Tulder MW. Spinal manipulative therapy for acute low back pain: an update of the Cochrane review. Spine (Phila Pa 1976). 2013;38(3): E158–177. doi: 10.1097/ BRS.0b013e31827dd89d. 12. Lance JW. Symposium synopsis. In Feldman RG, Young RR, Koella WP, eds. Spasticity: Disordered Motor Control. Chicago: Year Book Medical. 1980: pp. 485–494. 13. Bohannon RW, Smith MB. Interrater reliability of a modified Ashworth scale of muscle spasticity. Phys Ther. 1987;67(2): 206–207. 14. Braddom RL, Chan L, Harrast MA. Physical Medicine and Rehabilitation, 4th edn. Philadelphia, PA: Saunders/ Elsevier. 2011. 15. Cuccurullo SJ. Physical Medicine and Rehabilitation Board Review, 2nd ed. New York, NY: Demos Medical Publishing. 2009.

Section 3 Chapter

32

Musculoskeletal Pain

Patient with ankle pain Jose E. Barreto and Thomas K. Bond

Case study A 42-year-old female presents with ankle pain and a history of previous “ankle sprains.” The patient has pain and swelling associated with running activities and lately even with walking activities.

1. What is the differential diagnosis? a. Chronic ankle sprain b. High ankle sprain (syndesmosis injury) c. Talar dome fracture or osteochondral defects (OCD) d. Peroneal tendon subluxation e. Midfoot injury or fracture The most common form of ankle pain experience acutely is the “sprained ankle.” Most people have either experienced this personally or know someone who has. An ankle sprain is the “spraining” or injuring via stretching, and, in severe cases, tearing of ankle ligaments. Thus, a “sprain” is the injuring of a ligament or ligaments; whereas, a “strain” is the injuring of a muscle or its corresponding tendon. There are different grades or degrees of severity of ligament strains. Grade 1 or mild sprains, where only a few fibers are affected and there is no laxity (“looseness”) of the injured ligament, nor corresponding resulting instability of the joint which the ligament is stabilizing; Grade 2 or moderate sprains involve an incomplete or partial tear of the ligament which leads to mild laxity of the ligament and resulting joint instability; Grade 3 or severe sprains are characterized by complete/full-thickness tears of the ligament with resulting gross ligamentous laxity and joint instability. A chronic ankle sprain is a chronic pain state which can develop when an ankle sprain is either inappropriately rehabilitated, or the ankle fails to

improve despite receiving appropriate physical therapy and rehabilitation. The chronic ankle sprain is characterized by ligament laxity with resulting joint instability, thus leading to chronic, daily pain, and possibly, swelling and decreased functional ability.

2. What risk factors predispose patients to have a chronic ankle sprain? a. b. c. d. e.

Athletes Traumatic injuries Grade 2 or 3 ankle sprains Inadequate physical therapy or no rehabilitation Hypermobility syndrome

3. Why is this condition overlooked? There are about 23 000 ankle sprains in the USA every day[1] – that is, approximately 9 million ankle sprains each year in the USA alone. Unfortunately, it is estimated that up to 40% of these ankle sprains may become chronic leading to chronic ankle pain for some 3.5 million patients each year.[2] Many patients receive only “passive” treatment of just waiting for symptoms to improve on their own. Many of these patients go back to their normal activities and even back to the playing field without adequate physical therapy, particularly proprioceptive exercises. Some physicians think that an ankle sprain will heal on its own, even in severe cases.

4. Describe the anatomy and pathophysiology of a chronic ankle sprain The foot and ankle have two principal functions: propulsion and support. The ankle joint is made up

Case Studies in Pain Management, ed. Alan David Kaye and Rinoo V. Shah. Published by Cambridge University Press. © Cambridge University Press 2015.

235

Chapter 32: Patient with ankle pain

of two joints: the talocrural and the subtalar joints. The talocrural joint – the true ankle joint which is made of the tibia, fibula, and talus – is a uniaxial, modified hinge, synovial joint responsible for dorsiflexion and plantarflexion. On dorsiflexion the talus is wedged between malleoli, so there is no inversion or eversion; this is the closed packed position of this joint.[3] The subtalar (talocalcaneal) joint is a synovial joint with three degrees of freedom, where supination and pronation occurs. It is supported by lateral and medial talocalcaneal ligaments but the main support comes from the interosseous talocalcaneal ligament which limits eversion. The distal tibiofibular joint is important to mention here since it can suffer in cases of high ankle sprain. This joint is a fibrous or syndesmosis type of joint. It is supported by four ligaments – anterior and posterior tibiofibular ligaments, inferior transverse ligament, and interosseous ligament. The pathophysiology/mechanism of injury of an ankle sprain typically is via extreme ankle inversion (foot rolling in), eversion (foot rolling out), or rotation (either rotating in or rotating out). The most common type of ankle sprain injury mechanism is the inversion/internal rotation: as the foot rotates out of the normal plane of space (foot on the floor), the sequence of injury usually is as follows: the first ligament injured is the antero-talofibular ligament (ATFL), which is located in the “front” of the ankle. With continued rotation or inversion of the foot, the next ligament to be injured is the CFL or calcaneofibular ligament – located on the “side” of the ankle. Lastly, with continued rotation/inversion, a severe injury will occur, i.e., the dreaded “Grade-3 ankle sprain,” which in addition to the ATFL and CFL, will also cause injury to the posterior talofibular ligament (PTFL) – located at the “back” of the ankle.

5. How do you diagnose a chronic ankle sprain? Patients will usually report a twisting or rolling of the ankle with or without an audible or perceived “pop,” which is typically the sound of the ligaments being injured. The symptoms of the sprain depend on the severity of the injury and the time course; thus, a mild/Grade 1 sprain may only demonstrate minimal pain and swelling at the time of ligament injury; whereas, more severe grades may show copious

236

swelling, hematoma formation, severe pain, and inability to bear weight. It is also important to inquire if there was a history of previous sprain/injury, as well as the mechanism of injury. Onset of injury, ability to walk, run or bear weight, and whether there is pain elsewhere in the leg are also standard questions for the ankle history. The patient with chronic ankle sprain will typically give a history of multiple sprains, and an ankle which is “always swollen.” They may also state the ankle is “weak” and “rolls very easily” – demonstrative of ligamentous laxity and resulting joint instability. Physical examination starts with inspection for abnormal alignment, antalgic gait, color change, skin texture change, or visible edema. Next palpation for tissue texture changes, palpable edema, and tenderness is assessed. Range of motion testing, both active (AROM) and passive (PROM), are performed bilaterally to assess for asymmetry, restriction, or painful arc. Strength testing (AROM – against resistance) is used to assess integrity of the MTU (muscle–tendon unit) and its nerve supply. This is followed by “special tests” which are used to assess the integrity of the ligaments and thus the stability of the joint. Examples of the special tests for the ankle include: the anterior drawer test (ankle at 90 degrees, grasp heel and pull forward along with a posterior force with the other hand on distal tibia); the talar tilt test (ankle at 90 degrees, the heel is firmly adducted or inverted; the normal end feel is firm – increased laxity compared to the other side suggests damage to CFL); the external rotation test (foot external rotation with patient sitting and knee at 90 degrees and holding the tibia in fixed position; pain indicates syndesmosis injury – high ankle sprains); and the squeeze test (squeeze tibia and fibula together, pain distally indicates syndesmotic sprain). Diagnostic imaging typically begins with x-rays, which include the standard three views: ap (anteroposterior), lateral, and mortis views. With chronic ankle sprain, because of the ligament laxity and resulting joint instability, there may be osteoarthritis of the ankle joint. Additionally, there can be “holes” in the cartilage of the bones of the ankle joint resulting from abnormal gross joint movements, again from ligament laxity and joint instability. These lesions are called osteochondral defects (OCD). These osteochondral cartilage defect lesions are typically noted in the “talar dome”, or dome of the talus bone of the ankle mortise. OCD lesions can sometimes be seen on x-ray imaging, but are more

Chapter 32: Patient with ankle pain

easily demonstrated with CT scan or MRI. More recently, in the past several years, ultrasound (US) imaging has been utilized in sports/orthopedic medicine to evaluate the ligament damage in chronic ankle sprains. US allows the physician to evaluate the patient using no radiation, and also allows visualization of the ligaments, tendons, and joints dynamically in real-time – something which is impossible with other forms of imaging. The MRI is usually the study of choice due to its capacity to assess both intra-articular and extraarticular manifestations of lateral ankle sprains. However, it is less cost-effective and may have inferior resolution for partial tears compared to ultrasound. Laboratory studies are not helpful.

osteoarthritis (OA) of the knee. However, there are only case reports of anecdotal improvement in chronic ankle pain from OA. There is no scientific rationale or data to support the use of viscosupplementation/HA injections for chronic joint instability from damaged ligaments.

Acupuncture There have been a few case reports and low-quality studies in the literature showing short-term improvement in pain and QOL (quality of life) scores with acupuncture in acute ankle injuries. There was no evidence for the use of acupuncture with chronic ankle pain and joint instability.

Dry-needling

6. How should I treat this patient? Conservative approaches The typical initial treatment paradigm for the chronic ankle patient is physical therapy (PT). The PT interventions and modalities are targeted to address improving joint stability, ankle proprioception, and muscle activation and strengthening. Other techniques employed include taping (e.g., kinesiotaping), bracing, and modalities such as ice, heat, e-stim, and ultrasound.[4] Unfortunately, these techniques typically do not lead to resolution of pain and dysfunction in the chronic ankle pain patients with instability, as these techniques do not tighten ligaments which are damaged and in a laxity state. Typical analgesic therapy consists of non-steroidal anti-inflammatories or opioids.

Dry-needling is a technique which has recently come back into favor with physical therapists as a way to stimulate an immune system proliferative/healing response. Although there is an intuitive scientific rationale for its use in the treatment of the chronic ankle pain patient with joint hypermobility, there is no data showing its efficacy.

Radiofrequency ablation/radiofrequency coblation

There have been many interventions attempted for chronic ankle pain patients. Some of the more common ones are listed below:

These pain management procedures are performed by destroying or “ablating” one or more of the sensory nerves innervating the ankle. Recent studies have shown “good to excellent” results when measuring patient’s pain scores in the short-term of 3–6 months. These results returned to original baseline pain after 6 months – likely due to the fact these procedures do not address function (the ligament damage and resulting joint hypermobility), but only pain (by burning the nerve). Thus, the pain begins to return as the nerves begin to regenerate. Still, the evidence is robust as a pain management procedure for shortterm relief.[5]

Corticosteroid injections

Regenerative injection procedures

Although commonly used by physicians in the USA, steroid injections have not been shown to improve pain or dysfunction in chronic ankle pain; nor have they been shown to decrease need for pain medications, or slow time to surgical intervention.

Prolotherapy “Prolotherapy,” or proliferative injection therapy, is an injection technique performed with the intent of stimulating a “proliferative immune/healing response,” leading to a healing, and thus tightening, of damaged ligaments and tendons (Figures 32.1 to 32.3). Prolotherapy had been advocated for chronic joint pain and hypermobility secondary to supporting ligament damage for many decades. Quality evidence proving its

Procedural

Viscosupplementation/hyaluronic acid injections Viscosupplementation has shown some promise with chronic pain in other joints, particularly patients with

237

Chapter 32: Patient with ankle pain

Figure 32.1. Ankle – lateral – prolotherapy injection: 1. posterior talofibular and posterior tibiofibular ligaments; 2. calcaneofibular ligament; 3. anterior talofibular ligament; 4. sinus tarsi; 5. bifurcate ligament (calcaneonavicular and calcaneocuboid ligaments) and dorsal cuboideonavicular ligament; 6. peroneus brevis tendon.

Figure 32.3. Ankle – plantar – prolotherapy injection: 1. plantar fascia – long plantar and short plantar ligaments; 2. plantar aponeurosis.

treatment as demonstrated by reports of reduced pain levels, increased range of motion, extended ability to exercise, reduced depression, reduced anxiety, and a reduction in medications needed.[6] The second case series report demonstrated post-prolo tissue healing via imaging evidence on high-resolution ultrasound and MRI of three patients with chronic ankle pain and arthritis from joint hypermobility.[7] Additional RCTs are needed. See Figures 32.1 to 32.3.

Platelet-based procedures Figure 32.2. Ankle – medial – prolotherapy injection: 1. anterior tibiotalar ligament; 2. tibiocalcaneal ligament; 3. posterior tibiotalar ligament; 4. tibionavicular ligament; 5. plantar arch ligaments (plantar calcaneonavicular, plantar cuboidonavicular); 6. tibialis posterior tendon; 7. tibialis anterior tendon; 8. plantar fasciaaponeurosis.

efficacy in this patient population is lacking. However, there have been two case series published supporting both clinical improvement and evidence of tissue healing on follow-up imaging studies. A case series report was done on patients treated for unresolved, chronic ankle pain at a volunteer charity clinic having limited resources and personnel between 2000 and 2005. Treatment consisted of injecting a dextrose solution at specific ankle sites to stimulate healing of ligaments, tendons, and joints. Patients, including those who were told by previous doctors that “nothing more could be done” or that “surgery was the only option,” responded favorably to

238

These procedures, like Prolotherapy, are performed with the intention to stimulate healing through activation of the body’s immune response. Platelets are acquired via blood draw and concentrated by various methods. The standard, commercial grade centrifuge/ platelet processor can deliver a platelet-rich plasma (PRP) with concentrations of 3–7 times that of erythrocyte (RBC) poor serum. The Regenexx process can produce a platelet concentrate – the so-called Regenexx-SCP – of up to 20 times baseline. Platelet-rich plasma: Recent data has shown efficacy for PRP treatments in chronic ankle pain with OCD of the talus. These results demonstrated increased function and decreased pain scores up to 6 months post-procedure. Additional investigation including RCTs is needed.[8] Regenexx-SCP or “super-concentrated platelets”: Small case series for SCP treatments have demonstrated efficacy in improving function and decreasing pain.[9] Additional investigation including RCTs is needed.

Chapter 32: Patient with ankle pain

Surgical treatment options Open surgical repair for chronic ankle instability/hypermobility There are many different surgical techniques for addressing gross hypermobility of the ankle. One of the more well-known of these procedures is the “Broström-Gould repair,” which has been shown to be effective at correcting instability, and returning patients to pre-injury levels of function, even highlevel athletes. However, this and other surgical techniques been shown to be effective only for severe/ 3rd-degree ankle sprains with widening of the ankle mortise, and for those with certain associated fractures such as malleolar, distal fibular fractures, or detached talar dome fractures. Additionally, fractures at the 5th metaphyseal–diaphyseal junction (Jones’ fractures) may require surgical fixation as well in order to return the patient to high levels of function.

References 1.

2.

3.

Kannus P, Renström P. Treatment for acute tears of the lateral ligaments of the ankle: operation, cast, or early controlled mobilization. J Bone Joint Surg Am. 1991;73(2):305–312. Gerber JP, Williams GN, Scoville CR, Arciero RA, Taylor DC. Persistent disability associated with ankle sprains: a prospective examination of an athletic population. Foot Ankle Int. 1998;19(10): 653–660. Hubbard TJ, Hicks-Little CA. Ankle ligament healing after an acute ankle sprain: an evidencebased approach. J Athl Train. 2008;43(5):523–529.

4.

Open surgical repair for chronic ankle pain, OA/other causes Joint fusion: ankle arthrodesis. Ankle arthrodesis, or fusion, has been performed for many years with relative success controlling pain scores. More recently, studies have shown that Arthroscopy-assisted Ankle arthrodesis improves not only pain scores, but some aspects of function as well.[10] Total ankle arthroplasty (TAA). TAA has traditionally seen worse outcomes than other total joint replacements, such as knee and hip. However, with recent technologic advancements, some surgeries performed at TAA centers with highly specialized surgeons demonstrated improvement in function and pain scores in 46 patients (approx. 90% of study subjects) in one small study. Further research is needed including RCTs.

Kemler E, van de Port I, Backx F, van Dijk CN. A systematic review on the treatment of acute ankle sprain: brace versus other functional treatment types. Sports Med. 2011;41(3):185–197.

5.

Yeap EJ, Chong KW, Yeo W, Rikhraj IS. Radiofrequency coblation for chronic foot and ankle tendinosis. J Orthop Surg (Hong Kong). 2009;Dec;17(3): 325–330.

6.

Hauser R, Hauser MA, Cutla J. Dextrose prolotherapy injections for chronic ankle pain. Practical Pain Management. January 1, 2010.

7.

Fullerton BD. High-resolution ultrasound and magnetic resonance imaging to document tissue repair after prolotherapy:

a report of 3 cases. Arch Phys Med Rehabil. 2008;89(2):377–385. 8.

Smyth NA, Murawski CD, Haleem AM, et al. Establishing proof of concept: platelet-rich plasma and bone marrow aspirate concentrate may improve cartilage repair following surgical treatment for osteochondral lesions of the talus. World J Orthop. 2012;3(7):101–108. doi: 10.5312/wjo.v3.i7.101.

9.

http://www.regenexx.com/ category/recent-results/anklecase-results/

10. Wang JL, Liu YJ, Li ZL, Wang ZG, Wei M. Outcome evaluation of arthroscopy-assisted ankle arthrodesis. Zhongguo Gu Shang. 2011;24(9):719–722.

239

Section 3 Chapter

33

Musculoskeletal Pain

Patient with lateral epicondylosis or other focal tendinopathy Jose E. Barreto and Jeff Ericksen

Case study A 40-year-old male presents with a 2-month history of lateral elbow pain. He has taken over-the-counter medications with no relief.

1. What is the differential diagnosis? a. Lateral epicondylosis (LE) b. Posterior interosseous nerve (PIN) entrapment c. Partial or full-thickness tear of the extensor tendons d. Cervical radiculopathy at C6–C7 e. Rheumatoid arthritis Lateral epicondylosis is a painful condition of the lateral/extensor area of the elbow. This condition is also known as “tennis elbow” and lateral epicondylitis, but absence of inflammatory cells in the area have recently caused a change in the name to epicondylosis.[1,2] It is caused by repetitive or overuse injury of the common extensor mechanism; primarily the extensor carpi radialis brevis (ECRB). There is a higher incidence in patients older than 35 years. It has no gender predilection and affects 1–3% of the population.[3,4]

2. What risk factors predispose patients to developing sacral insufficiency fractures? a. Repeated wrist extension movements (occupational) b. Faulty biomechanics or technique (sports or occupational) c. Inadequate equipment d. Racquet sports

3. Describe the anatomy and pathophysiology of a lateral epicondylosis The elbow’s primary function is to help position the hand in an appropriate location for function. It has three cubital articulations: ulnohumeral (trochlear) joint, radiohumeral joint, and the superior or proximal radioulnar joint. It has a continuous capsule and a joint cavity with the three joints. It has 2 degrees of freedom. The ulnohumeral (trochlear) joint is a uniaxial hinge joint with an axis that is downward and medial which created the carrying angle. The radiohumeral joint is a uniaxial hinge joint between capitulum (humerus) and head of radius, it allows flexion and extension movements. The superior or proximal radioulnar joint is a uniaxial pivot joint. The annular ligament keeps its proper position. This joint allows the rotation movements of the head of the radius on supination and pronation. The lateral epicondyle and the lateral supracondylar ridge form the origin of the extensor tendons. These include the extensor carpi radialis longus, ECRB, extensor digitorum communis (EDC), and the extensor carpi ulnaris (ECU). Of these, the ECRB, EDC, and ECU are part of the common extensor tendon. Repetitive use and contraction of the forearm muscles, particularly eccentric contractions, can cause microtrauma and subsequent degeneration, immature repair, and tendinosis. Recent studies have confirmed the presence of fibroblasts, vascular hyperplasia, and disorganized collagen (angiofibroblastic hyperplasia) with a paucity of acute or chronic inflammatory cells. Limited blood supply of these tendons may be partly responsible for the tendinosis.[1,2]

Case Studies in Pain Management, ed. Alan David Kaye and Rinoo V. Shah. Published by Cambridge University Press. © Cambridge University Press 2015.

240

Chapter 33: Patient with lateral epicondylosis or other focal tendinopathy

Also, following the tensegrity model, if you have a loose distal wrist joint then the proximal segment (the elbow) needs to compensate and overwork to stabilize the distal segment since the forearm extensor muscles cross two joints.

4. How do you diagnose a lateral epicondylosis? The pain is located at the elbow, sharp in quality, and is associated with occasional radiation distally and weakness of the wrist extensors. There can be swelling in the area and symptoms of numbness and paresthesias; in this case suspect an entrapment of the deep branch of the radial nerve (posterior interosseous nerve – PIN). On examination, there is tenderness to palpation over the lateral epicondyle and the area extending 1–2 cm distally. Also, examine your patients for tenderness on the insertion of the biceps tendon (focal palpation below radial head with the forearm in pronated position). This can be an indication of insertional tendinosis, which can occur in patients with sports or occupations that demand frequent supination movements. Special tests include the resisted wrist extension (RWE) test (pain reproduced with wrist extension against examiner’s resistance), the ECRB test (patient holds the elbow extended, the forearm pronated while making a fist and extending the wrist – the examiner applies resistance and pain indicates a positive test for LE involving the ECRB muscle), and the resisted middle finger test (pain on resisted extension of the middle finger – this tightens the fascial origin of the ECRB). Also, examine the neck and shoulders as these areas can cause referred pain to the elbow. A neurologic exam will help assess the status of the radial nerve and its branches which may get involved as well (entrapment of the deep branch of the radial nerve may occur as it passes through the arcade of Frohse/radial tunnel in the supinator muscles). Radiographic studies may help rule out an intraarticular pathology, bone involvement, calcification, or exostosis at the epicondyle or in the tendon close to the tendon attachment. This may be seen in 20% of cases, but has no prognostic indications. Ultrasound may show increased blood flow near the lateral epicondyle and tendinosis appears as hypoechoic swelling of the involved tendon with possible hyperechoic calcification and adjacent bone irregularity.

Magnetic resonance imaging can help assess intra-articular pathology and the soft tissues including ligaments and will identify tendinosis. If the patient is having pain and/or inflammation in other areas, consider lab work including an ESR and C-reactive protein (CRP).

5. Is there any other diagnostic testing that should be done? Grip strength, elbow, and wrist range of motion (ROM) examination will help assess functional status.

6. How should I treat this patient? Conservative approaches At the initial phase of treatment, the focus should be on pain relief. This can be accomplished by PRICEMM – protect, ice, compression, elevation, medication, modalities. Physical therapy should include stretching of wrist flexors and extensors (slightly), along with strengthening of the wrist (progress from isometric to concentric to eccentric), elbow, and shoulder muscles (rotator cuff). Eccentrics could be done with weight or with resistance given by the other hand. Other physical medicine treatment modalities include therapeutic ultrasound, heat, and soft tissue therapy. To control force loads you can use a counterforce brace (forearm strap), improve sports technique, develop two handed backhand stroke for tennis players, and control intensity, duration and frequency of activities. Prevention strategies include exercises to address flexibility and strength of the spine, shoulder, scapular stabilizers, and arm and forearm muscles. Tennis players should be instructed on proper technique such as striking the ball in front of the body with the wrist and elbow extended, allowing for upper arm and torso (not the wrist extensors) to provide stroke power. The racquet should be lightweight and of low vibration material (graphite, epoxies) and appropriate grip size (distance from tip of ring finger to the proximal palmar crease, along its radial border) and low string tension.

Procedural Interventions for LE include corticosteroids, although this is controversial and studies have shown it causes more harm.[5] Newer and more accepted treatments

241

Chapter 33: Patient with lateral epicondylosis or other focal tendinopathy

Figure 33.1. Lateral elbow prolotherapy injection: 1. extensor carpi radialis longus tendon; 2. lateral (radial) collateral ligament; 3. common extensor tendon; 4. intra-articular injection; 5. lateral (radial) collateral ligament; 6. annular ligament; 7. lateral collateral ligament– anterior bundle.

include prolotherapy and platelet-rich plasma which have shown improvement.[6,7] Others include tenotomy and stem cell injections. Other treatments include surgery, ECSW, and acupuncture. For prolotherapy injections (Figures 33.1 and 33.2), it is important to treat the lateral collateral

References

Figure 33.2. Medial elbow prolotherapy injection: 1. pronator teres tendon; 2. common flexor tendon; 3. medial (ulnar) collateral ligament-posterior and anterior bands.

and annular ligaments since they play a big role in the stability of this area. Don’t forget to assess and treat the insertion of the biceps tendon below the radial head with the forearm in pronated position. This can be an indication of insertional tendinosis, which can occur in patients with sports or occupations that demand frequent supination movements.

nerve decompression: is outcome influenced by the occupational disease compensation aspect? Orthop Traumatol: Surg Res. 2011;97(2): 159–163.

1.

Bishai SK, Plancher KD. The basic science of lateral epicondylosis: update for the future. Tech Orthop. 2006;21(4):250–255.

2.

Walz DM, Newman JS, Konin GP, Ross G. Epicondylitis: pathogenesis, imaging, and treatment. RadioGraphics. 2010;30:167–183.

4.

Allander E. Prevalence, incidence and remission rates of some common rheumatic diseases and syndromes. Scand J Rheumatol. 1974;(3):145–153.

3.

Bigorre N, Raimbeau G, Fouque P-A, et al. Lateral epicondylitis treatment by extensor carpi radialis fasciotomy and radial

5.

Scarpone M, Rabago DP, Zgierska A, Arbogast G, Snell E. The efficacy of prolotherapy for lateral epicondylosis: a pilot

242

study. Clin J Sport Med, 2008; 18(3):248–254. 6.

Coombes BK, Bisset L, Brooks P, Khan A, Vicenzino B. Effect of corticosteroid injection, physiotherapy, or both on clinical outcomes in patients with unilateral lateral epicondylalgia: a randomized controlled trial. JAMA. 2013; 309(5):461–469.

7.

Mishra A, Pavelko T. Treatment of chronic elbow tendinosis with buffered platelet-rich plasma. Am J Sport Med. 2006;34:1774–1778.

Section 3 Chapter

34

Musculoskeletal Pain

Knee osteoarthritis with emphasis on percutaneous regenerative medicine Jason Tucker, Christopher Centeno, and Jeff Ericksen

Case study A 52-year-old female (BMI 22) presents with a several year history of activity-induced right-sided knee pain accompanied by stiffness most bothersome upon first rising in the morning, prior to retiring at night, and after cooling down from heavy exercise.

1. What are the most common etiologies of chronic knee pain? The prevalence of the respective perpetuators of pain is dependent on age. In middle to elderly ages the following are commonly seen:  Knee osteoarthritis: Extrapolation from observational studies suggests that osteoarthritis of the knee has a lifetime incidence of close to 1 in 2 individuals with approximately 1 in 5 over the age of 45 suffering from the disease.[1] Since many of these epidemiologic studies require radiographic evidence of disease for diagnosis, prevalence rates may be erroneously low (see imaging section below)  Meniscal injury (degenerative > traumatic)  Crystal and autoimmune rheumatologic disease  Tendinopathies

2. What is knee osteoarthritis and what is the pathophysiologic basis for its development? The knee is a tricompartmental, synovial joint structure. The three cardinal compartments are the medial and lateral tibiofemoral compartments and the patellofemoral joint (PFJ), which are all lined by articular

cartilage (AC). The cell constituents of AC are chondrocytes and more recently it has been found to also possess progenitor cells.[2] In a non-diseased joint, unassisted cell components possess very limited ability to self-replicate, but they play an integral role in maintaining a state of equated turnover of the extracellular matrix (ECM). The ECM, which consists of proteoglycans (mainly aggrecan) and hyaluronic acid with reinforcement provided by collagen (mainly type II), encompasses the aforementioned cellular components.[3] Knee osteoarthritis (KOA) is a generally agerelated, inflammatory mediated, degenerative joint disease (DJD) that manifests as joint stiffness and pain leading to reduction in functionality and subsequent deterioration in quality of life.[4] DJD is a term synonymous with osteoarthritis and provides a partially accurate macroscopic portrayal, but at its microscopic core, OA is best defined as a low-grade, self-perpetuating inflammatory state.[4] A disruption of the normally homeostatic state of balanced anabolism and catabolism of the ECM with a shift toward favoritism of a catabolic environment eventually leads to macroscopic hyaline cartilage degeneration[3] and unbeneficial synovial overgrowth and inflammatory changes in addition to bony hypertrophy (osteophyte formation).[5] This exemplifies the fact that OA is a whole joint disease with triad involvement of the AC, subchondral bone, and synovial tissue (and possibly the fat pad).[4] Furthermore, since cartilage is thought to be aneural, a predominance of pain may arise from neurogenic inflammation whereby an overgrowth of sympathetic nerves within the synovium and subchondral bone combines with binding of pain producing biochemical peptides to these nerve endings.[6]

Case Studies in Pain Management, ed. Alan David Kaye and Rinoo V. Shah. Published by Cambridge University Press. © Cambridge University Press 2015.

243

Chapter 34: Knee osteoarthritis with emphasis on percutaneous regenerative medicine

The patient has a past medical history of an ipsilateral anterior cruciate ligament tear in her 20s. Other pertinent medical history includes morbid obesity (BMI 40) until gastric bypass surgery 3 years ago and a family history of OA in her mother and brother.

3. What are the common causes of KOA?[1] 1. Acute mechanical damage from a traumatic joint injury can create joint instability, catalyzing the evolution of OA from the ensuing chronic mechanical damage (see #2 below) over years or decades as a result of chronic cruciate and/or collateral ligamentous laxity and/or meniscal injury. Reasons for this may include: A. Sustained inflammatory milieu environment, including exposure of the joint to the harsh post-traumatic hemarthrosis atmosphere. B. Surgical imprecision in recreating the native biomechanical state and/or concurrent removal of injured meniscal tissue (ACL repair does not reduce the occurrence of posttraumatic OA). 2. Chronic mechanical damage is most commonly encountered when misalignment patterns lead to abnormal force distribution and consequent excess burden on focal areas. A. One prototypical example is obesity leading to an increased preponderance of varus knees, which can promote the development of or worsen medial compartment KOA. 3. Genetic predisposition from unfavorable hormonal influence and/or gene modulation, regulation, and expression. A. Aging females with a family history of osteoarthritis are at highest risk. 4. Metabolic syndrome (particularly obesity) is the most common modifiable risk factor (impact cannot be overemphasized taking into consideration this fulminant epidemic). 5. Iatrogenic chondrotoxic effects from intraarticular (IA) corticosteroids and anesthetics (lidocaine, ropivacaine, etc.).[7]

244

4. What other type(s) of arthropathy need to be ruled out and how do they differ from OA?[8] 1. Rheumatoid arthritis (RA) and an effused osteoarthritic joint in a stage of active degradation could be confused if using an indiscriminate approach. RA is an autoimmune disease that typically presents in a distinctive enough fashion to make differentiation clear for the astute physician. RA classically presents with systemic symptoms, symmetrical polyarthritic involvement, erythema, profound calor, pain, and morning stiffness that last longer than 20–30 minutes. It usually occurs in small distal joints, but can affect the knee in roughly 1 in 4 cases. Further dissimilarities are provided in the examination, imaging, and lab sections below. 2. Calcium pyrophosphate dihydrate disease (CPPD), formerly called pseudogout, is a build up of calcium pyrophosphate crystals in the hyaline or fibrocartilage of the joint. It can be confused with gout because of the acute nature of the disease. The knee is involved more than 50% of the time. Both basic calcium phosphate crystals (BCP) and calcium pyrophosphate (CPP) crystals are commonly seen in patients with KOA. Whether the calcification occurs as a result of OA or predisposes one to develop OA is still under investigation.[9] 3. Gouty arthopathy (GA) occurs in as many as 10% of adults aged at least 50 years, but affects the knee much less commonly than CPPD. GA and CPPD may be difficult to clinically distinguish (see lab section below). GA and RA are clinically differentiated by lack of systemic symptoms and unilateral involvement in gout. Additionally, there are major distinctions in fluid and serum analysis. 4. Septic arthritis is an acute monoarticular arthritis that occurs with the introduction of a pathogenic organism into the joint. The infection can be directly introduced into the knee or “seed” from the blood (bacteremia) or a distant site. Rapid destruction is characteristically seen. Examples include inhabitation of the STD, Neisseria gonorrhea (NG), into the joint or the eruption of a Staphylococcus aureus infection inside a prosthetic knee. Joint infections iatrogenically induced by percutaneous injection are very rare.

Chapter 34: Knee osteoarthritis with emphasis on percutaneous regenerative medicine

5. Spondyloarthopathies are also a consideration but usually present with other distinguishing features such as the erythematous scaly rash seen with psoriatic arthritis. OA lacks systemic symptoms and profound erythema, but subtle warmth is possible. Morning symptoms should last no longer than 20–30 minutes and the development of the presence of significant pain is usually more insidious. There is an oligo-arthritic variant of OA where multiple joints are affected, but it usually presents asymmetrically. There are scenarios where bilateral OA occurs, which highlights the importance of combining knowledge with clinical experience. Unlike with OA, disease-modifying agents (DMARDs) exist for RA, including methotrexate and sulfasalazine and more recently allogenic biologics. Symptom mitigating agents such as NSAIDs and glucocorticoids can be utilized during flare-ups. Acute treatment for GA and CPPD is similar and can involve NSAIDs, corticosteroids, or colchicine depending on the circumstances. Chronic treatment for CPPD is similar to acute treatment; on the other hand, a uricosuric agent or urate production inhibitor is initiated within a few weeks for GA. OA treatment is addressed in detail below. The rest of the exam shows an antalgic gait, a subtle genus varus deformity, atrophy of the vastus medialis obliquus, quadriceps (Q) angle of 15 degrees, crepitus with ROM, and a+ bulge sign. There is lowgrade laxity on Lachman’s test and lateral opening upon varus (medial to lateral force application) stress compared to the contralateral side. The hip and foot exam are normal.

5. What does a pertinent exam of the knee consist of in a patient that is suspected of having knee osteoarthritis?[8,10] 1. Gait analysis – those with KOA tend to walk with an antalgic gait, spending less time in stance phase on affected side (assuming the pathology is unilateral). 2. Inspection – genu varus and valgus deformities are indicators of increased predisposition for development of OA or the current presence of frank OA. Unless abnormal hip–knee–ankle

angle contributed to KOA evolution, it may not be seen early in the disease course. The development of a varus knee has a more ominous course because the natural stance phase adduction moment ubiquitously causes greater load medially than laterally.[11] Atrophy of the vastus medialis (VMO) component of the quadriceps and/or a large Q angle can lead to break down of the patella femoral junction (PFJ). VMO weakness results in maltracking of the patella against the femoral trochlea and frictioninduced patella cartilage damage (chondromalacia). 3. Palpation – joint line tenderness (JLT) can be an indicator of disease presence but has a low specificity because it can also be indicative of meniscal or coronary ligament damage or meniscal extrusion with or without the concurrent presence of KOA. Gross deformities from synovial hyperplasia and osteophytes will be more evident in advanced disease. 4. ROM – crepitation on passive ROM testing is a sensitive but non-specific indication of either uni-, bi-, or tricompartmental KOA. It can also signify chondromalacia patella, redundant soft tissue (plica syndrome), or injury to the meniscus. As the disease worsens, ability to both actively and passively range the joint diminishes. 5. Provocative maneuvers – Bulge sign, Lachman’s, anterior/posterior drawer, medial/lateral stability, and McMurray’s test are well described and beneficial to evaluate for effusion, disruption of meniscal integrity, and instability, respectively. Evaluation of the integrity of the fibrocartilage is important because damage to the meniscus is intimately associated with the development of subsequent KOA. It is important to examine the joint above and below the knee and to examine the contralateral knee for individualistic comparison. A PA radiograph shows definite osteophytes and probable medial joint narrowing.

6. What imaging is currently used to diagnose KOA and is it adequate? PA, lateral, and sunrise semi-flexed (30 degree knee bend) standing radiographs have historically been

245

Chapter 34: Knee osteoarthritis with emphasis on percutaneous regenerative medicine

the standard of care in the imaging work-up and diagnosis of KOA. In conjunction with the PA technique, the lateral view can provide additive information about the tibiofemoral joint and both the lateral and sunrise views are utilized to assess the PFJ. Experts are increasingly suggesting that there are likely pre-radiographic and even preimaging (see below) stages of KOA and that KOA should not be ruled out on the basis of a negative x-ray.[12] This is an essential point because there is ample support in the literature that disease in milder stages is more easily treated compared to end-stage disease. Technologic advancements in MR imaging and musculoskeletal ultrasound (MSKUS) are challenging the current standard of care, but have their own limitations.  MSKUS is limited by only being able to detect outer condyle damage due to inability to penetrate bone. It can also visualize synovitis, which can be an indicator of OA (as well as other arthropathies).  MRI is much more sensitive and can detect KOA in the pre-radiographic phase. It also has the ability to detect subchondral bone marrow lesions, which has been highly correlated with pain, rate of degradation, and progression to joint replacement. Unfortunately, more than 75% of asymptomatic individuals can have clinically irrelevant abnormalities on MR imaging.[13] Thus, analogous to other areas of the body, imaging is most advantageous when incorporated into the comprehensive clinical picture. Neither MSKUS nor MRI have yet to be routinely implemented into clinical practice, but there are protocols and classification systems evolving. OA, RA, gout, and CPPD all have distinctive appearances on radiographs. Briefly, in RA, bony erosions, washout osteopenia, and involvement of both the medial and lateral tibiofemoral compartments are all classic, whereas in gout, overhanging edges combined with tophi and bony erosions are virtually pathognomonic. In CPPD, chondrocalcinosis (CC), which is defined by horizontal white lines of CPPD crystals integrating into the articular and fibrocartilage, is characteristically seen. Four cardinal findings are classically described in OA and include osteophytes, subchondral sclerosis and cysts, and

246

unicompartmental joint space narrowing. Further comparisons are beyond the scope of this chapter. The patient states that she has not had any blood work for at least a year and wonders whether this will be beneficial in determining her diagnosis.

7. Are there any labs that need to be ordered when working up a patient with knee pain? A synovial fluid aspirate should be sent for cell counts and crystal analysis if there is a reasonable suspicion for an infectious or crystalline arthropathy (notably gonorrhea is notorious for not exposing itself on routine gram stain and culture). Depending on the scenario, pertinent serum labs include but are not limited to a CBC, ESR, CRP, RF, ANA, and anti-CCP. Further discussion regarding these labs is outside the scope of this chapter. Many believe that biomarkers will eventually be of utility in diagnosing, monitoring, and treating OA by providing the physician with an earlier indicator of OA commencement (pre-imaging phase), prognostic information, and gauge of response to intervention. “Chondrodestructive” marker examples include metalloproteinases and interleukins. Chondroprotective or “regenerative” marker examples are bone morphogenic protein and transforming growth factor beta. “Omics” are being promoted as key in seeing the biomarkers reach clinical pertinency.[3] On return visit, as the patient’s pain specialist contemplates management options, she conveys to him that she had a steroid injection into her knee 6 months ago which resolved her pain for 2 weeks and then it gradually recurred to its previous level. The following options are contemplated:  Refer to an arthroscopic specialist for a “scope and clean out.”  Reinjection of intra-articular corticosteroids (CS) or injection of viscosupplementation (VS) with or without an anesthetic in the mixture.  Refer to an experienced therapist for formal physical therapy.  Refer to a musculoskeletal (MSK) regenerative specialist for consideration of imaging guided regenerative injection therapy.  Refer to a clinical psychologist as part of a multidisciplinary approach.

Chapter 34: Knee osteoarthritis with emphasis on percutaneous regenerative medicine

8. What are the traditional non-surgical treatment options for early KOA? Diet and exercise Implementation of a low calorie, well balanced diet with a concomitant formal exercise plan combines reduced caloric intake and adequate caloric expenditure to reduce joint wear and tear. Notably, exposing a healthy joint to exercise in moderation does not seem to induce the development of OA and can actually impede its progression. Other more invasive weight loss methods, such as gastric bypass surgery, should only be recommended in recalcitrant cases.

Physical therapy Referral to a reputable PT to strengthen supporting musculature and address any muscle imbalances is imperative in the appropriate circumstances. Supporting musculature weakness and/or imbalance can contribute to misalignment (see above).[1]

Pharmacologic intervention Medications can be temporarily beneficial if used judiciously and if no contraindications exist. OTCs like Tylenol and COX inhibitors are tried prior to progressively “climbing the ladder” to tramadol and in end-stage disease other more potent opioid products.[14] The brief use of opioids is also reasonable in mild stages of painful KOA to aid the patient in getting “over the hump.” Glucosamine, chondroitin, MSM, fish oil, vitamin D, and various herbs have also been implicated in maintenance of and improving joint health.

Orthotics Realigning braces with either varus or valgus torques can be beneficial. Alternatively, medial and lateral wedges with and without subtalar strapping can be utilized for lateral and medial KOA, respectively.[15]

Psychology therapy Clinical psychologists can be part of a multidisciplinary approach. KOA patients frequently have insomnia and CBT is beneficial in these cases.[16]

Traditional percutaneous injections Conventional injection treatment consists of a blind, “anatomically guided” injection of CS into the joint space usually in combination with an anesthetic such as 1% lidocaine. The literature is unequivocally clear that its effect vanishes after a period less than a month[17] (see above section for discussion regarding chrondrotoxic concern for CS and anesthetics). VS products are being increasingly utilized and have proven to be a more effective treatment option than CS.[18] Effects typically last at least 6 months and there may be a positive cumulative effect with repeated treatment.[19] VS can be cost prohibitive in uninsured populations.

9. What are the innovative non-surgical management options for early KOA? The “status quo” has chronically resulted in unacceptable outcomes, which frequently culminate with prosthetic joint replacement that inherently come with dangerous risks. As a result, new and revolutionary treatment options are developing and are frequently referred to as regenerative injection therapy (RIT). Examples include dextrose prolotherapy, PRP, and autologous mesenchymal stem cells (MSCs). The terminology for these clinically promising treatments is in a state of flux primarily due to the paucity of knowledge about their biologic mechanism of action(s) and effect(s) on imaging. The term RIT[20] is used because it describes the underlying hypothesized mechanism of placing injectate into and/or adjacent to areas of damaged tissue to induce regeneration of the structure. There is less objective support of DP’s and PRP’s[21] ability to regenerate tissue compared with MSC’s. Notably, if there is sustained reinstatement of functionality and acceptable attenuation of pain, there is debate about the importance of re-obtainment of the joint’s pristine macrostructure state. Studies with long-term outcome measures will be necessary to further clarify this. There is considerable variance in treatment intervals with RITs due to the lack of mechanism knowledge. DP consists of a hyperosmolar dextrose solution (15–25%) that is injected into extra-articular structures and/or the intra-articular joint space. As opposed to PRP/MSC, it has been used for many decades[22] and

247

Chapter 34: Knee osteoarthritis with emphasis on percutaneous regenerative medicine

data supporting DP as an efficacious treatment for numerous types of MSK disorders, including KOA, is growing. Rabago et al recently reported in a three-arm double-blind RCT (N¼90) that DP outperformed normal saline (blinded) injections and PT (unblinded) over a period of 1 year.[23] Another recent KOA trial confirms that there may be a direct effect on pain.[24] Animal model data has suggested that DP strengthens, tightens, and regenerates EA structures via inflammatory mechanisms[22] and thus it may be best utilized primarily for tightening of thin, lax damaged extraarticular MSK structures, which likely indirectly has a positive effect in treating KOA (see above section on acute and chronic mechanical damage). Many feel that DP pioneered the ingenuity of the recent explosion of PRP and MSCs in clinical practice. In fact, EA DP, and IA MSC/PRP are being increasingly employed in combination. PRP is a centrifuged sub-portion of the blood that contains a concentration of platelets at least 2× higher than normal blood.[25] Although a select few are custom processing it in the lab, 99% of PRP is produced using commercial bedside centrifuges by either a single or double spin system. Within the past several years, generically produced PRP has been replaced by formulations that have varying concentrations of not only platelets, but also erythrocytes and leukocytes. Mounting evidence and expert opinion is that hematocrit poor and leukoctye controlled “highconcentration” PRP (> 5–10×) is more optimal than “bloody” PRP.[26] PRP has shown the ability to stimulate proliferation of MSCs and/or improve their chondrogenic differentiation potential.[27] Unpublished research supports the aforementioned formulation as being the most effective in proliferation of bone marrow MSCs (Centeno et al). Adding even more complexity is the decision of whether to “activate” the PRP prior to injection. Utilizing calcium, thrombin, collagen, or the freeze-thaw method prior to injection activates the platelet alpha granules to release precious growth factors that stimulate anabolic activity.[25,26] The prudency of activating IA PRP is not yet clearly defined. Although PRP KOA clinical studies are particularly divergent both in methodology and injectate composition (and many fail to provide essential details about preparation), evidence is virtually unanimously positive. About a dozen studies have culminated with Level I evidence demonstrating the short-term (6 months to 1 year) effectiveness of PRP for KOA.[25,28] Further

248

research will focus on the production of PRP formulations with the most chondrogenic potential to be used with or without MSCs. MSCs are multipotent, adult stem cells that show clinical promise as therapeutic agents in regenerative medicine.[29] They are defined by their ability to selfreplicate and adhere to plastic. Some argue they are more accurately called progenitor cells because of the comparative limited ability for unlimited cell type differentiation in relation to embryonic stem cells. They have been used for years for orthopedic and arthritic treatment purposes[30] with increasingly sophisticated formulation and processing over time. MSCs can be isolated from many anatomic locations, including whole marrow aspirate, muscle biopsy, adipose liposuction aspirate, and synovium. For orthopedic uses, these sources have been compared by many authors for their ability to heal bone and cartilage with differences being noted. As a rule, the closer the source tissue is to the target tissue being treated, the more effective the MSCs appear to be at differentiation into the target tissue. Presently, there are two main sources for the MSC treatment of KOA: bone marrow (BM) and adipose tissue (AT).[29] Significant controversy exists over whether adipose or bone marrow is the better source for orthopedic tissue repair. While adipose MSCs are more prevalent and are capable of orthopedic tissue differentiation, production of orthopedic tissues from this type of cell requires the use of considerably more growth factors and can be deceptively more invasive. In addition, the intrinsic chondrogenic potential of adipose-derived MSCs doesn’t appear to be as robust as bone marrow-derived MSCs.[29] Bench side and animal studies[31] over the past decade are paving the way for blossoming translational clinical research.[30,32] Based on a US National Library of Medicine search there is report of over 1000 patients receiving BM MSCs for arthritic and bone diseases as opposed to less than 10 for AT MSCs. Essentially all published studies are case reports and case series,[32] but randomized controlled trials evaluating the percutaneous effectiveness of MSCs for KOA are beginning to surface (ClinicalTrials.gov Identifier: NCT01504464). See Figure 34.1. Undoubtedly, MSCs are most effective when cultured because of the exponential expansion that results. Based on a systematic literature review of over 800 patients, this has been proven to be safe when

Chapter 34: Knee osteoarthritis with emphasis on percutaneous regenerative medicine

Figure 34.1. 46-year-old white female with chondral lesion who failed arthroscopic debridement. Before FSPGR sagittals (top) with 1 year follow-up after cultured MSC injection (bottom). Same 3.0T MRI scanner and image settings used for both images. (Courtesy of the Centeno-Schultz Clinic.)

performed under a strict protocol that respects innate, built in safety mechanisms.[33,34] For example, a cessation of propagation signal is sent out upon confluence of cells with one another.[29] MSCs can be increased in number by exposure to growth factors like TGF-β and others, which is contained in PRP and platelet lysate.[35] Used in conjunction with augmenting growth factors and/or scaffolds for synergistic purposes and/or maximization of duration of action, autologous or possibly allogenic expanded MSCs discriminately injected in earlier stages of disease using imaging guidance provide the most realistic chance of making dangerous and invasive joint arthroplasty obsolete, except for the most recalcitrant cases. In the USA, the FDA has chosen to label MSCs that are more than minimally manipulated (i.e., cultured or digested fat at the bedside) as a drug with all of the regulations and stipulations associated with new drug development. Their usage is currently strictly prohibited in this country without an Investigational New Drug (IND) exemption and an extensive regulatory process. American MSK MSC specialists are circumventing this prohibition by treating patients with cultured MSCs in other countries with less stringent regulations.[29] Acupuncture and neural therapy are alternative treatments that are not discussed in this chapter.

10. What are the traditional surgical treatment options for early KOA? Most surgical techniques for symptomatic early stages of KOA utilize an arthroscopic approach. Arthroscopic “clean out” and debridement of fraying cartilage and hypertrophied synovial tissue is routinely

performed, but is ill-advised unless utilized in specific circumstances (such as with removing a known loose body or rectifying mechanical locking/catching).[36] Microfracture surgery is a technique that attempts to introduce bone marrow mesenchymal and hematopoietic stem cells into the joint by drilling holes into the central portion of the femoral condyle. It is ineffective in producing hyaline cartilage, but can create a fibrocartilage in areas of damage. There is diminution of success with increasing age and time from procedure.[37] Definitive treatment for end-stage disease is total knee arthroplasty (TKA). TKA is efficacious, especially in more elderly individuals who have become sedentary, but it can be extremely dangerous, with an overall 90-day mortality rate of close to 1%.[37] Frequency of morbidity and mortality increases with increasing age and comorbidities.[38]

11. What are innovative surgical treatment options for KOA? Most of these surgical techniques also use an arthroscopic approach. Microfracture repair (with the addition of PRP, MSC, VS, and other scaffolds), osteochondral autograft system (OATS), and autologous chondrocyte implantation (ACI) make up a preponderance of these options. OATS involves taking a chunk of healthy cartilage from a non-weight-bearing area and sewing it into a hyaline cartilage defect that is suspected to be causing pain. Unfortunately, persistent issues with unsustained integration have plagued this procedure thus far. ACI has proven to be superior to OATS but is most effective when used shortly after symptoms develop. Surgically implanted MSCs are also being explored.[39]

249

Chapter 34: Knee osteoarthritis with emphasis on percutaneous regenerative medicine

Off loading and load redistributing techniques, including valgus osteotomies and tibiofemoral distraction, are not new ideas but have resurfaced as a solitary treatment or combination management with the above-mentioned methods. They appear to be effective and even disease modifying,[40,41] but their lack of practicality (requirement of weeks of nonweight-bearing), concern for complications, and presence of other less invasive alternatives (off loading braces) has resulted in these being performed infrequently. In unicompartmental disease, particularly for patients who are young and healthy, a partial knee replacement is an option. It attenuates recovery time but has poor longevity.[42]

References 1.

2.

3.

4.

5.

6.

250

Neogi T, Zhang Y. Epidemiology of osteoarthritis. Rheum Dis Clin North Am. 2013;39(1):1–19. Pretzel D, Linss S, Rochler S, et al. Relative percentage and zonal distribution of mesenchymal progenitor cells in human osteoarthritic and normal cartilage. Arthritis Res Ther. 2011;13(2):R64. De Ceuninck F, Sabatini M, Pastoureau P. Recent progress toward biomarker identification in osteoarthritis. Drug Discov Today. 2011;16(9–10): 443–449. Sokolove J, Lepus CM. Role of inflammation in the pathogenesis of osteoarthritis: latest findings and interpretations. Ther Adv Musculoskelet Dis. 2013;5(2): 77–94. Finnson KW, Chi Y, Bou-Gharios G, Leask A, Philip A. TGF-b signaling in cartilage homeostasis and osteoarthritis. Front Biosci (Schol Ed). 2012;1(4):251–268. Longo G, Osikowicz M, Ribeiroda-Silva A. Sympathetic fiber sprouting in inflamed joints and adjacent skin contributes to painrelated behavior in arthritis. J Neurosci. 2013;33(24): 10066–10074.

7.

8.

9.

12. Coalescence of surgery and injections Intense debate continues regarding whether surgical or percutaneous will be most advantageous moving forward. Level I evidence proving superiority of one over the other is logistically challenging and cumbersome. Analogous to interventional cardiology reducing the need for coronary artery bypassing grafting via sternotomy in the 1980s to 1990s, the advent of “Interventional Orthopedics,” coined by Centeno et al is primed to provide the best of both worlds. Research suggests that injection techniques, by fluoroscopy or arthroscopy,[43] allow precision placement of injectates; this avoids the risk of anesthesia and surgery.

Farkas B, Kvell K, Czömpöly T, Illés T, Bárdos T. Increased chondrocyte death after steroid and local anesthetic combination. Clin Orthop Relat Res. 2010; 468(11):3112–3120. Imboden J, Hellmann D. Current Diagnosis & Treatment in Rheumatology, 3rd edn (LANGE CURRENT Series). 2013. MacMullan P. Detection of basic calcium phosphate crystals in osteoarthritis. Joint Bone Spine. 2011;78(4):358–363.

observational study (Framingham Osteoarthritis Study). BMJ. 2012;345: e5339. 14. Van Laar M. Pain treatment in arthritis-related pain: beyond NSAIDs. Open Rheumatol J. 2012;6:320–330. 15. Segal NA. Bracing and orthoses: a review of efficacy and mechanical effects for tibiofemoral osteoarthritis. PM R. 2012;4(5 Suppl):S89–96.

10. Abhishek A, Doherty M. Diagnosis and clinical presentation of osteoarthritis. Rheum Dis Clin North Am. 2013;39(1):45–66.

16. Helminen. Effectiveness of a cognitive behavioral group intervention for knee osteoarthritis pain: protocol of a randomized controlled trial. Helminen EEBMC Musculoskelet Disord. 2013;29(14):46.

11. Vincent KR, Conrad BP, Fregly BJ, Vincent HK. The pathophysiology of osteoarthritis: a mechanical perspective on the knee joint. PM R. 2012;4(5 Suppl): S3–9.

17. Bellamy N, Campbell J, Robinson V, et al. Intra-articular corticosteroid for treatment of osteoarthritis of the knee. Cochrane Database Syst Rev. 2005 18;(2):CD005328.

12. Guermazi A. Why radiography should no longer be considered a surrogate outcome measure for longitudinal assessment of cartilage in knee osteoarthritis. Arthritis Res Ther. 2011;13(6):247.

18. Bellamy N, Campbell J, Robinson V, et al. Viscosupplementation for the treatment of osteoarthritis of the knee. Cochrane Database Syst Rev. 2006;19(2):CD005321.

13. Guermazi A. Prevalence of abnormalities in knees detected by MRI in adults without knee osteoarthritis: population based

19. Navarro-Sarabia F Coronel P, Collantes E, et al. A 40-month multicentre, randomised placebocontrolled study to assess the efficacy and carry-over effect of

Chapter 34: Knee osteoarthritis with emphasis on percutaneous regenerative medicine

repeated intra-articular injections of hyaluronic acid in knee osteoarthritis: the AMELIA project. Ann Rheum Dis. 2011;70(11):1957–1962. 20. Vora A. Regenerative injection therapy for osteoarthritis: fundamental concepts and evidence-based review. PM R. 2012;4(5 Suppl):S104–109. 21. Halpern B. Clinical and MRI outcomes after platelet-rich plasma treatment for knee osteoarthritis. Clin J Sport Med. 2013;23(3):238–239. 22. Rabago D, Slattengren A, Zgierska A. Prolotherapy in primary care. Primary Care: Clinics in Office Practice. 2010;37:69–80. 23. Rabago D. Dextrose prolotherapy for knee osteoarthritis: a randomized controlled trial. Ann Fam Med. 2013;11(3):229–237. 24. Rabago D, Kijowski R, Woods M, Patterson JJ. Association between disease-specific quality-of-life and magnetic resonance imaging outcomes in a clinical trial of prolotherapy for knee osteoarthritis. Arch Phys Med Rehabil. 2013;94(11):2075–2082. 25. Kon E, Filardo G, Di Matteo B, Marcacc M. PRP for the treatment of cartilage pathology. Open Orthop J. 2013;7:120–128. 26. Boswell SG, Cole BJ, Sundman EA, Karas V, Fortier LA. Platelet-rich plasma: a milieu of bioactive factors. Arthroscopy. 2012;28(3):429–439. 27. Mifune Y. The effect of plateletrich plasma on the regenerative therapy of muscle derived stem cells for articular cartilage repair. Osteoarthritis Cartilage. 2013; 21(1):175–185. 28. Patel S. Treatment with plateletrich plasma is more effective than placebo for knee osteoarthritis:

a prospective, double-blind, randomized trial. Am J Sports Med. 2013;41(2):356–364. 29. Centeno CJ, Faulkner S. Regenerative orthopedics. In Wislet-Gendebien S, ed. Advances in Regenerative Medicine. InTech. 2011. DOI: 10.5772/25478. Available from:http://www. intechopen.com/books/advancesin-regenerative-medicine/ regenerative-orthopedics. 30. Centeno CJ. Partial regeneration of the human hip via autologous bone marrow nucleated cell transfer: a case study. Pain Physician. 2006;9(3):253–256. 31. Mokbel AN. Homing and reparative effect of intra-articular injection of autologous mesenchymal stem cells in osteoarthritic animal model. BMC Musculoskelet Disord. 2011; 15(12):259. 32. Orozco L. Treatment of knee osteoarthritis with autologous mesenchymal stem cells: a pilot study. Transplantation. 2013; 95(12):1535–1541. 33. Centeno C, Schultz J, Cheever M, et al. Safety and complications reporting on the re-implantation of culture-expanded mesenchymal stem cells using autologous platelet lysate technique. Curr Stem Cell Res Ther. 2010;5:81–93. 34. Peeters CM, Leijs MJ. Safety of intra-articular cell-therapy with culture-expanded stem cells in humans: a systematic literature review. Osteoarthritis Cartilage. 2013;21(10):1465–1473. 35. Jonsdottir-Buch SM, Lieder R, Sigurjonsson OE. Platelet lysates produced from expired platelet concentrates support growth and osteogenic differentiation of mesenchymal stem cells. PLoS One. 2013;11:8(7).

36. Felson DT. Arthroscopy as a treatment for knee osteoarthritis. Best Pract Res Clin Rheumatol. 2010;24(1):47–50. 37. Mithoefer K, McAdams T, Williams RJ, Kreuz PC, Mandelbaum BR. Clinical efficacy of the microfracture technique for articular cartilage repair in the knee: an evidencebased systematic analysis. Am J Sports Med. 2009;37(10): 2053–2063. 38. Singh JA, Kundukulam J, Riddle DL, Strand V, Tugwell P. Early postoperative mortality following joint arthroplasty: a systematic review. J Rheumatol. 2011; 38(7):1507–1513. 39. Roelofs AJ. Cell-based approaches to joint surface repair: a research perspective. Osteoarthritis Cartilage. 2013;21(7):892–900. 40. Intema F, Van Roermund PM, Marijnissen AC, et al. Tissue structure modification in knee osteoarthritis by use of joint distraction: an open 1-year pilot study. Ann Rheum Dis. 2011; 70(8):1441–1446. 41. Gomoll AH. High tibial osteotomy for the treatment of unicompartmental knee osteoarthritis: a review of the literature, indications, and technique. Phys Sportsmed. 2011;39(3):45–54. 42. Schroer WC, Barnes CL, Diesfeld P, et al. The Oxford unicompartmental knee fails at a high rate in a high-volume knee practice. Clin Orthop Relat Res. 2013;471(11):3533–3539. 43. Koga H, Shimaya M, Muneta T, et al. Local adherent technique for transplanting mesenchymal stem cells as a potential treatment of cartilage defect. Arthritis Res Ther. 2008;10(4):R84.

251

Section 4 Chapter

35

Visceral Pain

Patient with chronic abdominal pain from pancreatitis Rodrigo A. Benavides Corder and Jianguo Cheng

Case study A 47-year-old man with a history of alcoholism presents for an evaluation of chronic abdominal pain. He complains of severe abdominal epigastric pain for the past 2 years, which is constant but waxes and wanes throughout the day. It is described as dull and burning and at times it radiates to his back. He associates nausea and vomiting with intense flares of pain. He was diagnosed with abdominal pain from chronic pancreatitis. He has undergone treatment with NSAIDs with no relief.

1. What is the difference between acute and chronic pancreatitis? Chronic pancreatitis is defined as a progressive inflammatory response of the pancreas that results in permanent structural damage and may lead to impairment of exocrine and endocrine function.[1] In contrast with acute pancreatitis, in which there is a transient inflammatory response to pancreatic injury, chronic pancreatitis involves a progressive process. Recurrent episodes of acute pancreatitis may lead to chronic pancreatitis over time, but the two conditions may be differentiated by distinct pathologic findings, etiology, and course (Table 35.1).[2]

2. What is the most common cause of chronic pancreatitis and which demographics are usually affected? The incidence of chronic pancreatitis in the western hemisphere amounts to 10/100 000. It is more common in men at a 3:1 ratio, and present most commonly between the ages of 40 and 50 years. The mortality rate is 30% 10 years after diagnosis.[4] The cause of premature death is usually associated with

Table 35.1. Distinguishing features between chronic and acute pancreatitis

Acute pancreatitis

Chronic pancreatitis

Pathology

Neutrophilic inflammatory reaction

Mononuclear infiltration and fibrosis

Serologic markers

Mostly presents with elevated amylase and lipase

Amylase and lipase tend to be within normal range

Symptoms

Almost always painful crisis

Even though pain and pancreatic insufficiency are the most common symptoms, there may be long asymptomatic periods of time

[1–3]

cardiovascular disease, and lung and esophageal cancer from alcohol and tobacco, rather than surgical complications, multiorgan failure, or sepsis secondary to acute exacerbations.[5] The most common cause of chronic pancreatitis is excessive alcohol abuse, which acounts for 70–80% of cases. Not all alcoholic patients develop pancreatitis, suggesting a possible genetic predisposition. Other causes include autoimmune, hereditary, metabolic (hypercalcemia, hyperlipidemia), or idiopathic.[5–7]

3. Describe the typical presentation of pain in chronic pancreatitis Abdominal pain is present in 90% of patients with chronic pancreatitis, and is the primary cause of

Case Studies in Pain Management, ed. Alan David Kaye and Rinoo V. Shah. Published by Cambridge University Press. © Cambridge University Press 2015.

253

Chapter 35: Patient with chronic abdominal pain from pancreatitis

hospitalization in most patients. The pain is typically epigastric, often radiates to the back, and is occasionally associated with nausea and vomiting. It may be partially relieved by sitting upright or leaning forward, and often worsens 15 to 20 minutes after eating.[4,8,9] Several patterns of presentation have been reported, with 55% of patients referring to some degree of pain at all times, whereas the rest describe it as intermittent. Early in the course of the disease the pain may occur as flares usually lasting less than 10 days, with pain-free intervals ranging from months to up to a year. As the condition progresses the symptoms become more continuous, with prolonged periods of daily pain or clusters of severe pain exacerbations.[4]

4. What is the pathogenesis of pain in chronic pancreatitis? The pathogenesis of pain in chronic pancreatitis is multifactorial, involving central and peripheral pathways. Understanding these pathways may lead to a recognition of possible therapeutic targets and factors that may explain why some of the more traditional surgical treatment methods have not been as successful as anticipated.[8] Nociceptive pain in chronic pancreatitis involves the inflammatory infiltration of sensory nerves that innervate this organ. This implies changes such as ischemia, increased pressure of the pancreatic ducts, and release of bradykinin, prostaglandins, and substance P, which in turn activate nociceptors.[10] The theory of increased pressure in the pancreatic duct secondary to obstruction as a major cause of pain has been the basis to treatment with endoscopic and surgical decompression techniques, but so far these have shown inconsistent results.[11–13] Other structures that may contribute to nociceptive pain in chronic pancreatitis include pseudocyst formation that compresses other neighboring organs, obstruction of the duodenum or ductus choledocus, retroperitoneal infiltration, and gastric or duodenal ulcers.[1] Another significant contributing factor comes from the ongoing interaction of inflammatory cells and nociceptive neurons in the pancreas, which in the long term causes neural injury and remodeling. This produces sensitization of peripheral nociceptive pathways, and the constant influx of nociceptive input can consequently result in central sensitization in the spinal cord and brain. These changes could be responsible for the

254

hyperalgesia and allodynia symptoms that may be persistent despite a total pancreatectomy.[8,14–17]

5. Which other clinical manifestations may contribute to the overall pain experience of patients? Apart from pain the other major clinical features of chronic pancreatitis include symptoms from exocrine glandular insufficiency that leads to malabsorption and limitations to digest complex foods. Fat malabsorption usually occurs prior to protein deficiencies since lipolytic activity decreases faster than proteolysis. Symptoms include steatorrhea and deficiencies in fat-soluble vitamins (A, D, E, K) and vitamin B12.[1,18] Diabetes is another important complication of chronic pancreatitis. It usually presents late in the course of the disease and is insulin dependent. Treating diabetes in chronic pancreatitis may be challenging as these patients also have impaired glucagon synthesis which makes them more prone to hypoglycemia.[18,19] Since diabetes in itself may also produce chronic pain symptoms such as painful diabetic neuropathy, it is important to stress adequate control. Other complications associated to chronic pancreatitis include bile duct or duodenal obstruction, pancreatic ascites, splenic vein thrombosis, pseudocyst formation, pancreatic cancer, and spells of superimposed acute pancreatitis especially in alcoholic patients.[4,20]

6. Which diagnostic studies are of value for diagnosis and staging of chronic pancreatitis? Apart from the classic symptoms of chronic pancreatitis, additional studies may be required to confirm the diagnosis or evaluate the progression of the disease. In contrast with acute pancreatitis, routine laboratory exams such as amylase and lipase serum levels may be completely normal or mildly elevated. The exocrine function of the pancreas may be assessed with the determination of elastase and fecal fat excretion, whereas endocrine function can be monitored through glucose and HbA1c determination.[21,22] Ultrasound and CT scan are the most commonly used imaging studies to demonstrate structural changes in the pancreatic parenchyma, such as calcifications, pseudocysts, and tumors.[23,24] Magnetic resonance

Chapter 35: Patient with chronic abdominal pain from pancreatitis

Table 35.2. Causal treatment for conditions associated with pain in chronic pancreatitis

Disorder

Treatment

Obstruction of the ductus choledocus

Endoscopic stenting or surgical choledochoenterostomy

Pancreatic duct stones presenting with outflow obstruction and dilation

Extracorporeal shock wave lithotripsy or endoscopic retrograde cholangiopancreatography

Pseudocysts causing pain because of their location or increased size

Endoscopic, radiologic, or surgical drainage

Duodenal obstruction

Gastrojejunostomy

Painful chronic pancreas

Figure 35.1. Treatment algorithm for chronic pancreatitis pain management.

Life style modificaons

Pharmacologic treatment according to the WHO ladder

[3,11,27–29]

pancreatography and endoscopic retrograde cholangiopancreatography are utilized to identify pancreatic duct abnormalities such as strictures or dilations. The latter has the advantage of allowing therapeutic interventions if required at the time of the exam. Endoscopic ultrasonography is also very useful for the identification of stones and obtaining biopsies of lesions of concern for malignancy.[23–26]

7. What is the first step in treatment of pain in chronic pancreatitis? Pain control for chronic pancreatitis may be challenging given the complexity of the disease and the surrounding social problems that are usually associated with the patient population. As a general first step in alleviating pain, it is important to recognize if there is an underlying cause for the chronic pancreatitis and treat it to reduce progressive pancreatic damage.[3,11] Some conditions associated to pain in chronic pancreatitis require priority corrective treatment, as early interventions may mitigate disease progression, achieve significant pain control, and preserve pancreatic function. A list of these conditions and management are illustrated in Table 35.2.[27–29]

8. Which are the initial steps in conservative management of chronic pancreatitis? A stepwise approach for the treatment of pain has been proposed as a general recommendation for pain

Gastroenterology assessment for potenal endoscopic/surgical treatment

Radiofrequency nervus splachnicus blockade

Spinal cord smulaon management. Figure 35.1 displays an algorithm adapted from Puylaert et al (2011) summarizing the suggested order of treatment interventions available.[3] Following the stepwise approach, the next intervention to be considered involves life style modifications that may result in pain reduction and improve survival rates. Cessation of alcohol intake is imperative, particularly if this is the cause for pancreatitis. Other interventions include small meals that are low in fat throughout the day, supplementation with medium chain triglycerides, and smoking cessation.[30–32] Pancreatic enzyme supplements have also been traditionally utilized, despite inconclusive results regarding

255

Chapter 35: Patient with chronic abdominal pain from pancreatitis

significant pain reduction. However, the studies that have shown positive results are mainly in patients with less advanced disease (no large pancreatic duct involvement or steatorrhea), women, and those with chronic idiopathic pancreatitis.[32–35]

9. What are the medical treatment options available for treatment of chronic pancreatitis? The WHO 3 step ladder is recommended as the guideline to follow when starting a chronic pancreatitis patient on analgesic medical treatment. The first step in cases of mild pain consists of a non-opioid analgesic. Treatment should start with a single agent and, if insufficient, combination therapy can be started with adjuvant medication. Paracetamol (acetaminophen) is the first choice medication as an initial agent, given its good analgesic properties and low gastrointestinal side effects if given at the recommended dosage. Non-steroidal antiinflammatory drugs (NSAIDs) can also be offered, specifically for nociceptive pain symptoms. The side effect profile of these medications must be weighted given that compared to paracetamol, NSAIDs are associated among others with dyspepsia, gastric ulcerations, and renal toxicity.[1,3,32]

improve neuropathic pain and may also help to treat associated depressive states.[38,39] Pregabalin, an anticonvulsive drug, has rendered positive results specifically for chronic pancreatitis when used as an adjuvant. This agent, along with gabapentin, has also been shown to be effective in the treatment of pain from diabetic neuropathy.[39–41]

12. What are the indications for the use of opioids in chronic pancreatitis? The second step in escalation of pain treatment in chronic pancreatitis is applied to cases of mild to moderate pain and combines a non-opioid analgesic with a weak opioid. In severe cases of pain a strong opioid such as morphine may be prescribed. Longacting medications are generally more effective, but some patients will require the addition of short-acting opioids for breakthrough acute spells. Significant side effects that are particularly relevant to this patient population are nausea and vomiting, constipation, opioid-induced hyperalgesia, and most troublesome, addiction. Most opioids have been shown also to increase pressure in the sphincter of Oddi, but there has been no conclusive data to demonstrate that this increase has an effect on the development or deterioration of chronic pancreatitis.[41–43]

10. Can COX-2 inhibitors be given for long-term pain management in chronic pancreatitis?

13. What is the reasoning behind the use of antioxidants in the treatment of chronic pancreatitis?

Given the increased risk of developing cardiac disease, selective COX-2 inhibitors are not recommended for long-term management of pain. Some studies have shown overexpression of COX-2 in chronic pancreatitis, but further research is required to determine if this translates to improved efficacy in pain control or reduced incidence of related complications such as pancreatic cancer or diabetes.[36,37]

Lower plasma levels of several antioxidants have been demonstrated in patients with chronic pancreatitis; the basis of this treatment is to prevent the oxidative stress that comes from an increased presence of oxygen radicals. Studies evaluating the efficacy of antioxidant therapy in reducing pain in patients with chronic pancreatitis have had conflicting results. Dietary supplements with antioxidants and medications such as allopurinol have been studied but to date there is no clear proof of its effectiveness.[44,45]

11. Which are the most common adjuvants used in medical treatment of chronic pancreatitis? Adjuvant therapy may be used at any point of the WHO ladder if monotherapy is insufficient. Tricyclic antidepressants such as amitriptyline and nortriptyline may be useful as these agents have been shown to

256

14. When are interventional options indicated and which procedures can be used? If pharmacologic treatment fails to provide adequate pain relief, the next option to consider is a more

Chapter 35: Patient with chronic abdominal pain from pancreatitis

interventional approach which consists of procedures such as celiac plexus block, splanchnic nerve radiofrequency ablation, or spinal cord stimulators. The sympathetic innervation of the abdominal viscera travels through the anterolateral horn in the spinal cord, the preganglionic fibers from T5 to T12 run through the sympathetic chain and coalesce forming the splanchnic nerves (splanchnicus major, minor, and imus). These nerves course along the paravertebral border and at the level crus of the diaphragm they join with vagal preganglionic parasympathetic fibers, sensory fibers of the phrenic nerve, and postganglionic sympathetic fibers to form the celiac plexus. This plexus wraps around the anterior abdominal aorta; in this structure the preganglionic fibers form ganglia in which they synapse, and the postganglionic fibers will then innervate organs including the pancreas, stomach, liver, and gallbladder.[1,3,46] The celiac plexus block can be performed percutaneously or under endosonographic guidance. The specific approach does not appear to have a major influence on clinical outcomes.[47,48] Local anesthetic alone or in combination with a steroid is usually injected. Results from clinical trials involving these techniques are mostly retrospective case series and have reported marginal benefit.[47–50] These blocks have been shown to provide significant short-term pain relief (weeks to months), but for extended symptom control repeated therapy will be required; the down side of it is that effectiveness of these blocks may decrease with time.[47–51] The most common side effects include transient hypotension that can be easily treated with IV fluids or diarrhea. The injection of neurolytic substances such as alcohol is generally not recommended for treatment of non-terminal cases given the risk for severe complications such as paraplegia from the injection into the arteries supplying the spinal cord.

15. Whatis the evidence behindthe use of radiofrequency ablation and spinal cordstimulationfortreatmentofpain in chronic pancreatitis? Implementing the grading strength of recommendation and quality of evidence scores from Guyatt et al, both the splanchnic nerve radiofrequency block and spinal cord stimulator implant are considered as a 2C+ grade recommendation. (Effectiveness only

demonstrated in observational studies, given that there is no conclusive evidence of the effect, benefits must be closely balanced with risks and burdens.)[3,52] The use of splanchnic nerve blocks for the treatment of upper abdominal pain has been described mostly in case series. In one study 40% of patients undergoing this procedure reported significant pain relief of up to 50% improvement at 6 months; in another study patients were followed for a mean period of 18 months, and obtained not only significant improvement in pain scores but also a decrease in the need for opioids and number of acute hospitalizations.[53,54] The most recent is a retrospective case series of 11 patients with chronic pancreatitis, in which the splanchnic nerve radiofrequency ablation provided significant relief in their pain score for a median period of 45 weeks; five patients underwent a second procedure with an effect comparable to the first intervention.[55] Side effects reported for this procedure include transient hypotension and diarrhea, post-procedural neuritis which can be medically treated, and as in all procedures in the thoracic level, pneumothorax is always a possible complication, for which a post-procedure control radiograph is recommended. The use of spinal cord stimulation as a treatment option for abdominal pain has also been mostly described in retrospective case studies. Patients may undergo differential and/or celiac plexus block beforehand to determine if adequate mapping of most of their pain source is possible. Once the stimulator is planned for implant, the lead tips are mostly positioned at the level of T5. After undergoing a successful lead implant trial, most patients in these studies described at least 50% improvement in pain control, with a decrease in long-term opioid use and improved quality of life markers.[56–59] The main complications of this procedure include migration or lead breakage and infections such as simple local cellulitis and epidural abscesses.

16. What are the endoscopic and surgical treatment options available? The aim of endoscopic treatment in patients with chronic pancreatitis is to provide adequate drainage of the pancreatic duct by decompressing the duct and restoring the outflow of pancreatic secretions.[11] Such procedures include extracorporeal shock wave lithotripsy, endoscopic retrograde cholangiopancreatography with sphincterotomy, and pancreatic duct

257

Chapter 35: Patient with chronic abdominal pain from pancreatitis

stenting. It is important to note that to date, there has been no correlation shown between pain and the presence of intraductal stones, and there are no guidelines to determine the level of obstruction that would require endoscopic treatment. The effects of these treatments are often good in the short term, but the effectiveness decreases with time, and after 5 years of endoscopic treatment only 14% of patients are still pain free.[60,62] The most common indication for surgery in chronic pancreatitis is intractable pain. Other indications include suspicion of neoplasm or local complications in adjacent organs. Conventionally patients are referred for surgical intervention after a long period of

References 1.

Steer ML, Waxman I, Freedman S. Chronic pancreatitis. N Engl J Med. 1995;332(22):1482–1490.

2.

Mariani A, Testoni PA. Is acute recurrent pancreatitis a chronic disease? World J Gastroenterology. 2008;14:995–998.

3.

Puylaert M, Kapural L, Van Zundert J. Pain in chronic pancreatitis. Pain Practice. 2011;11(5):492–505.

4.

Ammann RW, Muellhaupt B. The natural history of pain in alcoholic chronic pancreatitis. Gastroenterology. 1999;116(5): 1132–1140.

5.

Behrman SW, Fowler ES. Pathophysiology of chronic pancreatitis. Surg Clin North Am. 2007;87:1309–1324.

6.

Apte MV, Pirola RC, Wilson JS. Molecular mechanisms of alcoholic pancreatitis. Dig Dis. 2005;23:232–240.

7.

Lee JK, Enns R. Review of idiopathic pancreatitis. World J Gastroenterol. 2007;13:6296–6313.

8.

Pasricha PJ. Unraveling the mystery of pain in chronic pancreatitis. Nat Rev Gastroenterol Hepatol. 2012;9:140–151.

9.

Thuluvath PJ, Imperio D, Nair S. Chronic pancreatitis: longterm

258

medical pain management and endoscopic interventions. Up to 40% to 75% of patients with chronic pancreatitis will require surgery for pain in the course of their disease, but there has been no consensus toward the best timing for these procedures.[61,62] The surgical strategies available can be divided into three main groups: drainage procedures (lateral pancreaticojejunostomy), resectional procedures (pancreaticoduodenectomy, distal, or total pancreatectomy), and procedures combining drainage and resection (Frey procedure, Berger procedure). Most studies have shown improved effectiveness in pain control compared to endoscopic procedures with 40% of patients pain free after 5 years of the intervention.[11,60–63]

pain relief with or without surgery, cancer risk and mortality. J Clin Gastroenterol. 2003;36:159–165. 10. Kawabata A, Matsunami M, Sekiguchi F. Gastrointestinal roles for proteinase-activated receptors in health and disease. Br J Pharmacol. 2008;153(1): S230–S240. 11. Issa Y, van Santvoort HC, van Groor H. Surgical and endoscopic treatment of pain in chronic pancreatitis: a multidisciplinary update. Digest Surg. 2013;30:35–50. 12. Ebbehoj N, Borly L, Madsen P. Pancreatic tissue fluid pressure during drainage operations for chronic pancreatitis. Scand J Gastroenterol. 1990;25:1041–1045. 13. Manes G, Buchler M, Pieramico O. Is increased pancreatic pressure related to pain in chronic pancreatitis? Int J Pancreatol. 1994;15:113–117. 14. Ceyhan GO, Bergmann F, Kadihasanoglu M, et al. Pancreatic neuropathy and neuropathic pain: a comprehensive pathomorphological study of 546 cases. Gastroenterology. 2009;136:177–186. 15. Dimcevski G, Sami SA, FunchJensen P, et al. Pain in chronic pancreatitis: the role of

reorganization in the central nervous system. Gastroenterology. 2007;132:1546–1556. 16. Olesen SS, Frokjaer JB, Lelic D. Pain associated adaptive cortical reorganization in chronic pancreatitis. Pancreatology. 2011;10:742–751. 17. Vera-Portocarrero L, Westlund KN. Role of neurogenic inflammation in pancreatitis and pancreatic pain. Neurosignals. 2005;14:158–165. 18. Mergener K, Baillie J. Chronic pancreatitis. Lancet. 1997;350:1379–1385. 19. Malka D, Hammel P, Sauvanet A, et al. Risk factors for diabetes mellitus in chronic pancreatitis. Gastroenterology. 2000;119:1324–1332. 20. Ammann RW, Akovbiantz A, Largiader F, Schueler G. Course and outcome of chronic pancreatitis: longitudinal study of a mixed medical-surgical series of 245 patients. Gastroenterology. 1984;86:820–828. 21. Lieb JG, Dagranov PV. Pancreatic function testing: here to stay for the 21st century. World J Gastroenterology. 2008;14:3149–3158. 22. Keim V, Teich N, Moessner J. Clinical value of a new fecal elastase test for detection of

Chapter 35: Patient with chronic abdominal pain from pancreatitis

chronic pancreatitis. Clin Lab. 2003;49:209–215. 23. Graziani R, Tapparelli M, Malago R. The various imaging aspects of chronic pancreatitis. JOP. 2005;6:73–88. 24. Kim DH, Pickhardt PJ. Radiologic assessment of acute and chronic pancreatitis. Surg Clin North Am. 2007;87:1341–1358. 25. Kahl S, Glasbrenner B, Leodolter A, et al. EUS in the diagnosis of early chronic pancreatitis: a prospective follow-up study. Gastrointest Endosc. 2002;55:507–511. 26. Bozkurt T, Braun U, Leferink S, et al. Comparison of pancreatic morphology and exocrine functional impairment in patients with chronic pancreatitis. Gut. 1994;35:1132–1136. 27. Dumonceau JM, Delhaye M, Tringali A. Endoscopic treatment of chronic pancreatitis: European Society of Gastrointestinal Endoscopy Clinical Guideline. Endoscopy. 2012;44:748–800. 28. Adler DG, Lichtenstein D, Baron TH, et al. The role of endoscopy in patients with chronic pancreatitis. Gastrointest Endosc. 2006;63:933–937. 29. Parsi MA, Stevens T, Lopez R, Vargo JJ. Extracorporeal shock wave lithotripsy for prevention of recurrent pancreatitis caused by obstructive pancreatic stones. Pancreas. 2010;39:153–155. 30. Shea JC, Bishop MD, Parker EM, et al. An enteral therapy containing medium-chain triglycerides and hydrolyzed peptides reduces postprandial pain associated with chronic pancreatitis. Pancreatology. 2003; 3(1):36–40. 31. Maisonneuve P, Lowenfels AB, Müllhaupt B, et al. Cigarette smoking accelerates progression of alcoholic chronic pancreatitis. Gut. 2005;54:510–514. 32. Singh VV, Toskes PP. Medical therapy for chronic pancreatitis

pain. Curr Gastroenterol Rep. 2003;5:110–116. 33. Brown A, Hughes M, Tenner S, Banks PA. Does pancreatic enzyme supplementation reduce pain in patients with chronic pancreatitis: a meta-analysis. Am J Gastroenterol. 1997;92:2032–2035. 34. Slaff J, Jacobson D, Tillman CR, et al. Protease-specific suppression of pancreatic exocrine secretion. Gastroenterology. 1984;87:44–52. 35. Isaksson G, Ihse I. Pain reduction by an oral pancreatic enzyme preparation in chronic pancreatitis. Dig Dis Sci. 1983;28:97–102. 36. Schlosser W, Schlosser S, Ramadani M. Cyclooxygenase-2 is overexpressed in chronic pancreatitis. Pancreas. 2002;25:26–30. 37. Takahashi M, Mutoh M, Ishigamori R. Involvement of inflammatory factor in pancreatic carcinogenesis and preventive effects of anti-inflammatory agents. Semin Immunopathol. 2013;23(2):203–227. 38. Gilron I, Bailey JM, Tu D, et al. Nortriptyline and gabapentin, alone and in combination for neuropathic pain: a double-blind, randomised controlled crossover trial. Lancet. 2009;374:1252–1261. 39. Fioramonti J, Bueno L. Centrally acting agents and visceral sensitivity. Gut. 2002;51(1):91–95. 40. Olesen SS, Bouwense SA, WilderSmith OH, et al. Pregabalin reduces pain in patients with chronic pancreatitis in a randomized, controlled trial. Gastroenterology. 2011;141:536–543. 41. Gachago C, Draganov PV. Pain management in chronic pancreatitis. World J Gastroenterol. 2008;14:3137–3148. 42. Nusrat S, Yadav D, Bielefeldt K. Pain and opioid use in chronic pancreatitis. Pancreas. 2012;41(2): 264–270.

43. Helm JF, Venu RP, Geenen JE. Effects of morphine in the human sphincter of Oddi. Gut. 1988;29:1402–1407. 44. Siriwardena AK, Mason JM, Sheen AJ, et al. Antioxidant therapy does not reduce pain in patients with chronic pancreatitis: the ANTICIPATE study. Gastroenterology. 2012;143:655–663. 45. Banks PA, Hughes M, Ferrante M, et al. Does allopurinol reduce pain of chronic pancreatitis? Int J Pancreatol. 1997;22:171–176. 46. Loukas M, Klaassen Z, Merbs W. A review of the thoracic splanchnic nerves and celiac ganglia. Clin Anat. 2010;23 (5):512–522. 47. Gress F, Schmitt C, Sherman S, et al. A prospective randomized comparison of endoscopic ultrasound- and computed tomography-guided celiac plexus block for managing chronic pancreatitis pain. Am J Gastroenterol. 1999;94:900–905. 48. Kawamata M, Ishitani K, Ishikawa K, et al. Comparison between celiac plexus block and morphine treatment on quality of life in patients with pancreatic cancer pain. Pain. 1996;64:597–602. 49. Eisenberg E, Carr DB, Chalmers TC. Neurolytic celiac plexus block for treatment of cancer pain: a meta-analysis. Anesth Analg. 1995;80:290–295. 50. Gress F, Schmitt C, Sherman S, et al. Endoscopic ultrasoundguided celiac plexus block for managing abdominal pain associated with chronic pancreatitis: a prospective single center experience. Am J Gastroenterol. 2001;96:409–416. 51. Kaufman M, Singh G, Das S. Efficacy of endoscopic ultrasound-guided celiac plexus block and celiac plexus neurolysis for managing abdominal pain associated with chronic pancreatitis and pancreatic

259

Chapter 35: Patient with chronic abdominal pain from pancreatitis

cancer. J Clin Gastroenterol. 2010;44:127–134.

patients. Pain Pract. 2013. doi: 10.1111/papr.12030.

52. Guyatt G, Gutterman D, Baumann MH. Grading strength of recommendations and quality of evidence in clinical guidelines: report from an American college of chest physicians task force. Chest. 2006;129:174–181.

56. Kapural L, Cywinski J, Sparks D. Spinal cord stimulation for visceral pain from chronic pancreatitis. Neuromodulation: Technology at the Neural Interface. 2011. doi: 10.1111/j.15251403.2011.00381.x.

53. Raj PP, Sahinler MD, Lowe M. Radiofrequency lesioning of splanchnic nerves. Pain Pract. 2002;2:241–247.

57. Khan I, Raza S, Khan E. Application of spinal cord stimulation for the treatment of abdominal visceral pain syndromes: case reports. Neuromodulation. 2005;8:14–27.

54. Garcea G, Thomasset S, Berry DP. Percutaneous splanchnic nerve radiofrequency ablation for chronic abdominal pain. ANZ J Surg. 2005;75:640–644. 55. Verhaegh BP, van Kleef M, Geurts JW. Percutaneous radiofrequency ablation of the splanchnic nerves in patients with chronic pancreatitis: results of single and repeated procedures in 11

260

58. Kapural L, Nagem H, Tlucek H. Spinal cord stimulation for chronic visceral abdominal pain. Pain Med. 2010;11: 347–355. 59. Kapural L, Deer T, Yakovlev A. Technical aspects of spinal cord stimulation for managing chronic visceral abdominal pain: the

results from the national survey. Pain Med. 2010;11:685–691. 60. Maydeo A, Soehendra N, Reddy N. Endotherapy for chronic pancreatitis with intercanal stones. Endoscopy. 2007;39: 653–658. 61. Dite P, Ruzicka M, Zboril V. A prospective, randomized trial comparing endoscopic and surgical therapy for chronic pancreatitis. Endoscopy. 2003;35:553–558. 62. Layer P, Yamamoto H, Kalthoff L. The differential courses of early and late onset idiopathic and alcoholic chronic pancreatitis. Gastroenterology. 1994;107: 1481–1487. 63. Cahen DL, Gouma DJ, Nio Y. Endoscopic versus surgical drainage of the pancreatic duct in chronic pancreatitis. N Engl J Med. 2007;356:676–684.

Section 4 Chapter

36

Visceral Pain

Patient with chronic pelvic pain from endometrial fibrosis Maged Guirguis and Jianguo Cheng

Case study A 27-year-old female presents with chronic pelvic pain for 10 years. She describes a deep, squeezing, burning, boring pain of gradual onset in the lower pelvic region, worsened with movement and menses. In the last 2 years, her pain has escalated to persistent daily pelvic pain. A diagnostic laparoscopy revealed endometriosis.

1. What is chronic pelvic pain? Chronic pelvic pain is a symptom that can affect both women and men. CPP in men is referred to as chronic prostatitis/chronic pelvic pain syndrome (CP/CPPS) and is also known as chronic non-bacterial prostatitis. CPP in women is more complex. Most women experience pelvic pain at some time in their lives. The duration of pelvic pain considered to be chronic varies from 3 months to more than 6 months in the literature, and the location and pathology of the pain are largely unspecified.[1] The American Congress of Obstetricians and Gynecologists defines chronic pelvic pain as non-cyclical pain of at least 6 months’ duration that appears in locations such as the pelvis, anterior abdominal wall, lower back, or buttocks, and that is serious enough to cause disability or lead to medical care.[2] According to the CDC, chronic pelvic pain (CPP) accounted for approximately 9% of all visits to gynecologists in 2007. In addition, CPP is the reason for 20–30% of all laparoscopies in adults. The causes of CPP are not well understood and may be associated with gynecologic (e.g., endometriosis) and nongynecologic (e.g., irritable bowel syndrome [IBS]) conditions. Diagnosis of an underlying cause can be difficult because the pain is rarely associated with a single underlying disorder or contributing factor. Endometriosis accounts for about one-third of the cases of CPP in women. It is a chronic and progressive

condition characterized by the presence of endometrial tissue outside the uterus, which causes a chronic inflammatory process and a tendency for adhesion formation.[3–6]

2. Describe the epidemiology and prevalence of endometriosis Endometriosis is an estrogen-dependent inflammatory disease that occurs in women of reproductive age, and generally becomes inactive with menopause, unless a woman uses postmenopausal hormone therapy.[7] It affects approximately 10% of women in the general population with a peak incidence in the 25–30 age range.[8] It is one of the most common causes of CPP and infertility in reproductive-age women across all ethnic and socioeconomic backgrounds. Among women who undergo laparoscopy, endometriosis is found in one-third of cases. Among women with pelvic pain, a much higher prevalence of endometriosis of up to 82% occurs.[9] In the USA from 1990 to 1998, endometriosis was the third most common gynecologic diagnosis noted in the hospital discharge records of reproductive-age women.[7] CPP often debilitates women with endometriosis for years, has a high risk of emergency department visits, and is associated with time lost from work and significant physical and social debility. CPP costs approximately $881.5 million per year and is the cause of approximately 10% to 15% of hysterectomies in the USA.[8]

3. What is the pathophysiology of endometriosis? The pathophysiology of endometriosis remains poorly understood. The potential causes include: (a) retrograde menstruation (Sampson’s theory);[10] (b)

Case Studies in Pain Management, ed. Alan David Kaye and Rinoo V. Shah. Published by Cambridge University Press. © Cambridge University Press 2015.

261

Chapter 36: Patient with chronic pelvic pain from endometrial fibrosis

coelomic metaplasia;[11] (c) vascular and lymphatic spread;[12] and (d) altered immunosurveillance.[13] Retrograde menstruation is among the most widely accepted causes. Based on this theory, viable endometrial cells are shed through the fallopian tubes into the peritoneum during menstruation, accumulate in the dependent portions of the pelvis, and attach to pelvic walls and organs. The risk factors for developing endometriosis may include uterine outflow obstruction and conditions that predispose women to retrograde menstruation and increased exposure to menstrual flow such as early menarche, reduced parity, longer duration of menstrual periods, and shorter cycle length.[7] The risk appears to decrease with personal habits that relate to decreased estrogen levels (e.g., smoking, exercise).

4. What is the mechanism of pain associated with endometriosis? Pain may be due to nociceptive, inflammatory, or neuropathic mechanisms, and probably all three of these mechanisms are relevant to endometriosisassociated CPP. There is an increased synthesis of cytokines, nerve growth factor (NGF), prostaglandins, metalloproteinases, and estrogen when endometrial tissue attaches to mesothelial cells in endometriosis. This may create an increased production of factors that promote nerve sprouting and peripheral sensitization. Endometrial lesions may also produce pain by directly compressing or infiltrating nociceptors. Furthermore, pelvic pain may be related to the nociceptive nerve fibers in the proliferated blood vessels in endometriosis tissue. Blood vessels are innervated by sensory and sympathetic fibers. Direct innervation of ectopic endometrial growths by sensory and sympathetic fibers has been shown in women with endometriosis. Also small diameter, unmyelinated nerve fibers have been identified as nociceptive in the functional layer of the endometrium of women with endometriosis. Similar nerves have been noted in ectopic endometriotic lesions. Persistent nociceptive input from endometriotic lesions may lead to central sensitization via increased responsiveness of spinal cord dorsal horn neurons processing input from the implants and affected adjacent viscera. Increased excitability of viscero-visceral convergent neurons to the spinal cord has been associated with persistent neuropathic pain and hyperalgesia in this setting.[14]

262

5. How is endometriosis presented clinically? Most ectopic endometrial deposits grow on the ovaries, peritoneum, uterosacral ligaments, pouch of Douglas, and retrovaginal septum, often causing pain as the primary clinical presentation or infertility as the second most common presentation. The symptoms vary in their presentation and severity; however, the most common symptom is pelvic pain. In addition, dysmenorrhea, deep pelvic dyspareunia (pain with sexual intercourse), dyschezia (difficulty or pain with defecation), pain with micturition, low back pain, and infertility are some of the other presentations. Pelvic pain is cyclic and tends to increase in severity in the premenstrual period. It may be unilateral or bilateral and often decrease in severity after menses. However, some women may experience a constant non-cyclic debilitating pain that interferes with their daily lives. For unclear reasons the pain severity might not correlate well with severity of the pathology. Therefore, severe disease may go undiagnosed.[15] Irritable bowel syndrome, painful bladder syndrome, fibromyalgia, and migraine headache all have a higher chance to involve women with endometriosis. In addition, women experiencing chronic pain associated with endometriosis may be prone to depression, anxiety, and chronic fatigue as seen with chronic fatigue syndrome. Due to the diffuse nature of endometriotic lesions, the physical examination is commonly unrevealing. Bimanual palpation of the pelvic structures is of limited specificity in either localization or diagnosis of endometriosis. Tender nodules may be palpable along the uterosacral ligaments or within the recto-uterine cul-de-sac of Douglas, particularly if the exam is done just before menses. Performing a systematic rectovaginal examination is critical because the pathology is found in the dependent areas of the pelvis.[16]

What is the differential diagnosis of chronic pelvic pain in females?[17] Potential differential diagnoses of endometriosis that might cause CPP include:  Gynecologic: adenomyosis, neoplasms of the ovary, pelvic adhesions, pelvic inflammatory disease (PID), ovarian cysts, uterine leiomyoma (fibroids).

Chapter 36: Patient with chronic pelvic pain from endometrial fibrosis

 Urologic: urologic interstitial cystitis, recurrent urinary tract infection (UTI), and chronic urethral syndrome.  Gastrointestinal: gastrointestinal IBS, inflammatory bowel disease (IBD), diverticular colitis, chronic intermittent bowel obstruction, chronic constipation, and carcinoma.  Musculoskeletal: fibromyalgia and pelvic floor tension myalgia.  Neurologic: neuralgia, neuropathic pain, psychologic somatization, depression, and sleep disorders.

How to establish the diagnosis of CPP due to endometriosis? There are many barriers to confirm the diagnosis of CPP due to endometriosis. The clinical presentation is often variable between patients. The correlation between clinical presentation and surgical findings is often unreliable. Women may delay seeking medical attention in the mistaken belief that painful symptoms are part of a normal menstruation, especially if there is a family history of “difficult periods,” or fears of appearing unable to cope with “female problems.” Physicians may trivialize symptoms, attribute pain to being a normal part of menstruation, or dismiss symptoms as being imaginary. This failure of validation often delays referrals to gynecologists or specialists in chronic pain management. Endometriosis should be considered especially if patients present with dysmenorrhea after having previous pain-free menstrual cycles. CT, MRI, and ultrasonography have a high rate of false-positive diagnoses of structures such as blood vessels, small bowel, ovarian lesions, tubal ovarian abscess, and carcinoma. Therefore, these imaging techniques are not specific or sensitive enough to be a cost-effective preoperative diagnostic option. Diagnostic laparoscopy for visualization and biopsy of lesions is the gold standard for diagnosis. It helps rule out other causes of pelvic pain, such as upper genital tract infection, adenomyosis, and pelvic inflammatory disease. Non-invasive methods for predicting disease prior to surgery are only moderately successful.[18] Not all endometriosis lesions are seen at laparoscopy; lesions may be hidden by pelvic organs or adhesions, microscopic or similar in appearance to other malignant or benign lesions. Therefore, techniques such as peritoneal fluid aspiration, lysis of adhesive disease,

mobilization of adherent structures from the pelvic wall, and biopsy of lesions are typically performed to thoroughly examine the cul-de-sac and pelvic organs and confirm the presence of endometriosis. Histopathologic confirmation of endometriosis is established by the microscopic identification of ectopic endometrial epithelium and stroma, often with fibrosis and infiltration of hemosiderin-laden macrophages within the ectopic lesions. These pathognomonic features are documented in only approximately 70% of clinically suspicious cases.[19]

How to manage CPP due to endometriosis? The treatment of CPP is particularly challenging because it shares the features of nociceptive, inflammatory, and neuropathic pain in both the visceral and somatic structures with complex sensory and motor innervations. Management plans typically start with conservative treatment and progress to more invasive interventions. The treatment may consist of two elements, CPP itself as a diagnosis, and endometriosis as disease. Current therapy aims to reduce the pain and to address the pathology to delay recurrence for as long as possible.[20] Between 10% and 20% of women with endometriosis have a recurrence of the disease regardless of the treatment they receive.

Treatment of endometriosis Hormonal treatments The goal is to reduce the amount of estrogen in the body so that menstruation and the resultant internal bleeding and irritation are interrupted or decreased. This method is highly effective for reducing the majority of the implants and inflammation. The major hormonal drugs used can be classified into five main groups: 1. Combined oral contraceptives: The duration of the therapy is between 6 and 12 months or it can be used indefinitely if the patient is satisfied with symptom relief and doesn’t want to become pregnant. About 75% of patients get relief. Potential side effects are irregular bleeding, weight gain, headache, thrombophlebitis, pulmonary embolism, hypertension, neuro-occular lesions, cerebral thrombosis and hemorrhage, benign liver tumors, and gallbladder disease. 2. Progestational agents: Medroxyprogesterone. The duration and effectiveness are similar to combined oral contraceptives. Potential side effects include

263

Chapter 36: Patient with chronic pelvic pain from endometrial fibrosis

weight gain, headache, abdominal discomfort, and irregular bleeding. 3. Gonadotropin releasing hormone (GnRH) analogs: Leuprolide, gosarelin, nafarelin. Side effects are hot flashes, vaginal dryness, emotional lability, loss of libido, and mild degree of bone loss during treatment which might not be reversible. Although useful in the management of chronic pain with endometriosis, GnRH analog treatment has been limited to 6 months by the US FDA due to hypoestrogenic side effects. 4. Gonadotropin inhibitors: Danazol. Treatment is usually not more than 6 months because of side effects. However, it can be extended to 9 months in severe cases. Potential side effects are headaches, flushing, sweating, atrophic vaginitis, acne, edema, hirsutism, deepening of voice, amenorrhea, and weight gain. 5. Aromatase inhibitors: Anastrozole and Letrozole. Side effects include mild headache, nausea, diarrhea, mild hot flashes, and increased risk of osteoporosis with long-term use. They are used only in conjunction with contraceptives. Treatment limited to 6 months due to decrease in bone mineral density. Surgical treatment Surgical removal of lesions, adhesions and cysts via excision, electrocautery, laser vaporization and restoration of pelvic anatomy is the preferred method of treatment for infertile patients with severe endometriosis. The recurrence rate of endometriotic lesions after 5 years is 19% with laparoscopic removal of lesions and 10% with hysterectomy and bilateral oophorectomy; this is compared with 53.4% with medical treatment.

Managing CPP associated with endometriosis Many women still have a recurrence of pelvic pain after medical and surgical management of endometriosis. The recurrence of pain may be due to remodeling of CNS (some of which occurred before surgery), role of reproductive tract in reactivating pain, and incomplete removal or recurrence of lesions.[21] Medical management A variety of analgesics, including non-steroidal antiinflammatory drugs (NSAIDs) have been used. According to a recent review, the effectiveness of NSAIDs (e.g., naproxen) in managing pain caused by endometriosis is inconclusive, and there was no evidence that a particular NSAID class was more effective. In some cases opioid therapy may be initiated to control episodes of severe pain. However, given the high profile of side effects of opioids, it is generally not recommended for prolonged use. Tricyclic antidepressants, dual-reuptake inhibitors of serotonin and norepinephrine, and antiepileptic drugs such as the calcium channel α(2)-δ ligands (e.g., gabapentin and pregabalin) may be useful. It is important to know that most patients must discontinue medical treatment of endometriosis to achieve a pregnancy. Endometriotic lesions and the associated symptoms unfortunately recur when medical treatment is discontinued and estrogen levels increase during the follicular phase of the menstrual cycle. Interventional and surgical management Procedures may be performed for diagnostic and/ or therapeutic purposes. An interventional algorithm for chronic pelvic pain has been formulated Figure 36.1. Superior hypogastric plexus block. From personal files of Rinoo V. Shah, MD, MBA.

264

Chapter 36: Patient with chronic pelvic pain from endometrial fibrosis

to target the visceral and somatic organs and their innervations:[14] – Trigger point injection/botulinum toxin. – Peripheral nerve block (ilioinguinal/ genitofemoral/pudendal). – Epidural steroid injection (thoracic/lumbar/ caudal). – Sympathetic nerve block (hypogastric/ ganglion impar). – Spinal cord stimulator: an advanced treatment option for patients who have failed conservative management.

References 1.

Trioloa O, Laganàa AS, Sturlese E. Chronic pelvic pain in endometriosis: an overview. J Clin Med Res. 2013;5(3):153–163.

2.

ACOG practice bulletin; No. 51. March 17, 2004.

3.

Mathias SD, Kuppermann M, Liberman RF, Lipschutz RC, Steege JF. Chronic pelvic pain: prevalence, health-related quality of life, and economic correlates. Obstet Gynecol. 1996;87:321–327.

4.

5.

6.

7.

Cheong YC, Smotra G, Williams AC. Non-surgical interventions for the management of chronic pelvic pain. Cochrane Database Syst Rev. 2014;3:CD008797. doi: 10.1002/14651858.CD008797. Stones RW, Mountfield J. Interventions for treating chronic pelvic pain in women. Cochrane Database Syst Rev. 2000;(4): CD000387.

– Intrathecal pump: careful selection of patients is very important. Most importantly, these patients should have cleared an independent psychologic evaluation prior to consideration of this therapy. – Surgical interventions: Beside surgical management of endometriosis mentioned above, presacral neurectomy (superior hypogastric plexus excision), paracervical denervation (laparoscopic uterine nerve ablation [LUNA]), and uterovaginal ganglion excision (inferior hypogastric plexus excision) may also be considered to attempt to relieve chronic pelvic pain.

8.

Hasson HM. Incidence of endometriosis in diagnostic laparoscopy. J Reprod Med. 1976;16:135–140.

9.

Neis KJ, Neis F. Chronic pelvic pain: cause, diagnosis and therapy from a gynaecologist’s and an endoscopist’s point of view. Gynecol Endocrinol. 2009;25 (11):757–761.

10. Sampson JA. Metastatic or embolic endometriosis, due to the menstrual dissemination of endometrial tissue into the venous circulation. Am J Pathol. 1927;3:93–110. 11. Mounsey AL, Wilgus A, Slawson DC. Diagnosis and management of endometriosis. Am Fam Phys. 2006;74(4):594–600. 12. Shaw RW. An Atlas of Endometriosis. Carnforth, UK: The Parthenon Publishing Group. 1993.

Stones RW, Mountfield J. Interventions for treating chronic pelvic pain in women. Cochrane Database Syst Rev. 2000;(2): CD000387. Review. Update in: Cochrane Database Syst Rev. 2000; (4):CD000387.

13. Dmowski WP, Gebel H, Braun DP. Decreased apoptosis and sensitivity to macrophage mediated cytolysis of endometrial cells in endometriosis. Human Reprod Update. 1998;4(5): 696–701.

Eskenazi B, Warner ML. Epidemiology of endometriosis. Obstet Gynecol Clin N Am. 1997;24:235–258.

14. Asante A, Taylor RN. Endometriosis: the role of neuroangiogenesis. Ann Rev Physiol. 2011;73:163–182.

15. Schenken RS. Treatment of human infertility: The special case of endometriosis. In Adashi EY, Rock JA, Rosenwaks Z, eds. Reproductive Endocrinology, Surgery, and Technology, Vol 2. Philadelphia: Lippincott-Raven. 1996. 16. Wellbery C. Diagnosis and treatment of endometriosis. Am Fam Phys. 1999;60(6):1753–1762. 17. Bloski T, Pierson R. Endometriosis and chronic pelvic pain: unraveling the mystery behind this complex condition. Nurs Womens Health. 2008; 12(5):382–395. 18. Howard FM. The role of laparoscopy as a diagnostic tool in chronic pelvic pain. Baillieres Best Pract Res Clin Obstet Gynaecol. 2000;14(3):467–494. 19. Brosens IA. Diagnosis of endometriosis. Sem Reprod Endocrinol. 1997;15(3):229–233. 20. Triolo O, Laganà AS, Sturlese E. Chronic pelvic pain in endometriosis: an overview. J Clin Med Res. 2013;5(3):153–163. 21. Stratton P, Berkley KJ. Chronic pelvic pain and endometriosis: translational evidence of the relationship and implications. Hum Reprod Update. 2011; 17(3):327–346.

265

Section 4 Chapter

37

Visceral Pain

Patient (male) with chronic pelvic pain from interstitial cystitis John Hau, Michael Truong, Eric S. Hsu, and Irene Wu

Case study A 43-year-old male presents to your clinic complaining of pain in his lower abdomen that is dull and “pressure-like” in sensation. This discomfort bothers him on a daily basis with mild pain throughout the day that worsens when he urinates and with ejaculation. Onset of symptoms was 6 months ago. He was recently evaluated by an urologist, and underwent a comprehensive work-up which was unremarkable. The patient was subsequently referred by his urologist to pain management. Review of his medical records shows a recent benign urinalysis, negative urine culture, and negative STD screenings. His review of systems is notable for fatigue, sexual dysfunction, and a long history of lower back pain.

1. What is the innervation of the male pelvis? The iliohypogastric nerve and ilioinguinal nerve originate from the L1 nerve root (with contribution from T12 in some patients). The iliohypogastric nerve follows a curvilinear course along the ilium and then perforates the transverse abdominus muscle, at which point it divides into an anterior and lateral branch. The anterior branch pierces the external oblique to provide sensory innervation to the skin above the pubis. The lateral branch provides sensory innervation to the lateral and posterior gluteus. The ilioinguinal nerve also follows a curvilinear course along the ilium and then perforates the transverse abdominus muscle at the level of the anterior superior iliac spine. It then travels through the inguinal ring and into the inguinal canal. The ilioinguinal nerve

provides sensory innervation to the upper inner thigh, base of the penis, and upper scrotum. The genitofemoral nerve originates from the L1 nerve root (with contributions from T12 and L2 in some patients). The nerve follows a curvilinear course along the ilium and descends anteriorly through the psoas major muscle. It continues to descend behind the ureter and divides just above the inguinal ligament into the genital and femoral branches. The genital branch courses through the inguinal canal and inguinal ring to provide innervations to the cremaster muscle and skin of the scrotom. The femoral branch enters the femoral sheath lateral to the femoral artery and innervates the skin of the anterior superior femoral triangle. The pudendal nerve originates from the S2, S3, and S4 nerve roots. The nerve courses inferiorly between the piriformis and coccygeal muscles. It then exits the pelvis via the greater sciatic foramen and passes around the ischial spine to re-enter the pelvis through the lesser sciatic foramen. At the level of the ischial spine, the pudendal nerve divides into three branches consisting of the inferior rectal nerve, the perineal nerve, and the dorsal nerve of the penis. These branches innervate the anal sphincter, perianal region, posterior two-thirds of the scrotum, muscles of the urogenital triangle, and dorsum of the penis. Autonomic input to the pelvis travels primarily via the superior hypogastric plexus. This structure is a coalescence of fibers that lie anterior to the L5 vertebral body. It is located just below the bifurcation of the aorta and is a continuation of the intermesenteric/ aortic plexus as the sympathetic fibers continue to descend along the posterior abdominal wall on either side of the vertebral bodies.

Case Studies in Pain Management, ed. Alan David Kaye and Rinoo V. Shah. Published by Cambridge University Press. © Cambridge University Press 2015.

266

Chapter 37: Patient (male) with chronic pelvic pain from interstitial cystitis

2. What is the differential diagnosis for a male presenting with chronic pelvic pain?  Overactive bladder, bladder outlet obstruction. Typically associated with complaints of urinary obstruction (urgency, frequency, nocturia) and voiding symptoms (abnormal stream, hesitancy, dribbling, dysuria).  Urethritis. Typically associated with a pertinent sexual history and discharge on examination. Neisseria gonorrhoeae and Chlamydia trachomatis are the most common pathogens.  Bacterial prostatitis and chronic urinary tract infections. Typically associated with positive urine and prostate fluid laboratory studies.  Chronic urinary calculi.  Urethral strictures. Typically occurs after a history of an inciting incident (such as prior infection, injury, surgery, or instrumentation). Patients may also present with frequent UTI as well.  Malignancy. The most common presentation in genitourinary cancers is painless hematuria, although pain and dysuria may develop from locally advancing disease.  Neurogenic bladder, including bladder spasms and detrusor sphincter dyssynergy.

3. How are prostate syndromes classified? What is the epidemiology of chronic prostatitis? The National Institutes of Health supports a classification approach to standardize the definitions of prostate syndromes. The terms chronic prostatitis and chronic pelvic pain (CPP) syndrome are used interchangeably in the literature. There are four classifications: I. acute bacterial prostatitis II. chronic bacterial prostatitis III. chronic prostatitis/chronic pelvic pain syndrome (CP/CPPS) IV. asymptomatic inflammatory prostatitis Patients with acute bacterial prostatitis present with symptoms of an acute infection (dysuria, pyuria, fevers). Escherichia coli is the most common pathogen. Chronic bacterial prostatitis presents with recurrent episodes of urinary tract infections with an infected

prostate gland identified on biopsy as the primary etiology of the recurrent infections. Chronic prostatitis/ CPPS patients have urologic pain as their primary complaint. Patients should be symptomatic for at least 3 of the 6 preceding months. Several exclusion criteria must be met, such as the absence of urethritis, cancer, urinary tract disease, urethral stricture, and neurologic disease involving the bladder. Patients can be further subclassified as inflammatory versus non-inflammatory based on the presence or absence of leukocytes in their prostatic secretions. Patients fall in the asymptomatic inflammatory prostatitis category when evidence of an inflamed prostate is found on biopsy. This occurs typically during evaluation of another genitourinary issue (i.e., prostate biopsy to evaluate for cancer), and patients are often asymptomatic otherwise. Although acute bacterial prostatitis and chronic bacterial prostatitis are the most well-studied of the prostate syndromes, they are the least common. The NIH consensus states that over 90% of symptomatic patients have CP/CPPS. In a population-based study by Nickel et al, men aged 20–39 have a prevalence of 11%, aged 40–49 have the highest prevalence at 13%, and older men aged between 70 and 74 are at 7% prevalence for CP/CPPS.

4. How should male CPP be evaluated? The primary objective in the evaluation of male patients with CPP is to identify potential etiologies that are treatable.

History A detailed discussion of the pain complaint is vital in the evaluation of male chronic pelvis pain. Location, timing, and associated symptoms should be elicited. The location of the pain may range anywhere from the lower abdomen down to the testicles and penis. Associated symptoms include pain with ejaculation, voiding, or bladder filling. Voiding symptoms such as urgency, frequency, and dysuria are frequently present. Coexisting irritable bowel syndrome or fibromyalgia should be noted. Assessing the impact on quality of life will also be useful. A history of urinary tract infections or sexually transmitted diseases may be suggestive of an infectious etiology, such as urethritis. A history of prior urologic disease, especially prior instrumentation or surgery, may be suggestive of an anatomical problem, such as urethral stricture.

267

Chapter 37: Patient (male) with chronic pelvic pain from interstitial cystitis

Physical exam The physical exam is strongly recommended to further screen for treatable etiologies in a patient with CPP. Structures that should be examined include the abdomen, suprapubis, groin, spermatic cord, epididymis, and testes. The prostate should be evaluated for tenderness, consistency, and nodules.

Laboratory Essential laboratory evaluations include urinalysis and urine culture to rule out acute disease. The Meares–Stamey 4-glass test examines urine and prostatic secretion. This allows for localization of inflammation in the upper versus lower urinary tract. This test can provide information on the primary site of infection in patients who have a positive urinalysis or urine culture. Urine cytology is important in men complaining of irritable voiding symptoms and/or microscopic hematuria. Residual urine volume determination provides information on the functioning of the lower urinary tract (i.e., obstruction).

Additional studies Further imaging and invasive studies are considered optional and should be pursued with clinical judgment. Cystoscopy should be performed if initial workup suggests lower urinary tract pathology. Clinical findings that support proceeding with cystoscopy include hematuria, suspicious cytology, abnormal urodynamics studies, or irritative voiding symptoms. Transrectal ultrasound of the prostate is useful in identifying treatable prostate abscess, cysts, or abnormalities in the ejaculatory system. Other pelvic imaging studies, such as CT scan and MRI, can identify potentially treatable pelvic pathology as well as evaluate the prostate, seminal vesicles, and ejaculatory duct for abnormalities.

5. What non-pharmacologic treatment options are available to patients with CPPS? A large number of patients with CPPS demonstrate muscle tenderness and tension, suggesting that there may be a component of myofascial pain and pelvic floor

268

muscle dysfunction. Therefore, non-pharmacologic therapies, which target decreasing pelvic floor muscle dysfunction, have been studied in the treatment of CPPS. These treatment options include biofeedback, pelvic floor retraining, and physical therapy based on myofascial trigger points targeted at reducing pelvic floor muscle dysfunction. Chronic pelvic pain syndrome is associated with depression and a reduced quality of life. Concurrent depression is a strong predictor for higher pain levels reported by men with CPPS. Therefore, CBT may be helpful in select patients. Acupuncture is an alternative and relatively safe treatment option for CPPS. There is data to suggest that acupuncture may be helpful in improving CPPS related pain. TENS units are a popular option in the treatment of many pain conditions. It functions by delivering electrical nerve stimulation of various durations and intensities to relieve pain. The electrical stimulation causes myelinated afferent nerves to activate inhibitory circuits to relieve pain at the targeted site.

6. What pharmacologic agents are used in the treatment of CPPS? There are a number of pharmacologic therapies available to the physician for the treatment and management of CPPS. The pharmacologic agents can be divided into two broad categories including those targeted at treating the underlying disease process with its associated symptoms and those that are aimed at treating pain. There are currently no uniformly accepted treatment guidelines in CPPS. Furthermore, the efficacy of these medications remains controversial as there are no robust clinical trials to date that show consistent benefit. Most studies show only a limited ability of these agents to reduce symptoms. The following medications are typically prescribed by an urologist or primary care physician for the medical management of CPPS. These pharmacologic agents are targeted at treating the underlying disease and symptoms of CPPS.  Antibiotics: Long-term antibiotics were once the mainstay of CPPS treatment. However, less than 10% of symptomatic patients have culturable bacteria in the urinary tract. A short initial course of antibiotics is considered to be an acceptable treatment option in patients with CPPS, but

Chapter 37: Patient (male) with chronic pelvic pain from interstitial cystitis

long-term antibiotic therapy in patients who have previously failed antibiotic therapy is not likely to be beneficial. The most commonly used antibiotics include quinolones (ciprofloxacin and levofloxacin) and tetracyclines. A 6-week course of antibiotics is usually adequate.  Alpha-blockers and 5-alpha reductase inhibitors: These medications are prescribed and indicated for patients with benign prostatic hypertrophy. Alpha-blockers inhibit overactivation of bladder neck smooth muscle while 5-alpha reductase inhibitors prevent prostate growth. These medications have been advocated in the treatment of CPPS because of the many similarities in the symptoms of CPPS and benign prostatic hypertrophy, such as voiding dysfunction. Current data suggests that there is some benefit with modest efficacy with these medications. It is generally accepted that antibiotics and alphablockers are used in combination as the first-line pharmacotherapy for the treatment of CPPS.  Pentosan polysulfate: This medication is postulated to replenish the glycosaminoglycan layer of the bladder. It is commonly used in patients with painful bladder syndrome and interstitial cystitis.  Phytotherapies: Certain natural therapies have been used for the treatment of CPPS with only modest efficacy. These include saw palmetto, bee pollen extract, and quercetin. Patients with CPPS will likely have undergone an evaluation and initiation of treatment by an urologist prior to seeing a pain management specialist. These patients will often present after starting treatment as listed above, and will likely be referred for management of their pain. Listed below are pharmacologic options available to the pain physician in helping to relieve pain secondary to CPPS.  Anti-inflammatories: NSAIDs and COX-2 inhibitors have been suggested to be helpful in the treatment of patients with CPPS to counteract the localized inflammation of the prostate. Likewise, an autoimmune mechanism has been implicated in CPPS, which makes corticosteroid therapy a possible treatment option.  Anticonvulsants: CPPS is increasingly viewed as a condition where neuropathic pain may play a significant role. Therefore, there has been a

dramatic increase in the use of anticonvulsants for management of CPPS. Gabapentin and pregabalin are anticonvulsant medications that have been used in the treatment of many neuropathic pain conditions such as postherpetic neuralgia, diabetic neuropathy, and fibromyalgia. They have a generally favorable side effect profile and are considered relatively safe medications.  Tricyclic antidepressants: Tricyclic antidepressants are centrally acting norepinephrine and serotonin reuptake inhibitors that have long been used in the treatment of neuropathic pain conditions. Their use has been advocated in the treatment of CPPS, and data suggests that in addition to reducing pain they may be beneficial in improving other symptoms such as urinary frequency.  NMDA receptor antagonists: It is postulated that NMDA receptor overactivation and excessive excitatory neurotransmission play a role in the development of certain chronic pain conditions through neuronal pathway remodeling. Therefore inhibition of the NMDA receptor can theoretically decrease pain by inhibiting these changes. Memantine is an NMDA receptor antagonist currently used in the treatment of Alzheimer’s disease. It may be an option to consider in patients with CPPS. Adverse effects due to memantine include dizziness, drowsiness, constipation, hypertension, and headaches.  Opioids: To date, there are no published controlled trials evaluating the efficacy of opioids in the treatment of CPPS. They are generally not considered very effective in the treatment of neuropathic pain. High doses are commonly required to achieve limited effects in patients with neuropathic pain. This is postulated to be due to downregulation of opioid receptors from overactivation of NMDA receptors as well as neuronal remodeling. Nevertheless, opioids are potent analgesics that are used in the management of many chronic pain conditions. If they are to be prescribed, it should be done so under the discretion of a physician knowledgeable of the myriad of opioids available. Combination therapy with a long- and short-acting agent is advisable. It is also suggested that opioids be used in combination with adjuvants such as anticonvulsants or antidepressants, as opposed to monotherapy. However, the use of opioids

269

Chapter 37: Patient (male) with chronic pelvic pain from interstitial cystitis

may be limited to its numerous side effects (nausea, respiratory depression, constipation) and complications (tolerance, abuse, and addiction).

7. What interventional procedures can be performed in the treatment of CPPS? Interventional procedures may be performed for diagnostic and therapeutic purposes. Temporary but consistent responses to nerve blocks may lead to longer lasting procedures such as pulsed radiofrequency ablation, neurolysis, or neuromodulation.  Trigger point injections: Patients with CPPS are thought to have a myofascial component with muscle tenderness and tension contributing to their pain. Therefore, trigger point injections targeted at these tender points have been recommended as a treatment option. These injections are performed at localized areas of tenderness. They are typically done with local anesthetic, saline, or with simple needling. Trigger point injections are relatively safe procedures that can be performed in the office setting.  Peripheral nerve blocks: The pelvis is innervated by nerves originating from the lumbosacral plexus. The iliohypogastric nerve (L1), ilioinguinal nerve (L1), genitofemoral nerve (L1, L2), and pudendal nerve (S2, S3, S4) are potential targets for nerve block. Some patients with chronic neuropathic non-malignant pain demonstrate improved pain in response to a series of injections, which may be due to reversal of neuroplastic changes. If patients demonstrate a consistent but temporary response to peripheral nerve blocks, neurolysis with pulsed radiofrequency ablation may be considered. Potential side effects of neurolysis include neuroma formation, deafferentation pain, permanent nerve injury leading to motor or sensory deficits, neuritis, sexual dysfunction, and bowel or bladder incontinence.  Sympathetic nerve blocks: The superior hypogastric plexus comprises postganglionic sympathetic fibers from T10 to L2 and parasympathetic fibers from the sacral ganglion. The superior hypogastric plexus is a potential target for intervention as it covers pain arising from pelvic visceral structures. Superior

270

hypogastric plexus block has been shown to decrease pelvic pain by 70% in patients with cervical, prostate, or testicular cancer. Although this data may not be directly extrapolated to patients with CPPS, it is an option to consider in patients who have failed conservative management.

8. How are the iliohypogastric, ilioinguinal, genitofemoral, and pudendal nerve blocks performed? Iliohypogastric and ilioinguinal nerve block The iliohypogastric and ilioinguinal nerves are often blocked together because these nerves may interconnect along their anatomical course. Variation in the sensory distribution of the iliohypogastric and ilioinguinal nerves frequently exists because of this interconnection. The nerve block is performed with the patient in the supine position. In the blind technique, the anterior superior iliac spine is identified and a point 1 inch medial and inferior is marked and prepared with sterile technique. This identified point is then entered with a 1.5-inch 25-gauge needle directed at an oblique angle toward the pubic symphysis. A solution containing local anesthetic with or without steroid is then injected in a fan-like manner after the needle pierces the fascia of the external oblique muscle. An ultrasound-guided technique was introduced by Eichenberger in 2006. In the ultrasound-guided technique, a linear 7.5 MHz probe is scanned 5 cm cephalad to the anterior superior iliac spine and rotated to an oblique plane with the lateral-caudal part of the transducer placed into contact with the iliac crest. At this point, the iliohypogastric and ilioinguinal nerve can be identified between the transverse abdominus muscle and the internal oblique muscle. These nerves are then blocked under real-time ultrasound guidance with an 8-cm 22-gauge blunt needle. Potential adverse effects of this nerve block include ecchymosis, hematoma formation, and peritoneal and abdominal viscera perforation.

Genitofemoral nerve block The genital branch of the genitofemoral nerve is blocked with the patient in the supine position. A point immediately lateral to the pubic tubercle

Chapter 37: Patient (male) with chronic pelvic pain from interstitial cystitis

and inferior to the inguinal ligament is identified. After preparation with antiseptic solution, a 1.5-inch 25-gauge needle is advanced through this point through the subcutaneous tissues and a solution containing local anesthetic with or without steroid is injected. Potential adverse effects of this nerve block include ecchymosis, hematoma formation, and peritoneal and abdominal viscera perforation.

Pudendal nerve block In males, the pudendal nerve block was traditionally performed via a transperineal approach in a lithotomy position with palpation of the ischial spine through the rectum. It is now frequently performed under fluoroscopy in the prone position with a technique originally described by Salahadin in 2004. To perform a pudendal nerve block, anterior–posterior views of the pelvic inlet are obtained under fluoroscopy with visualization of the ischial spine highlighted by 5 to 15 degrees of ipsilateral oblique angulation. A 3.5-inch 25-gauge needle is advanced to the tip of the ischial spine, where local anesthesic is deposited. Potential adverse effects of this nerve block include ecchymosis, hematoma formation, and peritoneal and abdominal viscera perforation.

Superior hypogastric plexus block The superior hypogastric plexus block can be performed blindly, but it is recommended that this block be performed under fluoroscopic guidance. The patient is placed in the prone position with the c-arm obliqued ipsilaterally 25 to 35 degrees and angled 25 to 35 degrees cephalad. A triangle consisting of the L5 transverse process superiorly, iliac crest laterally, and L5-S1 facet joint medially represents the window through which the needle should pass to reach the anterolateral margin of the lumbosacral junction where the superior hypogastric plexus is located. A 6-inch 22-gauge needle is advanced under fluoroscopic guidance to the anterolateral portion of the L5-S1 intervertebral disc. A small volume of contrast should be injected to obtain an appropriate neurogram, after which local anesthestic can be deposited around the plexus. Potential adverse effects of this nerve block include ecchymosis, hematoma formation, and peritoneal and abdominal viscera perforation.

9. What other treatment options are available if pharmacotherapy and targeted nerve blocks have failed to provide adequate pain relief? Neuromodulation This therapy has been widely and successfully used for the treatment of several chronic pain conditions over several decades. Neuromodulation has been used to relieve pain in a variety of conditions such as neuropathic pain, back and limb pain, facial pain, and angina. Melzack and Wall proposed the gate theory of pain in 1965, which provided a scientific basis for neuromodulation. The gate theory proposes that a “gate” regulates pain transmission from the dorsal horn in the spinal cord. This gate is closed when large afferent fibers are activated, thereby preventing the transmission of pain sensations. Neuromodulation is thought to work by introducing stimulation via exogenous electrical activity which depolarizes large afferent fibers in the dorsal column and closes the gate to pain transmission. Spinal cord stimulation is considered an advanced treatment for patients who have failed more conservative options. Studies have shown patients with improved pain visual analog scale scores, decreased pain disability indices, and decreased opioid use with spinal cord stimulation. Sacral nerve stimulation was first described in 1990 by Schmidt as a potential therapy for relieving medically refractory urinary frequency, urgency, and urge incontinence. Mounting evidence has shown that sacral nerve stimulation helps to improve pain scores in addition to urinary symptoms in patients with chronic pelvic and bladder pain. It is hypothesized that stimulation of the S3 nerve root causes fatigue of the external urinary sphincter and pelvic floor musculature thereby relieving pain and voiding dysfunction caused by muscle spasm. Spinal cord stimulation can be performed through various approaches and techniques, and therefore the optimal location for lead placement remains controversial. Some practitioners advocate T11-L1 level lead placement, while others advocate lead placement in the area of the sacral nerve roots. Furthermore, placement of sacral leads can be accomplished through different routes including retrograde lead placement, multipolar lead placement through the sacral foramen, or advancement through the sacral hiatus with alignment

271

Chapter 37: Patient (male) with chronic pelvic pain from interstitial cystitis

NIH-Chronic Prostatitis Symptom Index (NIH-CPSI) 1.

Pain or Discomfort In the last week, have you experienced any pain or discomfort in the following areas?

a.

b.

2.

Area between rectum and testicles (perineum) Testicles

Yes

No

1

0

1

c.

Tip of the penis (not related to urination)

d.

Below your waist, in your pubic or bladder area

6.

0

1

How often have you had to urinate again less than two hours after you finished urinating, over the last week? 0

Not at all

1

Less than 1 time in 5

2

Less than half the time

3

About half the time

4

More than half the time

5

Almost always

0

7. 1

0

Impact of Symptoms How much have your symptoms kept you from doing the kinds of things you would usually do, over the last week? 0

In the last week, have you experienced:

1

Yes a.

Pain or burning during urination?

b.

Pain or discomfort during or after sexual climax (ejaculation)?

2

No

1

0

1

0

3

8.

4.

How often have you had pain or discomfort in any of these areas over the last week? 0

Never

1

Rarely

2

Sometimes

3

Often

4

Usually

5

Always

9.

Which number best describes your AVERAGE pain or discomfort on the days that you had it, over the last week?

1

None Only a little

2

Some

3

A lot

Quality of Life If you were to spend the rest of your life with your symptoms just the way they have been during the last week, how would you feel about that? 0

Delighted

1

Pleased

2 3

0 NO PAIN

5.

1

2

3

4

5

6

7

8

9

10 PAIN AS BAD AS YOU CAN IMAGINE

Urination How often have you had a sensation of not emptying your bladder completely after you finished urinating. over the last week? 0

Not at all

1

Less than 1 time in 5

2

Less than half the time

3

About half the time

4

More than half the time

5

Almost always

Mostly satisfied Mixed (about equally satisfied and dissatisfied)

5

Mostly dissatisfied Unhappy

6

Terrible

4

Scoring the NIH-Chronic Prostatitis Symptom Index Domains Pain: Total of items 1a, 1b, 1c, 1d, 2a, 2b, 3, and 4

=

Urinary Symptoms: Total of items 5 and 6

=

Quality of Life Impact : Total of items 7, 8, and 9

=

Figure 37.1. NIH Chronic Prostatitis Symptom Index (NIH-CPSI).

272

Some A lot

How much did you think about your symptoms, over the last week? 0

3.

None Only a little

Chapter 37: Patient (male) with chronic pelvic pain from interstitial cystitis

of the electrodes adjacent to multiple sacral nerve roots. Regardless of the technique used, stimulation and efficacy of this therapy are assessed over a 7- to 10day trial period. If the stimulation achieved during the trial period is deemed to improve symptoms, the system consisting of an implantable pulse generator and electrodes is surgically implanted. Pudendal nerve stimulation has also been used in the treatment of CPP. The pudendal nerve is a peripheral nerve and stimulation can theoretically modulate the multiple spinal segments that give rise to the pudendal nerve. Pudendal nerve stimulation is accomplished by implantation of electrodes adjacent to the pudendal nerve in Alcock’s canal. Although data on its use is limited, pudendal nerve stimulation has been shown to improve voiding symptoms and pain scores. An emerging neuromodulation therapy is transperineal electromagnetic stimulation. This therapy employs an electromagnetic field to the perineum, which is thought to cause neural excitation and stimulation of the muscles of the pelvic floor. There is very limited data regarding the efficacy of this treatment at this time.

Intrathecal drug therapy An intrathecal pump delivering opioids may be considered in patients who have exhausted all other treatment options. It should be reserved for opioiddependent patients who have failed all other forms of management. Careful patient selection is recommended for this treatment modality.

10. How do you monitor the progress of treatment in CPPS? The efficacy of treatment can be measured by the NIH chronic prostatitis symptom index (NIH-CPSI). This

References 1.

2.

Anderson RU, Wise D, Sawyer T, Chan C. 6-Day intensive treatment protocol for refractory chronic prostatitis/chronic pelvic pain syndrome using myofascial release and paradoxical relaxation training. J Urology. 2011;185:1294–1299. Anothaisintawee T, Attia J, Nickel JC, et al. Management of chronic prostatitis/chronic pelvic pain

is a validated questionnaire that analyzes nine unique items addressing three different aspects of the patient’s symptoms, including pain, urinary function, and quality of life (Figure 37.1).

11. What is the treatment course in CPPS? CPPS is a chronic pain condition that is often difficult to diagnose and treat. Although no clear treatment guidelines exist, we recommend consideration of conservative therapy with physical therapy and pharmacotherapy as a first line of treatment. A multimodal pharmacologic approach targeting somatic, visceral, and neuropathic pain is recommended. If conservative therapy fails, interventional procedures such as targeted nerve blocks aimed at relieving somatic and visceral pain should be considered. Further interventional procedures including neuromodulation and intrathecal drug therapy should be reserved for patients who have failed less invasive options. Due to its complexity and chronicity, CPPS requires a multimodal approach to treatment coordinated by a multidisciplinary team, consisting of a primary care physician, pain physician, urologist, and physical therapist. To date, no therapies have emerged that consistently show high efficacy in the treatment of CPPS. Data for implantable therapies look promising and will likely play a larger role in the treatment of CPPS as more studies are performed. Our current understanding of CPPS is still lacking. More studies and research are necessary to better understand the underlying etiologies and pathophysiology of this complex disease process and to bring novel treatment strategies to light. Clinical features, diagnostic procedures, and results of surgical treatment in 68 patients. Ann Intern Med. 1985;103:271.

syndrome a systematic review and network meta-analysis. JAMA. 2011;305:78–86. 3.

4.

Cohen JC, Fagin AP, Hariton E, et al. Therapeutic intervention for chronic prostatitis/chronic pelvic pain syndrome: a systematic review and meta-analysis. PLoS One. 2012;7(8):e41941. Hall S, Bartleson JD, Onofrio BM, et al. Lumbar spinal stenosis.

5.

Lee SH, Lee SC. Use of acupuncture as a treatment method for chronic prostatitis/ chronic pelvic pain syndromes. Curr Urol Rep. 2011;12:288–296.

6.

Litwin MS, McNaughton-Collins M, Fowler FJ Jr, et al. The National

273

Chapter 37: Patient (male) with chronic pelvic pain from interstitial cystitis

Institutes of Health Chronic Prostatitis Symptom Index: development and validation of a new outcome measure. J Urology. 1999;162:369–375. 7.

8.

274

Krieger JN, Nyberg L, Nickel JC. NIH consensus definition and classification of prostatitis. JAMA. 1999;282:236. Nickel J. Clinical evaluation of the man with chronic prostatitis/ chronic pelvic pain syndrome. Urology. 2002;60:20–23.

9.

Nickel JC, Downey J, Hunter D, Clark J. Prevalence of prostatitislike symptoms in a population based study using the National Institutes of Health chronic prostatis symptom index. J Urology. 2001;165:842.

10. Salahadin, A, Shenouda P, Patel N, et al. A novel technique for pudendal nerve block. Pain Physician. 2004;7:319–322. 11. Schaeffer AJ. Etiology and management of chronic pelvic

pain syndrome in men. Urology. 2004;63:75. 12. Strauss AC, Dimitrakov JD. New treatments for chronic prostatitis/ chronic pelvic pain syndrome. Nat Rev Urol. 2010;7(3):127–135. 13. Touma, N, Nickel JC. Prostatitis and chronic pelvic pain syndrome in men. Med Clin N Am. 2011;75:86. 14. Yang CC. Neuromodulation in male chronic pelvic pain syndrome: rationale and practice. World J Urol. 2013;31:767–772.

Section 4 Chapter

38

Visceral Pain

Chronic rectal pain Brandon A. Van Noord, Irene Wu, and Eric S. Hsu

Case study A 71-year-old male with a past medical history of hypertension, coronary artery disease, atrial fibrillation, and prostate cancer presented to the clinic with a chief complaint of rectal pain. The pain started approximately 1 year prior to presentation after the patient underwent stereotactic body radiotherapy (SBRT) to treat Gleason 4+3, T2aN0M0, PSA 5.2 prostate cancer. Shortly after starting SBRT, he noticed bloody stools and was diagnosed with radiation proctitis on sigmoidoscopy. Medication management inadequately controlled his pain.

1. What is the differential diagnosis for rectal pain?          

Proctalgia fugax Hemorrhoids Pilonidal cyst or other infection Radiation proctitis Coccygodynia Pudendal neuralgia Pelvic or sacral insufficiency fractures Primary or metastatic disease of the rectum Referred pain from the urogenital tract Referred pain from the lumbosacral spine

neuronal plexuses before distributing nerve fibers throughout the pelvis. In general, the visceral afferent pain fibers and the sympathetic fibers travel together. Consequently, visceral pain is poorly localized, unlike precisely located somatic pain. Visceral afferent pain fibers have their cell bodies in the dorsal root ganglia and enter the spinal cord through the dorsal roots. The inferior hypogastric plexus is the major neuronal integrative center in the pelvis. It is a paired retroperitoneal structure located anterior to the sacrum on either side of the rectum (males) and the sides of the rectum and vagina (females). It innervates the bladder, proximal urethra, distal ureter, rectum, internal anal sphincter, and reproductive tract. Sympathetic innervation to the plexus originates from the thoracolumbar sympathetic chain (T10-L1) and arrives at the inferior hypogastric plexus via the superior hypogastric plexus. Parasympathetic afferent and efferent contributions to the inferior hypogastric plexus (S2-S4) coalesce as the pelvic splanchnic nerve before joining the plexus. Anterior to the sacrococcygeal junction, the two bilateral chains of the sympathetic ganglia fuse, terminate, and form the ganglion impar. This retroperitoneal structure receives visceral afferent pain fibers from the perineum, distal rectum, anus, distal urethra, vulva, and distal third of the vagina.

2. Describe the neuroanatomy of the pelvic viscera including the rectum

3. What are some unique features of rectal innervation and how do they contribute to rectal pain?

The autonomic nervous system (sympathetic and parasympathetic divisions), the somatic nervous system, and the afferent (sensory) division of the peripheral nervous system innervate the pelvic viscera. These diverse neural elements converge in

The smooth muscle of the rectum and internal anal sphincter are innervated by the sympathetic and parasympathetic divisions of the autonomic nervous system via the inferior hypogastric plexus. This starkly contrasts with the striated muscle of the

Case Studies in Pain Management, ed. Alan David Kaye and Rinoo V. Shah. Published by Cambridge University Press. © Cambridge University Press 2015.

275

Chapter 38: Chronic rectal pain

external anal sphincter and the levator ani, which are under the control of the somatic nervous system via the sacral plexus (S2–4) and the pudendal nerve. At the ischial spine, the pudendal nerve trifurcates to form the inferior rectal nerve, which innervates the external anal sphincter and the perianal region below the pectinate line, the perineal nerve, which supplies the posterior two-thirds of the scrotum/labia majora and the muscles of the urogenital triangle, and the dorsal nerve of the penis or clitoris. The unique innervation of the rectum explains why external but not internal hemorrhoids are exquisitely tender. Below the pectinate line, the pudendal nerve (sacral plexus) innervates the rectum. Above the pectinate line, the inferior hypogastric plexus innervates the rectum (see previous section). The unique innervation of the rectum also explains why an inferior hypogastric plexus block may need to be performed in addition to a ganglion impar block to adequately alleviate rectal pain.

fracture, contusion, or dislocation. The resulting inflammation and surrounding muscle spasm is painful. Repetitive microtrauma from prolonged sitting on a hard surface with poor posture can also lead to coccygodynia. Chronic trauma can lead to osteoarthritis of the sacrococcygeal and coccygeal joints. Difficult vaginal delivery can strain the sacrococcygeal ligament and lead to chronic coccygodynia. Finally, somatization may also contribute. Coccygodynia is diagnosed on physical exam. In coccygodynia, the coccyx is exquisitely point tender and movement causes sharp paresthesias that radiate into the rectum. Patients typically report pain with sitting, defecation, and sexual intercourse. While sitting, weight is distributed between the coccyx and the ischeal tuberosities. When leaning forward, weight redistributes from the coccyx onto the ischial tuberosities thereby improving pain. X-rays may be indicated if the physical exam is unclear or to rule out other concerning pathologies.

4. What is the typical work-up for rectal pain?

6. What are the pathophysiology and diagnostic criteria of proctalgia fugax?

 History  Physical exam (including rectal and vaginal exams)  Laboratory: CBC, PSA, ESR, ANA  Plain radiographs  Computed tomography  Magnetic resonance imaging  Radionucleotide bone scanning  Colonoscopy  Cystoscopy While pain is typically distressing to the patient, it often signals underlying pathology. Additionally, several pain disorders are diagnoses of exclusion. Therefore, it is important to initially maintain a broad differential diagnosis and pursue an appropriate diagnostic work-up before treatment. Pharmacologic and therapeutic interventions may mask pain secondary to underlying primary pathology.

Several different mechanisms contribute to the pathophysiology of proctalgia fugax. Internal anal sphincter smooth muscle spasm is the most frequently etiology. Pudendal neuralgia may also contribute to proctalgia fugax. Depression and anxiety are more common in patients with functional gastrointestinal disorders such as proctalgia fugax and may contribute to both the underlying pathophysiology and the decision to seek medical care. The three diagnostic criteria for proctalgia fugax are recurrent episodes of pain localized to the anus or rectum, episodes lasting from seconds to minutes, and no anorectal pain between episodes. In general, proctalgia fugax is a diagnosis of exclusion with no specific pathognomonic physical exam findings or confirmatory laboratory tests. Proctalgia fugax is more common in women and in patients with irritable bowel syndrome. Prolonged sitting and increased stress can increase both attack intensity and frequency.

5. What are the pathophysiology and diagnostic criteria of coccygodynia?

7. What are the pathophysiology and diagnostic criteria of radiation proctitis?

Coccygodynia is pain in the tailbone that radiates into the lower sacrum and perineum. Typically, it is caused by direct trauma that results in coccygeal

Radiation to the lower gastrointestinal tract or urogenital tract can result in radiation injury to the mucosa of the rectum and sigmoid colon. Mucosal

276

Chapter 38: Chronic rectal pain

injury is typically seen during the first 6 weeks after radiotherapy. Common symptoms include bleeding, diarrhea, and rectal urgency. Radiation proctitis can occur in 15% of patients following radiation therapy for pelvic malignancies. Symptoms spontaneously resolve after several months in only 35% of patients. The prevalence of radiation proctitis should increase as more patients elect minimally invasive treatment and as the proportion of elderly patients increases. Radiation proctitis should be suspected in patients who develop bleeding, diarrhea, and rectal urgency following radiation exposure. Colonoscopy and sigmoidoscopy can be used to confirm the diagnosis. To exclude other causes of proctitis, biopses may be warranted but should be used judiciously as they may contribute to fistula formation. CT scans are useful to exclude malignancy recurrence.

8. What are some treatment modalities of rectal pain?  Conservative

: : :

Donut pillow Sitz baths Ice/heat Adjuvants 

: :

TENS unit Complementary and alternative medicine/ acupuncture : Pain psychology: CBT, Relaxation biofeedback  Pharmacologic management

: : :

NSAID/COX-2 inhibitors Antiepileptic drug: gabapentin, pregabalin Tricyclic antidepressant: amitriptyline, desipramine, nortriptyline : Selective norepinephrine serotonin reuptake inhibitor: duloxetine, effexor : Nitroglycerin : Constipation: fiber/psyllium seed, surfactant (docusate), stimulant (bisacodyl/senna), osmotic agent (lactulose)  Interventional

: : : : :

BOTOX injection Ganglion impar block Pudendal nerve block Inferior hypogastric nerve block Sacrococcygeal nerve block

 Neuromodulation

: :

Sacral nerve stimulator Pudendal nerve stimulator

9. What are some topical medications used to treat rectal pain? Compared with enteral medication, topical medications are inherently advantageous in many ways. Topical medications only produce localized effects thereby minimizing systemic side effects and allergy risk. As topical medications only work on tissues directly underneath the skin application area, they can be added to an existing pharmacologic regiment with less worry about potential drug–drug interactions. A recent retrospective review analyzed over 1000 patients who received topical 1–2% amitriptyline and 0.5% ketamine for treatment of chronic pain. For those treated for rectal, genital, or perineal pain, the authors found that 85% had at least some pain relief with over 50% reporting substantial relief. Topically administered amytriptyline is believed to act primarily as a sodium channel blocker. Ketamine blocks NMDA receptors in peripheral nerves. Glutamate’s action on the NMDA receptor has been linked to sensitization in chronic pain. Since each medication works through a different mechanism of action, they should act synergistically. Local irritation was the only side effect noted in the study.

10. When is the ganglion impar block indicated and how is it performed? The ganglion impar innervates the coccyx, perineum, distal rectum, anus, distal urethra, and genitalia. Therefore, ganglion impar block is indicated to diagnose or treat sympathetically mediated pain involving any of these structures. Ganglion impar block has been successfully used to treat pain from radiation proctitis, coccygodynia, chronic perineal pain, pain from pelvic, gastrointestinal, and perianal malignancies, and postherpetic neuralgia. The ganglion impar block is performed under fluoroscopic guidance with the patient in the prone position. The first step is to use an AP view to identify the sacrococcygeal junction. A 20-gauge Tuohy needle is then advanced perpendicularly to the sacrococcygeal joint. A lateral view is then utilized to

277

Chapter 38: Chronic rectal pain

Figure 38.1. Ganglion impar block. From personal files of Rinoo V. Shah, MD, MBA.

Figure 38.2. Ganglion impar block. From personal files of Rinoo V. Shah, MD, MBA.

advance the needle through the joint until the tip is even with the anterior border of the coccyx. 1–2 ml of contrast is then injected after negative aspiration. Lenticular contrast spread between the rectum and the sacrococcygeal complex should be seen without vascular uptake. An AP view should again be obtained to verify bilateral spread. Local anesthetic with or without steroid can be injected. Following successful diagnostic block, neurolysis or radiofrequency ablation may be performed at the ganglion impar as well. In addition to side effects common to all blocks such as pain with injection, lack of efficacy,

278

Figure 38.3. Ganglion impar block. From personal files of Rinoo V. Shah, MD, MBA.

bleeding, and infection, there are several unique considerations when performing this block. As the ganglion impar lies directly posterior to the rectum, bowel perforation and tracking along the injection site are potential complications. In immunocompromised patients or after radiation therapy, infection and fistula formation can be potentially life threatening. See Figures 38.1 to 38.3.

Chapter 38: Chronic rectal pain

Figure 38.4. Trans-sacral peripheral nerve stimulation lateral fluoroscopic view. From personal files of Rinoo V. Shah, MD, MBA.

Figure 38.5. Trans-sacral peripheral nerve stimulation anteroposterior view. From personal files of Rinoo V. Shah, MD, MBA.

11. When is the pudendal nerve block indicated and how is it performed?

anesthetic solution is injected. The pudendal artery and nerve lie in close proximity to the pudendal nerve. Vascular or rectal injuries are special considerations when performing this block. See Figures 38.4 and 38.5.

Pudendal nerve block has a well-established indication in chronic perineal and pelvic pain treatment. The role of the pudendal nerve in proctalgia fugax is just beginning to be elucidated. In a recent study of patients with proctalgia fugax, pudendal nerve palpation elicited pain and precipitated an attack in 55 out of 68 patients. Subsequent pudendal nerve block completely resolved pain in 65% of patients and decreased pain in an additional 25% of patients. The pudendal nerve is most easily blocked before it trifurcates at the ischial spine using either a trans-vaginal or a trans-perineal approach. In either approach, the patient is placed in lithotomy. The nondominant hand is placed in either the vagina or rectum, the ischial spine is palpated, and a 20-gauge, 6-inch spinal needle is advanced through the sacrospinous ligament just beyond the ischial spine. After loss of resistance and negative aspiration, the local

12. When is the inferior hypogastric plexus block indicated and how is it performed? The inferior hypogastric plexus is the major neuronal integrative center for the efferent sympathetic fibers and visceral afferent pain fibers innervating the lower pelvic viscera, rectum, and genitalia. Therefore, inferior hypogastric plexus block is indicated in patients with malignant or non-malignant visceral or sympathetically mediated pelvic, rectal, or genital pain syndromes. The patient is positioned prone when using a transsacral technique to block the inferior hypogastric

279

Chapter 38: Chronic rectal pain

plexus. The c-arm is rotated cephalad to visualize the sacral foramen end-on as circles instead of ellipses. While the block is typically performed through S2, any of the sacral foramen may be used. A bent 3½inch, 22-gauge spinal needle is advanced to the most medial edge of the ventral sacral foramen under fluoroscopic guidance. The needle is then advanced 1 mm anteromedially toward midline and contrast is injected. After verifying proper cephalad and caudad spread along the anterior surface of the sacrum, the medication is injected. The procedure is then repeated on the contralateral side.

13. How is BOTOX used to treat rectal pain? BOTOX injection into the puborectalis muscle (part of the levator ani group of muscles) has been investigated to treat rectal pain associated with anismus. The puborectalis muscle attaches to the pubic bone and wraps around the rectum at the anorectal ring. Failure to relax from hypertrophy or spasm leads to constipation, tenesmus, and rectal pain. BOTOX-mediated relaxation improves defecation and pain.

References 1.

2.

3.

4.

280

Benzon HT, Hurley RW, Deer TR. Chronic pain management. In Barash PG. Clinical Anesthesia, 6th ed. Philadelphia: Lippincott Williams & Wilkins. 2009: p. 1519. Bhatnagar S, Khanna S, Roshni S, et al. Early ultrasound-guided neurolysis for pain management in gastrointestinal and pelvic malignancies: an observational study in a tertiary care center of urban India. Pain Pract. 2012; 12(1):23–32. Bouchoucha M, Hejnar M, Devroede G, et al. Anxiety and depression as markers of multiplicity of sites of functional gastrointestinal disorders: a gender issue? Clin Res Hepatol Gastroenterol. 2013;37(4): 422–430. Burnstock G. Innervation of the bladder and bowel. In Bock G,

5.

14. How can neuromodulation be used to help manage rectal pain? Neuromodulation changes pain perception by using exogenous electricity to alter the nervous system. While neuromodulation alters pain transmission, it does not treat the underlying pain etiology. The gate control theory is most frequently used to explain the mechanism behind neuromodulation. This theory holds that non-nociceptive fiber stimulation (A-β) can inhibit nociceptive fiber perception (A-δ and C). Descending, central pathways also modulate pain perception. Neuromodulation should only be considered if diagnostic nerve blocks of either the S3 nerve root or the pudendal nerve have proven efficacious. Initially, temporary leads are placed and stimulation is assessed over a 1-week trial period. If stimulation sufficiently improves symptoms, then permanent leads and an implantable pulse generator are surgically implanted. Sacral nerve stimulator leads are implanted either through the S3 sacral foramen to stimulate the nerve root or through the sacral hiatus to stimulate multiple sacral nerve roots. Pudendal nerve stimulators are implanted adjacent to the peripheral nerve at the ischial spine.

Whelan J, eds. Neurobiology of Incontinence. Ciba Foundation Symposium. Chicester: Wiley. 1990: pp. 2–26. Datir A, Connell D. CT-guided injection for ganglion impar blockade: a radiological approach to the management of coccygodynia. Clin Radiol. 2010;65(1):21–25.

6.

De Groat WC. Neurophysiology of the pelvic organs. In Rushton DN, ed. Handbook of NeuroUrology. New York: Marcel Dekker. 1994: pp. 55–93.

7.

Drossman DR, Rome III. The new criteria. Chinese J Digest Dis. 2006;7(4):181–185.

8.

Ger R. Surgical anatomy of the pelvis. Surg Clin N Am. 1988;68:1201–1216.

9.

Gupta D, Jain R, Mishra S, et al. Ultrasonography reinvents the originally described technique for ganglion impar neurolysis in

perianal cancer pain. Anesth Analg. 2008;107(4):1390–1392. 10. Hewitt DJ. The use of NMDAreceptor antagonists in the treatment of chronic pain. Clin J Pain. 2000;16(2 Suppl): S73–79. 11. Johnson PH. Coccygodynia. J Ark Med Soc. 1981;77:421–424. 12. Kabbara AI. Transsacrococcygeal ganglion impar block for postherpetic neuralgia. Anesthesiology. 2005;103(1): 211–212. 13. Karamanolis G, Psatha P, Triantafyllou K. Endoscopic treatments for chronic radiation proctitis. World J Gastrointest Endosc. 2013;5(7): 308–312. 14. Khosla A, Adeyefa O, Nasir S. Successful treatment of radiationinduced proctitis pain by blockade of the ganglion impar in an elderly patient with prostate cancer:

Chapter 38: Chronic rectal pain

a case report. Pain Med. 2013; 14(5):662–626. 15. Laughlin I, Hanling S, Hickey AH. Nonimplantable interventions in chronic pain management: Part 1. In Urman RD, Vadivelu N, eds. Pocket Pain Medicine. Philadelphia, PA: Lippincott Williams and Wilkins. 2011. Kindle Edition. 16. Lincoln J, Surnstock G. Autonomic innervation of the urinary bladder and urethra. In Maggi CA, ed. Nervous Control of the Urogenital System. Chur, Switzerland: Harwood Academic Publishers. 1993: pp. 33–68. 17. Maigne JY, Rusakiewicz F, Diouf M. Postpartum coccygodynia: a case series study of 57 women. Eur J Phys Rehabil Med. 2012;48(3): 387–392. 18. Moore KL, Agur AM. Essential Clinical Anatomy, 3rd edn.

Baltimore: Lippincott Williams & Wilkins. 2007: p. 215. 19. Nathan ST, Fisher BE, Roberts CS. Coccygodynia: a review of pathoanatomy, aetiology, treatment and outcome. J Bone Joint Surg Br. 2010;92:1622. 20. Waldman S. Pudendal nerve block. In Waldman SD, ed. Pain Review. Philadelphia, PA: Saunders Elsevier. 2009: p. 543. 21. Rao SS, Hatfield RA. Paroxysmal anal hyperkinesis: a characteristic feature of proctalgia fugax. Gut. 1996;39:609.

24. Takano M. Proctalgia fugax: caused by pudendal neuropathy? Dis Colon Rectum. 2005;48:114. 25. Toshniwal GR, Dureja GP, Prashanth SM. Transsacrococcygeal approach to ganglion impar block for management of chronic perineal pain: a prospective observational study. Pain Physician. 2007;10(5): 661–666. 26. Vancaillie T, Eggermont J, Armstrong G, et al. Response to pudendal nerve block in women with pudendal neuralgia. Pain Med. 2012;13(4):596–603.

22. Ron Y, Avni Y, Lukovetski A, et al. Botulinum toxin type-A in therapy of patients with anismus. Dis Colon Rectum. 2001;44(12): 1821–1826.

27. Wesselmann U, Burnett AL, Heinberg LJ. The urogenital and rectal pain syndromes. Pain. 1997;73:269–294.

23. Schultz DM. Inferior hypogastric plexus blockade: a transsacral approach. Pain Physician. 2007; 10(6):757–763.

28. Yang CC. Neuromodulation in male chronic pelvic pain syndrome: rationale and practice. World J Urol. 2013;31(4):767–772.

281

Section 4 Chapter

39

Visceral Pain

Pain in pregnancy Eugene Garvin, Jakun Ing, Irene Wu, and Eric S. Hsu

Case study An otherwise healthy 23-year-old female, gravida 1 para 0, at 29 weeks’ gestation presents with sudden onset low back pain with associated paresthesias. She states that the pain began after unloading bags of groceries from her shopping cart to her car. She felt a “pop” in her back but was able to drive home. The pain worsened over the evening and has since limited her mobility significantly. She remains mostly bedbound but is able to get around if she hunches forward and has something to hold on to. Her pain is constant and she rates it a 9/10 on the visual analog scale when she is walking and a 4/10 when she is lying flat in bed. The pain is mostly in her back but occasionally she gets lancinating pain radiating into her right posterior thigh but not past her knee. In addition, she has a “vibration and tingling” sensation over her posterior calf and lateral foot. She denies numbness, weakness, and bowel and bladder incontinence. Her obstetrician has ruled out any problems with the pregnancy that could account for her pain.

1. What are the common etiologies for pain in the pregnant patient? Normal changes during pregnancy predispose the pregnant patient to pain syndromes that are unique to the parturient. These changes also predispose the patient to pain conditions that can occur in nonparturients, such as lumbar disc herniation. Musculoskeletal changes that are associated with pregnancy include exaggerated lordosis of the lumbar spine, forward flexion of the neck, and downward movement of the shoulders. Joint laxity increases particularly in the pubic symphysis and sacroiliac joints. Relaxin, a polypeptide secreted by the corpus luteum,

contributes to this increased laxity, as it is known to soften ligaments around the pelvic joints and cervix, allowing for the eventual vaginal delivery. The anterior pelvis is tilted anteriorly, with increased use of hip extensor, abductor, and ankle plantar flexion muscles. Indeed fluid retention that occurs can lead to nerve compression.

Low back pain and disc disease Back pain occurs at some point during pregnancy in about 50% of women. Back pain most commonly occurs in the second half of pregnancy. In most cases, low back pain is secondary to altered posture, muscle weakness, joint laxity, or facet joint irritation. Disc herniation is rarely a cause for low back pain during pregnancy. Degenerative spondylolisthesis can be aggravated by pregnancy particularly at the L4-L5 level. Indeed direct pressure of the fetus head on the lumbosacral nerves has been postulated as the cause of radicular pain during pregnancy. Physical exam in the parturient is similar to the non-parturient. Special consideration to work-up (i.e., imaging) and treatment (i.e., pharmacologic and interventional management) are discussed in the following sections of this chapter.

Pelvic girdle pain Pelvic girdle pain (PGP) includes pelvic girdle syndrome (pain in all three pelvic joints) as well as unilateral or bilateral sacroiliac joint pain. Risk factors include prior cesarean delivery, multiparity, obesity, young maternal age, emotional stress, low educational level, and early menarche. PGP is usually described as a stabbing pain located in the area of the sacroiliac joint and can occur with pubic symphysis pain. The pain can radiate to the posterior thigh and is

Case Studies in Pain Management, ed. Alan David Kaye and Rinoo V. Shah. Published by Cambridge University Press. © Cambridge University Press 2015.

282

Chapter 39: Pain in pregnancy

worsened with weight-bearing and prolonged sitting. Palpation over the affected joint elicits the pain as well as joint provocation tests including Patrick’s (FABER), Yeoman’s, Gaenslen’s, and pelvic rock. Straight leg raise is usually negative. Treatment includes acupuncture, pelvic brace, and physical therapy. About 80% of women recover within 6 months after delivery.

Pubic symphysis separation (pubic diastasis, symphysiolysis, osteitis pubis) In non-pregnant women, the normal symphysis gap is 4–5 mm. With pregnancy this gap increases by 2–3 mm. The diagnosis of diastasis is separation of more than 10 mm. Risk factors for pubic symphysis separation include fetal macrosomia, multiparity, forceps delivery, rapid progression of labor, or intense contractions. It presents as suprapubic pain that can radiate to the back, hips and legs, and is associated with tenderness and swelling. Pain is increased by weight-bearing, walking, and getting up from a chair. Physical exam reveals pain on palpation of the pubic symphysis, bilateral pressure on the greater trochanter, or by hip flexion with legs in extension. Sometimes a gap can be felt on palpation but the degree of separation does not always correlate with severity of symptoms. Treatment is conservative with bedrest in lateral decubitus position, pelvic brace, physical therapy, and walking with a walker or crutches.

Hip pain and transient osteoporosis of the hip Osteonecrosis and transient osteoporosis of the hip are two relatively rare conditions that occur more frequently in pregnancy. Osteonecrosis of the hip is rare and its etiology is unclear. Physical findings are largely non-specific and MRI is the most sensitive test for diagnosis. The plain radiograph can remain normal despite disease and symptoms. The pathognomonic crescent sign (subchondral radiolucency) is evidence of subchondral collapse. Later stages reveal loss of sphericity, collapse of the femoral head, and ultimately joint space narrowing and degenerative changes. Treatment options include osteotomy, core decompression, and grafts. Transient osteoporosis of the hip also presents with pain and limited range of motion in the hip that is worsened with activity. There is also pain on palpation of the greater trochanter on physical exam.

Diagnosis can be made by MRI or plain radiograph. Treatment is conservative and includes prevention of weight-bearing and the use of crutches.

Knee pain and patellofemoral disorder Postural changes, joint laxity, and increased weight all contribute to knee pain during pregnancy. Patellofemoral disorder usually presents as pain behind the patella. Patella sleeves and quadriceps strengthening exercises can be helpful.

Leg cramps Leg cramps are common in the second half of pregnancy. Treatments include a hot shower or warm tub bath, ice massage, stretching and exercise, hydration, and use of long countered shoes.

Abdominal wall pain The parturient with abdominal wall pain must be evaluated carefully. Serious causes including miscarriage, ectopic pregnancy, and ovarian torsion must be ruled out. Two less serious causes include round ligament pain and abdominal wall hematoma. Both are caused by the increasing size of the uterus. Round ligaments stretch as the uterus rises and small hematomas may develop causing pain. In addition, the abdominal rectus sheath separates and this rapid stretch may also cause small hematomas leading to pain. Both etiologies of pain are treated conservatively with bedrest, local heat, and in severe cases mild analgesics.

2. How does the evaluation for low back pain compare from the parturient to the non-parturient? After careful history, a thorough musculoskeletal and neurologic exam should follow. Exam for pelvic and low back pain should begin with a full musculoskeletal and neurologic exam. Inspection for asymmetry, kyphosis, scoliosis, and gait abnormalities are usually preformed first. Palpation over the area where the patient reports the pain can often provide the most information as to its etiology. Range of motion of the lumbar spine should be assessed. Increased pain on flexion may be indicative of discogenic pain while increased pain on extension may indicate facetogenic pain. Straight leg raise should be performed to assess

283

Chapter 39: Pain in pregnancy

Table 39.1. Guidelines for imaging in the pregnant patient

Determine necessity and explain risks to mother Make attempts to delay imaging until the third trimester Determine the most efficacious use of radiation for the problem Consider MRI if indicated Avoid contrast agents Always weigh risks to fetus with benefit to patient Consent forms are neither required nor recommended. The patient should have the risks explained; however, adding a consent form may increase the perceived risk to the fetus

for radicular symptoms. Pelvic girdle syndrome or sacroiliac joint pain can often be distinguished from other causes of low back pain by physical exam. As discussed previously, sacroiliitis is indicated by positive provocation tests including Patrick’s (FABER), Yeoman’s, Gaenslen’s, and pelvic rock. Straight leg raise is usually negative. Reflexes, sensation, and strength should be evaluated in all muscle groups of the lower extremities. Pregnancy is not an absolute contraindication to plain radiographs. Plain radiographs will contribute vital information when bone lesions are suspected. The typical dose of radiation by three view radiographs is 1.5 rad. No detectable growth or mental abnormalities have been demonstrated with rad levels of less than 10 during pregnancy. EMG and NCS serve as good screening tests when radiculopathy is suspected. However, false-negative EMG’s are common. MRI has revolutionized imaging during pregnancy. Although it appears to be safe, no prospective randomized trials have evaluated harm to the fetus. Guidelines for use of neurodiagnostic imaging in the pregnant patient can be found in Table 39.1.

3. What pharmacotherapy for pain is safe to give in pregnancy? Whenever possible, medication use during pregnancy should be minimized. Normal changes during pregnancy include increased renal blood flow and glomerular filtration rate (GFR), dilutional hypoproteinemia, and increased total body water. These changes affect the absorption, distribution, and elimination of drugs

284

administered to the parturient. Drugs can cross the placenta and can affect fetal development. The period of teratogenesis corresponds with the critical period of organogenesis and begins from conception to the tenth week of gestation. In addition to teratogenesis, drugs have the potential to alter the normal physiology of pregnancy; for example, NSAIDs behave as a tocolytic, may decrease amniotic fluid volume, or place the fetus at increased risk of persistent pulmonary hypertension of the newborn. Possible malformations of the fetus during organogenesis include structural malformations, intrauterine fetal demise, altered fetal growth, altered neurobehavioral growth, acute neonatal toxicity, and withdrawal after birth. The US FDA require labeling of drugs using the Pregnancy Risk Classification system. Below is a table adapted to pain medications (Table 39.2). Notice that the only category A medicine (controlled studies in human have demonstrated no harm) is multivitamins. The only category X medicine (positive evidence in humans or significant risk to the fetus) is ergotamine. All other medications fall in between. Going from category B to D does not necessarily increase risk of teratogenicity. If a mild analgesic is indicated during pregnancy, Tylenol (category B) is the drug of choice. Ergotamine is contraindicated in pregnancy.

4. What role do non-pharmacologic therapies have in treatment for pain in pregnancy? There are a number of non-pharmacologic therapies that parturients may attempt for treatment of back pain. They can be managed as separate therapies or in a multimodal approach.

Exercise and physical therapy Exercise includes prescribed interventions that can include stretching, strengthening, and aerobic exercise. Exercise prescribed for low back pain in parturients is usually similar to that for non-parturients. These exercises may be performed at home or under the supervision of a physical therapist. Exercise performed in water is known as aquatherapy. Exercise may be beneficial for its psychologic benefits as well. Several studies have demonstrated benefit from 8- to 20-week exercise programs.

Chapter 39: Pain in pregnancy

Table 39.2. United States FDA pharmaceutical pregnancy categories

Pregnancy category A

Adequate and well-controlled human studies have failed to demonstrate a risk to the fetus in the first trimester of pregnancy (and there is no evidence of risk in later trimesters)

Multivitamins

Pregnancy category B

Animal reproduction studies have failed to demonstrate a risk to the fetus and there are no adequate and well-controlled studies in pregnant women OR animal studies have shown an adverse effect, but adequate and well-controlled studies in pregnant women have failed to demonstrate a risk to the fetus in any trimester

Acetaminophen, butorphanol, nalbuphine, caffeine, fentanyl, methadone, meperidine, morphine, oxycodone, hydrocodone, hydromorphone, oxymorphone, ibuprofen, naproxen, indomethacin, prednisone, prednisolone

Pregnancy category C

Animal reproduction studies have shown an adverse effect on the fetus and there are no adequate and well-controlled studies in humans, but potential benefits may warrant use of the drug in pregnant women despite potential risks

Amitriptyline, aspirin, ketorolac, betamethasone, cortisone, codeine, propoxyphene, gabapentin, lidocaine, propranolol, sumatriptan, sertraline, fluoxetine, buproprion

Pregnancy category D

There is positive evidence of human fetal risk based on adverse reaction data from investigational or marketing experience or studies in humans, but potential benefits may warrant use of the drug in pregnant women despite potential risks

Imiprimine, carbamezapine, diazepam, paroxetine, phenobarbital, phenytoin, valproic acid

Pregnancy category X

Studies in animals or humans have demonstrated fetal abnormalities and/or there is positive evidence of human fetal risk based on adverse reaction data from investigational or marketing experience, and the risks involved in use of the drug in pregnant women clearly outweigh potential benefits

Ergotamine

Acupuncture Acupuncture describes the ancient Chinese practice of needle penetration of skin at precise locations related to “Qi” and its flow through meridians. These needles may use manual or electrical stimulation, and have been theorized to stimulate endogenous opioid release. The practitioner must be careful not to stimulate meridian points associated with stimulation of labor. The practitioner may also place needles in places not thought to be meridian points, a practice known as sham or placebo acupuncture. Acupuncture appears to be effective in the treatment of common pain syndromes associated with the parturient.

Pelvic belt and pillow The pelvic belt is a form of brace meant to supply lumbar support for the parturient. It decreases the mobility and improves stability of the pelvis and sacroiliac joints. A pillow is a simple intervention used for support and decreasing pressure on painful joints. Both are non-invasive treatments that appear to have some effect in treating low back pain

in the parturient, either alone or in concert with other therapies.

Transcutaneous electronic nerve stimulation Transcutaneous electrical nerve stimulation (TENS) describes the use of electrical current to relieve pain. TENS appears to have limited use in the treatment of low back pain in the parturient.

Osteopathic manipulative treatment Osteopathic manipulative treatment (OMT) describes diagnosis and treatment of pain by an osteopathic physician involving stretching, pressure, and resistance. OMT appears to result in some benefit for low back pain in the parturient.

5. Whatroledonon-opioidmedications have in treating pain in pregnancy? If non-pharmacologic therapy has failed, a parturient with low back pain may benefit from non-opioid agents.

285

Chapter 39: Pain in pregnancy

Acetaminophen Acetaminophen is a category B medication and is generally the first-line medication in the treatment of pain in the parturient.

NSAIDs NSAIDs decrease pain by decreasing prostaglandin production. Prostaglandins are responsible for maintaining patency of the fetal ductus arteriosis and promoting fetal urine production which is vital to maintaining amniotic fluid volume. Use in the third trimester has been associated with closure of the ductus arteriosis and oligohydramnios. In general, NSAID use should be limited to as short a time as possible. However, first trimester exposure to aspirin, ibuprofen, and naproxen do not appear to increase teratogenic risk. Nevertheless, no NSAID should be used past 34 weeks’ gestation because this can lead to pulmonary hypertension and persistent fetal circulation in the newborn. Aspirin poses problems to the mother and the fetus owing to its inhibiting effects on platelets. Aspirin is therefore considered a category C medication. Theoretically, there is increased risk of peripartum hemorrhage, and an increased risk of fetal intracranial hemorrhage has been demonstrated. Low-dose aspirin has not been associated with maternal or neonatal complications. Other NSAIDs such as ibuprofen, naproxen, and indomethacin are considered category B medications. Although ketorolac is the only available injectable NSAID, it is poorly studied in pregnancy and should therefore be avoided. Ketorolac is considered a category C medication, as researchers found increased risk of dystocia in rodents.

Benzodiazepines Benzodiazepines are usually prescribed for anxiety and insomnia, but in the parturient with back pain, they could theoretically play a role as a muscle relaxant. Nevertheless, the association between benzodiazepines and potential congenital malformations has generally dissuaded their use in parturients. Diazepam is considered a category D medication and may be associated with cleft lip and cleft palate along with congenital inguinal hernia, though the data supporting this is unclear. Neonates born to

286

parturients on chronic benzodiazepines may be at risk for withdrawal.

Antidepressants Antidepressants, particularly SSRIs, have been used effectively to manage depression associated with pain, and should generally be continued during pregnancy, though there has been some concern about their use in parturients. All the SSRIs are considered category C medications, and some studies have demonstrated a small increase in risk of congenital malformations, particularly cardiac. There has also been some concern of neonatal abstinence syndrome and pulmonary hypertension in the neonate. SNRIs such as duloxetine, often used for both depression and neuropathic pain, are considered category C medications. Tricyclic antidepressants are considered category D medications. Animal studies suggest teratogenic potential, though this has generally not been shown in human studies.

Anticonvulsants Anticonvulsants, often used in the management of neuropathic pain, may be problematic in the parturient population. Valproic acid, phenytoin, and carbamazepine, all category D medications, have been associated with congenital malformations such as neural tube defects and fetal hydantoin syndrome. Gabapentin and pregabalin are both considered category C medications.

Local anesthetics Continuous administration of local anesthetic is generally confined to analgesic patches and creams. The most commonly used local anesthetic, lidocaine, is considered a category C medication. Mexiletine, used for neuropathic pain and taken orally, is also a category C medication.

Skeletal muscle relaxants In general, the use of skeletal muscle relaxants has not been well-studied in the parturient. Cyclobenzaprine is notable in that it is considered a category B medication. Others, including methocarbamol, metaxalone, carisoprodol, orphenadrine, baclofen, and tizanidine, are considered category C medications, without adequate data to demonstrate harm or safety to the human fetus.

Chapter 39: Pain in pregnancy

Steroids Oral steroids are sometimes prescribed to decrease the inflammation associated with lower back pain. Prednisone and prednisolone are largely inactivated by the placenta and therefore are considered category B medications. Other steroids such as hydrocortisone and dexamethasone are considered category C medications.

6. What role do opioids have in treating pain in pregnancy? If started on opioids, what is the risk to the mother and the fetus, both in gestation and the peripartum stage? Opioids pose challenges for the management of both opioid-naïve and opioid-dependent parturients. Most opioids are classified as category B medications, with the exception of codeine which is considered a category C medication (association with congenital malformation). However, opioids used for long periods of time may be considered category D for their risk of fetal opioid dependence and risk of fetal withdrawal syndrome. Chronic methadone use in parturients appears to be associated with a lower incidence of neonatal abstinence syndrome than chronic opioid use. Virtually all opioids have been safely used in parturients, including hydrocodone, oxycodone, fentanyl, morphine, and hydromorphone. Neuraxial opioids may help decrease overall narcotic requirement. Opioids may cause fetal bradycardia and a loss of fetal heart rate variability, which may confuse the clinical picture if fetal hypoxemia is involved. Opioids administered prior to delivery may also result in neonatal respiratory depression. Meperidine is effective for pain management but its metabolite, normeperidine, may accumulate after multiple doses. Mixed agonist–antagonists such as nalbuphine do not appear to offer any additional benefit, and nalbuphine in particular has been known to cause a sinusoidal fetal heart rhythm. Tramadol has been increasingly used to treat chronic pain in parturients for its lower incidence of opioid dependence. While neonates possess the necessary hepatic function to metabolize tramadol into its active metabolite, immature renal function delays clearance. Any neonate whose mother has been using

tramadol for chronic pain is at risk for neonatal abstinence syndrome. Opioid management in opioid-dependent parturients poses unique challenges. Chronic opioid use may result in opioid tolerance and the requirement of higher opioid doses. In addition, parturients managed on buprenorphine for chronic opioid dependence may demonstrate resistance to full µ-receptor agonists. On the other hand, acute withdrawal from opioids may trigger fetal withdrawal syndrome, resulting in fetal tachycardia and fetal demise. Note that naloxone may also trigger fetal withdrawal syndrome, and it should be used sparingly in the setting of opioid dependence in the parturient. If the patient is on chronic methadone, methadone therapy should be continued. Most likely, the parturient will require short-acting opioids for acute pain. Patients on buprenorphine may require larger doses of short-acting opioids for acute pain. Alternatively, buprenorphine may be discontinued in favor of methadone for maintenance treatment, making more µ receptors available for the use of short-acting opioids.

7. What interventions are available to the parturient? What are the risks associated with pain procedures for the parturient and the fetus? Interventional pain procedures raise challenging issues in the parturient. Many interventional pain consultants will defer procedures until after the pregnancy. The two most significant issues involve gestational age and quantity of absorbed radiation. Most procedures are generally performed under fluoroscopy, which can generate fetal radiation exposure, increasing the risk of mutations and abortion. Fluoroscopy may result in 1 to 5 rad each minute. Even dosage of 5 rad can be significantly harmful prior to the 15th week of gestation, which suggests that any procedures involving fluoroscopy should be postponed until at least then, if not until after delivery. As an alternative, certain procedures may be attempted under ultrasound guidance instead of fluoroscopy. Sacroiliac joint, transforaminal, translaminar, and caudal epidural steroid injections have all been successfully performed in the non-parturient under ultrasound guidance. Unfortunately, the outcome data for the success of ultrasound-guided

287

Chapter 39: Pain in pregnancy

procedures is limited, and specific challenges regarding the parturient have not been well described. While technical limitations of ultrasound guidance include the inability to visualize the spread of injectate and difficulty determining whether injections are intravascular, the biggest limitation of ultrasound guidance may be infrequent use and lack of familiarity with the technique. Another concern is the use of corticosteroid in these procedures. Among the corticosteroids commonly used in pain procedures, both triamcinolone and dexamethasone are category C medications, given findings of potential teratogenic effects in animals. Limited data involving human parturients has not associated corticosteroid exposure with increased risk of congenital malformations.

2. 3.

4.

8. Whatwouldbethesuggestedoverall therapeutic management for the parturient with lower back pain? All therapies for low back pain should involve a joint discussion between the pain management consultant and the patient regarding risks and benefits. In general, the goal should be to treat the pain adequately in the parturient without needlessly exposing the fetus to medications or radiation. 1. Non-pharmacologic measures such as exercise, physical therapy, pelvic belt, and pillow should be

References 1.

Alford DP, Compton P, Samet JH. Acute pain management for patients receiving maintenance methadone or buprenorphine therapy. Ann Intern Med. 2006;144:127–134.

2.

Berg G, Hammar M, MollerNielsen J, et al. Low back pain during pregnancy. Obstet Gynecol. 1988;71:71–75.

3.

Chen CP, Wong AM, Hsu CC, et al. Ultrasound as a screening tool for proceeding with caudal epidural injections. Arch Phys Med Rehabil. 2010;91:358–363.

4.

288

Fast A, Shapiro D, Ducommun EJ, et al. Low back pain in pregnancy. Spine. 1987;12:368–371.

5.

first-line therapies for managing lower back pain in the parturient. In the case of specific pathologies such as PGP, pubic symphysis separation, and transient osteoporosis of the hip, the patient should consider specific nonpharmacologic treatments such as those described in their respective sections earlier in this chapter. If medications become necessary, then acetaminophen should be the first choice. NSAIDs are a reasonable alternative or addition to acetaminophen. NSAIDs and aspirin should be avoided in the third trimester. Adjuncts such as cyclobenzaprine may also be considered. If pain remains uncontrolled, opioids should be considered as discussed above. If chronic opioid therapy is necessary, then methadone would be a reasonable choice. Long-term opioid therapy is associated with fetal dependence, but discontinuing chronic opioid therapy could result in fetal withdrawal syndrome. Neonates born to mothers on chronic opioids must be treated to prevent neonatal abstinence syndrome. Interventional pain procedures would be of last resort for the parturient given the risk of significant harm to the fetus, specifically regarding fluoroscopy. However, pregnancy is NOT an absolute contraindication. Ultrasound guidance should be considered to decrease the risk to the fetus.

5.

Finnegan LP, Connaughton JF, Kron RE, et al. Neonatal abstinence syndrome: assessment and management. Addict Dis. 1975;2:141–158.

9.

Johnson RE, Fudala PJ, Payne R. Buprenorphine: considerations for pain management. J Pain Symptom Manage. 2005;29: 297–326.

6.

Greher M, Scharbert G, Kamolz LP, et al. Ultrasound-guided lumbar facet nerve block: a sono-anatomic study of a new methodologic approach. Anesthesiology. 2004;100: 1242–1248.

10. Kuhnert BR, Kuhnert PA, Philipson EH, et al. Disposition of meperidine and normeperidine following multiple doses in labor. Am J Obstet Gynecol. 1985;151: 410–415.

7.

Heckma JD, Sassard R. Musculoskeletal considerations in pregnancy. J Bone Joint Surg. 1994;76A:1720–1730.

8.

Shah RV. The management of nonobstetric pains in pregnancy. Reg Anesth Pain Med. 2003; 28(4):362–363.

11. Laegreid L, Olegard R, Walhstrom J, et al. Abnormalities in children exposed to benzodiazepines in utero. Lancet. 1987;1:108–109. 12. Levy M, Spino M. Neonatal withdrawal syndrome: associated drugs and pharmacologic management. Pharmacotherapy. 1993;13:202–211.

Chapter 39: Pain in pregnancy

13. Mattison DR, Anguaco T, Miller FC, Quick GJ. Magnetic resonance imaging in maternal and fetal medicine. J Perinatol. 1989;9: 411–419. 14. Moise KJ, Huhta JC, Sharif DS, et al. Indomethacin in the treatment of premature labor: effects on the fetal ductus arteriosis. N Engl J Med. 1988; 319:327–331. 15. Montella BJ, Nunley JA, Urbaniak JR. Osteonecrosis of the femoral head associated with pregnancy: a preliminary report. J Bone Joint Surg Am. 1999;81(6):790. 16. Moses-Kolko EL, Bogen D, Perel J, et al. Neonatal signs after late in utero exposure to serotonin

reuptake inhibitors: literature review and implications for clinical applications. JAMA. 2005;293:2372–2383.

20. Slone D, Heinonen OP, Kaufman DW, et al. Aspirin and congenital malformations. Lancet. 1976;1:1373–1375.

17. Ostgaard HC, Zetherström G, Roos-Hansson E, Svanberg B. Reduction of back and posterior pelvic pain in pregnancy. Spine. 1994;19(8):894.

21. Snow RE, Neubert AG. Peripartum pubic symphysis separation: a case series and review of the literature. Obstet Gynecol Surg. 1997;52(7):438.

18. Pennick V, Liddle SD. Interventions for preventing and treating pelvic and back pain in pregnancy. Cochrane Database Syst Rev. 2013;8:CD001139.

22. Vleeming A, Albert HB, Ostgaard HC, Sturesson B, Stuge B. European guidelines for the diagnosis and treatment of pelvic girdle pain. Eur Spine J. 2008; 17(6):794–819.

19. Rayburn WF, Bogenschutz MP. Pharmacotherapy for pregnant women with addictions. Am J Obstet Gynecol. 2004;191: 1885–1897.

23. Zelson M, Lee SJ, Casalino M. Neonatal narcotic addiction. N Engl J Med. 1973;289: 1216–1220.

289

Section 4 Chapter

40

Visceral Pain

Postpartum pain Jeffry Chen, Eric S. Hsu, and Irene Wu

Case study A 33-year-old gravida 1, para 1 female who is now 16 weeks postpartum presents to clinic with a 4-week history of low back pain. Patient has a previous history of lumbar radiculopathy successfully treated with epidural steroid injection and low-dose opioids prior to pregnancy. Patient did not experience an exacerbation of the low back pain during pregnancy. She had an uncomplicated vaginal delivery with labor epidural. She continues to breastfeed.

1. What are common etiologies of back pain in a postpartum patient? Back pain, similar to the general population, is the most common presenting pain complaint in postpartum patients. Common etiologies of back pain in a postpartum patient are:  Acute muscle sprain/strain  Degenerative disk disease  Disc herniation  Facet arthropathy  Sacroiliac joint dysfunction  Myofascial pain syndrome  Scoliosis-related pain  Piriformis syndrome  Sacral insufficiency fracture  Coccygodynia  Spinal stenosis  Vertebral fractures  Spinal primary and metastatic lesions  Infectious such as osteomyelitis, paraspinal or epidural abscess, or discitis

2. What is the correlation, if any, between neuraxial analgesia and chronic back pain? Many postpartum patients who received neuraxial anesthesia for delivery may attribute their back pain to the neuraxial technique and will likely ask the pain physician for their opinion. Thus far, limited studies are available to suggest that labor epidural is related to chronic low back pain. In a retrospective study using postal surveys of over 11 000 women, MacArthur et al found that back pain was more common in those who delivered vaginally with epidural than those without epidural, 18.9% versus 10.5% respectively. Several subsequent prospective studies have failed to demonstrate a correlation between neuraxial anesthesia and chronic low back pain. MacArthur et al performed a prospective cohort study that followed up 329 women 1 year after vaginal delivery, 164 who chose labor epidural analgesia and 165 who did not. They found no difference in the prevalence of back pain between the two groups nor did they find a difference in the pain scores within the two groups. In a secondary analysis of a prospective study comparing epidural analgesia versus midwifery support, Orlikowski et al also found no relationship between epidural analgesia and back pain at 6 months. Howell et al conducted a follow-up study of a previously randomized control trial involving 369 women randomized to receive either epidural labor analgesia versus non-epidural labor analgesia. Of the original 369 women, 151 from the epidural group and 155 from the non-epidural group participated in the follow-up study. They found no

Case Studies in Pain Management, ed. Alan David Kaye and Rinoo V. Shah. Published by Cambridge University Press. © Cambridge University Press 2015.

290

Chapter 40: Postpartum pain

differences in the onset or duration of low back pain, disability, or movement restriction between the two groups. Furthermore, no studies have demonstrated a dose–response gradient between epidural anesthesia and back pain. It is therefore unclear whether lower local anesthetic concentration and subsequent lesser motor block reduces the likelihood of back pain.

3. What other pain syndromes are common or unique to the postpartum female? Chronic pelvic girdle pain Pregnancy-related hormone-induced ligament laxity in combination with low muscle endurance can impair the dynamic stability of the pelvis, thereby leading to increased pelvic girdle pain. Chronic pelvic girdle pain is experienced between the posterior iliac crest and the gluteal fold, predominantly near the sacroiliac joints, and may radiate to the posterior thigh. Pain may also be experienced in conjunction with, or exclusively in, the symphysis. The incidence of chronic pelvic girdle pain is approximately 45% during pregnancy and 25% in the postpartum period. The pain typically begins in the second trimester and will usually resolve within weeks to a few months post-delivery, although about 10% will continue to have chronic pain for many months to years.

Meralgia paresthetica Meralgia paresthetica is an entrapment neuropathy characterized by paresthesia and numbness in the anterolateral thigh, commonly described as a burning, stinging, or tingling sensation without other findings. The condition is commonly found in patients who are obese, pregnant, or other conditions associated with increased abdominal pressure. Complaints are usually unilateral. The etiology is typically entrapment of the lateral femoral cutaneous nerve (LFCN) as it courses around the anterior superior iliac spine (ASIS) and either under or through the inguinal ligament. As such, symptoms can usually be reproduced with palpation along the inguinal ligament just medial to the ASIS. The condition is typically self-limited and typically resolves within 4–6 months of presentation.

Chronic pelvic pain Chronic pelvic pain is a disabling condition defined as non-menstrual pain in the lower abdomen present for at least 6 months. Gynecologic factors such as endometriosis, intrapelvic adhesions, and chronic inflammatory disease are associated with CPP. Neurologic factors including pudendal neuropathy and neuropathy secondary to Pfannenstiel incision are also associated with CPP syndromes.

Chronic pain after cesarean delivery Chronic pain after delivery has been found to be more frequent after cesarean delivery as compared to vaginal delivery. Chronic pain after cesarean delivery is defined as abdominal wound scar pain persisting for more than 3 months post-delivery and unrelated to menstrual pain. Currently the available data shows a relatively low incidence of chronic pain after cesarean delivery with rates ranging from 1% to 18%. Patients can present with ongoing postPfannenstiel pain, which can occur at the level of incision or at the lateral ends of the scar. Entrapment of the iliohypogastric and/or ilioinguinal nerve by scar tissues has been identified as a possible cause of this pain.

Chronic perineal pain Perineal pain is extremely common after delivery affecting over 40% of women at 10 days postpartum. The pain usually subsides within weeks but it has been reported that 10% of patients may still experience pain at 18 months after delivery. Deep abdominal/ pelvic pain, dyspareunia, and pain while walking or sitting are common presenting symptoms. Difficult labor with prolonged second stage and assisted vaginal delivery with or without episiotomy are frequently mentioned as the triggering events. As a result, slowly resolving inflammatory reaction and scar tissue formation in the perineum may lead to chronic pain in the lower genital tract.

4. What are common risk factors for chronic postpartum pain? Risk factors for chronic postpartum pain include personal characteristics, pre-delivery, intra-delivery, and post-delivery factors.

291

Chapter 40: Postpartum pain

Pre-delivery  Psychosocial factors  Chronic pain conditions (i.e., back pain, migraines, menstrual pain, scar hyperalgesia from previous cesarean delivery)  Genetic susceptibility

During delivery  Type of anesthesia (i.e., general anesthesia, spinal). General anesthesia has been presumed to be associated with increased noxious input as compared to spinal anesthesia. The increased noxious input can lead to central sensitization in second-order neurons, such that the patients are at increased risk of persistent pain.  Surgical factors (i.e., emergency cesarean delivery, repeat incision > 2, length of Pfannenstiel incision, uterine exteriorization, closure of peritoneum)

Post-delivery  Severe and prolonged postoperative course resulting in “wind-up” and pain hypersensitivity  Postpartum depression

5. How should a patient with postpartum pain be evaluated? Targeted pain history     

Mode of onset Location Duration and frequency Character and severity Associated and precipitating factors (i.e., dysmenorrhea, vaginal bleeding, dyspareunia, numbness, paresthesias)  Pertinent socioeconomic considerations and psychiatric history  Type of delivery (i.e., spontaneous vaginal, assisted delivery, cesarean)  Previous exposure and duration of treatment with opioids or other medications used in chronic pain

Targeted physical examination  Musculoskeletal exam including an examination of the spine for focal areas of tenderness as well as a range of motion assessment

292

 Pelvic exam  Examination of Pfannenstiel incision, if present, for focal areas of tenderness  Motor examination to identify any associated extremity weakness, atrophy, or loss of muscle tone  Sensory examination to identify any focal areas of sensory loss

Imaging and other diagnostic modalities      

x-ray CT/MRI Ultrasound Electromyography and nerve conduction studies Laparoscopy Cystoscopy

In contrast to the parturient, there are no contraindications to radiographs in the postpartum patient. The postpartum patient can therefore safely undergo a comprehensive evaluation and work-up for her pain.

6. What non-pharmacologic treatment options for postpartum pain are available in a breastfeeding female? Physical therapy Physical therapy can include land- or water-based therapy. Physical therapy can range from general physical fitness and stretching exercises to aerobic activity. Spinal manipulation, which is defined as a high velocity thrust to a joint beyond its restricted range of movement, as well as soft tissue/myofascial release can also be included. Manual therapy is thought to influence the spinal “gating” mechanism and the descending pain suppression system at spinal and supraspinal levels to decrease pain. Osteopathic manipulation including stretching, gentle pressure, and resistance can also be considered.

Pelvic belts Pelvic belts can reduce mechanical loading and stabilize the lumbar spine along with pelvic floor muscles.

Acupuncture Acupuncture is needle puncture at classical meridian points, aimed at promoting the flow of “Qi” or

Chapter 40: Postpartum pain

energy. Acupuncture is thought to stimulate the body’s own pain-relieving opioid mechanisms.

TENS Electrical stimulation, including TENS, controls pain non-invasively and non-pharmacologically, and has a wide range of clinical applications. The TENS unit emits low-voltage electrical impulses that vary in frequency and intensity. These electrical pulses are thought to stimulate nerve pathways in the spinal cord, thereby blocking the transmission of pain. When applied to the lower back, the TENS unit emits electrical impulses which excite afferent nerves, and thus inhibits the transmission of painful stimuli arising from the region.

Biofeedback and relaxation training Biofeedback is accomplished through specialized equipment that provides information about physiologic responses occurring within the body (i.e., muscle activity). The technique has been shown to be effective in reducing peripheral nociceptive afferent activity and enhancing descending inhibitory pathways, through muscle relaxation. The technique also helps patients learn to cope more effectively with pain and its associated symptoms.

7. What pharmacologic agents are available for treatment of postpartum pain in a breastfeeding female? The breastfeeding female, similar to the pregnant female, presents a unique challenge when it comes to medication management such that the physician must also consider the well-being of the child in addition to the patient. The conditions that affect transfer of medications across the placenta also apply to transfer of medication to breast milk. Factors such as low molecular weight, minimal protein binding, high lipid solubility, and the un-ionized state all increase transfer of medication into breast milk. Typically only about 1–2% of the maternal dose is transferred to the fetus via breastfeeding. Breast milk is mainly synthesized and excreted during and immediately following breastfeeding; therefore, administration of medications after breastfeeding and when there is a long interval between feedings will help to minimize drug transfer.

Acetaminophen Acetaminophen does enter breast milk but dosage to the neonate is less than 2%. It is considered a safe medication during breastfeeding.

NSAIDs NSAIDs are acidic drugs with low lipid solubility and high protein binding, features that mitigate against substantial transfer into breast milk, compared with plasma. In general, NSAIDS, apart from aspirin in doses above 150 mg/day, are considered safe during lactation. Toxic effects on breastfed infants exposed to large doses of salicylates (2–4 g/day) have been known, as neonates have very slow elimination of salicylates. It should also be noted that NSAIDs have antiplatelet effects, so there is a potential effect on neonatal platelet dysfunction. NSAIDs exhibiting both COX-1 and COX-2 inhibition should not be taken by women who are breastfeeding neonates, especially infants who have thrombocytopenia or platelet dysfunction. The COX-2 specific inhibitors may be suitable because they show no or less platelet inhibition in adults.

Local anesthetics Both bupivacaine and lidocaine do enter breast milk, but only in minimal quantities even after continuous infusions during labor and even in higher doses as used for suppression of cardiac dysrhythmias. Local anesthetics are therefore considered safe for breastfeeding mothers.

Steroids Less than 1% of maternal prednisone appears in the neonate and this small amount is unlikely to affect the infant’s cortisol levels.

Antidepressants The tricyclic antidepressants appear to be relatively safe during breastfeeding. They attain similar levels in breast milk and maternal plasma, but do not accumulate in the neonate. Doxepin does not enter breast milk in high concentrations, although sedation and respiratory depression in a breastfeeding infant have been reported. Amitriptyline, nortriptyline, and imipramine appear to be the most appropriate medications for the lactating mother.

293

Chapter 40: Postpartum pain

The selective serotonin reuptake inhibitors are also relatively safe during breastfeeding. They enter the breast milk, but do not attain detectable levels in infant plasma and urine. There has been no evidence of neurologic pathology in the breastfed infant with selective serotonin reuptake inhibitors.

Benzodiazepines Benzodiazepines such as diazepam, temazepam, and clonazepam exhibit low milk:plasma ratios with undetectable serum levels in breastfeeding infants, unless exposure to these agents commenced in the antenatal period. However, sedation in infants is possible with benzodiazepine use during breastfeeding, as the medication metabolite can be detected in the infant for up to 10 days after a single dose.

Opioids Opioid analgesics are readily transferred into breast milk. Morphine and codeine concentrations in breast milk have been found to be equal or higher than maternal blood concentrations. Morphine, codeine, hydrocodone, fentanyl, and hydromorphone have been determined to be moderately safe for breastfeeding women. However, morphine is the preferred opiate to use by a breastfeeding mother due to its poor oral bioavailability. Additionally, hydrocodone should be used instead of codeine in the rare instance that the mother is an ultra-rapid metabolizer of codeine, which results in the breakdown of codeine into therapeutically excessive levels of morphine leading to central nervous system depression. Meperidine is converted to the active metabolite normeperidine. The half-life of normeperidine in the neonate is markedly prolonged and resultant accumulation increases the risk of neurobehavioral depression and seizures in the newborn.

8. What medications should be used with caution in the breastfeeding mother? Psychotropic drugs are of special concern when given to nursing mothers for long periods. Although there are very few case reports of adverse effects in breastfeeding infants, these drugs do appear in human milk and, thus, could conceivably alter short-term and long-term central nervous system function.

294

Gabapentin Limited studies exist for gabapentin, and thus the safety of breastfeeding while taking gabapentin should always be subject to a risk–benefit analysis for the individual mother and her infant.

Pregabalin Currently there are no studies that evaluate the passage of pregabalin into breast milk and therefore it should only be administered if absolutely necessary.

Antidepressants Limited data on the safety profile of serotoninnorepinephrine reuptake inhibitors and breastfeeding is available, so the use of such medications during breastfeeding is cautioned.

Lithium Lithium is copiously secreted into breast milk, at levels up to 50% of maternal serum levels. Immature neonatal renal function also increases the risk of toxicity.

9. What guidelines can a physician follow in management of postpartum pain in a breastfeeding mother? Postpartum pain in the breastfeeding female presents a significant challenge, in terms of effectively treating the pain while simultaneously minimizing exposure of the child to harmful medication. The following recommendations are suggested:  Undertake a risk–benefit assessment tailored to the individual patient. Evaluate the severity of the underlying disorder, the consequences of leaving it untreated, and potential adverse effects of medication on both mother and infant.  Non-pharmacologic interventions in the form of physical therapy, acupuncture, biofeedback, and relaxation therapy should be considered before prescribing medications.  If treatment is deemed appropriate, the smallest number of medications at the lowest possible dose consistent with control of the pain should be prescribed.  Breastfeeding mothers on medication should be monitored closely and educated with regard to early detection of signs of drug toxicity in their

Chapter 40: Postpartum pain

infants. A baseline examination of the infant followed by careful monitoring of signs of toxicity in the infant is strongly recommended.  Mothers should breastfeed prior to the scheduled medications, and use stored breast milk from “non-peak” periods rather than breastfeeding at times when the maternal plasma concentrations are high.  Interventional procedures can be considered when non-pharmacologic interventions have failed. Unlike the parturient, postpartum patients no longer have restrictions on interventional pain management. The patient can now undergo procedures that require fluoroscopy, which was

References 1.

2.

3.

4.

American Academy of Pediatrics Committee on Drugs. Transfer of drugs and other chemicals into human milk. Pediatrics. 2001;108: 776–789. Andrews V, Thakar R, Sultan AH, Jones PW. Evaluation of postpartum perineal pain and dyspareunia: a prospective study. Eur J Obst Gynecol Reprod Biol. 2008;137:52–156. Bloor M, Paech M. Nonsteroidal anti-inflammatory drugs during pregnancy and the initiation of lactation. Anesth-Analg. 2013;16:1063–1075. Van Diver T, Camann W. Meralgia paresthetica in the

previously contraindicated during pregnancy. With regard to positioning, the prone position is no longer problematic and the fully supine position is no longer associated with hypotension associated with aortocaval compression.  The rate and amount of transfer of injected steroids to breast milk is unknown at this time. Therefore, breastfeeding mothers should be cautioned regarding breastfeeding periprocedurally. Procedural interventions must be scheduled appropriately with careful monitoring of mother and infant to minimize transfer of maternally administered medications into breast milk.

parturient. In J Obstet Anesth. 1995;4:109–112. 5.

6.

7.

Ferreira CW, Alburquerque-Sendı NF. Effectiveness of physical therapy for pregnancy-related low back and/ or pelvic pain after delivery: a systematic review. Physiother Pract. 2013;29(6):419–431. Harney D, Patijn J. Meralgia paresthetica: diagnosis and management strategies. Pain Med. 2007;8(8):669–677. Howell CJ, Dean T, Lucking L, Dziedzic K, Jones PW, Johanson RB. Randomized study of long term outcome after epidural versus non-epidural analgesia during labor. BMJ. 2002; 325:357.

8.

Landau R, Bollag L, Ortner C. Chronic pain after childbirth. Int J Obstet Anesth. 2013;22: 133–145.

9.

Macarthur AJ, Macarthur C, Weeks SK. Is epidural anesthesia in labor associated with chronic low back pain? A prospective cohort study. Anesth Analg. 1997;85:1066–1070.

10. Moretti M. Breastfeeding and the use of antidepressants. J Pop Therapeut Clin Pharmacol. 2012;19(3):e387–390. 11. Sundquist J-C. Long-term outcome after obstetric injury: a retrospective study. Acta Obstet Gynecol Scand. 2012;91:715–718.

295

Section 5 Chapter

41

Headaches and Facial Pain

Patient with migraine headaches Natalia Murinova, Daniel Krashin, and Andrea Trescot

Case study A 21-year-old woman presents with headaches that have been present since she was 13 years old with onset of her menstrual periods. The headaches occur about 1 to 2 times a month, and can last up to 8 hours without treatment. Scintillating scotoma in her right visual field precede her typical headaches. These visual changes last for about 30 minutes, and are followed by a hemicranial throbbing headache. During these visual changes, she is unable to drive. The headache can be so disabling that she has to leave work, and go home. She prefers to lie down and go to sleep. She is really concerned about these headache episodes, and especially about her loss of vision. She thinks something is really wrong with her, and that she has a brain tumor or a stroke. She had normal CT of her brain and normal exam. In this case this patient is presenting with migraine with aura. Aura is seen only in 20% to 30% of patients with migraine. The visual changes of aura typically precede the migraine pain. Most commonly aura is described as a positive visual phenomena such as flashing lights compared to a stroke, which usually has negative symptoms (complete darkness with loss of vision instead of flashing lights). Fewer than 50% of patients with active migraine see a medical provider each year for their management.[1,2]. Many sufferers with severe disability never see a physician for their migraines.

1. Discuss the epidemiology of migraine Migraine is a primary headache disorder. Approximately 30 million Americans have severe migraines.[3] The prevalence of migraine is about 7% in men and about 20% in woman.[4] Peak prevalence occurs between the mid 20s and mid 40s; therefore, most

migraine occurs in a young, healthy population.[5] The difference in prevalence between genders cannot be attributed solely to hormonal differences; prevalence is substantially higher in women compared to men, even after 70 years of age.[5] Migraine with aura (MA) represents about a third of migraine patients. Disability related to the severe migraine is seen in 92% of women and 89% of men.[2] The productivity losses due to migraine in the USA are estimated to be 13 billion dollars per year.[6]

2. Discuss the pathogenesis of migraine The current accepted model of migraine is the trigeminovascular or neurovascular model, as activation of the trigeminovascular system is the main pathophysiology explaining migraine.[4] The aura, preceding the migraine pain, is a presumed wave of depolarization of cortex named cortical spreading depression (CSD). The CSD is a complex phenomenon. CSD involves changes of cortical activity, neuronal activation, and blood flow. These changes are accompanied by intense neural activation and release of neurotransmitters, inflammatory mediators, and other neural factors. The pain of migraine is attributed to inflammation and activation of nociceptive afferents in the meningeal and dural vasculature. Perivascular neurogenic inflammation is associated with increased nociception and release of inflammatory mediators and nociceptive neurotransmitters, which further sensitize and activate the vascular afferents. These stimuli further activate brainstem nuclei through trigeminal afferents into the trigeminal nucleus caudalis and then stimulate third order nuclei in thalamus and cortex. The brainstem activation closes the loop by sensitization of the trigeminovascular system, perpetuating the migrainous

Case Studies in Pain Management, ed. Alan David Kaye and Rinoo V. Shah. Published by Cambridge University Press. © Cambridge University Press 2015.

297

Chapter 41: Patient with migraine headaches

state. The likely origin of spontaneous migraine attacks is brainstem, identified on PET imaging.[7] In conclusion, migraine represents complex functional interactions between the trigeminocervical complex, brainstem nuclei, thalamus, cortex, and the cerebral vasculature.[8,9]

3. Clinical assessment and associated signs The most satisfying part of headache diagnosis is that just by talking to your patient, and recognizing certain patterns of headache, you can make a clinical diagnosis. Even though you will still want to proceed with a clinical examination, taking a good clinical history will often dictate the appropriate treatment, and you will avoid unnecessary tests. We treat people, not disorders, and that is why taking a history is truly “sometimes art.” The history is based on International Headache Society (IHS) criteria, considered the gold standard for headache diagnosis. When you elicit the characteristic history of unilateral headache located in the trigeminal area, with pounding that is so severe that the patient prefers to lie down quietly in a dark room, and has associated nausea, light sensitivity, sound sensitivity, and noise sensitivity, you are dealing with a “migraineur.” The neurologic examination is normal in migraine. The diagnosis of migraine can be done with no further investigations. I recommend clarifying the monthly number of headache days using a headache diary. If the headache number exceeds 15 days a month, the patient likely has chronic migraine. Chronic migraine is commonly triggered by medication overuse. Triggers of migraines are typically cumulative and include: bright lights, cigarette smoke, fasting, emotional stress, exercise, poor sleep, alcohol, caffeine, aged cheese and other foods with tyramine, chocolate, glutamate, nitrates, and menstruation. Proceed with further investigation with neuroimaging or lumbar puncture only if you elicit “red flags” in the history or exam.

International Headache Society Diagnosis Criteria for Migraine Migraine without aura (MO) diagnostic criteria[10]  “At least five headache attacks lasting 4–72 hours (untreated or unsuccessfully treated), which has two of the four following characteristics:

:

298

unilateral location;

: : :

pulsating quality of the pain; moderate or severe intensity aggravated by movement  at least one of the two following symptoms:

: :

phonophobia and photophobia nausea and/or vomiting  In children the attacks may be only 1 hour long.  If the patient has severe pain, aggravated by movement, but no associated migraine features, and has had at least five attacks of this type of headache, the usual term is probable migraine and treated as migraine NB Aura is not a part of the most common type of migraine; migraine without aura is more common than migraine with aura.

Migraine with aura (MA) diagnostic criteria[10] < 20% of all migraine cases  A. At least two attacks with at least three of the following: 1. One or more fully reversible aura symptoms indicating focal cerebral cortical and/or brain stem functions 2. At least one aura symptom develops gradually over more than 4 minutes, or two or more symptoms occur in succession 3. No aura symptom lasts more than 60 minutes; if more than one aura symptom is present, accepted duration is proportionally increased 4. Headache follows aura with free interval of at least 60 minutes (it may also simultaneously begin with the aura)  B. At least one of the following aura features establishes a diagnosis of migraine with typical aura: 1. Homonymous visual disturbance – the visual phenomenon accounts for 90% of auras; usually aura includes positive phenomena such as flashing lights (photopsia), bilateral scotoma, or a fortification spectrum (teichopsia). Complex visual disturbances include macropsia/micropsia or metamorphopsia (Louis Carol had migraines and used his migraine-related visual distortions as the basis for the book Alice in Wonderland).

Chapter 41: Patient with migraine headaches

2. Unilateral paresthesias and/or numbness (30% of auras) 3. Unilateral weakness 4. Aphasia or unclassifiable speech difficulty Usually there are other symptoms that can occur in conjunction with aura that include clumsiness (20%), apraxias, speech disturbances, déjà vu, jamais vu, and delirium.

4. Counseling on long-term risks of migraine White matter hyperintensities (WMH) are a common, incidental finding on the brain MRI. Although migraine sufferers most commonly have normal MRIs of their brains, the most common abnormality in migraineurs is WMH. Migraine patients are four times more likely to have WMH on MRI brain compared to non-migraineurs.[11] The cause of WMH in migraine is not known. Upon hearing that they have white matter changes or an abnormal brain MRI, patients typically become very concerned, and it is vital to inform them that these findings do not equal stroke and that their clinical significance is unclear. Likewise, it is prudent to explain that these findings are not related to multiple sclerosis, particularly since this phrase is commonly used in discussions of multiple sclerosis on the internet and elsewhere. Thus, it is beneficial to educate patients that these changes are often seen in migraine patients and do not represent an additional brain disease state.

Stroke Many studies have confirmed the association of migraine with ischemic stroke.[12–14] This is extremely rare. The large Atherosclerosis Risk in Communities (ARIC) study of middle-aged adults found that migraine with aura was strongly associated with ischemic stroke (odds ratio 2.81), while migraine without aura showed no increased risk.[15] A recent metaanalysis of stroke risk in migraine, which included the ARIC study in its analysis, found that migraine overall doubled the stroke risk, with a slight further increase of stroke risk in migraine with aura.[16] It is advisable to identify and address risk factors for stroke and cardiovascular disease in migraine patients, but there is no evidence at this time for specific interventions in this population. Studies demonstrate certain particularly risky subgroups; smokers and people with high blood

pressure or high cholesterol seem to have additive stroke risk with migraines. In patients with migraine with aura, smoking and high-dose birth control pills appear particularly risky for stroke and should be avoided.[14]

5. Diagnostic evaluation Neuroimaging is not indicated for migraine evaluation unless it is atypical. If the neurologic examination is normal, the diagnosis of migraine can be made with no further investigations. The US Headache Consortium recommends: “neuroimaging is not usually warranted in migraine patients who have a normal neurologic examination”; however, it should be considered when neuroimaging risk factors for intracranial pathology exist, as when a patient with a non-acute headache that lasts more than 4 weeks has an unexplained neurologic examination, or other “red flags.”

6. Medications: triptans Triptans are one of the most commonly prescribed specific treatments for migraine headache and are considered a first-line treatment for migraine (Table 41.1). Triptans are all agonists at the serotonin 5HT-1b/1d receptor.[17] Triptans have many effects which treat the pathophysiology of migraine: they inhibit vasoactive peptide release and CGRP release, act as vasoconstrictors, and inhibit nociceptive activation in the trigeminal nucleus caudalis, thus decreasing the pain of migraine.[18] It is noteworthy that the triptan response is not diagnostic of migraine; since they work at the trigeminal nucleus caudalis, they may also reduce pain from other headache and facial pain conditions, and even pain from carotid dissection. These medications are effective as abortive treatments for migraine and are generally well tolerated.[19] Triptans tend to work best when used early in the treatment of migraine attack.[20] Triptans are available in a wide variety, including oral, sublingual, nasal spray, and subcutaneous forms of administration. In the USA, migraineurs strongly prefer oral forms, even though they may be less effective in patients who experience nausea and vomiting during migraine.[21] A transdermal formulation of sumatriptan can be used in patients with severe nausea.[22] Triptans have no preventive effect, and can in fact lead to transformed migraine or medication overuse

299

Chapter 41: Patient with migraine headaches

Table 41.1. Available triptans

Generic name

Trade name

Forms

Dose (mg)

MR*

Max dose (mg) in 24 hrs

Year FDA approved

Sumatriptan

Imitrex

Inj

4; 6

1

12

1992

Sumatriptan

Imitrex

Tab

50, 100

2

200

1995

Sumatriptan

Imitrex

NS

20

2

40

1997

Sumatriptan

Zecuity

Transdermal

6.5 over 4 hours

Naratriptan

Amerge

Tab

2.5

4

5

1998

Rizatriptan

Maxalt

Tab

5; 10

2

20

1998

Rizatriptan

Maxalt MLT

OD

5; 10

2

20

1998

Zolmitriptan

Zomig

Tab

2.5; 5

2

10

2003

Zolmitriptan

Zomig ZMT

OD

2.5; 5

2

10

2003

Almotriptan

Axert

Tab

12.5

2

25

2001

Frovatriptan

Frova

Tab

2.5

2

5

2001

Eletriptan

Relpax

Tab

40

2

80

2002

Zolmitriptan

Zomig NS

NS

5

2

10

2003

Sumatriptan/ Naproxen

Treximet

Tab

85 500

2

170 1000

2008

2013

* May repeat in 1 or more hours as listed.

headache when used to excess; more than 8 days a month on average in most patients. Some studies have suggested that triptans can produce medication overuse headache more quickly, and at lower doses, than other headache treatments.[23] The agonism of 5HT-1b receptors causes vasoconstriction, which can cause chest pain in susceptible individuals. Triptans can also cause paresthesias. Since the coronary arteries can undergo vasoconstriction in response to triptans, they should not be used in individuals suffering from coronary artery disease (CAD).[24] The package insert for these medications reports contraindication in patients with CAD risk factors until they have undergone a “thorough cardiovascular evaluation,” which is not defined by the FDA. It is important to document cardiovascular risk assessment prior to prescribing triptans in patients with increased cardiovascular risk such as hypertension or older age. Since triptans are serotonin agonists and modulate multiple receptors including 5HT-2A, they are a potential cause of serotonin syndrome, particularly when used together with other serotonergic agents such as SSRI or SNRI antidepressants.[25] Serotonin syndrome can be a lethal consequence of excess serotonin in the central nervous

300

system. The Hunter criteria are simple: “the patient must have ingested a serotonergic drug and demonstrate at least one of the following signs: spontaneous clonus; inducible clonus plus agitation or diaphoresis; ocular clonus plus agitation or diaphoresis; tremor plus hyperreflexia; hypertonia PLUS temperature above 38°C PLUS ocular clonus or inducible clonus.”[26] In clinical practice, the serotonin syndrome is more often talked about than encountered, the incidence of serotonin syndrome due to the coadministration of antidepressants and triptans appears to be less than 0.03%.[27] Thus, expert recommendations suggest using these medications together, but titrating dosages cautiously, watching for clinical signs of this syndrome, and educating patients about the risks involved.[28]

7. Medications: other non-opioid abortive treatments Ergot medications, including ergotamine and dihydroergotamine, have been used for migraine treatment for many years. Do not mix triptans with dihydroergotamine (DHE-45) within 24 hours (FDA). There is risk of vasoconstriction similar to

Chapter 41: Patient with migraine headaches

Table 41.2. Non-triptan abortive treatment of migraines: medications that provide quick pain relief when attack occurs

Generic name

Trade name

Forms

Dihydroergotamine

DHE 45

Inj

Dihydroergotamine

Migranal NS

NS

Isometheptene Dichloralphenazone Acetaminophen

Midrin

Butalbital Acetaminophen Caffeine Butalbital Aspirin Caffeine

Dose (mg)

MR*

Max dose in 24 hrs

Year FDA approved

1

8

3 mg

1946

4

8

12 mg

1997

Tab

65 100 325

1

5 tabs

Never FDA approved; not recommended

Fioricet

Tab

50 325 40

4

4 tabs

1985; not recommended

Fiorinal

Tab

50 325 40

4

4 tabs

1976; not recommended

* May repeat in 1 or more hours as listed.

triptans, and the use of DHE with triptans could cause extreme vasoconstriction. Do not recommend acute treatment to your patients more than 1–2 days a week maximum, because of the risk of inducing rebound headaches with more frequent use of acute medications. When your patients are experiencing more than 4 headache days a month consider adding preventive therapy. See Table 41.2 for non-triptan abortive treatment of migraines.

8. Medications: opioids and butalbital-containing combination pills These medications are not considered first-line therapy for migraine treatment since they are habit forming and extremely prone to causing rebound headaches. The use of chronic opioid therapy for most patients with chronic daily headache is not recommended. Long-term opioid therapy should be reserved only for the patients that fail all the other available treatment options, such as hospitalization. It is rare in clinical practice to encounter patients who use daily analgesics without dose escalation. In these patients the physician must explain the risks and benefits of prolonged use, and document this well in the patient’s chart. Avoid barbiturates (butalbital), because these medications have very high risk of rebound. Patients who have been taking butalbital-containing medications long-term may require weaning to avoid barbiturate withdrawal.

“Migraine prevention is an important component of therapy aimed at reducing the attack frequency and severity.”[29] Physicians have a wide number of options of prophylactic drugs available to treat migraine patients.[30] Particular medication to use or avoid in a given patient is based on their individual characteristics and coexisting conditions.[30] Another reason for selecting a specific class of preventive therapy may be based more on presence of coexisting conditions (such as high/low blood pressure, obesity, sleep disorders, use of other medications, among others) and its tolerability than on an understanding of the mechanism of action of these drugs and the subtle differences in the pathology of each patient.[30] As a consequence, it is impossible to predict which patient will respond to which prophylactic medication.[30] For example a beta-blocker would be contraindicated for a migraineur with asthma but might be the drug of choice for a hypertensive patient.[30] Prophylactic therapies should be used in migraineurs who have 4 or more a migraine days a month and those who have inadequate response to abortive treatment, or require emergency room visits for their migraine management. Even if patients have one migraine a month that requires emergency room visits, consider starting preventive therapy. Please emphasize to the patient that this treatment should be taken daily even if they don’t have a headache, because these treatments will reduce the headaches only if they are taken every day for a period of several weeks via modulating their

301

Chapter 41: Patient with migraine headaches

receptors. Preventive therapy takes a long time, at least 6 to 8 weeks to achieve better headache control. I have patients who state that “the preventive medication did not work,” even though they have a more than 80% reduction in their headache frequency, because they expect 100% resolution, which is rare, and in most patients an unrealistic expectation. The side effects of preventive medications usually decrease with time. Monotherapy with one medication should be used first, but a combination of different preventive therapies is often warranted in patients with difficulties in managing their severe headaches. Wait at least 6 months for patient to have less than four headaches a month before discontinuing the medication. Often the preventive medication is chosen based on the side effects that can be beneficial for treatment of other associated conditions.

Beta-blockers (e.g., propranolol, atenolol, nadolol, timolol) Beta-blockers are preferred medications for prevention of migraine in patients with hypertension and anxiety. They should be avoided in patients with asthma, because of potential worsening bronchospasm. They are used with care in diabetes because of decreased awareness of hypoglycemia. Side effects include stomach distress, decreased blood pressure and heart rate, erectile dysfunction, and decreased libido.

Anticonvulsants (e.g., valproate, topiramate, gabapentin) Topiramate is FDA approved for treatment of migraines, and is the drug of choice for most migraine sufferers. Consider topiramate anytime you have a patient who desires weight loss, or where weight loss would be beneficial for treatment of their comorbid condition, such as diabetes. Topiramate can also be used in addiction. Topiramate has side effects of weight loss, paresthesias, spelling problems, and cognitive dysfunction. Be very careful about starting with a small dose of topiramate to prevent cognitive difficulties; these are reversible and disappear with time. Most side effects disappear with ongoing use over a few weeks, and can be significantly reduced by slow taper of medications with a weekly increase of its dose. Avoid topiramate in patients with anorexia, or where any weight loss would be undesirable.

302

Calcium channel blockers (i.e., verapamil, amlodipine) Calcium channel blockers are commonly used as firstline medications in patients with migraine with aura. They can worsen heart failure and heart block. Side effects include constipation and low blood pressure.

Tricyclic antidepressants (i.e., amitriptyline, nortriptyline, desipramine) Tricyclic antidepressants (TCAs) are commonly used as first-line agents in patients with insomnia, depression, and anxiety. Side effects of TCAs include dry mouth and weight gain.

Selective SNRIs (i.e., duloxetine, milnacipram) Side effects of SNRIs include nausea, difficulty with sleep, erectile dysfunction, and decreased appetite. They also include sweating, anxiety, and increased blood pressure from their norepinephrine modulation.

Treatment of chronic migraine For prophylactic treatment of chronic migraines use the same guide as for preventive therapy of acute migraines (see Table 41.3). Please always address the potential medication overuse, which is present in more than 80% of patients in specialty headache centers. Make sure that acute therapies (triptans, ergotamine derivatives, NSAIDs) are not used more than 8 days a month in migraine patients to prevent “rebound headaches.”

9. Functional measures for outcomes: headache diary Headache diaries can be a helpful tool for understanding the frequency and severity of headache symptoms. A simple record of migraine days and medication use can be as useful as a more elaborate form for clinical purposes. There are many aps available for smart phones that are excellent for keeping a headache diary. The Migraine Disability Assessment (MIDAS) is a brief questionnaire for patients to fill out. The MIDAS score has been shown to be highly correlated with the ratings of expert clinicians.[31]

Table 41.3. Prophylactic treatment of migraine

Generic name

Trade name

Dose (mg)

BETA-BLOCKERS

Possible side effects

Contraindications

Fatigue + BP+ HR * weight

Asthma depression

Propranolol

Inderal

20–360





Metoprolol

Toprol

50–100





Nadolol

Corgard

10–160





Timolol

Blocadren

10–40





Atenolol

Tenormin

25–100





FDA

YES

YES

+ BP+ HR constipation arrhythmia

CALCIUM CHANNEL BLOCKERS

Verapamil

Calan

80–480

Diltiazem

Cardizem

60–360

Amlodipine

Norvasc

2.5–10

Divalproex

Depakote

250–1500

Drowsiness, + hair *** weight, * LFTs bone marrow Δ

Gabapentin

Neurontin

300–3600

Dizziness, ** weight

Pregabalin

Lyrica

25–600

Topiramate

Topamax

25–400

++ weight, cognitiveΔ memory problems

Zonisomide

Zonegran

25–400

++ weight

Lamotrigine

Lamictal

25–100

Stevens-Johnson sy.

Levatiracetam

Keppra

ANTIEPILEPTICS YES

Kidney stones

YES

** weight dry mouth QTC prolongation

TRICYCLIC ANTIDEPRESSANTS

Amitriptyline

Elavil

10–50

** weight

Nortriptyline

Pamelor

10–50



Imipramine

Tofranil

25–50

“ Nausea, fatigue Serotonin syndrome

SSRIs Citalopram

Celexa

10–40

Escitalopram

Lexapro

5–20

Duloxetine

Cymbalta

30–120

Milnacipram

Savella

2.5–150

Venlafaxine

Exxefor

75–225

Onabotulinum

BOTOX

150

Sex. dysfunction

SNRIs

YES

303

Chapter 41: Patient with migraine headaches

Figure 41.2. Supraorbital nerve stimulation. From personal files of Rinoo V. Shah, MD, MBA.

nerve and occipital nerve stimulation may be effective in treatment-resistant migraine.[33]

Figure 41.1. Occipital nerve stimulation. From personal files of Rinoo V. Shah, MD, MBA.

10. Interventional approaches: BOTOX, peripheral nerve blocks, peripheral nerve stimulation (including occipital nerve stimulation and supraorbital nerve stimulation) Onabotulinum toxin injection therapy (BOTOX® or onabotulinum toxin A) was approved in October 2010 by the US FDA to treat chronic migraine in patients with 15 or more days of migraine a month. 155 units of onabotulinum toxin injection are administered at intervals of every 3 months for at least a 1-year cycle. Side effects of onabotulinum toxin injections include neck pain and headache. Acupuncture has been shown to be superior to placebo in some trials; however large-scale trials comparing acupuncture to sham needling showed that both were equally effective. Occipital nerve stimulation did not appear superior to sham simulation in the largest trial (n¼52) to date, although some of the secondary endpoints in the trial showed improvement; nonetheless, this treatment may be considered in patients with chronic migraine who have failed conservative treatment (Figures 41.1 and 41.2).[32] Very limited case studies have suggested that combined supraorbital

304

11. Preventive measures: physical therapy, modalities, and nutrition There are many dietary supplements that are used in the treatment of migraines. Riboflavin (vitamin B2) 50–400 mg a day is recommended for migraine prevention.[34] Vitamin E (tocopherol) at a dose of 100–200 IU daily is effective in the reduction of menstrual migraine.[35] Coenzyme Q10 (ubiquinol or ubiquinone) has been shown to improve migraines at doses of 50–600 mg a day compared to placebo.[36] Melatonin[37] at doses of 3–12 mg at night is also recommended in migraine prevention.[38] “The treatment of headache disorders with melatonin and other chronobiotic agents is promising, and there is great potential for their use in headache treatment.”[39] Biofeedback and behavioral therapy can be very beneficial for migraine patients. Furthermore, daily exercise can be very helpful in most migraine patients.

12. Thoughts from an expert, e.g., overlooked areas I find that the most satisfying part of the treatment of patients presenting with severe headache is that often the headache diagnosis that I elicit from my patients by recognizing their headache patterns, leads me to the correct diagnosis. Even though I still proceed with a clinical examination, taking a good clinical history really helps me to find the appropriate treatment for my patients, and helps me avoid tests they don’t need.

Chapter 41: Patient with migraine headaches

I find that the correct diagnosis is really the key. Whenever I am not succeeding with my headache patients, I go back to the beginning and try to find what I missed. I have many patients that are referred to me for evaluation of migraines. Most commonly my patients initially present with chronic daily migraines defined as having more than 15 days of migraines a month. The most common cause of their journey to transformation into chronic daily headache is medication overuse that is not admitted by them, because they don’t realize that their medication use can truly harm them. Unfortunately, migraine patients are susceptible to transformation from episodic headaches into chronic daily headaches. Treatment failure in headache patients is multifactorial. Often the diagnosis is wrong or incomplete. Migraine may be misdiagnosed as tension-type headache, cluster, or sinus headache. Always start with establishing your own diagnosis of your patient that is not responding to typical treatments. Use IHS criteria. Make sure that there is no secondary headache present – even subarachnoid hemorrhage can respond to triptans – and proceed with neuroimaging if this is appropriate. Other common reasons for failure is inappropriate use of pharmacotherapy. Patients often take preventive medication for 1 to 2 days, and if the headache is still there, they stop. It takes 6 weeks or

References 1.

Stang PE, Osterhaus JT, Celentano DD. Migraine: patterns of healthcare use. Neurology. 1994; 44(6):S47–S55.

2.

Lipton RB, Stewart WF, Diamond S, Diamond ML, Reed M. Prevalence and burden of migraine in the United States: data from the American Migraine Study II. Headache. 2001;41(7): 646–657.

3.

4.

Lipton RB, Diamond S, Reed M, Diamond ML, Stewart WF. Migraine diagnosis and treatment: results from the American Migraine Study II. Headache. 2001;41(7):638–645. Pietrobon D. Migraine: new molecular mechanisms. The Neuroscientist. 2005;11(4): 373–386.

5.

6.

more of using daily preventive medications to see any headache reduction, and preventive medications decrease the headaches by about 50% in their frequency and severity. If the patients continue overusing medications, such as using daily acetaminophen, their daily headaches can continue for years, and not respond to any preventive medications. I have seen hundreds of patients who are refractory “to all treatments,” only to find that they are overusing triptans (taking them daily), or other acute medications. Once we establish that this is the issue, the key is to convince the patient that certain medications taken in inappropriate doses will cause malignant transformation of their headaches, and once this occurs, they need to discontinue medications that they are overusing, before any medications become effective again. It takes several weeks to months to normalize the brain function. Procedures such as BOTOX therapy and occipital nerve blocks can be very helpful during these times as “bridge therapies” that will enable the patients to wean off overused medications, and start a new chapter in their life with appropriate use of acute and preventive medications. Part of the treatment also includes addressing their lifestyles and developing coping strategies to avoid anxiety and catastrophization. The use of opioid therapy for most headache patients is probably not warranted.

Stewart WF, Lipton RB, Celentano DD, Reed ML. Prevalence of migraine headache in the United States. JAMA. 1992;267(1):64–69. Hu XH, Markson LE, Lipton RB, Stewart WF, Berger ML. Burden of migraine in the United States: disability and economic costs. Arch Intern Med. 1999;159(8): 813.

7.

Weiller C, May A, Limmroth V, et al. Brain stem activation in spontaneous human migraine attacks. Nat Med. 1995;1(7): 658–660.

8.

Goadsby PJ, Lipton RB, Ferrari MD. Migraine: current understanding and treatment. N Engl J Med. 2002;346(4).

9.

Moskowitz MA. Pathophysiology of headache: past and present.

Headache. 2007;47 (Suppl 1): S58–63. 10. International Headache Society Classification Subcommittee. The International Classification of Headache Disorders: 2nd edition. Cephalalgia. 2004;24(Suppl 1): 9–160. 11. Kruit MC, van Buchem MA, Launer LJ, Terwindt GM, Ferrari MD. Migraine is associated with an increased risk of deep white matter lesions, subclinical posterior circulation infarcts and brain iron accumulation: the population-based MRI CAMERA study. Cephalalgia. 2010;30(2): 129–136. 12. Buring JE, Hebert P, Romero J, et al. Migraine and subsequent risk of stroke in the Physicians’ Health Study. Arch Neurol. 1995;52(2):129.

305

Chapter 41: Patient with migraine headaches

13. Kurth T, Schurks M, Logroscino G, Gaziano JM, Buring JE. Migraine, vascular risk, and cardiovascular events in women: prospective cohort study. BMJ. 2008;337:a636. 14. Schurks M, Rist PM, Bigal ME, et al. Migraine and cardiovascular disease: systematic review and meta-analysis. BMJ. 2009;339: b3914. 15. Stang P, Carson A, Rose K, et al. Headache, cerebrovascular symptoms, and stroke: The Atherosclerosis Risk in Communities Study. Neurology. 2005;64(9):1573–1577. 16. Spector JT, Kahn SR, Jones MR, et al. Migraine headache and ischemic stroke risk: an updated meta-analysis. Am J Med. 2010;123(7):612–624. 17. Tfelt-Hansen P, De Vries P, Saxena PR. Triptans in migraine. Drugs. 2000;60(6):1259–1287. 18. Bartsch T, Knight YE, Goadsby PJ. Activation of 5-HT1B/1D receptor in the periaqueductal gray inhibits nociception. Ann Neurol. 2004;56(3):371–381. 19. Ferrari M, Goadsby P, Roon K, Lipton R. Triptans (serotonin, 5-HT1B/1D agonists) in migraine: detailed results and methods of a meta-analysis of 53 trials. Cephalalgia. 2002;22(8):633–658. 20. Burstein R, Collins B, Jakubowski M. Defeating migraine pain with triptans: a race against the development of cutaneous allodynia. Ann Neurol. 2004;55(1):19–26.

23. Limmroth V, Katsarava Z, Fritsche G, Przywara S, Diener H-C. Features of medication overuse headache following overuse of different acute headache drugs. Neurology. 2002;59(7):1011–1014. 24. Young WB, Mannix L, Adelman JU, Shechter AL. Cardiac risk factors and the use of triptans: a survey study. Headache. 2000; 40(7):587–591. 25. Boyer EW, Shannon M. The serotonin syndrome. New Engl J Med. 2005;352(11):1112–1120. 26. Dunkley E, Isbister G, Sibbritt D, Dawson A, Whyte I. The Hunter Serotonin Toxicity Criteria: simple and accurate diagnostic decision rules for serotonin toxicity. QJM. 2003;96(9):635–642. 27. Shapiro RE, Tepper SJ. The serotonin syndrome, triptans, and the potential for drug–drug interactions. Headache. 2007; 47(2):266–269. 28. Evans RW, Tepper SJ, Shapiro RE, Sun-Edelstein C, Tietjen GE. The FDA alert on serotonin syndrome with use of triptans combined with selective serotonin reuptake inhibitors or selective serotoninnorepinephrine reuptake inhibitors: American Headache Society Position Paper. Headache. 2010;50(6):1089–1099. 29. Sprenger T, Goadsby P. Migraine pathogenesis and state of pharmacological treatment options. BMC Med. 2009;7(1):71.

21. Rapoport AM, Bigal ME, Tepper SJ, Sheftell FD. Intranasal medications for the treatment of migraine and cluster headache. CNS Drugs. 2004;18(10):671–685.

30. Waeber C, Moskowitz MA. Therapeutic implications of central and peripheral neurologic mechanisms in migraine. Neurology. 2003;61(8 suppl 4): S9–S20.

22. Schulman EA. Transdermal sumatriptan for acute treatment of migraineurs with baseline nausea. Headache. 2012;52(2): 204–212.

31. Lipton R, Stewart W, Sawyer J, Edmeads J. Clinical utility of an instrument assessing migraine disability: the Migraine Disability Assessment (MIDAS)

306

questionnaire. Headache. 2001; 41(9):854–861. 32. Silberstein SD, Dodick DW, Saper J, et al. Safety and efficacy of peripheral nerve stimulation of the occipital nerves for the management of chronic migraine: results from a randomized, multicenter, double-blinded, controlled study. Cephalalgia. 2012;32(16):1165–1179. 33. Reed K, Will K, Chapman J, Richter E. Combined occipital and supraorbital neurostimulation for chronic migraine headaches: an extended case series. Paper presented at 15th Congress of the International Headache Society 2011. 34. Condò M, Posar A, Arbizzani A, Parmeggiani A. Riboflavin prophylaxis in pediatric and adolescent migraine. J Headache Pain. 2009;10(5):361–365. 35. Ziaei S, Kazemnejad A, Sedighi A. The effect of vitamin E on the treatment of menstrual migraine. Medical Science Monitor. 2009; 15(1):CR16. 36. Sandor P, Di Clemente L, Coppola G, et al. Efficacy of coenzyme Q10 in migraine prophylaxis: a randomized controlled trial. Neurology. 2005; 64(4):713–715. 37. Peres M, Zukerman E, da Cunha Tanuri F, Moreira F, Cipolla-Neto J. Melatonin, 3 mg, is effective for migraine prevention. Neurology. 2004;63(4):757. 38. Miano S, Parisi P, Pelliccia A, et al. Melatonin to prevent migraine or tension-type headache in children. Neurol Sci. 2008;29(4):285–287. 39. Peres MF, Masruha MR, Zukerman E, Moreira-Filho CA, Cavalheiro EA. Potential therapeutic use of melatonin in migraine and other headache disorders. Expert Opin Investig Drugs. 2006;15(4):367–375.

Section 5 Chapter

42

Headaches and Facial Pain

Patient with cluster headache Natalia Murinova, Daniel Krashin, and Andrea Trescot

Case study A 38-year-old male presents with a 5-year history of episodic debilitating headaches. He reports a strictly unilateral right-sided unbearable headache that occurs only 2 months out of the year, waking him up at night from sleep. He denies nausea and vomiting but reports that, when severe, the headache is associated with right-sided eye tearing and nasal congestion. He has failed to respond to standard preventive treatment for migraine. His neurologic exam is normal, and he has never had MRI of his brain. He states that he can’t tolerate his headaches anymore, and feels suicidal due to severe pain. He was referred to you for treatment and evaluation of his severe “migraines.”

1. What is the epidemiology and clinical presentation of cluster headache? Cluster headache is one of the most agonizing pain syndromes known. This headache is described by the International Headache Society as consisting of excruciating unilateral head pain which lasts up to 180 minutes and is associated with ipsilateral cranial autonomic features. The diagnosis of cluster headache has been recognized for over 50 years, but the definitive diagnosis is often delayed for up to 5 years.[1] Cluster headaches are uncommon compared with migraines. More than 500000 people are estimated to suffer from cluster headache in the USA. It is diagnosed most commonly in young people. Cluster headache begins at age 30 or younger in 71% of patients, and only 3% of patients develop these headaches after age 51.[1,2] The gender ratio in cluster has changed over the past decades, with a ratio of 6:1 in

favor of males prior to the 1960s in the USA which has gradually equalized to a 2:1 ratio in the present population. This may be related to changes in lifestyle and gender roles, including the increasing prevalence of smoking in women.[3]

2. How is cluster headache diagnosed? The diagnosis depends on a detailed headache history. Any patient who presents with unilateral debilitating pain in the area of the first trigeminal branch, and who has associated same-sided autonomic symptoms, should be regarded as a potential cluster headache (CH) patient. More than 90% of patients have ipsilateral tearing and nasal congestion on the same side as their headache.[1] Patients with mild bilateral parasympathetic activation, such as bilateral tearing, are much more likely to have migraines. Therefore, the key is to recognize the association of unilateral parasympathetic activation with severe trigeminal nerve pain. If the patient has any abnormality on his neurologic examination, inconsistency in his history, or only partial response to treatment, or if other red flags such as infection, cancer or immunodeficiency are present, proceed with neuroimaging, preferably brain MRI with and without contrast.

3. What are the diagnostic criteria for cluster headache? Cluster headache is, in 80–90% of cases, episodic but is severely seriously disabling during those episodes. CH pain is described as the worst pain experienced. CH is defined by the International Headache Society as severe unilateral pain in or above the eye or in the temple, which lasts 15 minutes to 3 hours if

Case Studies in Pain Management, ed. Alan David Kaye and Rinoo V. Shah. Published by Cambridge University Press. © Cambridge University Press 2015.

307

Chapter 42: Patient with cluster headache

untreated.[4] The pain is accompanied by at least one of the following autonomic symptoms: “conjunctival injection or tearing, nasal congestion or rhinorrhea, eyelid edema, forehead and facial sweating, miosis and/or ptosis,” all ipsilateral to the pain, or a subjective sense of restlessness, agitation, and the inability to sit still.[4] The attacks can range from one every 2 days to one eight times a day, and tend to occur in groups (hence the term “cluster”), especially in the months of January and July.

4. What are the diagnostic pitfalls? Patients may present with the existing diagnosis of CHs but not fit the picture. For example, a patient referred for management of treatment-resistant CH might have headaches all year long; episodic cluster headaches usually cluster seasonally for about 4 weeks to 6 weeks out of a year, more often at night. Chronic CHs can occur, but are very rare; tumors and medication overuse need to be ruled out in these cases. Likewise, a patient identified as having “CH” might have headaches for 10 hours/day, much longer than the 35 minutes to 180 minutes usually seen in real CH. Patients who prefer to lie down during headache are also unlikely to have CH, as these patients typically are restless and pace, unable to get comfortable or sit still. Conversely, a patient with localized trigeminal pain around the eye lasting 30 minutes, and with unilateral parasympathetic symptoms, most likely has CH. A restless headache patient with relatively short excruciating headache with self-injurious behavior during their headache is more likely to have CH.

5. What is the differential diagnosis? a. Any primary trigeminal autonomic cephalalgia (TAC). The TACs include short-lasting unilateral neuralgiform headache attacks with conjunctival injection and tearing (SUNCT), paroxysmal hemicrania, and CH (listed from the shortest to longest duration). The TACs differ in their attack frequency and duration, as well as in their response to treatment.[4] b. Migraine headache without aura c. Hypnic headache d. Secondary headaches due to structural abnormalities such as brain tumor, aneurysm, or AVM[5] e. Chronic CH

308

6. Does the specific diagnosis matter? Yes, because the prognosis and treatment differ significantly between different headache types, and even between different types of TACs. This headache is excruciating and associated with suicidal ideation in a high frequency of patients. When correctly diagnosed most patients can get relief of their debilitating headaches. It is also important to distinguish the different types of TACs as all of them have different treatments.

7. What risk factors predispose patients to develop cluster headaches? Genetic studies show an increased risk of CH in relatives of CH sufferers, which has been hypothesized to reflect autosomal dominant inheritance with reduced penetrance, but no specific gene locus has been identified.[6] Seventy-three percent of CH patients have smoked or used chewing tobacco, almost all of them prior to the onset of CH.[1] The biggest risk for chronic CH is medication overuse.

8. Why is this condition overlooked? Cluster headache is taught as a medical curiosity, and is rarely encountered by primary care physicians, leading to the incorrect diagnosis of migraine. CH patients may also have migraine-like features such as nausea and light sensitivity. Physicians will sometimes discount the possibility of cluster in women, even though the gender ratio is only 2:1 in young patients.[1]

9. What is the anatomy and pathophysiology of a cluster headache? CH, like migraine headaches, involve the trigeminovascular system, which is involved both in sensory processing from the cranial vasculature as well as regulation of this vasculature.[7] Activation of the posterior hypothalamus appears to drive CH, particularly the suprachiasmatic nucleus. The irritability and restlessness observed during a CH is attributed to hypothalamic activation. CH patients also exhibit abnormal circadian rhythm of the melatonin release from the pineal gland, which is regulated by the suprachiasmatic nucleus in the hypothalamus. During a CH attack, the hypothalamus activates the trigeminovascular system. Ipsilateral trigeminal nerve activation results in excruciating face and eye pain. The parasympathetic nerve system, activated via the pterygopalatine nucleus,

Chapter 42: Patient with cluster headache

results in ipsilateral autonomic symptoms of the eye, face, and nasal passages.

10. Is there any other diagnostic testing that should be done? Neuroimaging should take the form of brain MRI with and without contrast to rule out secondary headache. Brain tumors and other structural abnormalities can present as CHs, and can partially respond to CH treatment.[5] In the patient with a normal neurologic exam and normal brain MRI, laboratory testing and EEG studies are not indicated. Once potentially lifethreatening secondary causes of headache have been ruled out, the IHS criteria should be followed to make a precise headache diagnosis.

11. How is cluster headache treated? Pharmacotherapy Treatment of CH has two pharmacotherapy strategies: symptomatic (acute) therapy administered by the patient at the time of his headache attack, and preventive therapy taken daily when the cluster bout begins to prevent further attacks.[8]

Acute treatment 100% oxygen. Oxygen given at 100% by face mask at 12 liters/minute was shown to be highly effective, ending 78% of CH attacks.[9] This treatment is very safe and CH patients may benefit from having a small oxygen tank at home to allow them to self-administer at the onset of CH. Triptan drugs have been shown to be effective abortive treatments for CH. Subcutaneous sumatriptan 6–12 mg is one of the most effective treatments.[10] Zolmitriptan orally 5–10 mg has also been shown to be effective.[11] Dihydroergotamine (DHE-45): Nasal DHE 1 mg given early in a CH attack can be a useful abortive agent.[12] In the ED setting, IV DHE given with an antiemetic can be helpful. Patients can be taught to self-inject DHE 1 mg IM at the onset of CH. Care must be taken to avoid giving these medications to patients with cardiovascular or cerebrovascular disease, where triptans and DHE are contraindicated.

Preventive treatments Prevention of CH relies on neuron membrane stabilizers. The most commonly used treatment is verapamil,

titrated up to doses of 240–360 mg a day.[13] High-dose verapamil can cause bradycardia and heart block and an EKG should be obtained after the verapamil was initiated, and especially with increasing doses. Gabapentin up to 900 mg or topiramate at 75 mg/day may also be helpful.[14] When treating frequent CH, be careful to evaluate for overuse of abortive medications. Daily use of triptans or opioids can convert CHs into chronic cluster, which is typically resistant to treatment until the medication overuse has been addressed.[15]

Procedural Occipital nerve block Injection of local anesthetic and corticosteroids has been shown to benefit CH in randomized controlled trials.[16] The side effects are usually limited to soreness at the injection site.

Neurostimulation Very limited studies of the occipital nerve stimulation and deep brain stimulation for intractable CH have been done with promising results but no strong recommendation can be made with the current state of evidence.[17]

12. How should I treat this patient? The most important thing is the correct diagnosis. This patient meets all the IHS criteria of CH. Secondary headache needs to be ruled out by neuroimaging with brain MRI. Most patients with CH require both acute and preventive treatment. The acute treatment should be administered during the onset of CH attack. Always start with oxygen therapy first, always at 100% via face mask, as nasal cannula is inadequate. The higher the blast of oxygen, the more likely the treatment will be effective. The patient should keep the oxygen on for at least 15 minutes, even if the attack resolves sooner, to prevent a recurrence. If oxygen is not effective, proceed with triptans or DHE by injection or nasal sprays. Oral abortive therapy should be avoided, since it takes too long to be effective. If there is contraindication to vasoconstrictive therapy, proceed with occipital nerve block; in refractory cases consider an occipital nerve stimulator. Always start patients on preventive therapy with verapamil and steroids if they have no contraindications, and continue therapy for at least 3 months.

309

Chapter 42: Patient with cluster headache

If patients are diagnosed with chronic CH, always rule out medication overuse first, because unaddressed medication overuse will make the prophylactic therapy ineffective. In chronic cluster patients it is reasonable to use procedures such as occipital nerve block if they fail to respond to standard prophylactic therapies and cannot use acute therapies due to rebound. A sphenopalatine ganglion block and radiofrequency lesioning has been beneficial in a case report.[18]

13. Are there any complications to worry about using DHE (dihydroergotamine) or triptans for acute treatment of cluster headache? There are potential risks of vasoconstrictive therapies in patients with a previous history of CAD or cerebrovascular disease; in these patients these medications are contraindicated. In patients with known coronary artery disease and no therapeutic benefit of oxygen, proceed with occipital nerve block. Triptans are potentially contraindicated with other SSRIs, or selective serotonin and norepinephrine reuptake inhibitors (SNRIs) because of risk of serotonin syndrome. The serotonin syndrome (SS) results from the use of individual or a combination of medications

References 1.

2.

3.

4.

310

Rozen TD, Fishman RS. Cluster headache in the United States of America: demographics, clinical characteristics, triggers, suicidality, and personal burden. Headache. 2012;52(1):99–113. Bjørn Russell M. Epidemiology and genetics of cluster headache. The Lancet Neurology. 2004;3(5): 279–283. Manzoni G. Gender ratio of cluster headache over the years: a possible role of changes in lifestyle. Cephalalgia. 1998;18(3): 138–142. International Headache Society Classification Subcommittee. The International Classification of Headache Disorders: 2nd edition.

that cause an increase in the intrasynaptic levels of serotonin. CH patients, especially chronic CH, can develop medication overuse headache. This is treated with discontinuing the overused medication; it is a debilitating condition that can cause excruciating daily headaches and needs to be addressed. CH patients should be carefully screened for medication overuse, especially those with a prior migraine history.[15]

14. What are the outcomes of treatmentof cluster headache patients? Longitudinal studies suggest that CH is a long-lasting condition with a small rate of spontaneous resolution.[19] Some fraction of patients with episodic CH will develop chronic CH, estimates for this range from 3.8 to 12.8%. This evolution appears to occur more frequently in patients whose CH episodes started later in life and in patients with frequent bouts of CH.[20] A population-based study of Danish headache patients showed that CH had a fairly good prognosis, yet suggested that this may represent patients learning to manage their pain symptoms themselves without requiring medical treatment rather than abolishing headache symptoms.[21] There is no data to suggest that abortive or preventive treatment of CH improves long-term outcomes. preventive pharmacologic treatment of cluster headache. Neurology. 2010;75(5):463–473.

Cephalalgia. 2004;24(Suppl 1): 9–160. 5.

6.

Favier I, van Vliet JA, Roon KI, et al. Trigeminal autonomic cephalgias due to structural lesions: a review of 31 cases. Archiv Neurol. 2007; 64(1):25. Southgate L, Scollen S, He W, et al. Elucidating the molecular genetic basis of cluster headache: delineation of the genetic architecture by exome sequencing. J Headache Pain. 2013;(Suppl 1): P34.

7.

Goadsby PJ. Pathophysiology of cluster headache: a trigeminal autonomic cephalgia. The Lancet Neurology. 2002;1(4):251–257.

8.

Francis GJ, Becker WJ, Pringsheim TM. Acute and

9.

Cohen AS, Burns B, Goadsby PJ. High-flow oxygen for treatment of cluster headache. JAMA. 2009; 302(22):2451–2457.

10. Ekbom K, Monstad I, Prusinski A, et al. Subcutaneous sumatriptan in the acute treatment of cluster headache: a dose comparison study. Acta Neurolog Scand. 1993;88(1):63–69. 11. Bahra A, Gawel M, Hardebo J-E, et al. Oral zolmitriptan is effective in the acute treatment of cluster headache. Neurology. 2000;54(9): 1832–1839. 12. Ziegler D, Ford R, Kriegler J, et al. Dihydroergotamine nasal spray for the acute treatment of

Chapter 42: Patient with cluster headache

migraine. Neurology. 1994;44(3 Part 1):447. 13. Gabai IJ, Spierings EL. Prophylactic treatment of cluster headache with verapamil. Headache. 1989;29(3):167–168. 14. Schuh-Hofer S, Israel H, Neeb L, Reuter U, Arnold G. The use of gabapentin in chronic cluster headache patients refractory to first-line therapy. Eur J Neurol. 2007;14(6):694–696. 15. Paemeleire K, Bahra A, Evers S, Matharu M, Goadsby PJ. Medication-overuse headache in patients with cluster headache. Neurology. 2006; 67(1):109–113.

16. Ambrosini A, Vandenheede M, Rossi P, et al. Suboccipital injection with a mixture of rapidand long-acting steroids in cluster headache: a double-blind placebocontrolled study. Pain. 2005; 118(1):92–96. 17. Pedersen JL, Barloese M, Jensen RH. Neurostimulation in cluster headache: a review of current progress. Cephalalgia. 2013; 33(14):1179–1193. 18. Shah RV, Racz GB. Long-term relief of posttraumatic headache by sphenopalatine ganglion pulsed radiofrequency lesioning: a case report. Arch Phys Med Rehabil. 2004;85(6):1013–1016.

19. Manzoni GC, Micieli G, Granella F, et al. Cluster headache: course over ten years in 189 patients. Cephalalgia. 1991;11(4): 169–174. 20. Torelli P, Cologno D, Cademartiri C, Manzoni GC. Possible predictive factors in the evolution of episodic to chronic cluster headache. Headache. 2000; 40(10):798–808. 21. Jensen R, Zeeberg P, Dehlendorff C, Olesen J. Predictors of outcome of the treatment programme in a multidisciplinary headache centre. Cephalalgia. 2010;30(10): 1214–1224.

311

Section 5 Chapter

43

Headaches and Facial Pain

Patients with tension headaches Natacha Telusca, Chrystina Jeter, and Kingsuk Ganguly

Case study A 35-year-old female with past medical history significant for anxiety and depression presents with complaints of headaches that have been worsening for over 3 years. She reports that the pain starts in the back of her neck and moves to the forehead. The pain is described as a tightening band around her forehead. The characteristic of the pain has been the same over the years, but the frequency has increased. She does not have any associated symptoms such as nausea, vomiting, blurry vision, photophobia, or photophonia. She is single and works as an attorney at a prestigious law firm. She uses aspirin and Tylenol, which provide some relief.

1. What is the differential diagnosis?       

Tension-type headache Occipital neuralgia Headache, migraine Headache, cluster Sinusitis Meningitis Trigeminal neuralgia

2. What is the epidemiology of these headaches? Headache is a major public health problem affecting a large segment of the world population. The patient in the above case study is likely suffering from a tensiontype headache (TTH), which is the most prevalent type of headache.[1] A Danish population-based study reported that the overall prevalence of TTH is 79–87%, and 2–5% for chronic TTH (CTTH).[2] TTH is slightly less common in men then women with a male to female ratio of 4:5.[3] Some of the reported

risk factors for developing TTH include poor self-rated health, inability to relax after work, and poor sleeping patterns.[3,4] The reported average age of onset of TTH is 25–30.[3] The global societal burden of TTH is substantial and greater than that of migraine.[1] Despite the high socioeconomic burden, TTH is the least studied of the primary headaches.[5] The second edition of the International Classification of Headache Disorders (ICHD) classified TTH into three main subtypes: infrequent episodic TTH, frequent episodic TTH, and chronic episodic TTH.[5] The criteria for infrequent TTH headache are episodes less than 1 day a month. Frequent episodic TTH is defined as at least 10 episodes occurring on 1 to 14 days per month. Chronic episodic TTH is described as having more headaches occurring on more than 15 days per month. Frequent TTH does not have significant impact on the individual, and chronic TTH has major impact on quality of life.[5]

3. Describe the clinical presentation In comparison to all of the primary headaches, TTH has the least features, which makes the diagnosis one of exclusion.[3] Patients with TTH typically present with bilateral, pressing, or tightening pain.[6] The pain is usually mild to moderate in intensity and is not aggravated by routine physical activity; patients usually do not have nausea, severe photophobia, or phonophobia.[6] A detailed history, general physical, and neurologic exam are important to make the correct diagnosis. In addition to the general history and physical exam, a 4- to 6-week headache diary is a very useful tool that can help the clinician to ensure the correct diagnosis of TTH.[7] The diary can also help differentiate between mild migraines and TTH.[8] Furthermore, the diary helps to identify triggers and

Case Studies in Pain Management, ed. Alan David Kaye and Rinoo V. Shah. Published by Cambridge University Press. © Cambridge University Press 2015.

312

Chapter 43: Patients with tension headaches

high intake of analgesics.[8] The general and neurologic physical exam is typically normal. Brain imaging is required if the clinical picture is unclear and secondary causes are suspected.[3]

4. What is the pathophysiology of tension-type headache? TTH has a very complex mechanism, which is not fully understood. Many factors may contribute to the development of TTH including genetic predisposition, environmental, psychological, peripheral, and central factors.[9] There is a clear relationship between TTH and psychologic distress and poor coping skills.[10] Patients with frequent TTH tend to have higher rates of anxiety and depression.[10] A study by Janke et al showed that depression increases susceptibility to TTH following laboratory stress, and was correlated with increased paracranial muscle tenderness.[11] In patients with frequent headaches, depression could exacerbate central sensitization increasing vulnerability to TTH.[11] Peripheral factors have been studied and shown to have an association with TTH. Patients with TTH tend to have increased pericranial muscle tenderness and myofascial trigger points,[12] and there is a positive correlation with the increased tenderness and the intensity and frequency of TTH.[13] Fernández-delas-Peñas et al reported that patients with chronic TTH have increased pericranial muscle tenderness and decreased pressure pain thresholds in comparison to healthy controls.[14] Though the exact mechanism of increased tenderness is not fully understood, there are several proposed mechanisms including sensitization of peripheral myofascial nociceptors, sensitization of second-order neuron at the level of the spinal dorsal horn/trigeminal nucleus, sensitization of supraspinal neurons, and decreased antinociceptive activity from supraspinal structures.[15] Central sensitization due to recurrent stimuli from pericranial muscle tissues may play a role in the conversion of episodic to chronic TTH.[12] Buchgreitz et al showed that there is a close relation between altered pain perception and chronification of headache.[13] Patients with chronic TTH have an increased number of active trigger points in the suboccipital muscles, and in the upper trapezius, sternocleidomastoid and temporalis muscles,[16] higher headache intensity, and longer headache duration than those with latent trigger points.[17]

5. Treatment The management of TTH involves nonpharmacological treatments, pharmacologic treatments, and invasive procedures. Patients with chronic TTH often have associated psychosocial factors such as stress, anxiety, and depression that can aggravate their pain,[9] therefore it is vital to identify and treat significant comorbidities concurrently.[8] Nonpharmacologic managements are commonly used and encouraged as part of the treatment for TTH; however the evidence for the efficacy of many of the treatments is lacking.[8] Some of the common modalities include electromyography (EMG) biofeedback, CBT, relaxation training, and acupuncture. A recent review by Sun-Edelstein and Mauskop concluded that EMG biofeedback (BFB) is effective in the treatment of TTH, and CBT and relaxation training may have some benefits.[18] Stress management therapy, including relaxation and cognitive coping skills, is a modestly effective treatment for TTH.[19] Acupuncture is used in the prophylactic treatment for TTH; however there is lack of evidence for its efficacy. A Cochrane review by Linde et al including 11 trials with 2317 participants concluded that there is insufficient evidence to support the use of acupuncture in the treatment of TTH.[20] Interventional treatments that are used in the treatment of TTH include cervical epidural steroid injection, occipital nerve blocks, and upper cervical facet injections. There are limited studies to support the effectiveness of these procedures. However, these procedures are relatively safe and provide an integral part of multimodal care by attempting to target the pain generator. Occipital and supraorbital nerve stimulation play a role in occipital neuralgia and migraines. Similarly, these neuromodulatory procedures may be beneficial in chronic TTH that is refractory to more conservative treatments. A study by Leinisch-Dahlke found that occipital nerve blockage is not an effective treatment for TTH.[21] The two most common analgesics used in the acute treatment of TTH are acetaminophen and aspirin.[22] Randomized controlled studies have shown that 500 and 1000 mg aspirin,[22] 1000 mg of acetaminophen,[22,23] and 375 mg of Naproxen[23] are effective treatments for acute TTH. Preventive treatment should be considered in patients with CTTH. Tricyclic antidepressants have been shown to be an effective prophylactic treatment. Amitriptyline has been shown

313

Chapter 43: Patients with tension headaches

Table 43.1 Acute therapy of tension-type headache; Bendtsen et al recommended the following drugs for acute therapy of tension-type headache[3]

Treatment

Dose

Remark

Ibuprofen

200–800 mg

Gastrointestinal side effects, risk of bleeding

Ketoprofen

25 mg

Same side effects as for ibuprofen

Aspirin

500–1000 mg

Same side effects as for ibuprofen

Naproxen

375–550 mg

Same side effects as for ibuprofen

Diclofenac

12.5–100 mg

Same side effects as for ibuprofen, only doses 12.5–25 mg tested in TTH

Paracetamol

1000 mg (oral)

Less risk of gastrointestinal side effects as compared with NSAIDs

Caffeine comb.

65–200 mg

See below*

* “Combination with caffeine 65–200 mg increases the efficacy of ibuprofen and paracetamol but possibly also increases the risk for developing medication overuse headaches.”[3]

Table 43.2 Drugs for prophylactic therapy of tension-type headache; Bendtsen et al. recommended the following drugs for prophylactic therapy of tension-type headache[3]

Treatment

Daily dose

Drug of first choice Amitriptyline

30–75 mg

Drugs of second choice Mirtazapine Venlafaxine

30 mg 150 mg

Drugs of third choice Clomipramine Maprotiline Mianserin

75–150 mg 75 mg 30–60 mg

References 1.

2.

3.

314

Stovner L, Hagen K, Jensen R, et al. The global burden of headache: documentation of headache prevalence and disability worldwide. Cephalalgia. 2007; 27(3):193–210. Lyngberg AC, Rasmussen BK, Jorgensen T, et al. Has the prevalence of migraine and tension-type headache changed over a 12-year period? A Danish population survey. Eur J Epidemiol. 2005;20(3): 243–249. Bendtsen L, Evers S, Linde M, et al. EFNS guideline on the treatment of tension-type headache: report of

to decrease the frequency and duration of headache, and intake of analgesics.[24] A randomized placebocontrolled trial by Holroyd et al comparing amitriptyline and nortriptyline showed that combined therapy of antidepressant medication and stress management provide good outcome relative to single therapy.[19]

6. Conclusion TTH is a common problem affecting a large segment of the population. The pathophysiology is not fully understood. A detailed history, general physical and neurologic exam, and headache diary are essential to ensure the correct diagnosis. Tables 43.1 and 43.2 can be used to guide treatment.

headache disorder. Curr Opin Neurol. 2006;19(3):305–309.

an EFNS task force. Eur J Neurol. 2010;17:1318–1325. 4.

Lyngberg AC, Rasmussen BK, Jorgensen T, Jensen R. Prognosis of migraine and tension-type headache: a population-based follow-up study. Neurology. 2005;65:580–585.

5.

Headache Classification Subcommittee of the International Headache Society. The international classification of headache disorders. 2nd edition. Cephalalgia. 2004;24(Suppl 1): 1–160.

6.

Bendtsen L, Jensen R. Tensiontype headache: the most common, but also the most neglected,

7.

8.

Russell MB, Rasmussen BK, Brennum J, et al. Presentation of a new instrument: the diagnostic headache diary. Cephalalgia. 1992;12(6):369–374.

Bendtsen L, Jensen R. Pharmacological and nonpharmacological management of tension-type headache. Eur Neurolog Rev. 2008;3(1):119–122. 9. Bendtsen L, Fumal A, Schoenen J. Tension-type headache: mechanisms. Handbook Clin Neurol. 2010;97:359–366. 10. Heckman BD, Holroyd KA. Tension-type headache and

Chapter 43: Patients with tension headaches

psychiatric comorbidity. Curr Pain Headache Rep. 2006; 10(6):439–447. 11. Janke EA, Holroyd KA, Romanek K. Depression increases onset of tension-type headache following laboratory stress. Pain. 2004; 111(3):230–238. 12. Bendtsen L, Fernández-de-lasPeñas C. The role of muscles in tension-type headache. Curr Pain Headache Rep. 2011;15(6):451–458. 13. Buchgreitz L, Lyngberg AC, Bendtsen L, et al. Frequency of headache is related to sensitization: a population study. Pain. 2006;123(1–2):19–27. 14. Fernández-de-las-Peñas C, Cuadrado ML, Arendt-Nielsen L, et al. Increased peri-cranial tenderness, decreased pressure pain threshold, and headache clinical parameters in chronic tension-type headache patients. Clin J Pain. 2007;23(4):346–352. 15. Bendsten L. Central sensitization in tension-type-headache: possible pathophysiological mechanisms. Cephalalgia. 2000;20:486–508.

16. Fernandez-de-las-Penas C, Cuadrado ML, Arendt-Nielsen L, et al. Myofascial trigger points and sensitization: an updated pain model for tension-type headache. Cephalalgia. 2007;27(5):383–393. 17. Fernandez-de-las-Penas C, Alonso-Blanco C, Cuadrado ML, Gerwin RD, Pareja JA. Myofascial trigger points and their relationship to headache clinical parameters in chronic tensiontype headache. Headache. 2006;46:1264–1272. 18. Sun-Edelstein C, Mauskop A. Complementary and alternative approaches to the treatment of tension-type headache. Curr Pain Headache Rep. 2012;16: 539–544. 19. Holroyd KA, O’Donnell FJ, Stensland M, et al. Management of chronic tension-type headache with tricyclic antidepressant medication, stress management therapy, and their combination: a randomized controlled trial. JAMA. 2001;285(17):2208–2215. 20. Linde K, Allais G, Brinkhaus B, et al. Acupuncture for tension-type

headache. Cochrane Database Syst Rev. 2009(1):CD007587. 21. Leinisch-Dahlke E, Jürgens T, Bogdahn U, Jakob W, May A. Greater occipital nerve block is ineffective in chronic tension type headache. Cephalalgia. 2005; 25(9):704–708. 22. Steiner TJ, Lange R, Voelker M. Aspirin in episodic tension-type headache: placebo-controlled dose-ranging comparison with paracetamol. Cephalalgia. 2003;23:59–66. 23. Prior MJ, Cooper KM, May LG, Bowen DL. Efficacy and safety of acetaminophen and naproxen in the treatment of tension-type headache: a randomized, double-blind, placebo-controlled trial. Cephalalgia. 2002;22: 740–748. 24. Bendtsen L, Jensen R, Olesen J. A non-selective (amitriptyline), but not a selective (citalopram), serotonin reuptake inhibitor is effective in the prophylactic treatment of chronic tension-type headache. J Neurol Neurosurg Psychiatr. 1996;61(3):285–290.

315

Section 5 Chapter

44

Headaches and Facial Pain

Pain management in trigeminal neuralgia: clinical case illustrations Joaquin Maury, Alan David Kaye, and Harry J. Gould, III

Introduction Trigeminal neuralgia (TN) is an exquisitely painful and incapacitating illness that affects approximately 10 000 people in the USA each year. It is one of the most common causes of facial pain. In its classic form the condition presents spontaneously without an obvious precipitating cause or neurologic deficit and is notoriously difficult to treat. The refractory nature of the disease has motivated physicians to search for underlying causes for the pain and for ways to treat or manage the condition. These investigations in some cases have identified conditions associated with the pain that characterizes TN, that when treated have greatly ameliorated the otherwise intractable state. We will discuss the approach to the assessment and management of TN and consider the benefits and risks of implementing the different treatment modalities that are in current use.

Clinical cases Patient 1 is an 83-year-old male with hypertension and coronary artery disease who presented to clinic with a complaint of intermittent, 10/10 intensity pain in the distribution of the V2 division of the right CN V that had electrical, knife-like, and shooting qualities. The patient describes a 20 pound weight loss over the past 2 years, isolation, and limited ability to eat. The pain had begun 6 years earlier and initially occurred approximately once every 1–2 months in attacks that lasted only seconds. These attacks were eventually controlled with a regimen of carbamazepine and baclofen until 1 year prior to presentation when, despite the use of medication, the attacks returned and progressed to the point where they were unrelenting; occurring daily without significant periods of relief. Neither physical

examination nor MRI revealed a source of the pain. The patient was treated with percutaneous radiofrequency rhizotomy of the gasserian ganglion at 60°C for 60 seconds. The patient was pain free for 1 year following the procedure and only experienced mild facial hippesthesia for 2 months. Patient 2 is a 72-year-old, hypertensive female who presented with a 3-year history of paroxysmal, shooting pain of electrical or shock-like quality in the distribution of the left V2 division of CN V. Each episode of pain lasted only seconds, but the intensity was consistently rated at 10/10 on a numeric rating scale. The attacks were frequently triggered with stimulation of the left cheek, but no objective neurologic deficits were detected on physical examination. The patient would experience 5–7 episodes per day for 2 or 3 days at a time, with painfree periods of 2 weeks, between attacks. The patient described a 16 pound weight loss over the past 2 years. Magnetic resonance imaging revealed vascular compression of the left trigeminal nerve in the pontocerebellar basal cistern (Figure 44.1). Following surgical microvascular decompression (Figure 44.2), the patient was pain free for 3 years. Patient 3 is a 60-year-old male who presented to clinic complaining of intermittent, shooting, and electrical facial pain that involved the left V1, V2, and V3 territories of CN V. The pain had evolved over a 3-month period and had been preceded by double vision and numbness over the left side of the face. The physical examination revealed a decreased response of the left corneal reflex, hippesthesia to light touch and pin prick over the left side of the face, weakness of the left extraocular muscles, and right hemiparesis with mild spasticity, hyperreflexia (+3) and Babinski sign on the right. Computed tomography (CT) of the head revealed a homogeneous, 5 cm × 3.5 cm,

Case Studies in Pain Management, ed. Alan David Kaye and Rinoo V. Shah. Published by Cambridge University Press. © Cambridge University Press 2015.

316

Chapter 44: Pain management in trigeminal neuralgia: clinical case illustrations

Figure 44.1. T2-weighted magnetic resonance image of the brain taken in patient 2. The compression of the vascular loop over the left trigeminal root can be appreciated.

Figure 44.3. CT image of the head performed in patient 3. A mildly hyperdense mass (approximate 5 cm × 3.5 cm) can be seen with a pathologic diagnosis of condrosarcoma of the cranial base (petroclival bone) projected to the left prepontine and pontocerebellar cisterns as well as at the left cavernous sinus and medial aspect of the temporal fossa with compression of the brainstem and the basilar artery (circled).

Figure 44.2. Surgical intraoperative image taken during microvascular decompression in patient 2. It shows the pads placed between the trigeminal nerve and the vascular loops.

mildly hyperdense mass located in the left pre-pontine, petroclival area, and cavernous sinus (Figure 44.3). A complicated surgical plan was followed, involving independent and sequential pterional and retromastoid approaches in order to facilitate the tumor resection and decompression of neural structures (Figure 44.4). These procedures were followed by radiosurgical ablation of the left trigeminal dorsal root entry zone (DRZ). These procedures resulted in total pain relief, but a residual sensory loss has necessitated continued pharmacologic treatment to prevent corneal ulceration.

Figure 44.4. Postoperative CT image of the head performed in patient 3. It shows the decompression of neural structures after a different surgical approach.

317

Chapter 44: Pain management in trigeminal neuralgia: clinical case illustrations

1. What are the history, incidence, and epidemiology of trigeminal neuralgia? Trigeminal neuralgia, or tic doloureux, is an extremely disabling illness that was first described in 1672 by Johannes Bausch, later in 1773 by John Fothergill,[1] and is capable of causing such suffering that it has sometimes been referred to as the suicide disease.[2] The worldwide peak incidence of TN is estimated to be approximately 4.5 cases per 100 000 population[3] and occurs in the sixth to eighth decades of life with a female predominance slightly higher than males in a ratio of 1.7:1. It is one of the most common causes of facial pain.

2. What are the features of trigeminal neuralgia? TN is characterized by paroxysmal attacks of unilateral, lancinating, electric shock, or stabbing facial pain, generally involving the second (V2) or third (V3) division of the trigeminal nerve (CN V) and lasting < 1 second to 2 minutes.[4] Less commonly the divisions of CN V are involved simultaneously, V1/V2, V2/V3 or all three divisions.[4] The isolated involvement of V1 occurs in less than 5% of cases.[5] The painful attacks classically are not accompanied by objective neurologic deficits or other identifiable causes of facial pain, but there is frequent association with anomalous ectatic vascular compression of CN V as it courses in the subarachnoid space from the pons to the gasserian ganglion in Meckel’s cave.[2] Trigeminal pain that presents bilaterally with a superimposed aching and/or burning quality and is accompanied by the loss of corneal reflex, sensory deficits, or other neurologic symptoms comprise the secondary or symptomatic variants of TN.[4] Weight loss and a feeling of isolation are common. The condition frequently responds well to medical management, but in refractory cases, in patients who cannot tolerate medications, and frequently in cases of secondary TN, surgical modalities provide a viable and satisfactory approach to treatment.

3. What is the etiology and pathophysiology of trigeminal neuralgia? The etiology and pathophysiology of the classic lancinating, shock-like, and stabbing pain of TN remain

318

unknown, although 80–90% of cases are thought to occur through compression by an aberrant loop of an artery or vein. Other causes of TN via nerve compression include vestibular schwannoma (acoustic neuroma), meningioma, epidermoid or other cysts, saccular aneurysm, or ateriovenous malformation. TN caused by structural lesions other than vascular compression is classified as secondary TN. As illustrated by patient 1, this constellation of symptoms frequently presents in the absence of any neurologic deficits or identifiable morphologic abnormalities,[4] but in many cases (patient 2) the symptoms are found to coexist in the presence of ectatic vascular patterns in the posterior cranial fossa that compress the roots of CN V in the subarachnoid space between the pons and Meckel’s cave. The chronic compression in these instances may produce focal areas of demyelination and the development of spontaneous, ectopic firing and ephaptic connections at the site of compression or at the level of Redlich-Obersteiner’s zone due to abnormal microstructure (lower fractional anisotropy) leading to signal intensification[6–9] and possibly a descending modulation inhibition.[10] By contrast, the secondary or symptomatic forms of TN are more typically associated with a superimposed burning and/or aching quality of pain and as illustrated by patients 3 and 4 demonstrate neurologic deficits on physical examination and are accompanied by identifiable anatomical abnormalities, best revealed by MRI.[4,9]

4. How is a diagnosis of TN made? Some patients have a history of “pretrigeminal neuralgia,” which is said to be dull, continuous, aching pain in the jaw which evolves eventually into TN. This brief, milder pain is sometimes suspected to have a dental origin and unnecessary dental procedures are often performed in many cases. It should be noted that TN can be precipitated by dental procedures (e.g., dental extraction), resulting in increased confusion about the precise etiology of this problem. The course of TN is variable. Episodes may last weeks or months, followed by pain-free intervals. Recurrence is common, and some patients have continuous pain. Most often, the condition tends to wax and wane in severity and frequency of pain exacerbations.

Chapter 44: Pain management in trigeminal neuralgia: clinical case illustrations

5. What are the distinct management categories for trigeminal neuralgia? Conceptually, the pain associated with classic, idiopathic TN and its secondary or symptomatic form falls into two distinct management categories.[4,5,10,11] The classic form is a quintessential example of neuropathic pain; pain associated with an abnormally functioning nervous system. The pain of the secondary form is generally of mixed type with components of nociceptive pain associated with the tissue injury comprising the underlying pathology and characteristics of neuropathic quality pain. In addition to taking a good clinical and pain history the goal of assessment for TN is to determine any underlying and potentially manageable pathology. Treatment is then focused on elimination or control of the underlying cause and the reduction of pain primarily utilizing pharmacologic and interventional modalities. The first-line medications for treating TN are the adjuvant medications; medications with analgesic properties that are designed for another purpose. Because of its efficacy in treating the paroxysmal neurologic condition of epilepsy, carbamazepine was considered a reasonable candidate for treating the paroxysmal attacks of TN and was implemented in the treatment with remarkable success. Carbamazepine subsequently became the drug of choice for managing idiopathic TN, with 58–100% of patients realizing complete or near complete pain control at doses between 300 mg and 2400 mg daily.[12] Second-line management based on the augmentation of central inhibition was achieved with the addition of baclofen as an alternative treatment. Current options for consideration in the treatment of TN have expanded to include other adjuvant medications found to be beneficial for treating other neuropathic disorders due to comparable efficacy for pain control and improved profiles for adverse effects.[12] Other adjuvant medications that may have a role in treating TN including tricyclic antidepressants, selective serotonin and norepinephrine antidepressants, antiepileptic medications, local anesthetics, anti-spasmodics, cannabinoids, and NMDA antagonists have been discussed elsewhere in this book[13,14] and the subcutaneous and perineural application of botulinum toxin may soon be added to this list.[15,16] In cases of secondary TN approaches for managing nociceptive pain are appropriate and usually necessary.[11] The best analgesic regimens that provide coverage for

nociceptive pain include non-opioid (aspirin, acetaminophen, and NSAIDs) and opioid agents and when used with the adjuvant medications for neuropathic pain may provide analgesic synergy for nociceptive pain. Routine, on the clock, dosing of long-acting medications should be provided for managing baseline levels of chronic pain with additional immediate-release medications for breakthrough pain with attention to follow-up evaluation, assessment and management of adverse effects, and evidence of misuse, abuse and diversion of controlled formulations. Interventional alternatives to medical management, e.g., the percutaneous procedures, open surgery and radiosurgery, have been used with success in the management of intractable TN when conservative medical management fails to reduce pain and improve the quality of life due to lack of efficacy or intolerable adverse effects and are often essential early in the management of secondary TN.[17] The percutaneous peripheral branch blocks and local neural ablations target the peripheral branches of CN V and have a 50% incidence of pain recurrence within 1 year,[17] whereas other procedures including the radiofrequency thermo-rhizotomy (patient 1), balloon compression, and the glycerol rhizotomy target the gasserian ganglion. These procedures are neurodestructive in nature and are associated with a 50% risk of sensory loss, a 6% risk of dysesthesia, and a 4% risk of anesthesia dolorosa and corneal anesthesia.[4,17–19] Percutaneous radiofrequency trigeminal rhizotomy (PTR) and surgical microvascular decompression are the most reliable procedures for the treatment of TN due to their relatively low recidivism rate of approximately 5–15%.[4,20–22] Both techniques may be indicated when medical management fails or as an alternative procedure when the other has failed in the attempt to control pain. Percutaneous radiofrequency thermocoagulation is a minimally invasive therapeutic procedure that can be performed under monitored anesthetic care or conscious sedation. Patient is in a supine position. A skin wheal is raised lateral to labial commissure. An angiocatheter is slowly advanced toward the foramen ovale, using a submental fluoroscopic view. The inner buccal mucosa is palpated to ensure no needle perforation. Gloves are replaced. Thereafter, an RF curved blunt needle is advanced through the angiocatheter toward the foramen ovale. This part can be painful. The final needle position is confirmed near the base of

319

Chapter 44: Pain management in trigeminal neuralgia: clinical case illustrations

the posterior clinoid process. Testing at a low frequency (2 Hz) will stimulate the V3 branch of the trigeminal ganglion; a bite block or gauze should be placed in the mouth during this testing, since the masseter muscle will be stimulated and the jaw will clench/relax cyclically. Sensory stimulation at a higher frequency (50 Hz) will allow one to determine if the V1, V2, or V3 branches are within the proximity of the RF needle. A curved RF needle can be rotated to differentially capture one branch over another. RF lesioning should commence at a temperature lower and a duration shorter than that for spinal RF procedures: 60°C and 60 seconds. The corneal reflex should be monitored. A local anesthetic and steroid should be instilled before lesioning commences. In the recovery area, a cold pack may be placed on the face. Pulsed mode RF, which is not neurolytic

Figure 44.5. Submental fluoroscopic view. From personal files of Rinoo V. Shah, MD, MBA.

(temperatures less than 45°C), is another alternative to conventional RF. Figures 44.5 to 44.10 demonstrate fluoroscopic identification of the foramen ovale and sequential steps for RF needle placement (images courtesy of Rinoo V. Shah, MD, MBA personal files). Open procedures are reserved for managing underlying pathology in patients with secondary or symptomatic TN and include intracranial operations for the removal of tumors, clipping of aneurysms, and post-trauma anatomical reconstruction.

6. What is microvascular decompression for trigeminal neuralgia? Microvascular decompression was proposed and developed by Peter J. Jannetta[22,23] and has become the mainstay of interventional management of TN. The procedure has been used with remarkable success in patients where there is evidence of close proximity between elements of CN V and vascular structures, e.g., arteries and veins, that serve the brainstem and cerebellum and involves the placement of a buffering pad between the portion of CN V and the offending artery. The procedure is recommended for patients with inadequate medical control of pain, with greater than 5 years anticipated survival, and who are able to tolerate a small craniotomy and surgical anesthesia. Microvascular decompression generally produces pain relief in 90% of patients with over 80% remaining pain free for 1 year. Seventy-three to 75% of patients remain pain free at 5 years after the procedure.[5,22–26] Complications including sensory loss, cerebral spinal fluid Figure 44.6. Introducer/angiocatheter advanced to 3 o'clock position of left foramen ovale. From personal files of Rinoo V. Shah, MD, MBA.

320

Chapter 44: Pain management in trigeminal neuralgia: clinical case illustrations

Figure 44.7. Curved blunt RF needle steered into foramen ovalesubmental view. From personal files of Rinoo V. Shah, MD, MBA.

Figure 44.8. Lateral fluoroscopic view with landmarks. From personal files of Rinoo V. Shah, MD, MBA.

leaks, infarcts, and cranial nerve damage are infrequent, occurring less than 4–7% of the time; the average mortality rate associated with the operation is 0.2–0.5%.[4,21–25] A surgical variant of the Jannetta procedure was published in 2009 to benefit patients who during microvascular decompression were found intraoperatively not to possess a visible vascular compression. The procedure is called “nerve combing” and consists of splitting the nerve longitudinally into several fascicles.[25–27]

7. What are some of the advantages of microvascular decompression for TN? Microvascular decompression is the only nondestructive procedure for the treatment of TN. It also provides low risk of facial sensory loss with subsequent dysesthesias or anesthesia dolorosa. Finally, microvascular decompression is the only operation that addresses what is believed to be the primary underlying pathology of vascular compression.

321

Chapter 44: Pain management in trigeminal neuralgia: clinical case illustrations

Figure 44.9. Needle entering foramen ovale towards Meckel’s cave. From personal files of Rinoo V. Shah, MD, MBA.

Figure 44.10. Contrast instillation. From personal files of Rinoo V. Shah, MD, MBA.

8. Are there any other alternative techniques for trigeminal neuralgia? Stereotactic radiosurgery is the least invasive surgical option for treating patients with TN. It is being used more frequently and achieving complete or partial relief of pain in 85.6% of patients at 1 year and in 75.4% of patients at 33 months. Patients with atypical pain presentations have low relief with this method of treatment.[28–31]

322

Following on the success of managing Parkinson’s disease with deep brain stimulation, it has been postulated that direct central neurostimulation of selected targets along the pain pathway, e.g., the peripheral branches of CN V, the sphenopalatine and gasserian ganglia, mesencephalic and thalamic nuclei, the cingulate gyrus, and some portions of the motor cortex, may be effective in modulating the pain of TN.[10,31] Further development and refinement of the role of neuromodulating techniques for the

Chapter 44: Pain management in trigeminal neuralgia: clinical case illustrations

treatment of TN is warranted and will require additional multicenter trials with extended follow-up to validate efficacy and determine safety.[32,33]

of the underlying cause and the prudent use of available treatment modalities. Concomitant use of treatment modalities is often the most appropriate and effective in reaching the desired goal.

9. What are the key points regarding trigeminal neuralgia for the clinician?

Disclosure

Optimum management of the pain associated with TN requires careful and comprehensive assessment

The author(s) have no potential conflicts of interest to disclose.

References 1.

Stookey B, Ransohoff J. Trigeminal Neuralgia: Its History and Treatment. Springfield, IL: Charles C. Thomas. 1959.

2.

Eboli P, Stone JL, Audin S, Slavin KV. Historical characterization of Trigeminal Neuralgia. Neurosurgery. 2009;64:1183–1187.

3.

Tenser R. Trigeminal Neuralgia: mechanisms of treatment. Neurology. 1998;51:17–19.

4.

Gronseth G, Cruccu G, Alksne J, et al. Practice parameter: the diagnostic evaluation and treatment of trigeminal neuralgia (an evidence-based review). Neurology. 2008;71:1183–1190.

5.

Zakrzewska JM. Diagnosis and differential diagnosis of trigeminal neuralgia. Clin J Pain. 2002;18:14–21.

6.

Devor M, Amir R, Rappaport H. Pathophysiology of Trigeminal Neuralgia: The Ignition Hypothesis. Clin J Pain. 2002; 18:4–13.

7.

8.

DeSouza DD, Hodaie M, Davis K. Abnormal trigeminal nerve microstructure and brain white matter in idiopathic trigeminal neuralgia. Pain. 2013. Available from: http://dx.doi.org/j. pain.2013.08.029/. Lacerda Leal PR, Amedee Roch J, Hermier M, et al. Structural abnormalities of the trigeminal root revealed by diffusion tensor imaging in patients with trigeminal neuralgia caused by neurovascular compression: a prospective, double-blind,

controlled study. Pain. 2011;152:2357–2364. 9.

Anderson VC, Berryhill PC, Sanquist MA, et al. Height resolution three-dimensional magnetic resonance angiography and three-dimensional spoiled gradient-recalled imaging in the evaluation of neurovascular compression in patients with trigeminal neuralgia; a doubleblind pilot study. Neurosurgery. 2006;58:666–673.

10. Leonard G, Goffaux P, Mathieu D, et al. Evidence of descending inhibition deficits in atypical but not classical trigeminal neuralgia. Pain. 2009;147:217–223. 11. Prasad S, Galetta S. Trigeminal neuralgia: historical notes and current concepts. The Neurologist. 2006;15:87–94. 12. Sindrup S, Jensen T. Pharmacotherapy of trigeminal neuralgia. Clin J Pain. 2002;18: 22–27. 13. Gould HJ III. Management of painful neuropathies. Curr Treat Opt Neurol. 2007;9:75–84. 14. Piovesan EJ, Randunz V, Utiumi M, et al. Influence of NMDA and non-NMDA antagonists on acute and inflammatory pain in the trigeminal territory: a placebo control study. Arq NeuroPsiquiatria. 2008;66:837–843. 15. Fabregat G, De Andres J, Villanueva-Perez, VL, AsensioSamper JM. Subcutaneous and perineural botulinum Toxin Type A for neuropathic pain. Clin J Pain. 2013;29:1006–1012.

16. Allam N, Brasil-Neto J, Brown G, Tomaz C. Injections of Botulinum toxin Type A produce pain alleviation in intractable trigeminal neuralgia: case report. Clin J Pain. 2005;21:182–184. 17. Amirnovin R, Neinat JS, Roberts JA, Eskandar EN. Multimodality treatment of trigeminal neuralgia. Stereotact Funct Neurosurg. 2005;83:197–201. 18. Gary P, Nurmikko T. Peripheral and Gasserian ganglion level procedures for the treatment of trigeminal neuralgia. Clin J Pain. 2002;18:28–34. 19. Kanpolat Y, Savas A, Bekar A Berk C. Percutaneous controlled radiofrequency trigeminal rhyzotomy for the treatment of idiopathic trigeminal neuralgia: 25 years experience with 1600 patients. Neurosurgery. 2001;48:524–534. 20. Zundert JV, Brabant S, Van de Kelft E, Vercruyssen A, Van Buyten JP. Pulsed radiofrequency treatment of the Gasserian ganglion in patients with idiopathic trigeminal neuralgia. Pain. 2003;104:449–452. 21. Zakrzewska J, Coakham HB. Microvascular decompression for trigeminal neuralgia. Curr Opin Neurol. 2012;25:296–301. 22. Jannetta PJ, McLaughlin MR, Casey KF. Technique of microvascular decompression. Neurosurg Focus. 2005;18:1–5. 23. Koopman J, De Vries LM, Dieleman JP, et al. A nationwide study of three invasive treatments

323

Chapter 44: Pain management in trigeminal neuralgia: clinical case illustrations

for trigeminal neuralgia. Pain. 2011;152:507–513. 24. Cole CD, Liu JK, Apfelbaum RI. Historical perspectives on the diagnosis and treatment of trigeminal neuralgia. Neurosurg Focus. 2005;18:1–10. 25. Ma Z, Li M. Nerve combing for trigeminal neuralgia without vascular compression. Clin J Pain. 2009;25:44–47. 26. Sekula RF Jr, Frederickson AM, Jannetta PJ, et al. Microvascular decompression for elderly patients with trigeminal neuralgia: a prospective study and systematic review with meta-analysis. J Neurosurg. 2011;114(1):172–179. 27. Zakrzewska J, Lopez BC. New techniques for surgical

324

management of trigeminal neuralgia. Letter to the editor. Pain. 2004;109:520–529. 28. Shetter AG, Rogers CL, Ponce F, et al. Gamma knife radiosurgery for recurrent trigeminal neuralgia. J Neurosurgery. 2002;97(5): 536–538. 29. Maesawa S, Salame C, Flickinger JC, et al. Clinical outcomes after stereotactic radiosurgery for idiopathic trigeminal neuralgia. J Neurosurg. 2001;94;14–20. 30. Chang, JW, Chang JH, Park YG, Chung SS. Gamma knife radiosurgery for idiopathic and secondary trigeminal neuralgia. J Neurosurg. 2000; 93(Suppl 3):147–151.

31. Dhople AA, Adams JR, Maggio WW, et al. Long-term outcomes of gamma knife radiosurgery for classic trigeminal neuralgia: implications of treatment and critical review of the literature. J Neurosurg. 2009; 111:351–358. 32. Marques A, Chassin O, Morand D, et al. Central pain modulation after sub-thalamic nucleus stimulation: a crossover randomized trial. Neurology. 2013;81:633–640. 33. Machado A, Ogrin M, Rosenow JM, Henderson JM. A 12-months prospective study of gasserian stimulation for trigeminal neuropathic pain. Stereotac Funct Neurosurg. 2007;85(5): 216–224.

Section 5 Chapter

45

Headaches and Facial Pain

Patient with chronic glossopharyngeal neuralgia/post-tonsillectomy pain A. Raj Swain

Case study

2. Describe the signs and symptoms

A 65-year-old White male presents for the evaluation and treatment of chronic throat pain. The patient has sharp, stabbing pain in the back of the right side of his throat which radiates upward and to the right ear. Pain is episodic and lasts approximately 30 seconds but occurs 20 to 30 times per day. The patient rates his pain 10/10 when it occurs and notes that he becomes nauseous and lightheaded during these episodes. The patient notes that his symptoms began after he had a tonsillectomy performed to treat obstructive sleep apnea. Initially he was without pain but over the last 6 months pain has worsened. He notes that pain is particularly severe when he swallows a cold beverage. His primary care physician began an initial work-up of the problem including CT scan and MRI of the brain and neck. All studies were negative. The patient has been trialed on membrane stabilizing medication including gabapentin and pregabalin with minimal improvement. What is the diagnosis?

Clinically, GN shares many features with trigeminal neuralgia. Pain is neuropathic in quality and is paroxysmal in nature. A major difference is the presence of pain in the tonsillar region and posterior pharynx in GN compared to the face in TN.[5,6] There can be transmission of pain to the mandibular region and to the ipsilateral ear. Attacks are often triggered by ordinary activities such as swallowing, chewing, speaking, coughing, sneezing, throat-clearing, or by head rotation.[5] Several patients reported that touching the external auditory canal, the side of the neck, and the skin anterior to the ipsilateral ear triggered pain.[3] Swallowing of cold liquids is especially likely to cause pain. The passage of food or liquid brushing the area rather than the act of swallowing causes pain. Association with trigeminal neuralgia is not uncommon and can occur in approximately 10% of cases.[7] Painful episodes can range from less than 1 second in duration to 2 minutes. However, the episodes can occur in rapid succession (status neuralgicus). When it is triggered by swallowing, feeding can be impaired and marked weight loss can occur. Pain is unilateral and a bilateral presentation may be indicative of underlying multiple sclerosis. In approximately 2% of patients, severe bradycardia may be present with resultant loss of consciousness.[7]

1. Whatisglossopharyngealneuralgia? Glossopharyngeal neuralgia (GN) was first described by Weisenberg in a 35-year-old male patient with a mass at the cerebellopontine angle.[1] The patient had been treated for 6 years for trigeminal neuralgia. Harris described the idiopathic type and established the term glossopharyneal neuralgia in 1921.[2–4] The estimated incidence is approximately 0.8 cases per 100000 patients (0.9 and 0.5 in men and in women, respectively). The left nerve is more often involved than the right.[3,4] The incidence of GN increases with age. The syndrome rarely presents before the age of 20 and most patients start having symptoms after the age of 50.[5,6]

3. Discuss the etiology and pathophysiology The etiology of GN can be divided into two groups: those without discernible cause, often referred to as idiopathic or essential GN and those with an underlying pathologic process such as neoplasm, infection or inflammation, or structural abnormality such as

Case Studies in Pain Management, ed. Alan David Kaye and Rinoo V. Shah. Published by Cambridge University Press. © Cambridge University Press 2015.

325

Chapter 45: Patient with chronic glossopharyngeal neuralgia/post-tonsillectomy pain

elongated styloid process, referred to as secondary GN[6] (Table 45.1). The precise pathophysiology of GN is unknown but likely involves both central and peripheral mechanisms. GN has been linked to vascular compression of the nerve entry zone resulting in demyelination and ephaptic transmission. Vascular compression has also been thought to induce repetitive activation of primary afferent signals in the nerve and has led to hyperactivity and hyperexcitability Table 45.1. Etiology of glossopharyngeal neuralgia[4]

1. Idiopathic (essential) form a. Vascular compression b. Central (pontine) dysfunction

of the central neurons. Activation of NMDA receptors has been introduced as another possible explanation.[3] Cardiovascular changes such as bradycardia and asystole are likely due to the close connection between the vagus nerve and the glossopharyngeal nerve and likely vago-glossopharyngeal reflex arch. Irritative or ischemic lesions in the glossopharyngeal nerve may create afferent nerve impulses that reach the solitary nucleus of the midbrain and via collateral tracts, reach the dorsal motor nucleus of the vagus nerve; these events may cause reflex bradycardia and even, asystole. Convulsive movements, limb clonus, smacking movements of the limbs, and upward turning of the eyes are signs of cerebral hypoxia secondary to bradycardia.[3]

2. Secondary (symptomatic) a. Neoplasm i. pharyngeal tumor ii. tongue and tonsillar tumors iii. skull base tumors iv. cerebellopontine angle tumors v. brainstem tumors b. Vascular malformations i. persistent hypoglossal artery ii. arteriovenous malformations iii. fusiform aneurysm c. Infection i. tonsillitis/pharyngitis ii. petrositis iii. parapharyngeal abscess iv. tuberculosis v. arachnoiditis d. Demyelination i. multiple sclerosis e. Trauma i. missile wounds ii. skull base fracture iii. post-tonsillectomy iv. post neck dissection v. post craniotomy vi. post radiation f.

Elongated styloid process i. Eagle’s syndrome

g. Other i. Chiari malformation ii. choroid plexus overgrowth iii. Tornwald’s cyst iv. vagus nerve stimulation

326

4. Diagnosis The diagnosis of GN is made based upon clinical evaluation. There is no specific test for this condition[8,9] (Tables 45.2 and 45.3). Initial evaluation should involve otolaryngology to exclude other diagnoses. GN should be considered in pain patterns including the throat and the ear. Pain triggered by palpation of the palate or tonsil is consistent with the condition.[8] Trigger zones can be identified by anesthestizing the palate with local anesthetic. Imaging with CT scan or MRI of the cranial base, cervical spine, and vertebral vasculature can identify tumors, demyelinating lesions, or vascular compression causing symptoms.[8–10] High-resolution CT scan of the cervical spine can detect the presence of an elongated styloid process, Table 45.2. IHS Diagnostic criteria for classical glossopharyngeal neuralgia[7]

A. Paroxysmal attacks official pain lasting from a fraction of a second to 2 minutes and fulfilling criteria B and C B. Pain has all of the following characteristics: a. unilateral location b. distribution within the posterior part of the tongue, tonsillar fossa, pharynx, or beneath the angle of the lower jaw and/or in the ear c. sharp, stabbing, and severe d. precipitated by swallowing, chewing, talking, coughing, and/or yawning C. Attacks are stereotyped in the individual patient D. There is no clinically evident neurologic deficit E. Not attribute it to another disorder

Chapter 45: Patient with chronic glossopharyngeal neuralgia/post-tonsillectomy pain

Table 45.3. IHS Diagnostic criteria for symptomatic glossopharyngeal neuralgia[7]

A. Paroxysmal attacks official pain lasting from a fraction of a second to 2 minutes and fulfilling criteria B and C B. Pain has all of the following characteristics: a. unilateral location b. distribution within the posterior part of the tongue, tonsillar fossa, pharynx, or beneath the angle of the lower jaw and/or in the ear c. sharp, stabbing, and severe d. precipitated by swallowing, chewing, talking, coughing, and/or yawning C. Attacks are stereotyped in the individual patient D. There is no clinically evident neurologic deficit E. A cause of lesion has been demonstrated by special investigations and/or surgery

suggestive of Eagle syndrome.[4,11–13] Definitive diagnosis can be made when the nerve is blocked at the jugular foramen or topical anesthesia of the pharynx stops pain.[8] GN is a rare condition and clinical overlap with other conditions often leads to misdiagnosis. The differential diagnosis for GN should include trigeminal neuralgia, superior laryngeal neuralgia (superior laryngeal branch of CN X), and nervus intermedius neuralgia (somatic sensory branch of CN XII).[3,4,8] Trigeminal neuralgia is also a cranial neuralgia with similar features to GN. There are multiple connections between the mandibular division of CN V and CN IX which can cause confusion. However, the distribution of pain is different (V: nasal ala, upper lip, or cheek versus IX: tonsillar fossa and posterior pharynx). Triggering factors also differ between the two conditions (V: cold breeze, touching or washing the face, shaving, talking, or chewing versus IX: swallowing).[8,14,15] Other subtle differences exist between TN and GN – GN is more common on the left, while TN is more common on the right.[16] TN is more likely to have a bilateral presentation and is more likely to be associated with multiple sclerosis. Superior laryngeal neuralgia (SLN) can also be confused with GN because of throat pain and association to swallowing.[17,18] SLN produces sudden shock-like pain on the side of the thyroid cartilage, pyriform sinus, angle of the jaw, and occasionally in the ear. Attacks are usually triggered by

talking, swallowing, yawning, or coughing. Local anesthetic blockage of the superior laryngeal nerve is useful from both a diagnostic and prognostic standpoint.[17,19] Nervus intermedius neuralgia (NIN) is misdiagnose as GN when the only symptom is sensory loss in the ear.[20] The glossopharyngeal nerve lies in proximity to the nervus intermedius. NIN pain is intermittent, stabbing, and electric pain deep in the ear. The trigger is often non-noxious stimulation of the ear canal or can follow swallowing or talking. Usually pain is triggered spontaneously.[8] The patient is pain free between attacks.

5. Treatment The treatment of GN can be pharmacologic or interventional (surgical). The first line of treatment is pharmacological. Interventional options should be considered for drug intolerance, inefficacy, allergies, or side effects.[8] The pharmacologic treatment of GN includes anticonvulsant and membrane stabilizing medications such as carbamazepine, phenytoin, oxacarbazepine, gabapentin, or pregabalin. Opioid analgesic medications have generally been ineffective but tricyclic antidepressant medications have been effective in combination with anticonvulsants and membrane stabilizers or alone.[8,9,21,22] Atropine should be used for cardiovascular symptoms. Atropine administration will treat hemodynamic changes but will not treat painful attacks. Interventional treatments range from nerve blocks to neurosurgical procedures. Nerve blocks can be performed with or without neurolytic agents and can include additives such as steroid or ketamine. The interventional treatments with the highest rates of pain relief are rhizotomy and microvascular decompression of cranial nerves IX and X. Microvascular decompression is the first choice treatment as it has the highest initial and long-term success rates.[23,24] Rhizotomy is a safe and effective back-up procedure if microvascular decompression is difficult.[8,23,24] If exploratory surgery does not identify a compressing vessel, CN IX can be sectioned along with the upper rootlets of CN X. This procedure can lead to dysphagia and vocal cord paralysis.[8,23] The pain from Eagle syndrome can be cured with resection of the elongated styloid process through minimally invasive surgery. Other available techniques include radiofrequency nerve ablation, balloon compression, proton beam therapy,

327

Chapter 45: Patient with chronic glossopharyngeal neuralgia/post-tonsillectomy pain

gamma knife therapy, and neurostimulation.[8,25] Shah et al described the successful pulsed radiofrequency lesioning of the glossopharyngeal nerve for secondary glossopharyngeal neuralgia, e.g., post-

References 1.

2.

Weisenberg TH. Cerebellopontine tumor diagnosed for six years as tic doloureaux: the symptoms of irritation of the ninth and twelfth cranial nerves. J Am Med Ass. 1910;54: 1600–1604. Harris W. Persistent pain in lesions of the peripheral and central nervous system. Brain. 1921;44:557–571.

3.

Evans RW, Torelli P, Manzoni GC. Glossopharyngeal neuralgia. Headache. 2006;46:1202.

4.

Teixeira MJ, de Siqueira SRDT, Bor-Seng-Shu E. Glossopharyngeal neuralgia: neurosurgical treatment and differential diagnosis. Acta Neurochir. 2008;150:471–475.

5.

DeSimone R, Ranieri A, Bilo L, et al. Cranial neuralgias: from physiopathology to pharmacological treatment. Neurol Sci. 2008;29:S69–S78.

6.

Slavin K. Glossopharyngeal neuralgia. Semin Neurosurg. 2004;15(1):72–79.

7.

Rozen TD. Trigeminal Neuralgia and glossopharyngeal neuralgia. Neurol Clin Am. 2004;22:185–206.

8.

Blumenfeld A, Nikolskaya G. Glossopharyngeal neuralgia. Curr Pain Headache Rep. 2013;17:343.

9.

Headache Classification Subcommittee of the International Headache Society. The International Classification of Headache Disorders, 2nd edition. Cephalalgia. 2004;24(Suppl 1): 9–160.

10. Elias J, Kuniyoshi R, Carloni WV, et al. Glossopharyngeal neuralgia associated with cardiac syncope.

328

tonsillectomy pain. This technique is less invasive than some of the surgical options and if successful, offers a bridge type therapy between non-interventional and surgical approaches.[26]

Arq Bras Cardiol. 2002;78(5): 149–152. 11. Son KB. The glossopharyngeal nerve, glossopharyngeal neuralgia and the Eagle’s syndrome: current concepts and management. Singapore Med J. 1999;40:659–665. 12. Rozen TD. Trigeminal neuralgia and glossopharyngeal neuralgia. Neurol Clin. 2004;22(1):185–206. 13. Kawasaki M, Hatashima S, Matsuda T. Non-surgical therapy for bilateral glossopharyngeal neuralgia caused by Eagle’s syndrome, diagnosed by threedimensional computed tomography: a case report. J Anesth. 2012;26(6):918–921. 14. Zakrzewska JM. Medical management of trigeminal neuropathic pain. Expert Opin Pharmacother. 2010;11:1239–1254. 15. Zakrzewska JM, McMillan R. Trigeminal Neuralgia: the diagnosis and management of this excruciating and poorly understood facial pain. Postgrad Med J. 2011;87(1028):410–416. 16. Katusic S, Williams DB, Beard CM, et al. Incidence and clinical features of glossopharyngeal neuralgia, Rochester, Minnesota, 1945–1984. Neuroepidemiology. 1991;10(5–6):266–275. 17. Bruyn GW. Superior laryngeal neuralgia. Cephalagia. 1983;3(4): 235–240. 18. Aydil U, Kizil Y, Koybasioglu A. Less known non-infectious and neuromusculoskeletal systemoriginated anterolateral neck and craniofacial pain disorders. Eur Arch Otorhinolaryngol. 2012; 269(1):9–16.

19. Baugh RF, Baugh A, Bunge F. Superior laryngeal nerve syndrome and the evaluation of anterior neck pain. Am J Otolaryngol. 2012;33(4):481–483. 20. Tubbs RS, Steck DT, Mortazavi MM, et al. The nervus intermedius: a review of its anatomy, function, pathology, and role in neurosurgery. World Neurosurg. 2013;79 (5–6):763–767. 21. Dalessio DJ. Diagnosis and treatment of cranial neuralgias. Med Clin North Am. 1991;75(3): 605–615. 22. Fromm GH. Clinical pharmacology of drugs used to treat head and face pain. Neurol Clin. 1990;8(1):143–151. 23. Rey-Dios R, Cohen-Gadol AA. Current neurosurgical management of glossopharyngeal neuralgia and technical nuances for microvascular decompression surgery. Neurosurg Focus. 2013;43(3):E8. 24. Ozenci M, Karaoguz R, Conkbayir C, Altin T, et al. Glossopharyngeal neuralgia with cardiac syncope treated by glossopharyngeal rhizotomy and microvascular decompression. Europace. 2003;5(2):149–152. 25. Anderson WS, Kiyofuji S, Conway JE, et al. Dysphagia and neuropathic facial pain treated with motor cortex stimulation: case report. Neurosurgery. 2009; E626. 26. Shah RV, Racz GB. Pulsed mode radiofrequency lesioning to treat chronic post-tonsillectomy pain (secondary glossopharyngeal neuralgia). Pain Pract. 2003;3(3): 232–237.

Section 5 Chapter

46

Headaches and Facial Pain

Patient with sphenopalatine neuralgia Mohit Rastogi, Natalia Murinova, and Alan David Kaye

Case study A 38-year-old male presents with unilateral facial pain that has been present for several years. There were no inciting events and the patient has been evaluated by several dentists and had extensive dental work without any benefit. His MRI of the brain is normal, and his neurologic exam is unremarkable. The pain is unilateral, always on the same side, intermittent and severe; it is described as a sharp, shooting pain located below the right orbit and deep behind the nose. During his attacks of pain, he describes unilateral nasal drainage, and ipsilateral eye irritation and lacrimation. He is not sure what causes his pain, and he is very upset with the lack of effective treatments thus far. He would like to know his diagnosis and treatment options. He has never been treated with any prescription medications other than opioids. He feels that you are his last hope, and he feels suicidal due to severe pain.

1. What is the differential diagnosis? a. b. c. d. e. f.

Migraine headache Cluster headache (CH) Glossopharyngeal neuralgia Postherpetic neuralgia Atypical facial pain Sphenopalatine neuralgia

2. What is Sluder neuralgia or sphenopalatine ganglion neuralgia? Sphenopalatine neuralgia is also called Sluder neuralgia, because Sluder was the first to describe this entity in 1908.[1] Sluder described a complex clinical syndrome that he associated with sphenopalatine ganglion.[2] He also suggested that treatment directed

at the sphenopalatine ganglion might alleviate the symptoms of the neuralgia.[2] There have been many descriptions of patients with sphenopalatine neuralgia over the last 90 years; in these cases the recommended treatment was directed at the sphenopalatine ganglion. It is suggested that what Sluder described as sphenopalatine ganglion neuralgia is a type of neurovascular headache with activation of the trigeminal nerve and autonomic nervous system, similar to CH.[1] There are a group of physicians who believe that there is no separate diagnostic entity of sphenopalatine neuralgia, and that these patients’ symptoms are better characterized as CH or as other trigeminal autonomic cephalalgias, such as short-lasting unilateral neuralgiform headache attacks with cranial autonomic symptoms (SUNA) and short-lasting unilateral neuralgiform headache attacks with conjunctival injection and tearing (SUNCT).[3] Neurologic literature suggests that Sluder’s neuralgia could be a type of trigeminal autonomic cephalalgia separate from CH.[4]

3. What is the anatomy of the sphenopalatine ganglion? The sphenopalatine ganglion (SPG) lies in the pterygopalatine fossa, a small 2 cm pyramidal space located posterior to the middle nasal conchae and anterior to the pterygoid canal. It is also known as pterygopalatine ganglion or Meckel’s ganglion. SPG is a parasympathetic ganglion and is one of four ganglions associated with the trigeminal nerve. The pterygopalatine fossa contains vascular structures (the internal maxillary artery and its branches) and nerves (the maxillary nerve of the trigeminal second branch, the sphenopalatine ganglion with its afferent and efferent branches). Sympathetic and parasympathetic fibers are both present in the sphenopalatine

Case Studies in Pain Management, ed. Alan David Kaye and Rinoo V. Shah. Published by Cambridge University Press. © Cambridge University Press 2015.

329

Chapter 46: Patient with sphenopalatine neuralgia

ganglion; the parasympathetic fibers are distributed to the nasal mucosa and the lacrimal branch (secretomotor supply).

4. What are the major nerve structures involved with the sphenopalatine ganglion? The maxillary branch of the trigeminal nerve (V2) passes through the foramen rotundum and through the sphenopalatine fossa and the sphenopalatine ganglion (SPG) is found just below this. It receives contributions from both the trigeminal nerve as well as facial nerve (via the greater petrosal nerve). The internal maxillary artery is also found in the sphenopalatine fossa. Sensory innervation is provided through the maxillary nerve (via sphenopalatine branches) and facial nerve (greater superficial petrosal nerve), and supplies parts of the orbit, nasal membranes, soft palate, and the pharynx. There are also parasympathetic contributions via the greater superficial petrosal nerve supplying the lacrimal gland and nasal and palatine mucosa. Additionally there are sympathetic contributions from the deep petrosal nerve from the superior cervical ganglion.

5. What is the proposed etiology of sphenopalatine neuralgia? There is no clear etiology for primary sphenopalatine neuralgia; however factors that can cause secondary sphenopalatine neuralgia include anatomical abnormalities in the form of mass effect (such as tumors), nasal deformities, vascular abnormalities (impinging internal maxillary artery), as well as infections. There are many patients in whom the neuralgia is primary, with no secondary cause found. In the primary sphenopalatine neuralgia, pain is thought to be due to activation of the trigeminal nerve, and the activation of the parasympathetic nervous system accounts for autonomic symptoms that include ipsilateral lacrimation and nasal congestion.

6. What is the clinical presentation of sphenopalatine neuralgia? In 1908 Sluder, for the first time, described sphenopalatine ganglion neuralgia and outlined the characteristic clinical picture. It is an extremely uncommon condition, more common in males and in people of

330

middle age. The pain is strictly unilateral and involves the orbital area, the inner canthus of the eye, and the root of the nose, with extension of pain to the maxillary sinus. The onset and the course of sphenopalatine neuralgia is characteristic: the attacks consist of continuous, sometimes mild pain beginning with little or no warning, with paroxysms of throbbing pain reaching maximum intensity in 1 to 10 minutes. These paroxysms may persist for hours, and are described as sharp knife-like sensations, distinct from those of trigeminal neuralgia. The presentation of sphenopalatine neuralgia is similar to trigeminal autonomic cephalalgias such as CH. The pain is usually unilateral and is often associated with ipsilateral autonomic features such as lacrimation, miosis, and rhinorrhea. Trigeminal activation is a common feature in many primary headaches, such as migraine, CH, and tension-type headache. However, what differentiates the group of trigeminal autonomic cephalalgias are the associated autonomic nervous system features such as tearing, nasal congestion, and eye redness. Within the classification of trigeminal autonomic cephalalgia are CH, paroxysmal hemicrania, and SUNCT. The parasympathetic features that are seen are likely mediated by activation of the trigeminal system involving the sphenopalatine ganglion.[5,6] Patients will often present with unilateral facial pain (can rarely be bilateral) located within the cheek, the periorbital region, mouth, and sometimes referred pain into the neck and shoulders (shared cervical root fibers). The distribution of the pain is in the V2 distribution of the trigeminal nerve. They often have dull continuous pain with intermittent pain spikes described as sharp, stabbing, moderate pain. Sphenopalatine neuralgia is usually continuous, with or without exacerbations that are typically much longer in duration than in CHs. Cluster headaches often appear in cluster periods with usually shorter duration of exacerbations and are more likely to afflict males than females, whereas the opposite is true for sphenopalatine neuralgia.[6–8] Cluster headaches are often very severe, unilateral, located in the orbital, temporal region, lasting 15–180 minutes and occurring from once every other day to eight times a day. The attacks are associated with one or more of the following, all of which are ipsilateral: conjunctival injection, lacrimation, nasal congestion, rhinorrhea, forehead and facial sweating, miosis, ptosis, and eyelid edema. Most patients are restless or agitated during an attack.[9]

Chapter 46: Patient with sphenopalatine neuralgia

7. What is the recommended work-up for suspected sphenopalatine neuralgia? The history is the most important part in the diagnosis of headaches. Use International Headache Society criteria for headache diagnosis. Thorough history should be taken to identify characteristics of the pain such as onset, palliation/provocation, quality, radiation, severity, and temporal elements of the pain. Physical examination can sometimes show dysesthesia in the V2 nerve distribution. Because the sphenopalatine ganglion is a relay center, the pain can be varied and may present with pain in the head and neck, gums, and even the teeth. Imaging modalities include MRI of the brain to rule out any possible structural abnormalities that may be impinging the neural structures.

8. What is the recommended medical management of sphenopalatine neuralgia? There is no evidence-based literature on recommended pharmacologic treatment of sphenopalatine ganglion neuralgia. The management of other trigeminal autonomic cephalalgias that resemble sphenopalatine ganglion neuralgia differs. The options for medical management of sphenopalatine neuralgia include antiepileptic medications: carbamazepine (200–1200 mg/ day) or oxcarbazepine (600–1800 mg/day). The other options include: lamotrigine, phenytoin, gabapentin, pregabalin, topiramate, and valproate. If the provider is suspecting CH, verapamil is the preferred preventive medication.

9. What are the interventional therapies for treatment of sphenopalatine neuralgia? There are many described techniques in addressing the sphenopalatine ganglion. These range from minimally invasive transnasal techniques relying on mucosal uptake to more invasive techniques with needle placement directly at the ganglion.[10] The minimally invasive techniques are commonly diagnostic. They allow identification of the ganglion as a pain generator. The transnasal approach involves the placement of a soft cotton tip applicator soaked with

an anesthetic of choice (lidocaine 4%) through the nares, along the superior border of the middle turbinate reaching the posterior wall of the nasopharynx. A second applicator can be applied superior and posterior to the first. Fluoroscopic guidance can be used for temporary, diagnostic, and permanent block of the SPG. The patient is positioned supine on the fluoroscopy table. A lateral image is taken until the pterygoid plates are superimposed over each other. The skin surface is anesthetized over the area of the pterygopalatine fossa. The entry point of the RFA needle is below the zygomatic arch, between the posterior condyle and coronoid process of the mandible. The needle is advanced until contact is made with the lateral wall of the pterygoid plate. At this time an anteroposterior view is obtained and the needle is advanced to the nasal wall. If the pterygoid plate is contacted, the needle should be redirected cephalad and anterior to slip off the bone into the fossa.[7] Sensory stimulation should reproduce paresthesias in the nasal cavity at the base of the nose. Stimulation in the soft palate or the maxilla would indicate the tip is near the maxillary nerve or near one of its branches. Contrast can be used to verify correct needle position as well as distinguish intravascular position. This is followed by injection of local anesthetic and then radiofrequency ablation. If chemical neurolysis is planned then following the administration of local anesthetic agent, neurolytic agent can be injected.

10. What are the complications of the sphenopalatine block? There are many complications that must be anticipated during sphenopalatine block. Positioning the needle creates a risk for traumatic perforation of the nasal wall and the orbit. Traumatic injury to the maxillary nerve is also a risk as is bleeding (epistaxis) and hematoma formation. There is a risk for seizure from local anesthetic uptake. Hemodynamic instability and bradycardia are also potential risks during the procedure.

Summary Sphenopalatine neuralgia is also called Sluder neuralgia, because Sluder was the first to describe this entity in 1908.[1] There have been many descriptions of patients having sphenopalatine neuralgia over the last 90 years; the treatment was directed at the

331

Chapter 46: Patient with sphenopalatine neuralgia

sphenopalatine ganglion. Sphenopalatine neuralgia presents with activation of the unilateral branch of the trigeminal nerve causing significant orbital and nasal pain, associated with unilateral activation of the parasympathetic system. Some pain experts and neurologists have argued over the years that sphenopalatine neuralgia may just be a presentation of CH;[3] however there is an opposing group of physicians who believe that it is a separate entity. They believe that it is another type of trigeminal autonomic cephalalgia, in the same group as CH, but distinct from CH. There is significant evidence in the literature that procedures performed in the sphenopalatine ganglion are not only helpful for sphenopalatine neuralgia, but any severe intractable trigeminal neuralgia,

References 1.

Sluder G. The Role of the Sphenopalatine (or Meckel’s) Ganglion in Nasal Headaches. AR Elliott Publishing Company. 1908.

2.

Sluder G. The syndrome of sphenopalatine ganglion neuralgia. Am J Med Sci. 1910;111:868–878.

3.

Ahamed SH, Jones NS. What is Sluder’s neuralgia? J Laryngol Otology. 2003;117(6):437–443.

4.

Oomen KPQ, Van Wijck AJM, Hordijk GJ, De Ru JA. Sluder’s neuralgia: a trigeminal autonomic cephalalgia? Cephalalgia. 2010;30(3):360–364.

332

especially intractable CH. Therefore we propose that if the provider is not sure if he or she is dealing with intractable sphenopalatine ganglion neuralgia versus CH, they should call this condition a probable trigeminal autonomic cephalalgia. If this severe pain is resistant to medical therapy, consider proceeding with the sphenopalatine ganglion procedures outlined above. The sphenopalatine ganglion plays a role in cephalalgias as well as facial pain. Narouze demonstrated that percutaneous radiofrequency ablation of the sphenopalatine ganglion is an extremely effective modality of treatment for patients with intractable chronic CHs.[7] Headaches and facial pain can be debilitating and therapies targeting the sphenopalatine ganglion have demonstrated effectiveness in palliation of pain.

5.

Narouze S, Vydyanathan A. Ultrasound-guided cervical facet intra-articular injection. Tech Reg Anesth Pain Manage. 2009;13(3): 133–136.

6.

Shah RV, Racz GB. Long-term relief of posttraumatic headache by sphenopalatine ganglion pulsed radiofrequency lesioning: a case report. Arch Phy Med Rehabil. 2004;85(6):1013–1016.

7.

8.

Narouze S, Kapural L, Casanova J, Mekhail N. Sphenopalatine ganglion radiofrequency ablation for the management of chronic cluster headache. Headache. 2009;49(4):571–577. Oomen KP, van Wijck AJ, Hordijk GJ, de Ru JA. Effects of

radiofrequency thermocoagulation of the sphenopalatine ganglion on headache and facial pain: correlation with diagnosis. J Orofacial Pain. 2012;26(1):59. 9.

Headache Classification Committee of the International Headache Society. The International Classification of Headache Disorders, 3rd edition (beta version). Cephalalgia. 2013; 33(9):629–808.

10. Raj PP, Lou L, Erdine S, Staats PS, Waldman SD. Radiographic Imaging for Regional Anesthesia and Pain Management. Cambridge: Cambridge University Press. 2005.

Section 5 Chapter

47

Headaches and Facial Pain

Temporomandibular joint disorders Timothy Furnish

Case study The patient, JH, is a 52-year-old male with 10 years of chronic headaches, neck pain, and right- greater than left-sided jaw pain. He complains of a waxing and waning course over the years, with pain that is worse when job related stress is elevated. He works as a social worker, and lives alone. The pain is described as aching and dull, made worse with chewing or jaw opening. He occasionally notices a clicking noise in the left temporomandibular joint. The neck pain is most problematic in the upper cervical region on the left radiating to the angle of the mandible. He reports no history of trauma to the face or neck and no history of infections or surgeries. He takes ibuprofen intermittently but without much benefit. He is on alprazolam for anxiety and has been on antidepressants in the past for depression and anxiety.

1. What is temporomandibular joint disorder (TMD) and how does it typically present? Temporomandibular disorders are a subgroup of craniofacial pain disorders involving the temporomandibular joints, the muscles of mastication, and associated head and neck musculoskeletal structures.[1] The term temporomandibular joint disorder (TMD) has become the most widely used term in the literature to characterize pain and dysfunction emanating from the musculoskeletal structures in the temporomandibular joint region. The typical presentation for TMD is pain in the area of the temporomandibular joint (TMJ) with decreased motion of the mandible and sounds emanating from the TMJ with jaw motion. Pain is often

unilateral but may be present on both sides, usually worse on one side than the other. The pain may be worse in the morning in patients who clench or grind teeth during sleep. It is common for the pain to radiate to surrounding structures such as the periorbital region, ears, or neck. Patients may or may not have a history of trauma to the joint or recent history of dental work. Pain is reproduced with palpation and jaw opening. Normal activities such as chewing, talking, or yawning often exacerbate the pain. Some patients present with decreased mouth opening, locking of the jaw, or lateral deviation of the jaw. Associated symptoms may include ear pain or fullness, tinnitus, dizziness, neck pain, and headaches.[1,2] The most widely accepted classification system for the diagnosis of TMD is the Research Diagnostic Criteria for TMD, which provides criteria for myofascial and intra-articular signs and symptoms (Table 47.1).

2. What are the relevant anatomical structures related to TMD? The TMJ connects the mandible to the skull and allows for the opening and closing of the jaw. It is a bi-condylar joint, meaning that the condyles located on either side of the mandible function in unison with opening and closing. Compared to other synovial joints in the human body the TMJ is unique in that the articular surfaces are covered in dense fibrous connective tissue instead of hyaline cartilage.[2] Separating the condyles from the articular fossa within the skull is a fibrocartilaginous disc (Figure 47.1). The disc serves to separate the bony structures, provide for smoother motion, and absorb stresses on the joint. The joint is surrounded by ligaments, which form the joint capsule.[3] Branches

Case Studies in Pain Management, ed. Alan David Kaye and Rinoo V. Shah. Published by Cambridge University Press. © Cambridge University Press 2015.

333

Chapter 47: Temporomandibular joint disorders

Table 47.1. RDC/TMD Axis I: Physical Dx

Group I: Muscle Disorders

Ia: Myofascial pain  

Pain in jaw, face, temples at rest or activity Pain with palpation of  3 muscles/sites on either side

Ib: Myofascial pain with limited jaw opening    Group II: Disc Displacements

Myofascial pain as defined in Ia Pain-free unassisted mandibular opening < 40 mm Maximum assisted opening ≤ 5 mm greater than pain-free opening

IIa: Disc displacement with reduction  

Clicking in TMJ on opening and closing; eliminated on protrusive opening; reproducible on 2 out of 3 trials; OR Clicking in TMJ on both vertical range of motion and lateral excursion or protrusion; reproducible on 2 out of 3 trials

IIb: Disc displacement without reduction with limited opening     

History of significant limitation in opening Maximum unassisted opening ≤ 35 mm Passive stretch increases opening by ≤ 4 mm over max unassisted opening Contralateral excursion of 7 mm and/or uncorrected deviation to ipsilateral side on opening Absence of joint sound or presence of joint sound not meeting criteria in IIa

IIc: Disc displacement without reduction, without limited opening History of significant limitation of mandibular opening Maximum unassisted opening > 35 mm Passive stretch increases opening by  5 mm over unassisted opening Contralateral excursion  7 mm Presence of joint sounds not meeting criteria in IIa Imaging reveals disc displacement without reduction Group III: Arthralgia, Osteoarthritis, Osteoarthrosis

IIIa: Arthralgia Pain on one or both joint sites with palpation One or more of the following self-reports of pain: pain in region of joint, pain in joint with maximum unassisted opening, pain during assisted opening, pain in joint during lateral excursion Course crepitus is absent IIIb: Osteoarthritis of the TMJ Arthralgia as defined in IIIa is present Either course crepitus or radiologic signs of arthrosis are present IIIc: Osteoarthrosis of the TMJ Absence of all signs of arthralgia Either course crepitus or radiologic signs of arthrosis are present

of the auriculotemporal and masseteric branches of the mandibular nerve innervate the temporomandibular joints.[4] At rest the disc sits between the condyle and the mandibular fossa. During jaw opening the condyle rotates within the fossa then the mandible translates anteriorly and the condyle slides forward out of the mandibular fossa onto the articular eminence

334

of the maxilla. The disc moves posteriorly with rotation then slides anteriorly with the condyle during translation. The reverse takes place upon jaw closing.[4] The joint’s function is to allow for mastication and speech. The primary muscles that act on the joint to elevate the mandible and close the mouth are the temporalis muscle and the masseter muscle. The lateral

Chapter 47: Temporomandibular joint disorders

Figure 47.1. Temporomandibular joint.

pterygoid muscles are the primary jaw opening muscles and produce the forward translation of the condyle out of the mandibular fossa. Other muscles acting on the joint include the suprahyoid digastric muscles.[1] The joint has complex mechanics, which allow for motion in multiple axes. The primary motion is a rotational hinge movement to open and close the jaw in conjunction with forward translation or gliding of the condyle out of the mandibular fossa. Both motions are required for complete jaw opening. The joint also allows for medio-lateral translation and posterior translation.[3] A potentially predisposing factor in TMD is joint hypermobility.

3. What is the cause of pain in TMDs? Unilateral face pain in the region of the temporomandibular joint (TMJ) was originally attributed to malposition of the joint within the glenoid fossa due to a loss of posterior teeth. In the 1930s the otolaryngologist James Costen described pain from the TMJ in patients with missing mandibular teeth that remitted with correction of their mandibular over closure after fitting them with dentures or partial prosthetic teeth.[1] It was believed that occlusal disruptions between mandibular and maxillary teeth resulted in hyperactivity and spasm of muscles. Missing teeth,

abnormal tooth eruption, malocclusion, or dental caries may be a cause of TMD. However, studies have not shown a clear association between occlusal factors and TMD onset or severity. The most widely accepted theory now holds that the cause of pain is multifactorial and will involve biological, behavioral, social, emotional, and environmental factors alone or in combination. There is substantial evidence that patients with TMD exhibit more depression and anxiety than asymptomatic controls.[2] It has been proposed that psychologic stress related parafunctional activities may cause muscle fatigue, spasm, and pain.[5] Levels of distress or depression have also been shown to predict an increased likelihood of seeking treatment for TMD and a poorer prognosis.[6] However, the association between anxiety, depression, and TMD does not necessarily indicate causation and could instead be an effect of dealing with chronic pain.[7] Bruxism, excessive jaw clenching, may lead to muscle ischemia and pain. This may be an initiating or perpetuating factor for TMD. Other parafunctional activities have been associated with TMD such as teeth clenching and grinding. However, bruxism is quite common and the majority of people who grind teeth either awake or asleep do not develop TMD.[5,8] Myofascial etiologies for TMJ pain have focused on the muscles of mastication and parafunctional activities that result in fatigue of the muscles. The majority of patients with TMD have tenderness in the muscles of mastication with low prevalence of radiologic joint pathology, which may indicate a myofascial etiology.[2] As with other myofascial pain disorders, the overuse of muscles resulting in fatigue and muscle tension may result in weakness, stiffness, spasm, and pain. Some patients with TMJ muscle pain have a history of trauma, which may be a factor in initiation of TMD pain. This may be macrotrauma such as head injury, excessive jaw opening, or long dental procedures. It could also be microtrauma from occlusal discrepancies or bruxism. How these injuries may result in TMD remains unclear but could involve strain to muscles or ligaments, capsulitis, or muscle spasms.[2] Whiplash injury and endotracheal intubation have also been associated with TMD in some studies.[7,9] Disc displacement is a potential cause of TMD and the most common arthropathy associated with TMD. In a TMJ disc displacement the disc is forced out of its normal position resulting in bone on bone contact

335

Chapter 47: Temporomandibular joint disorders

between the condyle and articular fossa. There may be a popping sound as the disc is forced out of normal alignment with mouth opening and another popping sound with mouth closing as the disc moves back into alignment. Lastly, there may be intra-articular degeneration. This may take the form of an inflammatory arthritis such as ankylosing spondylitis, rheumatoid arthritis, infection, or gout. In juvenile rheumatoid arthritis up to 50% of patients will have TMJ involvement with jaw pain and reduction in mouth opening. For adults with rheumatoid arthritis TMJ involvement is usually late in the course of the disease. Non-inflammatory osteoarthritis may also occur as a consequence of excessive wear, trauma, or previous joint surgery. This is not usually associated with generalized osteoarthritis in other joints.[1]

4. How common is TMD? There are many symptoms associated with TMD including: pain in the vicinity of the joint; dysfunctional motion of the mandible; sounds emanating from the joint with movement; and regional pain in the face, periorbital area, jaw, or neck. The presence of one or more of these symptoms in the general population is relatively common, ranging from 6% to 93%.[1,2,10] The majority of these, however, do not require treatment. Symptoms of a severity that result in seeking treatment occur in between 3.6 and 7% of the population.[2] The diagnosis of TMJ most commonly occurs in young to middle aged patients in their 20s–50s. Women are more commonly affected than men with reported ratios from 3:1 to 9:1.[1,11] Among TMD subtypes muscle disorders are the most common with disc displacement second most common. Inflammatory degenerative disorders are diagnosed in approximately a third of patients.[11]

5. What would be included in the differential diagnosis? Headaches, dental conditions, and neuropathic facial pain syndromes will top the list of the differential diagnoses. Potentially similar presentation may occur with: 1. Migraine 2. Tension headache

336

3. Primary or metastatic tumors to the mandible 4. Intracranial tumors 5. Trigeminal neuralgia 6. Temporal arteritis 7. Congenital deformity 8. Dental carries 9. Myocardial infarction 10. Salivary gland disorders 11. Primary or secondary headache disorders 12. Maxillary sinusitis 13. Ear infection 14. Cervical spondylosis The location and onset of pain in conjunction with mandibular function, physical examination for painful muscles or TMJ, and the presence or absence of joint sounds will contribute significantly to narrowing of the differential.

6. How is TMD evaluated and diagnosed? The diagnosis of TMD is made largely based on the clinical history and a careful examination. Imaging studies may be of use in evaluating for disc displacement or to rule out other pathologies such as osteoarthritis, rheumatoid arthritis, tumors, or dental carries as a cause of pain. Plain film studies should include an anteroposterior view for condylar shape and a lateral view with jaw open and closed to see condylar movement and joint space. Magnetic resonance imaging can be used to assess the disc or evaluate for other soft tissue pathology or infection. Similarly laboratory studies are useful only to rule out other potential causes of pain such as infection, rheumatologic conditions, or myocardial infarction. In 1992 the Research Diagnostic Criteria for temporomandibular joint disorders (RDC/TMD) was introduced to standardize diagnostic criteria in order to improve the consistency between studies of TMD.[11] Primarily developed as a research tool, the RCD/TMD is the only comprehensive diagnostic scheme for TMD. The RCD/TMD is broken down into axis I criteria for physical diagnosis and axis II criteria for psychosocial assessment. The axis I criteria divide the physical diagnoses into three categories: (1) muscle disorders; (2) disc displacements; and (3) inflammatory disorders: arthralgia, osteoarthritis, and osteoarthrosis (Table 47.1).[11,12]

Chapter 47: Temporomandibular joint disorders

In evaluating patients with suspected TMD the examiner should observe for any asymmetry of the face in both a horizontal and vertical plane. The head and neck should be observed at resting position for any abnormal extension of the head or deviation of the neck suggesting cervical spine pathology or dystonia. Evaluation of the oral cavity should look for missing or malformed teeth, dental caries, or malocclusion with underbite, overbite, or crossbite (teeth of the mandible medial or lateral to the teeth of the maxilla on closing).[4] In TMD the palpation of masticatory muscles and TMJ should reproduce the patient’s pain. Palpation of the joint is done with the examiner’s finder over the mandibular condyles. The examiner must also palpate the temporalis, masseter, and medial and lateral pterygoid muscles for tenderness, hypertrophy, or spasm. The presence of an intra-articular disc displacement will result in clicking sounds with opening and closing and if the disc is not reducing may result in jaw locking. If clicking sounds are present have the patient open and close the jaw with the jaw protruded and then retruded. If opening with protrusion eliminates the clicking and retrusion accentuates the clicking this may suggest an anterior disc displacement with reduction. Degenerative disease of the joint may present with crepitus. Patients can also present with limitations in jaw opening, which is defined as < 40 mm inter-incisor distance with assisted opening.[4,13]

7. What are the potential treatments for TMD? The goals of therapy for TMD are to reduce pain and increase function and quality of life. In the majority of cases TMD can be treated with non-invasive therapies.[1] The multifactorial etiology for TMD pain has resulted in a wide variety of treatment approaches including physical therapies, psychological, pharmacologic, and surgical treatments. Dentists are often the first to see and treat TMD. One of the most common forms of therapy employed by dentists is the use of oral appliances or splints.[13] This treatment extends from the theory that bruxism is a significant cause of pain and dysfunction. Stabilization splints are custom fitted to the patient’s teeth and worn at night to prevent grinding. These splints are made of a semi-pliable plastic that cover the upper and/or lower teeth to correct malocclusion and lessen the effects of grinding or clenching.[3] (Figure 47.2).

Figure 47.2. Temporomandibular disorder splint. Courtesy of Stephen Ward DDS.

There is short-term evidence for the use of these splints to reduce TMD related pain but limited longterm evidence.[10] Missing teeth should be replaced in order to correct malocclusion that could be contributing to TMD. Physical therapies for TMD have included acupuncture, manual therapy, electrotherapy, heat, and jaw exercises. Studies of these techniques while small and non-rigorous have tended to show benefit over no treatment. Acupuncture has been found superior to no therapy or other conservative treatment in several studies.[10] Jaw exercises are another common therapy and may involve stretching and strengthening exercises.(3) Jaw exercises have shown benefit when compared to controls in several studies.[10] Education of the patient to avoid extreme jaw movements and eat softer foods may improve pain or limit worsening of the condition. The use of moist heat or cold packs, active exercises, and manual manipulations may improve mouth-opening distance in patients with disc displacement or arthritis. Programs of biofeedback, relaxation techniques, and proprioceptive training have shown benefit over placebo and occlusal splint therapies.[14] Little evidence exists for the use of TENS, laser therapies, or ultrasound for the relief of TMD pain.[10] As with many other chronic pain conditions, psychotherapy has been shown to improve pain and function. This is especially true when significant anxiety or depression is a component of the presentation. Studies of psychologic therapies for TMD pain have included biofeedback and CBT. As with many

337

Chapter 47: Temporomandibular joint disorders

modalities of TMD pain treatment, the studies of CBT and biofeedback suffer from poor design, bias risk, and small study size. However, a systematic review of these studies found evidence for the use of CBT with or without biofeedback for reducing pain, depression, and the interference of TMD symptoms with activity.[15] Pharmacologic therapies have included oral agents and injectable drugs. Unfortunately there are few well-designed studies. A recent Cochrane review found 11 poor-quality studies and determined that there was inconclusive evidence for analgesics, benzodiazepines, anticonvulsants, and other agents. Despite this lack of evidence, the short-term use of nonsteroidal anti-inflammatory agents and muscle relaxants has been recommended for acute flairs.[1] Antidepressants from the tricyclic and serotoninnorepinephrine reuptake classes have been used successfully in other chronic musculoskeletal pain conditions and have been employed in the treatment of TMD. There have been several case series reporting benefit with chemodenervation of masseter and temporalis muscles with botulinum toxin. These studies report improvement in mouth opening and pain relief that occurs after the development of botulinuminduced reduced muscle strength.[16] As with larger joints such as knees and hips, there have been studies looking at the intra-articular injection of steroids or hyaluronic acid. These studies have shown pain reduction for both but little difference in effect between the steroids and hyaluronic acid.[17] Surgery for TMJ pain is reserved for those patients who are refractory to more conservative therapies. Most studies have involved the use of arthroscopy, arthrocentesis, discectomy, and joint replacement. Studies of arthrocentesis, arthroscopy, and discectomy have generally had similar results for disc displacement with reportedly high success rates but these studies lacked rigorous study designs.[10] Arthrocentesis is the least invasive of the surgical techniques. A needle is inserted into the joint and flushed with sterile solution with the goal of reducing or dislodging a stuck disc.[13] Arthroscopy of the TMJ allows for direct visualization of the joint space and disc via a surgically placed athroscope. A displaced disc may be reduced and disc fragments or inflamed tissue removed.[13]

338

Anchors can be placed to hold a chronically dislocated disc in the proper position. Discectomy is reserved for severe cases of TMD with a dislocated and damaged intra-articular disc. The removal of the disc may result in increased loading and wear on the condyles and articular fossa due to lost absorbency of the disc.[13] Some surgeons have used tendon allografts as a disc replacement material to protect the articular cartilage from further degeneration after discectomy. Joint replacement surgery is the most invasive form of TMJ surgical treatment. In TMJ arthroplasty a severely damaged joint is removed and replaced with a prosthetic condyle and/or fossa. This therapy is generally reserved for severe ankylosis, degenerative disease, aseptic condyle necrosis, rheumatoid arthritis, tumor, or traumatic damage to the joint structures.[3] The maximum life expectancy of TMJ prostheses is 10–15 years. Many early prosthetic devices failed prematurely and in the early 1990s the FDA halted manufacture of TMJ prosthetic devices due to lack of safety and efficacy data. Since the late 1990s four implants have been approved but are categorized as high-risk medical devices by the FDA.[3]

Conclusion Temporomandibular disorders are a frequent cause of facial pain and dysfunction. The diagnosis and management of TMD is complicated by the multifactorial etiology of TMD. Various disciplines have had a role in treating pain that originates from or around the TMJs including dentists, oral surgeons, physical therapiests, otolaryngologists, and pain specialists. This has resulted in a wide variety of therapies of varying invasiveness but unfortunately few large, well-controlled trials. The lack of good evidence for the treatments available has resulted in limited guidance for treating clinicians. Therapies such as physical therapy, education, avoidance of excessive jaw opening and hard foods, and psychosocial treatments such as biofeedback and CBT may lack grade I evidence but are the least invasive options and should be considered as first line. Oral appliances such as splints are also of limited invasiveness and worth early consideration.

Chapter 47: Temporomandibular joint disorders

References 1.

2.

3.

Scrivani SJ, Keith DA, Kaban LB. Temporomandibular disorders. N Engl J Med. 2008;359: 2693–2705. Suvinen TI, Reade PC, Kemppainen, Kononen M, Dworkin SF. Review of aetiological concepts of temporomandibular pain disorders: toward a biopsychosocial model for integration of physical disorder factors with psychological and psychosocial illness impact factors. Eur J Pain. 2005;9: 613–633. Ingawale S, Goswami T. Temporomandibular joint: disorders, treatments, and biomechanics. Ann Biomed Eng. 2009;37:976–996.

4.

Magee DJ. Orthopedic Physical Assessment, 5th ed. St. Louis: Saunders Elsevier. 2008.

5.

Benoliel R, Svensson P, Heir GM, et al. Persistent orofacial muscle pain. Oral Dis. 2011;17:23–41.

6.

Epker J, Gatchel RJ. Prediction of treatment-seeking behavior in acute TMD patients: practical

application in clinical settings. J Orofac Pain. 2000;14:303–309. 7.

Oral K, Kucuk BB, Ebeogul B, Dincer S. Etiology of temporomandibular disorder pain. Agri. 2009;21:89–94. 8. Jerolimov V. Temporomandibular disorders and orofascial pain. Rad 504 Medical Sciences. 2009;33: 53–77. 9. Klobas L, Tegelberg A, Axelsson S. Symptoms and signs of temporomandibular disorders in individuals with chronic whiplash-associated disorders. Swed Dent J. 2004;28:29–36. 10. List T, Axelsson S. Management of TMD: evidence from systematic reviews and meta-analyses. J Oral Rehab. 2010;37:430–451. 11. Manfredini D, Guarda-Nardini L, Winocur E, et al. Research diagnostic criteria for temporomandibular disorders: a systematic review of axis I epidemiologic findings. Oral Surg Oral Med Oral Pathol Oral Radiol Endod. 2011;112:453–463. 12. Dworkin SF, Leresche L. Research diagnostic criteria for temporomandibular disorders: review, criteria, examinations and

specifications, critique. J Craniomandib Disord Facial Oral Pain. 1992;6:301–355. 13. Zakrzewska JM. Differential diagnosis of facial pain and guidelines for management. Br J Anesth. 2013;111:95–104. 14. Medlicott MS, Harris SR. A systematic review of the effectiveness of exercise, manual therapy, electrotherapy, relaxation training, and biofeedback in the management of temporomandibular disorder. Phys Ther. 2006;86:955–973. 15. Aggarwal VR, Lovell K, Peters S, et al. Psychosocial interventions for the management of chronic orofacial pain. Cochrane Database Syst Rev. 2011;11:CD008456. 16. Majid OW. Clinical use of botulinum toxins in oral and maxillofacial surgery. Int J Oral Maxillofac Surg. 2010;39:197–207. 17. De Souza RF, Lovato da Silva CH, Nasser M, Fedorowicz Z, AlMuharraqi MA. Interventions for the management of temporomandibular joint osteoarthritis. Cochrane Database Syst Rev. 2012;4:CD007261.

339

Section 6 Chapter

48

Cancer Pain

Cancer pain Paul A. Sloan

Case study A 65-year-old woman presents to her family physician with abdominal discomfort. Work-up quickly reveals a diagnosis of cancer of the pancreas with local invasion of the upper lumbar spine and destruction of the L1 vertebral body (Figure 48.1). Her previous medical history is significant only for hypertension and hyperlipidemia. She reports her pain as originating in the epigastrium and radiating through to the back. It is described as aching in character, constant in nature, and rated at 6/10 on a Numeric Rating Scale. Her only analgesic to date has been acetaminophen 500 mg 4 times daily. She notes some fatigue on exertion and a slight decrease in her appetite. A review of systems is negative for nausea, vomiting, or constipation.

1. What is the etiology of cancer pain? Our patient has a solid tumor that can directly invade the tissues and organs of the body, causing pain. The invasion posteriorly of the cancer into the spine, with destruction of the lumbar vertebral body, is causing the constant cancer pain. The exact physiology of bone pain is not fully understood, but may involve proliferation of osteoclast cells by tumor-induced release of acids, or excitation of nociceptors by prostaglandins and other factors released by the tumor stroma.[1] In addition, the cancer cells in bone may stimulate sensory neuron sprouting within the bone marrow resulting in painful sensation transmission.[1] Pain that is caused by direct invasion of organ and tissue is usually classified as nociceptive pain. Cancers may also directly invade nerves and nerve plexuses resulting in pain, classified as neuropathic pain. Invasion of the celiac plexus by the pancreatic cancer

Figure 48.1 Direct invasion of the lumbar spine by pancreatic cancer causing destruction of the vertebral body of L1. In spite of this significant spine disease, the patient had no lower extremity neurological symptoms.

Case Studies in Pain Management, ed. Alan David Kaye and Rinoo V. Shah. Published by Cambridge University Press. © Cambridge University Press 2015.

341

Chapter 48: Cancer pain

produces a vague abdominal visceral pain, while invasion of a brachial plexus from apical lung cancer will produce neuropathic pain of the upper extremity.[2] This pain related to direct invasion of nerve tissue is often described in terms such as burning, tingling, electric, or shooting. It may be somewhat more resistant to management with standard opioid and nonopioid analgesic therapies. In addition to pain directly related to the cancer, patients may experience acute and chronic pain following cancer treatments. For example, various chronic pain syndromes may occur after cancer surgery with an incidence up to 50%.[3] Radiation treatment may lead to painful skin conditions or painful neuritis. Chemotherapy may lead to painful peripheral neuropathy that can be challenging to control.

2. Could there be other non-cancer causes for the patient’s pain? Patients with a history of cancer may present with acute or chronic pain from non-cancer sources. Chronic low back pain from degenerative disc disease or facet arthropathy, chronic neck pain, and headaches from non-cancer etiologies (e.g., migraine) may all present in the cancer patient. Postherpetic neuralgia is more common in patients with cancer, presumably from suppression of the immune system. The physician must use the patient history, physical examination, and relevant laboratory examinations to diagnose and treat these conditions. For our patient, with no previous history of chronic pain, the pain location, radiation, duration, intensity, and imaging study all highly suggest pain from cancer invasion.

3. What is the epidemiology of cancer pain? Pain is a feared symptom among patients diagnosed with cancer because it occurs so frequently, and often on a chronic basis. When cancer is first diagnosed, pain will be a presenting symptom in 20–50% of patients.[4] The prevalence of cancer pain will vary with tumor type, being most common for head/neck cancer, followed by gynecological, gastrointestinal, and lung cancers.[5] In the advanced stages of disease, the prevalence of cancer pain increases further. A recent review showed 75% of cancer patients with

342

advanced and untreatable disease as having chronic pain.[6] Interestingly, hematologic cancers (excluding non-Hodgkin’s lymphoma), once thought to produce little pain, were predictive for the prevalence of pain and may be caused by the cancer itself, disease-related complications, or iatrogenic causes.[7] With modern cancer therapies, many patients are living with stable disease or having an outright cure from cancer. Unfortunately, approximately half of these patients still suffer with chronic pain. For patients with no evidence of active cancer for at least 6 months, the prevalence of moderate–severe chronic pain was still 41–49% in a recent European study.[6] The etiology of the chronic pain may be related to cancer therapy, residual tissue damage from the cancer infiltration, neuropathic pain from nerve injury, musculoskeletal pain from inactivity, or unrelated causes. Of concern to patients is the reality that they may be free of cancer, yet not free, or possibly ever free, of chronic pain. For example, a cohort of colorectal cancer survivors at least 5 years free of disease reported pain in 27% of patients.[4]

4. Are there any risk factors for the development of cancer pain? There are risk factors for cancer pain but as yet, we are unable to change or modify most of these issues. For example, the younger age group is associated with more prevalence of cancer pain, and of a more severe nature. The previous section discussed that the prevalence of cancer pain varies with the specific type of tumor. The patient is obviously unable to change these factors. In addition, genetic factors can influence both the ability to sense pain, as well as the analgesic effect of opioids. Recent studies have demonstrated that genetic differences in interleukin-8 correlated strongly as a predictor of severe pain in patients with lung cancer, and that polymorphism in interleukin-1 receptor antagonist gene is associated with variations in opioid consumption for postoperative pain.[8,9] It appears that there are genetic variations in the morphine receptor that may make morphine therapy less effective in certain patients, and may result in some patients being more sensitive to painful stimuli.[10] To date, the technology is insufficient to easily identify or modify these genetic traits, but in the near future we may be able to sequence a patient’s genetic makeup, identify the best opioid based on their genetic pattern, and prescribe

Chapter 48: Cancer pain

analgesic therapy tailored for the best possible outcome with the least side effect profile.[11] There are at least two risk factors for chronic pain that may be possible to modify. Studies on patients with postoperative pain and chronic pain following surgery show that preoperative emotional distress is a factor associated with postoperative pain, and that untreated severe postoperative pain is a risk factor associated with postsurgical chronic pain.[12] Since many cancer patients develop chronic pain following therapy, it is feasible that pre-therapy reduction in emotional distress and better relief of pain during and immediately after cancer therapies may lead to a reduction in post-therapy chronic pain.

5. What analgesic treatments are available to relieve this cancer pain? The first and most important therapeutic message is for the pain specialist to assure the patient, who is likely anxious and scared, that he/she will use every available option to address and treat the cancer pain, and that the patient will always be believed and never abandoned by the treating physician. This information is very important at the start of the patient–pain physician relationship because of the widely publicized difficulties with chronic pain patients and prescription opioid therapies. The patient may well fear, in today’s climate, that she may be accused of misuse of opioid medications or discharged from a pain clinic in the future. Assurances by the pain specialist up front that pain relief will be a top priority of her care will relieve many anxieties by the patient and assist in the therapeutic relationship. The full spectrum of analgesic treatments has been used in the treatment of cancer pain, typically starting from the least powerful and progressing to the most strong analgesic medications or invasive therapies. The categories of options available are: (1) modify the pathology of the pain generators, through the use of radiotherapy, chemotherapy, immunotherapy, etc.; (2) elevate the pain threshold with analgesic medications such as NSAIDs, acetaminophen, opioids, antiseizure drugs, adjuvant analgesics; and (3) interrupt pain pathways through surgical or anesthetic techniques. Pain relief through radiotherapy/ chemotherapy has an accepted role in the management of cancer pain, but is beyond the scope of this chapter. Analgesic medications will be briefly reviewed, focusing on the use of opioids for cancer

pain. Common anesthetic neurolytic procedures for cancer pain relief will be presented.

6. What is the World Health Organization Analgesic Ladder for the treatment of cancer pain? Traditionally, non-opioid analgesics such as NSAIDs and acetaminophen have been used as a first-line therapy, followed by the addition/substitution of opioid analgesics. This is the brief outline of the World Health Organization Analgesic Ladder for the management of cancer pain.[13] Step 1 for mild pain uses NSAIDs and acetaminophen, Step 2 for moderate pain adds “weak” opioids (such as hydrocodone) to the NSAID base, and Step 3 for moderate–severe pain substitutes “strong” opioids (such as morphine, methadone, transdermal fentanyl) along with the NSAID base. In addition, adjuvant analgesics (such as tricyclic antidepressants, antiseizure drugs, corticosteroids) may be added to existing analgesics at any step of the ladder. In spite of a long history of accepted medical dogma, the WHO Ladder has been questioned in recent years. The use of Step 1 has been questioned (see below), and the advantage of using a weak opioid for Step 2 has been challenged by some and suggested that lower doses of strong opioids (such as morphine) would suffice for moderate pain.[14] Tramadol, with both opioid and non-opioid (inhibition of noradrenaline and serotonin reuptake) mechanisms of analgesia, has been suggested as an appropriate analgesic for Step 2.[15] In addition to the standard 3-step ladder, Step 4 has been proposed for severe cancer pain refractory to the first three steps of medications, and consists of the use of neurolytic blocks, spinal analgesics, and other invasive analgesic therapies.[16]

7. What analgesic medications should I use to treat this cancer pain? Non-steroidal anti-inflammatory drugs and acetaminophen The use of NSAIDs in the management of cancer pain has been found to be superior to placebo, but with no difference in side effects.[17] Given that they provide analgesia for typically mild pain only, a recent review of the literature advised that there is no proof that NSAIDs or acetaminophen should be used to start

343

Chapter 48: Cancer pain

analgesic therapy for cancer pain.[15] The authors also noted that there is no data available to guide the physician in the continuation of NSAIDs when a patient requires opioid analgesic therapy as well.[18] It is suggested that the analgesic effect of NSAIDs in combination with opioid must be determined on an individual basis. Since NSAIDs are associated with potentially serious side effects such as nausea, vomiting, diarrhea, constipation, headache, renal dysfunction, gastric ulcers, and platelet inhibition, this author believes there is insufficient evidence to recommend their routine use in the management of anything other than very mild cancer pain. If the physician chooses to start with a non-steroidal medication, there is no evidence to support the safety or efficacy of one NSAID over another.[17] Since our patient has rated her pain as moderate, therapy with an opioid analgesic is required.

Adjuvant analgesics Adjuvant analgesics are medications whose primary indication for use was something other than analgesia. For example, tricyclic antidepressants have a known analgesic effect outside of their antidepressant effect and show efficacy in the management of some chronic painful conditions.[19] Other adjuvant analgesics are listed in Table 48.1 and vary in both efficacy, safety, and side effect profile. Of the medications listed, gabapentin, pregabalin, and the tricyclic antidepressants are the most commonly used for pain management. This review does not permit a full discussion of these valuable drugs and the reader is referred to specific review articles for further information.[20]

Opioids Opioids are morphine-like drugs that bind with mu-, delta-, and kappa-opioid receptors (mainly on presynaptic nociceptive membranes) in the central nervous system to produce analgesia through decreased neuronal neurotransmitter release and decreased nociceptive impulse propagation.[21] They have been used to treat chronic cancer pain for the last four decades and found to be effective over months to years, often in an easy oral formulation.[22,23] Multiple studies have demonstrated that opioid analgesics when taken regularly for moderate or severe cancer pain are effective, with tolerable side effects, for approximately 85–90% of patients.[2] For the remaining 10–15% of patients

344

Table 48.1. Adjuvant analgesics used in the management of cancer pain

Medication class

Examples

Anticonvulsant

Gabapentin, pregabalin, carbamazepine

Antidepressant

Amitriptyline, desipramine, duloxetine, venlafaxine

Alpha-2-agonist

Clonidine, dexmedetomidine

Corticosteroids

Dexamethasone, prednisone

Local anesthetics

IV lidocaine, PO mexiletine

NMDA receptor antagonist

Ketamine, dextromethorphan

For bony metastasis

Bisphosphonates, calcitonin

Topical analgesic

Menthol, capsaicin cream, lidocaine transdermal

Muscle relaxant

Cyclobenzaprine, methocarbamol

Psychostimulant

Methylphenidate

with cancer pain that is difficult to control, interventional techniques are often used (see below). Opioids are the mainstay of the pharmacologic treatment of cancer pain, and morphine is the prototypical drug. Population studies reveal that all opioids are equally efficacious in the management of cancer pain; however there are significant interindividual differences such that an individual patient may achieve better analgesia (with fewer side effects) from one specific opioid over another. Individual genetic makeup is responsible for at least part of this phenomenon and requires that each physician titrate opioid analgesics carefully to the individual patient, with the expectation that rotation to a different opioid will often be necessary.[11,24] Principles of opioid therapy in the management of chronic cancer pain (Table 48.2) require that the medication be given on a regular basis, by the clock and not as necessary, in order to help reduce and prevent the recurrence of pain. The goal of therapy is the complete elimination of pain (a pain score of zero); however multiple published clinical opioid studies show that patients often titrate their opioid analgesics to a baseline of 1–3/10 on a numeric pain rating scale.[23,25–28] The reasons for this are not well studied, but may relate to patients titrating analgesia with opioid-related side effects.

Chapter 48: Cancer pain

Table 48.2. Principles of opioid therapy in the treatment of chronic cancer pain

1. Opioids should be given on a regular, timecontingent (not PRN) basis 2. Titrate the opioid to effect (analgesia) or intolerable side effect 3. Provide PRN rescue opioid doses in addition to the regularly scheduled opioid 4. Titrate the dose on an individual basis 5. Recognize that there is, in general, no ceiling effect and no maximum opioid daily dose 6. Treat breakthrough episodes of cancer pain 7. Anticipate and treat opioid-related side effects 8. Use simple oral or transdermal formulations if possible 9. Consider adjuvant analgesics as necessary 10. Consider interventional techniques for refractory cancer pain 11. Use analgesics in the context of whole person care

After initiation of opioid therapy (using the principles contained in Table 48.2), reevaluation of the patient will be required on a regular basis. With the initial use of opioids, daily (or more frequent) evaluation will be needed in order to titrate the opioid dose to effect or intolerable side effect. If many PRN opioids are required to achieve adequate analgesia, then the scheduled daily opioid dose will be increased as necessary. Many specialists use a PRN dose for breakthrough pain of 15% of the required, scheduled daily opioid dose. There is no upper limit to the daily opioid dose and each individual patient is prescribed a dose of opioids that achieves analgesia with acceptable side effect profile. If side effects occur, they should be treated rapidly so as not to hinder the upward titration of opioid to analgesia. If analgesia cannot be obtained without intolerable side effects from the initial opioid chosen, then rotation to a different opioid has been recommended, although without adequately controlled trials or conclusive evidence of the practice.[24] The oral route of administration, where possible, has been the traditional preferred route as it is simple, well tolerated, and often more inexpensive. Historical teaching has recommended the use of immediaterelease (IR) opioid formulations for initiation and dose titration of analgesics, with switching to an extended-release opioid formulation after achieving a stable opioid daily dose. More recent studies, however, suggest that using an extended-release (ER)

opioid formulation for initial dose titration is an acceptable practice.[29] There is little evidence that the ER formulations result in decreased side effects or better analgesia, yet they provide patient comfort with less focus on “taking the pill on time” and patient satisfaction.[21] The transdermal route of opioid administration is also well tolerated by the patient with transdermal fentanyl and transdermal buprenorphine available for pain management. At least 20 different opioids are available for the management of cancer pain. Morphine has been most studied and remains the prototype of a pure mu agonist.[21] Other commonly used opioids include hydromorphone, oxycodone, oxymorphone, methadone, and fentanyl. The clinician should avoid several opioids: (1) codeine (a prodrug for morphine, with variable metabolism to morphine resulting in overdose for some patients and inadequate analgesia for others), (2) meperidine (the metabolite normeperidine can cause seizures), and (3) remifentanil (ultrashort acting unless given by constant intravenous infusion). The most commonly used opioids will be briefly reviewed. Morphine is available as an IR or ER formulation with a long history of efficacy and safety. Immediate-release morphine should be given every 3–4 hours, while ER preparations can be dosed every 12 or 24 hours. Morphine is metabolized after oral ingestion in the liver, yet chronic dosing results in adequate plasma levels for the maintenance of analgesia.[23] Morphine, as with most opioids, should be used cautiously in patients with extensive liver or renal disease.[21] Starting doses for opioid-naive patients may be 30 mg daily, while there is no maximum daily dose apart from intolerable side effects. Hydromorphone is very similar to morphine and also is metabolized in the liver. The IR formulation should be given every 3–4 hours to maintain analgesia, while 12- and 24-hour ER preparations are available. Hydromorphone is approximately five times as potent as morphine and should be dosed accordingly. Oxycodone is well absorbed (bioavailability of 60–87%) and acts at both the mu and kappa-opioid receptors to produce analgesia. Oxycodone is metabolized in the liver by the cytochrome P450 enzyme system with a consequence that common medications may increase or decrease its metabolism.[21] The IR products are also dosed every 3–4 hours, with ER 12-hour preparations available. Oxymorphone has been used in a parenteral

345

Chapter 48: Cancer pain

formulation for many decades and is now also available in both IR and ER oral products. It is metabolized, like morphine, in the liver by glucuronidation. Although it has a very low oral bioavailability (10%) it provides cancer pain relief when given on a chronic basis.[21,28] Fentanyl has also been used for anesthesia in an intravenous format for many decades and formulated to a transdermal preparation 20 years ago.[27] The patch allows for a controlled rate of delivery of fentanyl to the microcirculation of the skin which is then directed to the central circulation. After initial patch placement, the peak plasma levels occur in 18 hours. Most patients find the convenient dosing at every 3 days to provide excellent cancer pain relief, in addition to being well tolerated.[27,30,31] Methadone is an inexpensive oral/intravenous opioid with several mechanisms of analgesic activity including mu-opioid receptor agonist, NMDA receptor antagonist, and serotonin reuptake inhibitor. It is well absorbed from the gastrointestinal tract (bioavailability of 60%); however the elimination half-life is very long (36 hours) and can be as much as 72 hours in the elderly.[32] Thus very cautious dose escalation must occur when titrating to pain relief and the clinician should wait 4 half lives (1 week) between dose escalations.

8. What if our patient has intermittent pain in spite of regular opioid therapy? Breakthrough pain is, unfortunately, a common condition of acute onset, short-lived and often intense pain that “breaks through” the baseline analgesia established by the regularly scheduled opioid therapy.[33] It is common in patients with cancer pain (incidence reported between 16% and 95%) and may be due to unknown causes, related to a particular activity (such as standing or walking) or movement (bending), or occur at the end of the scheduled opioid (end-of-dose failure).[33] A good pain history and physical examination should reveal which category the patient will fall under. End-of-dose failure is easily treated with an increase in medication dose, or a decrease in the dosing interval. If the breakthrough pain can be related to an activity, then pre-activity treatment with a short-acting opioid should be helpful. When the etiology cannot be determined, the patient should be given short-acting opioid medications to be used as necessary. The oral route is often chosen, but equally useful are short-acting opioids

346

delivered via sublingual, buccal, intranasal, or intravenous routes of administration.[34–36]

9. My patient is unable to take oral medications. What should I now do? Oral tablets, capsules, or liquids are a simple and effective route for opioid therapy; however many other routes of administration exist for the patient unable/unwilling to take oral analgesics. These routes include transdermal, sublingual, rectal, buccal, oral transmucosal, intranasal, intravenous, subcutaneous, epidural, intrathecal, and intracerebroventricular (ICV). The most common substitute is transdermal (fentanyl or buprenorphine), and often intravenous infusion. If the cancer pain is particularly difficult to manage, a patient-controlled intravenous analgesic device may help control the pain. The ICV route is rarely used today. Spinal analgesics (epidural and intrathecal) are extremely helpful in patients refractory to standard analgesic regimens and reviewed elsewhere in this book.[37]

10. What opioid side effects are common in the chronic treatment of cancer pain? Any opioid can produce any opioid-related side effect (Table 48.3), in any patient, at any time. The most common side effects[38] are nausea & vomiting, constipation, dry mouth, and sedation. With chronic dosing for cancer pain tolerance typically develops to nausea and sedation. If nausea persists, the addition of antiemetic therapy may be required or rotation for a trial with a different opioid. Constipation typically requires scheduled treatment with stool softeners and bowel stimulants. Methylnaltrexone and other PAMORA agents (peripherally acting mu opioid receptor antagonists) are very effective for the treatment of refractory opioid-related constipation.[39] The patient is often concerned about changes in cognitive function yet this proves to be an uncommon complication. Significant respiratory depression is very uncommon in the cancer pain patient, as is opioid misuse or intentional overdose. Spinal analgesics have a higher incidence of pruritus and urinary retention as side effects.[2] It was hoped that ER formulations of oral opioid products would provide less side effects yet this has not been shown to

Chapter 48: Cancer pain

Table 48.3. Opioid-related side effects

Nausea & vomiting Constipation Sedation Dry mouth Confusion, mood change, cognitive impairment, dysphoria Urinary retention Sweating, pruritus, myoclonus Respiratory depression Sleep disturbances Cough inhibition Bradycardia, myocardial depression Immunologic dysfunction Hormonal dysfunction Hyperalgesia Tolerance Physical dependence Psychologic addition Opioid misuse Intentional overdose Dizziness Delayed gastric emptying

occur. Typical opioid side effects are seen with all opioid ER products.[21]

11. Are there any non-drug therapies for cancer pain management? Many cancer patients find pain relief and comfort using non-drug therapies. These include music therapy, meditation, relaxation techniques, imagery, hypnosis, biofeedback, psychotherapy, massage, art therapy, aromatherapy, coping skills training, virtual reality, acupuncture, electroanalgesia, and transcutaneous nerve stimulation. A recent systematic review indicates that music therapy may have beneficial effects on pain and also on anxiety and mood for people with cancer.[40]

12. Woulda“nerveblock”be helpfulfor this patient? The term “nerve block” is often used by the public but has little specificity for the health professional. Blockade of the peripheral or central nervous system may be either temporary (with local anesthetic) or relatively permanent (with a chemical to destroy nerve tissue). The more permanent destructive nerve block is referred to as a neurolytic block.

Figure 48.2. Celiac plexus block – lateral view fluoroscopy. Direct invasion of the lumbar spine by pancreatic cancer causing destruction of the vertebral body of L1. In spite of this significant spine disease, the patient had no lower extremity neurologic symptoms. From personal files of Rinoo V. Shah, MD, MBA.

Temporary blockade of nervous tissue with local anesthetic may be required as a diagnostic block prior to a more permanent neurolytic procedure (such as celiac plexus block).[41] This would not be expected to provide long-term pain relief, but would help the physician decide on the appropriate use of a neurolytic block. However, the temporary nature of local anesthetic application may be prolonged with the use of an indwelling catheter system. This system can provide profound pain relief in patients with refractory cancer pain and may be continued at home with minimal cost or complication.[42,43] Local anesthetics are infused to block either a nerve plexus (brachial or lumbar plexus) or a peripheral nerve. Tumors in the head and neck, upper lung, or breast may invade the brachial plexus resulting in pain that should be amenable to local anesthetic infusion of the nerve plexus. Portable infusion pumps allow for both basal and patient-controlled bolus doses, with a typical local anesthetic (bupivacaine, ropivacaine) infusion rate of 3–8 ml/hour.[41] Our patient will require a different type of block since the pain pathways associated with pancreatic cancer will travel mostly through the sympathetic nervous system. Therefore, ablation of the celiac

347

Chapter 48: Cancer pain

administration of phenol was most successful in the management of sarcoma-related cancer pain.[45]

13. What is involved with a celiac plexus neurolytic block?

Figure 48.3. Celiac plexus block – anterior-posterior view fluoroscopy. From personal files of Rinoo V. Shah, MD, MBA.

plexus using alcohol or phenol may be used to treat upper abdominal pain associated with cancer of the pancreas (or stomach, distal esophagus) since the nociceptive fibers pass through this major sympathetic nervous system plexus.[41] In addition, because no motor or other sensory fibers pass through the celiac plexus, there should be little concern for motor or sensory complications. The celiac plexus neurolytic block is the most common neurolytic block currently performed.[44] In a similar fashion, ablation of the bilateral sympathetic ganglion at the superior hypogastric level will be used to treat cancer pain from pelvic tumors (gynecological), and ablation of the terminal ganglion of the sympathetic chain (ganglion of impar) may be useful in controlling pain related to rectal cancers. Prior to the success with spinal analgesics for cancer pain management other neurolytic blocks were more popular, including subarachnoid block for chest wall pain or perineal pain, and cranial nerve ablation for head and neck cancer pain.[41] Although neuraxial neurolytic procedures are not commonly performed today, they still have a role for the patient with refractory cancer pain as highlighted by a recent case report in which the epidural

348

The celiac plexus of the sympathetic nervous system chain lies anterior to the first lumbar vertebrae, in front of the diaphragm, behind the stomach and typically surrounding the origin of the celiac artery. This location renders the plexus accessible through percutaneous and endoscopic routes of blockade. The typical block involves 1 or 2 percutaneous 22-gauge needles inserted through the back to rest near the anterolateral surface of L1 vertebrae. Either alcohol or phenol is then injected for destruction of the celiac plexus and to obtain pain relief. An anterior approach has also been described through the abdominal wall. The block should always be performed using some form of imaging (fluoroscopy, CT scan, ultrasound) to help reduce the incidence of serious complications such as spinal cord or subarachnoid injection.[46,47] A recent innovation for celiac plexus block is the use of endoscopic ultrasound as a route of injection.[48] Regardless of technique employed, rare but catastrophic complications may still occur, such as spinal cord infarction with lower extremity paralysis, or death.[49,50] The block is usually performed with local anesthesia and sedation, as an outpatient procedure, and may be repeated if pain recurs after an initial period of analgesia.[46] The efficacy of celiac plexus block has been demonstrated in both retrospective and prospective studies, although randomized clinical trials are obviously difficult to complete. The often quoted efficacy rate of two-thirds of patients was recently confirmed in a prospective trial involving 220 patients with pain related to pancreatic cancer.[51] A review of clinical trials involving patients with cancer pain found a small, but significant, difference with improved pain control for patients receiving celiac plexus block compared with opioid therapy alone, along with fewer side effects compared with opioid analgesics alone.[52] The best timing for celiac plexus block is unclear. Some physicians will perform the block when pain is moderate in the hopes of obtaining long-term pain relief, and others will perform the block only when reasonable attempts with opioid analgesics have failed. A recent prospective comparative study in patients with pancreatic cancer pain found that first controlling pain with analgesics followed

Chapter 48: Cancer pain

by a celiac plexus block was more effective for pain relief and improved quality of life than performing the celiac plexus block at the start of opioid therapy.[53] It is also suggested that the repeat celiac plexus block performed for refractory pain may be less successful than the primary block.[51] Serious complications are rare; the most common side effects are transient hypotension (typically resolves after 48 hours) and chronic diarrhea (easily treated with anti-colonic medications).[46]

Conclusions Cancer pain will continue to affect the majority of oncology patients, especially during the final weeks or

References 1.

Halvorson KG, Sevcik MA, Ghilardi JR, et al. Similarities and differences in tumor growth, skeletal remodeling and pain in an osteolytic and osteoblastic model of bone cancer. Clin J Pain. 2006;22(7):587–600.

2.

Sloan PA. Neuraxial pain relief for intractable cancer pain. Curr Pain Headache Rep. 2007;11:283–289.

3.

Steegers MA, Snik DM, Verhagen AF, et al. Only half of the chronic pain after thoracic surgery shows a neuropathic component. J Pain. 2008;9(10):955–961. Marcus DA. Epidemiology of cancer pain. Curr Pain Headache Rep. 2011;15(4):231–234. van den Beuken-van Everdingen MH, de Rijke JM, Kessels AG, et al. Prevalence of pain in patients with cancer: a systematic review of the past 40 years. Ann Oncol. 2007;18(9):1437–1449. van den Beuken-van Everdingen MH, de Rijke JM, Kessels AG, et al. High prevalence of pain in patients with cancer in a large population-based study in The Netherlands. Pain. 2007;132 (3):312–320.

4.

5.

6.

7.

Niscola P, Tendas A, Scaramucci L, et al. Pain in malignant hematology. Expert Rev Hematol. 2011;4(1):81–93.

8.

9.

months of life. It continues to be feared by patients and health professionals alike and is usually associated with a decrease in quality of life. However, patients should be encouraged that the vast majority of persons (85–95%) will have their pain relieved with standard analgesics and adjuvant analgesics. For those patients with refractory cancer pain, interventional techniques offer the possibility of pain relief, although with a higher risk of significant complications. The near future may allow for genetic coding of individual patients to determine the most effective opioid analgesic regimen and with the least amount of side effects.

Reyes-Gibby CC, Wang J, Spitz M, et al. Genetic variations in Interleukin-8 and Interleukin-10 are associated with pain, depressed mood, and fatigue in lung cancer patients. J Pain Symptom Manage. 2012 [Epub ahead of print]. Candiotti KA, Yang Z, Morris R, et al. Polymorphism in the interleukin-1 receptor antagonist gene is associated with serum interleukin-1 receptor antagonist concentrations and postoperative opioid consumption. Anesthesiology. 2011;114(5): 1162–1168.

10. Sato H, Droney J, Ross J, et al. Gender, variation in opioid receptor genes and sensitivity to experimental pain. Molecular Pain. 2013;9:20. 11. Nagele P. Perioperative genomics. Best Pract Res Clin Anaesthesiol. 2011;25(4):549–555. 12. Katz J, Jackson M, Kavanagh BP, Sandler AN. Acute pain after thoracic surgery predicts longterm post-thoracotomy pain. Clin J Pain. 1996;12:50–55. 13. Glare P. Choice of opioids and the WHO ladder. J Pediatr Hematol Oncol. 2011;33(Suppl 1): S6–11. 14. Mercadante S. Management of cancer pain. Intern Emerg Med. 2010;5(Suppl 1):S31–35.

15. Leppert W, Luczak J. The role of tramadol in cancer pain treatment: a review. Supp Care Cancer. 2005;13:5–17. 16. Sloan PA. The evolving role of interventional pain management in oncology. J Support Oncol. 2004;2:491–506. 17. McNicol E, Strassels SA, Goudas L, et al. NSAIDs or paracetamol, alone or combined with opioids, for cancer pain. Cochrane Database Syst Rev. 2005;25(1): CD005180. 18. Mercadante S, Giarratano A. The long and winding road of non steroidal antiinflammatory drugs and paracetamol in cancer pain management: a critical review. Crit Rev Oncol Hematol. 2013;87 (2):140–145. 19. Hauser W, Wolfe F, Tolle T, et al. The role of antidepressants in the management of fibromyalgia syndrome: a systematic review and meta-analysis. CNS Drugs. 2012; 26(4):297–307. 20. Mitra R, Jones S. Adjuvant analgesics in cancer pain: a review. Am J Hosp Palliat Care. 2012;29 (1):70–79. 21. Sloan PA, Babul N. Extendedrelease opioids for the management of chronic nonmalignant pain. Exp Opin Drug Deliv. 2006;3:489–497.

349

Chapter 48: Cancer pain

22. Mount BM, Ajemian I, Scott JF. Use of the Brompton mixture in treating the chronic pain of malignant disease. Can Med Assoc J. 1976;115:122–124. 23. Thirlwell MP, Sloan PA, Maroun JA, et al. Pharmacokinetics and clinical efficacy of oral morphine solution and controlled-release morphine tablets in cancer patients. Cancer. 1989;63: 2275–2283. 24. Knotkova H, Fine PG, Portenoy RK. Opioid rotation: the science and the limitations of the equianalgesic dose table. J Pain Symptom Manage. 2009;38 (3):426–439. 25. Dhaliwal H, Sloan PA, Arkinstall W, et al. Randomized evaluation of controlled release codeine and placebo in chronic cancer pain. J Pain Symptom Manage. 1995;10:612–623. 26. Bruera E, Sloan PA, Mount BM, et al. A randomized, double-blind, double-dummy, crossover trial comparing the safety and efficacy of oral sustained-release hydromorphone to immediaterelease hydromorphone in patients with cancer pain. J Clin Oncology. 1996;14:1713–1717. 27. Sloan PA, Moulin D, Hays H. A clinical evaluation of TTSFentanyl for the treatment of cancer pain. J Pain Symptom Manage. 1998;16:102–111. 28. Sloan PA, Slatkin NE, Ahdieh H. Effectiveness and safety of oral extended-release oxymorphone for the treatment of cancer pain: a pilot study. Supp Care Cancer. 2005;13:57–65. 29. Webster LR, Brewer R, Morris D, et al. Opioid titration and conversion in patients receiving morphine sulfate and naltrexone hydrochloride extended-release capsules. Postgrad Med. 2011;123 (5):155–164.

350

30. Koyyalagunta D, Bruera E, Solanki DR, et al. A systematic review of randomized trials on the effectiveness of opioids for cancer pain. Pain Physician. 2012;15(3 Suppl):ES39–58.

41. Sloan PA. Cancer pain management, anesthesiologic interventions. In Schmidt RF, Gebhart GF, eds. Encylopedia of Pain. Heidelberg, Germany: Springer-Verlag. 2013.

31. Cachia E, Ahmedzai SH. Transdermal opioids for cancer pain. Curr Opin Support Palliat Care. 2011;5(1):15–19.

42. Sloan PA. Ultrasound-guided femoral nerve catheter for the treatment of refractory cancer pain. J Palliative Care. 2013;29:244–245.

32. Brown R, Kraus C, Fleming M, Reddy S. Methadone: applied pharmacology and use as adjunctive treatment in chronic pain. Postgrad Med J. 2004;80:654–659. 33. Payne R. Recognition and diagnosis of breakthrough pain. Pain Med. 2007;8(Jan):S3–7. 34. McCarberg BH. The treatment of breakthrough pain. Pain Med. 2007;8(Jan):S8–13. 35. Davis MP. Fentanyl for breakthrough pain: a systematic review. Expert Rev Neurother. 2011;11(8):1197–1216. 36. Dietrich E, Gums JG. Intranasal fentanyl spray: a novel dosage form for the treatment of breakthrough cancer pain. Ann Pharmacother. 2012;46 (10):1382–1391. 37. Hamann SR, Sloan PA, Witt WO. Low-dose intrathecal naloxone to enhance intrathecal morphine analgesia: a case report. J Opioid Manage. 2008;4:251–254. 38. Benyamin R, Trescot AM, Datta S, et al. Opioid complications and side effects. Pain Physician. 2008;11(2 Suppl):S105–120. 39. Thomas J, Karver S, Austin Cooney G, et al. Methylnaltrexone for opioid-induced constipation in advanced illness. N Eng J Med. 2008;358:2332–2343. 40. Bradt J, Dileo C, Grocke D, Magill L. Music interventions for improving psychological and physical outcomes in cancer patients. Cochrane Database Syst Rev. 2011;10(8): CD006911.

43. Buchanan D, Brown E, Millar F, et al. Outpatient continuous interscalene brachial plexus block in cancer-related pain. J Pain Symptom Manage. 2009;38 (4):629–634. 44. Sloan PA. The role of gastrointestinal endoscopic ultrasound-guided celiac plexus neurolytic block for pancreatic cancer pain. J Supp Oncol. 2006;4:463–464. 45. Candido KD, Philip CN, Ghaly RF, Knezevic NN. Transforaminal 5% phenol neurolysis for the treatment of intractable cancer pain. Anesth Analg. 2010;110 (1):216–219. 46. Birthi P, Sloan PA. Interventional treatment of refractory cancer pain. The Cancer J 2013;19: 390–396. 47. Koizuka S, Nakajima K, Mieda R. CT-guided nerve block: a review of the features of CT fluoroscopic guidance for nerve blocks. J Anesth 2013; Jul 20 [Epub ahead of print]. 48. Nishimura M, Togawa O, Matsukawa M, et al. Possibilities of interventional endoscopic ultrasound. World J Gastrointest Endosc. 2012;4(7):310–315. 49. Fujii L, Clain JE, Morris JM, Levy MJ. Anterior spinal cord infarction with permanent paralysis following endoscopic ultrasound celiac plexus neurolysis. Endoscopy. 2012;44 (Suppl 2):E265–266. 50. Gimeno-Garcia AZ, Elwassief A, Paquin SC, Sahai AV. Fatal

Chapter 48: Cancer pain

complication after endoscopic ultrasound-guided celiac plexus neurolysis. Endoscopy. 2012;44 (Suppl 2):E267. 51. McGreevy K, Hurley RW, Erdek MA, et al. The effectiveness of repeat celiac plexus neurolysis for pancreatic cancer:

a pilot study. Pain Practice. 2012;13:89–95. 52. Arcidiacono PG, Calori G, Carrara S, et al. Celiac plexus block for pancreatic cancer pain in adults. Cochrane Database Syst Rev. 2011;(3): CD007519.

53. Amr YM, Makharita MY. Comparative study between 2 protocols for management of severe pain in patients with unresectable pancreatic cancer: one-year follow-up. Clin J Pain. 2013; 29(9):807–813.

351

Section 6 Chapter

49

Cancer Pain

Patient presents with pancreatic cancer with persistent pain despite all other treatments Jay S. Grider

Case study A 56-year-old male presents to the interventional pain management center with ongoing abdominal pain on referral from the palliative care team. The pain is described as burning and sharp frequently, with constant “achy” and “crampy” pain typically. The patient is currently in extreme pain, rating it as a 10/10 during worst exacerbations. In addition to the epigastric pain, the patient also describes severe pain radiating to the back. The current analgesic regimen consists of 150 μg per day of transdermal fentanyl, 40 mg of sustainedrelease oxycodone twice daily, and hydromorphone 4 mg IR tablets 3–6 times per day as needed for breakthrough pain. Additionally the patient is taking Cymbalta 90 mg per day and gabapentin 300 mg three times daily for neuropathic-type abdominal pain according to the medical record. The patient has had surgical intervention in the form of a Whipple procedure which did not improve pain control. Chemotherapy and palliative radiation have also failed to improve the pain. The palliative care physicians have tried numerous opioid and adjunctive medication combinations in an attempt to provide analgesia but all have failed. They are considering a subcutaneous opioid infusion but would like to try a celiac plexus block first.

1. What barriers to patient care might an interventional pain physician face in cross-disciplinary care? The case outlined above represents several issues that must be addressed by the interventional pain physician treating patients with cancer-related pain. The first and most common issue surrounds the late stage at which oncologist and palliative care physicians

often seek aid from an interventional pain physician. As noted above the patient is in extreme pain and has likely wearied of the “lets try this and see if it works” clinical approach. Because of the frequent delay in consultation with specialists the family and loved ones of the patient are often very frustrated. In most cases the patient and family are having to deal emotionally with the terminal nature of the disease, but took comfort in the oft heard refrain “we will keep your loved one comfortable no matter what.” When this proves elusive or difficult to achieve, there is understandably a sense of betrayal. An interventional pain physician who is called upon to treat these patients would be wise to suggest that early involvement in the care of the patient allows the chance to outline pain treatment options before the pain becomes intractable. This early discussion allows rational decision making and informed consent. The interventional pain physician is often told by the patient, “we don’t care about the risks just do something” which short circuits a true informed consent process. The second issue with regard to late consultation surrounds the opioid dose titration that often occurs in the absence of comprehensive care involving intervention. Many well-meaning care teams aggressively titrate opioid doses resulting in tolerance, opioidrelated side effects, and occasionally opioid-induced hyperalgesia. While opioid therapy is well-established in cancer care, doses are often titrated early in a disease process that could be managed by other means.[1] When the patient subsequently reaches a late-stage in their illness, the opioid-related tolerance often results in diminished efficacy. The distressing discomfort as opioids prove less effective in the final hours and days of life can often be avoided by targeted early intervention. This allows aggressive opioid titration

Case Studies in Pain Management, ed. Alan David Kaye and Rinoo V. Shah. Published by Cambridge University Press. © Cambridge University Press 2015.

352

Chapter 49: Patient presents with pancreatic cancer with persistent pain despite all other treatments

at the very end of life to have the desired palliative effect. Building clinical relationships and care teams that result in early coordinated care avoids the seemingly emergent intervention on a patient in extremis and creates a continuum of care that is reassuring to the patient and their family.

2. What are common sources of pain in the pancreatic cancer patient? 1. Pain from oncologic process a. Tumor compression of surrounding viscera b. Tumor invading nearby plexus c. Metastatic pain of somatic or osteolytic sources 2. Pain from treatment process a. Radiation neuritis b. Chemotherapy-induced peripheral neuropathy c. Chronic postsurgical pain

3. What interventional techniques may be used? Since the specialties that care for the pancreatic cancer patient (or any intra-abdominal oncologic process for that matter) are often excellent at the application and titration of opioids and managing opioid-related side effects, the pain medicine physician is often asked to purely provide intervention for this group of patients.[1] As such the discussion will focus on these techniques. 1. Blocks a. Diagnostic b. Neurolytic i. Alcohol ii. Phenol iii. Radiofrequency c. Anatomic location i. Celiac plexus ii. Splanchnic nerve iii. Superior hypogastric plexus 2. Spinal cord stimulation 3. Neuraxial drug administration a. Continuous epidural analgesia b. Continuous intrathecal drug delivery i. Externalized ii. Subcutaneous

Blocks The use of the celiac plexus block in the treatment of cancer-related abdominal pain is well established. As the case demonstrates however, all interventional treatment for pancreatic cancer is often “lumped” into the celiac plexus block category by the uninitiated despite a wide variety of interventional treatment options. Educating the oncologist/palliative care team about the armamentarium of intervention available will perhaps change the perspective that abdominal cancer-related intervention is limited to one modality. The use of diagnostic blocks and therapeutic trials prior to definitive intervention in cancer-related pain is controversial.[2] Many interventional specialists correctly advocate clear discussion of risks and benefits of a procedure prior to performing it but force the patient to endure diagnostic procedures/therapeutic trials in an attempt to establish efficacy of a procedure.[2] This often occurs by an interventionalist who deals mostly with chronic benign pain and as such is reluctant to proceed directly to definitive or ablative procedures. Experiences and opinion differ and clearly therapy must be tailored to the patient. If the potential side effects of a treatment are distressing to the patient then a diagnostic block often helps gain their trust as many side effects are better tolerated if accompanied with excellent analgesia. The psychology of this concept cannot be understated as the pain becomes a routine part of the life of a late-stage cancer patient. The cancer patient often imagines the side effect described by the physician in addition to the pain they currently experience. It is important to underscore that, though there are risks, the benefit may be excellent analgesia from ablative or anesthetic procedures. Frequently patients have related to our team after an intervention has been performed that they had forgotten what it was like to be relatively comfortable. Many excellent resources describe the technical aspects of celiac plexus blockade and as such are beyond the scope of the current discussion. The interested reader is directed to technical resources describing the technique and selection of neurolytic agents.[2–10]Neuraxial neurolytic blocks have been described in previous decades but as better peripheral imaging and interventional techniques have evolved, there has been less enthusiasm for this route of neurolysis as the potential for unintended lesions is greater.[4–10]Cryoneurolysis has been used for

353

Chapter 49: Patient presents with pancreatic cancer with persistent pain despite all other treatments

cancer-related pain; however the probe size used in the modality can limit its usefulness for pancreatic interventions. The use of radiofrequency (RF) lesioning using both conventional energy and pulsed energy has been described.[2] One approach using RF that is recently described deserves attention; the use of radiofrequency ablation of the splanchnic nerves in the treatment of pancreatic cancer-related pain (PC-RP). Radiofrequency ablation of the splanchnic nerves may represent an alternative to chemical neurolysis with alcohol or phenol.[11] The advantages of this technique over chemical neurolysis are: (1) predictability of lesion, (2) safety, and (3) technical expertise well established in most pain centers that utilize RF techniques (versus relatively specialized use of alcohol or phenol). For example, one of the potential complications of celiac plexus block involves damage to the surrounding viscera. While this complication is rare, it does occur, and occasionally with devastating results such as bowel perforation and paralysis.[1] The splanchnic radiofrequency approach theoretically limits this risk as well as limiting the risk of spread of neurolytic agent to unwanted visceral structures or into the spinal canal. In the study by Papadopoulous et al, patients had improved analgesia at 6 months though no direct comparison to traditional celiac plexus block or to conventional medical therapy was made.[11] A meta-analysis on the effectiveness of traditional celiac plexus blockade suggested that 70–90% of patients had sustained benefit up to 3 months after the procedure or until death.[12] In keeping with the issues noted in the case discussion, the study found that when celiac plexus blockade employed earlier in the disease process is compared to conventional medication management with opioids, the interventional treatment arm demonstrated significantly better quality of life.[12] Similarly other studies have suggested that celiac plexus block and opioid therapy have similar outcomes with regards to analgesia; however the celiac plexus group had fewer side effects than the opioid therapy group.[12] The side effects of celiac plexus blockade are well described and include transient hypotension, back pain, and diarrhea. Severe procedure-related complications include hemorrhage, aortic dissection, and paralysis with the incidence of these being on the order of 1 in 700.[12] On occasion pain from the oncologic process encompasses more than the distribution of the celiac ganglion. In these situations the clinician should

354

consider adding a superior hypogastric plexus block via one of the many approaches described in the literature to provide complete analgesia.[3] Though rare, a ganglion of impar block may be required for painful symptoms of the anus or perineum which may develop due to a metastatic process. When pancreatic epigastric pain becomes more widespread, other means of treatment such as neuraxial intervention should be considered.

Spinal cord stimulation The use of this modality in cancer-related pain in general and for PC-RP specifically is not well established in the literature. While use of spinal cord stimulation for the treatment of chronic abdominal pain has a growing body of literature with variable success rates, the use of this modality in PC-RP is limited to case reports with clear neuropathic pain origins.[13,14]

Neuraxial drug administration The use of externalized epidural catheters placed at the dermatomal site of pain (in the case of PC-RP the T4–6 level) has been well described in the literature.[16] While the safety of this modality is relatively well established[15] there are many experts who suggest that this route of administration not be employed as there is a tendency toward fibrosis at the epidural catheter tip potentially resulting in interruption of therapy.[2,16] In the author’s institution, epidural analgesia for abdominal cancer-related pain has been successfully utilized for decades with only periodic fibrosis of the catheter tip. Further, since PC-RP is typically well localized another frequent criticism of epidural analgesia is avoided (i.e., the finding that large volumes of infusate must be used to cover an extensive area of pain). Unfortunately in our practice we have considered abandonment of frequent use of epidural analgesia mainly because of the discontinued production of the DuPen catheter (Baxter International, Deerfield, IL); named for Dr. Stuart DuPen a pioneer in the use of neuraxial analgesia for cancer-related pain. This catheter was well designed specifically for long-term epidural management and a suitable replacement product has proved elusive. The inability to utilize an epidural catheter constructed specifically for long-term use has led to frequent dislodgement in our clinical experience and

Chapter 49: Patient presents with pancreatic cancer with persistent pain despite all other treatments

ultimately reassessment of this route of medication delivery. Intrathecal drug delivery (IDD), either via an externalized catheter or implanted pump, has been suggested as a superior route of drug administration compared to either epidural administration or oral opioid administration.[16–26]The threshold of 3 months survival is often used to determine whether an implantable system should be used for analgesia versus an externalized catheter.[17] The merits and deficiencies of both routes of administration aside, the use of this 3 month time period as a criterion for choosing treatment is fraught with the potential for error as many practitioners have implanted an IDD system in a patient only to have them die within a short period of time, while others have chosen a percutaneous catheter only to see the patient live several months. Perhaps a better criterion would be the ability to tolerate an externalized catheter and the presence of care support infrastructure to help care for this modality. When the patient desires to be unencumbered from externalized tubing or does not have the requisite care system in place, perhaps an implanted pump may be best. This must be weighed against (1) the immunosuppression and wound healing difficulties of many chemotherapy/radiation patients, (2) the desire to spend what may be a significant amount of the remaining life span healing from a wound, and (3) the relatively limited number of community personnel who feel comfortable titrating implanted IDD therapy.[17] The final point should not be discounted as many home health agencies will assist with a percutaneous home intrathecal infusion but will not assist with the more technologically sophisticated IDD system, thus requiring the cancer patient to travel to the implanting physician to titrate analgesia. Selection of the route of drug administration is only part of the decision-making process. The drug titration paradigm and medication combinations selected are in fact where the patient benefit is derived. Clinician comfort with a wide variety of intrathecal opioids and dosing parameters is key to optimizing patient outcomes. The use of adjunctive medications such as local anesthetics, clonidine, and ziconotide can significantly improve analgesia for patients with mixed pain syndromes of somatic and neuropathic origin caused by cancer. The daily dose of intrathecal opioids and the titration schedule often may also necessarily be more aggressive than that in

the typical clinician experience with chronic benign pain.[17] It is not uncommon for patients to require 25–50% increases in medication dosage in late-stage cancer pain treatment to maintain analgesia.[17] Additionally, aggressive use of local anesthetics, with concomitant motor/sensory block, is often utilized in late-stage cancer pain management as function becomes less of a focus and palliative analgesia becomes more important. In these situations the interventional pain physician may need to have a frank discussion with the patient and family about the compromises in motor function that may be required.[17–26] Anecdotally, many patients who feared loss of mobility either from pain or motor weakness from aggressive local anesthetic administration may be more receptive to the concept when profound relief is obtained via epidural or intrathecal local anesthetic administration. The interested reader is directed to the multidisciplinary guidelines by Stearns and colleagues for more detailed information.[17]

4. What are the complications/ contraindications to neuraxial analgesia? The complications of celiac plexus block were outlined previously. It should be noted that most patients will experience hypotension and diarrhea and as such inpatient hydration may occasionally be needed in the first 48 hours. With regard to neuraxial complications rates, the rate of infection in long-term epidural catheters has been quoted at 6.1%; however the vast majority of those were superficial.[16] Likewise tunneled intrathecal catheters have a low risk of meningitis and infection while the most common complaint with this modality is postdural puncture headache with an incidence of 15%.[2] When these headaches occur they are normally self-limiting and resolve within a few days. Epidural blood patch may be considered in those with persistent headache. The guidelines pertaining to neuraxial analgesia and anticoagulation should govern intervention with these modalities.[26] The presence of spinal metastasis does not represent a contraindication to placement of an indwelling epidural or intrathecal catheter; however correlation with imaging to determine site of placement is imperative with recognition that pathologic barriers to medication spread may be present.

355

Chapter 49: Patient presents with pancreatic cancer with persistent pain despite all other treatments

References 1.

Raphael J, Hester J, Ahmedzai S, Barrie J, Farqhuar-Smith P. Cancer pain: part 2: physical, interventional and complementary therapies; management in the community; acute, treatment-related and complex cancer pain: a perspective from the British Pain Society endorsed by the UK Association of Palliative Medicine and the Royal College of General Practitioners. Pain Med. 2010;11:872–896.

2.

de Courcy JG. Interventional techniques for cancer pain management. Clin Oncol (R Coll Radiol). 2011;23:407–417.

3.

Manchikanti L, Singh V. Celiac plexus and splanchnic blockade. In Interventional Techniques in Chronic Non-Spinal Pain, 1st edn. Paducah, Kentucky: ASIPP Publishing. 2009: pp. 199–212.

4.

5.

6.

7.

356

Noble M, Gress FG. Techniques and results of neurolysis for chronic pancreatitis and pancreatic cancer pain. Curr Gastroenterol Rep. 2006;8:99–103. Ischia S, Ischia A, Polati E, et al. Three posterior percutaneous celiac plexus block techniques: a prospective randomized study in 61 patients with pancreatic cancer pain. Anesthesiology. 1992;76: 534–540. Wong GY, Schroeder DR, Carns PE, et al. Effect of neurolytic celiac plexus block on pain relief, quality of life, and survival in patients with unresectable pancreatic cancer: a randomized controlled trial. JAMA. 2004;291:1092–1099. Kaufman M, Singh G, Das S, et al. Efficacy of endoscopic ultrasoundguided celiac plexus block and celiac plexus neurolysis for managing abdominal pain associated with chronic pancreatitis and pancreatic cancer. J Clin Gastroenterol. 2010;44:127–134.

8.

Perello A, Ashford NS, Dolin SJ, et al. Coeliac plexus block using computed tomography guidance. Palliat Med. 1999;13:419–425.

9.

Mercadante S. Celiac plexus block versus analgesics in pancreatic cancer pain. Pain. 1993;52: 187–192.

10. de Oliveira R, dos Reis MP, Prado WA, et al. The effects of early or late neurolytic sympathetic plexus block on the management of abdominal or pelvic cancer pain. Pain. 2004;110:400–408. 11. Papadopoulos D, Kostopanagiotou G, Batistaki C. Bilateral thoracic splanchnic nerve radiofrequency thermocoagulation for the management of end-stage pancreatic abdominal cancer pain. Pain Physician. 2013;16:125–133. 12. Puli SR, Reddy JB, Bechtold ML, et al. EUS-guided celiac plexus neurolysis for pain due to chronic pancreatitis or pancreatic cancer pain: a meta-analysis and systematic review. Dig Dis Sci. 2009;54:2330–2337. 13. Yakovlev AE, Ellias Y. Spinal cord stimulation as a treatment option for intractable neuropathic cancer pain. Clin Med Res. 2008;6:103–106. 14. Yakovlev AE, Resch BE, Karasev SA, et al. Treatment of cancerrelated chest wall pain using spinal cord stimulation. Am J Hospice Palliat Med. 2010;27:552–556. 15. Ruppen W, Derry S, McQuay HJ, Moore RA. Infection rates associated with epidural indwelling catheters for seven days or longer: systematic review and meta-analysis. BMC Palliat Care. 2007;6:3. 16. Ballantyne JC, Carwood C. Comparative efficacy of epidural, subarachnoid, and intracerebroventricular opioids in patients with pain due to cancer. Cochrane Database Syst Rev. 2005;2:CD005178.

17. Stearns L, Boortz-Marx R, Du Pen S, et al. Intrathecal drug delivery for the management of cancer pain: a multidisciplinary consensus of best clinical practices. J Support Oncol. 2005;3:399–408. 18. Deer T, Krames ES, Hassenbusch SJ, et al. Polyanalgesic consensus conference 2007: recommendations for the management of pain by intrathecal (intraspinal) drug delivery: report of an interdisciplinary expert panel. Neuromodulation. 2007;10:300–328. 19. Kedlaya D, Reynolds L, Waldman S. Epidural and intrathecal analgesia for cancer pain. Best Pract Res Clin Anaesthesiol. 2002;16:651–665. 20. Hassenbusch SJ, Portenoy RK, Cousins M, et al. Polyanalgesic Consensus Conference 2003: an update on the management of pain by intraspinal drug delivery report of an expert panel. J Pain Symptom Manage. 2004;27:540– 563. 21. Williams JE, Louw G, Towlerton G. Intrathecal pumps for giving opioids in chronic pain: a systematic review. Health Technol Assess. 2000;4:1–65. 22. British Pain Society. Intrathecal Drug Delivery for the Management of Pain and Spasticity in Adults; Recommendations for Best Clinical Practice. British Pain Society London. 2008. 23. Gilmer-Hill HS, Boggan JE, Smith KA, et al. Intrathecal morphine delivered via subcutaneous pump for intractable cancer pain: a review of the literature. Surg Neurol. 1999;51:12–15. 24. Smith TJ, Staats PS, Deer T, et al. Randomized clinical trial of an implantable drug delivery system compared with comprehensive medical management of refractory cancer pain: impact on pain, drug-related toxicity and

Chapter 49: Patient presents with pancreatic cancer with persistent pain despite all other treatments

survival. J Clin Oncol. 2002;20:4040–4049. 25. Smith TJ, Coyne PJ, Staats PS, et al. An implantable drug delivery system (IDDS) for refractory cancer pain provides sustained pain control, less drug-related

toxicity, and possibly better survival compared with comprehensive medical management (CMM). Ann Oncol. 2005; 16:825–833. 26. Horlocker T, Wedel D, Rowlingson J, et al. Regional

anesthesia in the patient receiving antithrombotic or thromobolytic therapy: American society of regional anesthesia and pain medicine evidence-based guidelines (third edition). Reg Anesth and Pain Med. 2010; 35(1):64–101.

357

Section 6 Chapter

50

Cancer Pain

Pain management in hematological cancer: clinical case illustrations Quan D. Le, Alan David Kaye, and Harry J. Gould, III

Hematologic malignancies make up about 9% of the new cancer cases in the USA in 2013. Of the new hematologic cancer cases, approximately 53% were lymphoma, 32% were leukemia, and 15% were myeloma.[1] The management of pain in hematologic cancers presents a constellation of problems that are distinctly different from those associated with solid tumors. We will review four cases of patients who presented with pain associated with hematologic cancer that illustrate the unique complexity and breadth of the problems to be addressed. We will then discuss considerations that should be taken that affect the assessment of risks and the selection of analgesic treatment, and the monitoring of clinical response.

Case studies Patient 1 is an 89-year-old female with low-grade (inactive/stage 1) multiple myeloma who presented with fatigue and an acute exacerbation of her chronic back pain. The pain was located in the mid to lower back and was described as having a deep, aching, throbbing, sharp, and stabbing character with intermittent burning features. She rated her worst pain at 10/10 on a numerical rating scale. Her best and average pain levels were rated at 5/10 and 7/10, respectively. Severe point tenderness was not elicited on physical examination although general exacerbation of baseline tenderness was obtained with provocative maneuvers and multifocal high-intensity lesions were observed with magnetic resonance images of the lumbosacral spine (Figure 50.1). Patient 2 is a 51-year-old male with a history of acute myelogenous leukemia (AML) who presented to clinic in blast crisis. He underwent and tolerated chemotherapeutic induction but later presented with fatigue, occasional diffuse headaches, double vision, dysphagia, multiple cranial neuropathies, and gait and motor disturbances characterized by bilateral

Figure 50.1. Magnetic resonance image taken of the lower thoracic and lumbosacral region in patient 1. The heterogeneous appearing multifocal areas of high-intensity signal in the bone marrow may represent degenerative bone marrow changes, osteopenia, anemia, or lytic lesions.

weakness and ataxia. The patient also reported burning, tingling, and numbness in the face bilaterally affecting the forehead, cheek, and mandible, and a patchy distribution of paresthesia and dysesthesia in the neck, trunk, and limbs on both sides. Overall his neurologic deficits were more debilitating than his pain. He rated his worst pain intensity levels at 7/10 mostly at night on a numerical rating scale. His best

Case Studies in Pain Management, ed. Alan David Kaye and Rinoo V. Shah. Published by Cambridge University Press. © Cambridge University Press 2015.

358

Chapter 50: Pain management in hematological cancer: clinical case illustrations

and average pain levels were rated at 0/10 and 3/10, respectively. The diagnostic work-up for the new complaints included an evaluation of the CSF that revealed leptomeningeal carcinomatosis (LC). He subsequently underwent Ommaya reservoir placement for intrathecal therapy to suppress malignant cells in the CSF and received radiation therapy to the head to palliate local symptoms.[2] After completing a course of intrathecal chemotherapy and one radiation treatment, the patient developed pancytopenia, neutropenic fevers pneumonia, respiratory failure, and malnutrition that required admission to the intensive care unit for treatment and monitoring. Patient 3 is a 52-year-old female who carried a diagnosis of chronic myelogenous leukemia (CML), had a lymphoid blast crisis, and converted to acute lymphoblastic leukemia (ALL) approximately 3.5 years after diagnosis. She received and seemed to be tolerating chemotherapy that included intrathecal methotrexate and cytarabine (ARA-C) with prophylactic antibiotic, antifungal, and anti-viral agents. The patient’s pain was generally well managed. She routinely experienced occasional sharp pains, diffuse body aches, and fatigue upon walking about 1–2 blocks and reported pain levels that averaged approximately 4/10 in intensity. She also reported swelling, burning pain, and tingling and numbness in her hands and feet that was worse at night. She occasionally complained of sharp neck pain that originated in the left cervical area and spread anteriorly to the forehead. She denied asymmetric weakness, slurred speech, or swallowing difficulty. Physical examination revealed tightness over the left trapezius region. Her response to mechanical stimulation with von Frey monofilaments was decreased bilaterally in a stocking-glove distribution in the distal lower extremities. A Tinel sign could be elicited at the wrist bilaterally. Her deep tendon reflexes were slightly diminished, 1+, but symmetric in the extremities. Her cranial nerve and motor exams were normal. Patient 4 is a 46-year-old female with stage M1 AML who underwent induction chemotherapy and subsequently developed neutropenic fevers, malnutrition, deep vein thrombosis, pulmonary embolism, and jaundice with progressively worsening colicky right upper quadrant deep aching pain that required admission for a complicated hospital course. She also complained of worsening low back pain, diffuse body ache, and diffuse burning pain bilaterally in the upper and lower extremities. She rated her worst pain at 10/10 on

a numerical rating scale. Her best and average pain levels were rated at 7/10 and 8/10, respectively. Serum liver enzymes and alkaline phosphatase levels were elevated and CT scan and corroborative abdominal ultrasound revealed evidence of a distended, sludgefilled gallbladder. A hepatobiliary iminodiacetic acid (HIDA) scan revealed evidence of cystic duct obstruction or spasm, which supported the diagnosis of an acute cholecystitis. A biliary drain was placed.

1. Introduction Chronic or recurrent pain is a multifaceted problem that is familiar to over half the population. It is frequently difficult to manage and when poorly treated has a significant impact on function, mood, physical and psychologic health, and the enjoyment of life.[3] When pain is associated with neoplastic disease these negative features are amplified by the frequent futility associated with the underlying diagnosis which in turn further exacerbates the pain. Although most cancers pose similar challenges to managing pain, hematologic malignancies offer some unique features that complicate pain management. Unlike solid tumors, hematologic cancers due to altered blood cell morphology, numbers or function, tend to affect multiple organ systems because of alterations either in the oxygen-carrying capacity or the immunologic function of the blood.[4–6] These changes result in early fatigue, myalgias, and arthralgias.[7] Next, alterations in platelet numbers, functions, clotting times, and disease-related coagulable states are frequently associated with hemorrhages, ischemia, or thrombosis that lead to compromise in multiorgan function.[8–10] Finally, the compromise of components of the inflammatory system reduces the protective and restorative capabilities of the body for fighting off infections and healing.[11,12] Therefore the painful sequellae of hematologic compromise must be recognized and the treatments must consider the potential impact on already impaired tissues. Pain states associated with hematologic malignancies may be grouped and described in several ways but generally fall within three general classes; nociceptive related to tissue injury in the presence of a normally functioning nervous system, neuropathic associated with abnormal processing within either the central or peripheral nervous system or both, and mixed patterns that include components of nociceptive and neuropathic quality.[13–15] Each pain type can

359

Chapter 50: Pain management in hematological cancer: clinical case illustrations

involve somatic and visceral structures which to a large degree determine the distribution and characteristics of the pain complaint. When present in chronic disease, pain is either acute, presenting intermittently in response to activity or variations in medical state, or chronic, existing at a persistent level with fluctuations around a baseline intensity punctuated by episodes of exacerbated or breakthrough pain, associated with changes in medical condition or analgesic efficacy. Specific examples of how pain is manifested in hematologic cancer are illustrated in the following clinical cases. The considerations of potential risks and benefits that are necessary for selecting optimum interventions and analgesic therapy will be discussed.

2. Discussion The mechanisms of hematologic neoplasias dictate the unique types and patterns of pain generators that develop and both direct and impose limitations on the selection of treatment options available for the management of pain. As illustrated in the cases presented, pain that is associated with hematologic malignancies generally presents as complaints of multifocal or diffuse and poorly localized pain that involves multiple organs and organ systems. The classic and most frequent presentations include fatigue, myalgias, arthralgias, paresthesia, dysesthesia, burning pain, and bone pain. Patients are at high risk for spontaneous or procedure-related hemorrhage, ischemia, or thrombosis and are at significant risk for infection. The risk of iatrogenic injury involving the central and peripheral nervous system and the scarring of tissues and organs that lead to compromised function are high. Thus, the type of cancer plays a large role in the selection of treatments for pain control. Much of the pain associated with hematologic cancers is related to the infiltration and proliferation of malignant cells within regions that can accommodate limited amounts of tissue expansion, e.g., spine, long bones, and cranial vault (Figures 50.1 and 50.2). In order to survive in regions with limited space, malignant cells must invade and destroy normal space-occupying tissue thereby weakening the body’s supportive elements, compromising normal functions, and creating environments rich in the byproducts of inflammation. The bone marrow is a frequent site for tumor invasion in multiple myeloma. The compromise of the mineral components of skeletal

360

support and the decreased production of the oxygencarrying components of the blood commonly present as fatigue and acute exacerbation of bone pain as described by patient 1.[16,17] Bone pain is of mixed character with nociceptive and neuropathic features.[13] The bone pain in hematologic cancer is thought to be the result of malignant cell infiltration of the bone marrow, stimulating the direct and indirect release of inflammatory mediators and other factors (e.g., nerve growth factors, tumor necrosis factors) that cause tissue injury. These factors inhibit osteoblastic activity and induce tissue injury by enhancing osteoclastic activities that create an extracellular acidic environment and increase bone resorption.[18] Nociceptive bone pain, therefore, results from a combination of increased mechanical stress on mineral deficient bones, highly sensitive periosteal structures, and peripheral sensitization of primary nociceptive afferents. By contrast, neuropathic bone pain results from the sprouting and sensitization of the peripheral afferents that innervate the bone marrow, the mineralized bone and the periosteum and central sensitization of nociceptive signaling that is augmented during periods of inflammation.[19,20] Incident pain with volitional movement can be explained by the mechanical stress placed on structures with smaller reserve for reaching threshold stimulation thereby activating the low-threshold nociceptors and putative mechanotransducers in the weakened bone.[21] When as for patient 2, the malignant infiltration occurs within the cranial vault, tumor burden similarly places increased pressure on the sensitive leptomeninges at the base of the skull and additionally produces a neuro-inflammatory sensitization of the primary afferents in the meninges that lead to dull, global headaches, facial pain, cranial neuropathies, and multifocal dysesthesia/paresthesia in the neck, trunk, and limbs when the malignant cells extend into the spinal column. Pain associated with hematologic cancers also occurs as a consequence of altered hematopoiesis that results in cells of abnormal shape, life span, and function. The disturbance in hematopoiesis reduces the body’s ability to carry the products and byproducts of respiration, to recognize and fight off infections, and to effectively limit and repair injured tissue due to the lack of sufficient quantities and functional capabilities of blood components. When tissues receive insufficient oxygen, as was the case of patient 1, fatigue with activity develops early due to limited

Chapter 50: Pain management in hematological cancer: clinical case illustrations

Figure 50.2. Nuclear scans (A-P, left; P-A, right) from patient 2 illustrate highintensity signal that is widely and symmetrically distributed within the spine, pelvis, and major joints of the body. Additional focal lesions were identified in the right medial epicondyle and in the tibia bilaterally (arrows).

oxygen reserve. As the condition advances, as for patient 3, and hypoxia becomes more pronounced, even minor effort results in tissue and organ ischemia that triggers nociceptive pain, e.g., myalgias and arthralgias, to warn the patient of impending tissue injury. When tissue damage occurs, the cascade of inflammatory events leads to the peripheral sensitization of primary afferents as described above. The pain associated with ischemic pulmonary injury related to deep vein thrombosis and pulmonary embolism experienced by patient 4 further illustrates the potential consequences of insufficient or improperly functioning clotting elements seen in hematologic cancers. Ischemic injury, as well as tumor-induced nerve tissue

injury, paraprotein anti-nerve activity, amyloid deposition, and/or infections can underlie the numbness, swelling, burning, and tingling pain of neuropathic quality that patient 3 experienced in her hands and feet as part of her hematologic cancer. As with any painful condition, the treatment of the underlying cause is the best option for amelioration and potential resolution of the pain. Unfortunately, treatments for managing the underlying disease often take time to become effective if they are to be effective at all, may exacerbate the underlying pain in the process of treating, and in the worst cases may unintentionally produce additional pain related to the treatment rather than the disease. Our

361

Chapter 50: Pain management in hematological cancer: clinical case illustrations

challenge in managing pain of hematologic cancers is to understand the pathophysiology of the disease and the mechanisms that produce the pain complaint, to identify the generators of the pain and to thoroughly consider the risks involved when treating so that we do no additional harm in the process of trying to help as unfortunately occurred for patient 4. The primary modalities for attempting to cure hematologic cancers have improved over the years and now offer, if not a cure, significant hope for a longer life expectancy with higher quality, but these modalities have a high complication rate. Chemotherapy and radiation are the current mainstays of treatment. Unfortunately, the most effective vinca-alkaloids, taxanes, and platinum-based drugs are potentially toxic to nerve and can produce chemically induced peripheral neuropathies. The risk of neutropenic fevers from chemotherapy, increased mortality, and prolonged hospitalization is higher for hematologic malignancy compared to solid tumors.[4] Thus, it is prudent to use the lowest effective dosing of neurotoxic agents to minimize the risk of neural injury. Additionally, the use of oral cryotherapy or palifermin may minimize oral mucositis in multiple myeloma patients receiving high-dose melphalan prior to autologous hematopoietic stem cell transplantation.[22] Radiation can not only directly injure components of both the central and peripheral nervous system, but can injure bone marrow stem cells[23] and can damage surrounding soft tissues resulting in scarring, altered function, and nociceptive pain.[13] Bone marrow transplantation (BMT) and stem cell transplantation (SCT) that are commonly used treatments for hematologic malignancy increase the risk for other hematologic malignancies and neutropenic fevers and graft-vs.-host rejection can occur after bone marrow transplantation. Beyond the already diminished immune vigilance, BMT and SCT place additional stress on a potentially compromised clotting system and carry increased risk of tissue injury and infection that will be slow to heal. Moreover, tumor lysis syndrome (TLS) that affects multiple organ systems may result from cytotoxic therapy.[24] TLS is most commonly observed in hematologic malignancy especially leukemia and non-Hodgkin lymphoma, but may also be observed in high-grade solid tumors of the breast, liver, esophagus, colon, and rectum. Laboratory evidence indicates that TLS occurs in approximately 42% of patients

362

with intermediate-high-grade non-Hodgkin lymphomas and in 70% of children with acute leukemia who receive induction chemotherapy.[25,26] Options for providing analgesic relief for patients throughout the natural evolution of hematologic malignancies depend upon and are limited by the underlying disease with its associated compromise in function as well as the treatments needed to stay the course of the disease. As with all chronic pain conditions, the treatment choices fall into four distinct, but not mutually exclusive groups: pharmacologic, interventional, physical, and psychologic that are best managed by a multidisciplinary team focused on improving the quality of life for the patient.[27,28] Clearly, the pain associated with hematologic cancers is of mixed type, involving both nociceptive and neuropathic components that may be ameliorated or augmented by the beliefs, fears, and expectations of the patient and his/her social network. The orchestration of treatments must be appropriately selected and timed for maximum benefit. Pharmacologic management should take into account that patients with chronic disease will require a baseline treatment to reduce the general level of discomfort as well as medication for rescue during periods of increased stress or activity or in anticipation of a pain exacerbating therapy or treatment. The best analgesic regimens that provide coverage for nociceptive pain include non-opioid (aspirin, acetaminophen, and non-steroidal anti-inflammatory drugs (NSAIDs)) and opioid agents as well as adjuvant medications for providing potential analgesic synergy for nociceptive pain and coverage for the neuropathic components of pain. Routine, on the clock, dosing of long-acting medications should be provided for managing baseline levels of chronic pain with additional immediate-release medications for breakthrough pain, as provided for patient 2, and for pre-emptive management of predictable pain generating events, e.g., biopsies, procedures, physical therapy. Immediate-release or short-acting formulations are also useful for determining tolerance to the medication and for dose titration. The best medications for managing nociceptive components of pain associated with hematologic cancers depend upon the intensity and the temporal aspects of the pain. Pain of low intensity, typically reported in the range of 1–3/10 on a numeric scale, can frequently be managed with non-opioid analgesics as seen in the early treatment of pain that patient 2 experienced following Ommaya

Chapter 50: Pain management in hematological cancer: clinical case illustrations

reservoir placement. Celecoxib, a selective cyclooxygenase-2 inhibitor, may provide the best option because it does not affect clotting time or bone healing. By contrast, aspirin and other NSAIDs should only be used with caution in the presence of hypo-coagulable states, renal insufficiency, and spontaneous fractures. Moderate pain levels of 4–6/10, as were later experienced by patient 2, are likely to require weak opioid medications, e.g., codeine, or tramadol. Piroxicam or tramadol may be preferable for managing the chronic components of pain because of their long half-life and availability in extended-release formulations, respectively, which aid in compliance. Opioid medications including codeine, hydrocodone, morphine, oxycodone, hydromorphone, levorphanol, methadone, oxymorphone, and fentanyl are the mainstay for managing severe nociceptive pain, 7–10/10 in intensity. The oral route of administration is preferred but physical restrictions related to the progression of disease or complications associated with treatment, as occurred in patient 2, may require consideration of alternate routes of administration, e.g., transdermal, sublingual, subcutaneous, rectal, intravenous, patient-controlled analgesia and intrathecal routes. Intramuscular injection(s) are typically painful, and should be avoided if possible. The transnasal route is not typically recommended due to the variability of delivery and that the only formulation that is FDA approved for delivery via this route, butorphanol, works by a mixed agonist/antagonist mechanism like nalbuphine, pentazocine, and buprenorphine. These medications have a therapeutic ceiling and carry a risk of precipitating withdrawal when used in patients chronically treated with pure µ-agonist medications, e.g., morphine. The use of meperidine should be avoided for chronic use due to its short duration of action which requires frequent dosing and can lead to central nervous system toxicity from the build up of the toxic, normeperidine metabolite. Patients may realize additional benefit from analgesic medications when used concomitantly with largely equally efficacious adjuvant medications. The choice of adjuvant medication is based on the type of pain, cost, ease of dosing, favorable side effect profile, and predictable adverse effects that may in certain situations be advantageous. Bisphosphonates, which work through their inhibitory effects on osteoclastic activity, are a good adjuvant choice for bone pain. Osteoclastic inhibition results in less acidic proton

release that can sensitize and lower the nociceptors’ threshold and enhance bone resorption. Not surprisingly, bisphosphonates have also been shown to reduce the incidence of pathologic vertebral fractures and slow the process of skeletal demineralization. Nevertheless, caution must be exercised when using these medications because up to 51% of patients taking bisphosphonates have been observed to adversely experience osteonecrosis of the jaw.[29] Antidepressants and anticonvulsants are excellent choices for treating neuropathic pain. Tricyclic antidepressants (TCAs) like amitriptyline and nortriptyline are relatively inexpensive and are commonly prescribed for neuropathic pain. TCAs have antihistaminergic effects that may help with comorbid insomnia and anti-cholinergic effects of smooth muscle relaxation that may help with visceral organ spasm associated with neuropathic pain. Although the frequency of QT prolongation associated with TCAs is not well defined, TCAs should be used with caution in patients with a history of arrhythmia. Low-dose TCAs are typically safe and well tolerated. Other classes of antidepressants, especially the SNRIs, have been shown to be effective for treating neuropathic pain with few adverse effects. Nausea is common in this class of medication, so SNRIs should be started at low dose with gradual titration as tolerated to achieve desirable anti-neuropathic effect. Although the use of adjuvant medications is important in providing optimum analgesic coverage for patients with hematologic malignancies, it is important to consider potential complications related to their use especially when used concomitantly in pharmacologic treatment regimens due to complex interactions associated with their roles as substrates, inhibitors, or inducers of the cytochrome P-450 system. Chemotherapeutic agents like paclitaxel, doxorubicin, cyclophosphamide and tamoxifen, SNRIs like duloxetine and venlafaxine, and TCAs like amitriptyline, nortriptyline and desipramine may act as substrates and as inhibitors of the cytochrome P-450 system effectively slowing the metabolism of opioids and adjuvant medications, thereby increasing their serum concentrations and prolonging the effect of a given dose of medication. By contrast, many of the first generation anticonvulsants like carbamazepine and phenytoin may act as inducers of the cytochrome P-450 system which leads to more rapid metabolism of substrates thereby potentially decreasing the therapeutic effect or increasing the rate of generation of

363

Chapter 50: Pain management in hematological cancer: clinical case illustrations

active metabolites resulting in more rapid onset of therapeutic action and the potentiation of both analgesic and toxic effects of medications provided. The second generation anticonvulsants like gabapentin and pregabalin are more commonly prescribed for neuropathic pain, as was done for patient 3, due to relative efficacy, favorable side effect profile, minimal involvement with the cytochrome P-450 system, and the lack of drug–drug interactions which simplifies their use in complex pharmacologic regimens.[30] Other adjuvant medications that may help with neuropathic pain include local anesthetics (i.e., lidocaine gel/patch), anti-spasmodics, cannabinoids, and NMDA antagonists. Local anesthetics should be used with caution when treating focal pain in patients taking class 1 anti-arrhythmic medications. Patient 3’s response to treatment with tizanidine illustrates the benefits of using anti-spasmodics like baclofen and tizanidine for reducing neuropathic pain by augmenting GABAergic inhibition in the dorsal horn or by decreasing the release of excitatory neurotransmitters from the terminals of primary afferent neurons, respectively. The sedating effects of anti-spasmodics may also be advantageous for managing restlessness and insomnia with less hangover side effects compared to TCAs.[15] Cannabinoids are another class of adjuvant pain medication that has generated much interest and controversy in the management of cancer pain. Specific agents that may target cannabinoid receptors (CB1 and CB1) to maximize analgesia and minimize central nervous system depressant effects are being investigated. Current evidence indicates that cannabinoids are moderately effective for treating neuropathic pain with comparable side effects to other neuropathic agents.[31,32] The use of NMDA antagonists like ketamine and memantine at subanesthetic doses may also be considered to manage persistent neuropathic pain 7/10 or greater that is not responsive to other adjuvant medications.[33,34] Ketamine may have limited clinical benefit and should be used with caution in combination with opioids and standard adjuvant medications due to increased toxicity and adverse effects.[35,36] More controlled studies with longer follow-up are warranted to elucidate the role of ketamine in persistent chronic cancer pain.[37] Interventional management is a major component of an effective pain armamentarium that frequently provides rapid and definitive treatment of a painful

364

condition. This aspect of pain management will be considered in more depth in another chapter of this book, but will be briefly addressed here for completeness with respect to managing pain related to hematologic cancers. The selection and timing of appropriate interventions is important for optimum care which includes minimizing cost and improving quality of life. Generally, interventions are offered for managing acute painful conditions or for providing comfort when undergoing painful therapeutic procedures. In managing chronic pain of non-malignant origin, the implementation of interventional procedures is considered more appropriate when more conservative measures have failed to provide benefit because the benefit of many procedures is of limited duration requiring multiple interventions each of which carries risk of complications, e.g., infection, hemorrhage, or additional injury. When dealing with pain associated with terminal disease, however, the early provision of definitive procedures in the management of chronic pain is likely to be more appropriate than the prolonged trials with conservative measures since patients with predictably short-term survival would be less likely to require repeated procedures and would benefit from permanent relief while obviating the need for potentially compromising medications. Vertebroplasty and kyphoplasty are often used in treating pain associated with vertebral compression fractures that characteristically occur in multiple myeloma. Early vertebral augmentation for managing vertebral compression fractures with severe pain or risk of progressive deformity is recommended. Although vertebroplasty and kyphoplasty carry the risks of infections and incidental injury associated with the extravasation of polymethyl methacrylate, the overall complication rate is low and largely operator dependent. Finally, the early utilization of modalities that address the physical and psychologic aspects of pain that accompany terminal disease is an essential consideration in the management of pain in patients with hematologic cancers. These treatment modalities that include physical, integrative, cognitive behavioral, and psychosocial interventions are an important part of the comprehensive approach to pain management. The consideration and implementation can be a major contributor to achieving improved quality of life for individuals like patient 4, and will be addressed in depth elsewhere in this book.

Chapter 50: Pain management in hematological cancer: clinical case illustrations

Conclusion A thorough understanding of the pathophysiology and natural history of hematologic neoplasias and how it relates to pain is essential for providing optimum care and improving the quality of life for patients who entrust their lives and well-being to our care. Proper assessment and recognition of relative risks, benefits, and limitations associated with our treatment options will go a long way toward reducing

References 1.

American Cancer Society. Cancer Facts & Figures. 2013. http://www.cancer.org/ research/cancerfactsfigures/ cancerfactsfigures/cancer-factsfigures-2013.

2.

Nolan CP, Abrey LE. Leptomeningeal metastases from leukemias and lymphomas. Cancer Treat Res. 2005;125:53–69. Review.

3.

ABCNews. 2005. http://abcnews. go.com/Health/PainManagement/ story?Id=732395&page=4.

4.

5.

6.

7.

Lyman HG, Lyman HC, Agboola O. Risk models for predicting chemotherapy-induced neutropenia. The Oncologist. 2005;10:427–437. Soares M, Feres AG, Salluh IFJ. Systemic inflammatory response syndrome and multiple organ dysfunction in patients with acute tumor lysis syndrome. Clinics. 2009;64(5):479–481. Larson AR, Pui CH. Tumor lysis syndrome: definition, pathogenesis, clinical manifestations, etiology and risk factors. UpToDate 2013. http:// www.uptodate.com/contents/ tumor-lysis-syndrome-definitionpathogenesis-clinicalmanifestations-etiology-and-riskfactors. Chakraborty A, Wells R, Doherty M, Huynh C, Selby D. Joint pain in AML: successful pain control with radiotherapy. J Pain

the suffering that so often accompanies this constellation of disease. The goal of blood cancer pain management is to improve the individual’s quality of life. Pain may not be prevented or eliminated but can usually be reduced to a manageable and tolerable level. Part of pain care is to educate the patients and their families about this reality so they can have realistic expectations for their treatment as they meet the challenge of their disease.

Symptom Manage. 2008;35(6): 670–672. 8.

Franchini M, Frattini F, Crestani S, Bonfanti C. Bleeding complications in patients with hematologic malignancies. Semin Thromb Hemost. 2013;39(1): 94–100.

16. Rosen JC, Silbermann R, Roodman DG. Hematologic malignancies and bone. In Rosen CJ, Bouillon R, Compston JE, Rosen V, eds. Primer on the Metabolic Bone Diseases and Disorders of Mineral Metabolism, 8th ed. Wiley-Blackwell. 2013: p. 84.

9.

McMahon B, Stein LB. Thrombotic and bleeding complications in classical myeloproliferative neoplasms. Semin Thromb Hemost. 2013;39 (1):101–111.

17. Kariyawasan CC, Hughes DA, Jayatillake MM, Mehta AB. Multiple myeloma: causes and consequences of delay in diagnosis. Q J Med. 2007;100: 635–664.

10. Elice F, Rodeghiero F. Hematologic malignancies and thrombosis. Thromb Res. 2012;129(3):360–366. 11. Nørgaard M. Risk of infections in adult patients with haematological malignancies. The Open Infectious Diseases Journal. 2012;6:46–51. 12. Kurz K, Garimorth K, Joannidis M, et al. Altered immune responses during septicaemia in patients suffering from haematological malignancies. Int J Immunopathol Pharmacol. 2012;25(1):147–156. 13. Niscola P, Tendas A, Giovannini M, et al. Pain in blood cancers. Indian Journal of Palliative Care. 2011;17(3):175–183. 14. Levy HM, Chwistek M, Mehta SR. Management of chronic pain in cancer survivors. The Cancer Journal. 2008;14:401–409. 15. Gould HJ, III. Management of painful neuropathies. Curr Treat Options Neurol. 2007;9:75–84.

18. Oranger A, Carbone C, Izzo M, Grano M. Cellular mechanisms of multiple myeloma bone disease. Clin Develop Immunol. 2013:1–12. 19. Milligan ED, Watkins LR. Pathological and protective roles of glia in chronic pain. Nat Rev Neurosci. 2009;10(1):23–36. 20. Terman GW, Bonica JJ. Spinal mechanisms and their modulation. In Bonica J, ed. The Management of Pain, 2nd ed. Philadelphia: Lea & Febiger. 2001: pp. 73–152. 21. Ballantyne CJ, Cousins JM, Giamberardino AM. Bone cancer pain. Pain Clinical Updates by International Association for the Study of Pain. 2009;17(2):1–6. 22. Kobbe G, Bruns I, Schroeder T, et al. A 3-day short course of palifermin before HDT reduces toxicity and need for supportive care after autologous blood stemcell transplantation in patients with multiple myeloma. Ann Oncol. 2010;21:1898–1904.

365

Chapter 50: Pain management in hematological cancer: clinical case illustrations

23. Hussein MA, Vrionis FD, Allison R, et al. The role of vertebral augmentation in multiple myeloma: International Myeloma Working Group Consensus Statement. Leukemia. 2008:1–6. 24. McBride A, Westervelt P. Recognizing and managing the expanded risk of tumor lysis syndrome in hematologic and solid malignancies. J Hematol Oncol. 2012;5:75–85. 25. Kelly KM, Lange B. Oncologic emergencies. Pediatr Clin North Am. 1997;44(4):809–830. 26. Hande KR, Garrow GC. Acute tumor lysis syndrome in patients with high-grade non-Hodgkin’s lymphoma. Am J Med. 1993;94 (2):133–139. 27. Geeta MG, Geetha P, Ajithkumar VT, et al. Management of pain in leukemic children using the WHO analgesic ladder. Ind J Pediatr. 2010;77:665–668. 28. Niscola P, Cartoni C, Romani C, et al. Epidemiology, features and outcome of pain in patients with

366

advanced hematological malignancies followed in a home care program: An Italian survey. Ann Hematol. 2007;86:671–676. 29. Mhaskar R, Redzepovic J, Wheatley K, et al. Bisphosphonates in multiple myeloma: a network metaanalysis (review). The Cochrane Collaboration. 2012:5. 30. Smith SH. Opioid metabolism. Mayo Clin Proc. 2009;84(7): 613–624. 31. Ashton JC, Milligan ED. Cannabinoids for the treatment of neuropathic pain: clinical evidence. Curr Opin Invest Drugs. 2008;9:65–75. 32. Wang T, Collet JP, Shapiro S, Ware MA. Adverse effects of medical cannabinoids: a systematic review. Can Med Assoc J. 2008;178:1669–1678. 33. Leppert W. The role of ketamine in the management of neuropathic cancer pain: a Polish experience. In Wells CD, ed. Proceedings of the 3rd

International Congress on Neuropathic Pain, May 2010. Athens, Greece: NeuPSIG. 2010: pp. 27–30. 34. Kotlin’ ska-Lemieszek A, Luczak J. Subanesthetic ketamine: an essential adjuvant for intractable cancer pain. J Pain Symptom Manage. 2004;28:100–102. 35. Hardy J, Quinn S, Fazekas B, et al. Randomized, double-blind, placebo-controlled study to assess the efficacy and toxicity of subcutaneous ketamine in the management of cancer pain. J Clin Oncol. 2012;30: 3611–3617. 36. Salas S, Frasca M, PlanchetBarraud B, et al. Ketamine analgesic effect by continuous intravenous infusion in refractory cancer pain: considerations about the clinical research in palliative care. J Palliative Med. 2012;15:287–293. 37. Leppert W. Ketamine in the management of cancer pain. J Clin Oncol. 2013;31:1374.

Section 6 Chapter

51

Cancer Pain

Patient with metastatic breast cancer who had a mastectomy complicated by lymphedema Arash Asher and Jack Fu

Case study A 58-year-old female carries a diagnosis of left-sided, metastatic, hormone receptor negative breast cancer. She was originally diagnosed 3 years ago with local disease and it was treated with a left modified radical mastectomy, axillary dissection, and chemotherapy including Taxotere, carboplatin, and Herceptin. Two years after her diagnosis, she was found to have metastatic disease to the bone. She presented to her oncologist 6 months ago with swelling in the left upper extremity and pain in the left shoulder. She endorses a sense of “heaviness” of the arm and difficulty with certain activities of daily living such as washing her face, reaching overhead, and sleeping on the left arm. With physical examination, she has diffuse nonpitting edema of the left upper extremity, including the digits. She has myofascial pain along the left scapular muscles and pectoral muscles on the left side. She can only abduct and flex the left shoulder to about 80 degrees, both actively and passively. She is also significantly concerned about the cosmetic appearance of her arm as well as the emotional distress she is now experiencing.

1. What is the prevalence of upperbody morbidity and lymphedema after breast cancer? With advances in breast cancer treatment, such as the sentinel node biopsy and more refined radiation techniques, upper-body symptoms have decreased in severity but are still very common and significantly impact function and quality of life. Upper-body symptoms after breast cancer treatment include: weakness, stiffness, swelling (lymphedema), paresthesias, pain, and poor range of motion. At least 10% but

as many as 60% of women report at least one upperbody symptom from 6 months to 3 years after breast cancer surgery.[1] Another study found that greater than 50% of breast cancer survivors report these symptoms. Prevalence of pain ranges from 12% to 51%.[2] Therefore, it is clear that pain and other symptoms remain very common well after the treatment period. Lymphedema can occur after any cancer or treatment that affects the lymphatic system. The overall incidence of lymphedema after breast cancer can range from 8% to 56% at 2 years after surgery.[3] Eighty percent of patients experience onset within 3 years of surgery; the remainder develop lymphedema at a rate of 1% per year.[4]

2. What are known risk factors for the development or exacerbation of upperbody morbidity and lymphedema after breast cancer? The more invasive the treatment, the greater likelihood of morbidity and pain syndromes after breast cancer. For example, an axillary node dissection carries more risk than a sentinel node biopsy. Breast conserving surgery carries less risk than a simple mastectomy and a modified radical mastectomy. Radiation also confers risks: radiation to the breast and chest wall and regional nodes is associated with higher morbidity than the breast/chest wall alone. Patients undergoing axillary surgery and/or axillary radiation therapy are at higher risk for developing lymphedema. In addition, the extent of lymph node dissection increases the risk of developing lymphedema. For this reason, sentinel lymph node dissection has gained favor over axillary lymph node

Case Studies in Pain Management, ed. Alan David Kaye and Rinoo V. Shah. Published by Cambridge University Press. © Cambridge University Press 2015.

367

Chapter 51: Patient with metastatic breast cancer who had a mastectomy complicated by lymphedema

dissection because of decreased morbidity. In addition, obesity may be a risk factor for the development of lymphedema.[4] Other risk factors include the extent of local surgery, local radiation, delayed wound healing, a tumor that causes lymphatic obstruction, or scarring of the lymphatic ducts by surgery or radiation. Unlike previous contentions, moderate exercise or appropriate upper extremity strengthening regimens do NOT increase the risk of developing or exacerbating lymphedema after breast cancer. In fact, these interventions have been shown to improve quality of life and functional status after breast cancer treatment.[5] Patients with and at risk for lymphedema should be evaluated by a certified lymphedema therapist to develop an exercise program and routine that is appropriate for each breast cancer survivor. Furthermore, patients who do have lymphedema should wear a well-fitted compression garment during exercise. Patients without lymphedema do not need to wear a garment while doing exercise with the at-risk limb.

3. What is the differential diagnosis for pain syndromes after breast cancer?         

Lymphedema Neuropathic pain, including from injury to the intercostal brachial or thoracodorsal nerve with axillary lymph node dissection Radiation-induced fibrosis syndrome Postmastectomy phantom pain syndrome Myofascial pain disorder Axillary web syndrome Adhesive capsulitis (frozen shoulder) Postmastectomy pain syndrome (non-specific) Donor site morbidity (i.e., transverse rectus abdominis and latissimus dorsi)

4. What are some clinical manifestations of lymphedema after breast cancer? The onset of lymphedema after breast cancer is typically insidious. However, it can be triggered by local inflammation such as from an infection, burn, or injury to the involved extremity. Lymphedema is often characterized by stages:  Stage 1 – characterized by pitting edema.  Stage 2 – fibrotic changes occur with time.

368

 Stage 3 – Classically, lymphedema is characterized by non-pitting edema of an extremity, usually with involvement of the fingers. However, the distribution of the swelling may be limited to the proximal or distal portion of the extremity. Left untreated, lymphedema leads to chronic inflammation, hardening due to fibrotic skin changes, and at risk for more frequent infections. Patients with lymphedema may report a wide variety of symptoms such as heaviness, fullness, a tight sensation of the skin, or decreased flexibility of the affected joint. Arm lymphedema can cause difficulty with putting on previously well-fitting bracelets, rings, or watches. Range of motion and upper-body dressing, such as putting on a bra, can be more problematic as well as activities of daily living. In addition, lymphedema can cause significant psychologic distress to breast cancer patients due to changes in body image as well as the physical reminder of the cancer experience.

5. How is lymphedema diagnosed? Since lymphedema is progressive and early diagnosis leads to more effective treatment, early diagnosis is critical. The diagnosis relies upon: 1. History and physical exam. The history is usually more obvious when there is swelling in the involved extremity after a history of cancer and/or radiation involving that extremity. Other causes of limb swelling, such as a deep venous thrombosis, infection, or malignancy (e.g., superior vena cava compression), should be excluded. Also, diffuse swelling throughout the body should lead to evaluation for other causes such as hypoalbuminemia. Lymphedema is usually characterized by non-pitting edema, usually with involvement of the digits. 2. Circumferential upper extremity measurements. This is the most widely used method to diagnose upper extremity lymphedema. Tape measures are taken at defined intervals on both arms: the metacarpal-phalangeal joints, the wrist, 10 cm distal to the lateral epicondyles, and 15 cm proximal to the lateral epicondyles. This technique is accurate if it is done in precisely the same way each time. Differences of 2 cm or more at any point compared with the contralateral arm may be clinically significant. However, factors such as

Chapter 51: Patient with metastatic breast cancer who had a mastectomy complicated by lymphedema

obesity need to be taken into account, as a 2 cm difference in a very thin patient may have more relevance than a 2 cm difference in a very obese patient. Moreover, differences in muscle mass between the dominant and non-dominant arm need to be considered. Therefore, sequential measurements over time may be most meaningful clinically. 3. Lymphoscintigraphy. Occasionally, when the diagnosis is not clear based on the history and examination, imaging of the lymphatic system is indicated. Lymphoscintigraphy is a nuclear medicine study used for imaging lymph vessels and lymph nodes. Radiolabeled particles of protein are injected just under the skin of the region of the body to be imaged. It can demonstrate slow or absent lymph flow and areas of reflux. Radiologists who read and perform these studies need significant experience with lymphoscintigraphy for the diagnosis of lymphedema. 4. Other less frequently used or evolving methods for assessing lymphedema include near infrared fluorescence imaging, water displacement, and bioelectrical impedance measures. These techniques are beyond the scope of this chapter. Serial measurements of affected limbs using circumferential measurements, perometers, or water displacement assess the effectiveness of lymphedema treatment.

6. For patients at risk for lymphedema who do not have the condition, how should they be educated regarding preventive care? Patients at risk for developing lymphedema should be educated regarding the condition because they may recognize the early signs of lymphedema and seek treatment earlier. In addition, there are many anecdotal recommendations for preventive measures:  Skin and nail care: meticulous skin hygiene, use of skin moisturizers and topical antibiotics after small breaks in the skin, use of gloves for gardening and cooking, and awareness of signs of infection.  Upper extremity positioning: avoiding constrictive pressure on the affected arm.

7. What are the components of the “gold standard” treatment regimen for established lymphedema? The goal of lymphedema therapy is to control swelling and minimize complications. There are no cures for lymphedema and pharmacologic options (such as diuretics) are generally NOT helpful. Therefore, nonpharmacologic interventions are the mainstay of treatment. Complex decongestive therapy (CDT) is the most common form of treatment for lymphedema.[6] It has been shown to decrease swelling, reduce skin fibrosis, enhance quality of life, and reduce the risk of cellulitis.[7] It includes two phases:  Phase I: The main goal is to reduce the size of the limb and improve the quality of the skin. It is ideally performed 5 days/week until the reduction of the volume has reached a plateau. This can take several weeks and consists of the following aspects:

: :

: :

:

Manual lymph drainage: a manual technique to stimulate the superficial lymphatic vessels in order to bypass the areas of lymphatic congestion. Compression bandaging: multiple layers of inelastic material to discourage reaccumulation of lymph fluid. Typically, short-stretch bandages are used to create an effective compression gradient and also reduce fibrosis. Compression garments: If properly fitted, these can significantly enhance mobilization of edema fluid. Garment style and compression strength is customized to the patients’ needs. Lymphatic exercise: non-fatiguing exercises may mobilize some of the lymph fluid via muscle contractions. Aerobic exercise may also increase the tone of the sympathetic nervous system and allow the lymphatic system to pump more effectively. Skin care: the goal is to minimize dermal colonization and hydrate the skin to control dryness.

 Phase II:

: :

The patient is educated on a self-maintenance home program. The home program includes self-massage, lymphatic exercises, a skin care regimen, and

369

Chapter 51: Patient with metastatic breast cancer who had a mastectomy complicated by lymphedema

compression garments or bandages that are applied at home. The garments need to be replaced every 4–6 months.

8. What other treatments can be considered for lymphedema? Intermittent external pneumatic compression Also known as compression pump therapy, this can be useful as an adjunct in select patients for Phase I treatment of lymphedema or in some situations as a necessary component of Phase II treatment. Recommended pump pressures generally range from 30 to 60 mmHg and the length of treatment is typically about 1 hour. To maintain the reduction in volume, a compression garment or short-stretch bandages will need to be used between pump treatments. Potential contraindications for the pump include active infection, severe peripheral arterial disease, deep venous thrombosis, and recurrent cancer in the affected area.

Weight loss The risk for developing lymphedema increases with obesity. In addition, small randomized trials have shown that weight loss may improve breast cancerrelated lymphedema.[8] Therefore, weight loss should be part of the overall treatment regimen for overweight individuals with lymphedema.

Low-level laser therapy Preliminary studies have shown a benefit for low-level laser therapy in reducing lymphedema. Laser therapy may do this by decreasing fibrosis, possibly encouraging lymphangiogenesis, or other mechanisms.[9] More research is needed.

Surgery Surgery is rarely performed on patients with cancerrelated lymphedema. Surgical options include: microsurgical lymphaticovenous anastomoses, liposuction, and fasciotomy. Usually these are only considered when adequate trials of all usual methods of treatment have failed. Very few surgeons have experience with these procedures for lymphedema. Patients need to be referred to specialized centers for this.

370

9. What rare complications of lymphedema should practitioners be aware of? If a patient with a history of breast cancer develops overwhelming lymphedema many years after the primary surgery, other diagnosis should be considered. Firstly, the recurrence of the tumor needs to be excluded. In addition, the development of lymphangiosarcoma needs to be considered. This is classically seen many years after a mastectomy related to breast cancer (Stewart–Treves syndrome). On physical examination, the lesions appear as blue-red or purple with a macular or papular shape in the skin. The mean time between mastectomy and lymphangiosarcoma is 10.2 years with a median survival of 1.3 years.

10. Conclusions – how would you treat this patient in the vignette? This 58-year-old patient with metastatic breast cancer has the following pain management and rehabilitative needs:  Left upper extremity lymphedema  Physical exam evidence of adhesive capsulitis  Myofascial pain syndrome after mastectomy in the surrounding shoulder musculature I would first educate the patient about lymphedema since self-management is very important. This would include issues related to skin hygiene, avoiding trauma, burns, and tourniquet effects, and counseling regarding how this is a lifelong but manageable condition. I would then refer the patient to a physical or occupational therapist who is certified in lymphedema care and ideally has experience with breast cancer patients. I would want the patient to start with complex decongestive therapy to mobilize the lymph fluid and volume standpoint stabilization on either a customized compression sleeve and/or short-stretch bandages that she is taught to apply on her own. Once her lymphedema is addressed, the therapist can begin treating the pain and range of motion issues. I would typically begin with “myofascial release techniques,” gentle repeated stretching of the involved muscle groups in the pectoral and scapular muscles, and sustained trigger point compression. Often times, breast cancer patients develop scapular

Chapter 51: Patient with metastatic breast cancer who had a mastectomy complicated by lymphedema

protraction due to anterior chest wall tightness which needs to be addressed with physical therapy. If this is inadequate in controlling the pain, then I would consider a variety of injections to the areas of muscle pain including trigger point injections with a local anesthetic. Botulinum toxin injection can be considered

References 1.

2.

3.

4.

Hayes SC, Johansson K, Stout NL, et al. Upper-body morbidity after breast cancer. Cancer. 2012;118 (S8):2237–2249. Gartner R, Jensen MB, Nielsen J, et al. Prevalence of and factors associated with persistent pain following breast cancer surgery. JAMA. 2009;302(18):1985–1992. Paskett ED, Naughton MJ, McCoy TP, Case LD, Abbott JM. The epidemiology of arm and hand swelling in premenopausal breast cancer survivors. Cancer Epidemiol Biomarkers Prev. 2007;16(4):775–782. Petrek JA, Senie RT, Peters M, Rosen PP. Lymphedema in a

for patients who do not respond to the above regimens.[10] Finally, as the pain improves, then the patient can progress with physical therapy, addressing the range of motion restrictions and gentle strengthening exercises of the shoulder.

cohort of breast carcinoma survivors 20 years after diagnosis. Cancer. 2001;92(6):1368–1377.

Surg Oncol. 2007;14(6): 1904–1908. 8.

Shaw C, Mortimer P, Judd PA. Randomized controlled trial comparing a low-fat diet with a weight-reduction diet in breast cancer-related lymphedema. Cancer. 2007;109(10): 1949–1956.

Korpan MI, Crevenna R, FialkaMoser V. Lymphedema: a therapeutic approach in the treatment and rehabilitation of cancer patients. Am J Phys Med Rehabil. 2011;90(5 Suppl 1): S69–75.

9.

Carati CJ, Anderson SN, Gannon BJ, Piller NB. Treatment of postmastectomy lymphedema with low-level laser therapy: a double blind, placebo-controlled trial. Cancer. 2003;98(6): 1114–1122.

Hamner JB, Fleming MD. Lymphedema therapy reduces the volume of edema and pain in patients with breast cancer. Ann

10. Cheville AL, Tchou J. Barriers to rehabilitation following surgery for primary breast cancer. J Surg Oncol. 2007;95(5):409–418.

5.

Schmitz KH, Ahmed RL, Troxel AB, et al. Weight lifting for women at risk for breast cancerrelated lymphedema: a randomized trial. JAMA. 2010;304(24): 2699–2705.

6.

7.

371

Section 7 Chapter

52

Special Topics

A 57-year-old male with chronic pain syndrome, anxiety disorder, and hypertension is seeking mental health counseling Natacha Telusca and Kingsuk Ganguly

Why is it important to address the risk of suicide? Patients with chronic pain have a substantially higher risk of suicidal ideation and behavior than the general population. A systematic review by Tang and Crane reported that the risk of death by suicide in chronic pain patients is at least double the rate in the general population.[1] Chronic pain patients have a lifetime prevalence of suicide attempt ranging between 5% and 14%, and the prevalence of suicidal ideation is about 20%.[1] They identified several risk factors for suicidality in chronic pain, the type, intensity, and duration of pain and sleep-onset insomnia cooccurring with pain. They also concluded that helplessness and hopelessness about pain, the desire for escape from pain, pain catastrophizing and avoidance, and problem-solving deficits are important psychologic processes to the understanding of suicidality in chronic pain.[1] Edwards et al highlighted the magnitude of depressive symptoms and the degree of painrelated catastrophizing, a maladaptive cognitive/emotional pain-coping strategy, as the most consistent predictors of the presence and degree of suicidal ideation.[2] Psychiatric disease is a well-known risk factor for suicide; a great number of the chronic pain patient population suffers from comorbid psychiatric disorders. The greatest risk factors for suicidality are major depressive disorders, dysthemia disorders, and substance use disorders.[3] Drug overdose is the most commonly reported plan and method of suicide attempt.[4] This finding highlights the importance

of routine evaluation and monitoring of suicidal behavior in chronic pain particularly for patients with family histories of suicide, and those taking a high dose of opioid medications.[4] Antidepressant medications such as tricyclic antidepressants (TCAs), SSRIs, and serotoninnorepinephrine reuptake inhibitors (SNRIs) are used as adjuvants treatment for pain, and have been shown to be effective. When using these medications in chronic patients with comorbid mental illness such as schizophrenia, affective disorders, and bipolar, it is very important to use a multidisciplinary approach involving the the primary psychiatrist to help prevent adverse effects.

The patient doesn’t demonstrate any suicidal thoughts, has a good support system at home, and doesn’t have any concrete executable plans for self harm. You now feel comfortable discussing more conventional mental health strategies. What do you recommend? Cognitive behavioral therapy Cognitive behavioral therapy, commonly known as CBT, has been a clinically effective psychotherapy for a variety of disorders. CBT initially requires a therapist to help a patient with chronic pain syndrome identify his maladaptive beliefs that have led to dysfunctional behavior. Upon identification,

Case Studies in Pain Management, ed. Alan David Kaye and Rinoo V. Shah. Published by Cambridge University Press. © Cambridge University Press 2015.

373

Chapter 52: Chronic pain syndrome, anxiety disorder, and hypertension

behavioral therapy is then initiated to help the patient modify these maladaptive beliefs, with the goal of causing a permanent emotional and behavioral change.[5] The hope is that these changes can then lead to symptom reduction. In order for CBT to be effective, the patient must be self-motivated to undergo therapy, and then incorporate what he learns to be able to practice these skills outside of the clinical environment. Thus, these patients must be highly motivated and have critical thinking skills that allow them to understand what their distressing thoughts are.[5] CBT requires active collaboration with the therapist. Through this teamwork, the patient has to be able to identify what his problematic behaviors are and how they are precipitated. The patient can then adapt a more positive approach to his thinking of managing the symptoms of chronic pain. At the outset, it is appropriate for the therapist to take the lead in sessions to help the patient identify and modify distressing thoughts with the patient eventually assuming more control as therapy sessions progress. It is important for the patient to recognize his problems and establish goals to work toward. With goals set in mind, the patient can then focus on understanding how his thoughts can affect his emotion and behavior. This all may be overwhelming in the beginning, so the therapist is there to help the patient develop these skills in structured sessions. CBT sessions are intended to be time limited with the eventual goal that the patient can independently take these learned skills to avoid relapses.[5] The process of patients identifying cognitive processes that cause distress or changes in behavior often begin with guided discovery.[5] This process encourages patients to think about their behavior by asking questions to help discover their irrational thought patterns. Therapists also create behavioral experiments to test patients’ thoughts and beliefs and to assess the validity of those beliefs, with the goal being that the experiments can help dispel notions patients have of themselves. Meta-analyses of CBT in chronic pain patients have shown it to improve patients’ ability to cope with pain as well as reduce behavioral expression of pain.[6,7] A trustworthy relationship needs to be developed between the counselor and the patient in order for CBT to be successful. The counselor can then effectively describe the treatment plan to the patient in order to help the patient with alleviating

374

the stressors to his pain. If the patient accepts this feedback, he can then begin looking inward via the process of guided discovery and realize what his dysfunctional thoughts are. Experiments can be created by the counselor and patient to then discredit these beliefs. These automatic thoughts are constantly evaluated in multiple sessions so that the patient can eventually assess and develop goals to treat them. It is a treatment process that may require at least 2–4 months before positive results can be seen, although each patient must be evaluated clinically to see if results come sooner or later.[5] Thus, for the challenging patient with chronic pain, the results are usually not seen immediately. The patient has to be eventually astute enough to understand how his thoughts may affect his emotions, and play a role in his chronic pain syndrome. It is this cognitive model that the counselor must press upon the patient in order to help alleviate his symptoms. Counselors can also give patients assignments to take home and work on individually. These assignments are meant to help the patient develop self-improvement skills. With this multifaceted approach, the goal is for the patient to be able to problem solve through negative thoughts, deem them as invalid as well as learn to accept certain problems that may never be solved, and think of them in a different light.[5] The modification of expectations is an important skill that can be learned in CBT. As beneficial as CBT may be, the exposure of patients to negative situations may provoke anxiety. This may be relatively contraindicated in those with coexisting conditions such as dementia, or thought disorders such as schizophrenia. Furthermore, the stress of interpreting some thoughts may need to be approached at a more gradual level for patients with existing cardiac disease or hypertension.[5]

Relaxation training Relaxation training is a method used in stress management therapy to help one achieve a state of reduced psychologic tension. This can include, but is not limited to, meditation, yoga, tai chi, and other exercises that also incorporate rhythmic breathing. The underlying belief in this form of therapy is that the mind must relax in conjunction with the body.[8] Stressors, such as a chronic pain a patient experiences while performing a routine activity, will induce tension. This tension is not restricted to the source of

Chapter 52: Chronic pain syndrome, anxiety disorder, and hypertension

pain, and can be reproduced throughout one’s entire skeletal system. If one can focus on the relaxation of these muscles, this relaxation can then be transmitted to the neurologic system. As the patient continues to improve control over his relaxation therapy, he can be successful in preventing another acutely stressful event after a symptom flare up. Patients begin to learn by first recognizing the innate ability to contract and relax one’s muscles. The environment should be in a quiet area with no interruptions. Minimal external stimuli are critical. Movement, laughter, and talking should be prevented as much as possible. The patient then goes through a period of “progressive relaxation” by learning to focus on one muscle group of the body at a time. An hour’s worth of time should be dedicated to this. By developing control over one’s body, sympathetic discharge reduces, thus leading to a state of higher relaxation. In the beginning, it is appropriate for the therapist to voice the areas of the body that the patient can relax, but eventually the patient should be able to do this independently. In regards to mental concentration, the patient should not focus on his pain symptoms and whether or not they will improve. Rather, they should take a meditative approach and just allow the mind to come to a standstill.[8] Some patients may experience “relaxationinduced anxiety.” The initial stages of recognition of tense events may trigger this. The therapist should try to divert the patient’s mind with the use of pleasant imagery and “thought switching.” This approach may help the patient focus on more positive imageries.[8] Furthermore, relaxation training is not intended to be short or even temporary. In fact, the most beneficial thing would be for the patient to incorporate some form of relaxation therapy to his daily regimen. For some, the lack of instantaneous results can be frustrating. On average, it takes patients at least 3 months to be comfortable with learning these techniques. Thus, one must be self-disciplined when beginning. Also in the initial stages, patients may misinterpret relaxation therapy and make actual efforts to relax. For example, the patient may attempt to actively keep the leg down on the floor rather than let it hang loosely. This effort will actually increase tension. Chronic pain patients with neurotic disorders who may hold disorganized or distracted thoughts may be much more difficult to direct for relaxation training.

The focus on muscle groups can lead to cramping, which will obviously impede relaxation. Furthermore, it is not uncommon for one to feel muscle twitches as the muscles proceed to relaxation. This is a transient discomfort that should be addressed with reassurance.[8]

Psychoanalysis Psychoanalysis is a form of therapy based on principles of human behavior, experience, development, and motivation. This foundation is based on the belief that an individual’s unconscious thoughts can drive his actions and hence, affect his emotions and overall well-being.[9] Psychotherapy can be used as an excellent tool for chronic pain patients who have strong psychologic underpinnings for their pain disorder. Like the previously mentioned therapies, a strong relationship between the therapist and patient can help the patient uncover the underlying sources of his psychologic strains. Perhaps a traumatic or embarrassing event in the patient’s life that is associated with his chronic pain may hamper his ability to adequately cope with his disease. If the patient can re-experience these with the assistance of a therapist, it can help relieve some of the stress associated with his disease. By retracing the past, the patient can then help modify his behavior patterns after identifying psychologic sources of the pain. Psychoanalysis is very intensive, as it typically requires the patient to come in at least four times a week, lying on a couch, and openly expressing whatever is running through his mind. Unlike the other forms of therapy, the patient will not be sitting face to face with the therapist. By lying on a couch and facing away from the therapist, it is believed that this helps the patient express his feelings more openly rather than be concerned about the therapist’s reactions.[9] Psychoanalysis is a long process, and treatment can last for several months. Patients will commonly feel frustration, stress, and hindrances along the way as it is understandably difficult to bring up painful thoughts from the past that have often been hidden away for years. Many mental processes occur for the patient as well as the clinician. As unconscious thoughts are conjured up from the patient’s past, he may develop transference. Transference is the unconscious displacement of feelings toward the therapist that are associated with a person from the patient’s

375

Chapter 52: Chronic pain syndrome, anxiety disorder, and hypertension

past. The therapist’s behavior may elicit similarities from individuals that have affected or continue to affect the patient’s well-being. The therapist can develop countertransference, which are feelings about an individual from the therapist’s past that are displaced on to the current patient. This can help the therapist realize how other individuals may react to the patient’s chronic pain syndrome and how this adversely affects the patient. The stress of psychoanalysis will bring up defense mechanisms in the patient to help cope. Resistance is another unconscious process that can be encountered. For example, the patient may refuse to talk about certain painful topics or begin to skip therapy because of the uncomfortable sessions.[9] For a patient to be able and willing to open up about painful events in his past, mutual collaboration between the therapist and patient is a necessity. With a strong alliance, the therapist can help the patient increase his insight into his unconscious processes by validating his subjective experiences. The legitimization of past repressed experiences can subsequently help a patient move forward. As the therapist facilitates the transition of unconscious motives into consciousness, the patient can then recognize the reasons behind his thoughts, emotions, and behaviors, and begin to make changes. Successful treatment occurs when patients are insightful enough to be able to examine themselves independently.[9]

Psychiatric approach The psychiatric approach is a broad terminology that falls under the branch of medicine. The treatment is performed by a physician who is licensed in the medical specialty of psychiatry. A psychiatrist diagnoses and treats mental disorders by counseling and pharmacology. The psychiatric approach is based on basic science as well as cognitive psychology. As with the therapies previously described, an effective doctorpatient relationship in a comfortable environment is essential for the psychiatric approach to be successful.[10] A thorough history, including past medical and psychiatric, and physical examination are performed by the physician. After this initial examination, a clinical plan is created where the patient’s biological, social, psychological, and spiritual factors are all taken into account. With this information, the physician can develop a differential diagnosis and develop a treatment plan with multiple modalities.[11] This can include the meditation therapies described in this text,

376

CBT, psychoanalysis, relaxation training, and medication therapies to help manage any neurotransmitter imbalances. Pharmacologic therapies can include, but are not limited to: antidepressants such as SSRIs, anticonvulsants, and benzodiazepines.[11] As treatment progresses and outcomes are seen, it is acceptable for the differential diagnosis and treatment patterns to be adjusted. Obstacles will undoubtedly arise during the treatment process. The patient may be unwilling to take even more medications than what he is already on for his chronic pain, or he may find the frequency of his appointments to be burdensome. These challenges should be openly discussed with the psychiatrist so that a mutually designed treatment program can be achieved as treatment adherence is essential to see any benefit.[11] As a physician, psychiatrists can also order laboratory and diagnostic tests to aid in management. These can be used to help check serum levels of medications, to rule out metabolic diseases, or to obtain neuroimaging studies to evaluate any organic causes for psychiatric dysfunction. These multiple tools give psychiatrists a broad approach to manage mental health disorders.[11]

During this discussion, the patient presents a newspaper article touting the benefits of biofeedback. He says that one of his friends mentioned hypnosis. Can you explain these modalities to the patient? What is biofeedback? Biofeedback is a form of self-regulatory therapy in which individuals utilize specific instruments to measure and identify physical manifestations of their pain. The ultimate goal is to enable them to gain control over the specific pain response and in turn regulate that response in a more manageable way. This form of feedback can be very detailed, providing the individual with specific information about physiologic responses of which they are typically unaware, allowing them the opportunity to learn voluntary control over these unwanted responses.[12] Biofeedback for pain management usually involves feedback in the form of muscle tension measured at the site of pain or a standard location (e.g., frontalis muscles) or skin temperature. Feedback is measured

Chapter 52: Chronic pain syndrome, anxiety disorder, and hypertension

using electromyographic (EMG machines) for muscle tension and thermistors attached to the fingers to register temperature changes.[12] Changes in muscle tension and peripheral temperature are thought to be indications of sympathetic recruitment and stimulation. Using the above-mentioned instruments, patients learn to control these undesired stimuli and improve relaxation.[13] Efficacy for biofeedback has been proven for several painful conditions including tension and migraine headaches, low back pain, Raynaud’s phenomenon, as well as gynecologic pain conditions such as vulvar vestibulitis.[12,13] One limit of biofeedback relaxation techniques is that they require the assistance of a therapist in the initial sessions while the individual learns to recognize physiologic signals associated with their pain.[13]

What is hypnosis? Hypnosis for pain management is a technique that assists with improving the individual’s control over their pain symptoms. The goal of hypnosis is obtaining a state of deep relaxation and decreased awareness of pain signals. It typically begins with an induction phase comprising relaxing suggestions and attempts to obtain the individual’s focused attention. This phase is followed by a suggestion phase which includes pleasant imagery and metaphors to alter how their pain is perceived or experienced. Finally, a termination phase concludes the hypnotic session and promotes the suggestion that the pain-free state they achieved will persist after the session has ended.[14]

References

Inpatient versus outpatient psychologic therapy? While several modalities exist for outpatient evaluation and treatment of chronic pain conditions, there is little data to assist clinicians regarding the appropriateness of inpatient therapy. Inpatient programs provide intensive, structured medical therapy as well as monitoring and targeted therapy for various health behaviors. Patients who may be appropriate candidates are those who require detoxification, have a chronic pain condition lasting at least 6 months, have significant loss of function due to their pain, or require intensive psychologic therapy. Specifically, inpatient therapy seems to be more suitable for those who may benefit from a controlled environment in which they can focus on learning to manage their pain triggers and symptoms without the distractions of their daily home life, particularly if they have failed outpatient treatments.[18]

study. J Affect Disord. 2007; 101:27–34.

1.

Tang NK, Crane C. Suicidality in chronic pain: a review of the prevalence, risk factors and psychological links. Psychol Med. 2006:36(5)575–586.

2.

Edwards RR, Smith MT, Kudel I, Haythornthwaite J. Pain-related catastrophizing as a risk factor for suicidal ideation in chronic pain. Pain. 2006;126(1–3): 272–279.

5.

Bernal M, Haro JM, Bernert S, et al. Risk factors for suicidality in Europe: results from the ESEMED

6.

3.

When this type of therapy is initiated, assistance from a therapist is typically required. The ultimate goal is for the individual to master self-hypnosis and utilize their skills throughout the day, whenever they experience pain symptoms.[14] Hypnosis has been extensively studied and successfully implemented in malignancy-related pain.[14,15] There is also data suggesting its benefit in managing pediatric headache conditions, irritable bowel syndrome, tension headaches, and temporomandibular joint disorders.[16] There is also emerging evidence for its use in HIV neuropathic pain.[17]

4.

Smith MT, Edwards RR, Robinson RC, Dworkin RH. Suicidal ideation, plans, and attempts in chronic pain patients: factors associated with increased risk. Pain. 2004;111(1–2): 201–208. Beck JS. Cognitive Behavior Therapy Basics and Beyond, 2nd ed. New York: The Guilford Press. 2011. Morley S, Eccleston C, Williams A. Systematic review and meta-

7.

8.

analysis of randomized controlled trials of cognitive behaviour therapy and behaviour therapy for chronic pain in adults, excluding headache. Pain. 1999; 80:1–13. Butler AC, Chapman JE, Forman EM, Beck AT. The empirical status of cognitive behavioral therapy: A review of metaanalyses. Clin Psychol Rev. 2006;26:17–31. Lehrer PM, Woolfolk RL, Sime WE. Principles and Practice of Stress Management, 3rd ed. New York: The Guilford Press. 2007.

377

Chapter 52: Chronic pain syndrome, anxiety disorder, and hypertension

9.

American Psychoanalytic Association. About Psychoanalysis. 2009. http://www. apsa.org/About_Psychoanalysis. aspx (accessed June 18, 2013).

10. Sadock BJ, Sadock VA, Ruiz P. Kaplan & Sadock’s Comprehensive Textbook of Psychiatry, 9th ed. Philadelphia: Lippincott Williams & Wilkins. 2009. 11. Stern TA, Rosenbaum JF, Fava M, Biederman, J, Rauch SL. Massachusetts General Hospital Comprehensive Clinical Psychiatry, 1st ed. Philadelphia: Mosby, Inc. 2008. 12.

378

NIH Technology Assessment Panel on Integration of Behavioral

and Relaxation Approaches Into the Treatment of Chronic Pain and Insomnia. Integration of behavioral and relaxation approaches into the treatment of chronic pain and insomnia. JAMA. 1996;276(4):313–318. 13. Pistoia F, Sacco S, Carolei A. Behavioral therapy for chronic migraine. Curr Pain Headache Rep. 2013;17:304. 14. Sieberg C, Huguet A, von Baeyer CL, Seshia S. Psychological interventions for headache in children and adolescents. Can J Neurol Sci. 2012;39:26–34. 15. Fior H, Birbaumer N, Schugens MM, Lutzenberger W. Symptom-

specific psychophysiological responses in chronic pain patients. Psychophysiology. 1992; 29:452–460. 16. Jensen M, Patterson DR. Hypnotic treatment of chronic pain. J Behav Med. 2006;29: 95–124. 17. Dorfman D, George MC, Schnur J, et al. Hypnosis for treatment of HIV neuropathic pain: a preliminary report. Pain Med. 2013;14(7):1048–1056. 18. Flor H, Fydrich T, Turk DC. Efficacy of multidisciplinary pain treatment centers: a metaanalytic review. Pain. 1992;49:221–230.

Section 7 Chapter

53

Special Topics

Pediatric, infant, and fetal pain Christine Greco and Soorena Khojasteh

Case study 1 A sacrococcygeal teratoma with fetal hydrops was diagnosed in a fetus at 27 weeks’ gestation in a 28-year-old gravida 1 woman. The mother is otherwise healthy and there are no other abnormalities detected in the fetus. The pregnancy has been otherwise unremarkable. The decision is made to resect the teratoma.

1. Would this fetus require analgesia? There is much evidence that neural connections necessary to transmit information are present in the developing fetus. This evidence as well as evidence that fetuses can mount a stress response has led to an acceptance of providing analgesia to fetuses during painful surgical procedures. However there are many factors to consider including whether fetal immobility is needed, the risk of preterm labor, the invasiveness of the procedure, and any other underlying conditions in the mother or fetus. Studies by Fitzgerald and others have shown that peripheral, spinal, and supraspinal pathways necessary for nociception undergo significant maturation by the second trimester of human gestation.[1] Spinothalamic and thalamocortical connections are complete by 30 weeks and peripheral receptors are present throughout by 20 weeks’ gestation, indicating that painful stimuli are able to be transmitted from the periphery to the cortex by the end of the second trimester in the developing human fetus.[2] Descending inhibitory pathways such as the dorsolateral funiculus that would modulate pain develop postnatally.[3] This would suggest that neonates and preterm infants may have a more hyperresponsiveness to pain than older infants who have the ability to modulate pain via a more mature nervous system. Preterm infants as

young as 28 weeks have well characterized EEG patterns and somatosensory evoked potentials suggesting that nociceptive pathways from the periphery to the cortex are functional. Studies by Fisk[4–6] and others show that premature infants and fetuses exhibit stress in response to surgical stimuli which may have important effects on complication rates and mortality. Studies of neonates who received deep anesthesia with sufentanil had significantly reduced stress responses, complications, and mortality rates to surgery compared to neonates who received lighter anesthesia.[4] Fisk and others have shown that fetuses can mount a stress and physiologic response to invasive procedures.[5,6] Significant rises in cortisol and β-endorphin as well as redistribution of blood flow to the brain have been demonstrated in fetuses undergoing needle insertion into the hepatic vein. The stress response can be attenuated by administering fentanyl to the fetus.[6] This evidence suggests that painful stimuli reach the cortex in preterm infants although it is unclear whether painful stimuli are perceived as a conscious experience or as suffering. Studies of brain activation using nearinfrared spectroscopy in preterm neonates responding to a painful stimulus to the heel have shown increased signals over the contralateral cortex which implies a specific pattern of response to pain rather than a nonspecific activation seen with autonomic arousal.[7] There is also evidence to suggest that infants have the ability to form memory of pain. Infant males undergoing circumcision with no analgesia were compared to infant males who were not circumcised and infant males receiving EMLA® for circumcision.[4] The infants who received no analgesia with circumcision exhibited the greatest pain behaviors in response to immunizations later in life. There are data in animal models indicating that untreated pain may have

Case Studies in Pain Management, ed. Alan David Kaye and Rinoo V. Shah. Published by Cambridge University Press. © Cambridge University Press 2015.

379

Chapter 53: Pediatric, infant, and fetal pain

long-term consequences. Neonatal rat pups who received repeated inflammatory stimuli showed longterm changes in the dorsal horn synaptic organization and decreased pain thresholds.[8] Factors that have served as the basis for providing analgesia to fetuses during surgical procedures include an understanding of the neurodevelopmental biology of pain transmission, evidence of stress responses of fetuses and neonates which can be attenuated by opioids, behavioral responses to pain, and the potential for long-term negative consequences of untreated pain.

2. How can anesthetics/analgesics be delivered to a fetus?  Transplacental. Many drugs cross the placenta although this is affected by various factors such as lipid solubility of the drug and placental perfusion.

 Intravascular. Intravascular access can be achieved via the umbilical vein or hepatic vein.  Intramuscular. Intramuscular injection can be performed by inserting a needle under ultrasound guidance.

Case study 2 A 3-month-old infant had an inguinal hernia repair 6 hours ago under general anesthesia. Your assessment shows a still infant with his hands and fists clenched. His legs are drawn up. He occasionally grunts. His heart rate is 180/minute; his respiratory rate is 45/minute. He requires oxygen blow-by to maintain his oxygen saturation above 95%.

1. How would you assess pain in this patient? Pain assessment in infants is particularly challenging. Since self-report is not possible for infants and

Table 53.1 Crying Requires oxygen Increased vital signs Expression Scale (CRIES)

Date/time Crying – characteristic cry of pain is high pitched 0 – No cry or cry that is not high-pitched 1 – Cry high pitched but baby is easily consolable 2 – Cry high pitched but baby is inconsolable Requires O2 for SaO2 < 95% – babies experiencing pain Manifest decreased oxygenation. Consider other causes of hypoxemia, e.g., oversedation, atelectasis, pneumothorax 0 – No oxygen required 1 – < 30% oxygen required 2 – > 30% oxygen required Increased vital signs (BP* and HR*) – Take BP last as this may awaken child making other assessments difficult 0 – Both HR and BP unchanged or less than baseline 1 – HR or BP increased but increase in < 20% of baseline 2 – HR or BP increased > 20% over baseline Expression – The facial expression most often associated with pain is a grimace. A grimace may be characterized by brow lowering, eyes squeezed shut, deepening nasolabial furrow, or open lips and mouth 0 – No grimace present 1 – Grimace alone is present 2 – Grimace and non-cry vocalization grunt is present Sleepless – scored based upon the infant’s state during the hour preceding this recorded score 0 – Child has been continuously asleep 1 – Child has been awakened at frequent intervals 2 – Child has been awake constantly TOTAL SCORE

380

Chapter 53: Pediatric, infant, and fetal pain

preverbal children, most pain assessment tools for infants and young children combine behavioral and physiologic parameters. Historically infants were not thought to have the capability to fully perceive pain which was in part due to the lack of reliable methods of assessing pain. There is robust data on pain responses in infants showing that infants exhibit predictable response patterns to pain including behavioral patterns, increased stress hormone levels, and physiologic responses of heart rate, blood pressure, oxygenation, and respiratory rate. Infants display graded responses of heart rate, blood pressure, and oxygen saturation to escalating intensities of pain indicating that infants have the capability to discriminate severity of pain. Physiologic indicators of pain however can be non-specific and can also indicate other reasons for distress such as fear or hunger. Therefore, observed measures of behavioral parameters such as facial grimacing, posture, crying, sleep–wake cycles are often combined with more objective measures such as heart rate, blood pressure, and oxygen saturation to obtain a more accurate assessment of

pain. The Crying Requires oxygen Increased vital signs Expression Scale (CRIES) and the Premature Infant Pain Profile (PIPP)[9] (Tables 53.1, 53.2) are composite pain scales that have been well validated and are frequently used for infants and premature infants. Most infant pain scales are not valid for critically ill infants since sepsis, respiratory failure, acidosis, and other conditions will change behavioral and physiologic parameters. The FLACC scale[10] (Table 53.3) is a widely used pain scale that combines five types of pain behaviors, including facial expression, leg movement, activity, cry, and consolability and can be used in infants and older children, including those with developmental disabilities.[11] The assessment of this 3-month-old infant who recently had surgery as described above shows that he has severe pain using the CRIES. His blood pressure and heart rate are over 20% baseline values and he requires supplemental oxygen to maintain adequate oxygen saturation. He occasionally grunts and his posture indicates that he has such severe pain that he lies very still to avoid painful moving. It is not until

Table 53.2 Premature Infant Pain Profile (PIPP)

Indicators

0

1

2

3

GA in weeks

 36 weeks

32 to 35 weeks and 6 days

28 to 31 weeks and 6 days

< 28 weeks

Active Awake Opened eyes Facial movements present

Quiet Awake Opened eyes No facial movements

Active Sleep Opened eyes Facial movements present

Quiet Sleeping Opened eyes No facial movements

Maximal HR

"0 to 4 bpm

" 5 to 14 bpm

" 15 to 24 bpm

"  25 bpm

Minimal saturation

#0 to 2.4%

# 2.5 to 4.9%

# 5 to 7.4%

#  7.5%

Observe the NB for 15 sec Alertness

Record HR and SpO2

Observe NB for 30 sec Frowned forehead

Absent

Minimal

Moderate

Maximal

Eyes squeezed

Absent

Minimal

Moderate

Maximal

Nasolabial furrow

Absent

Minimal

Moderate

Maximal

Absent is defined as 0 to 9% of the observation time; minimal, 10% to 39% of the time; moderate, 40% to 69% of the time, and maximal as 70% or more of the observation time. In this scale, scores vary from zero to 21 points. Scores equal or lower than 6 indicate absence of pain or minimal pain; scores above 12 indicate the presence of moderate to severe pain. GA, gestational age; NB, newborn.

381

Chapter 53: Pediatric, infant, and fetal pain

Table 53.3 FLACC Pain Scale

Date/time Face 0 – No particular expression or smile 1 – Occasional grimace or frown, withdrawn, disinterested 2 – Frequent to constant quivering chin, clenched jaw Legs 0 – Normal position or relaxed 1 – Uneasy, restless, tense 2 – Kicking, or legs drawn up Activity 0 – Lying quietly, normal position, moves easily 1 – Squirming, shifting back and forth, tense 2 – Arched, rigid or jerking Cry 0 – No cry (awake or asleep) 1 – Moans or whimpers; occasional complaint 2 – Crying steadily, screams or sobs, frequent complaints Consolability 0 – Content, relaxed 1 – Reassured by occasional touching, hugging or being talked to, distractible 2 – Difficult to console or comfort TOTAL SCORE

he receives several IV boluses of morphine that you notice his fists, arms and legs relax, his eyes open, and he assumes a more relaxed posture. His vital signs return to normal after several IV morphine boluses although he continues to require supplemental oxygen.

2. How would you treat this infant’s pain? The primary method of treating this 3-month-old infant would be to administer systemic opioids. Morphine is one of the most widely used opioids in the treatment of pain in infants. Safe and effective opioid use in infants requires an understanding of agerelated differences in analgesic pharmacology. Neonates and young infants have delayed maturation of hepatic enzymes involved in the metabolism of analgesics such as opioids and amide local anesthetics.[12] This results in drug accumulation and an increased risk of toxicity. Although rates of hepatic enzyme maturation vary a great deal, enzymes involved in biotransformation and conjugation will mature considerably by the age of 6

382

months. In addition to immature hepatic enzyme systems, neonates and young infants also have decreased plasma concentrations of albumin and α1 acid glycoprotein which leads to decreased protein binding and greater concentrations of unbound, free, pharmacologically active drug, contributing to increased risk of toxicity of opioids. Neonates also have reduced glomerular filtration rates in the first few weeks of life resulting in slower elimination of many renally excreted drugs and many active metabolites of drugs that have undergone hepatic metabolism. Because of the many age-related differences in analgesic pharmacology, infants have an increased risk of opioid-induced hypoventilation and apnea. Safe and effective administration of opioids requires careful patient selection, dose titration, careful monitoring and observation, and aggressive treatment of opioid side effects. In order to decrease opioid requirements in infants postoperatively, many anesthesiologists will place a caudal block intraoperatively for an infant undergoing a hernia repair. Since this patient is no longer anesthetized, systemic opioids would be the preferred method of analgesia rather than placing a caudal block.

Chapter 53: Pediatric, infant, and fetal pain

Case study 3 A 6-year-old female with cerebral palsy had tendon lengthening surgery 1 day ago. She had a popliteal catheter placed postoperatively and has been comfortable with a 0.1% ropivacaine infusion. You are called by the nurse to evaluate her because of increased pain. On exam she is alert but looks moderately distressed.

1. How would you assess this patient’s pain? Young children are able to give self-reports of pain intensity and location. However, many factors affect the accuracy of self-report in children including concrete thinking, fear, hospitalization, separation from parents. A variety of pain assessment tools have been developed for young children, most involving pictorial representations of faces in varying degrees of distress. Many children tend to prefer faces scales although more than five choices seems to interfere with a child’s ability to indicate pain intensity. The Bieri Faces scale (Figure 53.1), the Wong–Baker faces scale (Figure 53.2), and the Oucher scale (Figure 53.3) are examples of faces scales that have been validated for use in children as young as 4 years of age. The Oucher scale is unique in that it uses actual photographs of children’s faces to depict pain and is available for a diversity of cultural groups. Parental feelings toward their child’s pain and illness can affect a child’s observed behavioral distress. Children whose

parents have heightened levels of anxiety tend to have increased levels of self-report pain scores during painful procedures. The Color Analog Scale (Figure 53.4) is a scale that portrays increasing pain with color gradations and has been validated among children as young as 5 years.[13] Older school age children and adolescents have the cognitive maturity to use standard pain scales; however, fear, hospitalization, pain, and illness may cause emotional regression in children. Pain scales intended for younger children such as the faces scales may be more accurate in these cases. The most appropriate pain scale for the 6-yearold child with cerebral palsy described above is the faces scale. Cerebral palsy is a non-progressive condition that primarily affects motor function but can be associated with a wide range of developmental disabilities. If the child has normal cognitive function, then the faces scale, such as the Wong–Baker Faces Scale, would be appropriate. The FLACC scale would be most appropriate if the patient has cognitive deficits.[11]

2. How would you manage her pain? Since this patient has had a well-functioning popliteal catheter that has been providing good pain relief, it would be most appropriate to first test the catheter to determine if it has become dislodged. If the child’s postsurgical pain has escalated despite a functioning popliteal catheter, then underlying etiologies for

Faces Pain Scale - Revised (FPS-R) 0

2

4

6

8

10

Figure 53.2. Faces: Wong–Baker Pain Rating Scale.

FACES: Wong-Baker Pain Rating Scale

0 No Hurt

2 Hurts Little Bit

4 Hurts Little More

6 Hurts Even More

8 Hurts Whole Lot

Figure 53.1. Faces Pain Scale – Revised (FPS-R).

10 Hurts Worst

383

Chapter 53: Pediatric, infant, and fetal pain

OUCHER!

Figure 53.3. The Oucher Scale.

10

9

the likelihood of toxicity with additional amide local anesthetics. After giving a test dose of chloroprocaine, this patient had a sensory and motor block and very good pain relief. After examining her more closely with the orthopedic surgeon, it was determined that her cast was too tight, causing increased pain. Once her cast was bivalved, she continued to have good pain relief with 0.1% ropivacaine infusion through her popliteal catheter.

Case study 4 8

7

6

5

A 10-year-old female presents to your clinic with a 2-month history of foot pain. She is a competitive gymnast and her mother tearfully describes that she “landed wrong” during gymnastics practice. She has not been able to go to school because of her pain since the injury and has been non-weight-bearing on the affected foot, using crutches for ambulation. Her brother recently stepped on her foot, causing much worsening of pain. On exam, you notice that her foot is cold, mottled appearing, mildly edematous, and she is not wearing shoes or socks due to severe allodynia. She maintains her ankle in extreme plantar flexion at all times which her mother reports occurred at the time of the initial injury.

4

1. What is your differential diagnosis? 3 2

1

0

worsening pain should be considered such as hematoma or infection. It is our practice to test epidural and peripheral nerve catheters in young children with chloroprocaine because of the fast onset and to reduce

384

   

Complex regional pain Fracture Conversion disorder Dystonia

The most likely diagnosis for this patient is complex regional pain syndrome type 1 (CRPS1). Motor and neurovascular characteristics of mottling, coolness, and allodynia are classic presentations of CRPS. Various motor findings such as tremulousness or fasciculations and dystonia can occur. However it is generally not typical for patients to maintain such an unusual posture seen in this patient with extreme, constant plantar flexion from the onset of the initial injury. This may be a result of severe pain and unwillingness to move with a subsequent contracture or a conversion disorder. There is evidence that distortion of brain sensory and motor maps of the body occurs in CRPS and it is possible that abnormal posture is related to these functional brain abnormalities.[14]

Chapter 53: Pediatric, infant, and fetal pain

Figure 53.4 The Color Analog Scale.

2. How does complex regional pain type 1 present in children? CRPS has unique epidemiologic features in children. It is more commonly seen in girls with an 8:1 ratio compared to boys and in 75–90% of cases, a lower extremity is involved.[15] The peak age of onset is around 12 years of age. CRPS is rare among young children, particularly less than the age of 6. Children typically present with pain described as burning, “pins and needles”; young children often have particular difficulty describing their pain since neuropathic pain is so different from nociceptive pain they have previously experienced. Approximately 20% of children will present with pain in the contralateral limb or remote limb. The majority of children recall a previous injury although most injuries are very minor. Sensory, motor, and neurovascular characteristics vary but allodynia and mottling are frequent findings. Many children have significant disability consisting of prolonged or frequent school absences, inability to bear weight, and social isolation, however the degree of disability varies. For many patients, changes in family dynamics tends to reinforce pain behaviors and disability.

3. How would you treat this patient? In both retrospective and prospective studies, evidence suggests that most children with CRPS will have improvements in function and pain through a rehabilitative approach without the use of nerve blocks.[16] For most children, initial treatment involves a multidisciplinary program of active and

aggressive physical therapy, occupational therapy, and CBT. Although medications trials are sometimes used, there is very little pediatric data supporting efficacy. Evidence suggests that a rehabilitative approach improves outcome in children without the use of pharmacologic agents or nerve blocks. Patients who do not respond to outpatient therapy may then need an inpatient or partial hospital program with intense physical therapy, occupational therapy, and CBT. Physical and occupational therapy should be directed at active limb mobilization, independent ambulation without the use of assistive devices, and aggressive desensitization techniques. Education of patients and parents about the non-protective nature of neuropathic pain, reinforcers of maladaptive pain behaviors, and disability are crucial for return of function. We tend to reserve the use of epidural catheters or peripheral nerve catheters for patients who continue to have significant limb swelling or dystonia after participation in a multidisciplinary rehabilitative program. There is some evidence that the use of peripheral nerve catheters provide excellent outcomes in children with CRPS; however long-term recovery was not established.[17] For upper extremity CRPS, we typically place infraclavicular or supraclavicular catheters and for lower extremity symptoms, we tend to place popliteal-sciatic catheters. We place epidural catheters for patients with bilateral lower extremity symptoms. Continuous ropivacaine infusions are generally used for peripheral nerve catheters. Patients with peripheral nerve catheters or epidural catheters are typically hospitalized for 4–5 days and receive daily inpatient psychologic therapy and twice daily physical and/or occupational therapies. For this

385

Chapter 53: Pediatric, infant, and fetal pain

patient with significant motor findings, it may be useful to perform an exam of her foot and ankle under anesthesia to determine if she has a fixed contracture. Radiographic imaging may also help to determine whether she has a fracture given her recent injury by her brother.

Case study 5 A 16-year-old female presents to your clinic with a 7-month history of periumbilical abdominal pain. She has been evaluated by a pediatric gastroenterologist but her mother is not satisfied and requests additional testing and another opinion. She has had a normal CT scan, normal upper and lower endoscopy, normal motility studies, and normal laboratory testing. She reports experiencing near constant abdominal pain, consistently located in the periumbilical region. She often has nausea and diarrhea with her pain but rarely has emesis. She denies fever but has had a 20-pound weight loss over the past year. She recently started high school but has missed much of the year due to her abdominal pain. The patient’s mother reports that she started a new school this year because of bullying at her prior school. On exam, you note that her abdomen is soft with no masses or tenderness.

1. Is this patient’s presentation consistent with functional abdominal pain? Are there any concerning features in this patient’s presentation? Functional abdominal pain (FAP) refers to a condition typically among school age children that is characterized by episodic abdominal pain in an otherwise healthy child who shows no evidence of structural conditions or inflammatory bowel disease. Abdominal pain is exceedingly common in school age children and accounts for 20% of school absences. Functional gastrointestinal disorders (FGID) in children are categorized by the Rome III classification; functional abdominal pain refers to a distinct pattern of FGID in children.[18] Children with FAP present with unique features. Most children are between the ages of 4 and 16 and describe periumbilical pain. Children younger than the age of 4 should prompt a further investigation to identify the underlying cause. Usually pain is described as episodic and rarely awakens the child at night. Fever, weight loss, a family history of

386

inflammatory bowel disease or abnormalities on exam warrant further work-up. The patient described above has many features consistent with FAP. She is 16 years old, localizes the pain to the periumbilical region, and is otherwise healthy. Her 20-pound weight loss is not consistent with FAP. A history of bullying, the recent move to a new school, and the start of high school are significant stressors for this patient and a thorough psychologic evaluation may help uncover underlying psychologic factors contributing to her pain and disability such as depression, anxiety, eating disorder, or school avoidance.

2. What diagnostic studies are indicated in this patient? A thorough history and physical examination should guide the diagnostic work-up. Although many patients have an extensive work-up, in general unfocused testing is not helpful in establishing an underlying diagnosis and may heighten parental and patient anxiety. A complete blood count, urinalysis, stool examination, and serum inflammatory markers are considered basic screening tests. Patients should be referred for further investigations who have abnormalities on basic screening tests or have other concerning signs such as fevers, weight loss, or a family history of inflammatory bowel disease. All patients with FAP should have thorough psychosocial history obtained to identify reinforcers of pain behaviors and disability. This patient’s history of significant weight loss most likely led to a more extensive work-up by her gastroenterologist. You discuss with the patient and her mother that her evaluation and work-up by her pediatric gastroenterologist seems appropriate and you encourage her to continue to have regular follow-up with you in your pain clinic as well as her current pediatric gastroenterologist.

3. How is FAP treated in children? Treatment of FAP depends on the clinical presentation, the severity of disability, and the degree of family dysfunction. History, physical examination, and testing should help to guide management. FAP likely arises from a host of biopsychosocial variables including genetic predisposition, inflammation, infection, stress, and depression. Once central sensitization occurs, other factors such as constipation or emotional stress act as triggers to create intensified pain.

Chapter 53: Pediatric, infant, and fetal pain

Treatment should include a multidisciplinary approach since both psychologic and biologic factors are involved. Treatment trials for constipation, lactose intolerance, and dyspepsia are sometimes useful and should be guided by the patient’s history, physical examination, and testing results. Some patients and parents are anxious about a more serious underlying condition causing pain such as a tumor and are relieved to have a diagnosis of FAP. Education of patients and parents is essential and should include discussions about non-protective pain, returning to function despite pain, avoiding over medicalization,

References 1.

2.

3.

4.

5.

6.

Fitzgerald M, Walker SM. Infant pain management: a developmental neurobiological approach. Nat Clin Pract Neurol. 2009;5(1):35–50. Fitzgerald M. The post-natal development of cutaneous afferent fibre input and receptive field organization in the rat dorsal horn. J Physiol (Lond). 1985; 364:1–18. Fitzgerald M, Koltzenburg M. The functional development of descending inhibitory pathways in the dorsolateral funiculus of the newborn rat spinal cord. Dev Brain Res. 1986;24:261–270. Taddio A, Katz J, Ilersich AL, Koren G. Effect of neonatal circumcision on pain response during subsequent routine vaccination [comment]. Lancet. 1997;349(9052):599–603. Giannakaoulopoulos X, Sepulveda W, Kourtis P, Glover V, Fisk NM. Fetal plasma cortisol and betaendorphin response to intrauterine needling. Lancet. 1994;344(8915):77–81. Fisk NM, Gitau R, Teixera JM, et al. Effect of direct fetal opioid analgesia on fetal hormonal and hemodynamic stress response to intrauterine needling. Anesthesiology. 2001;95:828–835.

and the need for maintaining regular school attendance. There is little data to support the use of medications for the treatment of FAP; however in select cases, tricyclic antidepressants may be helpful for sleep, and SSRIs may be indicated for the treatment of anxiety or depression. Evidence supports the use of cognitive behavioral therapies such as guided imagery, biofeedback, and self-hypnosis over the use of medications. The majority of children improve function and pain on an outpatient basis however; patients with more severe disability may require partial hospital or inpatient rehabilitative programs.

7.

Slater R, Boyd S, Meek J, Fitzgerald M. Cortical pain responses in the infant brain. Pain. 2006;123(3):332.

8.

Anand KJ, Coskun V, Thrivikraman KV, Nemeroff CB, Plotsky PM. Long-term behavioral effects of repetitive pain in neonatal rat pups. Physiol Behav. 1999;66(4):627–637.

9.

Stevens B, Johnston C, Petryshen P, Taddio A. Premature infant pain profile: development and initial validation. Clin J Pain. 1996;12(1):13–22.

10. Merkel SI, Voepel-Lewis T, Shayevitz JR, Malviya S. The FLACC: a behavioral scale for scoring postoperative pain in young children. Pediatr Nurs. 1997;23(3):293–297. 11. Malviya S, Voepel-Lewis T, Burke C, Merkel S, Tait AR. The revised FLACC observational pain tool: improved reliability and validity for pain assessment in children with cognitive impairment. Paediatr Anaesth. 2006;16(3):258–265. 12. Kearns GL, Abdel-Rahman SM, Alander SW, et al. Developmental pharmacology–drug disposition, action, and therapy in infants and children. N Engl J Med. 2003; 349(12):1157–1167. 13. Bulloch B, Garcia-Filion P, Notricia D, Bryson M,

McConahay T. Reliability of the color analog scale: repeatability of scores in traumatic and nontraumatic injuries. Acad Emerg Med. 2009; 16(5):465–469. 14. Maihofner C, Baron R, DeCol R, et al. The motor system shows adaptive changes in complex regional pain syndrome. Brain. 2007;130(Pt 10):2671–2687. 15. Wilder RT, Berde CB, Wolohan M, et al. Reflex sympathetic dystrophy in children: clinical characteristics and follow-up of seventy patients. J Bone Joint Surg [Am]. 1992;74A(6): 910–919. 16. Lee B, Scharff L, Sethna N, et al. Physical therapy and cognitivebehavioral treatment for complex regional pain syndromes. J Pediatr. 2002;141(1): 135–140. 17. Dadure C, Motais F, Ricard C, et al. Continuous peripheral nerve blocks at home for treatment of recurrent complex regional pain syndrome I in children. Anesthesiology. 2005;102(2): 387–391. 18. Rasquin, A Di Lorenzo C, Forbes D, et al. Childhood functional gastrointestinal disorders: child/ adolescent.Gastroenterology. 2006;130(5):1527–1537.

387

Section 7 Chapter

54

Special Topics

Patient with hearing impairment and chronic pain Mohit Rastogi

Case study A 42-year-old deaf male presents with a 6-week history of lower back pain.

1. What should the provider be aware of when communicating with the hearing impaired? The World Health Organization estimates that nearly 5% of the world’s population has disabling hearing loss. The Gallaudet Research Institute estimates that anywhere from 37 to 140 people out of 1000 in the USA deals with some type of hearing impairment. This impairment spectrum ranges from “hard of hearing” to the functionally deaf. The way in which this population communicates varies greatly as some use spoken language, while others, particularly among the deaf community use sign language. As is often the case among those groups of people that do not represent the majority, access to healthcare can be an issue. There are studies that report health professionals lack awareness in how to communicate effectively with the deaf.[1] Language and communication issues adversely impact care as these patients are more likely to defer needed medical care, miss appointments, and are less likely to have a provider with whom they meet regularly.[2] Under the Americans with Disabilities Act (ADA), hospitals must provide effective means of communication for patients, family members, and hospital visitors who are deaf or hard of hearing. This requires hospitals to provide (or have available) qualified on-site or video “interpreters, note takers, computer-aided transcription services, written

materials, telephone handset amplifiers, assistive listening devices, assistive listening systems, telephones compatible with hearing aids, closed caption decoders, open and closed captioning, telecommunication devices for deaf persons, videotext displays, or other effective methods of making aurally delivered materials available to individuals with hearing impairments.” Although providers are obligated to offer these services to patients, often other factors limit usage including a lack of payment (insurance) as well as a lack of awareness of providers of these services. With an estimated 80% of the general population expected to have episodes of lower back pain during their lives, one can assume most pain management specialists will encounter a patient with hearing impairment.

2. Why use sign language? There are many routes of communication available, one of the best being professional interpreter services. It has been demonstrated that this effective tool is often underused in the healthcare setting.[2,3] The lack of understanding of how to communicate and underutilizing available tools often result in healthcare professionals using non-professionals and family members to facilitate an encounter. There are multiple problems associated with this including the nonprofessional lacking any knowledge of medical terminology, and the risks of violation of privacy having a family member interpret. There is a difference in those who are deaf and those who belong to the deaf community as the latter is distinguished by preference of American Sign Language (ASL). It is important to identify

Case Studies in Pain Management, ed. Alan David Kaye and Rinoo V. Shah. Published by Cambridge University Press. © Cambridge University Press 2015.

388

Chapter 54: Patient with hearing impairment and chronic pain

the communication needs of the patient. Middleton and colleagues demonstrated that there can be significant variation in terms of what these needs might be, as some deaf patients prefer sign language exclusively, some may choose to use speech, while others may be comfortable communicating in written English; providers must be sensitive to these preferences.[4] Assuming most providers have limited exposure to sign language, utilization of professional sign language interpreters offers the most consistent service. There are a handful of companies participating in Communication Access Real-time Translation (CART), where speech is converted to text in real time, even remotely. The issue with written language communication is that literacy can be variable among the population and ASL grammar and syntax can differ greatly from written English. Although these other methods have limitations they are superior to the provider writing notes to communicate because of issues with fatigue in extended writing and often incomplete information conveyed. Important things to remember are to allow extra time for the consultation, make sure to look at the patient while speaking and listening, speak clearly to aid lip-readers, but don’t exaggerate speech as this distorts normal lip movements.

References 1.

Meador, HE, Zazove, P. Health care interactions with deaf culture. J Am Board Fam Pract. 2005;18(3): 218–222.

2.

Ku L, Flores G. Pay now or pay later: providing interpreter services in health care. Health Affairs. 2005;24(2): 435–444.

3.

4.

3. How do you evaluate the deaf pain patient? Communication is paramount during assessment of a pain patient and this should include the etiology of the hearing loss. The provider must recognize whether the hearing impairment is part of a constellation of symptoms from some other biologic process (i.e., Jervell and Lange–Nielsen syndrome, long QT, profound hearing loss from birth). A thorough medical history should be taken through interpretation services (sign language, CART) and a thorough pain questionnaire should be completed. There are commonly used tools used in the assessment of pain and these include the VAS, Pain Quality Assessment Scale (PQAS), and McGill Pain Questionnaire (MPQ) or its Short-Form.[5] Once assessment is completed the treatment plan should be clearly explained through the use of interpretation services as well as written information for the patient’s reference. If a patient is a candidate for an interventional procedure, consent could be obtained using any of the various interpretation services. Sedation can be challenging and the use of nonverbal communication can be helpful in conveying information. Deep sedation should be avoided as it can make communication more challenging.

Kale E, Syed HR. Language barriers and the use of interpreters in the public health services: a questionnaire-based survey. Patient Educ Couns. 2010;81(2): 187–191. Middleton A, Turner GH, BitnerGlindzicz M, et al. Preferences for communication in clinic from deaf people: a cross-sectional study. J Eval Clin Pract. 2010;16(4):811–817.

5.

Burckhardt CS, Jones KD. Adult measures of pain: The McGill Pain Questionnaire (MPQ), Rheumatoid Arthritis Pain Scale (RAPS), Short-Form McGill Pain Questionnaire (SF-MPQ), Verbal Descriptive Scale (VDS), Visual Analog Scale (VAS), and West Haven-Yale Multidisciplinary Pain Inventory (WHYMPI). Arth Care Res. 2003;49(S5): S96–S104.

389

Section 7 Chapter

55

Special Topics

Complementary and alternative medicine Ike Eriator and Jinghui Xie

Case study A 45-year-old college educated female with low back pain for 4 years is considering complementary and alternative therapies. She wants to know if the therapies will improve her pain and functioning.

1. How effective are conventional therapies for chronic low back pain? Low back pain is the most prevalent musculoskeletal pain condition. It affects over a quarter of the adult population in the USA over a 3-month period.[1] It is a common, expensive, and sub-optimally managed disorder. Individuals who have low back pain incur healthcare expenditures that are about 60% higher than that of individuals who do not have low back pain, and the incremental expenditures that are attributable to the management of this condition accounted for about 2.5% of all healthcare expenditures in the USA in 1998. In the current care of the patient with low back pain, the choice of treatment is often a reflection of the skill set of the professional consulted. And while the total estimated expenditures among individuals with spine problems increased by 65% from 1997 to 2005, the age- and sex-adjusted indicators of physical functioning, mental health, work and social limitations among individuals with low back pain were considerably worse in 2005 in comparison to 1997.[2] None of the most commonly prescribed conventional treatment regimens for pain are, by themselves, sufficient to eliminate pain or to have a major effect on physical and emotional function in most patients with chronic non-cancer pain. Of all the current treatment modalities for chronic non-cancer pain, the best evidence for pain reduction averages roughly 30% in

about half of the treated patients, and these pain reductions do not always occur with concurrent improvement in function.[3] There are two different systems of healthcare for back pain in the USA. About 60–70% of Americans who seek healthcare for back pain use conventional medicine, while about a third uses chiropractic medicine.[4] Low back pain is the most common medical condition for which adults in the USA use CAM therapies.[5]

2. What is complementary and alternative medicine, and who uses it? As defined by the National Center for Complementary and Alternative Medicine (NCCAM), complementary medicine refers to a group of diverse medical and healthcare systems, practices, and products that are not generally considered to be part of conventional medicine.[6] Alternative medicine refers to the use of approaches that are not part of conventional medicine as replacements for, rather than complements to, conventional medicine. Integrative medicine combines mainstream medical therapies with CAM therapies for which there is some highquality scientific evidence of safety and effectiveness. The World Health Organization defined complementary healthcare as a broad set of healthcare practices that are not part of the country’s own tradition and not part of the dominant healthcare system. About 38% of adults in the USA use CAM. About 75% of American adults have used at least one CAM modality for health improvement at some point in their lifetime and about 34 billion dollars is spent on CAM products and its practitioners annually. CAM accounts for about 1.5% of the total healthcare expenditure and about 11% of the total out of pocket

Case Studies in Pain Management, ed. Alan David Kaye and Rinoo V. Shah. Published by Cambridge University Press. © Cambridge University Press 2015.

390

Chapter 55: Complementary and alternative medicine

expenditure on healthcare.[7] CAM users tend to be well educated, female, and economically comfortable. Patients who use CAM tend to consider their health poorer than that of others and have higher levels of pain.[8] A retrospective analysis of data from 5750 patients who were evaluated in a University pain center over a 7-year period confirmed that about 35% of the patients used complementary and alternative medicine for pain control. The users of CAM were younger, more educated, and more dissatisfied with conventional pain care. Higher level of education, pain severity, and pain duration were persistent correlates of CAM use. Perceived pain control was a consistent enabling factor that was positively correlated with the use of CAM services.[9] About 12% of children in the USA use some form of CAM. Commonly used modalities include the “non-vitamin non-mineral natural products” like herbs, enzymes, manipulative modalities, breathing techniques, and yoga. Less commonly used modalities among children include homeopathy and diet therapies. The most important predictive factor for use of CAM by children is the utilization of CAM by the parent. There are more patient visits to CAM practitioners than to primary care physicians in the USA.[5] The popularity of CAM is related to the dissatisfaction with conventional healthcare and the belief that CAM is more in harmony with the participant’s lifestyle and belief. The holistic approach to care engenders a sense of control and empowerment over the illness. Moreover, the practitioners are more available, more caring, invest more time with the patients, and provide more confidence and optimism when compared to conventional practitioners.[8] Most patients use CAM to complement (rather than as an alternative to) conventional healthcare. Patients often do not limit themselves to a single modality of care – they do not see CAM and conventional medicine as being mutually exclusive.[10] Most patients who use CAM do not volunteer this information to their conventional healthcare providers. The popularity of CAM has driven the impetus for their incorporation into several medical school curricula as well as practices in many hospitals. Despite the popularity and the economic and social expenditures on CAM, the effects of these therapies on clinical outcomes and health remain controversial. Conclusions regarding the effectiveness of

different CAM approaches are guarded at best, since the meta-analyses and systematic reviews on most pain therapies tend to vary widely in terms of diagnostic criteria used for the different conditions, outcome measures studied, selection criteria for study inclusion, inconsistency in some of the treatment methods, and inclusion of patients from countries with different healthcare systems. Even when significant effect sizes are reported, the clinical impact of the measured outcomes is not always clear.[3]

3. Discuss the effectiveness of CAM therapies for low back pain CAM comprises those treatment modalities that are not a standard part of conventional or mainstream medical care, and as such there is no strict limitation of what should be included in this class. And if a therapy is proven to be effective using current evidence-based standards, it will probably move from complementary or alternative status to conventional status. Some critics have argued that there is no need for CAM as all scientifically proven methods will be adopted by conventional medicine. Chiropractic, osteopathy, and biofeedback may be considered as part of current conventional treatment, though in the past, they were regarded as part of CAM. Complementary and alternative medicine modalities can be classified into three subgroups:[6] 1. Natural products: includes a variety of products, such as herbs (also called botanicals), vitamins and minerals, and probiotics. These are widely marketed, readily available to consumers, and often sold as dietary supplements. 2. Mind and body practices: include a large and diverse group of procedures or techniques administered or taught by a trained practitioner or teacher. Several mind and body practices rank among the top complementary health approaches used by adults. Those most commonly used include deep breathing, meditation, chiropractic and osteopathic manipulation, massage, yoga, progressive relaxation, and guided imagery. This group includes: a. Acupuncture. b. Massage therapy c. Most meditation techniques, such as mindfulness meditation or transcendental meditation

391

Chapter 55: Complementary and alternative medicine

d. Movement therapies: including a broad range of Eastern and Western movement-based approaches; examples include Feldenkrais method, Alexander technique e. Relaxation techniques, such as breathing exercises, guided imagery, and progressive muscle relaxation f. Spinal manipulation g. Tai chi and qi gong: practices from traditional Chinese medicine that combine specific movements or postures, coordinated breathing, and mental focus h. Yoga g. Biofield therapies like healing touch i. Hypnotherapy j. Others. 3. Others, for instance, whole medical systems, traditional healers, Ayurvedic medicine, traditional Chinese medicine, homeopathy, and naturopathy.

Acupuncture An increasing number of hospitals offer acupuncture therapy, and there is a growing number of its practitioners in the USA. About 8 million adults used acupuncture in 2002. By 2007, the number had risen to 14 million. The National Certification Commission for Acupuncture and Oriental Medicine (NCCAOM) offers examination and certification in acupuncture, Chinese herbology, Asian bodywork therapy, and oriental medicine. Acupuncture, literally translated into needle (acus) and puncture (punctare), is a technique of healing in which very thin needles are inserted into certain points in the body to restore balance of life force or energy (Qi) flow throughout the body. It is an essential component of Traditional Chinese Medicine (TCM) and its use dates back over 3000 years in China and Korea. The practice is based on the theory that Qi flows through channels within the body called meridians and exists in a state of balance between dark (Yin) and light (Yang) properties. Qi is a force that allows for movement, growth, warmth, and development in the body. In good health, qi flows freely through the meridians. Diseases manifest when Qi becomes unbalanced and its flow through the meridians become obstructed. Because of the close proximity of meridians and Qi to the surface of the skin, the placement of needles on specific points along these meridians can help facilitate the

392

flow of Qi and rebalance the proportion of Yin and Yang.[11] There are 20 meridians (12 regular or principal meridians, and 8 extraordinary channels). The definition and characterization of the acupuncture points remain controversial. There are about 400 acupuncture points on the meridians, and several points outside the meridians. There is evidence to show that acupuncture leads to the release of endorphins and enkephalins, which plays a role in acupunctureinduced analgesia. Administration of opioid antagonists can partially counteract acupuncture-induced analgesia. Using functional MRI, there is evidence to support the fact that acupuncture at adjacent acupressure points (accupoints) elicits distinct cerebral activation patterns, and these specific patterns might be involved in the mechanism of the specific therapeutic effects of different acupoints.[12] Activation of the gate control system as well as a local effect leading to vasodilatation and increased blood flow may also contribute to the effects of acupuncture. Modern needles are made of stainless steel. They are sterilized and packaged in microbial barrier packaging paper. Packaging of needles is required to meet US FDA standards.[11] Although all permutations are based on traditional Chinese philosophy and medical practice, different forms of the practice including auricular, scalp, nose, and foot acupuncture has evolved. Patterns of needling may also vary. Variation may be in terms of depth of penetration, number of needles used, diameter of needle (0.12–0.34 mm), length of needle (0.5–3 inches), and the length of time the needle is left in place. The choice of which point to stimulate is a clinical decision based on the pathologic condition. After insertion, the needles are usually manipulated by the practitioner to stimulate the body’s healing response by lifting and thrusting, twirling and rotating, or by a combination of these movements. Through manipulation, the practitioners feel for tenseness around the needle, which signifies the patient’s experience of De Qi (a localized dull, heavy tingling) often described as the grab of the needle by the muscle. Treatment is often for 1–2 times every week for a minimum of a few weeks, and the needles are left in place for about 30 minutes. One dose of acupuncture is usually not enough to effect pain relief. A course of 6–8 sessions is typical. Sometimes, maintenance acupuncture is recommended, usually at longer intervals. Acupuncture is commonly carried out in conjunction with moxibustion – in which a

Chapter 55: Complementary and alternative medicine

dried herb (Artemisia vulgaris) is burned close to the skin in the area to deliver a heat stimulus. In injection acupuncture, herbals tinctures are injected into the site. In electro-acupuncture, an electrical current is added to the needle after insertion. Pulse stimulation treatment (PSTIM) uses a pulse neurotransmitter controlled by a microchip. It provides a continuous flow of intermittent, lowfrequency electrical pulses to specific peripheral nerves with end locations in the ear. The PSTIM needles connect to acupuncture points and pulses will be transmitted through the stimulating needle to the auricular points that have been identified. The pulse stimulation is commonly placed for 48–96 hours, repeated every 1 to 2 weeks for 4–6 times as a series. A systematic review of randomized control trials (13 trials involving 3025 patients) for different types of painful conditions found a small analgesic effect of acupuncture, which seems to lack clinical relevance and could not be clearly distinguished from bias. Whether needling at acupuncture points, or at any site, reduces pain independently of the psychologic impact of the treatment ritual remained unclear.[13] The Cochrane review (updated from 1996 to 2003) included 35 RCTs covering 2861 patients and concluded that there was insufficient evidence to make any recommendations about acupuncture or dry-needling for acute low back pain. However, for chronic low back pain, acupuncture was more effective for pain relief than no treatment or sham treatment, in measurements taken up to 3 months. Acupuncture for chronic low back pain was also more effective for improving function compared to no treatment, in the short-term. It was not more effective than other conventional and “alternative” treatments. When acupuncture was added to other conventional therapies, it relieved pain and improved function better than the conventional therapies alone, though the effects were only small.[14] The general conclusion at this time is that acupuncture for low back pain relief has only a small effect and is best used in conjunction with other therapy. The risks from acupuncture are minimal when clean needle technique and disposable needles are used by a licensed acupuncture practitioner. The use of anticoagulant therapy or the presence of a cardiac pacemaker should be discussed with the practitioner. Acupuncture is contraindicated in severe bleeding disorders, epilepsy, and first trimester of

pregnancy. Mild transient adverse effects including drowsiness, bleeding, bruising, and pain occur in about 10% of patients. Pneumothorax and infections leading to death are rare, but documented.

Massage Massage is a modality commonly used by patients with chronic low back pain as a supplemental therapy. It involves soft tissue manipulation using hands or a mechanical device on any body part. Wide variations in massage techniques make generalization from studies difficult. A systematic review[15] involving 13 randomized trials (1596 participants) that evaluated various types of massage therapy for low back pain concluded that massage was more likely to work when combined with exercises (usually stretching) and education. The amount of benefit was more than that achieved by joint mobilization, relaxation, physical therapy, self-care education, or acupuncture. No serious adverse events were reported by any participants in the included studies, apart from soreness during or shortly after the treatment and allergic reaction to the massage oil, manifesting as rash or pimples. Contraindications include deep venous thrombosis, phlebitis, burns, open wounds, fractures, and advanced osteoporosis.

Meditation Meditation refers to a broad range of practices that help to focus attention and bring a state of selfawareness as well as inner calm. There are two main classes of meditation. In concentration meditation, attention is restricted to a single point. In mindfulness meditation, there is a non-judgmental observation of emotions, thoughts, or sensation with the goal of cultivating a moment to moment awareness during the meditative process. There have been isolated reports of adverse effects like exacerbation of depression and anxiety.

Autogenic training Autogenic training (AT) refers to a series of mental exercises practiced regularly involving relaxation and autosuggestion. It usually involves six simple meditative exercises focusing the mind on the body’s experience of relaxation.

393

Chapter 55: Complementary and alternative medicine

Progressive muscle relaxation Progressive muscle relaxation (PMR) is a technique for learning to control the state of tension in the muscles. It involves contracting and relaxing muscle groups in a sequential order, building up to being able to combine the relaxation of different muscle groups and eventually relaxing the whole body at once.

Tai chi Tai chi is a Chinese martial art that combines meditation with slow gentle movements, deep breathing, and relaxation to enhance mental and physical strength. It is also based on the balance between the female receptive principle (yin) and the male creative principle. Tai chi may enhance strength, balance, coordination, and improve the cardiovascular system. It is similar in technique to qi gong.

Manipulation Spinal manipulative therapy (SMT) is the most commonly used CAM therapy for low back pain. It involves the use of high-impact-velocity impulse or thrust at the synovial joint at or near the end of the physiologic range of motion, while the practitioner controls the velocity, magnitude, and direction of the impulse. It differs from mobilization therapy in that in the latter, low-grade velocity and passive movement that remains within the patient’s range of motion and control are used to achieve a similar effect. Many different health professionals like chiropractors, osteopaths, and allopaths use some form of manipulative therapy in their practice. A Cochrane review of spinal manipulation therapy[16] included 39 RCTs for which meta-regression models were developed for acute or chronic pain and short-term and long-term pain and function. The results showed that for patients with acute low back pain, spinal manipulative therapy was superior to sham therapy, but the clinical effect size was small (10-mm difference on a 100-mm visual analog scale). SMT was also superior to therapies judged to be ineffective or even harmful. However SMT had no statistically or clinically significant advantage over general practitioner care, analgesics, physical therapy, exercises, or back school. The results for patients with chronic low back pain were similar. The reviewers concluded that there was no evidence that spinal manipulative therapy was superior to other standard

394

treatments for patients with acute or chronic low back pain. Cost is another important consideration.

Yoga Yoga is one of the most frequently used mind and body interventions. It is one of the six orthodox schools of Hindu philosophy, dates back to 3000 years ago, and has become a therapeutic tool. It focuses on stretching, controlled breathing, and distinctive postures. There are different types of yoga. A typical session lasts between 1 and 2 hours. A Cochrane review of 10 RCTs with a total of 967 chronic low back pain patients showed strong evidence for short-term effects on pain, back-specific disability, and global improvement.[17] There was also strong evidence for a longterm effect on pain, moderate evidence for a long-term effect on back-specific disability, but there was no evidence for either short-term or long-term effects on health-related quality of life. Yoga was not associated with serious adverse events. Yoga may therefore be recommended as an additional therapy to chronic low back pain patients. Since it can be implemented easily as a self-care method, its cost-effectiveness seems promising.

Biofield therapies Biofield therapies or spiritual healing or faith therapy involves the practitioner using hands, either on or above the individual’s body, to direct healing energy in order to facilitate general health and well-being through modification of the energy field. It is based on the premise that human beings have an energetic, spiritual dimension necessary for life’s sustenance. In a healthy person, this energy field is symmetrical and balanced. However, imbalance can arise from physical and psychologic symptoms and biofield therapies are used to restore and balance such energy field disturbances. Reiki, therapeutic touch, healing touch, Johrei, and polarity therapy are examples of contemporary biofield therapies. These various techniques differ mostly in the placement of the hands during the delivery of the therapy. Polarity therapy also involves physical manipulation of the body. Healing therapy involves the intentional channeling of healing energy from one or more practitioner to the individual person by means of the hands or thought. Reiki originated in the Tibetan sutras about 3000 years ago and the name is derived from two Japanese characters that together translate as “universal life energy.”

Chapter 55: Complementary and alternative medicine

It involves the placement of the practitioner’s hands at specific points on the body of the individual. Johrei (translates as “pure spirit”) uses a technique focused on removal of negativity, and health is improved through the direction of positive energy. A typical course may consist of about eight sessions. Care is needed in people with major psychiatric illnesses. A Cochrane review[18] involving 24 RCTs or controlled clinical trials with 1153 participants evaluated the effectiveness of healing touch, therapeutic touch, and Reiki. Participants exposed to touch therapy had on average of 0.83 units (on a 0 to 10 scale) lower pain intensity than unexposed participants. Results of trials conducted by more experienced practitioners appeared to yield greater effects on pain reduction. It was also apparent that the trials yielding greater effects were those from the Reiki studies. Two of the five studies evaluating analgesic usage supported the claims that touch therapies minimized analgesic usage.

Balneotherapy Balneotherapy is the treatment of disease by bathing, usually practiced at spas. Mineral water or medicinal clay is often used. It may involve use of cold or hot water, relaxation, stimulation, or massage through the moving water. A systematic review in 2006 concluded that there was encouraging evidence that spa therapy and balneotherapy may be effective for treating patients with low back pain.[19] In an RCT in which exercise in combination with balneotherapy for 10 sessions was compared to exercise plus physical modalities in patients with chronic low back pain, the balneotherapy with exercise group showed significant improvements in quality of life and flexibility of patients compared to the group that had exercise with physical therapy.[20]

Behavioral therapies Behavioral therapies that are practiced as part of CAM use the operant, cognitive, and respondent approaches. They are considered by some to be part of conventional therapies. Operant therapies are typically used to replace pain promoting and reinforcing behaviors with behaviors that lead to an increase in social engagement (such as work and family) and pain reduction (such as hobbies and physical activity). Cognitive therapy for low back pain entails the identification and augmentation of thoughts, patterns of

thought, feelings, and beliefs that have a negative impact on the adjustment of a patient, with the goal of modifying the patient’s experience of the diagnosis, associated disability, and/or lifestyle changes through the challenging of cognitions, feelings, and/or beliefs.[11] In such cognitive therapy, distraction and thought stopping techniques are used to replace negative things like “I am hopeless” with positive ones. Respondent therapy helps through relaxation techniques to modulate the experience of pain. Attempts are made to interrupt the physiologic response of the patient to the sensation of pain. This response is typified by muscle tension, which exacerbates the patient’s sensation of the pain, thereby continuing and amplifying the subsequent physiologic response. A Cochrane review of 30 studies (3438 participants) that evaluated these three behavioral therapies for chronic low back pain concluded that there was moderate quality evidence to show that the three therapies or their combination were similar in the short term and that operant therapy was more effective than waiting list controls in the short-term. There was also moderate evidence to show that behavioral treatment was more effective than usual care (which usually consists of physical therapy, back school, and/or medical treatments) in the short-term. Over the longer-term, there was little or no difference between behavioral treatment and group exercise for pain relief. And the addition of behavioral therapy to inpatient rehabilitation did not appear to increase the effect of inpatient rehabilitation alone.[21]

Prolotherapy Prolotherapy is also called regenerative injection therapy and involves the injection of various types of irritant solutions into joints, ligaments, and tendons to facilitate the body’s healing process through inflammation of the connective tissue. The deposition of new additional fibers helps to repair weak tendons or ligaments. Prolotherapy for low back pain is based on the premise that in patients with low back pain, disc herniations are often preceded by weak spinal ligaments. The laxity of the ligament provides the room necessary for the disc to herniate, and, even after the acute episode subsides, the lack of support by the connective tissue surrounding the disc, and the weakening of the ligament leads to changes in biomechanics that can cause the joint to become

395

Chapter 55: Complementary and alternative medicine

osteoarthritic. The most frequent indication for prolotherapy is chronic low back pain. Hyperosmolar dextrose is the most commonly used solution for prolotherapy. Other commonly used agents include lidocaine, glycine, phenol, and sodium morrhuate. The injection is usually administered at the connection of the ligament or joint to the bone. Treatment sessions are usually given at 2- to 6-week intervals for several months. A Cochrane review concluded that there was conflicting evidence regarding the efficacy of prolotherapy injections for patients with chronic low back pain and that when used alone, prolotherapy was not an effective treatment for chronic low back pain. But when combined with spinal manipulation, exercise, and other co-interventions, prolotherapy may improve chronic low back pain and disability.[22]

Neuroreflexotherapy In neuroreflexotherapy (NRT), several epidermal staples are implanted on specific dermatomes along the back with concurrent implantation of staples at referred tender points in the ear with the goal of interrupting the pain transmission and processing pathway. These devices are normally implanted at a superficial depth of less than 2 mm, and typically last up to 90 days in the back and 20 days in the ear before being removed if they do not fall out on their own. The metal pieces in the lower back are postulated to stimulate the release of peptides that inhibit the generation of pain messages, while the ones in the ear are postulated to activate pain-relieving mechanisms in the brain. Implantation is done on an outpatient basis and there has been no reported scarring associated with the procedure. NRT differs from acupuncture. The areas chosen for stimulation are based on innervation rather than proximity to the meridians. The practical methodologies are also different. NRT was developed in Spain in the 1970s and two decades later, the Spanish National Health System began offering the treatment to patients. A Cochrane review found that NRT appeared to be a safe and effective intervention for the treatment of chronic non-specific low back pain, and that it was more effective than placebo for chronic pain.[23]

Herbal therapy Herbalism is the use of plant extracts for therapeutic or preventive purposes. Many of the current

396

medications in conventional medicine have origins in plant-based materials. For instance, aspirin was synthesized from salicin, the active agent in willow bark, which has been used for inflammatory joint diseases since 50 BC. Morphine and codeine are constituents of opium, the dried exudate from the opium poppy (Papava somniferum) fruit. Some of the best evidence for use of CAM relates to the use of herbal extracts, which are effective for various pain syndromes. In a systematic review[24] of 10 RCTs of moderate or high qualities that included 1567 adults with nonspecific low back pain or chronic back pain, devil’s claw, in a standardized daily dose of 50 mg or 100 mg harpagoside, seemed to reduce pain more than placebo; a standardized daily dose of 60 mg reduced pain about the same as a daily dose of 12.5 mg of Vioxx (the comparative agent at the time of the study). Willow bark, in a standardized daily dose of 120 mg and 240 mg of salicin reduced pain more than placebo and a standardized daily dose of 240 mg reduced pain about the same as a daily dose of 12.5 mg of Vioxx. Cayenne, tested in plaster form, was also found to reduce pain more than placebo. Adverse effects reported were mostly mild, transient gastrointestinal complaints. Devil’s claw is contraindicated in pregnancy (because of the uterine stimulating effects), and in patients with peptic ulcers. It may increase the anticoagulant effects of warfarin. Willow may be associated with anaphylactic reactions, gastrointestinal irritations, and skin rashes and can also increase the anticoagulant effects of warfarin.

4. What is integrative medicine? Integrative medicine combines mainstream medical therapies with CAM therapies for which there is some high-quality scientific evidence of safety and effectiveness. In a prospective pilot randomized trial, investigators[25] demonstrated that it was feasible to assemble and train a clinical team of conventional and licensed CAM providers within an academic teaching hospital, and that the treatments delivered by CAM professionals within this model as applied to patients with LBP were safe. Although this was a short-term study that lasted only 12 weeks in carefully selected patients, the result also suggested that access to an expanded multidisciplinary, integrative care model may be beneficial in patients with low back pain.

Chapter 55: Complementary and alternative medicine

5. What information would you discuss with this patient seeking to use complementary and alternative medicine? Inquiring about and discussing CAM therapies with patients, especially those with severe and prolonged pain symptoms, may provide opportunities for improving the quality of pain care and patient safety. Providers should maintain an informed approach and remain open and empathetic to the needs of their patients. Western medicine works best for an acute physical disease where the anatomy and pathology are very clear. It is much less successful in chronic and poorly understood illnesses with psychosomatic features, typified by chronic low back pain. Patients with chronic low back pain often have to modify their lifestyle and activity level to better accommodate the physical impacts of the disorder. Such adjustments extend beyond the physical activity, and include the thoughts, emotions, lifestyles, and experiences associated with the pain. CAM therapies appear to provide the missing link through a more holistic approach. In the treatment of low back pain, there is evidence to support the use of herb (devil’s claw), acupuncture, spinal manipulation, massage, balneotherapy, and some of the other therapies discussed. An important consideration is that no single approach will benefit all patients. It is still unclear why some individuals respond well to some CAM modalities, while others do not. The evidence suggests that CAM modalities are best used in conjunction with other modalities. Patients often consider CAM therapies to be natural and harmless. However, such therapies can be associated with harmful effects. Acupuncture may be associated with bleeding, hematoma formation, and pneumothorax. Local discomfort, dizziness, headache, and stroke have occurred with manipulation. Contamination of CAM therapies with arsenic, lead,

References 1.

Deyo RA, Mirza SK, Martin BI. Back pain prevalence and visit rates: estimates from U.S. national surveys, 2002. Spine (Phila Pa 1976). 2006;31: 2724–2727.

2.

3.

mercury, caffeine, analgesics, and ephedrine has occurred. Relative doses of the potions or extracts are not regulated as in the case of pharmaceutically produced medications. Some of the agents used have active compounds, and interaction can occur with conventional medications. With more technologies and communication devices, there is a lot of information on CAM out there, especially through the internet. Patients must be careful because there is no filter mechanism for such information, and hardly any regulations. Claims of cures or successes are usually not subjected to any rigorous system of proof. Claims are often made on an individual or personal basis. Proof of success may just be anecdotal, and not scientifically validated. A cautionary acceptance is also warranted. Some of the products are not FDA approved. Patients need be directed to reliable sources of information on the subject. The credential and license of the CAM practitioner (if any) will need to be checked before referring the patient. Consideration of cost is also important. Many of the CAM therapies are not covered by third party payers. For instance, acupuncture therapy costs about $65–$125 per session. CAM methods are often blamed for being basically dependent on non-specific or placebo-like effects. However, the complexities of such non-specific effects also apply to conventional medicine. The popularity of CAM therapies all over the world is an accepted evidence-based fact. They may rely strongly on effectiveness instead of efficacy, but for the patient, it is effectiveness and safety that counts. Perhaps conventional medicine practitioners should learn from CAM and try to maximize any non-specific effects. The place of these therapies in current and future models of coordinated, evidence-based healthcare remains unclear. However, the future of medicine will likely be that of an integrative care that combines the best of complementary medicine with conventional treatment approaches using an evidence-based approach for all patients.

Martin BI, Deyo RA, Mirza SK, et al. Expenditures and health status among adults with back and neck problems. JAMA. 2008;299:656–664. Turk, DC, Wilson, HD, Cahana, A. Treatment of chronic non-

cancer pain. Lancet. 2011;377:2226–2235. 4.

Waddell, G. US health care for back pain. In The Back Pain Revolution, Churchill Livingstone. 1998: pp. 385–401.

397

Chapter 55: Complementary and alternative medicine

5.

6.

7.

8.

9.

Eisenberg, DM, Kessler, RC, Foster, C. et al. Unconventional medicine in the United States: prevalence, costs and patterns of use. N Eng J Med. 1993;328:246–252. NCCAM (National Center for Complementary and Alternative Medicine). What is CAM? http:// nccam.nih.gov/health/whatiscam/ (accessed August 15, 2013). Nahin RL, Barnes PM, Stussman BJ, et al. Costs of Complementary and Alternative Medicine (CAM) and Frequency of visits to CAM practitioners: United States, 2007: National Health Statistics Report # 18. Hyattsville, Maryland: National Center for Health Statistics. 2009. http://nccam.nih. gov/news/camstats/costs/ costdatafs.htm. (accessed August 15, 2013). Austin JA. Why patients use alternative medicine. J Am Med Assoc. 1998;279:1548–1553. Ndao-Brumblay SK, Green C. Predictors of complementary and alternative medicine use in chronic pain patients. Pain Med. 2010;11:16–24.

10. IOM (Institute of Medicine). Complementary and Alternative Medicine in the United States. Washington, DC: The National Academies Press. 2005. 11. Marlowe D. Complementary and alternative medicine treatments for low back pain. Prim Care Clin Office Pract. 2012;39:533–546. 12. Liu H, Xu J, Li L, et al. fMRI Evidence of Acupoints Specificity in Two Adjacent Acupoints. Evidence-Based Complementary

398

and Alternative Medicine, 2013: Article ID 932581, 5 pages. http:// dx.doi.org/10.1155/2013/932581 (accessed August 15, 2013). 13. Madsen MV, Gotzsche PC, Hrobjartsson A. Acupuncture treatment for pain: systematic review of randomized clinical trials with acupuncture, placebo acupuncture, and no acupuncture groups. BMJ. 2009;338:a3115 doi:10.1136/bmj.a3115. 14. Furlan AD, van Tulder MW, Cherkin D, et al. Acupuncture and dry-needling for low back pain. Cochrane Database Syst Rev. 2005; (1):CD001351. doi: 10.1002/ 14651858.CD001351.pub2 (accessed August 15, 2013). 15. Furlan AD, Imamura, M, Dryden T, Irvin E. Massage for low-back pain. Cochrane Database Syst Rev. 2008;(4):CD001929. doi:10.1002/ 14651858.CD001929.pub2 (accessed August 15, 2013). 16. Assendelft WJJ, Morton SC, Yu EI, Suttorp MJ, Shekelle PG. Spinal manipulative therapy for low-back pain. Cochrane Database Syst Rev. 2004;(1):CD000447. doi: 10.1002/14651858.CD000447. pub2. 17. Cramer H, Lauche R, Haller H, Dobos G. A systematic review and meta-analysis of yoga for low back pain. Clin J Pain. 2013;29: 450–460. 18. So PS, Jiang Y, Qin Y. Touch therapies for pain relief in adults. Cochrane Database Syst Rev. 2008; (4):CD006535. doi: 10.1002/ 14651858.CD006535.pub2 (accessed August 15, 2013).

19. Pittler MH, Karagülle MZ, Karagülle M, et al. Spa therapy and balneotherapy for treating low back pain: meta-analysis of randomized trials. Rheumatology (Oxford). 2006;45:880–884. 20. Kesiktas N, Karakas S, Gun K, et al. Balneotherapy for chronic low back pain: a randomized, controlled study. Rheumatol Int. 2012;32:3193–3199. 21. Henschke N, Ostelo RW, Van Tulder MW, et al. Behavioral treatment for chronic low back pain. http://summaries.cochrane. org/CD002014/behaviouraltreatment-for-chronic-low-backpain (accessed August 15, 2013). 22. Dagenais S, Yelland MJ, Del Marc C, Schoene ML. Prolotherapy injections for chronic low-back pain. Cochrane Database Syst Rev. 2007;(2):CD004059 (accessed August 15, 2013). 23. Urrútia G, Burton K, Morral A, Bonfill X, Zanoli G. Neuroreflexotherapy for nonspecific low back pain: a systematic review. Spine (Phila Pa 1976). 2005;30:E148–153. 24. Gagnier JJ, van Tulder MW, Berman BM, Bombardier C. Herbal medicine for low back pain. Cochrane Database Syst Rev. 2006; 2:CD004504. doi: 10.1002/14651858.CD004504. pub3 (accessed August 15, 2013). 25. Eisenberg DM, Buring JE, Hrbek AL, et al. A model of integrative care for low back pain. J Altern Complement Med. 2012;18: 354–362.

Section 7 Chapter

56

Special Topics

Ethical issues in the substance abusing pain patient Ike Eriator, Lori Hill Marshall, and Donald Penzien

Case study Mr. A is a 42-year-old male with chronic neck pain. He required an ACDF following a motor vehicle collision 3 years ago. He was referred from his primary care provider because he kept running out of his oxycodone before the end of the month. Mr. A indicates that oxycodone is the only thing that helps the pain. He smokes two packs of cigarettes a day. He admits to the use of marijuana and cocaine, but says it is for the pain. His past medical history is only significant for multiple accidents. He is requesting opioids for pain management.

1. What is the scope of the public health problem related to the use of prescription opioids for pain? About 100 million Americans have chronic pain.[1] And many people will argue that pain relief is a fundamental human right. Failure of a provider to properly control pain, for which opioids might be required, can result in substandard care and accusations of negligence. In addition, there are serious consequences of untreated chronic pain that impact the individual as well as the society including loss of function, productivity, economic losses, and increased risk of depression and suicide. Data on the effectiveness and safety of long-term opioid therapy for chronic pain are limited. Their use in chronic non-cancer pain remains controversial with respect to efficacy, adverse physical effects, as well as aberrant behaviors.[2] Aside from the physical adverse events, opioids carry a substantial risk of misuse. Studies of patients on long-term opioids for non-cancer pain therapy suggest that as many as 45% could be engaging in aberrant drug-taking behaviors.[3]

Opioids may have been the first group of drugs to be abused in history and are still among those most commonly used for non-medical purposes.[4,5] Since the 1980s when the medical literature began to support opioid therapy for chronic non-cancer pain, there has been a fourfold increase in opioid prescribing.[6] During the same period, unintentional opioid overdose deaths has increased fourfold and substance abuse treatment admissions for prescription opioid addiction has increased fivefold. In the USA, nonmedical use of prescription opioids is the second most prevalent type of illicit drug use after marijuana. In the USA, the misuse of prescription opioids is the fastest growing form of drug misuse and is the leading cause of accidental overdose and mortality.[7] Analysis of the national drug overdose deaths that occurred in 2010 showed that about 58% involved prescription pharmaceuticals and that opioid analgesics were the top drugs involved (75.2%). And nearly 30% of the opioid-related deaths did not involve any other drug class.[8] Patients who are prescribed opioids frequently or in higher doses have higher risk of mortality.[9] Observational studies have shown a correlation between opioid dose and risk of fatal overdoses and adverse events; patients taking a daily morphine equivalent dose of more than 100 mg had a ninefold increase in fatal overdoses while those on more than 120 mg daily equivalent doses of morphine had a twofold increase in substance-related health services utilization.[10,11]

2. What are the differential diagnoses of aberrant opioid-taking behaviors? There are many red flags that would suggest abuse, misuse, addiction or diversion, but no behavior is

Case Studies in Pain Management, ed. Alan David Kaye and Rinoo V. Shah. Published by Cambridge University Press. © Cambridge University Press 2015.

399

Chapter 56: Ethical issues in the substance abusing pain patient

universally linked to addiction, no matter how aberrant it might appear on face value.[12] Aberrant red flags include history of drug treatment, drugrelated arrests, driving under the influence of drug or alcohol, multiple motor vehicle accidents, family history of addiction, history of physical, emotional or sexual abuse, major psychiatric disorders (including personality disorder), bipolar disorder, history of emergency room visit for opioids, history of discharge from other pain clinic for non-compliance, history of obtaining controlled medications off the street, tobacco use, young age, thrill seeking personality, self-professed high tolerance, preference for more rapid onset opioids, unwillingness to obtain previous records, doctor-shopping, lack of interest in nonopioid alternatives, inability or unwillingness to give specimen for drug testing, slurred speech, altered mental status, unexplained pupillary constriction or dilatation, drug-related needle marks, dissonance between appearance and professed dysfunction, liver dysfunction, hepatitis B or C, escalation of medication dosage, requesting early refills, doctor-shopping, losing prescription medications, and tampering with prescriptions.[13] Potential risk factors for aberrant behavior are varied and include substance use disorders, pseudoaddiction, confusional states, failure of communication, personality disorders, and major mental disorders.[14] Multiple risk factors can coexist. Specific risk factors for prescription opioid misuse include younger age (16–45 years), mental illness, personal or family history of substance abuse, or legal history of substance abuse.[6] The differential diagnoses include: Inadequate analgesia (pseudoaddiction): Managed by increasing the dose as a test or trying opioid rotation. Opioid tolerance: Managed by increasing the dose as a test or trying opioid rotation. Opioid-Induced Hyperalgesia: Managed by decreasing dose, switching opioid, or discontinuing opioids. Addiction: Managed by referral for addiction treatment, discontinuing or tapering opioids. Self-medication for other symptoms: Managed by re-education and treating the other symptoms with non-opioids. Diversion: Managed by discontinuing opioids. Chemical coping: Patients are overly drug focused, overemphasizing the meaning of medication.

400

Managed by psychiatric consultation to decentralize medication from patient’s coping. Organic mental syndrome (confusional states): Managed by consultation of psychiatry/neurology. Depression, anxiety, and situational stressors: Consult psychiatry, pain-related psychotherapy.

3. What are some common terminologies related to drug abuse and addiction? Abuse refers to any use of an illegal drug or the intentional self-administration of a medication for a non-medical purpose such as altering one’s state of consciousness, for instance to getting high. Aberrant medication-taking behavior refers to a spectrum of behaviors that may reflect drug misuse. Addiction is a primary, chronic, neurobiologic disease (with genetic, psychological, and environmental factors influencing its development and manifestations), characterized by behaviors that include one or more of the 4Cs: impaired Control over drug use, Compulsive use, Continued use despite harm, and Craving. Diversion is the intentional removal of a medication from legitimate and dispensing channels. It occurs when opioids are shared or sold. Misuse is the use of a medication (for a medical purpose) other than as directed or as indicated, whether willful or unintentional, and whether harm results or not. Narcotic: The word originated from the Greek word for stupor or sleepiness, and it is increasingly used in a legal sense to describe medications associated with dependence, including opioids, cocaine, and marijuana. The word narcotic is imprecise from the scientific point of view, especially with regards to pharmacologic class and members. Opiates refer to drugs like codeine and morphine, whose origin is the opium poppy. Opioids is a generic term used to designate those agents, whether natural or synthetic, that combine with opioid receptors to produce physiologic effects and can be stereospecifically antagonized by naloxone. Opioids include the commonly used medications like morphine, methadone, meperidine, formerly referred to as “narcotics.” Pharming is a term coined by teenagers to describe raiding of medicine closets for prescription

Chapter 56: Ethical issues in the substance abusing pain patient

medicines which are tossed together in a “trail mix” at “pharm parties” and consumed by the handful. Physical dependence is a predictable physiologic response to chronic opioid exposure characterized by the development of a withdrawal syndrome with abrupt cessation or rapid dose reduction or the administration of an antagonist. Physical dependence is not an indication of maladaptive behaviors. Pseudoaddiction refers to the pattern of drugseeking behavior of pain patients receiving inadequate pain management that can be mistaken for addiction. It is characterized by behavior suggestive of addiction that is triggered by unrelieved pain. Patients seeking pain relief should be focused on pain relief and be willing to try non-opioid therapies while the addicted patient is solely focused on obtaining more opioid.[6] Tolerance is a state of adaptation in which exposure to a drug induces changes that result in a diminution of one or more of the drug’s effects over time, such that higher doses are required to achieve the same effect.

4. Describe the basic neurobiologic process underlying addiction Neurobiologic research has shown that when certain regions of the brain are stimulated with an electrode, intense pleasurable sensations are produced, such that the laboratory animal would prefer to self-administer the stimulation to these brain areas instead of eating or sex. All drugs of abuse activate these same pleasure centers and they all have in common the ability to induce the strong feelings of euphoria and reward. As a general rule, all addictive drugs activate this mesolimbic dopamine system. The release of dopamine in the nucleus accumbens reinforces memories of pleasant experience and boosts craving. Dopamine is the master molecule of addiction. The drugs of abuse target the brain’s reward system by flooding the circuit with dopamine. This prime circuit of addictive drugs is the mesolimbic dopamine system. It originates in the ventral tegmental area (VTA), a small area at the tip of the brainstem and projects to the nucleus accumbens (NA) and other areas such as the amygdala, the hippocampus, and the prefrontal cortex of the brain. Even selective activation of the dopamine neurons in this area is enough to produce the behavioral changes seen with addictive drugs. Furthermore, the systemic administration of drugs of abuse leads to release of dopamine in this mesolimbic system and

direct application of drugs into the VTA acts as a strong reinforcer.[4] The reinforcing effects of psychostimulant drugs are related to increased brain dopamine levels and the subjective perception of pleasure positively correlates with the amount of dopamine released.[15] Each addictive drug has specific molecular targets which when activated leads to distinct cellular mechanisms that activates the mesolimbic system. Opioids, cannabinoids, gamma hydroxybutyric acid (GHB), and the hallucinogens effect their actions through the G-protein coupled receptors. Nicotine, alcohol, the benzodiazepines, inhalants, and the dissociative anesthetics mediate their effects through the ionotropic receptors or ion channels. Cocaine, amphetamines, and ecstasy bind to monoamine transporters, blocking the uptake of dopamine or stimulating the nonvesicular release of dopamine. In the VTA, the mu-opioid receptors are selectively expressed on the GABA neurons and (through the inhibition of adenylyl cyclase) preferentially inhibit the GABA neurons through postsynaptic hyperpolarization and presynaptic regulation of transmitter release. The inhibition of these local inhibitory interneurons eventually leads to a disinhibition of the dopamine neurons, causing an increase in the amount of dopamine. Unlike the mu, the kappa-opioid receptors are expressed on the dopamine neurons and cause inhibition. Therefore, the mu-opioid receptors are associated with euphoria, while the kappa-opioid receptors are associated with dysphoria. When taken for recreational purposes, opioids are highly addictive, with a relative risk of 4 (on a scale where 1 is non-addictive and 5 is most addictive). With repeated use, addictive drugs induce neuroadaptive brain changes that include tolerance (escalation of dose to achieve the same effect). When the drug is no longer available, signs and symptoms of withdrawal may manifest resulting in the withdrawal syndrome, which defines physical dependence.[4] However, physical dependence is not always a correlate of drug abuse. Physical dependence can occur with other classes of drugs like bronchodilators or organic nitrate vasodilators. Physical dependence usually occurs with chronic use of addictive drugs, but only a small proportion of users develop that compulsive, relapsing use despite negative consequences that characterize addiction. Relapse is very common among addicts, even after a successful withdrawal.

401

Chapter 56: Ethical issues in the substance abusing pain patient

It is still not clear why some people develop addiction, while others do not. Drug vulnerability due to genetic influences accounts for about 38% of all cases and environmental and social factors account for the balance.[16]

5. What are the ethical principles that guide the use of opioids in pain control? The safe and effective use of opioids for the management of chronic pain is complex and requires an appropriate balance of the goal of relieving pain and suffering and the goal of not harming the patient or other members of society.[6] It requires judgments about needs and interests, risks and benefits, and fairness and responsibilities.[17] In essence, multiple ethical decisions and concerns have to be addressed at every step of providing pain control with opioids. Ethics refers to the moral principles embraced by an individual or a group designed to provide rules for the right conduct. The Law defines the minimum standards of performance that a society will tolerate and are enforced by the government. Moral and legal accountabilities may not be concordant. There is no objective yardstick for measuring pain. The best measure is still what the patient feels and communicates to the provider. Coupled with that is the fact that every pain has a sensory, cognitive, and emotional component, and is influenced by several psychosocial factors. Science has not been able to clearly untangle the relationship between pain and pleasure. While there is a pain pathway in the body, there is no neuroanatomical pleasure pathway. John Dryden, the 17th century poet laureate wrote that for all the happiness that mankind can gain, it is not in pleasure, but in rest from pain. Opioids have both pain-relieving and self-reinforcing properties that influence how patients use them. Morphine (a component of opium – which was known to the cradles of the ancient civilization) is still the current gold standard for the treatment of severe pain. This group of medications was originally used for the relief of suffering. William Osler wrote “Among the remedies which it has pleased Almighty God to give to man to relieve his sufferings, none is so universal and so efficacious as opium.”[5] Using opioids to treat chronic pain in a substanceabusing patient adds another dimension to this complex situation. Pain treatment and substance abuse can interact in multiple ways. Pain medication can

402

raise the risk of iatrogenic drug dependence. Substance-abusing patients are more likely to misuse or divert prescription medication.[14] Young adults who abuse prescription opioids are a similar cohort to other hard drug users.[18] A history of substance use disorder is a red flag for prescription medication abuse. If the patient is prescribed opioids, the risk of relapse of addiction or misuse of opioids must be acknowledged and anticipated. When opioids are withdrawn in the setting of chronic pain, relapse of opioid misuse is common in those patients with severe pain and depressive symptoms.[19] There are ethical principles that can provide guidance with regards to the use of opioids in the chronic pain patient. The principles of beneficence and non-maleficence are grounded in the traditional Hippocratic ethic, and are accepted as guides for action by healthcare providers. The principles of justice and respect for autonomy are linked to a drive for social justice and human rights. The principle of beneficence requires that the provider directly intervene with the goal of increasing the comfort, health, or well-being of the patient. It is closely related to compassion. Most providers are usually familiar with the benefits of opioid therapy and that it can reduce pain-related suffering and morbidity.[20] The degree of benefit is related to the level of pain and functional restriction. The benefit would be high for a patient with severe pain not successfully controlled by non-opioid medications and other modalities. For the patient who has not yet had a fair trial of non-opioid medications and other measures, the benefit may be moderate. For the patient who intends to sell the drug for profit or to consume it to satisfy an addiction, the result would be construed as harm rather than benefit.[17] The principle of non-maleficence is the primary moral obligation of the provider to avoid or minimize harm or risk of harm. The corollary of beneficence, non-maleficence, is often stated as “Do no harm” or “primum non nocere.” This principle also engenders the provider to avoid causing unwarranted injury or suffering to the patient. For instance, untreated pain can be associated with anxiety, depression, immune system changes, central nervous system sensitization, and even lead to drug abuse in susceptible individuals. The principle of non-maleficence also requires the provider to consider the side effects of opioid medications including the risks of abuse, misuse, and addiction. In patients where these side effects can be

Chapter 56: Ethical issues in the substance abusing pain patient

significant, the principle of non-maleficence can run counter to the principle of beneficence. In such cases, the principle of the double effect provides for undesirable secondary effects of a primary action, if the primary action provides benefit and the secondary effects are unlikely or less harmful to the patient than the target problem. Of course, if the possible harm is very likely or significant, then the possible benefit does not justify the fare. The possibility of harm is also significant when opioids are appropriate and safe for the patient, but could potentially harm others in the society through diversion. In the palliative or hospice care setting, concerns about addiction take a back seat to the goal of providing comfort, since problematic behaviors are usually not a significant problem in the palliative care population. Respect for autonomy: Autonomy refers to the capacity of a competent individual to make an informed, unforced decision. The main expression of the respect for autonomy is the process of informed consent. A patient is given broad choices of pain treatments that can be medically justified, and allowed to choose or decline any of the options. Autonomy requires adequate cognitive functioning and unimpaired judgment on the part of the patient to be able to comprehend the choices and make a rational decision. There are questions regarding the ability of patients with significant pain or suffering or compulsive drug use to weigh the pros and cons of opioid treatment.[20] Respect for autonomy does not mean that the patient dictates what the provider will prescribe. “Giving in” and prescribing any medication that is against sound medical judgment is a disservice to the patient and likely ethically indefensible, as it contradicts the principle of non-maleficence.[17] The principle of justice calls for treatment to be offered in a fair and equitable fashion to those who may benefit from it. It can also be described as the obligation to act on the basis of fair adjudication between competing claims. The principle is compatible with the practice of screening out people who seek opioids for non-medical use, addiction, or diversion. However, if screening were ineffective or inaccurate, it would also be unethical because it could result in injustice by excluding pain control measures for some people based on irrelevant characteristics.[17] For instance, when deciding on the use of opioid for cancer pain, the little old lady with degenerative arthritis who does not have any

history of smoking or substance abuse should not necessarily receive the exact same outcome as the 40–year-old male patient with non-specific low back pain and a history of active drug abuse. The provider who prescribes an opioid shares with the patient a dynamic ethical responsibility for the proper use of the drug. This responsibility requires attention on the part of the provider and collaboration on the part of the patient to ensure that it is used for the right purpose, the right patient, and at the right time.

6. How can opioids be safely used for chronic pain? All patients taking opioids long term have the potential for prescription opioid misuse. Therefore, as a universal precaution, all patients should be seen as being at risk, and therefore will require screening and assessment initially, and an ongoing monitoring for harm and benefits during treatment. There are several guidelines,[2] but the essential steps include a detailed review of risk assessment before initiation. This includes review of old records and information from previous providers, review of prescription monitoring program data, clinical assessment and diagnosis, drug screening, risk–benefit analysis of treatment options, and informed consent. Opioids are best used as part of multimodal treatment options. The opioid chosen should be individualized based on clinical assessment, pain diagnosis, and pain characteristics. During the initial phase of treatment, titration requires frequent clinical assessment for dose adjustment and monitoring of adherence to treatment. The on-going phase of treatment requires clinical evaluation for treatment benefits, side effects, and adherence including the 4As (analgesia, activities of daily living, adverse effects, and aberrant behaviors). Identifying and managing aberrant behaviors during treatment will require a combination of some or all of the following:[14] 1. A well-structured pain treatment agreement 2. Regular periodic clinical evaluation 3. Prescription monitoring system 4. Collateral information from other providers, prescription monitoring program and from the patient’s family members or significant others 5. Periodic random urine drug screening Patients with comorbid pain and substance abuse will require a full assessment of their current psychosocial

403

Chapter 56: Ethical issues in the substance abusing pain patient

circumstances including stresses and vulnerabilities in addition to the pain diagnoses, substance use diagnoses, and psychiatric comorbidities. A decision to initiate, continue, or discontinue the opioid therapy for the patient should be based on the risk–benefit ratio. Will other less risky therapy achieve the goals? Does the patient need to be on opioid therapy? In what setting is the opioid therapy going to be used? Will the opioid be used as part of multimodal, interdisciplinary care? Is the opioid therapy going to be for the short term or for the long term? In cases of opioid treatment where there is benefit in the absence of harm, it is easy to make the decision to continue the opioid therapy. If the benefit is small, in the absence of harm, then a dose increase may be considered. Higher doses of opioids are associated with increased risks,[10,11] and patients should therefore take the lowest effective opioid dose possible.[6] In cases where there is no benefit and no harm, a trial of a different opioid using equianalgesic dosing should be considered. If there remains no benefit and the patient is not meeting treatment goals, then the treatment benefit cannot outweigh the risks and a taper (if the patient is physically dependent) or discontinuation of the opioid is recommended. In situations where there is a lack of benefit even after switching to a different opioid, the opioid therapy should be tapered or discontinued. For the patient who is engaging in aberrant medication-taking behavior, a consideration of the complete differential diagnoses for that behavior is necessary. If due to addiction, then the benefit does not outweigh risks and the patient should be referred to addiction therapy. If due to pseudoaddiction, treatment adjustment is the appropriate step. Such adjustments may be in the form of other modalities to supplement pain control or increase in dose of the opioid or a switch to another medication. Time spent for discussion and communication is important. Patient education regarding realistic treatment expectations and opioid risks is important. It must be clearly communicated that the initial and subsequent opioid prescriptions are a test and not necessarily a commitment to long-term opioids. This discussion is often aided by using a risk–benefit framework which should focus attention on judging the treatments, not the patient. This discussion and explanation is helpful in clarifying the clinician’s role as a caregiver, and not as a law enforcement officer or a judge.[6]

404

It is also advisable to educate patients fully about the risks and benefits of the proposed treatment plan, and to document informed consent. Because of the inherent risks of opioids, clinicians should obtain informed consent from patients.[21] This should include a description of the potential opioid risks including adverse effects (short- and long-term), physical dependence, risk of drug interactions resulting in central nervous system and respiratory depression, addiction, and overdose. The informed consent should also describe the patient’s responsibilities including safe storage, disposal, and not diverting the medication. It is particularly important to warn the patient about the elevated risk of driving and overdose when the opioid is started and titrated. Opioid treatment agreement is a useful educational and informative tool that enhances the patient’s involvement in the treatment. It should include expectations for patients and providers and consequences for aberrant medication usage or behaviors. This agreement may also lay out policies such as the use of intermittent urine drug testing, pill counting, and prescription monitoring. It is also valuable to specify treatment goals such as the restoration of function, ability to participate in rehabilitation, or avoidance of emergency room visits for pain. Some providers also put a specific time horizon on functional goals, stating for example that after 4 months, the issue will be revisited and opioid prescribing will be stopped if the patient is not making progress or shows aberrant behavior.[21] The universal precautions when using opioid therapy include making a diagnosis with appropriate differentials, evaluating patients for the risks for abuse or addiction, getting an informed consent, use of treatment agreement, monitoring for treatment effectiveness, side effects and aberrancy, regular reassessment of the 4As, periodic review of diagnosis and treatment, and readjustment based on outcome data, and good documentations. Two commonly used screening tools include the Opioid Risk Tool (ORT) and the Revised Screener and Opioid Assessment tool for Patients with Pain (SOAPP-R). The Opioid Risk Tool is a brief 5-item self-report that is filled out by the provider or patient and classifies patients as low risk (0–3), moderate risk (4–7), or high risk (8 or higher) based on the total score. It helps to predict who is at risk before initiating opioid therapy. However, the yes or no questions as a screening tool can be obvious to patients and the

Chapter 56: Ethical issues in the substance abusing pain patient

answers might be tailored dishonestly if the patient senses that an honest answer may lead to an exclusion of opioid medications. The SOAPP-R evaluates the risk of opioid abuse, using 24 self-report items on a 0–5 scale which is then added together. A score of 8 or higher suggests increased risk. Both of these instruments provide a numerical value to assess risk. Studies suggest that clinical interviews and the long-form SOAPP-R are the most effective at predicting patients that are at high risk for aberrant behavior.[22] The concept of breakthrough pain has not been helpful in the management of chronic non-cancer pain. Giving short-acting opioids for breakthrough pain along with long-acting opioids usually results in the patients taking regular daily doses of both the long-acting and short-acting medications.[23] Breakthrough medications can be used during the initial titration phase, and then built into the long-acting regime. In cases where it is necessary to include breakthrough medications, the dose should be kept to about 10% or less of the total opioid dosage. Nonopioid modalities may also be considered for breakthrough pain. Monitoring is an important consideration when using opioid therapy and includes regular or frequent or unscheduled clinic visits, pill counts, use of prescription drug monitoring program data, and drug screening tests. Pill counts, which can help to monitor medication adherence, may be done during a scheduled or an unscheduled visit. If the patient forgets to bring the remaining pills, they should be given the opportunity to return within a few days for the count. Prescription Monitoring Program (PMP) can help the provider to ascertain whether the patient is obtaining controlled substances from different providers. Drug tests help to monitor for adherence to the prescribed medication and the avoidance of nonprescribed and illicit substances. Blood drug monitoring is invasive and is not very easy to use in the clinical setting. Hair drug monitoring provides better cumulative long-term information (weeks to months). Urine Drug Screening (UDS) is the gold standard for monitoring in the clinical setting. There is no consensus on the frequency of UDS; however, higher risk patients will require more frequent monitoring. Unscheduled UDS may have the greatest chance of detecting aberrancy. Specimens should be collected at the point of care. If the medication has not been taken in 72 hours, it will not likely be detected in the urine. Unexpected positive test results

may be due to illicit drug use, metabolic conversion of prescribed medication, medications from other legitimate sources (such as recent surgery), or incidental exposures, for instance in herbal medications or food such as poppy seeds. Unexpected negative results may be due to specimen manipulation, hoarding or diverting of medication, rapid drug metabolism due to enzyme induction, or the drug may be present but at levels below the cut-off for the laboratory test. Other possible causes include ordering the wrong assay, use of incorrect specimen, or interference from cross-reaction of agents. Unexpected urine drug test results should be carefully reviewed and confirmed. Calling the laboratory can help elucidate the findings. If the UDS is consistent with the patient using other substances during opioid treatment, a clinical decision will have to be made regarding further management. Many providers will reduce or eliminate prescribed opioids in response to these findings. If the patient was not having good pain relief, this may be mutually agreeable between the patient and the provider. The patient should be offered other pain treatments, and referred to chemical dependency treatment or 12-step support groups such as Narcotics Anonymous or Alcoholics Anonymous.[14] Active substance use problems need to be addressed, usually by a provider other than the pain specialist. There should be a good understanding and agreement with regards to who will prescribe which medication. If possible, one provider should prescribe all the medications, while the other specialists provide timely consultations and recommendations. Structured compliance checklists, motivational interviewing, and randomized drug tests can reduce aberrancy in high-risk pain patients.[24] Consultation with an addiction specialist or psychiatrist may be required as a condition of treatment. Some pain patients who have opioid addiction may be suitable candidates for buprenorphine or methadone maintenance therapy, provided by an addiction specialist. Patients should be provided with options for addiction treatment, with the understanding that when the addiction is controlled, options for pain management will increase. An expert consensus recommended that patients who are low risk and who have had only a few episodes of less serious aberrancy, such as asking for early refills, may be managed with education and more frequent monitoring, such as pill counts, providing shorter duration opioid prescriptions,

405

Chapter 56: Ethical issues in the substance abusing pain patient

mandatory reassessments before renewal, or repeated urine drug testing.[22] Patients with repeated aberrancies, or serious aberrancies such as positive cocaine in urine tests, or doctor-shopping, are candidates to be tapered off opioids entirely. Throughout this process, the provider should maintain professional decorum, especially in the face of unexpected findings that may suggest that the clinician has been “fooled” by the patient. If opioid therapy is to be discontinued, the reasoning should be explained to the patient. It should be made clear that it is the ineffective or risky treatment (and not the patient) that is being abandoned, and that other treatment options are available.

7. Considering the complexity and risks, why not just avoid offering opioid therapy altogether? Every well intentioned provider that uses opioid to treat pain has been scammed at one time or the other by a “pain patient” who is otherwise using the medication for purposes apart from pain control. Some providers, realizing that they have been scammed,

References 1.

2.

3.

4.

406

Simon LS. Relieving Pain in America: A Blueprint for Transforming Prevention, Care, Education and Research, Vol 26. Washington, DC: National Academies Press. 2012. Chou R, Ballantyne JC, Fanciullo GJ, Fine PG, Miaskowski C. Research gaps on use of opioids for chronic non-cancer pain: findings from a review of the evidence for an American Pain Society and American Academy of Pain Medicine clinical practice guideline. J Pain. 2009;10:147–159. Michna E, Jamison RN, Pham LD, et al. Urine toxicology screening among chronic pain patients on opioid therapy: frequency and predictability of abnormal findings. Clin J Pain. 2007;23: 173–179. Luscher C. Drugs of abuse. In Katzung B, Masters S, Trevor A, eds. Basic and Clinical

resolve that it will not happen again; they take the overly simplistic approach and stop offering opioid therapy altogether. Total avoidance of prescribing opioids is not an ethical option for most providers. Opiophobia refers to the irrational fear or unwillingness to use opioids for pain treatment. Those providers who have never been scammed are probably too strict in their criteria for prescribing opioids and are therefore leaving out some genuine pain patients who could otherwise benefit from opioid therapy. At the other extreme are the providers that are too liberal in their use of opioids for pain control and are intentionally or unintentionally aggravating the personal and public problems associated with the current opioid epidemics. The category of providers who are mis-prescribers include those who are dated (not keeping up with education on substance abuse), duped (gullible or co-dependent), drug dependent (addicted), and those who are deliberate dealers. The key is striking an appropriate balance between the two extremes, such that these medications can be available for the pain patients who need them, without promoting abuse, misuse, or addiction.[5,25]

Pharmacology. New York: McGraw-Hill. 2012: pp. 565–580.

opioid overdose-related deaths. JAMA. 2011;305:1315–1321. 11. Gomes T, Mamdani MM, Dhalla IA, Paterson JM, Juurlink DN. Opioid dose and drug-related mortality in patients with nonmalignant pain. Arch Intern Med. 2011;171:686–691.

5.

Eriator II. Opioid use in the United Sates. Fed Practitioner. 2003;20(11):50–64.

6.

Alford DP. Chronic back pain with possible prescription opioid misuse. JAMA. 2013;309:919–925.

7.

Compton WM, Volkow ND. Major increases in opioid analgesic abuse in the United States: concerns and strategies. Drug Alcohol Depend. 2006;81: 103–107.

12. Hay JL, Passik SD. The cancer patient with borderline personality disorder: suggestions for symptom-focused management in the medical setting. Psychooncology. 2000;9:91–100.

8.

Jones CM. Pharmaceutical overdose deaths, United States, 2010. JAMA. 2013;309:657–659.

13. Bailey JA, Hurley RW, Gold MS. Crossroads of pain and addiction. Pain Med. 2010;11:1803–1818.

9.

Paulozzi LJ, Kilbourne EM, Shah NG, et al. A history of being prescribed controlled substances and risk of drug overdose death. Pain Med. 2012;13:87–95.

14. Krashin D, Murinova N, Ballantyne J. Management of pain with comorbid substance abuse. Curr Psychiatry Rep. 2012;14:462–468.

10. Bohnert AS, Valenstein M, Bair MJ, et al. Association between opioid prescribing patterns and

15. Volkow ND, Wang GJ, Fowler JS, et al. Reinforcing effects of psychostimulants in human are associated with increases in brain

Chapter 56: Ethical issues in the substance abusing pain patient

dopamine and occupancy of D2 receptors. J Pharmacol Exp Ther. 1999;291:409–415. 16. Uhl G, Blum K, Noble E, Smith S. Substance abuse vulnerability and D-2 receptor genes. Trends Neurosci. 1993;16:83–88. 17. Katalik J. Controlling pain and reducing misuse of opioids: ethical considerations. Can Fam Physician. 2012;58:381–385. 18. Catalano RF, White HR, Fleming CB, Haggerty KP. Is nonmedical prescription opiate use a unique form of illicit drug use? Addict Behav. 2011;36:79–86. 19. Heiwe S, Lonnquist I, Kallmen H. Potential risk factors associated

with risk for drop-out and relapse during and following withdrawal of opioid prescription medication. Eur J Pain. 2011;15:966–970. 20. Cohen MJM, Jasser S, Herron PD, Margolis CG. Ethical perspectives: opioid treatment of chronic pain in the context of addiction. Clin J Pain. 2002;18:S99–S107. 21. Chou R, Fanciullo GJ, Fine PG, et al. Clinical guidelines for the use of chronic opioid therapy in chronic non-cancer pain. J Pain. 2009;10:113–130. 22. Moore T, Jones T, Browder J, et al. A comparison of common screening methods for predicting aberrant drug-related behavior

among patients receiving opioids for chronic pain management. Pain Med. 2009;10:1426–1433. 23. Manchikanti L, Singh V, Caraway DL, Benyamin RM. Breakthrough pain in chronic noncancer pain: fact, fiction, or abuse. Pain Physician. 2011;14: E103–117. 24. Jamison RN, Ross EL, Michna E, et al. Substance misuse treatment for high-risk chronic pain patients on opioid therapy: a randomized trial. Pain. 2010;150: 390–400. 25. Eriator II. Narcotic analgesics for chronic pain management. Curr Rev Pain. 1998;2(4):193–200.

407

Section 7 Chapter

57

Special Topics

Approach to the patient with abnormal drug screen Jeffrey Hopcian and Magdalena Anitescu

Case study 1 A 40-year-old woman comes to the pain clinic for a follow-up visit. She has a history of postlaminectomy syndrome and neuropathic pain unresponsive to epidural steroid injections. You are attempting to manage her pain medically. Currently she is taking a multimodal regimen of gabapentin, citalopram, and oral NSAIDs. Her other medications include protonix, ginko biloba, loratadine, and OTC antitussives. Today, she reports her pain is generally well managed, but on rare occasions following days of high activity, she experiences extreme pain unrelieved by her medications. She requests the addition of “something stronger” for as needed use. Review of her urine drug screen reveals a positive test for opioid-class medications.

1. Clinical assessment Whatisthe differential diagnosisfora patient with a positive drug screen? In general, the differential diagnoses for patients with a positive drug screen can be classified as falsepositive or true-positive. False-positive results may come from any number of sources including cross-reactivity or individual variances in drug metabolism. These issues are discussed in further detail later in this chapter. Truepositive test results can be attributed to a number of behaviors and diagnoses depending on the type of test, the specific result in question, and the actions of the patient. In this case, the differential diagnosis for true-positive drug testing includes the following: – Substance abuse – Substance dependence or addiction – Pseudoaddiction

– Chemical coping or self-medicating – Organic brain syndrome It is the responsibility of the treating physician to uncover the actions or intentions that result in a positive drug test. Furthermore, he/she must have a working knowledge of diagnostic terminology. Here, the term substance abuse is defined as a maladaptive pattern of use characterized by a failure to fulfill obligations, use in physically hazardous situations, use in spite of legal problems, or continued use despite social and interpersonal consequences.[1] Substance dependence is defined by negative physical or psychologic problems associated with use. It is characterized by tolerance (i.e., exposure resulting in diminished efficacy), withdrawal syndrome, and failed attempts to restrict use.[1] Addiction, while not defined in the DSM-IV, is often used synonymously with the term substance dependence, although its applications are broader. It implies some element of abuse. On the other hand, physiologic substance dependence can occur with substances not associated with abuse (e.g., beta-blockers).[2] The term pseudoaddiction was first described by Weissman in 1989 as a pattern of behavior resembling drug abuse that is actually aimed at obtaining relief from poorly controlled pain.[3] In this chapter, the term substance misuse is used to encapsulate any behavior of drug taking (prescription or illicit) that deviates from prescribed instructions, expectations, or the confines of legality.

What information exists regarding the epidemiology of drug abuse in the chronic pain population? Abuse of controlled substances in chronic non-cancer pain patients is a persistent, urgent issue. Well over 90% of patients referred to pain specialists are already

Case Studies in Pain Management, ed. Alan David Kaye and Rinoo V. Shah. Published by Cambridge University Press. © Cambridge University Press 2015.

408

Chapter 57: Approach to the patient with abnormal drug screen

receiving opioid therapy at the time of referral. Estimates for substance misuse rates in this population vary tremendously, owing in part to the complex interplay of other risks factors and previous exposure. Prevalence rates for low-risk patients are cited below 1% and for high-risk patients above 40%.[4–8] This is several-fold above the average for the general population, estimated at approximately 8%.[9] Moreover, patients taking prescribed controlled substances appear to have a higher prevalence of illicit drug use compared to patients who do not take controlled substances; 34% vs. 14%, respectively.[10] Risk factors for aberrant drug use have been explored in a number of studies. Generally accepted risk factors include family history of substance abuse, nicotine dependency, age < 45 years, pain involving more than three regions of the body, pain after a motor vehicle accident, history of psychiatric trauma (sexual abuse, post-traumatic stress disorder), and the presence of a psychiatric disorder such as depression, attention-deficit disorder, schizophrenia, personality disorder, post-traumatic stress disorder, somatoform disorder, and organic brain syndrome.[11–13] Given the high abuse potential of opiate medications, several screening tools have been developed to stratify the risk for opioid misuse before long-term opioid therapy is prescribed. Among these screening tools are the following: – The Opioid Risk Tool (ORT) – Pain Medication Questionnaire (PMQ) – Diagnosis, Intractability, Risk, Efficacy Score (DIRE) – Screener for Opioid Assessment for Patients with Pain-Revised (SOAPP-R) These tools attempt to stratify patients into low-, medium- or high-risk categories. They are typically measured against a formal psychologic evaluation, a generally accepted gold standard for risk assessment for substance misuse. Some studies have suggested that SOAPP-R has a superior sensitivity/specificity compared to other screening tools, correctly identifying 70–77% of patients who would eventually be discharged from a pain clinic for aberrant drug-taking behavior.[11]

What are the clinical implications of drug misuse in this patient population? Despite their widespread use, for drugs with high abuse potential (i.e., opioids, anxiolytics) there is

limited evidence to support their effectiveness for relieving chronic pain, improving functional status, or improving quality of life.[14–17] Rather, it is the opinion of many that opioid use in chronic pain works against the principles of chronic pain management such as improving self-efficacy and reducing reliance on the healthcare system. In one large-scale epidemiologic study, matched cohorts of chronic pain patients revealed worse pain, higher healthcare utilization, and lower activity levels among patients receiving opioids compared with those who did not.[18] Misuse of controlled and illicit substances has negative implications in this patient population. In addition to the physical dangers and social consequences of acute intoxication and drug overdose, drug misuse in this population may lead to a vicious cycle of tolerance and self-escalation of dosing. The negative downstream effects of this behavior include loss of effective agents for breakthrough or acute pain episodes, opioid-induced hyperalgesia, and withdrawal syndromes. In the last decade, fatal overdoses from prescription opioids have surpassed those from heroin and cocaine. In 2010, 2 million people initiated nonmedical use of prescription opioids and 2.4 million people initiated use of marijuana. The majority of opioid abusers obtained these drugs from a friend or relative (55%).[19] The cost of prescription drug abuse to society in 2007 was estimated at $55.7 billion, stemming from losses in the workplace as well as healthcare consumption and criminal justice system costs.[20]

2. Overview of drug testing What modalities and assays are available for clinical drug testing? Tests for the presence of prescription or illicit substances can be performed on a number of body samples including urine, blood, saliva, finger- or toe-nails, and hair. Urine tests have garnered favor for their minimally invasive nature as well as their ability to detect the presence of a wide variety of substances in a clinically valuable time window (ranging from hours to days, a clear advantage over serum testing). Additionally, there is a wide variety of commercial testing services available at relatively low costs. UDT in the clinical setting are divided into two categories: screening and confirmatory testing.

409

Chapter 57: Approach to the patient with abnormal drug screen

Urine drug screening typically utilizes an enzyme immunoassay (EIA), the main benefit of which is the ability to provide rapid results in an office-based, point-of-case setting. EIA is a purely qualitative assessment, utilizing competitive antibody binding to detect the presence of a particular compound in the urine. Typically, results are available in 5 minutes. The major drawbacks to EIA are that it fails to distinguish between different drugs in the same class while at the same time failing to demonstrate sufficient reactivity with (i.e., will not detect) all drugs in a given class. This is especially true in the case of opiates, benzodiazepines, and barbituates. For example, opiate EIAs are typically reactive with codeine and morphine and may fail to react with synthetic or semisynthetic opioids such as hydrocodone or oxycodone. Many opioid metabolites fail to react on EIA screening. Furthermore, EIA is subject to cross-reactivity with other agents (Table 57.1).[21] For these reasons, EIA may provide false-negative or false-positive results. Current best practice recommendations and guidelines of various sources agree; if urine drug screening with EIA is to be utilized, it is essential to also perform confirmatory testing with more sensitive/ specific assays. Confirmatory UDT is performed with gas chromatography/mass spectroscopy (GC/MS) or with liquid chromatography/mass spectroscopy (LC/MS-MS). In chromatography, a carrier (gas or liquid) medium moves a sample over a column which separates the molecules of interest in the sample via differential interaction with the column particles. The sample is then fed into a mass spectrometer which ionizes and then identifies a molecule based on a mass-to-charge ratio, a unique “fingerprint” which allows identification to occur with high sensitivity and specificity. MS-MS, or tandem mass spectroscopy, has even greater sensitivity and specificity than single-phase MS. These assays can provide a quantitative analysis, although the clinical utility of this quantitative assessment remains controversial.

What limitations should be considered when interpreting urine drug test results? The practitioner must be familiar with the limitations of the various UDT assays. As referenced above, EIA may provide false-negative and false-positive results for a number of reasons and should always be followed up with confirmatory testing. Although

410

Table 57.1. Cross-reactivity in ELISA urine drug screening[21]

Drug

Potential crossreactants

Amphetamines Amantadine

Methylphenidate

Buproprion

Phentermine

Chlorpromazine

Phenylephrine

Despiramine

Promethazine

Dextroamphetamine Pseudoephedrine Ephedrine

Ranitidine

Labetalol

Ritodrine

MDMA

Selegiline

Methamphetamine Trazodone Benzodiazepines Oxaprozin

Sertraline

Cannabinoids

Dronabinol

NSAIDs

Hemp food products

Proton pump inhibitors

Cocaine

Coca leaf products Topical cocaine anesthetics

Opioids

Dextromethorphan Quinines Heroin

Quinolines

Poppy seeds

Rifampin

Methadone

Diphenhydramine

Verapamil

Phencyclidine

Dextromethorphan Imipramine Diphenhydramine

Ketamine

Doxylamine

Meperidine

Ibuprofen

Tramadol

Adopted from reference[21].

GCMS and LCMS are considered the gold standard for UDT, they too have limitations. If a quantitative result is obtained with confirmatory testing, it should not be used to assess dose adherence. Wide variability in UDT results stems from individual differences in pharmacokinetics, pharmacodynamics, and pharmacogenetics. For example, morphine has a two- to threefold variability in absorption after oral, sublingual, and intramuscular dosing. Variation in absorption stems from a number of factors, including individual differences in transporter-mediated drug movement and metabolism (e.g., polymorphic cytochrome P-450 enzymes resulting in “ultra-rapid”

Chapter 57: Approach to the patient with abnormal drug screen

metabolizers). Variability in first-pass metabolism affects the amount of drug that reaches the systemic circulation. Without concomitant data of timed serum concentration the quantity of excreted drug in urine is not reliable evidence of dose compliance.[22] Physicians should be familiar with the basic metabolic pathways of prescribed opioids, as UDT assays often detect metabolites of these drugs rather than the parent drug itself (Figure 57.1). An important variable relevant to a UDT includes selection of a cut-off threshold, the level above and below which a sample is considered positive or negative, respectively. The Department of Health and Human Services (DHHS) has established specific cut-off guidelines for drug testing within the workplace. These cut-off levels are designed to minimize false-positive results (e.g., poppy seed cross-reactivity with cannabinoids or dextromethorphan crossreactivity with opiates). Levels below the cutoffs are

reported as negative results, which increases the rate of false-negatives. Their utility in the workplace notwithstanding, the DHHS cut-off levels are widely regarded as inappropriate for the clinical setting which monitors substance misuse.[21] Urine chemistry has a significant impact on UDT. Urine pH affects the concentration of drugs excreted. Dietary choices can alter the acid–base status of the urinary system: high-protein diets lead to more acidic urine and vegetarian diets to alkaline. Disease states such as diabetes mellitus and uremia may affect urine acidity and, consequently, the concentration of drugs in the urine. Several products are available to assist in adulterating urine samples in such a way so as to disrupt UDT assays. Typically, these products alter the chemical environment of the urine sample (e.g., pH). There are commercially available products to test for the integrity of a urine sample and for the presence of

Figure 57.1. Opioid metabolic pathways.

411

Chapter 57: Approach to the patient with abnormal drug screen

adulterants, although the sensitivity of these assays varies.[21] Other sources of error may occur at the time of collection, including swapping samples with a “clean” donor and dilution of urine samples with sink/toilet water. Other errors are possible during sample handling and processing. Urine sample integrity depends on evaluation of temperature (cooler samples raise suspicion of sample swapping or substitution), acidity (gross abnormalities may indicate the presence of adulterants), and creatinine and specific gravity (measures of concentration that help identify diluted samples).

What guidelines exist regarding drug testing in the clinical setting? Ultimately, the prescribing physician must determine his/her own best practice depending on the individual patient context and the nature of practice. A number of publications offer guidelines based on available evidence and experience. In 2012, the American Society for Interventional Pain Physicians (ASIPP) published “Guidelines for Responsible Opioid Prescribing in Chronic NonCancer Pain.” Included in the guidelines is a recommendation regarding the monitoring of patients on chronic opioid therapy. They suggest Adherence Monitoring utilizing a number of tools including Prescription Drug Monitoring Programs (PDMP), UDT, pill counts, and behavioral assessments. Additionally, these guidelines recommend applying a risk-based monitoring protocol in which the frequency of UDT and PDMP assessments is based on perceived risk for drug misuse.[23–25] Clear benefits to this approach include the evidence-based allocation of resources, thereby maximizing the benefit/cost ratio. On the other hand, some other practitioners recommend standardizing adherence monitoring across all patients on chronic opioid therapy with regularly scheduled UDT, regardless of perceived risk. Proponents of this measure argue that risk stratification protocols are imperfect and fail to identify patients who will later go on to misuse drugs. However, the application of different monitoring policies to different patients introduces practitioner bias and prejudice in treatment behaviors. That said, established risk factors for substance misuse have been demonstrated and screening tools validated such that patient selection for screening and associated containment of

412

associated healthcare costs may be a reasonable goal for practice development.[11–13]

3. Clinical management How would the results of this urine drug test affect your future management of this patient? The results of this patient’s initial drug screen are positive for opioid use in the absence of a prescribed opioid. However, a review of her other medications reveals the likely presence of dextromethorphan; a compound known to have cross-reactivity with opioids on enzyme immunoassay (EIA) drug screening. It is very possible that this patient is not using opioid medications and the results of her drug screen are simply a false-positive. If this patient is using opiate medications she is doing so inappropriately since you have not prescribed an opioid and she has not disclosed an opioid as one of her home medications. At this point, it remains inconclusive whether this patient is using opioid medications and more information is needed. The next step is to send her urine sample for confirmatory testing, which will provide more reliable information about the presence or absence of opioids in her system. Additionally, a search of a prescription drug monitoring program (PDMP) for your state and nearby states would be prudent and may uncover any opiate prescriptions that may have been filled by the patient surreptitiously.

Case study 2 A 35-year-old man with no significant past medical history was the victim of a car collision 6 months ago during which he suffered whiplash and multilevel lumbar disc herniation. He now suffers from persistent episodic tension-type headaches, chronic knee pain, and low back pain which persists despite lumbar microdiscectomy. He declines interventional options at the present time. His only prescription is tramadol which provides him only minimal relief. He has requested stronger medication from his primary care physician, prompting referral to your pain clinic. At the initial assessment, he denies using prescription drugs other than tramadol. He endorses occasional alcohol consumption (5–6 beers per week). He is a one packper-day cigarette smoker. Confirmatory urine drug

Chapter 57: Approach to the patient with abnormal drug screen

testing with GCMS at the initial clinic visit reveals a positive test for marijuana.

1. Clinical assessement What are this patient’s risk factors for illicit substance abuse? Commonly accepted risk factors for illicit substance abuse include gender (males more than females), personal history of substance abuse including alcohol and tobacco, family history of substance abuse, age < 45 years, history of preadolescent sexual abuse, organic brain syndrome, and the coexistence of psychiatric disorders and psychologic stress. Several studies have shown that patients with chronic pain are at additional risk for substance misuse if opioids or benzodiazepines are prescribed, if pain exists in more than three regions of the body, and if pain is the result of a motor vehicle accident. The presence of multiple prescriptions from multiple pharmacies and high daily doses of opiates (> 120 mg of oral morphine equivalents) indicates an increased likelihood of substance misuse.[11–13,26] The patient’s age, history of tobacco use, distribution of pain across three body regions, and history of pain from a motor vehicle accident suggest he is at increased risk for substance misuse.

What is the differential interpretation of these results? Potential interpretations of this test result include the following: – Recreational user – Addiction – Pseudoaddiction – Environmental exposure in the non-user The differential interpretation for this positive test result does not include a false-positive from crossreactivity since it was obtained with confirmatory gas chromatography/mass spectroscopy testing. Immunoassays used in urine drug screening, however, are subject to this source of error, and a positive test result should be evaluated as a possible falsepositive from cross-reactivity. Some controversy exists regarding the potential for environmental exposure to marijuana to cause a positive result on a UDT. In several studies, second-

hand exposure to marijuana smoke and consumption of hemp-containing foods (e.g., hemp-oil) led to positive urine test results. These exposures led to trace amounts of tetrahydrocannabinol (THC) in urine samples, but no study subjects met a level of 50 ng/ dL (established by the DHHS as a cut-off for a positive test result).[21]

2. Illicit drug abuse and detection What are the clinical trends and implications of substance abuse? In the last decade, fatal overdoses of prescription opioids have surpassed those from heroin and cocaine. In 2010, 2 million people initiated non-medical use of prescription opioids and 2.4 million people initiated use of marijuana. Of opioid abusers, 55% obtained these drugs from a friend or relative.[19]

What is the time frame for detection of marijuana in urine drug testing? What is the time frame for detection of other drugs with abuse potential? Detection times for marijuana use vary depending on quantity and frequency of use as well as individual differences in metabolism and excretion (Table 57.2). In general for urine cannabis testing, a single smalldose exposure is detectable for < 2 days and chronic large-dose exposure may be detectable for over 3 months.[27]

Would you include marijuana in your routine clinical drug screening? Why or why not? Marijuana continues to be a source of some debate and controversy. The issue of how to address marijuana in the chronic pain patient is a point of great practice variation. The United States Drug Enforcement Administration (DEA) lists marijuana as a schedule 1 drug and marijuana use, production, possession, and distribution is a federal offense (http:// www.whitehouse.gov/ondcp/state-laws-related-tomarijuana). For that reason, many physicians include an agreement in patient contracts to abstain from marijuana use and decline to prescribe opioids to marijuana users. Other practitioners reference state laws regarding usage of marijuana for medical purposes and adopt a

413

Chapter 57: Approach to the patient with abnormal drug screen

Table 57.2. Detection times for urine drug testing

Drug

Time

Alcohol

7–12 hrs

Amphetamines

48 hrs

Benzodiazepines Midazolam

12–48 hrs

Lorazepam

3 days

Diazepam

30 days

Cannabis Single use

3 days

Moderate (4×/wk)

5–7 days

Daily use

10–15 days

Chronic daily use

> 30 days

Cocaine

2–4 days

Ketamine

1–6 days*

Opiates Fentanyl/sufentanil

< 24 hrs

Fentanyl metabolites

48 hrs

Codeine

48 hrs

Heroin

48 hrs

Morphine

48–72 hrs

Hydromorphone

2–4 days

Oxycodone

2–4 days

Methadone

3 days

Phencyclidine

8 days

Adopted from references.[21,29–33] * Depending on assay and exposure.

practice of providing opioid prescriptions to known marijuana users. Still others adopt a “don’t ask, don’t tell” policy regarding marijuana, often asking for cannabis to be removed from the received results of a UDT. Many state medical licensure regulatory bodies consider marijuana users as drugs abusers, thereby opening up to liability the physician who willingly prescribes controlled substances to a known cannabis user. Research investigating the utility of marijuana in chronic pain has been limited by suboptimal study design; to date, no randomized controlled trials of marijuana per se have been performed. In 2001 a

414

systematic review of randomized trials of synthetic cannabinoids for the treatment of cancer pain, noncancer pain, and acute postoperative pain suggest that cannabinoids have analgesic effects on a par with the effects of codeine. The side effects of cannabinoid use are common and may limit the widespread clinical utility of cannabinoids.[28] Each practitioner must decide how medico-legal risk and personal beliefs will affect his/her pain practice with regard to marijuana and make full disclosure of this decision to patients.

3. Clinical management What are the next steps in your management of this patient? The patient’s positive test result is discussed with him in a frank and non-threatening manner. Regardless of the policy of the clinic regarding marijuana, it is important to understand why this patient has chosen to use marijuana. Is his use recreational in nature or is he attempting to self-manage an ailment? If his marijuana use is recreational, the impact of this on pain treatment is described. Will he continue to receive an opioid prescription from you in spite of his recreational use of an illicit drug? Is this use in breach of his clinical contract? Strict adherence to contracts and guidelines is advisable on the part of patients and practitioners. If his marijuana use is consistent with addiction, resources for addiction treatment and rehabilitation are offered. If he uses marijuana to supplement suboptimal treatment of chronic pain (i.e., pseudoaddiction), the goals of his treatment are revisited and his treatment plan may be revised.

Case study 3 A 60-year-old woman with a long-standing history of fibromyalgia comes to the clinic for follow-up. At the time of her referral 6 months ago, she was taking a purely opioid-based pain regimen prescribed by her primary care physician: a total of 24 mg of oral hydromorphone daily. You have begun transitioning her to a multimodal drug regimen of milnacipran, cyclobenzaprine, and hydrocodone/acetaminophen (APAP) tablets when needed for breakthrough pain. At her last visit 3 weeks ago she received a prescription for 30 tablets of hydrocodone/APAP with instructions to take one tablet every 6 hours as needed for pain, with

Chapter 57: Approach to the patient with abnormal drug screen

an agreement to limit herself to no more than an average of one tablet per day. Review of her UDT from today’s visit reveals urine negative for hydrocodone. She reports she is out of hydrocodone/APAP tablets and requests a refill. In a review of your state’s PDMP for this patient over the last month, you discover that she received a prescription for oral hydromorphone tablets from an emergency room physician 2 weeks ago.

1. Clinical assessment What is the differential interpretation of these findings? The differential interpretation of these findings includes the following:  Substance abuse/addiction  Illegal distribution of prescription medications  Pseudoaddiction

What steps would you take to determine a diagnosis? To determine a diagnosis, a UDT is performed to screen for the presence of the aforementioned opioids. In their absence, illegal distribution or selling of prescription drugs is suspected. If the UDT confirms the presence of these drugs, the practitioner may consider engaging the patient in an open and honest discussion of the PDMP findings to ascertain the intent of her behavior. Distinguishing between pseudoaddiction and drug abuse behaviors is

References 1.

American Psychiatric Association. Diagnostic and Statistical Manual of Mental Disorders: DSM-IV. Washington, DC: American Psychiatric Association. 1994: pp. 181–183.

2.

O’Brien CP, Volkow V, Li TK. What’s in a Word? Addiction Versus Dependence in DSM-V. Am J Psychiatry. 2006;163: 764–765.

3.

Weissman DE, Haddox JD. Opioid pseudoaddiction: an iatrogenic syndrome. Pain. 1989;36:363–366.

difficult and often subjective. The diagnosis may remain unclear for a time. “Red flags” in a history from a drug abuser (note the term abuse does not include pseudoaddiction) are “textbook” descriptions of conditions, declining a physical examination, reports of lost or stolen prescriptions, specific knowledge about opioids, and claiming non-opioid medications do not work. The practitioner must also be wary of the patient who works to elicit feelings of guilt or sympathy in the provider.[29]

2. Clinical management What are your next steps in management of this patient? Assuming this patient admits openly to her emergency room visit during the interview and cites her motivation as severe, intractable pain that is unrelieved by her current prescription regimen, a reasonable next step would be to adjust her current prescription plan accordingly and reinforce her pain clinic contract. It may be advisable to decrease the time between follow-up visits to minimize the opportunity for aberrant behavior until appropriate treatment goals are reached. Should drug-seeking behavior persist despite reasonable attempts to escalate pain control, suspicion of drug abuse, and perhaps illegal distribution of her medications (i.e., selling, theft from another household member) is raised, and continuing to provide opioid prescriptions should be reconsidered.

4.

Manchikanti L, Damron KS, McManua CD, Barnhill RC. Patterns of illicit drug use and opioid abuse in patients with chronic pain at initial evaluation: a prospective, observational study. Pain Physician. 2004:7:431–437.

6.

5.

Fishbain DA, Cole B, Lewis J, Rosomoff HL, Rosomoff RS. What percentage of chronic nonmalignant patients exposed to chronic opioid analgesic therapy develop abuse/addiction and/or aberrant drug-related behaviors? A structured evidence-based review. Pain Med. 2008;9: 444–459.

7.

8.

Manchikanti L, Pampati V, Damron KS, et al. Prevalence of prescription drug abuse and dependency in patients with chronic pain in western Kentucky. J Ky Med Assoc. 2003;101: 511–517. Cone EJ, Caplan YH, Black DL, Robert T, Moser F. Urine drug testing of chronic pain patients: licit and illicit drug patterns. J Anal Toxicol. 2008;32:530–543. Michna E, Jamison RN, Pham LD, et al. Urine toxicology screening among chronic pain patients on opioid therapy: frequency and predictability of abnormal

415

Chapter 57: Approach to the patient with abnormal drug screen

findings. Clin J Pain. 2007;23: 173–179. 9.

Substance Abuse and Mental Health Services Administration. Results from the 2007 National Survey on Drug Use and Health: National Findings. NSDUH Series H-34, DHHS Publication No. (SMA) 08–4343. Rockville, MD: Substance Abuse and Mental Health Services Administration. 2008.

10. Manchikanti L, Damron KS, Beyer CD, Pampati V. A comparative evaluation of illicit drug use in patients with or without controlled substance abuse in interventional pain management. Pain Physician. 2003;6:281–285. 11. Owen GT, Burton AW, Schade CM, Passik S. Urine Drug Testing: Current Recommendations and Best Practices. Pain Physician. 2012;15:119–133. 12. Webster LR, Webster RM. Predicting aberrant behaviors in opioid-treated patients: preliminary validation of the Opioid Risk Tool. Pain Med. 2005;6:432–442. 13. Manchikanti L, Cash KA, Damron KS, et al. Controlled substance abuse and illicit drug use in chronic pain patients: an evaluation of multiple variables. Pain Physician. 2006;9:215–226. 14. Noble M, Treadwell JR, Tregear SJ, et al. Long-term opioid management for chronic noncancer pain. Cochrane Database Syst Rev. 2010;1: CD006605. 15. Furlan AD, Sandoval JA, MailisGagnon A, Tunks E. Opioids for chronic noncancer pain: a metaanalysis of effectiveness and side effects. Can Med Assoc J. 2006;174:1589–1594. 16. Martell BA, O’Connor PG, Kerns RD, et al. Systematic review: opioid treatment for chronic back pain – prevalence, efficacy and

416

association with addiction. Ann Intern Med. 2007;146:116–127. 17. Manchikanti L, Ailnani H, Koyyalagunta L, et al. A systematic review of randomized trials of long-term opioid management for chronic non-cancer pain. Pain Physician. 2011;14:91–121. 18. Eriksen J, Ekholm O, Sjogren P, Rasmussen NK. Development of and recovery from long-term pain: a 6-year follow-up study of a cross-section of the adult Danish population. Pain. 2004;108: 154–162. 19. Substance Abuse and Mental Health Services Administration. Results from the 2010 National Survey on Drug Use and Health: Summary of National Findings, NSDUH Series H-41, HHS Publication No. (SMA) 11–4658. Rockville, MD: Substance Abuse and Mental Health Services Administration. 2011. 20. Birnbaum HG, White AG, Schiller M, et al. Societal costs of prescription opioid abuse, dependence and misuse in the United States. Pain Medicine. 2011;12:657–667. 21. Moeller KE, Lee KC, Kissack JC. Urine drug screening: practical guide for clinicians. Mayo Clin Proc. 2008;83:66–76. 22. Nafziger AN, Bertino JS. Utility and application of urine drug testing in chronic pain management with opioids. Clin J Pain. 2009;25:73–79. 23. Manchikanti L, Salahadin A, Sairam A, et al. American Society of Interventional Pain Physicians (ASIPP) Guidelines for Responsible Opioid Prescribing in Chronic Non-Cancer Pain: Part 1 – Evidence assessment. Pain Physician. 2012;15:1–66. 24. Manchikanti L, Salahadin A, Sairam A, et al. American Society of Interventional Pain Physicians (ASIPP) Guidelines for Responsible Opioid Prescribing in

Chronic Non-Cancer Pain: Part 2 – Guidance. Pain Physician. 2012;15:67–116. 25. Trescot AM, Helm S, Hansen H, et al. Opioids in the Management of Chronic Non-Cancer Pain: An Update of American Society of the Interventional Pain Physicians’ (ASIPP) Guidelines. Pain Physician. 2008;11:5–62. 26. Christo PJ, Manchikanti L, Ruan X, et al. Urine drug testing in chronic pain. Pain Physician. 2011;14:123–143. 27. Verstraete AG. Detection times of drugs of abuse in blood, urine, and oral fluid. Ther Drug Monit. 2004;26:200–205. 28. Campbell FA, Tramer MR, Carroll D, et al. Are cannabinoids an effective and safe treatment option in the management of pain? A qualitative systematic review. BMJ. 2001;323: 1–6. 29. Monheit B. Prescription drug misuse. Aust Fam Physician. 2010;39:540–546. 30. Parkin MC, Turfus SC, Smith NW, et al. Detection of ketamine and its metabolites in urine by ultra high pressure liquidchromatography-tandem mass spectrometry. J Chromatography B. 2008;876:137–142. 31. Silverstein JH, Rieders MF, McMullin M, Schulman S, Zahl K. An analysis of the duration of fentanyl and its metabolites in urine and saliva. Anesth Anal. 1993;76:618–621. 32. Mozayani A. Ketamine: effects on human performance and behavior. Forensic Sci Rev. 2002;14:123–131. 33. Fraser AD, Bryan W, Isner AF. Urinary screening for midazolam and its major metabolites with the Abbott ADx and TDx analyzers and the EMIT d.a.u. benzodiazepines assay with confirmation by GC/MS. J Anal Toxicol. 1991;15:8–12.

Section 7 Chapter

58

Special Topics

Physician exposed to excessive radiation Vikram B. Patel

Interventional pain physician has to go to occupational medicine office regarding radiation exposure

e. The reason for the state official’s visit is to educate the personnel regarding the exposure risks and to inspect and evaluate the radiation-producing equipment for any flaws.

A 38-year-old interventional pain specialist has been in practice for 3 years. The practice is growing and is very busy. He is performing close to 50 procedures a week under fluoroscopic guidance. He recently got a notice from the state regarding his excessive exposure and was interviewed by a state department official in his clinic. The state official also evaluated the fluoroscopy machine and interviewed the radiology technician.

The risk of occupational exposure to physicians performing interventional procedures under fluoroscopic guidance cannot be underestimated. In the past 30 years the use of fluoroscopically guided procedures has increased dramatically. In a study published in 2012 the mean operator radiation dose per case measured over personal protective devices at different anatomic sites on the head and body ranged from 19–800 (median ¼ 113) μSv at eye level, 6–1180 (median ¼ 75) μSv at the neck, and 2–1600 (median ¼ 302) μSv at the trunk. Operators’ hands often received greater doses than the eyes, neck, or trunk. Large variations in operator doses suggest that optimizing procedure protocols and proper use of protective devices and shields might reduce occupational radiation dose substantially.[2]

1. What is the reason for the state inspection? a. Every state is required by law to keep track of radiation producing equipment and license it after a full inspection. b. A yearly inspection and evaluation of the equipment is also required for compliance and the certificate is required to be displayed in the vicinity of the machine. c. OSHA (Occupational Safety and Health Administration) requires that “Every employer shall supply appropriate personnel monitoring equipment, such as film badges, pocket chambers, pocket dosimeters, or film rings, and shall require the use of such equipment.”[1] d. Excessive radiation exposure is monitored and reported to the state by the radiation exposure monitoring device manufacturing companies on a regular basis and the exposure is linked to the social security number of the person for cumulative lifetime exposure and to monitor its limits.

2. What are the OSHA standards regarding ionizing radiation?[1] Some of the important OSHA regulations regarding ionizing radiation are: a. Definition: Radiation includes alpha rays, beta rays, gamma rays, x-rays, neutrons, high-speed electrons, high-speed protons, and other atomic particles; but such a term does not include sound or radio waves, or visible light, or infrared or ultraviolet light. b. The dose to the whole body, when added to the accumulated occupational dose to the whole body, shall not exceed 5 (N18) rems, where “N” equals the individual’s age in years at his last birthday.

Case Studies in Pain Management, ed. Alan David Kaye and Rinoo V. Shah. Published by Cambridge University Press. © Cambridge University Press 2015.

417

Chapter 58: Physician exposed to excessive radiation

c. The employer maintains adequate past and current exposure records which show that the addition of such a dose will not cause the individual to exceed the amount authorized in this subparagraph. As used in this subparagraph dose to the whole body shall be deemed to include any dose to the whole body, gonad, active bloodforming organs, head and trunk, or lens of the eye. d. No employer shall permit any employee who is under 18 years of age to receive in any period of one calendar quarter a dose in excess of 10% of the limits. e. Every employer shall make such surveys as may be necessary for him to comply with the provisions in this section. Survey means an evaluation of the radiation hazards incident to the production, use, release, disposal, or presence of radioactive materials or other sources of radiation under a specific set of conditions. When appropriate, such evaluation includes a physical survey of the location of materials and equipment, and measurements of levels of radiation or concentrations of radioactive material present. f. Every employer shall supply appropriate personnel monitoring equipment, such as film badges, pocket chambers, pocket dosimeters, or film rings, and shall require the use of such equipment. g. Radiation area. Each radiation area shall be conspicuously posted with a sign or signs bearing the radiation caution symbol described in subparagraph (1) of this paragraph and the words:

4. What are the penetrating characteristics of various radiation beams? a. Please see Figure 58.1 for understanding the penetrating power of the various ionizing radiation beams (ultraviolet (?), alpha, beta, x-rays, gamma, neutron). b. Non-ionizing radiation beams examples: microwave, radio waves, infrared, radiofrequency. c. Certain important terminology regarding radiation exposure:[4] i. Exposure: The intensity or quantity of radiation. It is measured in roentgen (R) units. The System Internationale (SI) unit is coulomb (C). ii. Radiation absorbed dose (RAD): Actual absorbed energy by a given biologic tissue sample. It applies to the energy deposited by any kind of radiation in any kind of material. The SI unit is gray (Gy). iii. Radiation equivalent man (rem): Unit used to describe the biologic effects of radiation. It is affected by different types of radiation and irradiation conditions. The SI unit is sievert (Sv) (100 rem ¼1 Sv). iv. Millisievert (mSv): Unit used to measure amount of radiation exposure and measured by x-ray dosimeters. The total yearly dosage should not be more than 50 mSv. v. For all practical purposes 1 R¼1 RAD¼1 rem. vi. The dosage of radiation is cumulative over lifetime and should not be underestimated. In most situations, these effects are permanent.

CAUTION RADIATION AREA

3. Who sets and enforces the medical radiation dosage standards?[3] Individual states regulate the practice of medicine by licensing doctors, including radiologists. A licensed doctor is then permitted to use his or her experience and discretion when deciding how much radiation should be used to diagnose or treat a patient. For that reason, the use of radiation in medical imaging is exempt from federal dose limits.

418

Figure 58.1. Ionizing radiation beams and their penetrating power. Only concrete can stop the x-ray beam. Air can also act as a barrier due to rapid decay of the beam over distance.

Chapter 58: Physician exposed to excessive radiation

Table 58.1. Ionizing radiation exposure, biologic effects, and onset

Biologic effect Exposure (rem) single dose

Time of onset of symptoms

5–10

Changes in blood chemistry

50

Nausea

55

Fatigue

70

Vomiting

75

Hair loss

90

Diarrhea

100

Hemorrhage

400

Possible death

Within 2 months

1000

Destruction of intestinal lining, internal bleeding, and death

Within 1–2 weeks

2000

Damage to the central nervous system, loss of consciousness, death

Within minutes, hours, days

Hours

2–3 weeks

5. What are the biologic effects of ionizing radiation? a. See Table 58.1 for the biologic symptoms of ionizing radiation. Although there is no established “safe” level of radiation, these values are representative of tolerable amounts of radiation by a human body before any symptoms appear.[5] b. These symptoms are cumulative. EG symptoms of vomiting would already have caused changes in blood chemistry and bone marrow. c. A single dose of 5000 RAD can cause cerebral edema and death.

6. What are the maximum permissible doses (MPD) of radiation? a. There are certain levels of radiation which are established for an acceptable dose of radiation while performing fluoroscopic procedures (see Table 58.2).

Table 58.2 Maximal permissible doses of radiation to body

Organ

MPD

Thyroid

50 rem

Extremities

50 rem

Ocular lens

15 rem

Gonads

50 rem

Whole body

5 rem

Pregnant women

0.5 rem to fetus

b. The typical exposure during a chest x-ray is about 0.02 mSv; interventional fluoroscopic procedures can be as high as 250 times a regular chest x-ray.[3] c. The exposure to a physician during a typical lumbar epidural steroid injection is about 0.03 mrem at a distance of 1 m.

7. What can I do to reduce my exposure to radiation while performing the fluoroscopic procedures? a. Distance is one of the main factors that reduce the exposure. The decay in the penetrating strength of x-rays is exponential such that a distance of 1 m reduces the exposure by > 99%. However, scattered radiation beams should not be ignored. b. Exposure time is another factor that should be considered. A continuous delivery of x-ray is far more exposing than a single shot image, hence short bursts of fluoroscopy rather than continuous mode is preferable. c. Use of collimation: Using collimation to reduce the emission of x-rays is one of the most effective methods to reduce the exposure emitted by a fluoroscopy machine. It may even help enhance the image by eliminating overexposed fields (such as thoracic area) (Figure 58.2). The collimation area can be rotated to obtain proper coverage of the intended target. Metal plates block the emitting x-rays and don’t allow them to exit the tube. d. Use of pulse mode on a fluoroscopy machine: Pulsed mode delivers x-rays in pulses rather than continuous beam. Some machines allow adjustment of the pulse rate and can reduce the

419

Chapter 58: Physician exposed to excessive radiation

A

B

C

Figure 58.2. Image (B) is without collimation, image (A) shows circular (Iris) collimation while image (C) shows linear collimation.

exposure dramatically; however the image quality can suffer and become grainy. Using pulsed mode for regular images while advancing the needle or while marking the entry point should be considered. e. Use of barriers: Using leaded barriers to protect oneself is the most important protection if the above two are not practical. The following barriers are commonly used: i. Leaded apron: These come in various types, but a wrap-around design is preferable to limit the exposure while turned around and not facing

420

the fluoroscopy unit. Typical lead aprons have a 0.5 mm equivalent lead. A separate thyroid shield of similar thickness is also a must.[6] ii. Leaded eye wear (typical leaded glasses provide a 0.35 mm equivalent thickness).[7] iii. Leaded radiation attenuating gloves should be used (0.25 mm lead equivalent) while working within the field of x-ray exposure. iv. Extremities may be protected with the use of movable barriers or fixed flexible barriers attached to the procedure table.

Chapter 58: Physician exposed to excessive radiation

8. What are the recommendations by US Environmental Protection Agency regarding radiation exposure?[8] a. It is strongly recommended that, other than for the patient being examined, only staff and ancillary personnel required for the procedure, or those in training, be in the room during the fluoroscopic examination. b. No body part of any staff or ancillary personnel involved in a fluoroscopic examination should be in the primary beam; if not avoidable, it should be minimized. c. It is essential that all personnel in the room during fluoroscopic procedures be protected from scatter radiation by either whole-body shields or protective aprons. d. The two-dosimeter method (one dosimeter should be worn at the collar outside the apron. A second dosimeter may be worn on the abdomen under the apron) may be preferable for monitoring personnel in the room during highdose interventional procedures. Monthly dose monitoring can also be implemented to ensure that staff members who use garments with < 0.5 mm lead equivalent thickness continue to maintain an occupational dose below the required dose limits. e. Thyroid and eyes should be protected if the potential exposure to the worker will exceed 25% of the annual regulatory dose limits for those organs. f. It is strongly recommended that lead personnel protective equipment (e.g., aprons, gloves, thyroid collars) be evaluated at least annually for lead protection integrity using visual and manual inspection. g. Each facility should train staff who operate x-ray producing equipment or who are routinely exposed to radiation by the equipment. Training should be provided initially prior to utilization of the equipment and at least annually thereafter. The training should be commensurate with risk to the staff and to the patient. It should include the risks from exposure to ionizing radiation, regulatory requirements, recommendations of this guidance document, facility requirements, proper operation of the equipment, methods for maintaining doses to staff within regulatory

limits and as low as reasonably achievable, and guidance for protecting the patient and embryo or fetus. h. It is strongly recommended that the technical quality assurance program includes testing by a qualified physicist of all imaging equipment producing x-rays. The equipment should be tested after installation but before first clinical use, annually thereafter, and after each repair or modification that may affect patient dose or image quality. Testing after repair or modification should be performed before clinical use of the equipment. The testing, including a summary of methods, instruments used, measurements, and deficiencies identified, should be documented in a written report signed by the qualified physicist. i. Each medical center should have a written policy for the safe use of fluoroscopic equipment. This policy should apply to all fluoroscopy equipment, whether fixed, mobile, or portable, e.g., mobile C-arm systems and mini C-arm systems. This policy should: i. require testing of the fluoroscopic equipment by or under the direction of a qualified physicist; ii. require training and credentialing of persons operating or directing the operation of fluoroscopic equipment; iii. specify procedures for the safe use of the equipment, including dose management and recordkeeping; and iv. require a clinical QA/QI program for fluoroscopy.

Summary a. In summary, an interventional pain physician should be very aware of the radiation exposure hazards and take every precaution to avoid excessive radiation exposure. b. The most effective measures are:[9] i. Distance: stay as far as possible from the x-ray tube (the smaller cylindrical portion of the C-Arm). ii. Use burst mode rather than continuous fluoroscopy in most instances. iii. Use collimation and pulsed mode whenever possible.

421

Chapter 58: Physician exposed to excessive radiation

iv. Position the x-ray tube on the side opposite the operator.[10] v. Use radiation attenuating gloves,[10] lead apron (preferably wrap-around), thyroid shield, and leaded eye wear. vi. Monitor radiation exposure on a regular basis and wear two dosimeters for better measurement of the exposure as well as the protective barriers.

References 1.

2.

3.

4.

422

OSHA. Occupational Safety and Health Standards: Ionizing Radiation Standard, 29 CFR 1910.96: Occupational Safety and Health Administration. 1996. Kim KP, Miller DL, Berrington de Gonzalez A, et al. Occupational radiation doses to operators performing fluoroscopicallyguided procedures. Health Phys. 2012;103(1):80–99. US Environmental Protection Agency. Radiation Protection Guidance for Diagnostic and Interventional X-Ray Procedures: Federal Guidance Report No. 14. Washington, DC: EPA.GOV. 2013. http://www.epa.gov/ radiation/federal/fgr-14.html. United States Nuclear Regulatory Commission. Radiation Protection and the NRC. Washington, DC: United States

vii. Follow ALARA measures (as low as reasonably achievable) with regards to the amount of radiation utilized. viii. Cover all personnel in the procedure room with appropriate barriers and the patient’s body parts not being treated. ix. Remember that the radiation dose is cumulative over lifetime and the effects in most situations are permanent.

Nuclear Regulatory Commission. 2010. http://www.nrc.gov/ reading-rm/doc-collections/ nuregs/brochures/br0322/r1/ br0322r1.pdf. 5.

6.

7.

US Environmental Protection Agency. Health Effects of Ionizing Radiation. Washington, DC: EPA.GOV. http://www.epa.gov/ radiation/understand/ health_effects.html. Lee SY, Min E, Bae J, et al. Types and arrangement of thyroid shields to reduce exposure of surgeons to ionizing radiation during intraoperative use of C-arm fluoroscopy. Spine. 2013 [Epub ahead of print]. Burns S, Thornton R, Dauer LT, et al. Leaded eyeglasses substantially reduce radiation exposure of the surgeon’s eyes during acquisition of typical fluoroscopic views of the hip and

pelvis. J Bone Joint Surg Am. 2013;95(14):1307–1311. 8.

US Environmental Protection Agency. FEDERAL GUIDANCE REPORT NO. 14: Radiation Protection Guidance for Diagnostic and Interventional X-Ray Procedures. Washington, DC. 2012. EPA-402R–10003.

9.

Lee K, Lee KM, Park MS, et al. Measurements of surgeons’ exposure to ionizing radiation dose during intraoperative use of C-arm fluoroscopy. Spine. 2012; 37(14):1240–1244.

10. von Wrangel A, Cederblad A, Rodriguez-Catarino M. Fluoroscopically guided percutaneous vertebroplasty: assessment of radiation doses and implementation of procedural routines to reduce operator exposure. Acta Radiol. 2009; 50(5):490–496.

Section 7 Chapter

59

Special Topics

Patient becomes paralyzed following a lumbar transforaminal epidural steroid injection Scott E. Glaser

Case study The patient, a 47-year-old male lawyer, consulted with his friend, an orthopedic doctor, after suffering the onset of right lower back pain and associated right leg pain wrapping around his thigh to his anterior knee while training for a marathon 3 weeks ago. There was no numbness and the patient did not feel there was any weakness although certain activities were difficult to perform secondary to the pain. The pain was severe enough that he could not continue his training regimen. The lawyer was able to work but could not sit for more than 15 minutes and rescheduled an upcoming trial. The surgeon’s differential diagnosis included a hip injury, knee injury, or radiculopathy. Upon examination, the patient had no pain to palpation over the knee or hip, had full range of motion (FROM) of both joints without pain, and there was no evidence of joint swelling or laxity and no crepitus. There was no evidence of neurologic deficit on motor testing or sensory testing. The surgeon did note a positive straight leg raise at 45 degrees with exacerbation of pain in the buttock. Reverse straight leg raise (SLR) was also positive. The patient’s patellar reflex was subtly decreased on the right compared to the left. Because of the neurologic deficit and the severity of the patient’s discomfort, the surgeon considered diagnostic testing to assess his radiculopathy. He did not order an EMG as typically EMGs will not show evidence of nerve damage for 8 weeks after the onset of symptoms. Additionally, EMGs are associated with low sensitivity, i.e., a high false-negative rate. He also did not order a CT scan because of its decreased sensitivity compared to MRI for imaging soft tissue anatomy and the risk of exposure to ionizing radiation. He did order a MRI of the patient’s lumbar spine which revealed a

dessicated L3–4 disc with a right lateral protrusion and a high-intensity zone. It also revealed disc dessication and loss of disc height at L4–5 and a central bulge as well as similar findings at L5-S1. The orthopedic doctor diagnosed a L3 radiculopathy and recommended conservative, expectant, treatment utilizing physical therapy as well as symptom management with medications. His decision was based on the consensus that radicular pain should be treated conservatively when possible for the first 6–8 weeks. He recommended a NSAID for control of pain and the adjuvant medication Neurontin to reduce the neuropathic components of the pain (burning, tingling, shock-like sensations). He also prescribed Hydrocodone as needed for severe pain with appropriate warnings regarding the risks of opioids. The patient wanted an injection or surgery immediately because of the severity of his symptoms and the effect on his livelihood. The surgeon did not accede to his request but offered a course of oral steroids and physical therapy. He explained the increased risk of systemic versus depot steroids including a higher risk of hyperglycemia, avascular necrosis of the hip, mood disturbances, and adrenal suppression. He also discussed the higher risk of therapeutic failure secondary to the lack of site specificity. He also discussed the risks of surgery including peri-operative complications (DVT, infection) and future complications of even a simple discectomy (worsening pain, destabilization of the spine, epidural fibrosis). The lawyer chose conservative treatment without steroids. Upon follow-up 2 weeks later, the patient’s pain was worse and he again requested more specific treatment of the source of his pain. The surgeon offered him a fluoroscopically directed epidural steroid injection. He discussed the

Case Studies in Pain Management, ed. Alan David Kaye and Rinoo V. Shah. Published by Cambridge University Press. © Cambridge University Press 2015.

423

Chapter 59: Patient becomes paralyzed following a lumbar transforaminal epidural steroid injection

alternative routes (caudal, inter-laminar, and transforaminal). He recommended the transforaminal route due to its higher level of site specificity, i.e., delivery of the highest concentration of active ingredient (depot steroid, local anesthetic) to the area of inflammation/injury, i.e., the disc–nerve interface. He stated that there was not unequivocal literature supporting its increased efficacy versus the other routes but that the ability to place the highest concentration of depot steroid near the site of nerve inflammation considering that the disc was displaced laterally and almost outside the epidural space made it most likely to provide symptom relief. He did review all of the risks of all of the routes and reviewed the risks of steroids. The orthopedic surgeon utilized the traditional safe triangle technique for the procedure and inserted a 22-g 3.5 inch spinal needle into the superior and anterior aspect of the L3–4 foramen. Lateral views confirmed placement just under the pedicle posterior to the posterior vertebral body of L3. AP view revealed the needle bevel to be under the pedicle at the 6:00 position if the pedicle is imagined as a round clock face. He then injected contrast, Isovue 300, under live fluoroscopy on an AP view and visualized an outline of the nerve root and flow into the epidural space superiorly. He then attached a syringe containing triamcinilone 40 mg, 0.5 ml of Marcaine 0.5%, and 0.5 ml of preservative free normal saline (PFNS) and injected the solution slowly. On an AP view, the dye was obviously diluted by the steroid solution and had spread further along the nerve root. The surgeon removed the needle and left the room. The patient was rolled on to the gurney and it was noted that he could not bend his knees. In the recovery area it was noted that he could barely move his ankles and he felt numb up to his umbilicus. The patient’s vital signs remained stable. After examining the patient and calling an anesthesiologist colleague, the surgeon believed that he had caused a spinal anesthetic. He observed the patient for a few hours but the decreased sensation and movement did not begin to resolve and actually spread higher on the abdomen and the patient stopped moving his ankles. He then transferred the patient to the hospital. A neurosurgeon and neurologist were consulted in the ER. They recommended an MRI as both suspected an epidural hematoma. The MRI revealed subtly increased signal intensity in the central gray matter of the thoracic spinal cord. The neurosurgeon then

424

recommended treatment with high-dose steroids on an empiric basis based on his experience with spinal cord trauma. A rehab specialist was consulted. The next day, another MRI was performed which revealed an increased signal in the gray matter of the spinal cord on a T2 image and swelling in the distal thoracic cord. The patient had no notable improvement from the steroid treatment.

1. What are the possible complications after a transforaminal epidural steroid injection? Possible complications from a transforaminal epidural steroid injection (TFESI) include epidural abscess, discitis, epidural hematoma, spinal headaches, intravenous injection, trauma/damage to the exiting nerve root with persistent neurologic deficit and/or pain, steroid side effects/complications secondary to systemic steroid exposure, and spinal cord damage and paraplegia.[1] Spinal cord damage including paraplegia following transforaminal injections in the lumbar and thoracic spine is now a well known and feared phenomena.[1–3] There are 18 cases in the literature and the knowledge of many more that are sub judice.

2. What is the vascular anatomy of the blood supply to the spinal cord? The anterior part of the thoracolumbar spinal cord is supplied by the anterior spinal artery (ASA), which receives feedings from the anterior radicular medullary arteries (ARMAs). The largest ARMA is called the artery of Adamkiewicz (AKA). This artery is known for its size, hairpin turn, and remote anastamosis with the ASA. It reinforces blood flow to the majority of the thoracolumbar and sacral spinal cord. It is typically a singular artery although in some patients there may be more than one. The AKA has been found in every single foramen in the lower thoracic and lumbar spine. It is more often located in the upper lumbar and lower thoracic foramina on the left side.[4,5] ARMAs travel from the aorta around the waist of the vertebra and then enter the intervertebral foramina. These segmental vessels that pass through the intervertebral foramina divide into posterior and anterior radicular arteries, which follow the posterior

Chapter 59: Patient becomes paralyzed following a lumbar transforaminal epidural steroid injection

and anterior roots respectively. At variable levels the radicular arteries continue coursing medially to anastamose with the anterior and the posterior spinal arteries.[4,5]

3. Where is the location of the arteries in the foramen? The location of the ARMAs and artery of Adamkiewicz within the lumbar and thoracic foramen is consistent. These arteries are always found in the anterior and superior aspect of the foramen. They are anterior and superior to the dorsal root ganglion/ventral root complex. The most inferior of these arteries was found in the Murthy study just caudal to the inferior end plate, i.e., at the uppermost portion of the disc.[5–7] They become invested with the anterior and superior aspect of the nerve root.

4. What is the pathogenesis and mechanism of injury leading to paraplegia? The mechanism of injury leading to reduced blood flow through the arterial radicularis magna (ARM) or another vital medullary artery is secondary to local phenomena due to disruption of the vessel and possibly distal embolization. Possible local phenomena include intimal flaps, dissection, vasospasm, locally produced thrombus, and transection of the artery. Sources of emboli include distal thromboembolism following local thrombus formation, particulate steroid, and air bubbles contained in syringes and tubing. These mechanisms are not exclusive of each other and may coexist.[3,8–10] Evidence is mounting that the primary cause of ischemia is secondary to local phenomena related to arterial trauma. This likelihood was initially based on a case report in which no intravascular uptake was noted and dye was diluted in a washout picture confirming extravascular placement of the injectate.[3] These local mechanisms of injury have been confirmed by another case report which included the only spinal angiogram performed following one of these complications.[8] In this case, the segmental artery at the level of the procedure could not be cannulated secondary to clot a few hours after the procedure and the angiogram performed at an adjacent level revealed irregularities in the collateral

arteries. The neuroradiologists concluded this was secondary to local thrombus and vasospasm.

5. What changes in technique have been proposed to improve the safety of TFESI? Consensus is growing that the only foolproof method to ensure the lowest risk of paraplegia is to perform the procedure in a manner that reduces the risk of interaction between the bevel of the needle and the arteries in the foramina.[2,3,6,7,9,11] This requires that the safe triangle technique needs to be abandoned. It has been shown that the exterior diameter of a 22-g spinal needle is only slightly smaller than the external diameter of the average ARMA at the level of the foramen and therefore most likely larger than the internal diameter. Thus, the difficulty in cannulating the artery blindly, if even physically possible, detecting intravascular placement, and then safely removing the needle is obvious. The likelihood of arterial injury, intimal disruption, dissection, vasospasm, and local thrombus is also obvious. The damage to these small arteries would be potentiated by all of the movements of the needle that occur during the performance of the procedure including changing syringes, twisting Luer locks, etc. There have been new anatomic and radiologic studies of the arterial blood supply to this area of the spinal cord and especially the ARM or artery of Adamkiewicz.[6,7] These confirm the random, unpredictable nature of the thoracic and lumbar foraminal levels traversed by the medullary arteries explaining cases of paraplegia following a right side low lumbar transforaminal injection. They also reaffirm the consistent location of these arteries within the foramen in the superior and anterior aspect. Therefore, it is recommended that an approach be utilized that places the needle below the nerve and posterior to the disc in Kambin’s triangle (Figures 59.1 and 59.2).[12] This provides the greatest margin of safety as it minimizes the risk of all of the possible mechanisms of injury including local injury and embolization. This approach has also been shown to be efficacious.[13–15] Another alteration to the technique that has been promoted is to utilize needles without cutting bevels, i.e., blunt needles, conceivably reducing the risk of trauma to and cannulation of the artery in the

425

Chapter 59: Patient becomes paralyzed following a lumbar transforaminal epidural steroid injection

Pedicle

Safe triangle Ventral ramus Disc

Dorsal ramus Figure 59.1. The “safe triangle.” This triangle is viewed from an A-P view. Note that the hypotenuse is the superior aspect of the exiting nerve root.

detect intravascular cannulation and injection to reduce the risk of spinal cord damage. The proponents of these techniques recommend digital subtraction imaging which increases the sensitivity for the detection of vascular dye flow. They also recommend a lidocaine test dose and monitoring for a transient neurologic deficit of some sort to alert the practitioner that the needle is in an important artery.[20,21] Another editorial challenges the role of these ad hoc safety initiatives and cautions that wholesale adoption of these approaches may lead physicians away from fully examining all possible root causes; the danger is that a false sense of security occurs while increasing the difficulty of performing a therapeutic procedure.[22]

6. What are the pros and cons of the suggested technique modifications?

Figure 59.2. Schematic drawing of Kambin’s triangle. This triangle is viewed obliquely. Note the hypotenuse is the inferior aspect of the exiting nerve root. Also note that the upper aspect of the disc and inferior endplate of the cephalad vertebra may be included in the triangle. The ARM has been found at those levels.[7]

foramen.[16] A further technique modification that has been proposed is the use of non-particulate steroid to reduce the risk of embolus of particulates, theoretically reducing the risk of occlusion and infarction.[17,18] Lastly, some experts contend that the primary cause of the infarcts is secondary to embolism of the steroid particles and detection of intravascular arterial cannulation and removing the needle from the artery is sufficient to reduce the risk of spinal cord damage. A recent editorial challenges the role of particulate steroids as the culprit in thoracolumbar TFESI paraplegia.[19] They promote enhanced techniques to

426

The pros of performing transforaminal injections in areas of the foramen remote from the pathway of the medullary arteries are clear. This eliminates the risks associated with trauma to the artery as well as the risk of embolism while maintaining the ability to instill therapeutic medications near the nerve root and in the epidural space. It appears that the only risk possibly increased by this approach is the increased risk of intra-annular or intra-discal dye injection. This may in turn lead to a higher risk of discitis. This can be a significant complication with the sequelae of death secondary to sepsis or local bony destruction requiring surgical debridement and spinal fusion. However, most cases of discitis respond to conservative treatment without sequelae. This is in stark contrast to paraplegia. Thus far, there are no case reports of discitis following a TFESI utilizing the Kambin triangle approach. The pros of utilizing a blunt needle are felt to be reduced risk of cannulation and damage to the artery should it be encountered theoretically reducing the risk of local phenomena (thrombus, dissection, intimal flaps, muscle spasm) and steroid embolism. The cons are greater patient discomfort and difficulty in placement and, therefore, widespread adaptation of these needles. Additionally, it is not clear that the trauma to the artery from such a needle would be sufficiently less than a sharp bevel and whether the possible local sequelae leading to ischemic injury to the spinal cord would be eliminated.

Chapter 59: Patient becomes paralyzed following a lumbar transforaminal epidural steroid injection

The pros of utilizing non-particulate steroids are a theoretically reduced risk of arterial occlusion. The cons are a lack of protection from all other mechanisms of injury. Additionally, this medication may act like an air embolism in watershed arteries such as those to the spinal cord unless completely and rapidly miscible with the blood. Lastly, there appears to be diminished therapeutic effectiveness compared to particulate steroids. The pros of utilizing enhanced techniques to detect intravascular cannulation and injection (DSI, lidocaine test doses) are a reduced risk of injecting medications that may cause distal embolisms. The cons, again, are the lack of protection from all other mechanisms of injury, errors in detection, and the dissociation between anatomic (DSI) and physiologic (lidocaine test dose) diagnostic accuracy. There is also much greater exposure to radiation when utilized in the DSI mode. Additionally, there is already a case report of paralysis following a TFESI during which DSI was utilized and failed to detect intravascular uptake.[23]

7. What is the treatment of paraplegia secondary to infarction? The treatment of this rare complication has been palliative in nature. Spinal angiograms have been recommended in an attempt to find a reversible lesion–muscle spasm dissection, or intimal flap. However, given the sensitivity of neural cells to hypoxemia, even if such a lesion was found and corrected, it is difficult to conceive of a significant difference in outcome. High-dose steroids have been utilized for empiric reasons based on their reported efficacy in cases of spinal cord trauma in mitigating neurologic damage. There is currently little evidence that they would provide similar benefit in cases of ischemic neurologic injury. Additionally, treatment of the closest disease model to spinal cord infarcts (cerebrovascular accidents) with steroids is controversial at best and there is not strong scientific evidence to support it. Lastly, exogenous steroids are associated

References 1.

Manchikanti L, et al. An up-date of comprehensive evidence-based guidelines for interventional techniques of chronic spinal pain: Part II: Guidance and recommendations.

with significant complications themselves as enumerated by the surgeon in this case.

8. What ethical and legal ramifications are there surrounding this procedure and the possible complications? The legal and ethical ramifications surrounding pain management interventions are no different than any other medical procedure. The physician’s decisionmaking process and technical expertise must always be utilized in an attempt to improve the patient’s medical condition but in a manner which above all does no harm. When complications occur, physicians are held to standards of care which are no longer local but generally accepted based on the dissemination of information. The standard of care evolves as medical knowledge and experience accrues. However, practitioners who have paralyzed patients with this technique have been accused of practicing below the standard of care because they have not utilized additional precautions such as DSI and test doses to detect intravascular uptake. Yet no pain management society and no guidelines state that these maneuvers are the standard of care and they are not routinely utilized. Additionally, these technique modifications are theoretical and proof of their efficacy in preventing vascular catastrophes is a hypothesis which cannot be proven secondary to the exceedingly low incidence of this complication. Experts who testify are ethically obliged to acknowledge the evolving nature of the standard of care as well as the lack of evidence supporting certain technique modifications. Their ethical duty is to attempt to clarify the medical issues in the case without falling prey to becoming advocates for either the plaintiff or the defense.[24] Experts have an ethical duty to conduct a root cause analysis and understand the limitations of their knowledge and predictive capacity, e.g., hindsight bias, survivorship bias, and Black Swan events.[9,22] traditional technique. Pain Physician. 2013;16:321–334.

Pain Physician. 2013;16: S49–S253. 2.

Atluri S, Glaser SE, Shah RV, Sudarshan G. Needle position analysis in cases of paralysis from transforaminal epidurals: consider alternative approaches to

3.

Glaser SE, Falco F. Paraplegia following a thoracolumbar transforaminal epidural steroid injection. Pain Physician. 2005; 8:309–314.

427

Chapter 59: Patient becomes paralyzed following a lumbar transforaminal epidural steroid injection

4.

5.

6.

7.

8.

9.

Crock HV, Yoshizawa H. Origins of arteries supplying the meninges and spinal cord. In The Blood Supply of the Vertebral Column and Spinal Cord in Man. Chicago: RR Donnelly & Sons. 1977. Alleyne CH Jr, Cawley CM, Shengelaia GG, Barrow DL. Microsurgical anatomy of the artery of Adamkiewicz and its segmental artery. J Neurosurg. 1998;89:791–795. Murthy NS, Maus TP, Behrns CL. Intraforaminal location of the great anterior radiculomedullary artery (artery of Adamkiewicz): a retrospective review. Pain Med. 2010;11:1756–1764. Kroszczynski AC, Kohan K, Kurowski M, Olson TR, Downie SA. Intraforaminal location of thoracolumbar anterior medullary arteries. Pain Med. 2013;14 (6):808–812. Lyders EM, Morris PP. A case of spinal cord infarction following lumbar transforaminal epidural steroid injection: MR imaging and angiographic findings. AJNR Am J Neuroradiol. 2009;30:1691–1693. Glaser SE, Shah RV. Root cause analysis of paraplegia following transforaminal epidural steroid injections: the ‘unsafe’ triangle. Pain Physician. 2010;13:237–244.

10. Tiso RL, Cutler T, Catania JA, Whalen K. Adverse central nervous system sequelae after selective transforaminal block: the role of corticosteroids. Spine J. 2004;4:468–474. 11. Zhu J, Falco FJ, Formoso F, Onyewu CO, Irwin FL.

428

Alternative approach for lumbar transforaminal epidural steroid injections. Pain Physician. 2011;14:331–341. 12. Kambin P, Savitz MH. Arthroscopic microdiscectomy: an alternative to open disc surgery. Mt Sinai J Med. 2000;67:283–287. 13. Park KD, Lee J, Jee H, Park Y. Kambin triangle versus the supraneural approach for the treatment of lumbar radicular pain. Am J Phys Med Rehabil. 2012;91:1039–1050. 14. Park JW, Nam HS, Park Y. Usefulness of posterolateral transforaminal approach in lumbar radicular pain. Ann Rehabil Med. 2011;35:395–404. 15. Park CH, Lee SH, Park HS. Lumbar retrodiscal versus postganglionic transforaminal epidural steroid injection for the treatment of lumbar intervertebral disc herniations. Pain Physician. 2011;14:353–360. 16. Heavner JE, Racz GB, Jenigiri B, Lehman T, Day MR. Sharp versus blunt needle: a comparative study of penetration of internal structures and bleeding in dogs. Pain Practice. 2003;3:226–231. 17. Benzon HT, Chew TL, McCarthy RJ, Benzon HA, Walega DR. Comparison of the particle sizes of different steroids and the effect of dilution: a review of the relative neurotoxicities of the steroids. Anesthesiology. 2007;106:331–338. 18. Okubadjero GO, Talcott MR, Schmidt RE, et al. Perils of intravascular methylprednisolone

injection into the vertebral artery. An animal study. Bone Joint Surg. 2008;90:1932–1938. 19. Shah RV. Paraplegia following thoracic and lumbar transforaminal epidural steroid injections: how relevant are particulate steroids. Pain Pract. 2013 [published online Aug 5, 2013]. 20. Jasper JF. Role of digital subtraction fluoroscopic imaging in detecting intravascular injections. Pain Physician. 2003;6:369–372. 21. Karasek M, Bogduk N. Temporary neurologic deficit after cervical transforaminal injection of local anesthetic. Pain Med. 2004;5:202–205. doi: 10.1111/j.1526-4637.2004.04028.x. 22. Shah RV. Paraplegia following thoracic and lumbar transforaminal epidural steroid injections: how relevant is physician negligence? J Neurointerv Surg. 2013 [published online Aug 28, 2013]. 23. Chang Chien GC, Candido KD, Knezevic NN. Digital subtraction angiography does not reliably prevent paraplegia associated with lumbar transforaminal epidural steroid injection. Pain Physician. 2012; 15:515–523. 24. Helm S, Glaser S, Falco F, Henry B. A medical-legal review regarding the standard of care for epidural injections, with particular reference to a closed case. Pain Physician. 2010;13(2): 145–150.

Section 7 Chapter

60

Special Topics

Postepidural steroid injection paraplegia Annemarie E. Gallagher and Devin Peck

Case study A 45-year-old previously healthy male construction worker sustained a low back injury on the job resulting in radicular pain. He subsequently underwent a fluoroscopic-guided epidural steroid injection to alleviate his symptoms. Within 1 week after the injection, he developed 10/10 excruciating pain in the lumbar region. Associated symptoms included new onset severe motor weakness and hypesthesia in the bilateral lower extremities.

1. What is the differential diagnosis? a. b. c. d. e. f. g.

Vessel transection Epidural hematoma Spinal cord infarction Intrathecal injection Epidural abscess Conversion disorder Spinal cord injury

An epidural hematoma is a potential complication of interventional spinal procedures, including epidural steroid injections and facet joint injections. Approximately 350 cases have been reported in the literature and have been secondary to such causes as traumatic needle placement, vertebral abnormalities, anticoagulant medication use, and coagulopathy. The incidence of epidural hematoma following an epidural injection has been suggested to be around 1:150 000.[1]

2. Review the anatomy and vasculature of the epidural space The epidural space is a potential space between the dura mater and the lining of the vertebral canal. Its borders

include the posterior longitudinal ligament anteriorly, the ligamentum flavum posteriorly, and the pedicles and intervertebral foraminae laterally. Superiorly, the border is the fusion of the spinal and periosteal layers of the dura mater at the foramen magnum and inferiorly, the sacrococcygeal membrane. The epidural space contains the dural sac – typically ending around the S2 level in adults – the spinal nerves, blood vessels, connective tissue, and fat.[2] Within the spinal canal, there is an arterial and venous network, both anteriorly and posteriorly, which is formed from the spinal arteries that enter at each vertebral level via the intervertebral foramina. The spinal arteries originate from the vertebral arteries, as well as the aorta. These arteries then anastomose with the artery of Adamkiewicz and run along the surface of the spinal cord, which as a result, has a rich blood supply. In addition, there is a watershed area where the thoracic and lumbar arteries meet the branches from the conus medullaris. Although there are a significant number of anastomoses throughout the spinal canal, arterial trauma can severely compromise blood supply to the spinal cord[2] (Figure 60.1).

3. If the patient developed lower extremity paralysis immediately following the epidural steroid injection, which diagnosis would be highest on the differential? a. b. c. d. e.

Epidural hematoma Spinal cord infarction Meningitis Secondary gain Epidural abscess

Case Studies in Pain Management, ed. Alan David Kaye and Rinoo V. Shah. Published by Cambridge University Press. © Cambridge University Press 2015.

429

Chapter 60: Postepidural steroid injection paraplegia

Dorsal somatic branch (pre-laminar branch) with dural branches Radiculomedullary artery

Anterior spinal Anterior artery central Nutrient vessels artery Anterior radiculomedullary arteries Posterior radiculomedullary arteries

Figure 60.1. Depiction of the artery of Adamkiewicz (radiculomedullary artery) and its relationship to the aorta and spinal nerve. Source: Kunihiro Y et al. MR angiography and CT angiography of the artery of Adamkiewicz: non-invasive preoperative assessment of thoracoabdominal aortic aneurysm. Radiographics. 2003;23:1215–1225, with permission.

Aorta Intercostal artery (segmental artery) Posterior branch Anterior branch Muscular branch

Posterior spinal arteries Posterior trunk (post-central branch) Ganglionic branch

Posterior plexiform network

Lower extremity paralysis and its rapid onset following an epidural steroid injection suggest the presence of profound ischemia, which may be secondary to thoracolumbar spinal cord infarction.[3] Infarction of the spinal cord can occur from intra-arterial injections resulting in vascular trauma, arterial spasm, air emboli, or emboli from particulate steroids. There has been and continues to be debate in the literature regarding the risks and benefits of particulate versus non-particulate steroids for transforaminal epidural steroid injections.[4] Two recent papers critically examine the role of particulate steroids and physician negligence as the root cause of paraplegia.[5,6]

4. If the patient underwent a transforaminal epidural steroid injection, describe the most favorable needle placement to minimize the risk of these serious postinjection symptoms? a. Far lateral to the infero-lateral margin of the pedicle b. Superoanterior triangle in the foramen

430

c. Inferior aspect of the foramen d. Avoid performing transforaminal epidural steroid injections The artery of Adamkiewicz, also known as the anterior segmental medullary artery or the arteria radicularis magna (ARM), provides blood supply to the lower twothirds of the spinal cord via the ASA. It typically arises from a left posterior intercostal artery (in 69–85% of cases). It may enter the spinal canal at any intervertebral foramen from T8–L1 (with the highest incidence at T10), although it has been documented that the artery may enter at a broader range of levels from T2 to L5.[3,7,8] To avoid the artery of Adamkiewicz during a transforaminal epidural steroid injection, entering the inferior portion of the foramen is of utmost importance.[3,4,6–8] A retrospective study performed by Murthy et al demonstrated that the artery of Adamkiewicz was located in the superior one-half of the foramen in 97% of patients. In 88% of patients, this artery was located in the upper one-third of the foramen, and only 2% of the time was it located in the lower one-third. The artery of Adamkiewicz was never seen in the inferior one-fifth of the

Chapter 60: Postepidural steroid injection paraplegia

neuroforamen.[9] The diameter of the artery averages 1 mm in the foramen.[10] There is, however, debate regarding the optimal location for needle placement in the foramen. With needle entry into the inferior portion of the foramen, there is an increased risk of contact with the nerve root. If nerve root is contacted, an awake patient will report paresthesia and the needle should be immediately withdrawn and redirected. While there is debate regarding the safety of utilizing sedation for spinal injections, it is important to maintain a level of consciousness consistent with the ability to report paresthesias.

5. Which of the following symptoms should be the most worrisome for the clinician? a. b. c. d. e. f.

Motor weakness Hypesthesia in the bilateral lower extremities Hypesthesia in one lower extremity Incontinence Headache Patient’s anger

An epidural hematoma can lead to spinal cord impingement. Back pain and neurologic deficits – including impaired gait, motor weakness, hypesthesia, a sensory level, and neurogenic bowel and bladder – should alert the physician to the possibility of spinal cord compression. The patient will typically appear to be in significant distress. Back pain tends to be severe, constant, and localized and may have a radicular component, mimicking a disc herniation. Pain can be made worse with increased intraspinal pressure, as caused by the valsalva maneuver, lifting, coughing, or sneezing. Spinal cord impingement may progress to paraparesis or paraplegia, depending on the extent of compression. This is a medical emergency and prompt diagnosis and treatment are imperative in an attempt to prevent permanent disability from spinal cord injury (Figure 60.2).

6. What safety measures could the physician have taken to try to prevent such complications secondary to the epidural steroid injection? It is important for the physician to have a clear fluoroscopic image to help guide needle placement

during the injection. The image should always be in the center of the screen, and AP, lateral, and oblique images should be utilized. Cases have been documented in which the image was not centered, and as a result vascular flow of contrast was not visualized, as the flow pattern was mainly off screen.[10] In order to determine if needle placement is intravascular, contrast should be injected under live fluoroscopy. An additional measure that may be taken to avoid catastrophic neurologic sequelae following epidural steroid injection includes administration of a test dose of local anesthetic. Any symptoms following such an injection would lead to cessation of the procedure, preventing permanent symptoms.[11,12] Fluoroscopic digital subtraction angiography may also enhance the detection of vascular flow during the procedure, particularly when injecting at the levels where the artery of Adamkiewicz is more likely to be found.[4,12,13] Careful attention to a patient’s coagulation status is also imperative for the prevention of complications. While a routine coagulation profile is not recommended, vigilance with regard to the patient’s use of anticoagulants and anti-platelet agents is vital. The American Society of Regional Anesthesia and Pain Medicine has published guidelines which can serve to guide clinical decision-making.[14] Other potential safety measures include the use of non-particulate steroids, the use of blunt needles, and the avoidance of patient sedation during the procedure.[4] However, many of these initiatives are ad hoc, hence, they may not improve safety. Shah[5,6] discusses issues about adoption of ad hoc safety initiatives without fully examining the root cause – this leads to erroneous conclusions. If these initiatives significantly raise complexity and costs with a procedure, alternative injection techniques should be considered.

7. What is the most reliable way to diagnose an epidural hematoma? a. b. c. d. e.

x-ray Magnetic resonance imaging CT scan Electromyography Spinal tap

Radiographic imaging is a useful tool in diagnosing the presence of a spinal epidural hematoma. MRI is a non-invasive modality that can localize and measure the extent of a hematoma. It can also demonstrate any

431

Chapter 60: Postepidural steroid injection paraplegia

Figure 60.2. (A) T1-weighted MRI of lumbar spine showing a spinal epidural hematoma. (B) T2-weighted MRI of lumbar spine showing a spinal epidural hematoma. Source: Klein JP, Hsu L. Neuroimaging during pregnancy. Semin Neurol. 2011; 31(4):361–373.

impact on the spinal cord. In particular, T2-weighted MRI images will show a compressed spinal cord as a hyperintense region, suggestive of intramedullary edema.[15]

8. How should the clinician interpret the results, depending on when the modality was utilized following symptom onset? Sagittal MRI views can clearly demonstrate a biconvex hematoma in the dorsal epidural space, with welldefined contours that taper at the superior and inferior margins. During the acute stage (i.e., within 30 hours of onset), on T1-weighted images, the hematoma will show up as a hypointense signal compared

432

to the spinal cord, and on T2-weighted images, it will be a hyperintense signal. During the subacute stage (> 30 hours after onset), the hematoma will be a heterogeneous area of hyperintensity on both T1 and T2 images. The heterogeneity seen is secondary to hemoglobin degradation during this time period. The epidural fat will appear to have a cupped shape, and the dura will be displaced on sagittal and axial views, indicating that the pathology is epidural in location.[16]

9. What is the treatment for an epidural hematoma? Acute management Patients presenting with signs of spinal cord compression must undergo a complete neurologic examination.

Chapter 60: Postepidural steroid injection paraplegia

Their neurologic status must be monitored for the possibility of progression. Any acute progression will make surgical intervention more likely. In order to evacuate a spinal epidural hematoma, a laminectomy may be performed. A study by Foo and Rossier reported that 95% of patients presenting with incomplete sensorimotor deficit and about 45% of those with complete deficit returned to baseline function following surgery.[17,18] Surgical outcome appeared to be more favorable if the epidural hematoma was in the lumbosacral region and if it was confined to one vertebral level.[17,18] In an attempt to prevent neurologic progression and permanent deficit, most patients must undergo prompt decompression within 36 hours, necessitating the use of reliable diagnostic modalities in the acute phase.[17]

Chronic management Patients who develop spinal cord injury need extensive rehabilitation, including physical and occupational therapy, in order to learn techniques that will aid with such activities of daily living as transfers, bathing, and clean intermittent self-catheterization. It is important to keep in mind that about 70% of those with spinal cord injury will develop musculoskeletal nociceptive pain due to shoulder overuse, as

References 1.

2.

3.

4.

5.

Nam KH, Choi CH, Yank MS, Kang DW. Spinal epidural hematoma after pain control procedure. J Korean Neurosurg Soc. 2010;48(3):281–284.

Malhotra G, Abbasi A, Rhee M. Complications of transforaminal cervical epidural steroid injections. Spine. 2009;34(7): 731–739. Shah RV. Paraplegia following thoracic and lumbar

10. What is the role of intravenous steroids in acute spinal cord injury? The role of intravenous steroids in acute spinal cord injury is controversial. The pathophysiology of spinal cord injury consists of a primary mechanical insult (such as needle puncture or injection), followed by secondary processes which take place at the cellular and molecular level.[20] While these processes are not completely understood, potential pharmacologic interventions have been investigated in animal models.[21] Currently, treatment with intravenous methylprednisolone is the most common modality to prevent these secondary mediators of cellular damage. The National Acute Spinal Cord Injury Study (NASCIS) was first published in 1984, and was most recently updated in 1997.[22] These authors recommend a high-dose methylprednisolone protocol within the first 8 hours of injury but this is not recommended for penetrating injuries.

transforaminal epidural steroid injections: how relevant are particulate steroids. Pain Pract. 2013 [published online Aug 5, 2013]. 6.

Richardson J, Groen GJ. Applied epidural anatomy. Cont Educ Anaesth Crit Care Pain. 2005; 5(3):98–100. Glaser S, Shah RV. Root cause analysis of paraplegia following transforaminal epidural steroid injections: the ‘unsafe’ triangle. Pain Physician. 2010;13:237–244.

this joint becomes the main weight-bearing joint, and about 60–70% of patients will also develop neuropathic pain. It is important to treat the sources of this pain to optimize the patient’s participation in therapeutic interventions.[19]

7.

8.

Shah RV. Paraplegia following thoracic and lumbar transforaminal epidural steroid injections: how relevant is physician negligence? J Neurointerv Surg. 2013 [published online Aug 28, 2013]. Boll DT, Bulow H, Blackham KA, et al. MDCT angiography of the spinal vasculature and the artery of Adamkiewicz. AJR Am J Roentgenol. 2006;187:1054–1060. Murthy NS, Maus TP, Behrns CL. Intraforaminal location of the greater anterior radiculomedllary artery (artery of Adamkiewicz): a retrospective review. Pain Med. 2010;11(12):1756–1764.

9.

Atluri S, Glaser SE, Shah RV, Sudarshan G. Needle position analysis in cases of paralysis from transforaminal epidurals: consider alternative approaches to traditional technique. Pain Physician. 2013;16(4): 321–334.

10. Glaser SE, Falco F. Paraplegia following a thoracolumbar transforaminal epidural steroid injection. Pain Physician. 2005;8:309–314. 11. Karasek M, Bogduk N. Temporary neurologic deficit after cervical transforaminal injection of local anesthetic. Pain Med. 2004;5:202–205. 12. Kennedy DJ, Dreyfuss P, Aprill CN, Bogduk N. Paraplegia following image-guided transforaminal lumbar spine epidural steroid injection: two

433

Chapter 60: Postepidural steroid injection paraplegia

case reports. Pain Med. 2009; 10(8):1389–1394. 13. Baker R, Dreyfuss P, Mercer S, et al. Cervical transforaminal injection of corticosteroids into a radicular artery: a possible mechanism for spinal cord injury. Pain. 2003;103:211–215. 14. Horlocker TT, et al. Regional Anesthesia in the Patient Receiving Antithrombotic or Thrombolytic Therapy: American Society of Regional Anesthesia and Pain Medicine EvidenceBased Guidelines (Third Edition). Reg Anesth Pain Med 2010;35: 64–101. 15. Taniguchi LE, Pahl FH, Lucio JED, et al. Complete motor recover after acute paraparesis

434

caused by spontaneous spinal epidural hematoma: case report. Emer Med. 2011;11: 1–4. 16. Chang F, Lirng J, Chen S, et al. Contrast enhancement patterns of acute spinal epidural hematomas: a report of two cases. Am J Neurorad. 2003;24:366–369. 17. Fukuia MB, Swarnkara AS, Williams RL. Acute spontaneous spinal epidural hematomas. Am J Neurorad. 1999;20:1365–1372. 18. Foo D, Rossier A. Preoperative neurological status in predicting surgical outcome of spinal epidural hematomas. Surg Neurol. 1981;15:389–401. 19. Kirshblum S, Gonzalez P, Nieves J, et al. Spinal cord injuries. Phys

Med Rehab Brd Rev. 2010;2: 595–598. 20. Bydon M, et al. The current role of steroids in acute spinal cord injury. World Nsgy. 2013;13: S1878–8750. 21. Kwon BK, et al. A systematic review of non-invasive pharmacologic neuroprotective treatments for acute spinal cord injury. J Neurotrauma 2011;28:1545–1588. 22. Bracken MB, et al. Administration of methylprednisolone for 24 or 48 hours or tirilazad mesylate for 48 hours in the treatment of acute spinal cord injury: results of the third national acute spinal cord injury randomized controlled trial. JAMA 1997;277:1597–1604.

Section 7 Chapter

61

Special Topics

Complications: patient with dural puncture following cervical interlaminar epidural steroid injection Niteesh Bharara and Frank J. E. Falco

Case study A 52-year-old right-handed male construction worker presents with a 4-month history of neck, right shoulder, and right arm pain. His pain increases when he looks up or rotates his head to the right. He has noticed that the pain is worse at night causing him trouble sleeping. His previous treatments include narcotics, anti-inflammatory medication, and physical therapy which did not help. On physical examination, he reports pain on palpation of the cervical paraspinal muscles. The neurologic exam for muscle strength, muscle stretch reflexes, and sensation is normal. Spurling’s test reproduces the patient’s symptoms into the arm along the biceps and into the wrist, thumb, and index finger. He has not noticed changes in gait or balance, bowel or bladder function, or strength. An interlaminar epidural steroid injection is performed in the office without complications. The following day the patient returns to the office complaining of a severe headache located in the frontal and occipital region aggravated by sitting upright and relieved by lying down.

1. What is the differential diagnosis? a. b. c. e. f.

Tension headache Migraine headache Subdural hematoma Spontaneous intracranial hypotension Pneumocephalus

The dural puncture headache was first described in 1899 by August Bier, MD after he was given spinal anesthesia by his assistant. This is essentially a headache that presents after a needle is placed in the dural space causing a tear or hole. The cause of the

headache is not known, however most investigators have agreed that the cause is from leaking of CSF from the dural hole. In a series reported by Vandane and Dripps, the incidence of postdural puncture headaches was twice as common in women compared to men. The mean age for postdural puncture headaches was 20–40 years whereas the lowest incidence occurred after the fifth decade. Dural puncture headaches and spontaneous intracranial hypotension present very similarly, except spontaneous intracranial hypotension occurs in the absence of dural puncture, surgery, or trauma. The most common headaches are tension-type headaches which are typically dull, persistent pain that extends over the entire head.

2. What are the signs and symptoms of a postdural puncture headache? The postdural puncture headache is characteristically located in the frontal and/or the occipital region and described as severe and spreading in nature. The pain is exacerbated by head movement or being in the upright position, and relieved by lying down. It might be associated with nausea, vomiting, auditory and/or visual symptoms, vertigo, and paresthesias of the scalp and upper limbs. Pain might radiate to the neck and be associated with stiffness. In the absence of a postural component to the headache symptoms, the diagnosis of a postdural puncture headache is questionable. The postdural puncture headache can occur immediately after the dural puncture; however, this is rare and its occurrence should prompt the physician to look for alternative causes. Ninety percent of headaches will occur within 3 days of the procedure

Case Studies in Pain Management, ed. Alan David Kaye and Rinoo V. Shah. Published by Cambridge University Press. © Cambridge University Press 2015.

435

Chapter 61: Complications of dural puncture

and 66% will occur within 2 days. The headaches will rarely first present after 3 days, but can present as late as 14 days after dural puncture.

3. Describe the anatomy and pathophysiology for postdural puncture headaches The spinal dura mater is a dense connective tissue layer that extends from the foramen magnum to the second segment of the sacrum. It contains the spinal cord and the spinal nerve roots pierce through it as they exit the neural foramen. The dura mater fibers are arranged parallel to the longitudinal plane and are several layers thick. The thickness of the posterior dura varies significantly between individuals and the thickness at a particular spinal level is unpredictable. Cerebrospinal fluid (CSF) is located deep to the dura. The CSF production occurs mainly in the choroid plexus and produces approximately 500 ml daily. The CSF volume in an adult at any one time is approximately 150 ml and the CSF pressure in the lumbar region when lying down is noted to be 5 to 15 cm H2O and over 40 cm H2O when standing. Leakage of CSF through a dural puncture appears to be the main cause of postdural puncture headaches. The rate of CSF loss through the dural perforation is usually greater than the rate of CSF production. There is no dispute that leakage of CSF plays an important part in producing the headache, however the exact mechanism is unclear. There have been two plausible explanations. The first is that the drop in CSF pressure causes traction on the intracranial structures when in the upright position leading to the characteristic headache. The second involves the compensatory vasodilation following the drop in CSF pressure. The Monro–Kellie doctrine states that the sum of the volumes of the brain, CSF, and intracranial blood remains constant. So a decrease in the CSF volume will result in compensation with cerebral vasodilation which causes the characteristic headache.

4. How is the diagnosis of a postdural puncture headache made in this patient? The clinical history for a dural puncture headache is a headache and other associated symptoms that are posturally related, i.e., significantly worse when being

436

upright and much better after lying down. However, if there is doubt regarding the authenticity of a postdural puncture headache, additional tests can be used to confirm the diagnosis. A diagnostic lumbar puncture may demonstrate a low opening CSF pressure. A non-contrast MRI may demonstrate diffuse dural enhancement with evidence of brain sagging. A CT myelography or retrograde radionuclide myelography can be used to locate the source of the CSF leak.

5. What are the non-interventional treatments for a postdural puncture headache? A postdural puncture headache is usually self-limiting and lasts only a few days before resolving spontaneously. However, there are conservative measures to limit the discomfort from this type of headache which can be incapacitating. Because the regular course is one of spontaneous resolution it has been recommended that conservative therapy be attempted for the first 24 hours. Once a postdural puncture headache is diagnosed, conservative management usually begins with bed rest and avoidance of the upright position. The patient might also benefit from lying in the lateral decubitus position, as this puts less tension on the dura and results in a decrease in CSF leakage. Oral hydration has been a popular therapy for patients with a postdural puncture headache; however, there have been no studies that have shown vigorous hydration has any therapeutic or symptomatic benefit in a normally hydrated patient. Those patients that are unable to take oral hydration and become dehydrated should be given intravenous fluids. Over-the-counter analgesics such as acetaminophen and non-steroidal anti-inflammatories can be attempted and may provide symptomatic control. Steroids, vasopressors, alcohol, and ergotamine have been used with little benefit. Caffeine has been used for postdural puncture headaches for many years and has been studied thoroughly. Sechzer and Abel performed a double-blinded trial, and found that patients who received caffeine sodium benzoate 500 mg intravenously had better relief of postdural puncture headache than those who received a placebo. The mechanism of action is unknown, but it is believed to be secondary to caffeine being a cerebral vasoconstrictor and a potent CNS stimulant. Caffeine is inexpensive and fairly risk free;

Chapter 61: Complications of dural puncture

therefore, it is a reasonable first option. The recommended treatment dose is 300 to 500 mg of oral or intravenous caffeine once or twice daily. To keep the dosage in perspective, a cup of coffee contains 50 to 100 mg and a can of soda contains 35 to 50 mg. There have been reports of intravenous administration of caffeine resulting in seizure, so this should be attempted with caution. A tight abdominal binder can also be used to alleviate the headache symptoms. The rise in abdominal pressure from an abdominal binder will be transmitted to the epidural space and increase the CSF pressure. Unfortunately these binders are very uncomfortable and are not tolerated well by many patients.

6. What are the interventional treatments for a postdural headache and how are they performed? Interventional treatments have been practiced for many decades; however, they should not be attempted until after conservative treatment has failed for the first 24 hours. Placement of blood in the epidural space, which is commonly known as an epidural blood patch, has been a popular treatment for a postdural puncture headache since the 1960s. There has been significant controversy with the optimal volume of injected blood. A volume of 10 ml of blood was first widely accepted; however, Crawford et al found better results with the use of 20 ml. Controversy also exists with the time of placement of the epidural blood patch. There have been discussions of performing this procedure very early or even prophylactically. The exact mechanism for the therapeutic benefits of an epidural blood patch is not known. It has been proposed that the injected blood can exert a mass effect in the epidural space compressing the dural sac and lessening the CSF leakage. This is believed to cause immediate relief of the symptoms and lasts approximately 3 hours. The mass effect will then start to disappear and the blood will form a thin layer adherent to the dural sac sealing the dural puncture. It has been found that a properly executed epidural blood patch is 96% to 98% successful in resolving a postdural puncture headache. The procedure should be performed at the 24-hour mark if conservative management has failed for the best possible outcome. The procedure requires two people to perform. The first must gain intravenous access and withdraw a

total of 20 ml of blood in a sterile fashion. The second person will then place a needle in the epidural space and inject the blood at a rate of 1 ml every 3 seconds. The blood will not provide an adequate clot if the blood is injected too quickly. Major complications are very rare following an epidural blood patch; however, minor complaints such as a backache occur 16% of the time and might last as long as 3 months. Other risks include infection, bleeding, and arachnoiditis. There have been two cases of facial nerve palsies following a blood patch that resolved spontaneously. Epidural administration of saline has also been proposed to relieve symptoms after a dural puncture. Epidural injection of 20 ml of saline can temporarily increase the CSF pressure and cause relief of symptoms for approximately 24 hours. An epidural saline infusion has also been proposed for more long-lasting benefits. Injection of dextran 40 or gelatin has also been shown to be effective in the management of a postdural puncture headache. Dextran and gelatin have been reported as an alternative to an epidural blood patch and should be considered in situations when the blood of the patient cannot be used for the patch. Examples of these situations include bacteremia or the patient who is a Jehovah Witness.

7. What are the factors to avoid a dural puncture? The incidence of postdural puncture headaches was 66% in 1998 and this high incidence was likely secondary to the large gauge cutting spinal needles. There has been a reduction in the incidence of dural puncture headaches to 11% with the introduction of newly designed spinal needles. There is a direct correlation between the size of the needle and the risk of a postdural puncture headache. A study has shown the incidence ranged from 18% with use of a 16-gauge needle to 5% with a 26-gauge needle. Tip design has also been found to be influential in the reduction in the incidence of dural puncture. Blunt pencil point spinal needles such as a Whitacre spinal needle are known to be less traumatic and hence superior to cutting point needles such as the Quinke spinal needle. Lambert et al reported the rate of postdural puncture headaches to be 1.2% compared to 2.7% for a 25-gauge Whitacre versus Quinke spinal needle. Orientation of the bevel spinal needle affects the incidence of postdural puncture headaches. The dura

437

Chapter 61: Complications of dural puncture

has been described to run in a longitudinal fashion. Introduction of the spinal needle with the bevel parallel to the long axis produced less dural trauma when compared to perpendicular placement. This is likely because the introduction of the needle in parallel to the dural fibers results in less tension and in turn less trauma. The angle of insertion should also be as acute as possible when introducing the needle in the epidural space. Ready et al revealed that an acute-angle insertion of a needle results in decreased leakage of CSF and is less likely to produce a postdural puncture headache. It was once believed that an oblique angle of penetration theoretically creates a flap valve to seal the perforation; however, this has not been shown in studies.

8. What are the complications from a dural puncture? Although rare, complications are well recognized following dural puncture. The most well published

References 1.

2.

3.

4.

438

Lybecker H, Djernes M, Schmidt JE. Post dural puncture headache onset, duration, severity and associated symptom: an analysis of 75 consecutive patients with PDPH. Acta Anaesthesiol Scand. 1995;39:605–612. Vandom LD, Dripps RD. Long term follow-up of patients who received 10,098 spinal anesthetics: syndrome of decreased intracranial pressure (headache and ocular and auditory difficulties). JAMA. 1956; 161:586–591. Moskowitz M. The trigeminovascular system. In Oleson J, Tflet-Hansen P, Welch K, eds. The Headaches. New York, NY: Raven. 1993: pp. 97–104. Stewart WF, Shechter A, Rasmussen BK. Migraine prevalence: a review of population

complication is transient cranial nerve palsies. Any of the cranial nerves can be involved; however, the 3rd, 4th, 6th, 7th, and 8th are the most common cranial nerves. The incidence of cranial nerve palsies is 1:10 000 to 3.7:100 000. The sixth cranial nerve is the most susceptible and it was believed that this was due to its length; however, cranial nerve four is actually longer. The sixth cranial nerve is the most vulnerable because of its length and because it is fixed to the wall of the cavernous sinus. Tension is applied to this nerve as the brain begins to sag from the decrease in CSF pressure. Cases of subdural hematoma or cerebral hematoma have also been reported after a dural puncture. These bleeds can occur in young, healthy individuals even with the use of a small gauge needle. The proposed mechanism is that a constant leak of CSF will result in the brain sagging. Traction is then placed on delicate blood vessels causing them to rupture and bleed.

based studies. Neurology. 1994; 44(Suppl):S17–23. 5.

6.

Spencer HC. Postdural puncture headache: what matters in technique. Reg Anesth Pain Med. 1998;23:374–379. Buettner J, Wresch K-P, Klose R. Postdural puncture headache: comparison of 25 gauge Whitacre and Quinke needle. Reg Anesth. 1993;18:166–169.

7.

Fink BR, Walker S. Orientation of fibres in human dorsal lumbar dura mater in relation to lumbar puncture. Anesth Analg. 1989;69:768–772.

8.

Jawalekar SR, Marx GF. Cutaneous cerebrospinal fluid leakage following attempted extradural block. Anesthesiology. 1981;54:348–349.

9.

Sechzer PH, Abel L. Post spinal anaesthesia headache treated with

caffeine: evaluation with demand method – Part I. Current Therap Resp. 1978;24:307–312. 10. Cesarini M, Torrielli R, Lahayl F et al. Sprotte needle for intrathecal anaesthesia for cesarean section: incidence of postdural puncture headache. Anesthesia. 1990;45: 656–658. 11. Sharma SK, Gambling DR, Joshin GP, et al. Comparison of 26 gauge atraucan and 25 gauge whitacre needles: insertion characteristic and complication. Can J Anesth. 1995;42:706–710. 12. Gormley JB. Treatment of post spinal headache. Anesthesiology. 1960;21:565–566. 13. Baysinger CL, Merk EJ, Harte E, et al. The successful treatment of dural puncture headache after failed epidural blood patch. Anesth Analg. 1986;65:1242–1244.

Section 7 Chapter

62

Special Topics

Complications: a patient with serotonin syndrome Natalia Covarrubias, Amirpasha Ehsan, and Danielle Perret Karimi

Case study

What is serotonin syndrome?

J.H. is a 58-year-old female who was followed at an outpatient pain clinic for the past 6 months. Her past medical history includes hypertension, lumbar degenerative disc disease, superimposed myofascial pain, and depression. Her pain had been treated with Flexeril 5 mg TID and tramadol 100 mg BID for the past 4 months. Additionally, she was taking Elavil 50 mg daily for depression. At her regularly scheduled followup appointment at the pain clinic, she reported feeling anxious, sweaty, with a recent history of uncontrolled hypertension. She noted these symptoms after her last psychiatry appointment 5 days ago when her doctor increased her Elavil to 100 mg daily. She denied use of any over-the-counter medications or illicit drugs. She was not taking any other prescription medications other than the previously mentioned prescriptions. Physical exam revealed tachycardia, hypertension, and normal body temperature. She appeared anxious, shaky, and diaphoretic. She was alert and oriented to person, place, and time. Strength was 5/5 throughout the bilateral upper and lower extremities. Reflexes were 2+ in the bilateral upper extremities and 3+ in the bilateral patella and Achilles tendons. Sensation was intact in bilateral upper and lower extremities. Based on her history and physical, the diagnosis of serotonin syndrome was determined. All serotonergic medications were discontinued and she was sent home with strict instructions to be monitored by her husband and to proceed to the nearest emergency room should she worsen. She returned 24 hours later reporting resolution of her symptoms.

Serotonin syndrome (SS) is a complication which can occur due to drug interactions resulting in an increase of serotonin in the central nervous system.[1] The syndrome is a concentration-dependent reaction which can develop in any individual, by medications that synergistically increase serotonin.[2,3]

Which drugs have been implicated in SS? Many drugs have been implicated in causing SS, including: antidepressants (tricyclics, SSRIs, and serotonin-norepinephrine reuptake inhibitors [SNRIs]), antibiotics (linezolide), opiates and pain medications (fentanyl, oxycodone, tramadol, methadone, cyclobenzaprine), antiemetics, amphetamines, antimigraine drugs, and over-the-counter medications (dextromethorphan, St. John’s wort).[1,3–6] In the case of patient J.H., she was taking tramadol, cyclobenzaprine, and Elavil. Tramadol weakly inhibits serotonin uptake and has been implicated in various case reports resulting in SS when used with an SSRI.[1,6–9] Cyclobenzaprine is structurally related to tricyclic antidepressants, thus is thought to have some similar actions, such as possibly increasing central serotonin transmission.[10] Cyclobenzaprine has been shown to precipitate SS in multiple case reports.[10–12] Serotonin syndrome typically occurs hours after initiating a serotonergic medication, increasing a dosage, or after combining serotonergic medications.[11] It presents as a triad of altered mental status, autonomic hyperactivity, and neuromuscular abnormalities.[7] Symptoms depend on the severity of the increased serotonin and have a variable

Case Studies in Pain Management, ed. Alan David Kaye and Rinoo V. Shah. Published by Cambridge University Press. © Cambridge University Press 2015.

439

Chapter 62: Complications: a patient with serotonin syndrome

presentation (see Table 62.1). The diagnosis can be based on the Hunter’s criteria, which has been shown to be more sensitive and specific than Sternbach’s criteria.[2] Hunter’s criteria requires use of a Table 62.1. Signs and symptoms of serotonin syndrome[3,5,7]

Mild

Tachycardia, diaphoresis, akathisia, twitching, tremor, restlessness, hyperreflexia

Moderate

Tachycardia, hypertension, hyperthermia (> 40°C), hyperactive bowel signs, clonus greater in lower extremities, horizontal ocular clonus, mild agitation/ hypervigilance, slightly pressured speech, irritability

Severe

Severe hypertension, tachycardia, tachypnea, muscular rigidity, generalized tonic–clonic seizures, clonus, hyperthermia (> 41°C), hypertonicity, confusion, agitated delirium, rhabdomyolysis, multiple organ failure

serotonergic agent and one of the following presentations: spontaneous clonus; inducible clonus with agitation or diaphoresis; ocular clonus with agitation or diaphoresis; tremor and hyperreflexia; hypertonicity, and temperature > 38°C with ocular clonus or inducible clonus.[2] Sternbach’s criteria requires 3 of 10 clinical features to be present: agitation, diaphoresis, diarrhea, fever, hyperreflexia, incoordination, mental status changes, myoclonus, shivering, or tremor.[13] Other differential diagnoses should be ruled out and a neuroleptic should not have been started or increased prior to symptom onset to ensure an appropriate diagnosis.[13]

How is SS diagnosed? Diagnosis is based on a good history and physical examination as there are no specific laboratory tests.[3,5] Laboratory tests can be utilized to rule out other causes of the patients’ clinical presentation. A history of medication use should include illegal drugs, overthe-counter medications and supplements, as well as

Table 62.2. Presentations of SS, NMS, malignant hyperthermia, and anticholinergic poisoning

Serotonin syndrome  Use of serotonergic medication  Occurs less than 12 hours after ingestion  Hypertension, tachycardia, tachypnea, hyperthermia  Mydriasis  Sialorrhea  Diaphoresis  Hyperactive bowel sounds  Hypertonicity in lower extremities more than upper extremities  Hyperreflexia and clonus  Agitation and coma

Malignant hyperthermia  Administration of an inhalational anesthetic  Occurs within 30 minutes to 24 hours after administration  Hypertension, tachycardia, tachypnea  Hyperthermia up to 46°C  Normal pupils and mucosa  Mottled skin with diaphoresis  Hypoactive bowel sounds  Rigor mortis-like rigidity  Hyporeflexia  Agitation

Neuroleptic malignant syndrome  Use of a dopamine antagonist  Occurs in 1–3 days after ingestion  Hypertension, tachycardia, tachypnea, hyperthermia  Normal pupils  Sialorrhea  Dysphagia and dysarthria  Diaphoresis, pallor  Normal bowel sounds  Normal reflexes and muscle tone  “Lead-pipe” rigidity  Bradyreflexia and bradykinesia  Agitation

Anticholinergic poisoning  Use of an anticholinergic  Occurs within 12 hours of use  Hypertension, tachycardia, tachypnea, hyperthermia  Mydriasis  Dry oral mucosa  Urinary retention  Hot, red, dry, patchy, and flushed skin  Hypoactive or absent bowel sounds  Normal muscular tone and reflexes  Agitation with delirium

Adapted from references.[3,5,14]

440

Chapter 62: Complications: a patient with serotonin syndrome

prescribed medications. As mentioned earlier, all of these have been implicated in causing SS.

What is the differential diagnosis? Differential diagnoses should include, but are not limited to, neuroleptic malignant syndrome (NMS), malignant hyperthermia, anticholinergic poisoning, meningoencephalitis, sepsis, delirium tremens, opioid withdrawal, and heat stroke. A thorough history and physical examination is the key to proper diagnosis (see Table 62.2). Treatment involves removing the precipitating agent and providing supportive care.[14] Removal of the serotonergic medication usually allows resolution of symptoms in 24 hours.[13] In mild cases, administration of a benzodiazepine can assist with symptoms. In moderate cases, serotonin antagonists are often administered in addition to correction of hyperthermia with external cooling. In the setting of severe hyperthermia, sedation, with neuromuscular paralysis and intubation, should occur to eliminate excessive muscle activity thus preventing further complications.[3] Serotonin antagonists such as

References 1.

2.

3.

4.

5.

Cooper BE, Sejnowski CA. Serotonin syndrome: recognition and treatment. AACN Advanced Critical Care. 2013;24(1):15–20; quiz 1–2. Dunkley EJ, Isbister GK, Sibbritt D, Dawson AH, Whyte IM. The Hunter Serotonin Toxicity Criteria: simple and accurate diagnostic decision rules for serotonin toxicity. QJM: Monthly Journal of the Association of Physicians. 2003;96(9):635–642. Boyer EW, Shannon M. The serotonin syndrome. N Engl J Med. 2005;352(11):1112–1120. Ables AZ, Nagubilli R. Prevention, recognition, and management of serotonin syndrome. Am Fam Phys. 2010; 81(9):1139–1142. Iqbal MM, Basil MJ, Kaplan J, Iqbal MT. Overview of serotonin syndrome. Ann Clin Psychiatry. 2012;24(4):310–318.

cyproheptadine and chlorpromazine can be utilized for moderate to severe SS.[14] Cyproheptadine is only available in tablet form, but can be crushed and administered via a nasogastric tube. Doses reported range from 4 mg to 30 mg, with below 16 mg seeming to be ineffective.[14] Iqbal et al recommend cyproheptadine 4 mg to 8 mg every 6 hours.[5] Chlorpromazine is administered intramuscularly at a recommended starting dose of 50 mg, though larger doses have been reported.[14] Following appropriate treatment, some physicians may be weary of restarting serotonergic medications. Symptoms of SS are directly related to the level of intrasynaptic serotonin, thus medications which alter this level can prevent a reoccurrence of SS.[15] Physicians should take caution when prescribing multiple agents with serotoninergic properties, and warn patients of possible side effects such as akathisia, tremor, and changes in mental status. In the cases that such medications are used together, serotonergic medications should be started at different times, each maintained at the lowest effective dose, and the patient should be well monitored.

6.

Walter C, Ball D, Duffy M, Mellor JD. An unusual case of serotonin syndrome with oxycodone and citalopram. Case Rep Oncol Med. 2012;2012:261787.

7.

Nelson EM, Philbrick AM. Avoiding serotonin syndrome: the nature of the interaction between tramadol and selective serotonin reuptake inhibitors. The Ann Pharmacother. 2012;46(12):1712–1716.

8.

9.

Kitson R, Carr B. Tramadol and severe serotonin syndrome. Anaesthesia. 2005;60(9):934–935. Shahani L. Tramadol precipitating serotonin syndrome in a patient on antidepressants. J Neuropsychiatry Clin Neurosci. 2012;24(4):E52.

10. Keegan MT, Brown DR, Rabinstein AA. Serotonin syndrome from the interaction of cyclobenzaprine with other serotoninergic drugs. Anesth Analg. 2006;103(6):1466–1468.

11. Mestres J, Seifert SA, Oprea TI. Linking pharmacology to clinical reports: cyclobenzaprine and its possible association with serotonin syndrome. Clinical Pharmacol Therapeut. 2011; 90(5):662–665. 12. Day LT, Jeanmonod RK. Serotonin syndrome in a patient taking Lexapro and Flexeril: a case report. Am J Emerg Med. 2008;26(9):1069 e1–3. 13. Sternbach H. The serotonin syndrome. Am J Psychiatry. 1991;148(6):705–713. 14. Gillman PK. The serotonin syndrome and its treatment. J Psychopharmacol. 1999; 13(1):100–109. 15. Rastogi R, Swarm RA, Patel TA. Case scenario: opioid association with serotonin syndrome: implications to the practitioners. Anesthesiology. 2011;115(6): 1291–1298.

441

Section 7 Chapter

63

Special Topics

Office-based buprenorphine to wean patients off opioids Natalia Murinova, Daniel Krashin, Cliff Gevirtz, and Alan David Kaye

Case study A 37-year-old male presents with a 7-year history of chronic daily headaches, severe daily whole-body pain that is consistent with fibromyalgia, severe insomnia, depression, and anxiety. His neurologic exam is normal, and he has a normal MRI scan of his brain. He states that he can’t tolerate his headaches anymore, and feels “suicidal” due to the severe pain. After talking to him, you learn that the primary treatment for his severe disabling pain has been daily long-acting morphine, along with large doses of oxycodone and acetaminophen. You discuss with the patient that most of his chronic pain, including his chronic daily headaches and fibromyalgia, have been exacerbated by medication overuse, since using opioids more than 8 days a month can cause medication overuse headaches and central sensitization.[1] He agrees that he would like to detoxify off opioids, however he has been unable to decrease them in the past, because of withdrawal symptoms, and he has required higher amounts of opioids for treatment. He has history of heroin abuse. He feels he is dependent on opioids, and can’t live without them. Your recommendation is to proceed with comprehensive treatment that includes opioid agonist treatment with buprenorphine, since medicationassisted therapy is the standard of care in opioiddependent adults.

1. How common is opioid dependence? Opioid dependence affects millions of people worldwide, including more than 2 million people in the USA. The number of people estimated to have “nonmedical pain reliever dependence,” i.e., dependence on prescription opioids, has been rising each year and was estimated at 1.4 million in 2011. In 2011 the

NSDUH (National Survey in Drug Use and Health) estimated that there are 369 000 people over the age of 12 dependent on heroin.[2] Admissions to substance abuse treatment centers for prescription opioid dependence have also been rising according to the Treatment Episode Data Set (TEDS), as have emergency room visits due to overdoses according to the Drug Abuse Warning Network (DAWN).[2]

2. What is the best approach to address opioid dependence in the USA? Comprehensive treatment that includes psychosocial treatment and medication-assisted therapy is the standard of care in opioid-dependent adults in the USA and other countries. From medication-assisted therapy the most evidence supports treatment with the full mu-opioid agonist methadone, and the partial mu-opioid agonist buprenorphine.[3,4] These treatment modalities appear to be safe, effective, and cost-efficient. The availability of buprenorphine as an outpatient treatment for opioid dependence has greatly expanded the availability of opioid agonist treatments. The increasing prevalence of opioid dependence in the USA, involving both prescription opioids and heroin, has fueled the demand for this treatment. The Drug Enforcement Agency has estimated that there are roughly 800 000 buprenorphine treatment spaces for opioid dependence, but this falls well short of requirements.[5]

3. What are the risk factors for opioid addiction? 1. Mental health disorders[6] 2. Male sex[6] 3. Younger adults[6]

Case Studies in Pain Management, ed. Alan David Kaye and Rinoo V. Shah. Published by Cambridge University Press. © Cambridge University Press 2015.

442

Chapter 63: Office-based buprenorphine to wean patients off opioids

4. Individuals with greater days supply of prescription opioids dispensed[6] 5. Substance abuse[6] 6. Family history of substance abuse

4. How do you diagnose opioid dependence? In DSM-5 the diagnoses of opioid dependence and opioid abuse have been consolidated into opioid use disorder.[7] There is no distinction between “dependence” and “abuse” except for severity. There are 11 criteria for substance use disorders in DSM-5, including taking the substance longer or in greater amounts than intended, being unable to cut down or stop, spending time obtaining, using or recovering from the substance, social and work impairment due to the use, continuing to use despite this impairment and giving up on important activities, using the substance although it puts one at increased risk, and developing tolerance or withdrawal symptoms.[7] Only two positive criteria are needed for diagnosing mild opioid dependence.[7] The only new criterion, which was not present in DSM-IV, is craving for opioids.[7] Almost all patients who use chronic opioids will therefore meet at least the last two criteria for opioid use disorder: tolerance and withdrawal.

5. What are some of the features that make buprenorphine a desirable option for the treatment of opioid addiction? 1. 2. 3. 4. 5.

Partial mu-receptor agonist characteristic High affinity to opioid receptor Kappa-receptor antagonist Long half-life and slow elimination Favorable safety profile

Buprenorphine is derived from thebaine, a morphine alkaloid.[8] Buprenorphine has several actions physiologically. It has a high affinity to the mu-receptors and this leads to slow dissociation from the receptor.[9] Because of this high affinity to the mu-receptor it has fewer withdrawal symptoms upon discontinuation.[9] Buprenorphine is a partial mu-receptor agonist and kappareceptor antagonist.[9] Some authorities have proposed that at higher doses, the kappa-opioid antagonism overwhelms the mu-opioid effects and that this accounts for the partial agonist status of buprenorphine.[10]

Buprenorphine prevents onset of withdrawal syndrome in opioid-dependent patients. Increasing the dose of buprenorphine beyond a certain point, usually 32 mg a day or less, may have a “ceiling effect,” meaning that further dose increases do not produce additional opioid effects. This decreases its abuse potential and risk of overdose. Buprenorphine has a high affinity for the mureceptor, and this affinity is able to overcome the opioid antagonism of naltrexone. This has been used clinically to “rescue” patients from acute opioid withdrawal induced by naltrexone.[11] Conversely, the high affinity and partial agonism at the mu-opioid receptor means that patients currently using a full agonist opioid may be thrown into withdrawal by the administration of buprenorphine. Buprenorphine’s effects include positive mood, sedation, and respiratory depression seen as typical opioid effects. Buprenorphine in doses up to 32 mg sublingual dose were well tolerated, with a typical daily dose of 16 mg. There is evidence for buprenorphine’s ceiling effect on respiratory depression, associated with a wide margin of safety. In studies of cancer-related pain, transdermal buprenorphine is highly effective, with no evidence of a ceiling effect for pain relief.[12] Opioid-dependent patients are at especially high risk for relapse early in treatment. Buprenorphine’s high mu-receptor affinity may decrease the anticipated positive effects from other opioid use, thus reducing relapse rates.

6. What are the pharmacokinetics and pharmacodynamics of buprenorphine and the best form of administration? Buprenorphine has been used parenterally as an effective analgesic for many years. Ongoing injection use is avoided in treatment of substance use disorder, since many of these patients are used to injecting illicit substances such as heroin. Sublingual buprenorphine easily dissolves when placed under the tongue within 3 to 10 minutes in most patients. Clinical effects can last up to 96 hours following one dose, and because of this once a day dosing is typical.[13] Due to the slow dissociation from receptors the halflife of buprenorphine is 37 hours.[13] The onset of action is about 30 to 60 minutes with peak effect at 100 minutes.[13] If the patient accidentally swallows the buprenorphine, it will have little systematic effect

443

Chapter 63: Office-based buprenorphine to wean patients off opioids

due to extensive first-pass metabolism. If they try to inject the combined buprenorphine-naloxone compound intravenously, the naloxone will precipitate immediate, severe withdrawal. Buprenorphine is highly protein-bound and has a high volume of distribution.[13] Buprenorphine has a large first-pass effect with oral use, leaving only 10% in the systemic circulation. It is therefore used sublingually or parenterally, and recently a long-acting transdermal patch has become available for pain treatment. The liver metabolizes buprenorphine primarily by CYP P450 3A4 into the active metabolite norbuprenorphine via N-dealkylation, but few adverse effects have been found with enzyme inhibition or induction.[14] Both norbuprenorphine and its parent drug are glucuronidated and eliminated in bile as well.

subdermal implant intended to treat opioid dependence, which is pending FDA approval.

7. Is dose adjustment required for buprenorphine in the presence of hepatic and renal disease?

If a qualified physician intends to prescribe buprenorphine for treatment of opioid dependence and addiction, first he/she has to complete an 8-hour educational program with a post-test which can be done online or in person. Then proof of this training has to be submitted to the DEA to be placed in the registry of approved providers. The SAMHSA/CSAT (Substance Abuse and Mental Health Services Administration/Center for Substance Abuse Treatment) approves this training. The DEA then issues a unique identification DEA number and the qualified physician has two distinct DEA numbers. The initial number of patients the qualified physician can treat is 30, but with secondary notification to SAMHSA/CSAT these numbers can be increased to 100 after 1 year.

Adjustment of buprenorphine dose is not required in renal disease.[13] Even patients with chronic renal failure on dialysis have no difference from healthy controls in their pharmacokinetics of this drug, dose-adjusted blood levels were similar, and there is no increased incidence of sedation or toxicity.[15,16] A recent study suggested that the dose does not need to be adjusted in mild to moderate liver disease, either.[17]

8. What forms of buprenorphine are available for treatment of opioid dependence? Sublingual buprenorphine for the treatment of opioid dependence comes in two forms: buprenorphine hydrochloride and buprenorphine hydrochloride with naloxone hydrochloride in a ratio of 4:1. Sublingual buprenorphine is available in 2 mg or 8 mg films marketed as Subutex® (Reckitt Benckiser Group), and buprenorphine with naloxone is marketed as Suboxone® (Reckitt Benckiser Group), and is available in films with doses of 2/0.5 mg, 4/1 mg, 8/2 mg, 12/3 mg, which was developed to deter the abuse of buprenorphine. There are also generic forms of both available as tablets. Other forms of buprenorphine include a transdermal patch, used for pain management, and a

444

9. Why is buprenorphine not used via oral route? While buprenorphine can be given as a tincture in alcohol, it undergoes extensive first-pass metabolism, limiting its oral bioavailability to about 10%. The preferred route is sublingual, which has high bioavailability. Naloxone has poor absorption sublingually, but deters abuse of the drug by injection because the effect of buprenorphine would be countered by naloxone.

10. How can I get certified to prescribe buprenorphine for this indication?

11. How should I start this patient on buprenorphine? The best success of treatment of buprenorphine is if the patient with opioid dependence is experiencing early withdrawal before administration of buprenorphine. Buprenorphine displaces full opioid agonists at the mu-receptor, because of its high affinity for the mu-receptor. However buprenorphine is only a partial agonist. If a patient is taking a full opioid agonist and is not in withdrawal, adding buprenorphine will precipitate a withdrawal. Treatment should be started with supervised administration in the office. Ideally patients should be seen at weekly intervals initially, with weekly prescriptions of the medication, for at least the first month of treatment.

Chapter 63: Office-based buprenorphine to wean patients off opioids

12. What is the standard induction protocol? As discussed in section 11, the first dose should be administered in the clinic while the patient is showing mild to moderate withdrawal symptoms. Since this is not entirely predictable, many providers leave a window of 2 or more hours for the first visit to ensure that the patient is genuinely in withdrawal before starting the medication. The patient need not be present in clinic or directly observed the entire time. The first dose should be 2 to 4 mg, which is administered and which the patient takes there in the office. The smaller initial dose is less likely to cause adverse reactions or precipate withdrawal, but may be inadequate to reverse the symptoms of withdrawal. The patient should be reassessed after 60–120 minutes and may need a second dose if they continue to experience withdrawal symptoms. The first day total dose should not exceed 8 mg, or 12 mg in patients with high opioid requirements. The patient should be reassessed the next day. If they continued to have cravings or clear signs and symptoms of withdrawal after receiving 8–12 mg of buprenorphine on the first day, the dose may be increased by an additional 4 mg, administered in the office and then reassessed as before. The mechanics of using the sublingual medication should also be reviewed, including keeping the film or tablet under the tongue, not swallowing it, and not talking or eating until it has been absorbed. The dose may be increased up to a total of 16 mg on the second day. Most patients will achieve adequate treatment on doses up to 16 mg. If the total dose of 16 mg was given the previous day and the patient continued to have clear withdrawal or opioid cravings, the dose may be increased further, starting at 18 to 20 mg. Higher doses than 16 mg may be associated with higher risk of diversion. The maximum dose is 32 mg, but few patients appear to require this in practice.

13. What is the safety of using buprenorphine in the treatment of opioid dependence? For the majority of opioid-dependent patients, induction onto buprenorphine is well tolerated and uncomplicated.[18] No significant morbidity has been seen in heroin addicts treated with 8 mg of sublingual

buprenorphine.[19] Overdoses have been reported in cases of abusing high doses of buprenorphine together with benzodiazepines.

14. Can you use buprenorphine for pain control? Several commercial preparations of buprenorphine are available for the treatment of pain, including parenteral injectable buprenorphine and a transdermal patch. These forms are not indicated for treatment of opioid dependence. Some pain providers have used the sublingual buprenorphine as a treatment for chronic pain in high-risk individuals. This is legal in the USA but off-label and there is limited evidence to guide this practice. Recent publications on this topic include the review by Daitch et al and a pilot study by Rosenblum et al, and the interested reader is encouraged to review these.[20,21]

15. What are the clinical outcomes using buprenorphine? Buprenorphine opioid treatment combined with psychosocial addiction treatment seems to be better than non-pharmacologic treatment alone. In one study of long-term buprenorphine maintenance, 59% of patients on buprenorphine therapy remained in treatment after 6 months.[22] The investigators concluded that psychosocial addiction treatment and abstinence during the first week after induction were key to patient retention.[22] In another study assessing treatment engagement at 18–42 months, 77% remained on buprenorphine/naloxone.[23] Those on buprenorphine/naloxone were more likely to report abstinence, be in 12-step addiction treatment, remain employed, and enjoy improved functional status.[23]

Summary Opioid dependence is best treated with a comprehensive multidisciplinary approach that includes medication-assisted treatment with opioid agonists and psychosocial treatment. Evidence documents reduction of illicit opioid use, improved function, and prolonged retention in treatment. The availability of buprenorphine through office-based treatment in the community, independent of specialized addiction treatment centers, has improved patients’ access to medication-assisted treatment for opioid dependence.

445

Chapter 63: Office-based buprenorphine to wean patients off opioids

Buprenorphine’s unique characteristics, including partial mu-receptor agonism, high affinity to muopioid receptor, kappa-receptor antagonism, and favorable safety profile, make it favorable for treatment of opioid dependence. Many studies have shown

References 1.

2.

3.

4.

5.

6.

7.

8.

446

Tepper SJ, Tepper DE. Breaking the cycle of medication overuse headache. Cleveland Clin J Med. 2010;77(4):236–242. Mental Health Services Administration. Mental Health Services Administration (SAMHSA),(2012) Results from the 2011 National Survey on Drug Use and Health: Summary of National Findings. US Department of Health and Human Services. 2012. Mattick RP, Kimber J, Breen C, Davoli M. Buprenorphine maintenance versus placebo or methadone maintenance for opioid dependence. Cochrane Database Syst Rev. 2008;2. Mattick RP, Breen C, Kimber J, Davoli M. Methadone maintenance therapy versus no opioid replacement therapy for opioid dependence. Cochrane Database Syst Rev. 2009;3. Drug Enforcement Administration. Automated Records and Consolidated Orders System (ARCOS), 2010 Report on Buprenorphine Distribution. 2010. Edlund MJ, Steffick D, Hudson T, Harris KM, Sullivan M. Risk factors for clinically recognized opioid abuse and dependence among veterans using opioids for chronic non-cancer pain. Pain. 2007;129(3):355–362. American Psychiatric Association. Diagnostic and Statistical Manual of Mental Disorders: DSM-5. Arlington, VA: American Psychiatric Association. 2013. Downing JW, Leary WP, White ES. Buprenorphine: a new potent long-acting synthetic analgesic:

that this treatment, done properly, is an effective, safe, and cost-effective modality for opioid dependence. In combination with psychosocial treatments, buprenorphine therapy is an essential adjunctive therapy for opioid dependence in the outpatient setting.

comparison with morphine. Br J Anaesth. 1977;49(3):251–255. 9.

Leander JD. Buprenorphine has potent kappa opioid receptor antagonist activity. Neuropharmacology. 1987;26(9): 1445–1447.

10. Cowan A, Lewis JW, Macfarlane IR. Agonist and antagonist properties of buprenorphine, a new antinociceptive agent. Br J Pharmacol. 1977;60(4):537–545. 11. Urban V, Sullivan R. Buprenorphine rescue from naltrexone-induced opioid withdrawal during relatively rapid detoxification from high-dose methadone: a novel approach. Psychiatry (Edgmont). 2008; 5(4):56. 12. Muriel C, et al. Effectiveness and tolerability of the buprenorphine transdermal system in patients with moderate to severe chronic pain: a multi-center, open label, uncontrolled, prospective observational study. Clin Ther. 2005;27:451–462. 13. Elkader A, Sproule B. Buprenorphine. Clin Pharmacokinet. 2005;44(7): 661–680. 14. McCance-Katz EF, Sullivan LE, Nallani S. Drug interactions of clinical importance among the opioids, methadone and buprenorphine, and other frequently prescribed medications: a review. Am J Addictions. 2010;19(1):4–16. 15. Hand CW, Sear JW, Uppington J, et al. Buprenorphine disposition in patients with renal impairment: single and continuous dosing, with special reference to metabolites. Br J Anaesth. 1990; 64(3):276–282.

16. Summerfield RJ, Allen MC, Moore RA, Sear JW, McQuay HJ. Buprenorphine in end stage renal failure. Anaesthesia. 1985;40(9): 914. 17. Johnson RE, Fudala PJ, Payne R. Buprenorphine: considerations for pain management. J Pain Symptom Manage. 2005;29(3): 297–326. 18. Fiellin DA, Kleber H, TrumbleHejduk JG, McLellan AT, Kosten TR. Consensus statement on office-based treatment of opioid dependence using buprenorphine. J Subs Abuse Treat. 2004;27(2): 153–159. 19. Lange WR, Fudala PJ, Dax EM, Johnson RE. Safety and sideeffects of buprenorphine in the clinical management of heroin addiction. Drug Alcohol Depend. 1990;26(1):19–28. 20. Daitch J, Frey ME, Silver D, et al. Conversion of chronic pain patients from full-opioid agonists to sublingual buprenorphine. Pain Physician. 2012;15(3 Suppl). 21. Rosenblum A, Cruciani RA, Strain EC, et al. Sublingual buprenorphine/naloxone for chronic pain in at-risk patients: development and pilot test of a clinical protocol. J Opioid Manag. 2012;8(6):369–382. 22. Stein MD, Cioe P, Friedmann PD. Buprenorphine retention in primary care. J Gen Intern Med. 2005;20(11):1038–1041. 23. Parran TV, Adelman CA, Merkin B, et al. Long-term outcomes of office-based buprenorphine/ naloxone maintenance therapy. Drug Alcohol Depend. 2010; 106(1):56–60.

Section 7 Chapter

64

Special Topics

Patient on chronic opioids who wants to have anesthesia-assisted detoxification Cliff Gevirtz and Alan David Kaye

Case study A 47-year-old man presents with a request for opiate detoxification. The patient previously had seven back surgeries, failed trials for an intra-thecal pump, and a spinal cord stimulator. He is taking oxycodone CR 300 mg per day and fentanyl sublingual spray for breakthrough pain but these were not really reducing his pain significantly. He also had problems with his insurance covering the opiates. He had tried self tapering as well as transfer to buprenorphine twice, but both attempts left him in too much discomfort to continue. He denied using any drugs of abuse and has a private consultancy which enables him to seek out the best treatments in the world.

1. What are the various terms related to addiction and withdrawal? The terminology of detoxification, dependence, withdrawal, and addiction are often used imprecisely and incorrectly by members of the medical and legal professions: a. Detoxification: clearance of the intoxicating drug from the body. b. Dependence: a physical or psychologic state in which a person displays withdrawal symptoms if drug use is halted suddenly. c. Withdrawal syndrome: refers to symptoms including restlessness, rhinorrhea, lacrimation, diaphoresis, myosis, piloerection, and cardiovascular changes associated with an increase in catecholamine levels.

2. What are the indications for anesthesia-assisted opiate detoxification (AAOD)? It should be emphasized that only a highly selected subgroup of patients may benefit from this procedure. It is specifically NOT for all patients. Indeed, many patients who are seeking a “quick fix” to their problem are NOT appropriate candidates. The appropriate candidates include those who have been unable to abstain from using heroin even with methadone substitution despite adequate motivation, those who are unable to completely stop methadone and who continue along with 10 to 20 milligrams per day, as well as patients who are socially and occupationally active and cannot go through the usual long detoxification procedures without jeopardizing their jobs. There is also a subset of patients who have had numerous previous approaches to detoxifications only to withdraw due to discomfort. These patients often relate more than a dozen previous attempts. Sports stars and celebrities who need to achieve sobriety by a particular date, in order to continue their careers, may also be appropriate candidates if they are committed to long-term sobriety and not just a quick solution to their problems. Pain patients who have become physically dependent on opiates but are not receiving a significant amount of pain relief from them are also suitable candidates. Patients’ preferences are an important variable to consider as some patients may have tried various approaches to detoxification multiple times. Its use should be restricted to patients with only opioid dependence as simultaneous dependence on other substances can complicate the procedure.

Case Studies in Pain Management, ed. Alan David Kaye and Rinoo V. Shah. Published by Cambridge University Press. © Cambridge University Press 2015.

447

Chapter 64: Patient on chronic opioids who wants to have anesthesia-assisted detoxification

3. What are the contraindications to anesthesia-assisted opiate detoxification? Contraindications include:  Pregnancy  History of cardiac or pulmonary decompensation or evidence of the same on clinical examination  History of prolonged QTc interval  End-stage renal disease  End-stage liver disease  Concurrent co-dependence on benzodiazepines, alcohol, or cocaine  Current suicidal or homicidal ideation It is important to note that several medications used during the detoxification process may prolong the QTc interval to a dangerous extent which can result in a Toursades des points arrthymia. It is postulated that this prolongation may be the cause of sudden cardiac death seen 12–48 hours after detoxification.

4. What pre-assessment is needed for anesthesia-assisted detoxification? There are a myriad of recipes for conducting AAOD, which differ in the exact procedure employed. However, the following components are essential to any such regime: 1. An initial assessment, which involves obtaining a detailed history regarding the drug-intake and general medical and psychiatric illness. A detailed history of past attempts at detoxification and the reason for failure should be documented. An assessment of the need for as opposed to methadone maintenance, buprenorphine maintainance, and tapering. 2. Formulating a treatment plan which addresses and effectively treats each element of the opiate withdrawal syndrome and the post-procedure follow-up, i.e., ongoing psychiatric care must be in place prior to starting detoxification. 3. Obtaining a detailed written informed consent. The patient should be clearly and adequately informed of the available treatment options including methadone taper and buprenorphine taper (see Chapter 63), the comparative costs

448

incurred for each, and the relative risks/ advantages inherent in them.

Pre-anesthetic testing The key issue is to identify the damage that the substance abuse may have already caused. Apart from a detailed physical examination, recommended investigations are: a complete blood count; an electrocardiogram; serum chemistry tests including liver function test, tests for HIV, hepatitis B virus (HBV), and hepatitis C virus (HCV); and chest x-ray.

4. What is the actual methodology to perform AAOD? Premedication High-dose α-2 agonist blockade is introduced incrementally to reduce the sympathetically mediated effects of withdrawal. Clonidine may be administered orally, transdermally, or intravenously. The total dose used prior to induction is usually above 0.5 mg and is determined by the patient’s hemodynamic ability to tolerate the dosage. It is important to understand that trying to save a few dollars in costs by minimizing dosages leads directly to uncontrolled symptomatology including hypertensive crisis and elevations in the objective and subjective clinical withdrawal scales post procedure. Dexmedetomidine may be used intravenously as it is more selective, has a shorter duration of action, and is easier to titrate than clonidine. Antiemetic medications (droperidol or ondansetron) are given simultaneously in dosages similar to those used to treat chemotherapy-induced vomiting. Another approach uses prior induction onto buprenorphine for 1 week prior to AAOD which is believed to mitigate the intensity of the withdrawal syndrome.[1]

Monitoring Thorough anesthetic monitoring of the vital functions is needed along with the ASA basic monitoring standard. A Foley catheter is helpful to monitor renal function. Hensel et al[2] used bispectral index (BIS) monitoring to regulate the depth of anesthesia with the advantage of being able to significantly reduce the total dose of propofol required, time to recovery from anesthesia, and objective withdrawal symptoms.

Chapter 64: Patient on chronic opioids who wants to have anesthesia-assisted detoxification

Induction and maintenance Anesthesia is induced with propofol and succinylcholine or a non-depolorizer can be used as the muscle relaxant in a rapid sequence of induction and intubation. Intubation is needed since opiates decrease gastric emptying and vomiting can be a prominent component of the withdrawal syndrome if adequate antiemetic medications are not administered. Maintenance anesthesia is done with an infusion containing a combination of midazolam[3] and propofol[4] or any inhalational agent. Most of the studies report the use of propofol, though methohexital[5,6] has also been used. A test dose of the opioid antagonist is used to gauge the adequacy of the α-2 blockade and is followed by a high-dose intravenous infusion of naloxone,[7] or nalmefene[8] in normal saline or naltrexone[8] via an orogastric tube into the stomach. Because a volume shift into the intestines is expected after administration of opiate antagonists, a liberal amount of lactated ringers solution is infused to maintain fluid balance. Thereafter, patients are monitored for withdrawal signs. The major signs of withdrawal under anesthesia are piloerection and myoclonic jerking as the other signs are masked by the use of the α-2 agonist. Anesthesia is maintained until the patient responds negatively to another test dose of opioid antagonist. This is usually after 6 to 8 hours but may be longer in cases of methadonemaintained patients. It is also important to remember that since the patient went to sleep with the presumption of a full stomach, extubation should only be performed when the laryngeal reflexes have returned.

4. How do you know when the detoxification is complete? One method for determining completion of the procedure is to measure spontaneous minute ventilation immediately after administration of the narcotic antagonist. It has been observed to more than double from the normal volume. When the minute ventilation falls to 75% of the peak value, the detoxification is usually complete.

What post-procedure monitoring is required? The usual policy is to discharge the patient within 24–36 hours. However, some patients who keep on

complaining of persistent withdrawal symptoms may need to be kept for a few additional days and managed symptomatically. The major complaints are usually insomnia, muscle aches, and abdominal discomfort. The use of sedatives to treat insomnia must be tempered with the risk of starting a different dependency. However, the number one reason for failure in the first few days post detoxification is the inability to sleep and some short-term measures need to be taken to restore the circadian rhythm. Muscle aches or bone pain are usually treated with an NSAID. Abdominal cramping is treated with Bentyl and with octreotide. A thorough physical examination for any anesthetic complications, persistent withdrawal symptoms, and psychiatric symptoms is needed before discharge to prevent “bounce-backs” to the emergency room. Patients who are transferred directly to an inpatient psychiatric facility to initiate an aftercare abstinence-oriented program often have better success in maintaining abstinence.

5. What complications can occur in AAOD? There are only a few case reports available; [9–12] hence, the actual prevalence cannot be determined. Further, as the procedure has been commercialized, vested interests might hinder complete and accurate reporting of the complications. The following complications can be anticipated and have been observed during the procedure:  Emesis and diarrhea: These are the prominent features of the withdrawal syndrome. Therefore antacids and antiemetics in high doses are used prophylactically. Ranitidine should be avoided as it may cause tachycardia, vomiting, insomnia, and elevation of liver enzymes in higher doses. Diarrhea may be prophylactically treated with octreotide, a synthetic polypeptide. It inhibits the anterior pituitary, suppressing the pancreatic secretions and thus inhibiting gastric acid, serotonin, and vasoactive intestinal peptide (VIP) secretion which, in turn, decreases gastrointestinal motility. Loperamide should be avoided as it is absorbed into the systemic circulation and may increase the signs of withdrawal post-procedure.  Sepsis: Some centers also advocate the use of a single dose of an antibiotic, e.g., cephalozin to prevent infection.

449

Chapter 64: Patient on chronic opioids who wants to have anesthesia-assisted detoxification

 Gastric ulcer: Cushing’s ulcers have been reported.  Cardiovascular complications: These include cardiovascular stimulation, QT prolongation[13] and bradycardia, bigeminy, and cardiac arrhythmia.  Suppression of thyroid hormones.  Psychiatric complications: These include dysphoria, an acute psychotic episode requiring haloperidol, suicide attempts on Day 3 and Day 5 post-procedure.  Deaths: Deaths have been reported 16–40 hours following the procedure.[14] In most of the cases the cause has been found to be pulmonary edema, upper gastrointestinal ulceration, and aspiration. One patient suffered from an intracerebral hemorrhage, presumably due to poor control of blood pressure. It was also seen that in these cases, the use of clonidine was very limited and it was not continued after the procedure. In most of these cases, the ASA basic monitoring standards were not followed.  Continuation of withdrawal symptoms: Many patients continue to experience moderate withdrawal symptoms hours after the anesthesia or sedation ended, including nausea, vomiting, diarrhea, and sleep disturbances.[15,16] Others report only mild to moderate symptoms for the next 3–4 days. In addition, the severity of withdrawal may also be related to the anesthetic used. It must be emphasized that aggressive control of these symptoms is necessary or the patient will immediately relapse. This means large doses of antiemetics, anti-spasmodics, and sedatives for sleep.  Duration and severity of withdrawal symptoms: Proponents suggest that the procedure is a rapid and painless method of detoxification. Therefore, an important outcome is the comparison of the duration and severity of withdrawal symptoms associated with ultra-rapid detoxification and other detoxification strategies. It has been a measure of contention about how painless this procedure really is. While the worst part of withdrawal occurs while the patient is anesthetized, it is clear that there will be discomforts and this point must not be minimized when discussing the procedure with the patients.

450

6. How does AAOD compare to traditional methods of detoxification? Completion of detoxification: 30–91% of patients may drop out of traditional inpatient detoxification regimes. Using sedation or anesthesia, one is assured of 100% completion of detoxification. It may be noted, however, that this 100% completion rate is by virtue of the method itself (because the patient is unconscious or deeply sedated and cannot physically run away until the basic procedure is over). Thus, the detoxification completion rate, of and by itself, should not be used as an outcome measure in effectiveness evaluation of ultra-rapid opiate detoxification (UROD).

7. How can the opiate-dependent patient maintain abstinent state?  Induction onto an antagonist: Both during the procedure and immediately following longacting opiate antagonists are administered. Naltrexone pellets (vide infra) have been inserted subcutaneously and can last up to 6 months. Use of injectable nalmefene in large doses intramuscularly administered will provide mu-receptor blockade for a week. A new preparation of intramuscular depot naltrexone, which was originally developed to help in alcohol abstinence, has been demonstrated to last a month. The later three methods insure “enforced abstinence” as the mureceptor blockade is exceedingly difficult to overcome.  Naltrexone pellets are custom formulated and placed subcutaneously through a small incision. Depending on the formulation and the content of pellets placed, significant plasma levels of naltrexone have been demonstrated for up to 6 months.  In a 3.4 g sustained-release naltrexone preparation produced by GoMedical Industries, Perth, Western Australia, the period of significant plasma antagonist levels was documented to extend to approximately 188 days (plasma levels greater than 2 ng/ml). At least two opiate-dependent addicts tried to overcome the blockade during this period and experienced no effect.

Chapter 64: Patient on chronic opioids who wants to have anesthesia-assisted detoxification

8. Does AAOD actually lead to prolonged abstinence? Rate of abstinence during both the short-term 6-month period of protracted withdrawal symptoms and longer-term abstinence are important outcome measures; indeed, one may argue that the postprocedure abstinence rate (short- or longer-term) constitutes one of the “gold standards” for the effectiveness evaluation of AAOD, provided the other therapeutic elements (naltrexone, psychosocial treatments, etc.) are comparable between the AAOD group and the control groups. Few studies are available to suggest that AAOD leads to a shorter or longer duration of abstinence. Other measures of outcome might include patient satisfaction, program evaluation, and finally cost– benefit efficiency. It should be remembered that the patient populations treated are not similar. For example, patients dependent on heroin might respond differently than those dependent on crude opium and the response may vary according to the duration of dependence or prior attempts at traditional detoxification. In an extensive review of the literature, O’Connor and Kosten[17] studied the research design and methodologic characteristics of 9 AAOD and 12 other rapid opiate detoxification studies. These researchers noted that most of these studies used general anesthesia except two, which used midazolam and propofol. Only three studies included a control group; only two studies used random allocation of patients and only one was blind. Most of the studies focused on the completion of detoxification or severity of withdrawal symptoms over a duration ranging from 6 hours to 12 days. It is important to note that O’Connor has a patent (US Patent# 6,541,478) on ultra-rapid detoxification for nicotine, but has not published any studies on its efficacy. Three studies evaluated outcome beyond the acute detoxification period. Legarda and Gossop[18] reported that all of the 11 subjects were still taking naltrexone after 30 days. Seoane et al[19] reported that 93% of the patients were abstinent after 1 month, although the methodology used to assess the same was not specified. They could not make a quantitative assessment of the effectiveness of AAOD due to the variability of the patients and the techniques employed along with the small number of patients

enrolled and suggested the need for more long-term studies on outcome including safety and efficacy. Albanese et al[20] studied two cohorts of patients (n¼123) for 6 months and found two key facts: (1) most failures occurred within the first 2 weeks and (2) the decline in sobriety was rather steady over the remainder of the study period with less than 50% still sober after 6 months. Bell et al[21] reviewed the literature from 1998 to 2000 and identified 21 studies on naltrexoneaccelerated procedures. They concluded that the withdrawal syndrome was quite protracted in many of the studies as reflected in the duration of inpatient stay, which varied from 24 hours to 8 days with a mean duration of 3–4 days. Only 10 out of 21 studies reported on long-term outcomes. Five of these were concerned with detoxification under anesthesia or deep sedation. The follow-up duration ranged from 3 months to 1 year. The abstinence rates ranged from 20% at 6 months to 68% at 12 months. One study compared relative abstinence rates in subjects undergoing AAOD with those in a methadone tapering group and found it to be significantly higher (67%) in the former group as compared to 33% in the latter. In terms of induction onto naltrexone, all those who completed the AAOD regime were started on naltrexone. In a retrospective follow-up study, Lawental[22] compared subjects who had undergone AAOD with those who had undergone a 30-day inpatient detoxification program (IDP) as judged by abstinence rates after 12 and 18 months. He found that only 22% in the former group reported abstinence as compared to 42% in the latter. He also commented on the costeffectiveness of the alternatives to AAOD. Krabbe et al[23] compared abstinence rates and withdrawal effects of AAOD with standard methadone tapering in a prospective 3-month follow-up trial. They found significantly higher abstinence rates and lesser and milder withdrawal symptoms in the subjects who had undergone AAOD at 1 and 2 months follow-up duration. A similar trend continued at 3 months follow-up although the differences were no more significant. Bochud Tornay et al[24] followed up 16 patients over a period of 30 months after AAOD and found that 14 of these relapsed. Collins et al[25] conducted a prospective randomized trial comparing AAOD to buprenorphine detoxification in heroin addicts. They reported that there was no benefit in comparison to buprenorphine

451

Chapter 64: Patient on chronic opioids who wants to have anesthesia-assisted detoxification

detoxification and the additional cost and complications argued against performing AAOD. The major problems with this report include a greater than 70% loss of follow-up in patients in both treatment groups, failure to prophylaxis against the objective signs of withdrawal, e.g., administering antiemetics only after the patient vomited, using small amounts of α-2 agonists, and not using octretide to reduce gastrointestinal distress. In sum, if you don’t aggressively prevent the symptoms then the post-procedure course will be stormy and the objective and subjective clinical withdrawal scales will be unnecessarily elevated. Further compromising the independence of this report is the fact that one of the authors has a financial stake in the competing drug buprenorphine. This was fully disclosed in the article, but still raises the question of observer bias. In his accompanying editorial, O’Connor[26] said this study sounded the death knell of AAOD. The irony of this comment is that, as previously mentioned, he obtained a patent on using the AAOD for detoxification from nicotine. The sample sizes of many of these trials are small, even many recent trials (e.g., Krabbe et al[27]) are not randomized and patient and recruitment characteristics (e.g., whether or not they had to pay a fee for enrolment) might influence the results. Thus it would be premature to draw any firm conclusions from the presently available evidence on the efficacy of AAOD. A more recent study from Iran[28] with a retrospective design tried to estimate the retention rate in naltrexone maintenance treatment. Studying 45 opiate-addicted male patients who were detoxified by 6 hours of intravenous naloxone infusion under general anesthesia with midazolam, propofol, clonidine, and atracurium. Withdrawal signs were evaluated by the objective opiate withdrawal scale (OOWS, range 0–13) up to 24 hours after awakening. After UROD, naltrexone 50 mg/day was prescribed for 9 months with assessments in 4-week intervals. Adverse events after AAOD were prolonged unconsciousness (n¼1), transient confusion (n¼8), and depressed mood (n¼6). The total sample showed a median OOWS score of 2 (mild withdrawal syndrome). 96% (43/45) of the patients could be discharged the day after UROD. Thirty-six patients (80%) continued naltrexone therapy for the entire 9-month observation period. They concluded that UROD and subsequently induction of naltrexone maintenance therapy can be regarded as safe and

452

effective in patients with pure opiate addiction. They concluded that cultural and economical factors may not correspond to European and American treatment modalities.

9. What evidence-based reviews have been conducted of AAOD? A Cochrane review[29] and a NICE review[30] of the procedure has found little merit in the procedure. But neither group undertook a formal meta-analysis. The NICE review found some benefit to the procedure in pooled analysis but found the high rate of complications as compared to current medical therapies unacceptable. Again, if you don’t pursue aggressive prophylaxis against the withdrawal symptoms then severe complications can occur.

10. Are there potential benefits of AAOD? The short-term effectiveness of this procedure is claimed to be 100%. Even if the claim regarding this “100% effectiveness” is contentious (see above), the fact remains that all the patients entering UROD therapy complete the detoxification process. A greater number of patients would enter long-term management protocols (usually with naltrexone maintenance and psychosocial treatments) and thus would at least have a chance to remain abstinent. The major problem is that the Objective and Subjective clinical withdrawal scales may be markedly elevated unless each symptom is aggressively controlled. This is clearly where the art of the procedure varies. As an example, one protocol used ondanestron as a rescue dose only whereas others used prophylaxis with chemotherapy levels of the medication. The procedure may be especially useful for a subset of patients who do not enter treatment for the fear of undergoing the painful conventional detoxification process. For the patients who are undergoing the process, it becomes more humane as they do not have to undergo the suffering and discomforts associated with the conventional detoxification procedures. If we reflect on the Oath of Maimonides, it is one of the responsibilities of the physician to ensure minimal suffering and relief of pain. In undertaking this procedure, the operator must commit to aggressively prevent these discomforts.

Chapter 64: Patient on chronic opioids who wants to have anesthesia-assisted detoxification

11. Should this procedure be conducted at all? The foremost issue is the risk of significant morbidity and mortality associated with the procedure, given the fact that there is practically none associated with the standard detoxification procedures. However, some practitioners believe that if such a risk can be taken in other procedures associated with relief of pain, then the same should be justifiable in this too. This issue assumes special relevance when consideration is given to the fact that there is always a possibility of under-reporting of complications related to the procedure due to vested financial interests. The high cost of the procedure is another factor. The procedure requires an extensive setup, close coordination between psychiatry and anesthesia services, and detailed monitoring post-procedure, which are not always easily available. It is also clear that this procedure should be conducted in an ambulatory surgery unit or a hospital intensive care unit or equivalent. It is not an Officebased anesthesia procedure. The major problem, which is not addressed by all the naysayers, is what to do with that cohort of patients who are on that very dose of opiates that renders them unsuitable for buprenorphine or who cannot enter methadone maintenance due to lack of treatment slots or time constraints. This cohort numbers in the tens of thousands and the concept that they should wait until therapy becomes available seems untenable, when the mortality rate of opiate abuse is approximately 2% per year of abuse.

12. What ethical conundrums occur with AAOD? There are important ethical aspects to be considered. Some researchers have patented their versions of this procedure. The American Medical Association has taken a strong stand against these patents, calling them unethical. From the point of view of patients,

References 1.

Stine SM, Kosten TR. Use of drug combinations in the treatment of opioid withdrawal. J Clin Psychopharmacol. 1992;12:203–209.

2.

they may be in extreme distress when seeking treatment and hence vulnerable to exploitation by the sellers of a quick fix to their problem. Further, there may be a selection bias operating at the treatment entry level itself, which might spuriously influence the short-term results. The commercial providers often exploit this as a “magic bullet,” claiming excellent outcome as if that is a generalized truth applicable to all opioid-dependent patients. Other ethical concerns are: safety issues (see above), full disclosure to the patient regarding the procedure with its pros and cons, and providing the patient with a menu of viable options for treatment for which AAOD is only one rather than the “ultimate” choice. The American Society of Addiction Medicine (ASAM) has issued elaborate recommendations[31] for UROD incorporating many of the above ideas. It recommends that any method of opioid detoxification is only a first step, and is not in itself an effective treatment of opioid addiction.

13. What is the role of clonidine in mitigating withdrawal? Symptoms associated with the withdrawal syndrome, such as restlessness, rhinorrhea, lacrimation, diaphoresis, myosis, piloerection, and cardiovascular changes are mediated through increased sympathetic activity. Thirty-fold increases in the levels of epinephrine and lesser increases in norepinephrine can be observed during withdrawal from opioids. During opioid withdrawal, neural activity in the locus ceruleus, the major noradrenergic nucleus in the brain, is greatly increased. This surge is responsible for many of the symptoms seen during withdrawal. Clonidine, an α2-agonist, has been shown effective in suppressing noradrenergic hyperactivity, relieving withdrawal symptoms. Without clonidine or an equivalent α2-agonist agent, AAOD would result in large increases in both total and fractionated catecholamine levels. These increases would be associated with unacceptable morbidity and mortality.

Hensel M, Kox WJ. Safety, efficacy and long-term results of a modified version of rapid opiate detoxification under general anesthesia: a prospective study in methadone, heroin, codeine and

morphine addicts. Acta Anesth Scand. 2000;44:326–333. 3.

Streel E, Dan B, Bredas P, et al. Interference with withdrawal signs of naloxone induced opiate

453

Chapter 64: Patient on chronic opioids who wants to have anesthesia-assisted detoxification

withdrawal under anaesthesia is anaesthetic specific in opiate dependent rats. Life Sci. 2001;70:517–522. 4.

5.

6.

7.

Cucchia AT, Monnat M, Spagnoli J, Ferrero F, Bertschy C. Ultra-rapid opiate detoxification using deep sedation with oral midazolam: short and long term results. Drug Alcohol Depend. 1998;52:243–250. Demaria PA Jr, Rogers C, Braccia G. Propofol for sedation during rapid opiate detoxification. Am J Psychiatry. 1997;154:290–291. Kienbaum P, Scherbaum N, Thurauf N, et al. Acute detoxification of opioid addicted patients with naloxone during propofol or methohexital anesthesia: a comparison of withdrawal symptoms, neuroendocrine, metabolic and cardiovascular patterns. Crit Care Med. 2000;28:969–976. Resnick RB, Kestenbaum RS, Washton A, Poole D. Naloxone precipitated withdrawal: a method for rapid induction onto naltrexone. Clin Pharmacol Ther. 1977;21:409–413.

8.

Kaye AD, Gevirtz C, Bosscher HA, et al. Ultrarapid opiate detoxification:a review. Can J Anaesth. 2003;50:663–671.

9.

Gold CG, Cullen DJ, Gonzales S, Houtmeyers D, Dwyer MJ. Rapid opioid detoxification during general anesthesia: a review of 20 patients. Anesthesiology. 1999;91:1639–1647.

10. Bell J, Young M, Masterman S, et al. A pilot study of naltrexone accelerated detoxification in opioid dependence. Med J Aust. 1999;171:26–30. 11. London M, Paul E, Gkolia I. Ultra-rapid detoxification in hospital. Psy Bull. 2000;23: 544–546. 12. Rozen RG, de Kan R, van den Brink W, Kerkof AF, Geerlings PJ.

454

Dangers involved in rapid opioid detoxification while using opioid antagonists: dehydration and renal failure. Addiction. 2002;97:1071–1073. 13. Allhoff T, Renzing Kohler K, Kienbaum P, Sack S, Scherbaum N. Electrocardiographic abnormalities during recovery from ultra short opiate detoxification. Addict Biol. 1999;4:337–344.

22.

23.

14. Dyer C. Addict died after rapid opiate detoxification. BMJ. 1998;316:170. 15. Seoane A, Carrasco G, Cabre L, et al. Efficacy and safety of two new methods of rapid intravenous detoxification in heroin addicts previously treated without success. Br J Psychiatry. 1997; 171:340–345.

24.

16. Laheji RJ, Krabbe PF, de Jong CA. Rapid heroin detoxification under general anesthesia. JAMA. 2000;283:1143.

25.

17. O’Connor PG, Kosten TR. Rapid and ultrarapid opioid detoxification techniques. JAMA. 1998;279:229–234.

23 February 2001. http://www. health.nsw.gov.au/public-health/ dpb/publications/pdf/ rapiddetoxification_cir2001-17. pdf. Lawental E. Ultra-rapid opiate detoxification as compared to 30-day inpatient detoxification programme: a retrospective study. J Subst Abuse. 2000;11:173–181. Krabbe PF, Koning JP, Heinen N, et al. Rapid detoxification from opioid dependence under general anesthesia versus standard methadone tapering: abstinence rates and withdrawal distress experiences. Addict Biol. 2003; 8:351–358. Bochud Tornay C, Favrest B, Monnat M, et al. Ultra-rapid opiate detoxification using deep sedation and prior oral buprenorphine preparation. Drug Alcohol Depend. 2003;69:283–288. Collins ED, Kleber HD, Whittington RA, Heitler NE. Anesthesia-assisted vs buprenorphine or clonidineassisted heroin detoxification and naltrexone induction. JAMA. 2005;294:903–913. O’Connor PG. Methods of detoxification and their role in treating patients with opioid dependence. JAMA. 2005; 294(8):961–963.

18. Legarda JJ, Gossop M. A 24-hour inpatient detoxification programme for heroin addicts: a preliminary investigation. Drug Alcohol Depend. 1994;35:91–93.

26.

19. Seoane AG, Carrasco L. Cabr A. et al. Efficacy and safety of two new methods of rapid intravenous detoxification in heroin addicts previously treated without success. Br J Psychiatry. 1997; 171:340–345.

27. Krabbe P, Koning J, Heinen N, et al. Rapid detoxification from opioid dependence under general anaesthesia versus standard methadone tapering: abstinence rates and withdrawal distress experiences. Addiction Biol. 2003;8(3):351–358.

20. Albanese AP, Gevirtz C, Oppenheim B, et al. Outcome and six month follow-up of patients after ultra-rapid opiate detoxification. J Addict Dis. 2000;19:11–28. 21. Bell J, Kimber J, Lintzeris N. Guidelines for rapid detoxification from opioids. NSW Health, circular no. 2001/17, file no. 00/1287, issued on

28. Naderi-Heiden A, Naderi A, Naderi MM, et al. Ultra-rapid opiate detoxification followed by nine months of naltrexone maintenance therapy in Iran. Pharmacopsychiatry. 2010; 43(4):130–137. 29. Gowing L, Ali R, White J. Opioid antagonists under heavy sedation or anaesthesia for opioid

Chapter 64: Patient on chronic opioids who wants to have anesthesia-assisted detoxification

withdrawal. The Cochrane Library, Oxford: Update Software. 2002;2. 30. Pilling S, Strang J, Gerada C. Psychosocial interventions and

opioid detoxification for drug misuse: Summary of NICE guidance. BMJ. 2007;335:203–205. 31. Public policy statement on Opioid Antagonist Detoxification

Under Sedation or Anesthesia (OADUSA). American Society of Addiction Medicine (ASAM). J Addict Dis. 2000;19: 109–112.

455

Section 7 Chapter

65

Special Topics

Munchausen syndrome and pain Santhosh A. Thomas and Sachin K. Bansal

Case study A 60-year-old woman with multiple cervical and lumbar spine surgeries underwent a successful cervical and thoracic spinal cord stimulator trial. Permanent spinal cord stimulator leads and batteries were implanted with similar coverage of pain. Over the ensuing 2 months, the patient reported either loss of coverage or no coverage. There was no evidence of lead migration, fracture, or damage. The batteries were functioning normally. After every reprogramming attempt, the patient left satisfied feeling coverage despite minimal changes in settings. What is the most likely diagnosis? “The only antidote for mental suffering is physical pain” Karl Marx

Munchausen syndrome is a psychiatric disorder characterized by individuals who invent illness or injury, in order to draw attention, sympathy, or to meet deep emotional needs. Patients consciously and voluntarily produce particularly unending, dramatic, or severe physical symptoms of disease. The patients frequently give plausible and dramatic history, all of which is forged. They repeatedly go to great lengths to avoid the detection of their deception. This makes it more difficult to notice these presentations as part of a serious mental disorder. Often times, a series of exhaustive medical examinations by numerous practitioners have been performed. Usually the individual has undergone extensive testing, all of which is often inconclusive as to the origin of their pain and symptoms. While these patients can present with any complaint, such as psychologic trauma or a disease, perhaps the

most common complaint is pain or discomfort in a region of their body. Patients regularly complain of non-organic pain including back pain, abdominal pain, and headaches that is difficult to evaluate or diagnose. They often have cuts on their bodies. Patients with Munchausen are not sick but want to be sick or injured. They are very knowledgeable about their “illness” and are very convincing. Patients have extensive knowledge of the hospital or healthcare facility and are willing to travel great distances to address their concerns. They time and again have a good understanding and can use medical terminology during their visit. Munchausen is generally classified as a factitious disorder. When evaluating a patient who presents with pain, one often encounters a very large charge detailing numerous office visits, laboratory work-up, and radiologic work-up. Because of the abundant visits, the patient often views him or herself as disenfranchised and feels that he or she has not been treated fairly. The patient may gripe about other care takers to the evaluating practitioner and state that they were not cared for appropriately and adequately, or that a proper and through work-up was not performed. Healthcare workers often believe that these patients are purposely feigning their illness to seek attention; however recently there has been a drive to reclassify Munchausen as a somatoform disorder with the thought that these patients may not be consciously drawing attention to themselves. Nonetheless, the current belief is that the syndrome is a factitious one in which patients are aware that they are feigning symptoms and are doing so for attention and sympathy (not for external benefits such as monetary awards).

Case Studies in Pain Management, ed. Alan David Kaye and Rinoo V. Shah. Published by Cambridge University Press. © Cambridge University Press 2015.

456

Chapter 65: Munchausen syndrome and pain

What is the origin and history of this condition? The term “munchausen” derives from Baron Münchhausen, a German nobleman, who was known to embellish and fabricate fantastic stories about his life. Often times these stories involved feats which were virtually impossible. In 1785, writer Rudolf Raspe even published a book titled “The Surprising Adventures of Baron Münchhausen.” It was not until 1951 that Richard Asher, an eminent British endocrinologist, noticed a pattern of behavior in individuals characterized by such fabrication, especially in regards to symptoms of illness. Noting the history of Baron Münchhausen, he named the syndrome Munchausen syndrome and went on to describe the syndrome in The Lancet.

How is this diagnosed? Diagnosis of Munchausen’s syndrome can be quite complicated. The practitioner must ensure that their encounter with the patient is not viewed as being offensive or obtrusive in any manner. In such circumstances, the patient is likely to take a defensive stance. The likely result of such a scenario would be the perpetuation of the patient’s syndrome and the consultation of another practitioner elsewhere in this regard. One must be careful not to arrive at this diagnosis blindly or based upon a hunch. It is essential to review pertinent medical documentation, laboratory workup, and appropriate films to ensure that there is no organic cause for the patient’s pain. In circumstances of Munchausen’s syndrome manifested by complaints of pain, one must rule out other pain syndromes such as fibromyalgia, myofascial pain syndrome, and central pain syndrome among others. This can be a challenging task, and is best performed in conjunction with a comprehensive team consisting at least of a psychiatrist skilled at diagnosing and managing these patients. Often times the patient may be unreceptive or hostile toward seeing a psychiatrist or psychologist, believing that this indicates that the practitioner suspects a factitious nature to his or her symptoms. In addition, it is important to recognize that Munchausen’s syndrome is a severe and chronic disorder. As mentioned above, patients have

generally exhausted medical evaluations and even treatments for their reported pain symptoms. When a patient presents with complaints of pain without such a history of chronicity, multiple medical evaluations and hospitalizations, and extensive work-up, Munchausen should be very low on the differential diagnosis. They may be reluctant to meet family and prior physicians as this exposes them to the risk of being discovered and revealing inconsistencies in the presentation. In fact, in regards to the frequency of Munchausen’s syndrome in the chronic pain population, Fishbain et al in the Clinical Journal of Pain demonstrated that of the patients who present with chronic pain, very few have Munchausen’s syndrome. In this study, out of 2860 patients who had chronic pain, only four were shown to have Munchausen’s syndrome. This totals 0.14%. Despite the remoteness of this study, it certainly demonstrates that one should examine all other possibilities before considering Munchausen’s a viable diagnosis in regards to chronic pain. Nonetheless, Munchausen’s continues to be a significant problem for all practitioners but perhaps the most impacted are emergency room providers who are often the first ones to encounter these patients. Patients with Munchausen’s syndrome are adept in making up symptoms or cause illness in several ways. They may give the healthcare provider or a loved one false medical history as having cancer or HIV. They may belong to internet support groups. They may be able to fake symptoms commonly manifesting as heart attack, abdominal pain, diarrhea, back pain, seizures, and passing out. These symptoms fit precisely into textbook description of the illness. They have a common exception as they do not respond to conventional treatment. Self harm is not only limited to cutting or burning themselves but can include ingesting medications such as diabetic medications, anticoagulants, and chemotherapy. One case report from San Francisco General Hospital details a young individual who purposely injects himself with insulin and subsequently requires intravenous therapy, endotracheal intubation, and admission to an intensive care unit. They are not shy with impairing healing of wounds which may be self-inflicted by reopening them or contaminating them. They are eager and willing to have medical tests, procedures, and operations.

457

Chapter 65: Munchausen syndrome and pain

What are the treatment options? First and foremost, the evaluating physician must rule out any organic cause for the patient’s pain. While evaluating these patients is timely and often costly, the diagnosis of Munchausen’s syndrome should not be made without this investment of time and evaluation. Often times when an individual has Munchausen’s syndrome they have other concomitant psychiatric disorders such as anxiety disorder, depression, or even borderline personality. Treatment of their Munchausen’s syndrome by a trained psychiatrist cannot be appropriately performed without addressing these other disorders. Frequently, patients are dismissed as being hypochondriacs, which is a mistake. Hypochondriacs truly believe they are ill and seek treatment accordingly. Patient’s with Munchausen’s syndrome know and believe that they are not ill, but still possess a desire to be ill to attract attention (not physical or monetary benefit, however). This is different from Munchausen syndrome by proxy, which is a relatively rare form of child abuse that involves exaggeration or fabrication of symptoms by a primary caretaker. The caretaker acts as if the individual he or she is caring for has a physical or mental illness when the person is not actually sick. “By proxy” indicates that a parent or another adult is embellishing or manufacturing the symptoms in a child not in himself or herself. In regards to pain, there are numerous ways in which patient’s may simulate their symptoms. Sometimes they may go to such extremes as to cause themselves physical injury, such as cutting or burning themselves. They may fabricate stories as to how these lesions occurred. This can be quite dangerous as some individuals will go to great lengths to harm

References 1.

Asher R. Munchausen’s syndrome. Lancet 1951;1 (6650):339–341.

2.

Bretz SW, Richards JR. Munchausen syndrome

458

3.

themselves in order to seek treatment. There have been documented cases of individuals even amputating limbs. Therefore while Munchausen’s syndrome may start as an inorganic disorder of feigning disease, it may translate into severe organic dysfunction. If suspected, in particular in the inpatient setting, Munchausen’s syndrome should always be evaluated by a psychiatrist. While overall the prognosis for patients with Munchausen’s syndrome is thought to be poor, with efficient evaluation and treatment and avoidance of unnecessary procedures or testing, the prognosis may improve. Huffman et al from the Department of Psychiatry at Massachusetts General Hospital describe this in reference to a particularly troublesome female patient. Huffman goes on to state that using such caution when evaluating these patients and in particularly minimizing the chance for the patient to harm himself or herself is key in the effective management of Munchausen’s syndrome. Furthermore, regularly scheduled appointments with the patient and his or her treating psychiatrist can give the patient the attention he or she seeks without having to utilize unnecessary resources or distress the patient. The cause of Munchausen’s syndrome is unknown but these patients often have experienced severe illness when they were young, or have had severe emotional or physical abuse or neglect. Although no known cure exists for Munchausen’s syndrome, if the patient is able to modify their behavior, reduce his or her misuse and overuse of the available medical resources, and understand the serious consequences due to self induced conditions, we may be able to help these patients with their psychologic condition. Medicine may be used to treat the related or coexisting disorders such as depression, anxiety, or personality disorder.

presenting acutely in the emergency department. J Emerg Med. 2000; 18(4):417–420. Fishbain DA, Goldberg M, Rosomoff RS, Rosomoff HL. More Munchausen with

chronic pain. Clin J Pain. 1991;7 (3):237–244. 4.

Huffman JC, Stern TA. The diagnosis and treatment of Munchausen’s syndrome. Gen Hosp Psychiatry. 2003;25 (5):358–363.

Section 7 Chapter

66

Special Topics

Insomnia and chronic pain Mark Etscheidt and Paul A. Sloan

Case study A 57-year-old male presents at a multidisciplinary pain clinic with chronic low back pain and radicular symptoms into the right lower extremity. He is overweight and ambulates slowly with the use of a cane. He had gradual onset of pain over 18 years ago with no clear precipitant. He underwent cervical discectomy for radiculopathy 6 years ago with complete recovery. An MRI 1 year ago identified spinal stenosis at the L4–5 level. Subsequent lumbar discectomy and fusion was of no benefit in reducing his pain. His activity is restricted to that around the house. He has not drunk any alcohol in the past 30 years since being diagnosed with chronic pancreatitis. He reported heavy alcohol use on weekends in his early 20s, but never any legal problems. He currently smokes one pack of cigarettes daily, drinks four cups of coffee in the morning and one in the evening, and denies any use of illegal/recreational drugs. Current medications include chlordiazepoxide/clidinium, lamotrigine (50 mg), duloxetine, clonazapam (0.5 mg), fenofibrate, quetiapine, trazadone (150 mg), and hydrocodone/ acetaminophen (10 mg BID). He has had “anxiety attacks” the past 10 years, but psychiatric treatment has decreased the frequency of these attacks to about two episodes per month. He admits to being depressed, with his last suicide attempt being 1½ years ago. Sleep is impaired with delayed sleep onset, frequent night-time awakenings with shortness of breath, daytime fatigue, and heavy snoring.

1. How common are sleep disorders among chronic pain patients? The prevalence of insomnia in the general population is 10–20%.[1] Over 75% of psychiatric patients

experience sleep disorders, and nearly all patients with anxiety disorders (with the exception of simple phobias) have difficulties with initiation and maintenance of sleep.[2] Since chronic pain is often accompanied by emotional and psychosocial difficulties, it is reasonable to expect that chronic pain patients would exhibit sleep disorders. Complaints of impaired sleep are common among patients with chronic pain, with 65–78% of chronic pain patients describing themselves as poor sleepers.[3,4] Even for those chronic pain patients who described themselves as good sleepers the incidence of sleep onset and sleep maintenance difficulties was 29% and 26% respectively.[4] In a study of back pain patients, 46.7% had clinically significant insomnia, 32.5% had sub-threshold insomnia, and 20.8% had no insomnia.[5] Much like chronic pain, insomnia is associated with significant morbidity in terms of health problems and healthcare utilization, work absenteeism and reduced productivity, and risk of non-motor-vehicle accidents.[6] The total direct, indirect, and related costs of insomnia has been conservatively estimated at $30 to 35 billion annually in the USA.[7]

2. What type of sleep disorder is this patient struggling with? Insomnia is primarily diagnosed with a thorough clinical interview. The clinical evaluation of insomnia should include exploration of a detailed sleep history along with a medical, substance use, and psychiatric history.[8] Insomnia can be classified as either primary or secondary. Primary insomnia is an independent disorder of sleep, although psychiatric, medical, and drug problems may be a consequence of this insomnia disorder

Case Studies in Pain Management, ed. Alan David Kaye and Rinoo V. Shah. Published by Cambridge University Press. © Cambridge University Press 2015.

459

Chapter 66: Insomnia and chronic pain

(See the American Psychiatric Association Diagnostic and Statistical Manual of Mental Disorders, 5th ed. 2013).[9] Secondary insomnia is when a sleep disturbance is a by-product of other problems such as psychiatric, medical, or drug use. If a chronic pain patient begins experiencing sleep difficulties after the onset of their pain, this would likely be considered secondary insomnia. Conditioned insomnia is typified by insomnia problems emerging during a stressful time, but which persist after resolution of the stress. Patients may experience heightened arousal because of anticipatory fear of not being able to fall asleep, and they often have ruminative thoughts centering on their frustration with poor sleep and the potential negative consequences upon their life. Circadian rhythm disorders are typified by patient complaints that they cannot stay awake or fall asleep at the desired time. Shift-workers or those traveling abroad are most likely to have these complaints. In phase delay syndrome the major sleep period is delayed, and it is perceived as incongruent with the patient’s expectation as to when they should fall asleep. Some chronic pain patients may experience a phase delay when they are no longer required to rise at a specific time, resulting in extension of time spent in bed in the morning with a subsequent shift in their sleep cycle. Sleep-related breathing disorders include obstructive sleep apnea and central sleep apnea. Signs suggestive of sleep apnea include daytime sleepiness, snoring, sleep stoppage (observed by others), dry mouth, obesity, and narrow airway. A high incidence of obstructive and central sleep apnea was found in chronic pain patients on opioids.[10] In particular, there was a direct relation between the central apnea index and the daily dosage of methadone and benzodiazepines. Certain changes such as weight gain often associated with chronic pain patients due to a more sedentary lifestyle may contribute to the development of sleep apnea. Decreased activity can contribute to weight gain for many chronic pain patients. Obesity, especially visceral obesity, in conjunction with insulin resistance is associated with sleep apnea.[11] Sleep apnea treatments include CPAP, dental appliances, or surgical intervention. Polysomnography is indicated when a breathing or movement disorder is observed or suspected.

460

3. What information is needed to assist in the diagnosis and treatment of this sleep problem? Interview The clinical evaluation of insomnia identifies specific sleep complaints (e.g., delayed sleep onset, frequent night-time awakenings, short duration of sleep, daytime fatigue, snoring, nightmares, etc.). The history of onset and life events correlating with sleep difficulties should be explored. Physical and mental status examination can assist in elucidating precipitating and/or perpetuating factors.

Polysomnography Sleep lab evaluation is not indicated for routine evaluation of chronic insomnia.[8] It is indicated if breathing or movement disorders are suspected. Certain chronic pain disorders such as fibromyalgia may warrant polysomnography especially if insomnia predated the onset of the pain symptoms.

Sleep diary The diary will include time to sleep, time to awaken, sleep latency, total sleep time, and total time in bed. In treatment outcome research, insomnia is often operationalized as a sleep-onset latency greater than 30 minutes and a sleep efficiency ratio of less than 85%.[12] The typical sleep diary includes time to bed, time until fell asleep, sleep interruptions, time until returned to sleep, wake-up time, time got out of bed, satisfaction with sleep, and any medications and/or substances used for sleep. The diary is the key to making medication and behavioral recommendations to improve sleep.

Psychometric tools There are a number of psychometric tools to evaluate insomnia. The Insomnia Severity Index is a 7-item questionnaire that has been shown to be a reliable and valid instrument to quantify perceived insomnia severity, and can be a useful tool as a screening device or as an outcome measure in insomnia treatment research.[13] The Epworth Sleepiness Scale consists of eight items that are useful to measure daytime sleepiness, and it is brief and repeatable.[14] The

Chapter 66: Insomnia and chronic pain

Pittsburgh Sleep Quality Index and Insomnia Severity Index consists of 19 items, with five other items to be completed by a person who has observed the person’s sleep. It is designed to measure sleep over the prior month and discriminates between good and poor sleepers.[15] The Pittsburgh Sleep Quality Index and Insomnia Severity Index have been shown to be useful screening tools for evaluating insomnia in low back pain patients.[16]

Other medications such as beta-blockers, bronchodilators, and calcium channel blockers can cause sleep difficulties. Risk–benefit must be weighed along with alternative medications.

4. Are there any drugs that may be contributing to his sleep problem?

Proper sleep hygiene is important for all experiencing sleep difficulties. However, there is limited evidence that sleep hygiene alone is effective in the treatment of insomnia.[8] A number of recommendations have been offered.[9,18] 1. Allow enough time for sleep, which is generally 7–9 hours. 2. Establish a regular sleep–wake cycle. It is especially important to have a consistent wake-up time (within a 1-hour window) as arousal time is an important synchronizer of circadian rhythms. 3. When you get up, stay up until your anticipated bedtime. If you have difficulty sleeping at night, avoid napping during the daytime. 4. Don’t go to bed until you feel sleepy. Laying down in bed should be associated with sleep. 5. Avoid stressful events and worry just before bedtime. Find a way to relax prior to bedtime such as meditation, reading, or watching TV. 6. The bedroom environment should be cool, dark, and quiet. Generally, watching television in bed is not recommended for those with sleep difficulties. However, for some patients, a television or radio may prevent anxiety provoking cognitive ruminations and thereby assist with improved sleep onset for some patients. However, if left on it will be disruptive to sleep maintenance. A timer to shut off the television or radio would be essential if used for this purpose. 7. A relaxing night-time routine is important. Avoid stressful conversation or projects just prior to bedtime. 8. The bedroom should be reserved almost exclusively for sleep (with the exception of sexual activity). In this way laying down in bed will become a more potent conditioned stimulus for sleep. 9. Avoid caffeine, alcohol, and nicotine.

Caffeine is a central nervous system stimulant. Many pain patients feel a need to use caffeine to combat daytime fatigue. In the absence of anxiety disorders, it may be reasonable for patients to consume a reasonable amount of caffeine upon awakening for the day. However, because of its long half-life (average 4 hours range 2–10 hours), it should be avoided for at least 6 hours, and preferably 10 hours before bedtime. While mild to moderate consumption may have little effect on non-pain patients, the combination of caffeine intake and pain may result in heightened arousal and impair sleep in patients who had no sleep difficulties associated with caffeine intake prior to the onset of their pain problem. Alcohol is a central nervous system depressant. While alcohol can facilitate sleep onset and deepen initial sleep, as it is metabolized, withdrawal symptoms emerge which are disruptive to continuation of sleep. Opioids have a complicated and poorly understand effect on sleep. Although they may cause initial drowsiness, the long-term use may lead to sleep disturbances and a decrease in quality of sleep.[17] Prospective clinical trials are needed to evaluate the effect of chronic opioids on sleep. Nicotine is a potent central nervous system stimulant which can cause problems initiating and sustaining sleep. Although smokers typically perceive nicotine as relaxing (most likely due to removal of nicotine withdrawal), the net result is that of autonomic activation which contributes to problems in initiating and maintaining sleep. Because of its multiple negative health consequences the best recommendation is to quit. For those who refuse to quit, reducing or eliminating nicotine 1–2 hours before bedtime should be urged.

5. What are some initial recommendations you might make to address sleep difficulties?

461

Chapter 66: Insomnia and chronic pain

10. Regular exercise for 30 minutes at least 3 days per week can improve sleep. However, vigorous exercise should be avoided for 3 hours before bedtime (typically not an issue for chronic pain patients). 11. Find a bed/mattress that is comfortable to you. Replace worn out mattresses (about every 5–10 years depending on the quality of the mattress). 12. Avoid large, later dinners/snacks.

6. Are there any medications you might consider to assist with his sleep? Prescription Benzodiazepines can improve sleep continuity. However, they can affect sleep architecture by increasing stages 1 and 2 and decreasing deep stage sleep (i.e., stages 3–4).[9] Benzodiazepines shorten sleep latency and increase total sleep time. Short-term efficacy of benzodiazepines and zolpidem is well documented.[19] Short-term use of hypnotics should be considered in conjunction with cognitive and/or behavioral therapies for sleep.[8] Since the efficacy of benzodiazepines ceases when medication is withdrawn, behavioral interventions are indicated for more permanent improvement in sleep. Long-term use of benzodiazepines can be dependency-producing and there are significant side effects upon withdrawal, including rebound sleep disruption and lower seizure threshold. Medication management with chronic pain patients should take into account the safety of longterm use. Undesirable side effects include adverse cognitive events, adverse psychomotor events, and daytime fatigue.[20] Receiving hypnotic prescriptions was associated with greater than threefold increased hazards of death even when prescribed on an intermittent basis.[21] Because they are highly dependency-producing, they should be used with caution. Tolerance can develop within 1 to 2 weeks, leading to rebound insomnia beyond baseline levels. Patients may become chronic users of benzodiazepines despite loss of efficacy because of the adverse effects of stopping these drugs. If used, a benzodiazepine with a short half-life is preferred, on a time limited basis. Chronic pain patients with anxiety disorders are especially prone to becoming dependent on these medications.

462

Antidepressants are recommended in the treatment of insomnia include trazadone, amitriptyline, doxepin, and mirtazapine.[8,22] They are typically prescribed in low doses. Adverse effects include sedation, headache, sweating, weight changes, nausea, and vomiting. There is also a risk potential for adverse cardiovascular effects. Antidepressants with sleeppromoting effects such as doxepin, mirtazapine, trazodone, and trimipramine may be useful for chronic pain patients with depression and/or neuropathic pain. Unlike other tricyclic antidepressants, low-dose doxepin, at doses of 3 mg and 6 mg, has been demonstrated to be free of anticholinergic effects and facilitates sleep maintenance.[23,24] Due to the activating effects of some antidepressants such as fluoxetine, paroxetine, imipramine, desipramine, venlafaxine, and bupropion, the risk–benefit must be appraised for each patient. The SNRIs duloxetine and milnacipran were not superior to placebo in reducing sleep problems for fibromyalgia patients, and the dropout rates due to adverse events were higher than for placebo.[25] Melatonin receptor agonists include ramelteon and agomelatine. Compared to melatonin, they have a longer half-life and greater affinity for melatonin receptors.[26] Ramelteon has been shown to produce significant improvements in both subjective and polysomnographic sleep latency, total sleep time, and latency to REM for patients 18–64 years.[26] There were no differences between ramelteon compared to placebo with regard to adverse events, with the most common side effects being headache, somnolence, and sore throat. Non-benzodiazepine hypnotic drugs (NBHD) include zolpidem, zopiclone, and zaleplon. They demonstrate hypnotic efficacy similar to that of benzodiazepines.[27] While objective and subjective sleep latency improved compared with placebo, the size of this effect was small and needs to be balanced with concerns about adverse effects, tolerance, and potential addiction.[28] Stimulants like modafinil and methylphenidate are used to combat excessive sleepiness associated with narcolepsy.

Non-prescription Alcohol is a central nervous system sedative and can facilitate sleep onset. However, as the alcohol is

Chapter 66: Insomnia and chronic pain

metabolized a withdrawal syndrome occurs. While a glass of wine with a meal is unlikely to negatively impact upon sleep, having several drinks up until bedtime can disrupt and shorten sleep. Melatonin is a neurohormone naturally secreted by the pineal gland that is involved in sleep and immune function. Melatonin production is controlled by an endogenous circadian rhythm system and appears to be suppressed by light.[29] It can be useful for shifting the sleep–wake cycle and can be useful for sleep onset difficulties in chronic pain patients.[30]

7. What other non-pharmacologic treatments are available? Psychologic and behavioral interventions are indicated in the treatment of primary and secondary insomnia.[8] Psychologic treatments for insomnia can reduce the use of hypnotic drugs and are durable with improvement persisting for at least 1 year among the more treatment-adherent patients.[31] Cognitive behavioral treatment of insomnia (CBT-I) has been shown to be useful for management of insomnia associated with arthritis and fibromyalgia. In a comparison of cognitive behavioral therapy for insomnia and sleep hygiene for patients with fibromyalgia, only the CBT-I group improved in several sleep variables, fatigue, daily functioning, pain catastrophizing, anxiety, and depression. The sleep hygiene group improved only in sleep quality.[32] CBT-I interventions include stimulus control and sleep restriction. Stimulus control involves extinguishing the association between the bedroom environment and behaviors causing hyperarousal. For example, many chronic pain patients will isolate themselves in their bedroom during pain exacerbations. Behavioral analysis would predict that the bedroom would become a conditioned stimulus for an aroused state which would be incompatible with sleep. Furthermore, lying in bed awake is idle time for chronic pain patients. The most salient stimulus such patients process is the pain itself, which can cause heightened arousal which in turn is incompatible with sleep. Sleep restriction reduces the amount of time spent in bed compared with the estimated amount of time the patient spends sleeping. Based upon the sleep diary findings, sleep restriction is initiated with time allowed in bed limited to actual sleep time. The

minimum amount of time should not be restricted to less than 4 hours. Time is then gradually increased to an optimal amount for the patient based upon sleep efficiency. Sleep duration and sleep efficiency are key variables ascertained from the patient’s sleep diary. A 1–2 week baseline is recommended. Once average sleep duration is identified, sleep onset and awakening times are set. Patient adherence to the schedule is important. They must get up at the agreed hour. However, they should not go to bed unless they are sleepy. If they awaken during the night and cannot fall back asleep within a reasonable period of time (e.g., 15–20 minutes) they should get out of bed and leave the bedroom until they feel like they can fall asleep; at which time they can return to their bed. When sleep efficiency (time asleep/time in bed × 100) improves to 85% or more for 5 consecutive nights, time allowed in bed is incremented by 15 minutes. It is not unusual for chronic pain patients to report only 2–3 hours of sleep at night. For example, a patient may also report that they only get 3 hours of sleep because they go to bed at midnight, have delayed sleep onset, awaken frequently, have delayed return to sleep, and finally get out of bed at 7 AM. Sleep restriction may require the patient to not go to bed until 3 AM and to get out of bed at their regular 7 AM time. This paradoxical prescription often changes the patient’s cognitive set from worrying about not getting to sleep to trying to stay awake, something inherently much more difficult and therefore less stressful. Patients who are concerned about the short amount of time spent in bed need reassurance. The longer one remains awake during the day, the higher the sleep drive (i.e., the need to go to and stay a sleep). So even on nights where they get little sleep, it will heighten sleepiness the next night. Cognitive therapy is indicated to address negative cognitions that are incompatible with sleep. Chronic pain patients often find themselves lying in bed with their mind focused on their problems (e.g., financial worry, vocational uncertainties, the well-being of their family, the meaning of their pain, etc.). Chronic pain patients also can become preoccupied with the inability to fall asleep and/or maintain sleep much like those with primary insomnia. Cognitive therapy, such as teaching problem-solving strategies and correcting misconceptions, can be very helpful in

463

Chapter 66: Insomnia and chronic pain

defusing the anxiety associated with maladaptive appraisals of sleep and life events. Exercise can increase sleep drive, although it should be avoided for 3 hours before bedtime. Metaanalytic studies have yielded moderate effect sizes of exercise for sleep. Exercise increased total sleep time and delayed REM sleep onset (10 minutes), increased slow-wave sleep, and reduced REM sleep (2–5 minutes).[33] Aerobic exercise promotes sleep in patients with chronic primary insomnia.[34,35] Exercise has also been shown to improve self-reported sleep quality in patients with major depression.[36] The sedentary lifestyle of many chronic pain patients likely contributes to fragmentation of sleep. Dozing off during the day due to too much idle time or spending excessive time lying in bed is common among chronic pain patients. Human instinct tends to inhibit activity and foster rest in response to pain. While this is appropriate for acute pain, inactivity can become a perpetuating factor in chronic pain and chronic insomnia. For chronic pain patients, regular exercise in late afternoon or early evening can be helpful in improving sleep, reducing stress, and improving pain tolerance. Patients should start at an intensity commensurate with their physical limitations and increase exercise duration and intensity as tolerated.

References 1.

Buysse DJ, Reynolds CF 3rd, Monk TH, Berman SR, Kupfer DJ. The Pittsburgh Sleep Quality Index: a new instrument for psychiatric practice and research. Psychiatry Res. 1989;28:193–213.

2.

Morin CM. Insomnia: Psychological Assessment and Management. New York: Guilford Press. 1993.

3.

Bahouq H, Allali F, Hmamouchi I, Hajjaj-Hassouni N. Prevalence and severity of insomnia in chronic low back pain patients. Rheumatology Int. 2013;33: 1277–1281. Morin CM, Gibson D, Wade, J. Self-reported sleep and mood disturbance in chronic pain

4.

464

Relaxation can be helpful for both chronic pain and insomnia.[37,38] Relaxation training was a component of cognitive behavioral treatment of insomnia in chronic pain patients.[39] However, relaxation is not as effective as CBT-I in the management of primary insomnia.[40] Relaxation is especially useful for insomnia when chronic pain patients present with anxiety, musculoskeletal, or headache difficulties. Relaxation techniques can be used to displace cognitive rumination and diminish muscle tension that may impede sleep onset.

8. Conclusions When chronic pain patients present with multiple factors that negatively impact upon sleep, one would surmise heightened insomnia. Thus a chronic pain patient exhibiting depression/anxiety symptoms along with engaging in little to no exercise, smoking cigarettes, and drinking caffeine up until bedtime will inevitably complain of poor sleep. A multidisciplinary approach is required to help this patient make progress with their chronic pain and sleep disorder problems. The combination of medication and CBT-I should be initiated for insomnia difficulties among chronic pain patients with the goal to gradually reduce patient dependency on medications for sleep. chronic insomnia in adults. J Clin Sleep Med. 2008;4: 487–504.

patients. Clin J Pain. 1998;14: 311–314. 5.

6.

Purushothaman B, Singh A, Lingutla K, et al. Prevalence of insomnia in patients with chronic back pain. J Orthop Surg. 2013;21:68–70. Daley M, Morin CM, LeBlanc M, et al. Insomnia and its relationship to health-care utilization, work absenteeism, productivity and accidents. Sleep Med. 2009;10:427–438.

7.

Chilcott LA, Shapiro CM. The socioeconomic impact of insomnia: an overview. Pharmacoeconomics. 1996;10 (Suppl 1):1–14.

8.

Schutte-Rodin S, Broch L, Buysse D, Dorsey C, Sateia M. Clinical guideline for the evaluation and management of

9.

Morin CM, Espie CA. Insomnia: A Clinical Guide to Assessment and Treatment. New York: Kluwer Academic/Plenum Publishers. 2003.

10. Webster LR, Choi Y, Desai H, Webster L, Grant BJB. Sleepdisordered breathing and chronic opioid therapy. Pain Medicine. 2008;9:425–432. 11. Vgontzas AN, Papanicolaou DA, Bixler EO, et al. Sleep apnea and daytime sleepiness and fatigue: relation to visceral obesity, insulin resistance, and hypercytokinemia. J Clin Endocrinol Metab. 2000;85:1151–1158. 12. Lacks P, Morin CM. Recent advances in the assessment

Chapter 66: Insomnia and chronic pain

and treatment of insomnia. J Consult Clin Psychol. 1992;60:586–594. 13. Bastien CH, Vallières A, Morin CM. Validation of the Insomnia Severity Index as an outcome measure for insomnia research. Sleep Med. 2001;2:297–307. 14. Johns MW. A new method for measuring daytime sleepiness: the Epworth sleepiness scale. Sleep. 1991;14:540–545. 15. Buyssee DE. Insomnia. JAMA. 2013;309:706–716. 16. Alsaadi SM, McAuley JH, Hush JM, et al. Detecting insomnia in patients with low back pain: accuracy of four selfreport sleep measures. BMC Musculoskelet Disord. 2013;14:196. 17. Panagiotou I, Mystakidou K. Non-analgesic effects of opioids: opioids’ effects on sleep (including sleep apnea). Curr Pharm Des. 2012;37:6025–6033. 18. Riete M, Weissberg M, Ruddy J. Clinical Manual for Evaluation and Treatment of Sleep Disorders. Washington, DC: American Psychiatric Publishing. 2009. 19. Nowell PD, Mazumdar S, Buysse DJ, et al. Benzodiazepines and zolpidem for chronic insomnia: a meta-analysis of treatment efficacy. JAMA. 1997;278:2170–2177. 20. Glass J, Lanctôt KL, Herrmann N, Sproule BA, Busto UE. Sedative hypnotics in older people with insomnia: meta-analysis of risks and benefits. BMJ. 2005;331 (7526):1169. 21. Kripke DF, Langer RD, Kline LE. Hypnotics’ association with mortality or cancer: a matched cohort study. BMJ Open. 2012;2: e000850. 22. Mittur A. Trazadone: properties and utility in multiple disorders.

Expert Rev Clin Pharmacol. 2011;4:181–196. 23. Krystal AD, Durrence HH, Scharf M, et al. Efficacy and safety of Doxepin 1 mg and 3 mg in a 12-week sleep laboratory and outpatient trial of elderly subjects with chronic primary insomnia. Sleep. 2010;33:1553–1561. 24. Richey SM, Krystal AD. Pharmacological advances in the treatment of insomnia. Curr Pharm Des. 2011;17:1471–1475. 25. Häuser W, Urrútia G, Tort S, Uçeyler N, Walitt B. Serotonin and noradrenaline reuptake inhibitors (SNRIs) for fibromyalgia syndrome. Cochrane Database Syst Rev. 2013;1: CD010292. 26. Liu J, Wang LN. Ramelteon in the treatment of chronic insomnia: systematic review and metaanalysis. Int J Clin Pract. 2012;66:867–873. 27. Voshaar RC, van Balkom AJ, Zitman FG. Zolpidem is not superior to temazepam with respect to rebound insomnia: a controlled study. Eur Neuropsychopharmacol. 2004;14:301–306. 28. Huedo-Medina TB, Kirsch I, Middlemass J, Klonizakis M, Siriwardena AN. Effectiveness of non-benzodiazepine hypnotics in treatment of adult insomnia: meta-analysis of data submitted to the Food and Drug Administration. BMJ. 2012; 345:e8343. 29. Macchi MM, Bruce JN. Human pineal physiology and functional significance of melatonin. Frontiers in Neuroendocrinol. 2004;25:177–195. 30. Srinivasan V, Spence DW, Pandi-Perumal SR, Trakht I, Cardinali DP. Jet lag: therapeutic use of melatonin and possible application of melatonin analogs.

Travel Med Infect Dis. 2008; 6:17–28. doi: 10.1016/j. tmaid.2007.12.002. Epub 2008 Jan 28. 31. Morgan K, Dixon S, Mathers N, Thompson J, Tomeny M. Psychological treatment for insomnia in the regulation of long-term hypnotic drug use. Health Technol Assess. 2004; 8:1–68. 32. Martınez MP, Miró E, Sanchez AI, et al. Cognitive-behavioral therapy for insomnia and sleep hygiene in fibromyalgia: a randomized controlled trial. J Behav Med. 2013 [Epub ahead of print]. 33. Driver HS, Taylor SR. Exercise and sleep. Sleep Med Rev. 2000;4:387–402. 34. Passos GS, Poyares D, Santana MG, et al. Effects of moderate aerobic exercise training on chronic primary insomnia. Sleep Med. 2011;12:1018–1027. 35. Reid KJ, Baron KG, Lu B, et al. Aerobic exercise improves selfreported sleep and quality of life in older adults with insomnia. Sleep Med. 2010;11:934–940. 36. Rethorst CD, Sunderajan P, Greer TL, et al. Does exercise improve self-reported sleep quality in non-remitted major depressive disorder? Psychol Med. 2013; 43:699–709. 37. Integration of behavioral and relaxation approaches into the treatment of chronic pain and insomnia. NIH Technology Assessment Panel on Integration of Behavioral and Relaxation Approaches into the Treatment of Chronic Pain and Insomnia. JAMA. 1996;276:313–318. 38. Hyman RB, Feldman HR, Harris RB, Levin RF, Malloy GB. The effects of relaxation training on clinical symptoms: a metaanalysis. Nurs Res. 1989;38: 216–220.

465

Chapter 66: Insomnia and chronic pain

39. Currie SR, Wilson KG, Pontefract AJ, deLaplante L. Cognitive-behavioral treatment of insomnia secondary to chronic pain. J Consult

466

Clin Psychol. 2000;68: 407–416. 40. Edinger JD, Wohlgemuth WK, Radtke RA, Marsh GR, Quillian

RE. Cognitive behavioral therapy for treatment of chronic primary insomnia: a randomized controlled trial. JAMA. 2001;285:1856–1864.

Section 7 Chapter

67

Special Topics

Opioid-induced constipation John Michels, Hamilton Chen, Danielle Perret Karimi, and Justin Hata

Case study A 50-year-old male with a history of severe lumbar stenosis presented to your clinic with pain intractable to NSAID therapy. After starting the patient on opioid therapy, he returns to your clinic with symptoms of straining, incomplete evacuation, and hard, dry stools.

1. What is the differential diagnosis? Opioid-induced constipation is the constellation of GI symptoms resulting from opioid use. Despite the effectiveness of opioids for pain management, opioidinduced constipation may significantly affect the quality of life for patients on opioid treatment.[1] In an opioid-naive patient presenting with constipation after starting opioids, opioid-induced constipation should be highly suspected. Other differential diagnosis of constipation may be divided into functional, structural, metabolic, neurologic, psychogenic, and drug causes (Table 67.1).[2] These causes must be considered in the evaluation of a patient presenting with constipation. Examples of functional causes of constipation include: diet, sedentary lifestyle, or motility disturbance. Examples of structural causes of constipation include: fissures, hemorrhoids, diverticulosis, or mass lesions resulting in obstruction. Examples of metabolic causes include: diabetes, hyperparathyroidism, hypercalcemia, hypokalemia, hypothyroidism, uremia, and pregnancy. Examples of neurogenic causes include: stroke, multiple sclerosis, spinal cord injury, and brain injury. Examples of psychogenic causes include: anxiety, depression, and somatization. Examples of drug causes include: opioids, anticholinergics, antidepressants, calcium channel blockers, psychotropics, diuretics, and levodopa.

Table 67.1. Differential diagnosis of constipation

Functional

Diet, motility disturbance, sedentary

Structural

Anorectal fissures, hemorrhoids, diverticulosis, mass lesions with obstruction

Metabolic

Diabetes, hyperparathyroidism, hypercalcemia, hypokalemia, hypothyroidism, uremia, pregnancy

Neurogenic

Stroke, multiple sclerosis, spinal cord injury, brain injury

Psychogenic

Anxiety, depression, somatization

Drugs

Opioids, anticholinergics, antidepressants, calcium channel blockers, psychotropics, diuretics, levodopa

Adapted from Arce DA, Ermocilla CA, Costa H. Evaluation of constipation. Am Fam Physician. 2002;65:2283–2290.

2. What is the prevalence of opioidinduced constipation? Opioids are effective and commonly used medications for both cancer pain and non-cancer pain. A largescale survey performed in multiple countries demonstrated that 19% of survey responders suffered pain for greater than 6 months and 28% of responders were treated with either a weak or strong opioid.[3] A 2005 study estimated that over 4.3 million US adults are taking opioids regularly in any given week.[4] Among all patients on opioid therapy, constipation is usually the most common adverse effect.[5] A survey conducted in the US found that opioid users had considerably less bowel movements per week compared to non-opioid users (5.3 versus 8.6).[6]

Case Studies in Pain Management, ed. Alan David Kaye and Rinoo V. Shah. Published by Cambridge University Press. © Cambridge University Press 2015.

467

Chapter 67: Opioid-induced constipation

In cancer patients, the true prevalence of opioidinduced constipation is difficult to estimate. This is due to the metabolic, physiologic, and structural changes from the disease that may further contribute to constipation. In prior studies, estimates of the prevalence of opioid-induced constipation range from 23% to 63%.[7,8] In patients with non-malignant pain, the prevalence of opioid-induced constipation is estimated at 15–90%.[9] The variation in the prevalence of opioid-induced constipation is likely secondary to patient heterogeneity, study design, and the lack of a standardized definition for constipation.[9]

3. Describe the relevant anatomy of the bowel as it relates to opioid-induced constipation The GI tract comprises multiple layers: the mucosa, submucosa, muscularis propria, and serosa. The tract is innervated by the enteric nervous system, which comprises the myenteric plexus and the submucosal plexus. The myenteric plexus is located between the smooth muscle layers of the bowel in the muscularis propria, while the submucosal plexus is located in the submucosa.[10,11] Because the myenteric plexus is located between two muscle layers and extends along the intestine, it is predominantly involved in motor activity of the gut. The submucosal plexus is predominantly involved in secretion and absorption of the GI tract. Both the myenteric plexus and the submucosal plexus contain opioid receptors that mediate their function.

4. Describe the opioid receptors in the bowel and its relation to the physiology of the bowel Opioid receptors comprise seven transmembrane Gprotein coupled receptors. The major classes of opioid receptors include mu, delta, and kappa. These receptors are made by both endogenous and exogenous mechanisms. In the central nervous system these receptors are mainly involved with pain mediation but may also mediate autonomic outflow to the gut. In the GI system, the opioid receptors mediate GI physiologic processes. When opioid agonists bind to these receptors, transmission is blocked via both presynaptic and postsynaptic sites of action.[12] This

468

Table 67.2. Effects of opioid receptors on bowel

Decreased gastric motility Inhibition of small and large intestine propulsion Increased amplitude of non-propulsive segmental contractions Constriction of sphincter of Oddi Increased anal sphincter tone Diminished secretions Increased water absorption from bowel contents

results in a number of physiologic actions, including an inhibition of peristaltic and secretory activities in the GI tract, constriction of the sphincter of Oddi and anal sphincter, and increased water absorption from bowel (Table 67.2).

5. Describe the clinical presentation of opioid-induced constipation Constipation related to opioid use is usually part of a constellation of symptoms commonly referred to as opioid-induced bowel dysfunction, which is characterized clinically by: (1) hard, dry stools; (2) straining; (3) incomplete evacuation; (4) bloating; (5) abdominal distention; and (6) increased gastric reflux.[13] These symptoms are a result of the effects of opioids on the receptors in the myenteric plexus and submucosal plexus. Severity of opioid-induced constipation may depend on the type of opioid, dosage of opioid, polypharmacy, formulation, and prior opioid exposure. Opioid-naïve patients taking high doses of oral opioids are at risk for the most severe symptoms. Among the opioids, opioid-induced constipation seems to be more common in codeine compared to other opioid medications.[14] Transdermal opioids such as fentanyl are associated with less constipation than the oral opioids.

6. What is the role of the patient’s history in determining constipation caused by opioid use? The history should begin with a detailed inquiry into the patient’s normal pattern of defecation, the frequency with which the current problem differs from the normal pattern, the perceived hardness of the

Chapter 67: Opioid-induced constipation

stools, whether the patient strains in order to defecate, and any other symptoms the patient may be experiencing. Basing the diagnosis of constipation on simply asking the patients whether they are constipated is associated with marked under-reporting of the problem in patients who have physical evidence of constipation, such as the presence of hemorrhoidal disease. A constipated patient may be otherwise totally asymptomatic or may complain of one or more of the following: 1. Abdominal bloating 2. Pain on defecation 3. Rectal bleeding 4. Spurious diarrhea 5. Low back pain The following may also suggest that the patient may have difficult rectal evacuation: feeling of incomplete evacuation, need for digital extraction of feces, or tenesmus. An inquiry concerning the amount of time spent on the toilet while waiting to defecate may also be of benefit. Patients should be asked to describe in detail what happens when they try to defecate and what maneuvers (pharmacologic or physical) they have used to facilitate this process.

7. What is the benefit of the physical examination in a patient with opioidinduced constipation? General physical examination is often of no benefit in deciding on treatment. Evaluation for other causes of constipation may be warranted and can be assessed through general evaluation of the abdomen, pelvis, and rectum if clinically warranted. The anorectal examination may be used to evaluate the effects of the patient’s constipation. The presence of fissures and the nature of the patient’s hemorrhoidal columns should be noted and characterized.

8. Is there a role for imaging in evaluating opioid-induced constipation? There is a small role for imaging in evaluation of constipation secondary to opioid use. Imaging studies

are used to rule out acute processes that may be causing colonic ileus or to evaluate causes of chronic constipation. In patients with acute abdominal pain, fever, leukocytosis, or other symptoms suggesting possible systemic or intra-abdominal processes, imaging studies are used to rule out sources of sepsis or intra-abdominal problems. Lower gastrointestinal (GI) endoscopy, colonic transit study, defecography, anorectal manometry, surface anal electromyography (EMG), and balloon expulsion may be used in the evaluation of constipation. Consider sigmoidoscopy or colonoscopy for colorectal cancer screening in patients older than 50 years. Barium enema may be used for patients who fail colonoscopy.

9. How important is fluid intake for treatment of opioid-induced constipation? Fluid intake is one of the keys to treatment. Patients should be advised to drink at least eight glasses of water daily. Counseling may be required to achieve this goal. Milk and milk products should be minimized if these prove constipating. In some patient populations, coffee, tea, and alcohol account for the majority of the fluid volume consumed. Patients should be made to understand that because of the diuretic effects of these products, this may prove to be counterproductive. Patients should decrease their consumption of coffee, tea, and alcohol as much as possible and should consume an extra glass of water for every drink of coffee, tea, or alcohol.

10. What is the roll of diet and fiber? Fiber may be an inexpensive option to assist in longterm management of opioid-induced constipation. It is important to convey to patients that bulk-forming agents generally do not work rapidly and must be used on a long-term basis. Additionally it is important to convey that large bulky stool volumes due to fiber intake, in the setting of opioid agonism, without stimulant agents on board, may result in difficult fecal passage. Dietary fiber is available in diverse natural sources, such as fruits, vegetables, and cereals. Ingestion of natural fiber sources is nutritionally superior to supplementation with purified fiber. However, advising patients to eat more fruit and vegetables is frequently unsuccessful. Patients do respond

469

Chapter 67: Opioid-induced constipation

reasonably well to prescriptions and often seek them; accordingly, prescribing a fiber supplement, such as wheat, psyllium, or methylcellulose, is often useful. Fruits that typically assist in the treatment of constipation include the “P” fruits: pears, peaches, papaya, pineapple, and prunes as well as mangoes (“Ps make a man-go”).

11. Are fiber-based laxatives safe for patient’s with fecal impaction? Fiber-based laxatives may be dangerous in those with fecal impaction and may actually cause impaction, especially without adequate water intake.

12. Is exercise effective? Although some controversy exists about the effectiveness of exercise in constipation treatment, encouraging as much aerobic exercise as possible seems reasonable. Even brisk walking may help stimulate bowel motility and, certainly, is unlikely to hurt most patients. For less mobile patients, upright positioning and use of gravity may assist in constipation treatment. Vaginal sexual intercourse may assist in the treatment of female patients with constipation.

13. Why are stimulants effective in opioid-induced constipation? Stimulants are typically the most effective pharmacotherapeutic agents for the treatment of opioidinduced constipation. Sennosides and bisacodyl counteract the effects of opioid-induced impairment of intestinal motility by inducing defecation through the stimulation of colonic peristalsis. Sennosides act directly on the intestinal mucosa or nerve plexus, which stimulates peristaltic activity, increasing intestinal motility. Senna usually produces its action 8–12 hours after administration. Senna typically comes in 8.6 mg tablets. Starting dose is two tablets taken at bedtime, not to exceed four tablets twice a day. Bisacodyl stimulates peristalsis by possibly stimulating the colonic intramural neuronal plexus. It alters water and electrolyte secretion, resulting in net intestinal fluid accumulation and laxation. It provokes defecation within 24 hours and may cause abdominal cramping. Patients should continue taking stimulant laxatives unless diarrhea occurs. If diarrhea does become

470

a problem, the dosage of laxative or softener is decreased until the patient is comfortable.

14. What is the mechanism of action of docusate and is it effective for opiateinduced constipation? Docusate is the stool softener most widely used in palliative care. It acts to increase secretions in the gastrointestinal tract, as well as absorption of these secretions by hard stool. Unfortunately, the drug has very little utility when given alone for opioid-induced constipation. Because gastric motility is decreased in these patients, softening of the stool may not alleviate the symptom unless prescribed in combination with a stimulant.

15. What is the mechanism of action of osmotic laxatives such as lactulose and sorbitol? They produce an osmotic-directed influx of fluid into the small intestine, thus increasing peristalsis as well as softening stool. Further peristalsis may be augmented by the breakdown of these sugars, which may lower intestinal pH. Both agents may be given rectally in similar dosages.

16. What is the mechanism of action of magnesium sulfate? Magnesium sulfate causes osmotic retention of fluid, which distends the colon and increases peristaltic activity; it promotes emptying of the bowel.

17. Should suppositories and enemas be considered as first-line treatment modalities? Suppositories and enemas are alternatives to oral therapy for palliative care. They may be uncomfortable and embarrassing for some patients and should not be first-line agents. Enemas are effective, and several are readily available in premade kits. In the setting of fecal impaction, enemas should be given before oral laxative agents (“pull before you push”) in order to avoid emesis. An aggressive oral laxative regimen can be started after the first successful bowel movement.

Chapter 67: Opioid-induced constipation

When enemas are necessary, the following agents are available:  Saline and tap water enemas, which draw water into the colon. (Tap water enemas should be given with caution in all patient populations due to the risk of fluid and electrolyte abnormalities, which may result with continued application.)  Soap sud enemas, which not only draw water into the colon, but help lubricate the stool.  Mineral oil enemas, which moisten and soften the hardened stool.  Emollient enemas, which contain substances that soften the stool. Examples are Microenema and Colace.  Phosphate enemas, which cause contractions in the muscles of the colon. Examples are Fleet phosphosoda.

18. Is there a role for opioid receptor antagonists in the treatment of opioidinduced constipation? Naloxone and nalmefene are pure opioid antagonists that may relieve constipation in individuals suffering from opioid-induced constipation. These agents are thought to exert their effect on local opioid receptors in the gastrointestinal tract. The exact mechanism of action is theorized to reside within the myenteric plexus, where most gastrointestinal opioid side effects originate. Dosages approaching 24 mg/day may cause significant systemic absorption, and pain crisis and opioid withdrawal may begin to manifest in individuals maintained on opioid medications. Because of the risk of systemic opioid reversal, as well as cost considerations, naloxone is infrequently prescribed for this indication. Nalmefene, a derivative of naltrexone, was assessed for its potential role in the treatment of constipation induced by these analgesics. Although promising due to perceived selectivity for the gastrointestinal tract, a high rate of withdrawal symptoms was reported, and it was concluded that the active glucuronide form of the drug was not selective enough for the gastrointestinal tract to be administered for this purpose. Methylnaltrexone (Relistor) and other PAMORA agents, known as peripherally acting mu opioid receptor antagonists, act to reverse constipation without

affecting analgesia or precipitating withdrawals. It cannot cross the blood–brain barrier, and so has antagonist effects throughout the body, counteracting effects such as constipation, but without affecting opioid effects in the brain such as analgesia. The use of methylnaltrexone may be limited by expense and is used when other laxatives have not worked. It is injected under the skin every other day as needed. Two other agents have also been developed in this class. Alvimopan (Entereg) was approved for shortterm, in-hospital use in 2008 and with expanded indications in 2013 to include accelerating the time to upper and lower gastrointestinal recovery following surgeries that include partial bowel resection with primary anastomosis with a maximum dosing of 7 days. Naloxegol (Movantik) is a pegylated derivative of naloxone with low permeability and limited central nervous system distribution and application was approved by the FDA in 2013 for the treatment of opioid-induced constipation for patients with inadequate response to laxatives.

19. What is opioid rotation? Opioid rotation is a method of reducing constipation through either use of an alternative opioid or administration of the same opioid via a different route. Opioids may have a direct local effect on constipation, and parenteral or transdermal administration (versus oral administration) may partially alleviate these symptoms. Patients who experience increasing constipation while maintaining a stable opioid regimen may benefit from switching to a different opioid. Because opioid cross-tolerance is incomplete, switching to a lower equianalgesic dosage of a different opioid may allow for continued analgesia with the potential for a lower likelihood of adverse effects. Specific opioids, such as oxycodone and fentanyl, are associated with decreased nausea and constipation, respectively.

Conclusion Opioid-induced constipation is bothersome yet often manageable in patients receiving palliative care. Although it is often considered imprudent to treat the side effects of a drug with additional drugs, it is appropriate practice considering the desirability of adequate pain relief and symptom management. When an opioid is to be started, a thorough history of exposure to the agent, side effects experienced, as

471

Chapter 67: Opioid-induced constipation

well as baseline bowel habits must be performed. By anticipating side effects and having a sound understanding of the pathophysiologic mechanisms by which opioids may cause constipation, the practitioner may confidently select the most appropriate agents to alleviate these symptoms based on the specific drug’s known mechanism of action.

References 1.

2.

3.

4.

5.

472

Klepstad P, Borchgrevink PC, Kaasa S. Effects on cancer patients’ health-related quality of life after the start of morphine therapy. J Pain Symptom Manage. 2000;20:19–26. Arce DA, Ermocilla CA, Costa H. Evaluation of constipation. Am Fam Physician. 2002; 65: 2283–2290. Breivik H, Collett B, Ventafridda V, Cohen R, Gallacher D. Survey of chronic pain in Europe: prevalence, impact on daily life, and treatment. Eur J Pain. 2006; 10(4):287–333. Parsells Kelly J, Cook SF, Kaufman DW, et al. Prevalence and charateristics of opioid use in the US adult population. Pain. 2008;138(3):507–513. McNicol E, Horowicz-Mehler N, Fisk R, et al. Management of

Constipation should be managed systematically with a planned schedule of interventions before starting an opioid. The protocol may be site or practitioner specific and should be adhered to judiciously. A heightened awareness of side effects when managing pain with opioids is more likely to improve the quality of treatment that patients receive.

opioid side effects in cancerrelated and chronic noncancer pain: a systematic review. Pain. 2003;4(5):231–256. 6.

Pappagallo M. Incidence, prevalence, and management of opioid bowel dysfunction. Am J Surg. 2001;182:11S–18S.

7.

McMillan SC. Assessing and managing opiate-induced constipation in adults with cancer. Cancer Control. 2004;11:3–9.

8.

9.

Meuser T, Pietuck C, Radbruch L, et al. Symptoms during cancer pain treatment following WHOguidelines: a longitudinal followup study of symptom prevalence, severity and etiology. Pain. 2001;93(3):247–257. Panchal SJ, Muller-Schwefe P, Wurzelman JI. Opioid-induced bowel dysfunction: prevalence, pathophysiology, and burden. Int

J Clin Pract. 2007;61(7): 1181–1187. 10. Sternini C, Patierno S, Selmer IS, Kirchgessner A. The opioid system in the gastrointestinal tract. Neurogastroenterol Motil. 2004;16(Suppl 2):3–16. 11. Holzer P. Treatment of opioidinduced gut dysfunction. Expert Opin Investig Drugs. 2007;16: 181–194. 12. Holzer P. Opioid receptors in the GI tract. Regul Pept. 2009; 155(1–3):11–17. 13. Walsh TD. Prevention of opioid side effects. J Pain Symptom Manage. 1990;5:362–367. 14. Ahmedzai S, Brooks D. Transdermal fentanyl versus sustained-release oral morphine in cancer pain: preference, efficacy, and quality of life. J Pain Symptom Manage. 1997;13:254–261.

Section 7 Chapter

68

Special Topics

Complications: vasovagal response during pain procedures Frank J. E. Falco and Nomen Azeem

Case study A 53-year-old man presents with a history of chronic right-sided neck pain with radicular symptoms of his upper extremity which began following a motor vehicle accident several years ago. Diagnostic testing includes a cervical MRI which reveals a right paracentral disc herniation at the C6–C7 level with foraminal narrowing and encroachment on the right C7 nerve root. An electrodiagnostic evaluation consisting of needle electromyography (EMG) and nerve conduction study (NCS) revealed a chronic right C7 and C8 motor radiculopathy. He has failed conservative treatment including non-steroidal anti-inflammatory drugs (NSAIDs), opioid pain medications, and chiropractic care. He has also completed 3 months of physical therapy with minimal improvement in his pain symptoms. A cervical epidural steroid injection with fluoroscopic guidance was scheduled and he was instructed to discontinue all NSAIDs for 7 days prior to his procedure. He is also instructed to discontinue all oral consumption of fluids and food at midnight the night prior to the procedure. On the day of the procedure, his vital signs are recorded within normal limits and he has intravenous access placed in his right arm. He is then prepped for a cervical epidural steroid injection. The procedure is initiated without complication. Shortly, following the initial insertion of a 25-gauge needle to anesthetize the entry point of the spinal needle the patient becomes hypotensive, bradycardic, and lethargic. Although he is able to respond to commands, he becomes disoriented and subsequently has a syncopal episode. It is determined that he has experienced a vasovagal episode during the interventional procedure. All needles are withdrawn and the patient is administered IV atropine and regains consciousness. The patient is transported to the post-procedure care

area where his vital signs are monitored over the course of the next 60 minutes. He is discharged once his vital signs are within normal limits and he is alert.

1. What is the differential diagnosis? a. b. c. d. e. f. g. h. i. j. k. l.

Hypovolemia Hypoglycemia Myocardial infarction Cardiac arrhythmia Cerebral vascular accident Thyroid dysfunction Endocrine abnormality Seizure Electrolyte imbalance Allergic reaction Intravascular injection Spinal block

2. What is a vasovagal response? A vasovagal episode also known as a vasovagal response is a malaise when a patient is exposed to a specific trigger that is mediated by the vagus nerve of the parasympathetic nervous system. It can lead to a number of symptoms including light headedness, pallor, nausea, sweating, confusion, weakness, tinnitus, tunnel vision, warmness, and blurry vision. These symptoms may be transient in nature but at times may lead to syncope. Vasovagal episodes can occur in a variety of situations and settings. This response can be a danger to the patient in the setting of interventional pain procedures. Vasovagal reactions can be disturbing to patients and staff alike and at a minimum disrupt procedures and patient flow in a busy practice.

Case Studies in Pain Management, ed. Alan David Kaye and Rinoo V. Shah. Published by Cambridge University Press. © Cambridge University Press 2015.

473

Chapter 68: Complications: vasovagal response during pain procedures

Further, the rate of vasovagal reactions can influence decisions regarding the risk to benefit ratio of conscious sedation during neuraxial injections. The most commonly known triggers for vasovagal reactions include painful or unpleasant stimuli, stress, prolonged standing, and dehydration. These triggers are often associated with interventional procedures.

3. How does a vasovagal response occur? The pathophysiology of vasovagal episodes involves the parasympathetic nervous system. Direct hypothalamic activation of the medullary cardiovascular centers triggered by emotional stress or pain can stimulate a vasovagal response. The nucleus tractus solitarius of the brainstem is activated directly or indirectly by a triggering stimulus resulting in simultaneous enhancement of the parasympathetic system and decrease of the sympathetic nervous system. This paradoxical tone leads to a cardioinhibitory response, characterized by a drop in heart rate (negative chronotropic effect) and in contractility (negative inotropic effect) leading to a decrease in cardiac output that is significant enough to result in a loss of consciousness. The reduced sympathetic activity causes a relaxation of arterial resistance which allows less venous return further decreasing cardiac output and blood delivery to the brain.

4. What are the risk factors/causative factors for a vasovagal response during pain procedures? Vasovagal episodes usually occur as a response to a trigger that can be physical or psychological. If these triggers develop immediately prior to the initiation of a procedure or during the procedure a vasovagal response may occur. Common triggers include orthostasis, hunger, dehydration, nausea, vomiting, hyperthermia, abdominal straining, urination, defecation, intense laughter, stress, extreme emotion, carotid massage, fear, and pain.

5. What is the association of vasovagal response with pain procedures? During an interventional pain procedure the fear or sensation of pain are the primary triggers for a

474

vasovagal reaction. Injection phobia, a persistent irrational fear of needles and injections, belongs to the category of simple phobias. It is also classified as a form of blood injury phobia which relates to anxiety when viewing blood, wounds, or deformities. One reason for this classification is that patients with an injection phobia may show a vasovagal reaction during the injection. This reaction, consisting of initial tachycardia followed by bradycardia, hypotension, shock, vertigo, syncope, diaphoresis, and nausea, is one of the key characteristics of blood injury phobia. Injection phobics who show a vasovagal reaction also tend to react in a passive, non-resisting way during the injection. Although, injection phobia can play a role, the cause of vasovagal responses during pain procedures might be multifactorial. According to Deacon et al, results indicated that only a small minority of patients experienced significant anxiety symptoms during venipunctures. Further, incidence of vasovagal reactions and vasovagal syncope was extremely low. Only 7 of 3315 patients reported losing consciousness during the procedure. In describing their previous experiences with blood draws and injections, most patients indicated that they experienced only minimal pain, disgust, fear of fainting, and concerns that needles might pose a health hazard.

6. In which type of pain procedures are vasovagal responses more common? Although the general phobia of needles and injections have caused vasovagal episodes in a variety of pain procedures, there are certain procedures that are known to have a higher association. In 2008, Trentman et al reported vasovagal reactions during interlaminar cervical epidural steroid injections might be much more frequent than previously reported and more common than during lumbar epidural steroid injections. In the study of 249 patients there was an additional vasovagal reaction for every 14 patients who were treated with cervical epidural injections in comparison with those treated with lumbar epidural injections. Thus, a higher level of vigilance might be warranted for patients undergoing interlaminar cervical epidural steroid injections. In a study done by Botwin et al, 1.7% of a sample size of 345 patients experienced vasovagal reaction during cervical interlaminar

Chapter 68: Complications: vasovagal response during pain procedures

epidural steroid injections. According to Gruber et al, out of 257 patients that underwent caudal epidural steroid injections there was a 0.8% incidence of vasovagal episodes.

7. How do you diagnose a vasovagal response? The diagnosis of a vasovagal response during a pain procedure must begin by assessing the vital signs during an injection procedure. Once the procedure has been terminated and the patient is stable, one must begin by ruling out all other causes for their symptoms. This can include hypoglycemia, dehydration, electrolyte imbalance, etc. The patient should have blood drawn for assessment of electrolytes, hydration assessment, glucose level, liver function, and thyroid function. An electrocardiogram or a 24hour holter monitor might be done to assess the heart. A tilt table test can be used to reproduce a vasovagal response.

References 1.

2.

3.

4.

5.

Trentman TL, Rosenfeld DM, Seamans DP, Hentz JG, Stanek JP. Vasovagal reactions and other complications of cervical vs. lumbar translaminar epidural steroid injections. Pain Practice. 2009;9:59–64. Botwin KP, Castellanos R, Rao S, et al. Complications of fluoroscopically guided interlaminar cervical epidural injections. Arch Phys Med Rehabil. 2003;84:1568–1589. Deacon B, Abramowitz J. Fear of needles and vasovagal reactions among phlebotomy patients. Anxiety Disorders. 2006;20: 946–960. Botwin KP, Gruber RD, Bouchlas CG, et al. Complications of fluoroscopically guided caudal epidural injections. Am J Phys Med Rehabil. 2001;80:416–424. American Psychiatric Association (APA). Diagnostic and Statistical

8. How would you treat a patient for a vasovagal response during a pain procedure? The optimal method to treat a vasovagal response is to prevent a vasovagal response by avoiding triggers once they are identified. In the case of pain procedures one must be mindful that vasovagal responses can lead to syncope and are usually self-limited. Unfortunately, it is impossible for a clinician to immediately determine the etiology or severity of a syncopal episode during a procedure. Relaxation techniques can be employed prior to a pain procedure for the psychogenic component which may contribute to a vasovagal response. If a patient develops a non-self-limiting vasovagal response during a pain procedure, first withdraw all instruments and needles that might be penetrating the body of the patient. Next, it is important to restore cerebral perfusion by administering intravenous fluids, pressor medications, and the use of atropine to counteract the bradycardia and increase cardiac output.

Manual of Mental Disorders, 3rd edn, revised (DSM III-R). Washington: APA. 1987.

11. Ellinwood EH, Hamilton GJ. Case report of a needle phobia. J Fam Pract. 1991;32:420–422.

6.

Daniels LK. Rapid in-office and in-vivo desensitization of an injection phobia utilizing hypnosis. Am J Clin Hypn. 1976;18:200–203.

7.

Marks I. Blood-injury phobia: a review. Am J Psychiatry. 1988;145:1207–1213.

12. Ferguson JM, Taylor CB, Wermuth B. Brief communication: a rapid behavioral treatment for needle phobics. J Nerv Ment Dis. 1978;166:294–298.

8.

Ost L-G. Blood and injection phobia: background and cognitive, physiological, and behavioral variables. J Abnorm Psychol. 1992;101:68–74.

13. De L Horne DJ, McCormack H. Behavioral psychotherapy for a blood and needle phobic mastectomy patient receiving adjuvant chemo-therapy. Behav Psychother. 1984;12: 342–348.

9.

Kleinknecht RA, Lenz J, Ford G, DeBerard S. Types and correlates of blood/injury-related vasovagal syncope. Behav Res Ther. 1990;28:289–295.

14. Hsu LKG. Novel symptom emergence after behaviour therapy in a case of hypodermic injection phobia. Am J Psychiatry. 1978;135:238–239.

10. Ost L-G, Sterner U, Lindahl I-L. Physiological responses in blood phobics. Behav Res Ther. 1984;22:109–117.

15. Kavanagh DJ, Knight DA, Ponzio V. In vivo practice for needle phobia: report on two cases. Behav Change. 1986;3:63–69.

475

Section 7 Chapter

69

Special Topics

Acute pain management: patient-controlled analgesia Nyla Azam and Devin Peck

Case study A 67-year-old female with a history of AML who recently completed chemotherapy with fludaribine, melphalan, and alemtuzumab now has oral mucositis with severe oral pain. The patient is unable to take anything by mouth and is receiving nutrition via total parenteral nutrition (TPN). She has been receiving IV morphine boluses, 2 mg every 4 hours, with initial pain relief that does not last until her next dose. Your service has been consulted to assess whether she would be a candidate for a PCA.

1. What is patient-controlled analgesia? Patient-controlled analgesia (PCA) is a method for controlling pain in which the patient is able to selfadminister pain medications – often opioids – via activation of a mechanical distribution system. PCA can be administered via any route of delivery, such as intravenous, oral, epidural, or via peripheral nerve block catheter. A landmark study done in 1973 by Marks and Sachar revealed that most inpatients had inadequate pain control, due largely to the mode of administration and delivery system of analgesics rather than the type of analgesics.[1] Roe demonstrated in 1963 that frequent small doses of IV opioids led to better pain control than the conventional IM opioid dosing.[2] Sechzer evaluated the response to frequent small doses of IV opioids self-administered via machine – more practical than frequent administrations by a nurse.[3] Studies evaluating the pharmacologic principles behind IV PCAs established that two prerequisites existed for effective analgesia: doses should be individualized and titrated to establish minimum

effective analgesic concentration, and a constant plasma opioid concentration must be maintained to avoid peaks and troughs.[4]

2. How does an IV PCA work? Modern PCA techniques use a microprocessorcontrolled infusion pump with an activation button used by the patient to self-administer analgesic medications. Basic PCA settings consist of an initial loading dose, demand dose, lockout interval, infusion rate, and an optional 1-hour and/or 4-hour limit. The initial loading dose can be administered by a healthcare provider to obtain initial analgesia. The pump can then be programmed to administer a demand dose – a fixed amount given intermittently – with time lockouts to prevent overdosing. During the lockout interval, the patient cannot receive another demand dose. The pump can also be programmed to include a continuous infusion rate, which is administered by the machine regardless of whether the patient activates a demand dose. The continuous rate can therefore be administered in conjunction with demand doses. While PCAs can be programmed to have a 1-hour or 4-hour dose limit, debate exists as to whether this improves patient safety.[4]

3. Why is an IV PCA beneficial? Patients can self-titrate pain medication and avoid the delay that accompanies requesting pain medication from their nurse. Because analgesics are selfadministered at smaller doses more frequently, patients are able to maintain a narrower range of plasma drug concentration. By maintaining steadier opioid levels it is possible to avoid the subtherapeutic troughs and excessive peak plasma concentrations seen with conventional bolus dosing. This can help

Case Studies in Pain Management, ed. Alan David Kaye and Rinoo V. Shah. Published by Cambridge University Press. © Cambridge University Press 2015.

476

Analgesia Sedation

Chapter 69: Acute pain management: patient-controlled analgesia

Minimal effective analgesic concentration

6AM

Figure 69.1. PCA administration chart. Reproduced with permission from Ferrante FM and Covino B. Patient Controlled Analgesia. Wiley Blackwell. 1990.

Pain

Opioid Concentration

PCA dose Conventional IM Dose

8AM

10AM

Noon

2PM

Time

avoid episodes of pain and the potential side effects of sedation and respiratory depression.[4,5] Systematic reviews have shown that PCAs offer better analgesic efficacy and that patients report greater satisfaction with an IV PCA compared to conventional dosing. These reviews also demonstrate that there is no evidence supporting a difference in the side effect profile, although other studies have shown that total opioid consumption is less with IV PCA compared to conventional dosing.[4,6] Patient acceptance of the technique is high, due in part to a sense of control over their own pain relief, reduction in the delay for receipt of pain medications, not receiving injections, and not having to interrupt or bother nurses.[6] An additional benefit of IV PCA is that it continuously collects data regarding patient utilization. When paired with flowsheet data on pain scores, this can be an invaluable tool in assessing a patient’s opioid requirements over time, and in designing an optimal oral regimen if desired (Figure 69.1).

4. What are some patient selection criteria for appropriate use of an IV PCA? The key element of a PCA is that the patient is in control of their analgesia. Respiratory depression is preceded by sedation, and a sedated patient is unable or unlikely to push their PCA button. Patients must therefore be cooperative and able to press their PCA button, limiting the PCA in patients younger than

3 to 5 years of age and those with some mental or physical handicaps.[5] Initiation of a PCA is often most appropriate in patients requiring frequent PRN dosing of medications, or when such dosing is anticipated (i.e., in the postoperative period or during a sickle cell pain crisis).

5. When would a continuous infusion on an IV PCA be appropriate? A continuous infusion rate theoretically allows for a constant opioid level and better analgesia, and may be appropriate for those with high baseline opioid tolerance and requirements. Routine use of a basal infusion rate for opioid-naive patients is rarely advisable. There may be an increase in total opioid use, and with that an increase in side effects.[6] In a study by Parker, the addition of a “nighttime only” continuous infusion of morphine to an existing PCA did not improve sleep or pain scores following abdominal hysterectomy in opioid-naive patients.[7] The presence of a continuous infusion removes, to a degree, a patient’s sense of control over their pain management. As such, they can perceive such therapy to be less effective. In the study by Parker, most programming errors that resulted in adverse side effects took place when a continuous infusion was part of the PCA regimen.[7] Basal rates bypass the safety mechanism of patient control, and can place the patient at higher risk for respiratory depression and sedation. Routine use of a basal infusion rate is

477

Chapter 69: Acute pain management: patient-controlled analgesia

therefore not recommended and should only be used in carefully selected patients.

therefore clinically insignificant effects on the neonate.[9]

6. Whatis theideal opioidforanIV PCA?

7. How do I initially program a PCA in my opioid-naive patients?

The ideal opioids for IV PCA use have a rapid onset, high efficacy, and intermediate duration of action without significant accumulation of active metabolites. The opiates most commonly used are morphine, hydromorphone and fentanyl.[5] All common opioids have been used, with morphine being the most studied.[4] PCA success may depend less on the type of opioid used, and more on how the PCA pump is programmed.[8] The decision regarding the choice of opioid should depend also on the effect of the metabolites of that opiate.[6] Morphine has an active metabolite, morphine-6glucuronide (M6G), which produces analgesia, sedation, and respiratory depression. M6G is eliminated renally, and patients with renal failure receiving morphine have been reported to have profound respiratory depression. It is recommended to avoid IV morphine PCAs for patients with serum creatinine > 2.0 mg/dL. Hydromorphone is hepatically metabolized and can be used as an alternative in morphineintolerant or renally impaired patients. Due to its lipophilic nature, fentanyl has a more rapid onset of action compared to morphine, and is well suited for use with a PCA. It is also a good alternative for patients with renal impairment as the drug is not renally metabolized.[4] Meperidine was historically the second most commonly used opioid for IV PCA use, but is no longer recommended because of its neurotoxic metabolite, normeperidine. Normeperidine accumulation can lead to anxiety, tremors, and grand mal seizures with potential loss of airway and anoxic brain injury or death. There is pharmacodynamic variability in patient response to meperidine, making dose titration difficult. Meperidine is contraindicated in patients with renal impairment, seizure disorder, and on monoamine oxidase inhibitors due to potentially lethal malignant hyperpyrexia.[4] The short-acting opioids sufentanil, alfentanil, and remifental have also been used for IV PCAs, with sufentanil being the most studied. When using sufentanil, due to its short-acting nature, a continuous infusion may be necessary.[4] Remifentanil is most useful in labor pain due to its ultra-short duration of action and

478

A PCA can be initiated with a starting bolus dose, a demand dose of morphine 1 mg or equianalgesic hydromorphone 0.2 mg, a 6–10 minute lockout interval with an optional 1-hour or 4-hour lockout. PCA should be viewed as maintenance therapy, and so pain should be controlled before the PCA is started.[6] The key to successful PCA management is establishing initial analgesia, which can be done with boluses of medication prior to starting the PCA pump. Pain medications can be titrated to a pain score of < 4/10 or RR < 12 breaths/minute.[4] The lockout time should be long enough for the patient to feel the full effect of the demand dose. If they are too long, the PCA may not be effective.[6]

8. How do patient characteristics such as age and weight affect IV PCA use? According to a study done by Macintyre and Jarvis, the best predictor of total IV PCA opioid use was patient age, with a formula for the average amount of morphine used in the first postoperative 24 hours being 100 age (in mg).[10] Elderly patients have lower opioid requirements and need smaller demand doses.[6] Although total dose is less for older patients, titration regimens are similar to those used in younger patients.[4] A study by Egbert and coworkers found that PCA use in elderly men led to better pain scores, less confusion, and fewer pulmonary complications, with the incidence of confusion being 2.3% in the PCA group and 18% in the conventional dosing group.[11] PCA use does require intact cognition, and so elderly patients with dementia and postoperative delirium are not suitable candidates.[6] Weight and gender were found to have no effect on the total opioid doses required.[4]

9. Whatifthepatientcontinuestohave pain while on the PCA? If your patient continues to have pain scores > 4/10 while on the PCA, first assess whether the patient is using the PCA and the demand dose activation button effectively. Often, patients will need re-education on

Chapter 69: Acute pain management: patient-controlled analgesia

how to use the PCA.[4] Potential negative patient perceptions of PCAs can also result in limited patient use. Some of those perceptions that have been identified include patients not trusting the PCA machine or fearing overdose or addiction. These perceptions, and overall pain scores, were higher in patients who did not receive education regarding PCA use.[6] Occasionally the activation button becomes nonfunctional and may need to be replaced. If the patient is receiving their demand doses and continues to have pain, then administer a bolus and increase the demand dose. Keep in mind side effects from PCA use. If the patient is experiencing nausea or drowsiness, they may not want a higher demand dose.[4] For an opioid-naive patient, one should consider a continuous infusion rate only if the PCA demand dose has been increased for at least 4 hours. Continuous infusion rates are rarely needed, however, and substantially increase the risk for respiratory depression. If a continuous rate is used, as a general rule of thumb, it should constitute less than 50% of the total opioid dose. For opioid-tolerant patients, such as those on chronic opioid therapy, especially sustained-release opioids, or those with cancer pain, the continuous infusion rate should make up about 80% of the total opioid requirement, with large demand doses with long lockout times for breakthrough pain.[4]

10. What are the side effects of an IV PCA? Conflicting data has been produced regarding the amount of opioids used via IV PCA compared to conventional intermittent dosing.[6] The common side effects are those seen with any opioid medications, and include nausea, vomiting, pruritis, sedation, confusion, and respiratory depression.[4] The most concerning side effect is respiratory depression, which is seen more commonly in elderly patients, those with sleep apnea, patients on sedatives or additional opioids, and those with hypovolemia.[6] The most common and most bothersome side effect is nausea and vomiting. Adding droperidol directly to the IV PCA mixture has been studied and shown to be significantly more effective than placebo in decreasing the incidence of nausea and vomiting. The optimal droperidol dose is 15–100 μg for every 1 mg of morphine, with the incidence of side effects of

sedation and dysphoria increased with doses > 4 mg per day.[4] However, in 2003 the FDA issued a black box warning regarding QTc prolongation with the use of droperidol. Therefore it has become a less frequently-used option for anti-emesis. Adding antiemetics to the PCA mixture has been controversial. A study done by Gan showed that administering droperidol was as effective as adding it to the PCA bag.[12] Small doses of pure opioid antagonists, such as nalmefene, also significantly decreased the incidence of nausea and vomiting without affecting analgesia levels.[4] Another commonly reported side effect is pruritis. While the mechanism of this is not fully understood, it appears that opioid receptors, along with serotonin and dopamine receptors and prostaglandins, all play a role. While antihistamines are often used to treat opioid-induced pruritis, it may be more effective to use an agent specific to these mechanisms. These may include mixed opioid receptor agonist-antagonists such as nalbuphine, serotonin receptor antagonists, NSAIDs, and dopamine receptor antagonists.[13] Of these agents, nalbuphine is likely the most commonly used.

11. How safe is an IV PCA? The safety of a PCA relies on the concept that activation of the demand dose is patient-centered, and if the patient is too sedated to push their activation button, severe respiratory depression will be avoided. The incidence of respiratory depression with IV PCA use is 0.25%, whereas for intermittent bolus dosing it is 0.9%.[4] Risk factors for respiratory depression with IV PCA use include advanced age, head injury, sleep apnea syndrome, obesity, respiratory failure, concurrent sedative medication use (especially benzodiazepines), hypovolemia, and renal failure.[4] Postoperative PCAs have been used safely in morbidly obese patients with sleep apnea when no continuous rate and a lower initial bolus dose is used.[6] Family members should be educated to not press the activation button while the patient is sleeping. Programming errors or equipment malfunction can be the reason behind morbidity and even mortality related to PCA. While rare, there have been reports of deaths from IV PCA due to programming errors by the provider that resulted in overdose. Thorough training of nursing personnel is

479

Chapter 69: Acute pain management: patient-controlled analgesia

paramount. Safety precautions such as a second nurse witnessing pump programming can help to prevent such errors.[4] Continuous infusion rates bypass the safety mechanism inherent in IV PCA use, and so must be used carefully in select patients. Continuous rates, even at low doses, are more likely to cause severe respiratory depression in opioid-naive patients, with the incidence found to be comparable to that found with a continuous infusion alone in a study by Schug and Torrie.[14] The use of continuous infusions in opioid-naive patients has also shown a greater incidence of nausea, vomiting, and sedation, along with higher total opioid doses but without improved pain relief. It is therefore safest for use in opioid-tolerant patients.[4] Ultimately, IV PCA use is widely accepted, and has been found to be safer and with fewer logistical problems than both conventional bolus dosing of opioids and intraspinal or epidural analgesia.[4]

12. What are the limitations of an IV PCA? Patients can still experience the same degree of side effects as they would with conventional opioid dosing. Although patients report better analgesia, IV PCAs are usually inadequate for certain types of postsurgical pain that may be better addressed with neuraxial analgesia or peripheral nerve blocks or catheters. The IV PCA can also be a physical hindrance to the patient, impairing mobility as the patient has to ambulate with a pole and IV tubing.[4]

13. Are there ways to improve the efficacy of IV PCA use? For postoperative pain control, adjuncts to the IV PCA can be used to decrease the total opioid requirement. NSAIDs, local wound infiltration with local anesthetic, and peripheral nerve blocks used in addition to the IV PCA help greatly in achieving that goal. Adding ketamine to IV PCAs has been postulated to be helpful in reducing pain.[4] Nociceptive stimuli activate the NMDA receptor as part of the pathophysiology of acute pain. Ketamine is an NMDA receptor antagonist that acts centrally to decrease hyperalgesia. Animal studies have shown a synergistic and additive effect between opioids and ketamine on pain control.

480

Ketamine has a potential role in decreasing opioid requirements and related side effects. Several studies were performed to evaluate the effect of adding ketamine to IV PCAs, with contradictory findings, most likely due to methodologic weaknesses.[15] The optimal ratio of morphine-to-ketamine leading to the lowest pain scores with a low incidence of side effects was 1:1 with a lockout of 8 minutes.[4] Studies using a higher ratio of ketamine to morphine demonstrated no improvement in pain scores or patient satisfaction and a higher incidence of dysphoria, leading to patients dropping out of the study.[4]

14. Can a PCA mask postoperative complications? Concerns have been raised that because patients can self-administer pain medication, they can selfmedicate for new pain without informing the medical staff, and can mask potentially dangerous and untreated conditions, such as urinary retention, myocardial infarction, pulmonary embolism, or compartment syndrome. A case report was submitted in which a patient was taken to the OR for compartment syndrome 36 hours after his initial surgery for lower limb trauma, after which a PCA had been started. It was noted however that the patient was monitored for sedation and pain scale for only the first 6 hours. The authors concluded that in their case, the PCA allowed for decreased nurse-to-patient contact time, allowing for the complication to arise. They state that PCA use is acceptable if patients continue to be monitored throughout the period of use.[16] With regular hourly monitoring, PCAs may allow for detection of changes in clinical condition with unexpected increases in opioid requirements.[6]

15. How does the cost of IV PCA compare to conventional dosing? A review by Jacox and colleagues showed that PCAs led to improved pain control and better patient satisfaction at a higher cost than conventional medication dosing. A significant portion of the cost is due to the equipment, drugs, and consumables. PCAs have not been shown to significantly reduce length of stay, another important factor in patient care.[17] To place it into perspective, however, the pharmacy cost of providing postoperative analgesia was found to be about 1% of the cost of joint surgery.[18]

Chapter 69: Acute pain management: patient-controlled analgesia

References 1.

Marks RM, Sachar EJ. Undertreatment of medical inpatients with narcotic analgesics. Ann Intern Med. 1973;78:173–181.

2.

Roe BB. Are postoperative narcotics necessary? Arch Surg. 1963;87:912–915.

3.

Sechzer PH. Objective measurement of pain. Anesthesiology. 1968;29:209–210.

4.

Grass JA. Patient-controlled analgesia. Anesth Analg. 2005; 101:S44–S61.

5.

Sherwood ER, Benzon HT. Patient-controlled analgesia. In Benzon HT, Raja SN, Molloy RE, Liu SS, Fishman SM, eds. Essentials of Pain Medicine and Regional Anesthesia. Philadelphia: Elsevier Inc. 2005: pp. 235–238.

6.

Macintyre PE. Safety and efficacy of patient-controlled analgesia. Br J Anaesth. 2001;87:36–46.

7.

Parker RK, Holtmann B, White PF. Effects of a nighttime opioid infusion with PCA therapy on patient comfort and analgesic requirements after abdominal

hysterectomy. Anesthesiology. 1992;76:362–367. 8.

pruritis. Drugs. 2007;67: 2323–2333.

Mather LE, Woodhouse A. Pharmacokinetics of opioids in the context of patient controlled analgesia. Pain Rev. 1997;4:20–32. 9. Hinova A, Fernando R. Systemic remifentanil for labor analgesia. Anaesth Analg. 2009;109:1925–1929. 10. Macintyre PE, Jarvis DA. Age is the best predictor of postoperative morphine requirements. Pain. 1996;64:357–364. 11. Egbert AM, Parks LH, Short LM, Burnett ML. Randomized trial of postoperative patient-controlled analgesia vs. intramuscular narcotics in frail elderly men. Arch Intern Med. 1990;150:1897–1903.

14. Schug SA, Torrie JJ. Safety assessment of postoperative pain management by an acute pain service. Pain. 1993;55:387–391.

12. Gan TJ, Alexander R, Fennelly M, Rubin AP. Comparison of different methods of administering droperidol in patient-controlled analgesia in the prevention of postoperative nausea and vomiting. Anesth Anal. 1995;80:81–85.

17. Jacox A, Carr DB, Mahrenholz DM, Ferrell BM. Cost considerations in patientcontrolled analgesia. Pharmacoeconomics. 1997;12: 109–120.

13. Ganesh A, Maxwell LG. Pathophysiology and management of opioid-induced

15. Carstensen M, Moller AM. Adding ketamine to morphine for intravenous patient-controlled analgesia for acute postoperative pain: a qualitative review of randomized trials. Br J Anesth. 2010;4:401–406. 16. Harrington P, Bunola J, Jennings AJ, Bush DJ, Smith RM. Acute compartment syndrome masked by intravenous morphine from a patient-controlled analgesia pump. Injury. 2000;31:387–389.

18. Macario A, McCoy M. The pharmacy cost of delivering postoperative analgesia to patients undergoing joint replacement surgery. J Pain. 2003;4:22–28.

481

Section 7 Chapter

70

Special Topics

Acute pain management: PCEA/continuous epidural catheters Qiao Guo, Minyi Tan, and Devin Peck

Case study A 63-year-old female with a history of multiple sclerosis is diagnosed with non-small cell lung cancer in the right upper lobe. She is scheduled for lobectomy and presents to the preoperative anesthesia clinic for postoperative analgesic management consultation.

1. What are the indications for placing an epidural catheter for postoperative analgesia? Epidural analgesia plays an important role in providing pain relief for surgical and obstetric patients. The most common indication for epidural catheter placement is labor. High lumbar and thoracic epidural catheter placements have gained increasing popularity in recent years for management of postoperative pain. The technique is most commonly employed for procedures in which a thoracic or an extensive abdominal incision is anticipated. The threshold for placement may be lower in patients of advanced age, those with pulmonary disease, those with comorbidities which may lead to poor functional capacity, or in whom systemic opioids may not be well tolerated. Epidural analgesia may also be considered in scoliosis surgeries, where a catheter might be placed by the surgeon.

2. What postoperative pain treatment options are available for patients undergoing thoracotomy? a. Epidural catheter i. Local anesthetic ii. Opioids iii. Adjuncts (such as clonidine)

b. Systemic analgesics i. Opioids 1. Traditional PRN analgesic regimen 2. Intravenous patient-controlled analgesia ii. Non-opioids 1. 2. 3. 4.

NSAIDs Acetaminophen Ketamine Dexmedetomidine

c. Regional block i. Intercostal nerve block ii. Paravertebral block iii. Intrapleural catheter[1]

3. What are the benefits of a thoracic epiduralpatient-controlledanalgesia as an analgesic technique? There are numerous advantages to utilizing a thoracic epidural as an analgesic technique: a. Sympathetic response The sympathetic surge resulting from surgery can increase stress on the myocardium and induce hypercoagulability. Sympathetic block can attenuate these effects, thereby decreasing mortality and morbidity in the perioperative period.[2] b. Cardiovascular system Cardiac parameters are altered with the infusion of local anesthetics via a thoracic epidural catheter. The major chronotropic control of the

Case Studies in Pain Management, ed. Alan David Kaye and Rinoo V. Shah. Published by Cambridge University Press. © Cambridge University Press 2015.

482

Chapter 70: Acute pain management: PCEA/continuous epidural catheters

heart is mediated via the T1-T4 levels, and could be affected by the spread of medication from a thoracic epidural. Wattwil et al measured the heart rate of volunteers by the means of echocardiography, and showed that heart rate was decreased both at rest and during exercise with use of a thoracic epidural. This response could benefit patients with a history of cardiac disease in which reduction of heart rate and myocardial oxygen demand is desired.[3] The review by Clemente and Carli discussed multiple studies examining changes in coronary perfusion with use of a thoracic epidural. It was concluded that thoracic epidural infusions improve the myocardial oxygen supply/demand ratio without jeopardizing coronary perfusion pressures within ischemic areas. In addition, the overall incidence of myocardial infarction with a thoracic epidural was shown to be decreased.[4] c. Respiratory system With a thoracic epidural in place, intercostal nerves that supply the rib cage are expected to be impaired. This should cause minimal impairment of ventilation if the patient’s diaphragm functions normally with an intact phrenic nerve. Most patients receiving thoracic epidurals undergo thoracic or abdominal surgeries which can result in significant respiratory impairment,[4] due in part to pain with deep breathing. Multiple studies have shown that patients with thoracic epidurals have improved respiratory function postoperatively when compared with those without epidural catheters.[5,6] With a thoracic epidural in place, deep breathing and coughing becomes more comfortable. The incidence of atelectasis and pneumonia are decreased, thereby decreasing the incidence of respiratory deterioration. d. Gastrointestinal system It is well established that gastrointestinal function benefits greatly from epidural placement. One of the major concerns for any major surgery is postoperative return of gastrointestinal function. Carli et al have shown that patients who have undergone colorectal surgery with an epidural have earlier return of bowel function when compared with patients receiving intravenous patient-controlled analgesia.[7] Increased

gastrointestinal motility is predominantly caused by inhibition of the sympathetic component of the autonomic system through the infusion of local anesthetics within the epidural space.[8] Enhanced sympathetic activity due to surgical manipulation, stress, and pain can also lead to gastrointestinal hypoperfusion with resulting intestinal paralysis. It has been shown in multiple studies that thoracic epidural infusions can improve gastrointestinal perfusion.[4] e. Tumor spread Use of epidural analgesia for tumor resection and manipulation may be beneficial in limiting the extent of tumor spread. In a review published in the British Journal of Anaesthesia, it was concluded that surgery promoted systemic tumor spread and decreased patients’ immune response and ability to eliminate tumor cells. Multiple retrospective studies have concluded that thoracic epidural or paravertebral techniques can decrease the recurrence of tumor and improve survival.[2] f. Thrombotic protection The perioperative period represents a time of increased risk for deep vein thrombosis (DVT). Epidural analgesia reduces this risk. The mechanism for this reduction is not completely understood. However, it has been theorized that increased lower extremity blood flow due to sympathetic blockade contributes to this effect. Shouhed et al identified multiple studies showing a decreased incidence of DVT with the use of an epidural as the primary analgesic technique.[9]

4. How should an epidural catheter be placed? What techniques can be used for assistance with a difficult placement? Monitors should be placed prior to the procedure, and sedation can be administered for anxious patients. Patients are usually placed in a sitting position with neck and back flexed. If the patient is unable to sit up, the procedure can be performed with the patient in the lateral decubitus position, taking care to ensure adequate flexion of the spine. Most epidural catheter placements are performed via palpation of anatomic landmarks which can be used to help determine the vertebral level. The most

483

Chapter 70: Acute pain management: PCEA/continuous epidural catheters

Table 70.1. Commonly used anatomic landmarks for epidural placement

Anatomic landmark

Vertebral correlation

Most prominent cervical spinous process

C7

Inferior border of scapula

T7

Iliac crest

L4

prominent spinous process is typically C7, the inferior border of the scapula can be used to identify the T7 level, and the iliac crest is approximated at the level of L4 (see Table 70.1). The epidural space can be accessed via various routes, including the interlaminar, transforaminal, and caudal route. In order for an epidural catheter to be placed, the interlaminar approach is most commonly utilized. There are several techniques for accessing the epidural space via the interlaminar route. These include the midline, paramedian, and Taylor approaches. A traditional midline approach for thoracic epidural placement can be difficult secondary to the increased angulation of the thoracic spinous process. Therefore, the paramedian approach is often recommended (see Figure 70.1A–C). The paramedian approach can also be useful in the lower lumbar area, specifically in patients who cannot be optimally positioned due to injury or pathology, or in those with spinal deformities such as scoliosis. Ultrasound can be used as an adjunct when anticipating a difficult placement. The modality can be used to identify the midline, to map out the location of the largest interlaminar space, and to estimate the depth of the epidural space. This can be helpful in patients with abnormal anatomy or who are morbidly obese. Fluoroscopy can be used for guidance with placing an epidural at a precise level. It is especially helpful in patients with distorted anatomy to visualize the interlaminar spaces, and to verify entry into the epidural space with the injection of contrast.

5. What are some risks of an epidural placement? a. Neuraxial bleed and epidural hematoma Trauma to the epidural vein by epidural placement or catheter insertion is generally clinically insignificant in patients with normal

484

coagulation. Hence, the development of neuraxial hematoma leading to neurologic catastrophe is a rare event. The risk of neuraxial hematoma development after epidural analgesia has been estimated at 1 in 168 000.[10] However, the development of new anticoagulants and antiplatelet medication creates a new challenge in managing patients receiving epidural catheters.[11–13] The American Society of Regional Anesthesia[14] has released guidelines in response to these new medical developments. Neuraxial hematoma may present as back pain, with motor weakness, sensory deficits and possibly bowel or bladder incontinence indicating the presence of spinal cord compression. Immediate MRI is warranted to confirm the diagnosis and immediate surgical intervention may be indicated. If so, decompression within the first 12 hours results in a greater likelihood of recovery.[15] b. Infection The risk of infection from epidural placement has been reported to be 1 in 110 000.[10] This risk can be minimized with meticulous sterile technique, the use of bacterial filters, antibiotic prophylaxis, and close monitoring of the epidural site. Local infection at the planned catheter placement site is a contraindication for placement. Studies examining whether systemic or distal localized infection have increased incidence of catheter colonization have shown conflicting results. However, epidural catheterization is generally not recommended in patients with untreated bacteremia or sepsis, unless the risk outweighs the benefit.[16] An epidural abscess may present several days to months following the procedure. It can present as localized back pain with tenderness at the site, fever, and leukocytosis. Patients can develop progressive weakness and eventual paraplegia if left untreated. Staphylococcus aureus is the most common pathogen involved with development of epidural abscesses. Antibiotic treatment targeting S. aureus should be started and continued until definitive culture and antibiotic sensitivity is available. Neurosurgery consultation should be obtained if neurologic symptoms develop. c. Nerve injury Direct trauma from needle or catheter insertion can cause injury to the spinal cord, conus

Chapter 70: Acute pain management: PCEA/continuous epidural catheters

B

A

C

Figure 70.1. (A–C) Illustrations showing angulation difference between lumbar (A) and thoracic (B) spinous process, and midline vs. paramedian (C) approaches. Image courtesy of Dr. Allen Tanner.

medullaris, and nerve roots. The incidence of persistent neurologic injury is 1 in 257 000 while that of transient neurologic injury that resolves within 1 year is 1 in 4300.[10] The spinal cord ends at the L1 vertebral body in most adults, but it

can end above or below this level.[17] Hence epidural placement even below L1 can potentially cause cord injury. Performance of the procedure in an awake patient can minimize this risk. In a prospective study by Auroy et al, 5 out of 30 413

485

Chapter 70: Acute pain management: PCEA/continuous epidural catheters

epidural anesthetics caused transient radiculopathy that correlated with paresthesia or pain felt during the epidural catheter placement.[18] While pain or paresthesia can occur with injury to a nerve root, spinal cord injury may be painless due to lack of sensory innervation. If such injury is suspected, MRI should be obtained. Electromyography may take up to 3 weeks to show axonal loss and can be used to determine the site of the injury. Neurologic injury in a patient in whom an epidural catheter has been placed may be due to other causes unrelated to the catheter. For instance the femoral nerve can be damaged from retraction or injury during pelvic surgery; the lateral femoral cutaneous nerve can be damaged from retraction near the inguinal ligament or by a large pfannenstiel incision; the common peroneal nerve can be injured when compressed against the fibular head with lithotomy positioning. Management of postoperative neuropathy should involve the anesthesiologist, surgeon, and neurologist when appropriate. d. Dural puncture Inadvertent dural puncture during epidural placement can result in the development of a postdural puncture headache. If the resultant loss of cerebral spinal fluid (CSF) occurs at a rate greater than that of CSF production (about 0.3 ml/ minute), intracranial hypotension will result. With the loss of CSF, pain-sensitive structures such as the meninges are stretched when the brain is pull downward by gravity. This worsens when the patient is in the upright position. The Monro-Kellie hypothesis states that since the cranial vault is of constant volume, the loss of one of the three component substances – brain parenchyma, blood, or CSF – necessitates a compensatory increase of another. Therefore, with loss of CSF, there is a compensatory increase in cerebral blood volume. The associated cerebral vasodilation is likely another contributor to pain. Cerebral vasoconstrictors such as caffeine, theophylline, and sumatriptan may be used for conservative management along with hydration. When conservative treatment fails, epidural blood patch can be performed. Symptoms are generally relieved within minutes of the procedure.

486

With use of the loss of resistance to air technique, dural puncture may risk pneumocephalus which can result in seizures, loss of consciousness, and other neurologic sequelae.[19,20] e. Respiratory depression The use of opioids in epidural infusions can lead to respiratory depression. However, the incidence is not higher than that seen with other routes of opioid administration.[21] Hydrophilic opioids such as morphine can result in delayed respiratory depression. This results from extensive cephalad spread of the medication in the CSF. Neuraxial administration of local anesthetics results in level-dependent blockade of the abdominal and intercostal muscles of respiration. Epidural block to the mid-thoracic level has very little effect on pulmonary function in patients without pre-existing lung disease. However, blockade at this level may cause dyspnea via proprioceptive blockade of the abdominal and intercostal muscles. Patients with severe pulmonary disease, or those who depend on the use of accessory muscles for adequate respiration, may require ventilator support with such blockade. Severe hypotension resulting from a high neuraxial block can lead to reduced medullary blood flow and central apnea. There may also be impairment of diaphragmatic function by blockade of the C3-C5 levels. High thoracic blocks result in slightly decreased vital capacity due to a decrease in expiratory reserve volume related to thoracoabdominal muscle paralysis. Tidal volume remains unchanged. f. Catheter migration Intrathecal or intravenous migration of catheters can occur after placement. The use of low-dose local anesthetics and opioids in the epidural infusion can prevent the sudden onset of high spinal anesthesia. A test dose of local anesthetic with epinephrine can identify intravenous migration resulting in transient tachycardia, and intrathecal migration resulting in sensorimotor blockade. Intrathoracic placement of epidural catheters can occur with either a midline or a paramedian approach.[21] The catheter or needle can puncture the pleura during insertion. Inadvertent

Chapter 70: Acute pain management: PCEA/continuous epidural catheters

placement of a pleural catheter can lead to inadequate analgesia, pneumothorax, and hemothorax.[22] g. Failed epidural Failure of an epidural catheter to provide analgesia can result from improper identification of the epidural space. Loss of resistance is a tactile technique that can be misinterpreted in patients with challenging anatomy, such as the obese. The presence of cysts within the interspinous ligament or ligamentum flavum may lead to a false loss of resistance. Such cysts are more common in the elderly. A false loss of resistance can also result from needle entry into the paraspinal musculature.[23] With accidental pleural catheter placement, loss of resistance can be felt as the Tuohy needle passes into the pleural cavity. Even with correct identification of the epidural space during needle placement, the catheter tip may enter an intervertebral foramen. As mentioned previously, a properly placed catheter can migrate out of the epidural space at any time. Finally, catheters may become nonfunctional due to kinking or clotting.

6. What are some medications used for epidural analgesia? a. Local anesthetics Epidurally administered local anesthetics such as bupivacaine and ropivacaine may result in decreased postoperative ileus, nausea, vomiting, and sedation, which can be associated with opioids. However, these agents are less commonly used alone due to the unpredictable incidence of motor blockade, regression of sensory block, and hypotension. Opioid supplementation is often needed to provide adequate analgesia. Ropivacaine has been shown to be less cardiotoxic and neurotoxic when compared with bupivacaine. While ropivacaine results in less motor blockade, bupivacaine provides better analgesia.[24] b. Opioids Opioids modulate nociceptive input presynaptically and postsynaptically in the dorsal horn of the spinal cord. They have the advantage of providing analgesia without causing motor or sympathetic blockade. Drugs in the epidural space

must cross both a hydrophilic zone as well as a hydrophobic zone. The hydrophilic zone comprises extracellular and intracellular fluid, while the hydrophobic zone comprises the cell membrane lipids of the arachnoid layer. Hydrophilic opioids such as morphine readily cross this hydrophilic zone, but are slower to cross the arachnoid membrane. This results in a slower onset and longer duration of action as the medication dwells in the CSF. This can result in the extensive cephalad spread of morphine associated with delayed respiratory depression. Lipophilic opioids such as fentanyl and sufentanil readily cross the lipid membrane of the arachnoid mater, resulting in more rapid onset. High lipid solubility increases the rate of clearance from CSF, resulting in a shorter duration of analgesia and less extensive cephalad spread. Rapid diffusion of lipophilic opioid out of CSF is facilitated by vascular uptake. The lack of spread in CSF leads to segmental analgesia. The level of catheter placement therefore becomes increasingly important. The use of fentanyl in epidural analgesia is somewhat controversial. Several studies have demonstrated little difference in plasma levels after 24 hours of infusion, total daily fentanyl use, incidence of side effects, and quality of analgesia between epidural and IV administration.[25,26] Sufentanil, however, showed lower levels of respiratory depression and sedation with epidural administration.[27] Opioids with intermediate lipophilicity, such as alfentanil and hydromorphone, can diffuse readily between both the lipid and aqueous zones. Analgesia experienced with epidural hydromorphone is similar to morphine with less incidence of pruritus and faster onset.[28] Alfentanil is more lipophilic than morphine, but less so than hydromorphone. It has less intravascular absorption when compared to other lipophilic opioids and a shorter duration of action. Thus continuous infusion with its use is required to provide adequate analgesia. c. Local anesthetics and opioids Local anesthetics and opioids act synergistically such that a high degree of analgesia can be maintained or enhanced even with reduction of the dose of each drug. This can decrease the incidence of

487

Chapter 70: Acute pain management: PCEA/continuous epidural catheters

adverse effects associated with both medications while providing the same level of pain relief. d. Adjuvants Adjuvants such as epinephrine, clonidine, and ketamine may be added to epidural infusions to enhance analgesic efficacy. Several studies have shown that these adjuvants can be used to provide better pain relief.[29–31] However, there is concern that ketamine may be neurotoxic and that clonidine may increase the incidence of hypotension. Furthermore, there is concern about the safety of using vasoconstrictors in the epidural space.

7. What are the options for infusion strategies? Epidural analgesia can be administered by continuous infusion, or by intermittent bolus with or without background infusion. Continuous infusions are typically used in epidural analgesia. This serves to maintain a good level of pain control while minimizing the cardiovascular effects of local anesthetics, as well as the respiratory effects of opioids. Continuous rates can also reduce the degree of unscheduled clinician intervention. Sensory block may regress with continuous infusion over time and supplemental medication may be needed. The addition of an intermittent epidural bolus is associated with decreased medication consumption, decreased clinician intervention, and improved patient satisfaction.

8. What special considerations exist for the use of neuraxial analgesia in patients with multiple sclerosis? Factors such as emotional stress, hyperpyrexia, and infection are known to cause exacerbations or relapses of MS symptoms. Studies have implicated spinal anesthesia in the development of postoperative exacerbations.[32] The effect of local anesthetics on

Reference 1.

488

Slinger PD, Campos JH. Anesthesia for thoracic surgery. In Miller RD, ed. Miller’s Anesthesia, 7th ed. Philadelphia: Churchill Livingstone. 2010.

the course of the disease is unclear. Epidural anesthesia and analgesia have been used in the obstetric population and do not appear to increase the incidence of relapse.[33] However, patients with relapses had received a concentration of bupivacaine greater than 0.25%.

9. For what other procedures might epidural analgesia be beneficial? There has been an increase in the use of epidural analgesia in scoliosis surgery. The catheter can be placed under direct visualization by the surgeon. This technique has been shown to result in decreased pain scores, more rapid return of bowel function, and improved patient satisfaction.[34]

10. What are the contraindications for placing an epidural catheter? a. Absolute i. Patient refusal ii. Uncorrected hypovolemia iii. Increased ICP iv. Infection at site v. Allergy to amide/ester LA b. Relative i. Coagulopathy (platelet count < 100 000) ii. Uncooperative patient iii. Severe anatomic abnormalities of spine iv. Sepsis v. Hypertension c. Controversial i. Inadequate training/experience ii. Elaborate tattoos at the needle insertion site iii. Positioning that compromises respiratory status iv. Anesthetized patient v. Previous back surgery[35]

2.

Freise H, Van Aken HK. Risks and benefits of thoracic epidural anaesthesia. Br J Anaesth. 2011;107(6):859–868.

3.

Wattwil M, Sundberg A, Arvill A, Lennquist C. Circulatory changes

during high thoracic epidural anaesthesia: influence of sympathetic block and of systemic effect of the local anaesthetic. Acta Anaesthesiol Scand. 1985;29: 849–855.

Chapter 70: Acute pain management: PCEA/continuous epidural catheters

4.

Clemente A, Carli F. The physiological effects of thoracic epidural anesthesia and analgesia on the cardiovascular, respiratory and gastrointestinal systems. Minerva Anestesio. 2008;74: 549–563.

5.

Manikian B, Cantineau JP, Bertrand M, et al. Improvement of diaphragmatic function by a thoracic extradural block after upper abdominal surgery. Anesthesiology. 1988;68: 379–386.

6.

Hendolin H, Lahtinen J, Länsimies E, Tuppurainen T, Partanen K. The effect of thoracic epidural analgesia on respiratory function after cholecystectomy. Acta Anaesthesiol Scand. 1987;31:645–651.

7.

Carli F, Trudel JL, Belliveau P. The effect of intraoperative thoracic epidural anesthesia and postoperative analgesia on bowel function after colorectal surgery: a prospective, randomized trial. Dis Colon Rectum. 2001;44: 1083–1089.

8.

Steinbrook RA. Epidural anesthesia and gastrointestinal motility. Anesth Analg. 1998;86:837–844.

9.

Shouhed D, Amersi F, Sibert K, Hemaya E, Siberman AW. Thromboprophylaxis and major oncologic surgery performed with epidural analgesia. JAMA Surg. 2013;148(1):81–84.

10. Ruppen W, Derry S, McQuay H, Moore A. Incidence of epidural hematoma, infection and neurologic injury in obstetric patients with epidural analgesia/ anesthesia. Anesthesiology. 2006;105:394–399. 11. Raj PP, Shah RV, Kaye AD, Denaro S, Hoover JM. Bleeding risk in interventional pain practice: assessment, management, and review of the literature. Pain Physician. 2004; 7(1):3–51. PubMed PMID: 16868610.

12. Shah RV, Kaye AD. Bleeding risk and interventional pain management. Curr Opin Anaesthesiol. 2008;21(4):433–438. 13. Manchikanti L, Falco FJ, Benyamin RM, et al. Assessment of bleeding risk of interventional techniques: a best evidence synthesis of practice patterns and perioperative management of anticoagulant and antithrombotic therapy. Pain Physician. 2013; 16(2 Suppl):SE261–318. 14. Horlocker TT, Wedel DJ, Rowlingson JC, et al. Regional anesthesia in the patient receiving antithrombotic or thrombolytic therapy: American Society of Regional Anesthesia and Pain Medicine Evidence-Based Guidelines (Third Edition). Reg Anesth Pain Med. 2010;35(1): 64–101. 15. Lawton MT, Porter RW, Heiserman JE, et al. Surgical management of spinal epidural hematoma: relationship between surgical timing and neurological outcome. J Neurosurg. 1995; 83(1):1–7. 16. Horkocer TT, Wedel DJ. Neurologic complication of spinal and epidural anesthesia. Reg Anesth Pain Med. 2000;25:83–98. 17. Broadbent CR, Maxwell WB, Ferrie R, et al. Ability of anaesthetists to identify a marked lumbar interspace. Anaesthesia. 2000;55:1122–1126. 18. Auroy Y, Narchi P, Messiah A, et al. Serious complications related to regional anesthesia: results of a prospective survey in France. Anesthesiology. 1997;87:479–486. 19. Lin HY, Wu HS, Peng TH, et al. Pneumocephalus and respiratory depression after accidental dural puncture during epidural analgesia: a case report. Acta Anaesthesiol Sin. 1997;35: 119–123. 20. Rodrigo P, Garcia JM, Ailagas J. General convulsive crisis related

to pneumocephalus after inadvertent dural puncture in an obstetric patient. Rev Esp Anesthesiol Reanim. 1997;44: 247–249. 21. Zaugg M, Stoehr S, Weder W, Zollinger A. Accidental pleural puncture by a thoacic epidural catheter. Anaesthesia. 1998;53: 60–71. 22. Furuya A, Matsukaw T, Ozaki M, Kumazawa T. Interpleural misplacement of an epidural catheter. J Clin Anesth. 1998;10:425–426. 23. Sharrock NE. Recordings of, and an anatomical explanation for, false positive loss of resistance during lumbar extradural analgesia. Br J Anaesth. 1979;51:253–258. 24. Halpern HS, Breen TW, Campell DC, et al. A multicenter, randomized, controlled trial comparing bupivacaine with ropivacaine for labor analgesia. Anesthesiology. 2003;98: 1431–1435. 25. Glass PSA, Estok P, Ginsberg B, et al. Use of patient-controlled analgesia to compare the efficacy of epidural to intravenous fentanyl administration. Anesth Analg. 1992;74:345–351. 26. Sandler AN, Stringer D, Panos L, et al. A randomized, double blind comparison of lumbar epidural and intravenous fentanyl infusion of post thoracotomy pain relief. Anesthesiology. 1992;77:626–634. 27. Miguel R, Barlow I, Morrell M, et al. A prospective, randomized, double-blinded comparison of epidural and intravenous sufentanil infusion. Anesthesiology. 1994;81:346–352. 28. Paech MJ, Pavy TJ, Orlikowski CE, Lim W, Evans SF. Postoperative epidural infusion: a randomized, double-blind, dose finding trial of clonidine in combination with bupivacaine and fentanyl. Anesth Analg. 1997;84:1323–1328.

489

Chapter 70: Acute pain management: PCEA/continuous epidural catheters

29. Chia Y-Y, Liu K, Liu Y-C, Chang H-C, Wong C-S. Adding ketamine in a multimodal patient controlled epidural regimen reduces postoperative pain and analgesic consumption. Anesth Analg. 1998;86:1245–1249. 30. Niemi G, Breivik H. Adrenaline markedly improves thoracic epidural analgesia produced by a low-dose infusion of bupivacaine, fentanyl and adrenaline after major surgery: a randomised, double-blind, cross-over study with and without adrenaline. Acta

490

Anaesthesiol Scand. 1998;42: 897–909.

patient with multiple sclerosis. J Clin Anesth. 1988;1(1):21–24.

31. Chaplan SR, Duncan SR, Brodsky JB, Brose W. Morphine and hydromorphone epidural analgesia. Anesthesiology. 1992;77:1090–1094.

34. Borgeat A, Blumenthal S. Postoperative pain management following scoliosis surgery. Curr Opin Anaesth. 2008;21: 313–316.

32. Bamford C, Sibley W, Laguna J. Anesthesia in multiple sclerosis. Can J Neurol Sci. 1978;5:41–44.

35. http://www.nysora.com. New York. [updated 3 January 2009; cited 30 August 2013]. Available from: http://www.nysora.com/ regional_anesthesia/ neuraxial_techniques/3026epidural-blockade.html.

33. Bader AM, Hunt CO, Datta S, Naulty S, Ostheimer G. Anesthesia for the obstetric

Section 7 Chapter

71

Special Topics

New vistas: continuous peripheral catheters/regional anesthesia in postoperative pain management Michael R. Rasmussen and Edward R. Mariano

Case study A 65-year-old male retired Navy officer, BMI of 32 kg/m2, presents for right total knee arthroplasty (TKA) for treatment of severe knee osteoarthritis resulting in intractable chronic knee pain. His medical history is also significant for hypertension, type 2 diabetes, dyslipidemia, and obstructive sleep apnea. His physical activity level is limited by chronic knee pain, for which he has taken escalating doses of oral opioid medication (oxycodone/acetaminophen tablets), making exercise tolerance difficult to assess. He quit smoking more than 10 years ago and drinks 3–4 beers per week. He is anxious about surgery and specifically expresses concern about postoperative pain.

1. What are some of the important epidemiologic considerations in TKA? The primary indication for TKA is chronic, disabling knee pain not responsive to drug therapy or other conservative therapeutic measures.[1] Although osteoarthritis is the most common etiology prompting TKA, other causes may include rheumatoid arthritis, juvenile rheumatoid arthritis, osteonecrosis, and arthritis caused by other types of inflammatory diseases. The overarching goal of TKA is to enhance quality of life by relieving pain and improving functional status, a goal achieved in 90% of patients.[2] In fact, the historical success of this procedure, in part, has led to increasing demand for primary TKA. In the USA, among Medicare beneficiaries, primary TKA volume increased 161.5% from 1991 to 2010.[3] In 2010, more than 600 000 TKA procedures were performed in the USA, and projections for 2030 estimate over 3 million primary TKA procedures.[4] Estimating the total cost of a single TKA surgery can be difficult; however

according to one recent study, costs associated with this procedure can add up to over $36 000.[5]

2. What is the rationale for standardizing the perioperative pain managementpathwayforTKApatients? With such a tremendous investment of healthcare dollars and other resources at stake, as well as the lives of millions of patients, it is imperative to coordinate perioperative care in a manner that maximizes efficiency, safety, patient satisfaction, and positive patient outcomes. Standardized patient care pathways (also referred to as “clinical pathways”) involving multiple healthcare disciplines have been extensively studied for lower extremity joint replacement.[6] One essential component in the care of these patients is standardizing the perioperative management of pain; effective analgesic protocols for joint replacement patients can speed postoperative recovery and avoid potential complications.[7] Furthermore, protocols that incorporate specific techniques, such as continuous peripheral nerve blocks (CPNB), may decrease the time required for patients to meet criteria for discharge eligibility[8] and possibly reduce hospitalization costs.[9] The rationale for developing and implementing a multimodal analgesic protocol is to create a perioperative patient care experience that is consistent, effective, safe, and reliable.

3. What is a multimodal analgesic protocol? The experience of postoperative pain involves complex biological, emotional, and psychosocial processes involving many different receptors, neurotransmitters,

Case Studies in Pain Management, ed. Alan David Kaye and Rinoo V. Shah. Published by Cambridge University Press. © Cambridge University Press 2015.

491

Chapter 71: Continuous peripheral catheters/regional anesthesia in postoperative pain management

and hormones. By employing an approach that involves multiple medications from different classes, multimodal analgesia modulates pain at various “stops” along this complex path, allowing for reduced dosing of each individual agent. The American Society of Anesthesiologists (ASA) recommends the use of multimodal analgesic strategies for acute pain management whenever possible in an effort to optimize individual drug efficacy while minimizing the risk of adverse drug effects.[10] The pain experience can vary greatly from one patient to the next even after the same surgical procedure. Therefore, the most effective analgesic pathway should provide a standardized framework, yet allow for customization at the individual level based on patient comorbidities, preoperative analgesic usage, prior perioperative pain experiences, type of surgery, and patient preferences.[10]

4. What are potential components of an effective multimodal analgesic protocol? A multimodal analgesic protocol may consist of multiple medications and interventions that target pain in different ways (Table 71.1).

Local anesthetics Local anesthetics play a vital role in the multimodal analgesic protocol for many types of surgery and can be administered systemically[11] or by injection for perioperative pain management. Injections of local anesthetics can take the form of local infiltration analgesia (LIA),[12] neuraxial regional anesthesia, or peripheral regional anesthesia, either as singleinjection or CPNB. Protocols may even incorporate local anesthetics in a combination of routes (e.g., LIA plus regional anesthesia).[13] Neuraxial (i.e., spinal or epidural) regional anesthesia techniques can provide very effective postoperative analgesia; however these techniques are not selective for the surgical site, may require prolonged urinary catheterization, and produce other untoward effects such as pruritus, hypotension, and impaired ambulation.[14] As an intraoperative technique, spinal anesthesia may deserve special consideration; a recent large retrospective study has associated spinal anesthesia with decreased morbidity and mortality following joint arthroplasty.[15] Compared to general anesthesia, patients who receive spinal anesthesia experience lower

492

Table 71.1. Potential components of an effective multimodal analgesic protocol

Local anesthetics

Local infiltration analgesia (LIA) Neuraxial regional anesthesia Peripheral regional anesthesia Systemic

Opioids

Injectables including intravenous patient-controlled analgesia (PCA) Oral short- and long-acting preparations

Non-opioid systemic analgesics

Acetaminophen Non-steroidal anti-inflammatory drugs (NSAIDs) including cyclooxygenase-2 (COX-2) inhibitors Gabapentin or pregabalin N-methyl-D-aspartate (NMDA) receptor antagonists Glucocorticoids Antidepressants Alpha-2 agonists

rates of pulmonary embolism, respiratory failure, pneumonia, stroke, infections, acute renal failure, and need for blood products.[15] However, the duration of postoperative analgesia for a single-injection spinal anesthetic technique is limited even when hydrophilic opioids are used. Peripheral nerve blockade offers the advantage of selectivity for the operative limb and sparing of the non-operative limb. The sensory innervation of the knee is complex and derives from both the lumbar plexus and sacral plexus. Although both the femoral and sciatic nerves can be anesthetized to provide near complete analgesia following TKA, the routine addition of a sciatic nerve block remains controversial.[16] A sciatic nerve block does produce a transient footdrop from anesthesia of the common peroneal nerve, potentially confounding any surgical injury, in addition to impairing mobility and increasing fall risk. Standardization of the regional anesthesia component of a multimodal analgesic protocol for TKA should be undertaken in cooperation with surgeons, nurses, and physical therapists to achieve the overall perioperative goals, ensuring patient safety and comfort, while balancing the advantages and disadvantages of individual regional anesthetic techniques. Femoral nerve blockade, specifically femoral CPNB, has gained broad usage for TKA and is often considered the gold standard in terms of regional

Chapter 71: Continuous peripheral catheters/regional anesthesia in postoperative pain management

Table 71.2. Common continuous peripheral nerve block (CPNB) catheter sites and associated surgical indications

CPNB catheter site

Planned surgical site

Interscalene brachial plexus

Shoulder, proximal humerus

Supraclavicular brachial plexus

Shoulder, proximal humerus, elbow, forearm, wrist, hand

Infraclavicular brachial plexus

Elbow, forearm, wrist, hand

Axillary brachial plexus

Wrist, hand

Posterior lumbar plexus

Hip, thigh

Femoral nerve

Knee, thigh

Adductor canal (saphenous nerve)

Knee

Sciatic nerve

Leg, ankle, foot

analgesia for this surgery.[17] For TKA, the main limitation of single-injection femoral nerve blocks is duration since patients often begin intensive physical therapy on the first postoperative day (POD) after the block has resolved. CPNB involves the insertion of a catheter in the proximity of a target nerve or plexus, and CPNB techniques have been used for a variety of surgical indications (Table 71.2). For patients undergoing TKA, the femoral nerve is typically the primary target. Infusion devices can deliver local anesthetic directly to the target nerve on an ongoing basis for days after surgery, and the regimen varies according to the device (e.g., basal rate only vs. basal rate plus patient-controlled bolus vs. scheduled intermittent bolus only). In a recent meta-analysis comparing CPNB to single-injection peripheral nerve blocks, CPNB results in lower patient-reported worst pain scores and pain scores at rest on POD 0, 1, and 2; by POD 3, pain outcomes appear similar.[18] In addition, CPNB patients experience less nausea, consume less opioids, sleep better, and report greater satisfaction with pain management.[18] However, the management of CPNB patients, especially lower extremity joint arthroplasty patients, takes a hands-on approach, and not all anesthesiology practices are equipped with an acute pain service. A healthcare provider must be available at all times to manage common issues associated with CPNB and

assess patients at least once daily for analgesic efficacy and potential side effects.[19] The TKA population is special in that typical local anesthetic infusions administered via femoral CPNB, although effective for analgesia, can produce clinically significant and unwanted quadriceps weakness which can impair early mobilization and increase fall risk.[20] For this reason, there has been increasing interest in a relatively new peripheral nerve block technique that delivers local anesthetic distal to the main femoral nerve in the adductor canal of the thigh,[21] therefore resulting in less quadriceps weakness[22] and facilitating early ambulation.[13] The adductor canal block is nearly a pure sensory block, which primarily blocks the saphenous nerve, nerves to the patellar plexus, and the motor nerves to the vastus medialis muscle and is amenable to CPNB.[21] Recent retrospective studies have shown that replacing femoral CPNB with either single-injection adductor canal block plus LIA[13] or continuous adductor canal block plus LIA[23] for TKA patients results in greater ambulation on POD 1. The LIA technique involves the injection of a large volume of long-acting local anesthetic (e.g., ropivacaine) by the surgeon directly into the soft tissues in and around the knee joint and generally lasts 6–12 hours.[12] Adjuvants such as NSAIDs and epinephrine, with or without opioids, are also commonly included in the LIA solution and injected concurrently.

Opioid analgesics Opioid analgesics, mu-opioid receptor agonists, continue to play a central role in perioperative pain management. Although potent and effective analgesics, they are associated with a multitude of undesirable side effects (e.g., respiratory depression, lethargy, constipation, nausea and vomiting, immune suppression, and cancer cell growth). However, despite these side effects, opioids should continue to be part of the perioperative analgesic pathway. While intravenous opioid PCA is commonly employed for postoperative pain management, it may not be necessary for patients undergoing lower extremity joint replacement.[6] For example, the authors’ institution uses a three-tiered approach for post-TKA opioid administration. All patients receive scheduled doses of oral opioids in the form of opioid/ acetaminophen combination tablets; alternatively, scheduled long-acting opioid formulations can be used. Additional doses of short-acting oral opioids are available as

493

Chapter 71: Continuous peripheral catheters/regional anesthesia in postoperative pain management

needed for breakthrough pain. Intravenous opioids are prescribed on an as-needed basis and are reserved for severe breakthrough pain not alleviated by oral formulations. This three-tiered approach for opioid dosing ensures a basal level of opioid analgesia to prevent patients’ “getting behind” on pain control, especially during physical therapy, and also allows for additional analgesics in different formulations upon request if pain is not relieved. Monitoring the amount of daily breakthrough medication use and pain scores for each patient allows for individual titration. The specific opioid dosing for each tier depends on various patient-specific attributes such as prior or current use of opioids, weight, age, comorbidities, metabolic function, and responsiveness to pre- and intraoperative opioid dosing. For patients on chronic opioid therapy prior to undergoing TKA, preoperative baseline doses of opioids should be continued in addition to the previously discussed analgesic protocol.

Non-opioid systemic analgesics Non-opioid systemic analgesics such as non-steroidal anti-inflammatory drugs (NSAIDs) cyclooxygenase-2 (COX-2) inhibitors, antidepressants, alpha-2 agonists, calcium channel blockers (e.g., gabapentin and pregabalin), glucocorticoids, acetaminophen, NMDA receptor antagonists, and local anesthetics can be effectively combined with opioids as part of a postoperative multimodal, opioid-sparing analgesic regimen.[24,25] In general, patients already treated with such medications preoperatively should continue them throughout the perioperative period unless contraindicated.[10] COX-2 inhibitors, when given preoperatively in TKA patients, have been shown to reduce opioid dosing requirements and opioid-related adverse effects.[26] The inclusion of NSAIDs and acetaminophen is quite common in multimodal analgesic pathways, and their effectiveness in decreasing opioid consumption has been well described; they should be given on a scheduled basis.[27] The perioperative use of calcium channel blockers such as gabapentin and pregabalin also demonstrates opioidsparing effects in multiple studies and should be considered as part of any multimodal pathway;[28] dose reduction or avoidance may be necessary for patients of advanced age or significant comorbidity burden. Alpha-2 agonists such as clonidine and dexmedetomidine have proven their opioid-sparing

494

capabilities,[25] but may be complicated by systemic hypotension when given in higher doses or when administered intrathecally. The analgesic utility of glucocorticoids has recently been described in a meta-analysis of studies using single dose intravenous dexamethasone; patients who received dexamethasone reported lower postoperative pain scores at 2 and 24 hours, decreased opioid consumption, and shorter postanesthetic care unit (PACU) stays.[29]

5. What are some special considerations for patients with chronic pain and chronic opioid therapy? Many patients presenting for TKA already experience chronic pain and are at increased risk of hyperalgesia and wind-up postoperatively. They may also present with chronic pain of other etiologies such as spine pain or other arthritic joint pains. These comorbid chronic pain syndromes can make acute pain management after surgery challenging, and these patients should receive special consideration. Nowhere else is the use of multimodal analgesia more important than in the opioid-dependent, chronic pain patient presenting for surgery, especially lower extremity joint replacement. In addition to the other multimodal medications already discussed, especially local anesthetic techniques such as perioperative CPNB, an intraoperative low-dose ketamine infusion is an especially useful adjunct in the care of chronic pain patients, especially those with pre-existing hyperalgesia on chronic opioid therapy. Although ketamine acts on many different receptor types throughout the body,[29] the most notable is the NMDA receptor. Perioperative ketamine has been shown to decrease supplemental opioid requirements and wound hyperalgesia[30] and may even have longer-term benefits in preventing the development of persistent pain.[31] In a prospective study on chronic pain patients undergoing major spine surgery, the addition of a low-dose ketamine infusion resulted in decreased pain scores and opioid consumption in the immediate postoperative period, and these benefits continued for weeks after surgery.[32] These beneficial effects seem to be most prominent in more painful surgeries such as joint replacement because ketamine is thought to reduce central sensitization of dorsal horn neurons caused by prolonged, intense pain; therefore, ketamine not only

Chapter 71: Continuous peripheral catheters/regional anesthesia in postoperative pain management

Table 71.3. Suggested multimodal analgesic protocol for total knee arthroplasty patients from the authors’ institution

Preoperatively

-

Assessment of patient’s comorbidities and preoperative analgesic regimen (e.g., opioids especially) Fall prevention education form reviewed and signed by patient and regional anesthesiologist Insertion of adductor canal continuous peripheral nerve block catheter and bolus with 1.5% mepivacaine

Intraoperatively

-

General or neuraxial anesthetic (preferred) Low-dose ketamine (0.5 mg/kg bolus + 0.25 mg/kg/hr infusion) for patients with chronic pain syndrome or preoperative opioid use undergoing general anesthesia Local infiltration analgesia by surgeon at the end of surgery with 0.2% ropivacaine + ketorolac + epinephrine

Postoperatively

-

Perineural infusion of 0.2% ropivacaine via adductor canal catheter Scheduled medications: oral oxycodone, acetaminophen, and diclofenac As-needed medications: oral oxycodone and intravenous (IV) morphine No routine IV opioid patient-controlled analgesia

addresses acute postsurgical pain, but also the development of persistent postsurgical pain.[33,34]

6. How does the diagnosis of obstructive sleep apnea (OSA) influence the analgesic protocol? Local anesthetics are potent, non-sedating analgesics, and selective regional anesthesia techniques such as CPNB should be used whenever possible in patients with OSA according to the ASA Practice Guidelines for the perioperative management of these patients.[35] OSA patients are at increased risk for pulmonary

References 1.

Cross WW 3rd, Saleh KJ, Wilt TJ, Kane RL. Agreement about indications for total knee arthroplasty. Clin Orthop Relat Res. 2006;446:34–39.

2.

NIH Consensus Statement on total knee replacement. NIH Consensus and State-of-the-Science Statements. 2003;20(1):1–34.

3.

Cram P, Lu X, Kates SL, et al. Total knee arthroplasty volume, utilization, and outcomes among Medicare beneficiaries, 1991–2010. JAMA. 2012;308 (12):1227–1236.

4.

Ruiz D, Koenig L, Dall TM, et al. The direct and indirect costs to

5.

6.

7.

complications in the perioperative period due to their increased susceptibility to the respiratory depressant effects of anesthetics and opioid analgesics. A multimodal analgesic protocol that minimizes opioid use is recommended (Table 71.3).

Conflict of interest Dr. Mariano has received unrestricted educational funding for teaching programs from I-Flow Corporation (Lake Forest, California, USA) and B. Braun Medical Inc. (Bethlehem, Pennsylvania, USA). These companies had absolutely no input into any aspect of manuscript preparation.

society of treatment for end-stage knee osteoarthritis. J Bone Joint Surg Am. 2013;95(16):1473–1480. Odum SM, Troyer JL, Kelly MP, Dedini RD, Bozic KJ. A costutility analysis comparing the cost-effectiveness of simultaneous and staged bilateral total knee arthroplasty. J Bone Joint Surg Am. 2013;95(16):1441–1449. Hebl JR, Kopp SL, Ali MH, et al. A comprehensive anesthesia protocol that emphasizes peripheral nerve blockade for total knee and total hip arthroplasty. J Bone Joint Surg Am. 2005;87 (Suppl 2):63–70. Hebl JR, Dilger JA, Byer DE, et al. A pre-emptive multimodal

pathway featuring peripheral nerve block improves perioperative outcomes after major orthopedic surgery. Reg Anesth Pain Med. 2008;33 (6):510–517. 8.

Ilfeld BM, Le LT, Meyer RS, et al. Ambulatory continuous femoral nerve blocks decrease time to discharge readiness after tricompartment total knee arthroplasty: a randomized, triplemasked, placebo-controlled study. Anesthesiology. 2008;108 (4):703–713.

9.

Ilfeld BM, Mariano ER, Williams BA, Woodard JN, Macario A. Hospitalization costs of total knee arthroplasty with a continuous

495

Chapter 71: Continuous peripheral catheters/regional anesthesia in postoperative pain management

femoral nerve block provided only in the hospital versus on an ambulatory basis: a retrospective, case-control, cost-minimization analysis. Reg Anesth Pain Med. 2007;32(1):46–54. 10. Practice guidelines for acute pain management in the perioperative setting: an updated report by the American Society of Anesthesiologists Task Force on Acute Pain Management. Anesthesiology. 2012;116 (2):248–273. 11. Dahl JB, Mathiesen O, Kehlet H. An expert opinion on postoperative pain management, with special reference to new developments. Expert Opin Pharmacotherapy. 2010;11 (15):2459–2470. 12. Kerr DR, Kohan L. Local infiltration analgesia: a technique for the control of acute postoperative pain following knee and hip surgery: a case study of 325 patients. Acta Orthop. 2008;79 (2):174–183. 13. Perlas A, Kirkham KR, Billing R, et al. The impact of analgesic modality on early ambulation following total knee arthroplasty. Reg Anesth Pain Med. 2013;38 (4):334–339. 14. Zaric D, Boysen K, Christiansen C, et al. A comparison of epidural analgesia with combined continuous femoral-sciatic nerve blocks after total knee replacement. Anesth Analg. 2006;102(4):1240–1246. 15. Memtsoudis SG, Sun X, Chiu YL, et al. Perioperative comparative effectiveness of anesthetic technique in orthopedic patients. Anesthesiology. 2013;118 (5):1046–1058. 16. Ilfeld BM, Madison SJ. The sciatic nerve and knee arthroplasty: to block, or not to block–that is the question. Reg Anesth Pain Med. 2011;36(5):421–423. 17. Ilfeld BM, Mariano ER, Girard PJ, et al. A multicenter, randomized,

496

triple-masked, placebo-controlled trial of the effect of ambulatory continuous femoral nerve blocks on discharge-readiness following total knee arthroplasty in patients on general orthopaedic wards. Pain. 2010;150(3):477–484. 18. Bingham AE, Fu R, Horn JL, Abrahams MS. Continuous peripheral nerve block compared with single-injection peripheral nerve block: a systematic review and meta-analysis of randomized controlled trials. Reg Anesth Pain Med. 2012;37(6):583–594. 19. Mariano ER. Making it work: setting up a regional anesthesia program that provides value. Anesthesiol Clin. 2008;26 (4):681–692, vi. 20. Ilfeld BM, Duke KB, Donohue MC. The association between lower extremity continuous peripheral nerve blocks and patient falls after knee and hip arthroplasty. Anesth Analg. 2010;111(6):1552–1554. 21. Lund J, Jenstrup MT, Jaeger P, Sorensen AM, Dahl JB. Continuous adductor-canalblockade for adjuvant postoperative analgesia after major knee surgery: preliminary results. Acta Anaesthesiol Scand. 2011;55 (1):14–19. 22. Jaeger P, Nielsen ZJ, Henningsen MH, et al. Adductor canal block versus femoral nerve block and quadriceps strength: a randomized, double-blind, placebo-controlled, crossover study in healthy volunteers. Anesthesiology. 2013;118 (2):409–415.

25. Buvanendran A, Kroin JS. Multimodal analgesia for controlling acute postoperative pain. Curr Opin Anaesthesiol. 2009;22(5):588–593. 26. Buvanendran A, Kroin JS, Tuman KJ, et al. Effects of perioperative administration of a selective cyclooxygenase 2 inhibitor on pain management and recovery of function after knee replacement: a randomized controlled trial. JAMA. 2003;290(18):2411–2418. 27. Elia N, Lysakowski C, Tramer MR. Does multimodal analgesia with acetaminophen, nonsteroidal antiinflammatory drugs, or selective cyclooxygenase-2 inhibitors and patient-controlled analgesia morphine offer advantages over morphine alone? Meta-analyses of randomized trials. Anesthesiology. 2005;103(6):1296–1304. 28. Dauri M, Faria S, Gatti A, et al. Gabapentin and pregabalin for the acute post-operative pain management: a systematicnarrative review of the recent clinical evidences. Curr Drug Targets. 2009;10(8):716–733. 29. De Oliveira GS Jr., Almeida MD, Benzon HT, McCarthy RJ. Perioperative single dose systemic dexamethasone for postoperative pain: a meta-analysis of randomized controlled trials. Anesthesiology. 2011;115(3):575–588. 30. Schmid RL, Sandler AN, Katz J. Use and efficacy of low-dose ketamine in the management of acute postoperative pain: a review of current techniques and outcomes. Pain. 1999;82(2):111–125.

23. Mudumbai SC, Kim TE, Howard SK, et al. continuous adductor canal blocks are superior to continuous femoral nerve blocks in promoting early ambulation after TKA. Clin Orthop Relat Res. 2014;472(5): 1377–1383.

31. Remerand F, Le Tendre C, Baud A, et al. The early and delayed analgesic effects of ketamine after total hip arthroplasty: a prospective, randomized, controlled, double-blind study. Anesth Analg. 2009;109 (6):1963–1971.

24. Young A, Buvanendran A. Recent advances in multimodal analgesia. Anesthesiol Clin. 2012;30(1):91–100.

32. Loftus RW, Yeager MP, Clark JA, et al. Intraoperative ketamine reduces perioperative opiate

Chapter 71: Continuous peripheral catheters/regional anesthesia in postoperative pain management

consumption in opiate-dependent patients with chronic back pain undergoing back surgery. Anesthesiology. 2010;113 (3):639–646. 33. Laskowski K, Stirling A, McKay WP, Lim HJ. A systematic review of intravenous ketamine for

postoperative analgesia. Can J Anaesth. 2011;58(10):911–923. 34. Kohrs R, Durieux ME. Ketamine: teaching an old drug new tricks. Anesth Analg. 1998;87 (5):1186–1193. 35. Gross JB, Bachenberg KL, Benumof JL, et al. Practice guidelines for the

perioperative management of patients with obstructive sleep apnea: a report by the American Society of Anesthesiologists Task Force on Perioperative Management of patients with obstructive sleep apnea. Anesthesiology. 2006;104 (5):1081–1093.

497

Section 7 Chapter

72

Special Topics

Methadone and treatment of chronic pain Daniel Krashin, Natalia Murinova, and Andrea Trescot

Methadone is familiar to most laypersons as a treatment for heroin addiction, dispensed at special clinics. Methadone is also a powerful, inexpensive, and effective analgesic, but it has many unique and potentially lethal aspects that have to be well understood before using it for pain relief. Addiction treatment with methadone requires a special license, but any provider who can prescribe Schedule II medications can write for methadone for pain. Unfortunately, because no special training is required to prescribe methadone and because poor insurance coverage is leading to the increased use of inexpensive methadone, there has been a tragic dramatic increase in opioid deaths attributed to methadone. This chapter reviews the indications, contraindications, drug interactions, metabolism, genetic issues, ethics, and initial evaluation and treatment for methadone.

Case study Ms. X is a 33-year-old white female, HIV-positive former IV drug abuser, who has supported herself for many years working as a voice actress. Her HIV is well controlled with zidovudine, emtricitabinetenofovir, and etravirine. She states she is compliant with her medication therapy. She has been “clean” for 3 years. She has chronic peripheral neuropathy, which is bilateral and progressive. She has tried gabapentin and tramadol, with limited relief. Her infectious disease specialist, who was providing her HIV care, treated her with methadone 30 mg per day for her pain; however, he retired, and her new physician is uncomfortable with continuing to prescribe methadone. The provider’s main concerns include recent reports of deaths from methadone. Her provider is also concerned that the patient was seeking early refills on more than one occasion, and her prior

history of drug abuse. The patient is informed she may be taken off methadone. She becomes visibly upset and states that she “ feels my life is not worth living in pain.” Ms. X has been given a referral to the pain clinic. She has “enough medication” to get her to the pain clinic, and has high expectations; she had heard “you are the best.”

1. What are the clinical uses of methadone? Methadone is a synthetic opioid which was developed by the Germans during World War II, but not put into widespread use until after the war due to difficulty understanding the proper dosing, difficulties that continue to bedevil prescribers of methadone.[1] Methadone’s unique properties make this drug particularly well suited to certain uses in medicine, particularly in the areas of chronic pain treatment and in the treatment of opioid dependence. The federal government has estimated that over 268 thousand people are enrolled in methadone maintenance programs for the treatment of opioid dependence,[2] while over 720 thousand use methadone to treat chronic pain.[3] The reasons for this wide use are multiple. The half-life of methadone is long, allowing it to be used as a long-acting medication once a day to prevent withdrawal or 2–3 times a day to treat pain. Methadone is, in the USA, cheaper by an order of magnitude than most other longacting opioids. Methadone also has high bioavailability and lacks neurotoxic metabolites. The NMDA antagonism of methadone may also make it less prone to cause hyperalgesia and more effective in treating neuropathic pain.

Case Studies in Pain Management, ed. Alan David Kaye and Rinoo V. Shah. Published by Cambridge University Press. © Cambridge University Press 2015.

498

Chapter 72: Methadone and treatment of chronic pain

2. What about using methadone in the chronically ill patient with HIV? Patients with chronic illnesses beyond their chronic pain condition require additional consideration and care. In the case of the patient with HIV, pain is a common comorbidity. Some chronic conditions may directly influence the metabolism of methadone, such as hepatic disease, but more common are increased vulnerabilities to the adverse effects of methadone, such as increased risk of respiratory depression with in sleep apnea or COPD patients. Drug-drug interactions are also a common issue, and many HIV antiretroviral drugs have potential interactions with methadone, as will be discussed below.

3. How does her history of substance abuse change methadone prescribing? Patients with past histories of opioid abuse frequently have significant opioid tolerance and may require higher opioid doses for management of pain.[4,5] It should be noted that it is illegal to treat opioid dependence with opioid agonists either for detoxification or opioid maintenance unless they are being prescribed as part of a methadone maintenance program or by a registered Suboxone® (buprenorphine) prescriber. However, pain treatment with opioids is allowable, and should be clearly documented as such. Methadone can be abused and diverted just as any other opioid medication, and with the growing use of methadone for pain, growing numbers of overdoses have been observed with methadone.[6] For any chronic opioid therapy patient, but particularly one with red flags such as psychiatric illness or substance abuse history, it is essential to follow best practices in pain management. These include the use of opioid agreements, patient education and informed consent, and the use of prescription monitoring programs. More direct assessments of compliance such as pill counts and urine drug screens should also be used. Patients should use only one provider and one pharmacy to obtain their opioids and need to understand that doctor-shopping and seeking early refills is a violation of the treatment agreement. Long-acting opioids are also recommended to decrease the “pop a pill, feel better” effects of short-acting opioids.

4. What is the cost of methadone? Methadone for chronic pain is fairly inexpensive, with an estimated cost of $27 for a month supply of 90 10 mg tablets.[7] The oral liquid and IV formulations are significantly more expensive.

5. How common is accidental overdose in methadone use? Accidental overdose is a common cause of death with methadone treatment. The long and variable half-life, which results in patients receiving the full effect of a new methadone prescription more than a day after having started a new prescription, potentially results in patients developing sedation and respiratory depression at unexpected times, often at night. The other pitfall in methadone dosing is the non-linear dose-response curve. The ratio of morphine equivalency increases dramatically with dosing, so that methadone potency ranges from four times that of morphine at the lowest doses to 12 times morphine at doses of 120 mg/day. Thus, increasing the dose in seemingly small increments can greatly increase the opioid potency, and patients on a stable dosing regimen who take an extra methadone tablet in response to increased pain can easily put themselves in the toxic range. Dose titration for methadone should therefore be slow and cautious and take the long time to steady-state into account.[8] If the patient on methadone needs additional medication for acute pain, such as after a surgical procedure or trauma, it may be safer to use short-acting opioids along with the standard methadone dose.

6. What are other risks of methadone use? Methadone can be lethal even after a single dose if the patient has genetic predisposition to long QTc, which can cause a torsades de pointe cardiac arrhythmia syndrome. In a case familiar to the authors, a patient with undiagnosed congenital prolonged QT interval suffered cardiac arrest after receiving a single methadone dose, surviving only because this occurred in a large medical center and she was immediately able to receive cardiac care. This adverse effect, not shared by other opioids, occurs because methadone binds to the KCNH2 cardiac channel and prolongs the action potential in a dose-dependent fashion.[9] High-dose

499

Chapter 72: Methadone and treatment of chronic pain

methadone increases the corrected QT interval.[10,11] In patients with a normal screening EKG, another should be checked once the patient is stable on their dose of methadone to reassess QT interval while on this medication.[12,13] Methadone doses should be lower in patients with known liver impairment, since this will increase methadone dose and the likelihood of its adverse effects. Drug-drug interactions can both increase methadone levels in the body and also raise the risk of arrhythmia when multiple medications that affect the QTc are used concurrently.[14] Cocaine has been reported to cause cardiac arrest in patients on chronic methadone.[15] Ehret et al found that long QT syndrome was common in patients using methadone to treat opioid dependence, and that it was commonly associated with the use of CYP3A4 inhibitors, hypokalemia, and hepatic dysfunction.[16]

7. What are drug-drug interactions of methadone? Methadone is hepatically metabolized in the liver by the P450 enzyme system, specifically CYP3A4 and 2D6 (and perhaps 2B6). Methadone has numerous potential interactions; a recent systematic review found over 50 potential clinically significant interactions with drugs and supplements.[17] Grapefruit juice, which is a well-known inhibitor of intestinal CYP3A4, has been shown to cause a mild increase in methadone levels.[18] Medications that affect the CYP3A4 and 2D6 may increase or decrease methadone blood levels, resulting in inadequate analgesia or toxicity. With patients on complex medication regimens, the risk of methadone treatment is greatly increased. Many HIV antiretroviral regimens can interact with methadone.[19] Methadone in turn can alter HIV drug metabolism, possibly leading to the development of viral resistance. It is therefore essential to coordinate the treatment of HIV and pain treatments, particularly if more than one provider is prescribing for the patient. The specific discussion of medication interactions in this chapter is necessarily brief, and given the complexity and frequent changes associated with antiretroviral treatment, it is usually beneficial to closely coordinate with the infectious disease provider. The most common offenders are the nucleoside reverse-transcriptase inhibitor (NRTI) medications, such as efavirenz and nevirapine, which act as enzyme

500

inducers, increasing methadone metabolism and reducing the area under the curve (AUC) by up to half.[20] However, etravirine does not usually cause significant dosing changes.[21] Protease inhibitors (PI) can also decrease the AUC of methadone, particularly lopinavir with ritonavir, darunavir with ritonavir, tipranavir, and nelfinavir.[22] The enzyme induction often takes some weeks to result in decreased methadone effect, but discontinuing the enzyme inducer can result in rapid decrease of methadone metabolism, requiring prompt adjustment of dosage.[23] The NRTIs can also be affected by methadone, with decreased levels of abacavir and increased levels of zidovudine.[24]

8. What is the role of genetic testing? Genetic testing is not currently considered part of the standard of care for pain treatment, and the evidence base for the use of genetic testing in pain management is very limited. However, some genetic polymorphisms have been found to be associated with the risk of opioid dependence and with the methadone doses necessary for successful maintenance therapy.[25] Genetic variations that influence the amount and activity of the P450 hepatic enzymes would, in theory, be useful in predicting responses to opioids and dosage requirements. In methadone treatment, the majority of studies have been done in methadone maintenance patients. Both CYP2D6[26] and CYP3A4[27] have been associated with changes in methadone levels. The ABCB1 gene, which codes for the P-glycoprotein transporter that acts to remove methadone from the central nervous system, has also been shown to influence dosage requirements in methadone maintenance.[28]

9. What is the metabolism and mechanism of methadone? “The action of methadone is mixed and complicated.”[1] Methadone’s primary action is binding μ-opioid receptor; however, methadone also acts as an NMDA antagonist. In addition, methadone acts as a serotonin and norepinephrine reuptake inhibitor.[29] The NMDA antagonism may be responsible for methadone being less likely to cause opioid-induced hyperalgesia and to be more effective in neuropathic pain than pure μ-opioid agonists such as morphine. In the sciatic-constriction rat model of neuropathic pain,

Chapter 72: Methadone and treatment of chronic pain

it has been found that only about 60% of the pain relief is reversible with naloxone, with the other 40% operating through non-opioid receptors, most likely the NMDA receptor.[30] Methadone is highly bioavailable when taken orally; at 80% it is three times more available than morphine.[31] The oral form has onset of action within 30 to 60 minutes of ingestion. Methadone is highly lipophilic and protein bound. The half-life of methadone is long and variable, anywhere from 8 to 59 hours, so that steady-state is often not achieved until 3 or 4 days after starting the medication. While daily dosing is sufficient to prevent opioid withdrawal, the duration of analgesic effect is usually 6–8 hours and thus it is usually prescribed every 8 hours for pain. When tapering or discontinuing methadone, it is prudent to decrease the dosage no more than every 3 days. Since it is so lipophilic, chronic methadone use results in drug deposition in the body’s fat stores. The primary metabolism of methadone is hepatic, using CYP3A4, which converts methadone through N-demethylation to EDDP (2-ethylidene-1,5-dimethyl3,3-diphenylpyrrolidine), an inactive and non-toxic metabolite.[32] The methadone-to-EDDP ratio can be used as a gauge of methadone metabolism, and serum EDDP level can be used as a gauge of methadone compliance.[33] CYP2D6 and CYP1A2 are also involved in methadone metabolism. Methadone should be used cautiously with hepatic impairment.

10. What are the advantages of methadone in this patient? HIV patients have a high rate of comorbid pain conditions, estimated at between 30 and 60%.[34] In patients with HIV, comorbid substance abuse appears to be associated with less improvement in pain ratings.[35] This patient presents with neuropathic pain, which is a common cause of chronic pain in the general population, with an estimated prevalence of 8% in a recent French study.[36] In HIV patients, neuropathic pain is much more common, and a study of 1500 HIV clinic patients found evidence of sensory neuropathy in over half, and complaints of neuropathic pain in 38% of these, or 22% of the entire sample.[37] Methadone is long-acting and inexpensive, which may make it more available and useful for this patient. Methadone may also be more effective for her neuropathic pain than other opioids. Other

classes of medications such as tricyclic antidepressants, gabapentin, and serotonin-norepinephrine reuptake inhibitors should be considered first line in neuropathic pain treatment.[38] Methadone appeared to be an effective treatment for neuropathic non-cancer pain in several case studies.[39,40]

11. What are the risks of methadone in this patient? Methadone has a narrow therapeutic index and a higher risk of toxicity or accidental overdose than other opioids. The rates of methadone poisoning and unintentional methadone poisoning have both increased in the USA dramatically since 1999, likely due to the increased rate of methadone prescribing for pain.[41] These have been observed both in patients receiving prescriptions for pain as well as in abusers of diverted methadone.[42] She also faces increased risk of a dangerous drug interaction with her HIV medications. If she returns to substance abuse, she faces increased risks of overdose or toxicity. Substance abusers are also more likely to sell their pain medication and to be nonadherent with treatment.

12. What are the ethical considerations? The choice to treat patients with chronic opioid therapy is a complex ethical issue. The ethical duties to help patients and reduce suffering must be balanced with the duty to do no harm. While chronic opioid treatment offers the promise of pain relief and restoration of function, many problems have become obvious over time. Many patients on high-dose opioids develop dangerous side effects, opioid tolerance and opioid-induced hyperalgesia. The adverse effects of opioid treatment have become ever more evident.[43] In high-risk patients with psychiatric illness or active addiction, who are more likely to have adverse events or bad outcomes, the ethical balance must take these risks into account. The risks of iatrogenic addiction, overdose, and diversion are real and must be taken seriously.

13. What should be the initial approach to this patient? Although Ms. X reports using gabapentin in the past, a significant number of patients have never had adequate trials of anticonvulsants or other

501

Chapter 72: Methadone and treatment of chronic pain

membrane modulators. Patients often either do not use them long enough or at adequate doses for a full trial, or may not have realistic expectations for these medications. If the starting dose is too high or titrated too quickly, the side effects may be intolerable. The provider should take a detailed history, focusing on details of any medication trials, including the duration of trial and the maximum dose tolerated. Comorbid anxiety and depression needs to be addressed, as the patient may be using opioids for

References 1.

Chen K. Pharmacology of methadone and related compounds. Ann N Y Acad Sci. 1948;51(1):83–97.

2.

Substance Abuse Mental Health Services Administration. National survey of substance abuse treatment services (N-SSATS). Data on Substance Abuse Treatment Facilities. Rockville, MD: SAMHSA/Office of Applied Studies. 2008.

3.

Krantz MJ, Martin J, Stimmel B, Mehta D, Haigney MC. QTc interval screening in methadone treatment. Ann Intern Med. 2009;150(6):387–395.

4.

Hughes J, Bickel W, Higgins S. Buprenorphine for pain relief in a patient with drug abuse. Am J Drug Alcohol Abuse. 1991; 17(4):451–455.

5.

Umbricht A, Hoover D, Tucker M, et al. Opioid detoxification with buprenorphine, clonidine, or methadone in hospitalized heroin-dependent patients with HIV infection. Drug Alcohol Depend. 2003;69 (3):263–272.

6.

7.

502

Maxwell JC, McCance-Katz EF. Indicators of buprenorphine and methadone use and abuse: what do we know? Am J Addictions. 2010;19(1):73–88. GoodRx. GoodRx Methadone Page. GoodRx.com 2013; http:// www.goodrx.com/methadone.

8.

9.

treatment of her severe anxiety. However, when antidepressants are prescribed, it further complicates her therapy, as many antidepressants are CYP2D6 inhibitors and can change the blood levels of the opioids prescribed.[44,45] Given her substance abuse history, Ms. X will need close monitoring. Methadone, with its long half-life and NMDA inhibition, is the most appropriate choice of chronic opioid therapy for this patient, but her provider will need expert guidance to be successful and safe with this treatment.

Davoli M, Bargagli AM, Perucci CA, et al. Risk of fatal overdose during and after specialist drug treatment: the VEdeTTE study, a national multi-site prospective cohort study. Addiction. 2007; 102(12):1954–1959. Kornick CA, Kilborn MJ, Santiago-Palma J, et al. QTc interval prolongation associated with intravenous methadone. Pain. 2003;105(3):499–506.

10. Krantz MJ, Lewkowiez L, Hays H, et al. Torsade de pointes associated with very-high-dose methadone. Ann Intern Med. 2002;137(6):501–504. 11. Gil M, Sala M, Anguera I, et al. Qt prolongation and Torsades de Pointes in patients infected with human immunodeficiency virus and treated with methadone. Am J Cardiol. 2003;92(8): 995–997. 12. Cruciani RA. Methadone: to ECG or not to ECG. . .That is still the question. J Pain Symptom Manage. 2008;36(5):545–552. 13. Chugh SS, Socoteanu C, Reinier K, et al. A community-based evaluation of sudden death associated with therapeutic levels of methadone. Am J Med. 2008;121(1):66–71. 14. Piguet V, Desmeules J, Ehret G, Stoller R, Dayer P. QT interval prolongation in patients on methadone with concomitant drugs. J Clin Psychopharmacol. 2004;24(4):446–448.

15. Krantz MJ, Rowan SB, Mehler PS. Cocaine-related torsade de pointes in a methadone maintenance patient. J Addict Dis. 2005;24(1): 53–60. 16. Ehret GB, Voide C, Gex-Fabry M, et al. Drug-induced long QT syndrome in injection drug users receiving methadone: high frequency in hospitalized patients and risk factors. Arch Intern Med. 2006;166(12):1280. 17. Kapur BM, Hutson JR, Chibber T, Luk A, Selby P. Methadone: a review of drug-drug and pathophysiological interactions. Crit Rev Clin Lab Sci. 2011;48(4): 171–195. 18. Benmebarek M, Devaud C, Gex-Fabry M, et al. Effects of grapefruit juice on the pharmacokinetics of the enantiomers of methadone. Clin Pharmacol Ther. 2004; 76(1):55–63. 19. Wynn G, Cozza K, Zapor M, Wortmann G, Armstrong S. Medpsych drug-drug interactions update. Antiretrovirals, part III: antiretrovirals and drugs of abuse. Psychosomatics. 2005;46(1):79–87. 20. Gruber V, McCance-Katz E. Methadone, buprenorphine, and street drug interactions with antiretroviral medications. Curr HIV/AIDS Rep. 2010;7(3): 152–160. 21. Kakuda TN, Schöller-Gyüre M, Hoetelmans RM. Pharmacokinetic interactions

Chapter 72: Methadone and treatment of chronic pain

between etravirine and nonantiretroviral drugs. Clin Pharmacokinetics. 2011;50(1): 25–39. 22. Kharasch E, Walker A, Whittington D, Hoffer C, Bedynek P. Methadone metabolism and clearance are induced by nelfinavir despite inhibition of cytochrome P4503A (CYP3A) activity. Drug Alcohol Depend. 2009;101(3):158–168. 23. Lüthi B, Huttner A, Speck R, Mueller N. Methadone-induced Torsade de pointes after stopping lopinavir–ritonavir. Eur J Clin Microbiol Infect Dis. 2007;26(5): 367–369. 24. McCance-Katz E, Rainey P, Jatlow P, Friedland G. Methadone effects on zidovudine disposition (AIDS Clinical Trials Group 262). J Acquir Immune Defic Syndr Hum Retrovirol. 1998;18(5):435–443. 25. Doehring A, von Hentig N, Graff J, et al. Genetic variants altering dopamine D2 receptor expression or function modulate the risk of opiate addiction and the dosage requirements of methadone substitution. Pharmacogenet Genomics. 2009;19(6):407–414. 26. Fonseca F, de la Torre R, Díaz L, et al. Contribution of cytochrome P450 and ABCB1 genetic variability on methadone pharmacokinetics, dose requirements, and response. PLoS One. 2011;6(5):e19527. 27. Chen C-H, Wang S-C, Tsou H-H, et al. Genetic polymorphisms in CYP3A4 are associated with withdrawal symptoms and adverse reactions in methadone maintenance patients. Pharmacogenomics. 2011;12(10): 1397–1406. 28. Coller JK, Barratt DT, Dahlen K, Loennechen MH, Somogyi AA. ABCB1 genetic variability and methadone dosage requirements in opioid-dependent individuals. Clin Pharmacol Ther. 2006;80(6): 682–690.

29. Codd EE, Shank RP, Schupsky JJ, Raffa RB. Serotonin and norepinephrine uptake inhibiting activity of centrally acting analgesics: structural determinants and role in antinociception. J Pharmacol Exp Ther. 1995;274(3):1263–1270. 30. Sotgiu ML, Valente M, Storchi R, Caramenti G, Biella GE. Cooperative N-methyl-d-aspartate (NMDA) receptor antagonism and μ-opioid receptor agonism mediate the methadone inhibition of the spinal neuron pain-related hyperactivity in a rat model of neuropathic pain. Pharmacol Res. 2009;60(4):284–290. 31. Kristensen K, Blemmer T, Angelo HR, et al. Stereoselective pharmacokinetics of methadone in chronic pain patients. Ther Drug Monitor. 1996;18(3): 221–227. 32. Ferrari A, Coccia CPR, Bertolini A, Sternieri E. Methadone: metabolism, pharmacokinetics and interactions. Pharmacol Res. 2004;50(6):551–559. 33. Preston KL, Epstein DH, Davoudzadeh D, Huestis MA. Methadone and metabolite urine concentrations in patients maintained on methadone. J Anal Toxicol. 2003;27(6):332–341.

and adverse clinical impact of human immunodeficiency virusassociated sensory neuropathy in the era of combination antiretroviral therapy: the CHARTER Study. Arch Neurol. 2010;67(5):552. 38. Finnerup NB, Otto M, McQuay HJ, Jensen TS, Sindrup SH. Algorithm for neuropathic pain treatment: an evidence based proposal. Pain. 2005;118(3): 289–305. 39. Moulin D, Palma D, Watling C, Schulz V. Methadone in the management of intractable neuropathic noncancer pain. Can J Neurol Sci. 2005;32(3): 340–343. 40. Altier N, Dion D, Boulanger A, Choinière M. Management of chronic neuropathic pain with methadone: a review of 13 cases. Clin J Pain. 2005;21(4): 364–369. 41. Fingerhut LA. Increases in poisoning and methadone-related deaths: United States, 1999–2005. National Center for Health Statistics. Health E-Stat. 2008. 42. Hall AJ, Logan JE, Toblin RL, et al. Patterns of abuse among unintentional pharmaceutical overdose fatalities. JAMA. 2008;300(22):2613–2620.

34. Larue F, Fontaine A, Colleau SM. Underestimation and undertreatment of pain in HIV disease: multicentre study. BMJ. 1997;314(7073):23–28.

43. Ballantyne JC, LaForge KS. Opioid dependence and addiction during opioid treatment of chronic pain. Pain. 2007;129(3): 235–255.

35. Tsao J, Dobalian A, Stein J. Illness burden mediates the relationship between pain and illicit drug use in persons living with HIV. Pain. 2005;119(1–3):124–132.

44. Preskorn SH, Greenblatt DJ, Flockhart D, et al. Comparison of duloxetine, escitalopram, and sertraline effects on cytochrome P450 2D6 function in healthy volunteers. J Clin Psychopharmacol. 2007;27(1): 28–34.

36. Bouhassira D, Lantéri-Minet M, Attal N, Laurent B, Touboul C. Prevalence of chronic pain with neuropathic characteristics in the general population. Pain. 2008;136(3):380–387. 37. Ellis RJ, Rosario D, Clifford DB, et al. Continued high prevalence

45. Kotlyar M, Brauer LH, Tracy TS, et al. Inhibition of CYP2D6 activity by bupropion. J Clin Psychopharmacol. 2005;25(3): 226–229.

503

Section 7 Chapter

73

Special Topics

Drug testing Steven Michael Lampert, Richard D. Urman, and Alan David Kaye

Case study A 45-year-old unemployed construction worker presents to the pain center with a complaint of chronic axial low back pain for 5 years after a work-related injury. An MRI of the lumbar spine ordered by his primary care physician is remarkable for L3-L4 and L4-L5 bulging discs and some minor facet arthropathy without any other significant findings. He denies any recent history of drug abuse but he admits to smoking 2 packs of cigarettes daily and drinking a 6-pack of beer per week. On further questioning, he admits to cocaine and marijuana use but “nothing in the last 5 years.” He recently moved from a neighboring town where his primary care doctor was managing his pain with oxycodone 10 mg every 4 hours as needed for pain. He reports that his pain improved 30–40% with the oxycodone from his baseline pain level. His new primary care doctor has referred him for recommendations regarding chronic opioid management. In his evaluation, you decide to order a urine drug test.

1. Whyisitimportanttododrugtesting in patients with chronic pain managed with opioids? Urine drug testing is often part of a standard opioid agreement. Urine drug testing is done to document compliance with medication management as well as screen for concomitant use of illicit drugs of abuse. Recent well publicized reports have clearly documented the growing prescription opioid epidemic and associated increase in morbidity and mortality including opioid-related death.[1] With this in mind, it is incumbent on the physician prescribing opioids to monitor for compliance and drugs of abuse. Furthermore, published data show that physicians are not

able to consistently and accurately determine which patients have problems with compliance or addiction based on behaviors and self-report alone.[2] A busy practitioner is frequently faced with aberrant results of urine drug testing, triggering an in depth discussion with the patient about implications for further continuing care. Ideally, the expectations and consequences of aberrant urine drug testing are thoroughly discussed at the onset of the doctor– patient relationship and clearly spelt out in the opioid agreement before opioid management is initiated.

2. What methods are available for urine drug testing? EIA (enzyme-mediated immunoassay) is often used as an initial screening test. It is a quick, relatively easy and inexpensive test to perform. Although somewhat sensitive, the test is not as sensitive or specific as other methods. For example, the test may be positive for opioids but not detect the specific opioid prescribed, especially synthetic opioids like fentanyl, oxycodone, and hydrocodone.[3] Liquid chromatography/tandem mass spectrometry (LC/MS-MS) or gas chromatography/mass spectrometry (GC/MS) are highly sensitive and specific techniques used to accurately identify and quantify specific drugs and their metabolites. They can be used to confirm an unexpected positive or negative EIA result.

3. How accurate are the urine drug tests? Analyzing almost 1000 samples, Manchikanti et al compared the diagnostic accuracy of immunoassay with liquid chromatography tandem mass spectrometry

Case Studies in Pain Management, ed. Alan David Kaye and Rinoo V. Shah. Published by Cambridge University Press. © Cambridge University Press 2015.

504

Chapter 73: Drug testing

Table 73.1. Drug testing sensitivity, specificity, and efficiency

Sensitivity/false negative rate (%)

Specificity/false positive rate (%)

Test efficiency (agreement) (%)

Morphine, hydrocodone, codeine, hydromorphone

92.2/7.8

93.1/6.9

92.5

Oxycodone

75.4/24.6

92.3/7.7

90

Methadone

96.1/3.9

98.8/1.2

98.7

Marijuana

90.9/9.1

98.0/2

97.8

Cocaine

25.0/75

100/0

99.4

Methamphetamines

40/60

98.8/1.2

98.5

Amphetamines

47/53

99.1/0.9

98.2

Table 73.3. Additional drug sensitivity in days

Table 73.2. Drug sensitivity in days

Hydrocodone

1–2 days

Oxycodone

1–3 days

Morphine

3–4 days

Methadone

5–10 days

Hydromorphone

1–2 days

Meperidine

1–2 days

Codeine

1–3 days

Heroin

1–3 days

Benzodiazepines

up to 30 days

Barbiturates

2–10 days

and found a high degree of agreement (90% for oxycodone and as much as 99% for cocaine) between the techniques.[4] Their findings are summarized in Table 73.1.

4. How long will a urine drug test remain positive for opioids, benzodiazepines, and barbiturates? Table 73.2 gives the times for how long a urine drug test remains positive for various opioids, benzodiazepines, and barbiturates.[5]

5. How long will a urine drug test remain positive for drugs of abuse? Table 73.3 gives the times for how long a urine drug test remains positive for drugs of abuse.[5]

Marijuana

1–3 days for casual use, 11 weeks for chronic use

Cocaine

1–3 days

Amphetamine

2–4 days

Methamphetamine

2–4 days

Heroin

1–3 days

Phencyclidine

2–8 days

6. This patient tells you he ran out of his medication and last took his oxycodone 2 days ago. Would you expect his urine drug test to be positive or negative? The test should be positive. Urine drug testing for oxycodone would be positive 1–3 days after the last dose of this medication.[5]

7. Can urine drug testing tell you how much of a given drug the patient is currently taking? No, even quantitative urine drug testing by chromatography/mass spectrometry cannot reliably indicate how much actual drug the patient is taking.[3] There is significant individual variability in how the body absorbs, distributes, and metabolizes a given drug. As an example, morphine absorption shows 2.5-fold variability between oral, buccal, and intramuscular routes. Furthermore, first pass metabolism can result in significant variability in the amount of a given drug reaching the systemic circulation.

505

Chapter 73: Drug testing

Table 73.4.

Test or drug category

Drugs that may cause a false positive result

Amphetamines

Amantadine (Symmetrel), bupropion (Wellbutrin), chlorpromazine, desipramine (Norpramin), fluoxetine (Prozac), L-methamphetamine (in nasal decongestants), labetalol (Normodyne), methylphenidate (Ritalin), phentermine, phenylephrine, phenylpropanolamine, promethazine (Phenergan), pseudoephedrine, ranitidine (Zantac), thioridazine, trazodone (Desyrel)

Benzodiazepines

Oxaprozin (Daypro), sertraline (Zoloft)

Cocaine

Topical anesthetics containing cocaine

Opiates

Dextromethorphan, diphenhydramine (Benadryl), fluoroquinolones (ciprofloxacin, levofloxacin, and ofloxacin), poppy seeds, quinine, rifampin, Verapamil (in methadone assay)

Phencyclidine

Dextromethorphan, diphenhydramine, ibuprofen, imipramine (Tofranil), ketamine (Ketalar), meperidine (Demerol), thioridazine, tramadol (Ultram), venlafaxine (Effexor)

Tetrahydrocannabinol

Dronabinol (Marinol), non-steroidal anti-inflammatory drugs (ibuprofen, naproxen (Naprosyn), and sulindac (Clinoril)), proton pump inhibitors (pantoprazole (Protonix))

8. What medications and substances can trigger a false positive urine drug test result? Immunoassay techniques are subject to crossreactivity. Drugs that may cause a false positive result are listed in Table 73.4.[6]

9. How reliable is a physician’s history or “gut instinct” in predicting aberrant drug use? Combining patient history with urine drug testing is more reliable than either technique alone. In a university-based pain clinic study of 122 patients with non-cancer pain treated with opioids, 21% of patients with no behavioral issues had a positive urine drug screen for either an illicit drug or a non-prescribed controlled substance.[2] Behavioral issues were defined as “lost or stolen prescriptions, consumption in excess of prescribed dosage, visits without appointments, multiple drug intolerances and allergies, and frequent telephone calls.”[2] A study by Michna and colleagues found that “gut instinct” was a poor predictor of an abnormal urine drug test.[7] In a university-based pain clinic, retrospective analysis of data from 470 patients found no relationship between urine drug test result and sex, pain

506

site, type of opioid dose or number of opioids prescribed, or prescribing physician.[7] The only correlation found in this population was that younger patients were found to have used illicit substances (mean age 44.1 ± 9.3) and/or additional non-prescribed opioids (mean age 44 ±10) more often than older patients.[7]

10. What are the most common inappropriate drugs found on a urine drug testing? Marijuana is the most common illicit drug found on urine drug testing, followed by cocaine. Opioids are the most common non-prescribed controlled drug found on urine drug testing followed by benzodiazepines, barbiturates, and ethanol.[2]

11. Is drug testing mandatory or required by law? Although drug testing may be required in specific occupations like mass transit, airline, trucking, or sports participation, there is no current federal requirement to screen chronic pain patients on long-term opioids.[8] States, like New York, for example, have published recommendations that when long-term opioids are prescribed, a treatment agreement should specify that urine drug testing will be done.

Chapter 73: Drug testing

12. What would you do if the urine drug test is unexpectedly positive or unexpectedly negative? An unexpected positive or negative result should trigger a discussion with the patient about the results. How to handle different scenarios can be laid out in the opioid agreement so that the patient knows the boundaries and expected consequences of a given result before treatment is initiated or continued. There are no specific guidelines on how to handle an unexpected positive or negative urine drug test. However, from our experience, a positive test for cocaine, amphetamines, heroin or LSD would trigger a physician to consider immediate discontinuation of opioid prescribing for the patient and even a potential discharge from the practice based on the circumstances. Furthermore, due to the concern for noncompliance or even diversion, from our experience, discontinuation of opioid prescribing should also be considered for a patient that is prescribed regular opioids for chronic non-cancer pain who tests negative for the prescribed opioid. In either case, another option would be to discontinue the opioid and refer the patient to a substance abuse specialist.

References 1.

2.

3.

4.

Centers for Disease Control and Prevention. Vital signs: overdoses of prescription opioid pain relievers – United States, 1999–2008. MMWR Morb Mortal Wkly Rep. 2011;60:1487–1492. Katz NP, Sherburne S, Beach M, et al. Behavioral monitoring and urine toxicology testing in patients receiving long-term opioid therapy. Anesth Analg. 2003;97:1097–1102. Nafziger AN, Bertino JS Jr. Utility and application of urine drug testing in chronic pain management with opioids. Clin J Pain. 2009;25(1):73–79.

5.

6.

There are times when a patient tests positive for a different or an additional opioid that was not prescribed. Again, a discussion with the patient is indicated in order to attempt to find the source of the unexpected result. Based on this discussion and the opioid agreement in place, a reasonable plan can be made. For example, a patient consistently compliant with the opioid agreement that took a leftover tablet of hydrocodone and admits to it may be given a “warning” and required to increase the frequency of urine drug testing for a given period of time. However, for a patient that consistently tests positive for non-prescribed opioids, discontinuation of opioid therapy may be indicated as well as referral to a substance abuse specialist. Other scenarios are less clear. For example, some states like Colorado have legalized the use of marijuana. Although the patient may not have broken state law, patients that test positive for marijuana and alcohol are clearly at higher risk for opioid addiction and adverse events while on prescription opioid medications. A physician that continues to prescribe opioids to this patient risks contributing to the increasing epidemic of opioid-related death.[1,9]

Manchikanti L, Malla Y, Wargo BW, Fellows B. Comparative evaluation of the accuracy of immunoassay with liquid chromatography tandem mass spectrometry (LC/MS/MS) of urine drug testing (UDT) opioids and illicit drugs in chronic pain patients. Pain Physician. 2011;14 (2):175–187. US Department of Health and Human Services. American Society of Interventional Pain Physicians (ASIPP) Guidelines for Responsible Opioid Prescribing in Chronic Non-Cancer Pain: Part 2 – Guidance. Pain Physician. 2012;15:S67–S116. Standridge JB, Adams SM, Zotos AP. Urine drug screening: a

valuable office procedure. Am Fam Physician. 2010;81(5):635–640. 7.

Michna E, Jamison RN, Pham LD, et al. Urine toxicology screening among chronic pain patients on opioid therapy: frequency and predictability of abnormal findings. Clin J Pain. 2007;23 (2):173–179.

8.

Clinical Practice Guidance Number 2012.3: Guidance on Urine Drug Testing April 2012. http://www.oasas.ny.gov/AdMed/ recommend/guide3test.cfm.

9.

Federation of State Medical Boards. Model Policy on the Use of Opioid Analgesics in the Treatment of Chronic Pain, July 2013.

507

Index

Locators in bold refer to figures/tables AAN. see American Academy of Neurology AAOD (anesthesia-assisted opiate detoxification) 447–453 ABC (aneurysmal bone cyst) 162 abdominal binders, dural puncture headache 437 abdominal pain children 386–387 pancreatitis 253–255, 253–258 pregnancy 283 aberrant drug use screening. see drug screens aberrant medication-taking behavior, definition 400–401 abuse, definition 400–401 accidental overdose. see overdose accidents. see motor vehicle collisions acetaldehyde 64–65 acetaminophen brachial plexopathy 33 cancer pain 295–344 drug screens 414–415 lumbar spinal stenosis 169 migraine headache 301 pancreatitis treatment 256 postpartum pain 293 pregnancy pain 285, 285–286 tension headaches 313–314, 314 thoracic outlet syndrome 107 total knee arthroplasty 492, 494 acetyl-L-carnitine 67 active range of motion testing (AROM), ankle pain 236 activities of daily living (ADL) 69 acupuncture ankle pain 77, 237 chronic pelvic pain 268 HIV neuropathy 77 low back pain 392–393 migraine headache 304 phantom limb pain 40–41 postpartum pain 292–293 post-thoracotomy pain 44 pregnancy pain 285 spinal cord injury pain 19

508

temporomandibular joint disorder 337 tension headaches 313 thoracic spine pain 129 acute myeloid leukemia (AML) 359 acute pain management, patientcontrolled analgesia 476–480, 482–488, 484, 485 acute pancreatitis 253, 253. see also pancreatitis ADA (Americans with Disabilities Act) 388 Adams classification system, discography 147 addiction aberrant drug use 408 definitions 400–401, 408 neurobiological processes underlying 401–402 see also substance abuse adductor canal block, total knee arthroplasty 493 adherence monitoring, aberrant drug use 412 adhesiolysis 169–170, 177 adhesive capsulitis 24–25, 368 adiposis dolorosa. see Dercum’s disease adjuvant analgesics cancer pain 344, 344 epidural catheters 488 hematological cancer 363–364 pancreatic cancer pain 355 adjuvant therapy, pancreatitis 256 advanced glycation end products (AGEs) 58 aerobic exercise. see exercise agomelatine 462 AGS (American Geriatric Society) 10–11 AKA. see artery of Adamkiewicz alcohol and insomnia 461–463 pancreatitis 253 spasticity 233 urine drug testing 414, 506–507 alcohol-induced neuropathy (AIN) 70

clinical features 65 differential diagnosis 65 epidemiology/demographics 64 ethics 69–70 examination/work-up 66–67 pathophysiology 64–65 treatment options and complications 69 allodynia, poststroke pain 22 alpha-2 adrenergics brachial plexopathy 36 cancer pain 344 chronic pelvic pain 269 complex regional pain syndrome 48 spasticity 233 thoracic outlet syndrome 107 total knee arthroplasty 492, 494 see also clonidine; tizanidine alpha-2 delta ligands anxiety/depression 12 older patients, treatment options 9–11 phantom limb pain 40 postherpetic neuralgia 2–4, 2–15, 7–8, 7–8, 12–13 side effects 7–8, 7–8 substance abuse 12 see also gabapentin; pregabalin alpha-lipoic acid 57–58, 67 alternative medicine 390. see also complementary and alternative medicine American Academy of Neurology (AAN) 2–4, 2–15 American Association of Neurologic Surgeons 147 American College of Rheumatology 214 American Geriatric Society (AGS) 10–11 American Medical Association (AMA) 453 American Sign Language (ASL) 388–389 American Society of Addiction Medicine (ASAM) 453

Index

American Society of Anesthesiologists (ASA) 491–492 American Society for Interventional Pain Physicians (ASIPP) 412 American Society of Regional Anesthesia 484 Americans with Disabilities Act (ADA) 388 amitriptyline alcohol-induced neuropathy 68, 269 brachial plexopathy 33 headache 218–302, 303, 313–314, 314 hematological cancer 363 HIV neuropathy 76 insomnia 462 phantom limb pain 40, 41 postherpetic neuralgia 4, 7–8, 11, 13 postpartum pain 293–294 poststroke pain 25, 26 pregnancy pain 285 side effects 3–7 thoracic outlet syndrome 107 AML (acute myeloid leukemia) 359 amlodipine 302, 303 amphetamines, drug testing 410–416, 505–507 amputation 38. see also phantom limb pain anatomical details ankle 235–236 Baastrup’s disease 159–160, 160 bowel 468 brachial plexus 30 cluster headaches 308–309 dural puncture 436 epidural space 484, 485 facet joints 117–118, 117–138, 138–139 intervertebral discs 144–145 knee 243 lateral epicondylosis 240–241 lumbar spinal stenosis 165–166 male pelvis 266 pelvic viscera/rectum 275 sacroiliac joint 197 sacrum 202–203 sphenopalatine ganglion 329–330 spinal cord 424–425, 429, 430 temporomandibular joint 333–335, 335–337 thoracic region 123 anesthesia-assisted opiate detoxification (AAOD) 447–453 aneurysmal bone cyst (ABC) 162 ankle pain 235–239, 238 ankle sprain, differential diagnosis 235 ankylosing spondylitis (AS) 124–125, 125–126, 161–162, 214, 336

antalgic gait analysis 245 anterior radicular medullary arteries (ARMAs) 424–425 anterior scalene muscle (ASM) block 106 anterior spinal artery (ASA) 424–425 antero-talofibular ligament (ATFL) 235–236 antibiotics, chronic pelvic pain 268–269 anticholinergic drug side effects 7–8, 58–59, 68, 269 anticholinergic poisoning 440 anticonvulsants alcohol-induced neuropathy 67–68 brachial plexopathy 36 cancer pain 344 chronic pelvic pain 269 complex regional pain syndrome 48 diabetic neuropathy 58 HIV neuropathy 75–76 migraine headache 302, 303 phantom limb pain 40 pregnancy pain 286 thoracic outlet syndrome 107 see also carbamazepine; gabapentin; lamotrigine; pregabalin; topiramate; zonisamide antidepressants alcohol-induced neuropathy 68, 269 brachial plexopathy 36 cancer pain 344 chronic pain syndrome 373 diabetic neuropathy 58–59 hematological cancer 363 HIV neuropathy 76 insomnia 462 methadone prescription 502 phantom limb pain 40 postpartum pain 293–294 pregnancy pain 286 temporomandibular joint disorders 338 thoracic outlet syndrome 107 thoracic spine pain 128 total knee arthroplasty 492 see also SNRIs; SSRIs; tricyclic antidepressants antiepileptics. see anticonvulsants anti-inflammatories. see corticosteroids; nonsteroidal anti-inflammatory agents antioxidants, pancreatitis treatment 256 antiretroviral therapy 73, 76 anxiety/depression brachial plexopathy 33 cervicogenic headache 85 chronic pain syndrome 373–376

chronic pelvic pain 268 diabetic neuropathy 60 and insomnia 459 methadone prescription 502 myofascial pain syndrome 226–227 phantom limb pain 39, 41–42 postherpetic neuralgia 9–10, 12 poststroke pain 26–27 spinal cord injury pain 20 substance abuse 399–400 temporomandibular joint disorders 335 tension headaches 313 aquatherapy 284 ARIC (large Atherosclerosis Risk in Communities) study 299 ARM (arteria radicularis magna). see artery of Adamkiewicz ARMAs (anterior radicular medullary arteries) 424–425 AROM (active range of motion) testing, ankle pain 236 aromatase inhibitors 263–264 artery of Adamkiewicz (AKA) 424–425, 429–431, 432 arthritis. see rheumatoid arthritis arthrocentesis, temporomandibular joint disorders 338 ARV (antiretroviral therapy) 73, 76 AS. see ankylosing spondylitis ASA (anterior spinal artery) 424–425 ASA (American Society of Anesthesiologists) 491–492 ASAM (American Society of Addiction Medicine) 453 ASIPP (American Society for Interventional Pain Physicians) 412 ASL (American Sign Language) 388–389 aspirin 301, 313–314, 314, 319, 396 assessment instruments, opioid use for chronic pain 404–405 Association for the Study of Pain (IASP) 47 ATFL (antero-talofibular ligament) 235–236 atropine 327 attorneys, workers’ compensation system 224 aura migraine headache 297–299, 302 autogenic training (AT) 393 autoimmune disorders 215. see also fibromyalgia autonomic arousal fibromyalgia 216 postoperative analgesia 482–483

509

Index

autonomic dysreflexia, spinal cord injury pain 18–19 autonomic function testing diabetic neuropathy 53 HIV neuropathy 75 autonomic neuropathy 55 autonomy, respect for 403 axial back pain 138, 138 axial neck pain 116 Baastrup’s disease 159–163, 160, 161–162 back pain. see low back pain baclofen brachial plexopathy 36 intrathecal drug delivery 178–179 phantom limb pain 40 spasticity 233–234 spinal cord injury pain 18 thoracic outlet syndrome 107 trigeminal neuralgia 319 bacterial prostatitis 267 balloon kyphoplasty. see kyphoplasty balneotherapy 395 barbiturates, urine drug testing 505–507 BDNF (brain-derived neurotrophic factor) 216 Beers Criteria for Potentially Inappropriate Medications in Older Adults 11 behavioral therapy low back pain 395 migraine headache 304 postlaminectomy pain syndrome 179 beneficence, principle of 402–403 benfotiamine 69 benzodiazepines drug testing 410–416, 505–507 fibromyalgia 218 insomnia 462 postpartum pain 294 pregnancy pain 286 spasticity 233 see also diazepam Bertolotti’s syndrome 155–157 beta blockers and insomnia 461 migraine headache 302, 303 see also propranolol biaculoplasty 150 bicycle test 168, 172 Bieri Faces scale 383, 383 biofeedback chronic pain syndrome 376–377 migraine headache 304 phantom limb pain 40–41 postpartum pain 293

510

spinal cord injury pain 19 tension headaches 313 biofield therapies 394–395 biological effects, ionizing radiation 419, 419–422 biomarkers, knee osteoarthritis 246–247 biphosphonates 363 bipolar RFA (bRFA) 199, 210–211. see also radiofrequency ablation birth, postpartum pain 291–292 bisacodyl 470 bisphosphonates 48 bladder dysfunction 152, 267 blood sugar control, diabetic neuropathy 57 bone marrow transplantation (BMT) 361–362 bone metastases adjuvant analgesics 344 biophosphonates 363 hematological cancer 360 non-small cell lung cancer 207–212, 209, 211 pancreatic cancer 341 postpartum pain 290 bone mineral density (BMD) 183–184 bone scans. see nuclear bone scans bone scintigraphy 203 botulinum toxin cervicogenic headache/occipital neuralgia 91–92 chronic rectal pain 280 migraine headache 304 phantom limb pain 40–41 postmastectomy lymphedema 371 poststroke pain 25 spasticity 233 spinal cord injury pain 18 temporomandibular joint disorders 338 trigeminal neuralgia 319 bowel anatomy 468 opioid receptors 468, 468 BP. see brachial plexopathy BPNS (Brief Peripheral Neuropathy Screen) 74 braces brachial plexopathy 33 knee osteoarthritis 247 lateral epicondylosis 241 lumbar disc herniation 134 lumbar spinal stenosis 169 postpartum pain 292 pregnancy pain 285 thoracic spine pain 128 brachial plexopathy (BP) 30–37, 31, 36, 104, 109

brachial plexus 102–103 bradycardia 325–326 brain fog 216 brain regions implicated addiction 401–402 poststroke pain 24 brain tumors, differential diagnosis 309 breakthrough pain cancer pain 346 opioids, safe use 405 patient-controlled analgesia 478–479 total knee arthroplasty 493 breast cancer, metastatic 207, 367–371 breastfeeding 293–295 Brief Peripheral Neuropathy Screen (BPNS) 74 bronchodilators 461 Broström-Gould repair 239 bruxism 335 Budapest Consensus, complex regional pain syndrome 47 bupivacaine epidural catheters 487 intrathecal drug delivery 178–179 postpartum pain 293 buprenorphine 442–446, 451–452 burst compression fractures 182, 182–183 butalbital-containing medications 301, 301 buttock pain 168 CAD (coronary artery disease) 300. see also cardiovascular function caffeine dural puncture 436–437, 486 and insomnia 461 migraine headache 298, 301 pregnancy 285 tension headaches 314 calcitonin 41, 48, 169 calcium channel blockers and insomnia 461 migraine headache 302, 303 total knee arthroplasty 494 see also amlodipine; verapamil calcium pyrophosphate dihydrate disease (CPPD) 160, 244–246 CAM. see complementary and alternative medicine Canadian Pain Society (CPS) 2–3, 2–15 cancer pain 341–342, 349, 352–355 analgesic routes other than oral 346 analgesics 343–346, 344, 345 breakthrough pain 346 differential diagnosis 336

Index

epidemiology/demographics 342 glossopharyngeal neuralgia 326 male pelvic pain 267 nerve blocks 341–348, 347–349 non-drug therapies 347 risk factors for development 342–343 side effects of opioids 346–347, 347 WHO analgesic ladder 343 see also metastases cannabinoids alcohol-induced neuropathy 69–70 enzyme immunoassay 410–416 hematological cancer 364 HIV neuropathy 76–77 motor vehicle collision victim 412–414, 414 spinal cord injury pain 19 urine drug testing 414, 505–507 capsaicin, topical alcohol-induced neuropathy 68, 269 diabetic neuropathy 59 HIV neuropathy 76 postherpetic neuralgia 2, 2–15, 4–5, 7–8 carbamazepine alcohol-induced neuropathy 67–68 brachial plexopathy 36 diabetic neuropathy 58 phantom limb pain 40 poststroke pain 26 pregnancy pain 285 thoracic outlet syndrome 107 trigeminal neuralgia 319 cardiovascular function anesthesia-assisted opiate detoxification 447–448, 450 differential diagnosis 336 methadone prescription 499–500 postherpetic neuralgia 9–10, 12–13 postoperative analgesia 482–483 carpel tunnel syndrome 31–32 CART. see Communication Access Realtime Translation Castellvi classification, Bertolotti’s syndrome 156 catastrophizing, myofascial pain syndrome 226–227 catheter migration 486–487. see also epidural catheters cauda equine syndrome 152 caudal epidural injections, postlaminectomy pain 176 CBT. see cognitive behavioral therapy CDT. see complex decongestive therapy celecoxib hematological cancer 363 phantom limb pain 40

thoracic outlet syndrome 107 celiac plexus block cancer pain 348–349 pancreatic cancer pain 353–354 pancreatitis treatment 257 cement augmentation, sacral insufficiency fracture 204–205. see also vertebral augmentation central poststroke pain (CPSP) 22–27, 23–29 central sensitization, poststroke pain 24, 27 cerebral blood flow, fibromyalgia 216 cerebral palsy pain 383–384 cerebrospinal fluid (CSF) leakage. see dural puncture cerebrovascular disease 96 cervical disc damage 116 cervical facet joints 182 anatomy 117, 117–118 and axial neck injuries 89–119, 116–119 treatment outcomes 121 treatments 119–121 cervical joint subluxation 82 cervical radiculopathy 31–32, 32–34, 104, 109–114, 110, 240 cervical spinal cord stimulation 50 cervical stenosis/myelopathy 95–96 clinical features 96 definition 96–99 diagnostic criteria 97–475 differential diagnosis 96, 214 epidemiology/demographics 96–99 examination/work-up 97–99 imaging studies 97–99 natural history 97–99 ossification of posterior longitudinal ligament 99 pathophysiology 98 and rheumatoid arthritis 99 risk factors for development 97–99 surgery 98–99 treatments, non-surgical 98–99 cervical traction, cervical radiculopathy 112 cervicogenic headache clinical assessment 83–85 clinical features 82 convergence trigeminocervical nucleus convergence theory 85, 86 diagnostic criteria 85–87, 89 differential diagnosis 81–82 pain generators/referral patterns 87 subsets/types 86–89, 88 symptoms 85–86 treatments 89–93, 90, 91, 92

cesarean delivery, postpartum pain 291 chemodenervation, spinal cord injury pain 18–19 chemotherapy-induced pain/ peripheral neuropathy hematological cancer 361–362 pancreatic cancer pain 353 peripheral neuropathy 342 chicken pox 1 children, pain in. see pediatric pain chiropractic manipulation cervical radiculopathy 113 cervicogenic headache 84 low back pain 394 thoracic spine pain 129 chlorpromazine, serotonin syndrome 441 chronic ankle sprain, differential diagnosis 235 chronic low back. see low back pain chronic myelogenous leukemia (CML) 359 chronic pain syndrome 373–376 hearing impaired patient 388–389 and insomnia 459–464 methadone prescription 498–502 chronic pancreatitis 253, 253. see also pancreatitis chronic pelvic pain (CPP) 261, 266 classification systems 267 differential diagnosis 267 endometrial fibrosis 261–265 epidemiology/demographics 267 examination/work-up 267–268 NIH chronic prostatitis symptom index 272 postpartum pain 273–291 treatments 268–273 chronic prostatitis symptom index (NIH-CPSI) 272 chronic rectal pain 275–280, 278, 279 circadian rhythm disorders 460, 463 circumferential upper-extremity measurements, postmastectomy lymphedema 368–369 classification systems Bertolotti’s syndrome 156 cervicogenic headache 85 discography 147 interstitial cystitis 267 lumbar disc herniation 131–132, 132 lumbar spinal stenosis 166, 166–173 sacral insufficiency fracture 202–203 clicking sounds, temporomandibular joint disorders 337 clonazepam 233

511

Index

clonidine alcohol-induced neuropathy 68–269 anesthesia-assisted opiate detoxification 453 brachial plexopathy 36 diabetic neuropathy 59 epidural catheters 488 intrathecal drug delivery 178–179 phantom limb pain 40 spasticity 233 thoracic outlet syndrome 107 total knee arthroplasty 494 cluster headaches 307–310, 312, 329–330 CML (chronic myelogenous leukemia) 359 cocaine, drug testing 410–416, 505–507 coccygodynia 275–276, 290 Cochrane reviews acupuncture 393 anesthesia-assisted opiate detoxification 452 biofield therapies 394–395 cognitive therapy 395 manipulation 394 neuroreflexotherapy 396 postherpetic neuralgia 4 prolotherapy 395–396 yoga 394 codeine detection times, urine drug testing 414 postpartum pain 294 pregnancy pain 285 urine drug testing 505–507 coenzyme Q10, migraine headache 304 cognitive behavioral therapy (CBT) chronic pain syndrome 373–374 fibromyalgia 218–219 insomnia 463 temporomandibular joint disorders 337–338 tension headaches 313 cognitive impairment fibromyalgia 216 poststroke pain 25 cognitive therapy insomnia 463–464 low back pain 395 cohort analysis cluster headaches 310 interspinous spacers 171 neuraxial analgesia 290–291 cold treatment lateral epicondylosis 241 lumbar disc herniation 134 phantom limb pain 40–41

512

poststroke pain 25 post-thoracotomy pain 44 collimation, radiation exposure 418–420, 419 Communication Access Realtime Translation (CART) 389 compensation, injury 224–225 complementary and alternative medicine (CAM) definitions/user statistics 390–391 diabetic neuropathy 61 education 397 fibromyalgia 218–219 HIV neuropathy 77 low back pain 391–396 thoracic spine pain 129 types 392–396. see also specific CAMs by name complex decongestive therapy (CDT), postmastectomy lymphedema 369–370 complex regional pain syndrome (CRPS) 46–51, 49, 50 children, pain in 384–386 diagnostic criteria 47, 47 differential diagnosis 31–32, 32–34, 46, 104 compression/decompression, nerve and alcohol-induced neuropathy 65–73 and axial neck injuries 116–117 brachial plexopathy 33 cervical radiculopathy 110–111, 113–114 cervical stenosis/myelopathy 99 epidural lipomatosis 153 failed surgery for. see postlaminectomy pain syndrome glossopharyngeal neuralgia 325–326 and HIV neuropathy 73 lumbar spinal stenosis 165, 170–177 pancreatic cancer pain 353 trigeminal neuralgia 320–321 vertebral 124–125 compression fractures, lumbar spinal stenosis 168. see also lumbar compression fractures compression pump therapy, postmastectomy lymphedema 370 compression test, sacroiliac joint pain/ arthritis 197 compressive meningioma 230 computed tomography (CT) scanning Baastrup’s disease 161 Bertolotti’s syndrome 156 bone metastases 208, 209, 210 cervical radiculopathy 112 lumbar compression fracture 185 lumbar disc herniation 133

pancreatitis 254–255 sacral insufficiency fracture 203 sacroiliac joint pain/arthritis 198 trigeminal neuralgia 317 concentration meditation 393 conditioned insomnia 460 constipation, opioid-induced 467–472, 467–472 continuous epidural catheters, acute pain management 482–488, 484, 485 continuous peripheral catheters postoperative analgesia 491–495, 492, 493, 495 total knee arthroplasty 493 continuous peripheral nerve blocks (CPNB), total knee arthroplasty 494–495 contractures, spinal cord injury pain 17–18 contraindications/indications anesthesia-assisted opiate detoxification 447–448 epidural catheters 482, 488 neuraxial analgesia 355 patient-controlled analgesia 477–478 spinal manipulation 231 contrast discography 147 convergence trigeminocervical nucleus convergence theory, cervicogenic headache 85, 86 cooled radiofrequency ablation (cRFA) 199 coronary artery disease (CAD) 300. see also cardiovascular function cortical spreading depression (CSD) 297–298 corticosteroids ankle pain 237 brachial plexopathy 36 cancer pain 344 cervicogenic headache/occipital neuralgia 90 chronic pelvic pain 269 complex regional pain syndrome 48 facet joints, cervical 119 lateral epicondylosis 241 phantom limb pain 40, 40–41 postpartum pain 293 post-thoracotomy pain 44 pregnancy pain 287–288 thoracic outlet syndrome 107 total knee arthroplasty 492, 494 treatment complications 202 see also dexamethasone; methyl prednisone cortisol release, fibromyalgia 216 cortisone, pregnancy pain 285

Index

costochondritis (Tietze’s syndrome) 124–125, 126, 129 differential diagnosis 207 thoracic spine pain 124–125, 126 costovertebral joint pain 124–125, 129 counseling, migraine headache 299 COX-2 inhibitors pancreatitis treatment 256 postpartum pain 293 CPNB. see continuous peripheral nerve blocks CPP. see chronic pelvic pain CPPD. see calcium pyrophosphate dihydrate disease CPS (Canadian Pain Society) 2–3, 2–15 CPSP. see central poststroke pain cranial nerve palsies 438 craniocervical junction 84 cRFA (cooled radiofrequency ablation) 199 CRIES infant pain assessment instrument 380–381, 381 cross-disciplinary. see interdisciplinary management CRPS. see complex regional pain syndrome CSD (cortical spreading depression) 297–298 CT. see computed tomography scanning cyclobenzaprene brachial plexopathy 36 phantom limb pain 40 serotonin syndrome 439–441 thoracic outlet syndrome 107 cyproheptadine, serotonin syndrome 441 cystitis, interstitial. see chronic pelvic pain cytochrome P-450 system 363–364 buprenorphine 443–444 methadone 500, 502 urine drug screening 410 cytokines, HIV neuropathy 75 Danish study, cluster headaches 310 dantrolene spinal cord injury pain 18 spasticity 233 Daubert legal principle 83 DAWN (Drug Abuse Warning Network) 442 DCS (double crush syndrome) 105 DEA (Drug Enforcement Administration) 413–444 deaf patients 388–389 decompression, see compression/ decompression

deep brain stimulation (DBS) 60, 322–323 deep vein thrombus (DVT) 483 definitions addiction/withdrawal 447 complementary and alternative medicine 390 ethics 402 multi-modal analgesic protocol 491–492 opioids, substance abuse 400–401 patient-controlled analgesia 476 radiation exposure 417–418 serotonin syndrome 439 substance abuse 408 vasovagal response during pain procedures 473–474 degenerative disc disease, postpartum pain 290. see also lumbar spondylosis degenerative joint disease (DJD) 182, 243. see also knee osteoarthritis degenerative spondylolisthesis, pregnancy pain 282 Déjerine-Roussy syndrome 22. see also central poststroke pain delivery, postpartum pain 291–292 demographics. see epidemiology/ demographics demyelination, glossopharyngeal neuralgia 326 denervation complex regional pain syndrome 50 lumbar spondylosis 149 Denis classification, sacral insufficiency fracture 202–203 dental pain 318 dental problems, and emporomandibular joint disorders 336–337, 337 Department of Health and Human Services (DHHS), drug testing guidelines 411 dependence, drug, definition 447 depression. see anxiety/depression Dercum’s disease (adiposis dolorosa), thoracic spine pain 124–125 Derry review, postherpetic neuralgia 4 desipramine migraine headache 218–302 phantom limb pain 40 thoracic outlet syndrome 107 detection times, drug screens 413, 414 detoxification anesthesia assisted opiate 447–453 definitions 447 Devil’s claw, for low back pain 396

DEXA (dual energy x-ray absorptiometry), sacral insufficiency fracture 203–204 dexamethasone brachial plexopathy 36 phantom limb pain 40 pregnancy pain 287 thoracic outlet syndrome 107 total knee arthroplasty 494 dexmedetomidine, total knee arthroplasty 494 dextran, dural puncture headache 437 dextrose prolotherapy, knee osteoarthritis 247–249 DHE. see dihydroergotamine DHHS. see Department of Health and Human Services diabetes amyotropy 55 pancreatitis 254 postherpetic neuralgia treatment guidelines 12 diabetic neuropathy 62, 64 and alcohol-induced neuropathy 65–73 classification of peripheral nerve injuries 56 clinical manifestations 54–55 diagnostic criteria 54, 54 differential diagnosis 52–53 electromyography 55–56 epidemiology/demographics 52 examination/work-up 53–54 N-acetylcysteine 69 pathophysiology 56 pharmacological treatments 57–60 quality of life 62 treatment algorithm 57 treatment complications 61 treatments 56–61 Diagnosis, Intractability, Risk, Efficacy Score (DIRE), aberrant drug use 408–409 diagnostic criteria cervicogenic headache 85–87, 89 cluster headaches 307–308 coccygodynia 276 complex regional pain syndrome 47, 47 diabetic neuropathy 54, 54 fibromyalgia 214–216, 215–219 glossopharyngeal neuralgia 326–327 lumbar spinal stenosis 167 migraine headache 298–299 opioid dependence 443 proctalgia fugax 276 radiation proctitis 276–277

513

Index

diagnostic criteria (cont.) serotonin syndrome 300 somatoform disorder 217 temporomandibular joint disorders 333, 334–339, 336 tension headaches 312 diarrhea, anesthesia-assisted opiate detoxification 449 diary keeping insomnia 460, 463 migraine headache 298, 302 tension headaches 312–313 diazepam pregnancy pain 285 spasticity 233 spinal cord injury pain 18 diet. see nutritional support diffuse idiopathic skeletal hyperostosis (DISH) 124–125, 126, 126, 161 dihydroergotamine cluster headaches 309 migraine headache 300–301, 301 treatment complications 310 DIRE. see Diagnosis, Intractability, Risk, Efficacy Score disability cervicogenic headache 83–84 workers’ compensation system 224–225 disc disease, and temporomandibular joint disorders 334–339, 335–336 disc herniation, lumbar. see lumbar disc herniation discectomy, temporomandibular joint disorders 338 discogenic pain 144–150, 146, 148–320, 195–197 discography Bertolotti’s syndrome 157 lumbar disc herniation 134 lumbar spondylosis 146–148, 148 postlaminectomy pain syndrome 175 DISH. see diffuse idiopathic skeletal hyperostosis distal symmetric polyneuropathy (DSP) 72–73. see also HIV neuropathy distraction test, sacroiliac joint pain/ arthritis 197 diversion, definitions 400–401 DJD. see degenerative joint disease docusate, opioid-induced constipation 470 dopamine systems 216, 401–402 dorsal ramus syndrome (Maigne’s syndrome) 124–125, 127–128 double crush syndrome (DCS) 105

514

doxepin 462 DREZ (dorsal root entry zone) lesions 33 Drug Abuse Warning Network (DAWN) 442 drug-drug interactions, methadone prescription 500 Drug Enforcement Administration (DEA) 413–444 drug screens chronic low back pain case 504–507 fibromyalgia case 414–415 motor vehicle collision victim 412–414, 414 opioids, safe use for chronic pain 405 postlaminectomy pain syndrome 408–412, 410–416 see also substance abuse Dryden, John 402 dry-needling, ankle pain 237 DSP. see distal symmetric polyneuropathy dual energy x-ray absorptiometry. see DEXA duloxetine brachial plexopathy 33 migraine headache 303 phantom limb pain 40 pregnancy pain 286 spinal cord injury pain 19 duloxetine hydrochloride 59 DuPen catheter 354 dura mater, anatomy 436 dural puncture headache 175, 435–438, 486 dural tension test, lumbar disc herniation 133 DVT (deep vein thrombus) 483 Eagle syndrome 326, 326–327 ear infection, differential diagnosis 336 eating disorders 202 ECRB. see extensor carpi radialis brevis Edelsberg meta-analysis, postherpetic neuralgia 4 education, patient complementary and alternative medicine 397 complex regional pain syndrome 48 opioids, safe use for chronic pain 404 postmastectomy lymphedema 369–370 temporomandibular joint disorders 337 efavirenz, drug-drug interactions 500

EFNS. see European Federation of Neurological Societies elavil, serotonin syndrome 439–441 elbow, lateral epicondylosis 240–242, 242 electrodiagnostic testing, cervical radiculopathy 112 electromyography (EMG) alcohol-induced neuropathy 66 brachial plexopathy 33 cervical radiculopathy 112 diabetic neuropathy 55–56 HIV neuropathy 74 lumbar disc herniation 134 lumbar spinal stenosis 172 thoracic outlet syndrome 106 emesis, anesthesia-assisted opiate detoxification 449 endogenous opioids, fibromyalgia 216 endometrial fibrosis 261–265 endorphins, acupuncture 392 endoscopic treatment, pancreatitis 257–258 enemas, opioid-induced constipation 470–471 enkephalins, acupuncture 392 enzyme immunoassay (EIA), drug screening 409–410, 410–416, 504 epidemiology/demographics aberrant drug use 408–409 alcohol-induced neuropathy 64 ankle pain 235 Baastrup’s disease 159 brachial plexopathy 30–31 cancer pain 342 cervical radiculopathy 109 cervical stenosis/myelopathy 96–99 cluster headaches 307 complex regional pain syndrome 46 diabetic neuropathy 52 dural puncture 435 endometrial fibrosis 261 fibromyalgia 216 hearing impaired patient 388 HIV neuropathy 72 insomnia 459 interstitial cystitis 267 knee osteoarthritis 243 lateral epicondylosis 240 low back pain 390 migraine headache 297 opioid dependence 399, 442 opioid-induced constipation 467–468 pancreatitis 253 phantom limb pain 38 postmastectomy lymphedema 367 poststroke pain 22–23, 23–29, 27 Scheuermann’s kyphosis 127

Index

spinal cord injury pain 16 temporomandibular joint disorders 336 tension headaches 312 thoracic outlet syndrome 102 total knee arthroplasty 491 trigeminal neuralgia 316, 318 vasovagal response during pain procedures 474–475 epidural anesthesia acute pain management 482–488, 484, 485 cancer pain 346 postlaminectomy pain 176 postpartum pain 290–291 thoracic spine pain 128–304 total knee arthroplasty 492 see also epidural steroid injections epidural blood patches, dural puncture headache 437 epidural hematoma 152, 429–432, 430, 484 epidural lesions, thoracic spine pain 124–125 epidural lipomatosis 152–153 epidural neuraxial analgesia, pancreatic cancer pain 354–355 epidural space, anatomy 484, 485 epidural steroid injections cervical radiculopathy 113 post-thoracotomy pain 44, 176 treatment complications 423–427, 429–433, 430, 432, 435–438, 484 Epworth Sleepiness Scale 460–461 ER (extended-release) opioids 345 ergotamine 300–301 estrogen-dependent inflammatory processes 261 ethics alcohol-induced neuropathy 69–70 anesthesia-assisted opiate detoxification 453 methadone prescription 501 opioids, substance abuse 402–403 paralysis following epidural steroid injections 427 ethyl alcohol. see alcohol European Federation of Neurological Societies (EFNS) 2–3, 2–15, 6 evidence-based medicine complementary and alternative medicine 397 postherpetic neuralgia 4–7 see also Cochrane reviews; outcome measures exercise brain-derived neurotrophic factor 216 fibromyalgia 218–219

and insomnia 462, 464 knee osteoarthritis 247 migraine headache 304 myofascial pain syndrome 226 opioid-induced constipation 470 postmastectomy lymphedema 367–368 pregnancy pain 284 see also physical therapy exposure to radiation. see radiation exposure extended-release (ER) opioids 345 extensor carpi radialis brevis (ECRB) 240–241 FABER test (Flexion, ABduction, External Rotation, and Extension) 198 Faces Pain Scale – Revised (FPS-R) 383 facet joints postpartum pain 290 pregnancy pain 282 thoracic spine pain 124–125, 127–129 see also cervical facet joints; lumbar facet joints facet synovitis 160 failed back surgery syndrome (FBSS) 174–179, 175, 225–228 faith healing 394–395 FAP (functional abdominal pain) 386–387 fat malabsorption, pancreatitis 254 FCE. see functional capacity evaluation (FCE) FDG (fluorodeoxyglucose) 161 fear-avoidance model of pain (FAM) 226 femoral nerve blocks 492–493, 493 fentanyl anesthesia-assisted detoxification 447–453 cancer pain 346 epidural catheters 487 intrathecal drug delivery 178–179 patient-controlled analgesia 478 phantom limb pain 40 postpartum pain 294 pregnancy pain 285 urine drug testing 414 thoracic outlet syndrome 107 fetal pain 379–380, 381 fiber, dietary 103–470 fibromyalgia 214–219 diagnostic criteria 214–216, 215–219 drug screens, abnormal 414–415 fibrosis, and postlaminectomy pain syndrome 175, 175

fight-or-flight response 216 FLACC scale (Face, Legs, Activity, Cry, Consolability scale) 381, 382, 383 flexion injuries 116 fluid intake dural puncture 486 opioid-induced constipation 422–469 fluorodeoxyglucose (FDG) 161 fluoroscopy epidural catheters 484 lumbar facet mediated pain 139–140 pregnancy pain 287 radiation exposure 417–422, 418–420 focal tendinopathy 240–242 folate deficiency (vitamin B9) 65, 73 foot pain 384–386 frequency-modulated electro-magnet neural stimulation therapy (FREMS) 61 Fryette’s laws of spinal motion 231, 231 functional abdominal pain (FAP) 386–387 functional capacity evaluation (FCE), cervicogenic headache 83 disability benefits 224–225 functional magnetic resonance imaging (fMRI), diabetic neuropathy 54 functional restoration programs (FRPs), myofascial pain syndrome 227 GA (gouty arthopathy) 244–246 GABA (gamma aminobutyric acid) 233, 401–402 gabapentin alcohol-induced neuropathy 67–68 brachial plexopathy 33, 36 cancer pain 344 diabetic neuropathy 58 fibromyalgia 218 lumbar spinal stenosis 169 migraine headache 303 pancreatitis treatment 256 phantom limb pain 40, 41 postherpetic neuralgia. see below postpartum pain 294 poststroke pain 25, 26 pregnancy pain 285 spinal cord injury pain 19 thoracic outlet syndrome 107 thoracic spine pain 128 total knee arthroplasty 492, 494

515

Index

gabapentin, for postherpetic neuralgia 2, 4, 9–10, 11–12 dosing/onset of analgesia 8, 9 new clinical data 5–6 older patients, treatment options 9–11 side effects of treatment 7–8 treatment guidelines 2–3 Gaenslen test, sacroiliac joint pain/ arthritis 197 gait analysis 245 Galveston Orientation and Amnesia Test (GOAT) 85 gamma aminobutyric acid (GABA) 233, 401–402 ganglion impar block, chronic rectal pain 277–320, 278 garlic 77 gas chromatography/massspectroscopy (GC/MS) 410 gastrointestinal function anesthesia-assisted opiate detoxification 450 constipation 467–472, 467–472 differential diagnosis 262–263 postoperative analgesia 483 see also bowel gastroretentive gabapentin 7–8, 8 gelatin, dural puncture headache 437 gender alcohol-induced neuropathy 64 migraine headache 297 opioid dependence 442–443 tension headaches 312 gene therapy, diabetic neuropathy 61 generalized, symmetrical polyneuropathy 55 genetic factors cancer pain 342–343 knee osteoarthritis 244 pancreatitis 253 postpartum pain 291–292 genetic testing, methadone prescription 500 genitofemoral nerve 266, 270–271 glossopharyngeal neuralgia 325–328, 326–327 glucocorticoids, total knee arthroplasty 492, 494. see also corticosteroids glutamate, fibromyalgia 218 GOAT (Galveston Orientation and Amnesia Test) 85 gonadotropin inhibitors, endometrial fibrosis 263–264 gonadotropin releasing hormone (GnRH) analogs 263–264 gouty arthopathy (GA) 244–246

516

graded motor imagery (GMI), phantom limb pain 40–41 grapefruit juice 500 greater trochanteric bursitis 168 growth hormone release, fibromyalgia 216 guidelines aberrant drug use 411–412 drug testing 411 epidural catheters 484 opioid prescription for chronic cancer pain 412 postherpetic neuralgia 2–6, 2–15 radiation exposure recommendations 418, 421 gut instinct, physician 506–507 headache cluster 307–310 dural puncture 175, 435–438, 486 poststroke pain 23, 23–25 tension 312–314, 314 see also cervicogenic headache; medication-overuse headache; migraine headache hearing impaired patient 388–389 heart function. see cardiovascular function heat treatment lumbar disc herniation 134 phantom limb pain 40–41 poststroke pain 25 post-thoracotomy pain 44 hematological cancer 358–360, 365 case descriptions 316–359, 358 interventional management 364 pain mechanisms 358–361, 360–361 pharmacological treatments 362–364 treatment interventions 361–362 hemiplegic shoulder pain (HSP) 23–25 hemorrhoids, differential diagnosis 275 hepatic function anesthesia-assisted opiate detoxification 447–448 buprenorphine 444 methadone prescription 499–500 postherpetic neuralgia 9, 9–10, 11–12 herbal remedies HIV neuropathy 77 low back pain 396 heroin, urine drug testing 414, 505–507 heterotopic ossification 19 high-intensity zones (HIZ), discogenic pain 146 hip pain, in pregnancy 283 hippocampus, fibromyalgia 216 history-taking constipation 468–469

insomnia 460 interstitial cystitis 267 postmastectomy lymphedema 368 sphenopalatine neuralgia 331 urine drug testing 506–507 HIV neuropathy 78 chronic pain syndrome 499 clinical presentation 73–74 differential diagnosis 72–73 epidemiology/demographics 72 examination/work-up 74–75 methadone prescription 500–501 pathophysiology 75 treatment 75–78 hormonal treatment, endometrial fibrosis 263–264 HSP (hemiplegic shoulder pain) 23–25 Hunter criteria, serotonin syndrome 300, 440 HVLA (high velocity, low amplitude) spinal manipulation 230 hyaluronic acid injections, ankle pain 237 hydrocodone drug screens, abnormal 414–415 phantom limb pain 40 postpartum pain 294 pregnancy pain 285 thoracic outlet syndrome 107 hydrocortisone, pregnancy pain 287 hydromorphone cancer pain 345 epidural catheters 487 intrathecal drug delivery 178–179 patient-controlled analgesia 478 postpartum pain 294 pregnancy pain 285 urine drug testing 414, 505–507 hydrophilic/hydrophobic zones, epidural catheters 487 hyperalgesia, poststroke pain 22 hypermobility, craniocervical junction 84 hypertension, and chronic pain syndrome 373–376 hypnosis, chronic pain syndrome 377 hypochondriacs 458 hypothyroidism, differential diagnosis 214 hysteria and hypochondriasis scale, MMPI 147 IASP. see International Association for the Study of Pain iatrogenic chondrotoxic effects, knee osteoarthritis 244 ibuprofen phantom limb pain 40

Index

pregnancy pain 285 tension headaches 314 thoracic outlet syndrome 107 ice treatment. see cold treatment ICHD (International Classification of Headache Disorders) 87, 312 IDD. see intrathecal drug delivery IDET (intradiscal electrothermal therapy) 150 IDP (inpatient detoxification program) 451 iliohypogastric nerve 266, 270 ilioinguinal nerve 266, 270 imaging studies ankle pain 236–237 Baastrup’s disease 160, 161, 161 Bertolotti’s syndrome 156 brachial plexopathy 33 cervical radiculopathy 112 cervical stenosis/myelopathy 97–99 diabetic neuropathy 54 HIV neuropathy 75 knee osteoarthritis 245–246 opioid-induced constipation 422–469 postpartum pain 292 pregnancy pain 284–289 sacroiliac joint pain/arthritis 198 thoracic outlet syndrome 106 see also computed tomography scanning; magnetic resonance imaging; radiography IMDs (intrathecal medication devices) 60. see also intrathecal drug delivery IME. see independent medical assessment imipramine migraine headache 303 postpartum pain 293–294 immobilization, sacral insufficiency fracture 204 immune function and alcohol-induced neuropathy 65–73 and HIV neuropathy 73 impairment, definition 83–84 incidence. see epidemiology/ demographics independent medical assessment (IME) cervicogenic headache 83–84 disability benefits 224 indications for treatment. see contraindications/indications infant pain 380–381, 380–382, 383. see also pediatric pain infection and alcohol-induced neuropathy 65

epidural catheters 484 glossopharyngeal neuralgia 326 and HIV neuropathy 73 and lumbar spondylosis 146 postpartum pain 290 inferior hypogastric plexus nerve block, rectal pain 279–280 inflammatory processes ankylosing spondylitis 125–126 axial neck pain 116 cervical radiculopathy 110–111 endometrial fibrosis 261–262 HIV neuropathy 75 knee osteoarthritis 243 pancreatitis 254 sacroiliac joint pain/arthritis 196 temporomandibular joint disorders 336 thoracic spine pain 128–304 information for patients. see education, patient informed consent, anesthesia-assisted opiate detoxification 448 injection therapy 198–199 ankle pain 237–238 Baastrup’s disease 162–163 Bertolotti’s syndrome 157 chronic pelvic pain 270 knee osteoarthritis 247, 250 low back pain 395–396 lumbar facet mediated pain 140–141 lumbar spinal stenosis 169 phantom limb pain 40–41 postlaminectomy pain syndrome 175–176 post-thoracotomy pain 44 temporomandibular joint disorders 338 thoracic outlet syndrome 106–107 thoracic spine pain 128–304 see also epidural steroid injections innervation chronic rectal pain 275–276 male pelvis 266 sphenopalatine ganglion 330 inpatient detoxification program (IDP) 451 inpatient psychological interventions 377 insomnia, and chronic pain 459–464 Insomnia Severity Index 460–461 inspections, radiation equipment 417 integrative medicine, low back pain 396. see also interdisciplinary management intercostal neuralgia 43, 124–125 interdisciplinary management

chronic pain syndrome 373 chronic pelvic pain 273 myofascial pain syndrome 227 pancreatic cancer pain 352–353 postlaminectomy pain syndrome 179 total knee arthroplasty 491–494, 492, 495 interleukin-8, cancer pain 342–343 intermittent claudication, lumbar spinal stenosis 168 intermittent external pneumatic compression, postmastectomy lymphedema 370 International Association for the Study of Pain (IASP) 22, 47 International Classification of Headache Disorders (ICHD) 87, 312 International Headache Society 23 cervicogenic headache/occipital neuralgia 85–87, 89 cluster headaches 307 migraines 298–299 interspinous bursitis 159–163, 160, 161, 161–162, 162 interspinous spacers, lumbar spinal stenosis 170–171 interstitial cystitis. see chronic pelvic pain interventional pain physicians, cancer pain 352–353 intervertebral discs, lumbar spinal stenosis 168. see also discogenic pain intra-articular corticosteroids, knee osteoarthritis 244 intracranial hypotension, differential diagnosis 435 intradiscal electrothermal therapy (IDET) 150 intradiscal surgery, axial neck pain 121 intramuscular drug delivery, fetal pain 380 intrathecal drug delivery (IDD) analgesic routes other than oral 346 chronic pelvic pain 273 pancreatic cancer pain 355 postlaminectomy pain syndrome 178–179 spasticity 233–234 intrathecal medication devices (IMDs) 60 intravascular drug delivery, fetal pain 380 ionizing radiation, biological effects 419, 419–422 Irving study, postherpetic neuralgia 5 jaw exercises 337

517

Index

Kambin triangle 426, 426 ketamine epidural catheters 488 hematological cancer 364 patient-controlled analgesia 480 phantom limb pain 40, 41 total knee arthroplasty 494–495 urine drug testing 414 Khaliq review, postherpetic neuralgia 4 kidney function. see renal function kissing spine, 159–163, 160, 161, 162 knee osteoarthritis (KOA) 243–244 common causes 244 differential diagnosis 244–245 examination/work-up 245–247 non-surgical treatment options 247–249, 249 postoperative analgesia 491–495, 492, 493 surgery 249–250 knee pain, in pregnancy 283 kyphoplasty bone metastases 210–212, 211 hematological cancer 364 lumbar compression fracture 188–191 sacral 204–205, 205–206 kyphosis, lumbar compression fracture 184 lactulose, opioid-induced constipation 470 laminectomy, lumbar spinal stenosis 171–177. see also postlaminectomy pain syndrome lamotrigine brachial plexopathy 36 migraine headache 303 phantom limb pain 40 poststroke pain 25, 26 thoracic outlet syndrome 107 language impairment, poststroke pain 25 large Atherosclerosis Risk in Communities (ARIC) study 299 Lasegue/Laséque/Lazarevic sign Bertolotti’s syndrome 156 lumbar disc herniation 133 laser therapy, postmastectomy lymphedema 370 last observation carried forward (LOCF) imputation method, postherpetic neuralgia 6 lateral epicondylosis (LE) 240–242, 242 lateral femoral cutaneous nerve (LFCN) 291 lateral shear test, cervicogenic headache 84 laxatives 470

518

LDH. see lumbar disc herniation leaded barriers, radiation exposure prevention 420 Leeds Assessment of Neuropathic Symptoms and Signs Pain Scale (LANSS) 54 leg cramps, pregnancy 283 legal implications motor vehicle collisions 82–83 myofascial pain syndrome 223–224 paralysis following epidural steroid injections 427 postlaminectomy pain syndrome 174, 175 urine drug testing 506–507 leukemia. see hematological cancer LFCN (lateral femoral cutaneous nerve) 291 lidocaine alcohol-induced neuropathy 68, 269 complex regional pain syndrome 48 diabetic neuropathy 59 lumbar spinal stenosis 169–170 postherpetic neuralgia 2–3, 2–4, 2–15, 6–8, 9–10 postpartum pain 293 pregnancy pain 285, 286 limaprost, lumbar spinal stenosis 169 liquid chromatography/massspectroscopy (LC/MS-MS) 410, 504 literature reviews see meta-analyses, research studies, systematic reviews lithium, postpartum pain 294 liver function. see hepatic function local anesthetics brachial plexopathy 36 cancer pain 344 complex regional pain syndrome 49 diabetic neuropathy 59 epidural catheters 487–488 phantom limb pain 40 postpartum pain 293 post-thoracotomy pain 44 pregnancy pain 286 thoracic outlet syndrome 107 total knee arthroplasty 492, 492–493, 493 see also mexiletine LOCF. see last observation carried forward long QT syndrome 499–500 low back pain 390 complementary and alternative medicine 391–396 differential diagnosis 423 drug testing 504–507

paralysis following epidural steroid injections 423–427 postpartum pain 290–295 pregnancy pain 282–284, 288 sacroiliac joint pain/arthritis 195–200, 196 LSD, urine drug testing 507 LSS, see lumbar spinal stenosis lumbar compression fracture 182, 182–191, 183–184, 184–185, 186–187, 187–191 lumbar disc herniation (LDH) 131–136, 132–133, 155 differential diagnosis 131, 137–138 pregnancy pain 282 thoracic spine pain 124–125 lumbar facet mediated pain 109, 131, 137–143, 138–139, 139–140, 140–141, 141–142, 155 lumbar spinal stenosis 167–168 and lumbar spondylosis, discogenic pain 144 postlaminectomy pain syndrome 176–177 sacroiliac joint pain/arthritis 195–197 lumbar spinal stenosis (LSS) 164, 166–173, 171–172 anatomy 165–166 classification systems 166, 166–173 diagnostic criteria 167 differential diagnosis 131, 137–138, 167–168 natural history 165 treatments 168–177 lumbar spondylosis, discogenic pain 144–150, 146, 148–320 lumbar surgery/postlaminectomy pain syndrome 174–179, 175 lumbar sympathetic nerve block 49 lumbar transforaminal epidural steroid injections 423–427 lumbosacral fusion 196 lumbosacral radiculitis, differential diagnosis 195–197 lumbosacral region, magnetic resonance imaging 358 lumbosacral transitional vertebra, Bertolotti’s syndrome 155–157 lung cancer, bone metastases 207–212, 209, 211 lymphangiosarcoma 370 lymphedema, postmastectomy 367–371 lymphoma. see hematological cancer lymphoscintigraphy, postmastectomy lymphedema 369 magnesium sulfate, opioid-induced constipation 470

Index

magnetic resonance imaging (MRI) axial neck pain 117–118 Baastrup’s disease 160, 161, 161 Bertolotti’s syndrome 156 bone metastases 208 cervical radiculopathy 112 cluster headaches 309 epidural hematoma 430 epidural lipomatosis 152–153 knee osteoarthritis 246, 249 lumbar compression fracture 185, 186 lumbar disc herniation 132–133, 133 lumbar facet mediated pain 139 lumbar spinal stenosis 167, 172 lumbar spondylosis, discogenic pain 145–146, 146 lumbosacral region 358 pancreatitis 254–255 paralysis following epidural steroid injections 424 sacral insufficiency fracture 203 trigeminal neuralgia 317 Maigne’s syndrome 124–125, 127–128 malabsorption, pancreatitis 254 malignancy. see cancer pain, metastases malignant hyperthermia, differential diagnosis 440 malingering, legal aspects 83 manipulation. see chiropractic manipulation, osteopathic manipulation therapy Manual Tender Point Survey, American College of Rheumatology 214–215 marijuana. see cannabinoids massage HIV neuropathy 77 low back pain 393 phantom limb pain 40–41 poststroke pain 25 post-thoracotomy pain 44 spinal cord injury pain 19 mastectomy, lymphedema 367–371 maxillary sinusitis, differential diagnosis 336 maximum medical improvement (MMI) cervicogenic headache 83–84 disability benefits 224 maximum permissible doses (MPD), ionizing radiation 419, 419 Meckel’s ganglion 329–330 mediastinis 207 medication-overuse headache 299–301, 442–446 cluster 308–311

migraine 298–300, 302, 305 tension 314 meditation, low back pain 393 Medtronic Corporation 179 melatonin, and insomnia 463 melatonin receptor agonists 462 meloxicam phantom limb pain 40 thoracic outlet syndrome 107 memory loss, cervicogenic headache 85 meningitis 312, 429–430 mental health counseling. see psychological interventions mental health disorders Munchausen syndrome 456–458 opioid dependence 442–443 meperidene patient-controlled analgesia 478 postpartum pain 294 pregnancy pain 285 urine drug testing 505–507 meralgia paresthetica, postpartum pain 291 mesenchymal stem cells (MSCs). see stem cell transplantation meta-analyses celiac plexus block 354 cognitive behavioral therapy 374 migraine headache 299 postherpetic neuralgia 4 see also systematic reviews metabolic factors and alcohol-induced neuropathy 65–73 and HIV neuropathy 73 metabolic pathways, opioids 411 metabolic syndrome, knee osteoarthritis 244 metastases lung cancer 483 rectal 275 skeletal. see bone metastases see also cancer pain methadone cancer pain 346 chronic pain syndrome 498–502 complex regional pain syndrome 48 postherpetic neuralgia 11–12 phantom limb pain 40 pregnancy pain 285 thoracic outlet syndrome 107 urine drug testing 414, 505–507 methocarbamol phantom limb pain 40 thoracic outlet syndrome 107 methyl prednisone brachial plexopathy 36

phantom limb pain 40 thoracic outlet syndrome 107 methylcobalamin, alcohol-induced neuropathy 68–69 methylene blue 149 methylnaltrexone 471 mexiletine HIV neuropathy 76 phantom limb pain 41 pregnancy pain 286 thoracic outlet syndrome 107 microvascular decompression glossopharyngeal neuralgia 327 trigeminal neuralgia 320–321 see also compression/decompression MIDAS (Migraine Disability Assessment) 298 migraine headache 297–298, 304–305 acupuncture 304 diagnostic criteria 298–299 diary keeping 298, 302 differential diagnoses 308, 312, 329, 336, 435 examination/work-up 298–299 long-term risks counseling 299 occipital nerve stimulation 304 pharmacological treatments 299–314, 300–301 prevention strategies 300–314, 303 milnacipam, migraine headache 303 mindfulness meditation, low back pain 393 minimally invasive lumbar decompression (MILD) 170 mirror therapy, phantom limb pain 40–41 misuse, definition 400–401 MMI. see maximum medical improvement Modified Ashworth Scale, spasticity assessment 232, 233 monitoring, safe use of opioids 405 mononeuropathy multiplex (MM) 72–74 Monro-Kellie hypothesis 486 Moore review, postherpetic neuralgia 4 morphine cancer pain 345 infant pain 382 intrathecal drug delivery 178–179 patient-controlled analgesia 478 phantom limb pain 40 postpartum pain 294 pregnancy pain 285 thoracic outlet syndrome 107 urine drug testing 414, 505–507

519

Index

mortality rates anesthesia-assisted opiate detoxification 450, 453 benzodiazepines 462 methadone prescription 498–500 oxycodone, chronic low back pain case 504 spinal cord injury pain 16 thoracotomy 482 total knee arthroplasty 492 motor cortex stimulation, poststroke pain 25 motor vehicle collisions 81 brachial plexopathy 30 drug screens, abnormal 412–414, 414 fibromyalgia 214 legal aspects 82–83 myofascial pain syndrome 223 sacroiliac joint pain/arthritis 195–200, 196 whiplash 81–82 MPD. see maximum permissible doses MRI. see magnetic resonance imaging multimodal analgesic protocols, total knee arthroplasty 491–494, 492, 495 multiple myeloma, case descriptions 358, 358 multiple sclerosis 96, 152, 325, 326, 327, 488 Munchausen syndrome 456–458 Munchausen syndrome by proxy 458 mu-receptors 443 muscle relaxants cancer pain 344 phantom limb pain 40 pregnancy pain 286 thoracic outlet syndrome 107. see also baclofen; cyclobenzaprene; methocarbamol; relaxation training muscle sprain/strain 144, 290 musculoskeletal pain differential diagnosis 262–263 postpartum pain 290 poststroke pain 23, 27 spinal cord injury pain 16, 17–21 musculoskeletal ultrasound (MSKUS), knee 246 music therapy 347 myelogenous leukemia (AML), case description 358 myelography, lumbar disc herniation 133–134 myocardial infarction. see cardiovascular function myofascial pain syndrome 82, 223–227

520

and axial neck injuries 116, 118 differential diagnosis 81–82, 116, 137–138, 195–197, 207, 214, 217, 368 and lumbar facet mediated pain 137–138, 140, 155 postpartum pain 290 and temporomandibular joint disorders 120–335, 334–339 and thoracic spine pain 124–125, 129 myofascial release techniques, lymphedema 370 myo-inositol 69–70 myosotis, differential diagnosis 214, 217 N-acetylcysteine 69–70 NA (nucleus accumbens) 401–402 nalmefine, opioid-induced constipation 471 naloxone, opioid-induced constipation 471 naltrexone 450 naproxen phantom limb pain 40 pregnancy pain 285 tension headaches 314 thoracic outlet syndrome 107 narcotic, definition 400–401 National Center for Complementary and Alternative Medicine (NCCAM) 390 National Certification Commission for Acupuncture and Oriental Medicine (NCCAOM) 392 National Institute for Health and Clinical Excellence (NICE) 452 National Institutes of Health. see NIH National Survey in Drug Use and Health (NSDUH) 442 NBHD (non-benzodiazepine hypnotic drugs) 462 neck distraction test 111 neck pain axial 89–121 opioids, substance abuse 399 vasovagal response during pain procedures 473–475 needle placement, epidural steroid injections 430–431, 432 nerve biopsy, HIV neuropathy 74 nerve blocks axial neck pain 89–119, 119–121 Bertolotti’s syndrome 157 cancer pain 341–348, 347–349, 353–354 cervical radiculopathy 113 chronic pelvic pain 270–271 chronic rectal pain 277–347, 278 cluster headaches 309–310

lumbar facet mediated pain 141, 143 lumbar sympathetic block 49 migraine headache 304 pancreatitis treatment 257 postlaminectomy pain syndrome 175–176 sphenopalatine neuralgia 331 thoracic outlet syndrome 106 thoracic spine pain 128 total knee arthroplasty 492–495, 493 trigeminal neuralgia 319 nerve compression. see compression nerve conduction studies (NCS) alcohol-induced neuropathy (AIN) 66 cervical radiculopathy 112 HIV neuropathy 74 lumbar disc herniation 134 thoracic outlet syndrome 106 nerve growth factor (rhNGF), HIV neuropathy 76 nerve injury, epidural catheters 484–486 nerve root avulsion, 31–35 nerve stimulation. see neurostimulation nervus intermedius neuralgia (NIN) 327 NeuPSIG. see Special Interest Group on Neuropathic Pain neural imaging studies, thoracic outlet syndrome 106 neuralgia glossopharyngeal 325–328, 326–327 sphenopalatine 329–332 see also postherpetic neuralgia; trigeminal neuralgia neuraxial analgesia pancreatic cancer pain 354–355 postpartum pain 290–291 total knee arthroplasty 492 treatment complications 355 see also epidural anesthesia neuraxial bleeding, epidural catheters 484 neurodiagnostic imaging guidelines, pregnancy pain 284–289 neurogenic claudication 167, 171 neuroleptic malignant syndrome 440 neuromas, post-thoracotomy pain 43–45 neuromodulation chronic pelvic pain 271–273 chronic rectal pain 277, 280 complex regional pain syndrome 50 phantom limb pain 41 trigeminal neuralgia 322–323

Index

neuropathic pain 16–17 bone metastases 360 cancer pain 341 differential diagnosis 368 endometrial fibrosis 262–263 hematological cancer 361–362 methadone prescription 501 painful diabetic peripheral neuropathy 52 phantom limb pain 38–39 spinal cord injury pain 16–20, 17–21 thoracic spine pain 123, 127–128 see also alcohol-induced neuropathy; complex regional pain syndrome; diabetic neuropathy; HIV neuropathy; peripheral neuropathy Neuropathic Pain Questionnaire (NPQ) 54 Neuropathic Pain Scale (NPS) 54 Neuropathic Pain Special Interest Group 76 Neuropathic Pain Symptom Inventory (NPSI) 54 neuroplasty therapy (adhesiolysis) 169–170, 177 neuroreflexotherapy (NRT), low back pain 396 neurostimulation cluster headaches 309 trigeminal neuralgia 322–323 see also transcutaneous electrical nerve stimulation neurotransmitter systems 218, 500–501. see also dopamine; gamma aminobutyric acid; norepinephrine; serotonin neurovascular model, migraine headache 297–298 nevirapine, drug-drug interactions 500 NICE (National Institute for Health and Clinical Excellence) 452 nicotine, and insomnia 461 NIH (National Institutes of Health) chronic prostatitis symptom index (NIH-CPSI) 272 NIN (nervus intermedius neuralgia) 327 nitric oxide (NO) 75 N-methyl-D-aspartate (NMDA) antagonists cancer pain 344 chronic pelvic pain 269 complex regional pain syndrome 48 hematological cancer 364 methadone 500–501 phantom limb pain 40 poststroke pain 24

total knee arthroplasty 492, 494–495 see also ketamine NNRTI. see nucleoside reversetranscriptase inhibitor nociceptive pain bone metastases 360 cancer pain 341 endometrial fibrosis 262 pancreatitis 254 spinal cord injury pain 17–21 thoracic spine pain 123 non-benzodiazepine hypnotic drugs (NBHD) 462 non-maleficence principle, substance abuse 402–403 non-small cell lung cancer, bone metastases 207–212, 209, 211 non-steroidal anti-inflammatory drugs (NSAIDs) brachial plexopathy 33, 36 cancer pain 295–344 chronic pelvic pain 269 complex regional pain syndrome 48 endometrial fibrosis 264 lumbar spinal stenosis 169 pancreatitis treatment 256 phantom limb pain 40 postpartum pain 293 poststroke pain 25 pregnancy pain 284, 286 thoracic outlet syndrome 107 thoracic spine pain 128–304 total knee arthroplasty 492, 494 trigeminal neuralgia 319 see also celecoxib; ibuprofen; meloxicam; naproxen norepinephrine 218. see also SNRI antidepressants North American Neuromodulation Society 178–179 North American Spine Society Diagnostic and Therapeutic Committee 147 nortriptyline alcohol-induced neuropathy 68–269 brachial plexopathy 33 hematological cancer 363 migraine headache 218–302, 303 pancreatitis treatment 256 phantom limb pain 40 postpartum pain 293–294 poststroke pain 25 tension headaches 313–314 thoracic outlet syndrome 107

NPQ (Neuropathic Pain Questionnaire) 54 NPS (Neuropathic Pain Scale) 54 NPSI (Neuropathic Pain Symptom Inventory) 54 NRT (neuroreflexotherapy), low back pain 396 NSAIDs. see non-steroidal antiinflammatory drugs NSDUH (National Survey in Drug Use and Health) 442 nuclear bone scans lumbar compression fracture 185, 186–187 sacroiliac joint pain/arthritis 198 nucleoside reverse-transcriptase inhibitor (NNRTI) medications 500 nucleus accumbens (NA) 401–402 Numerical Pain Rating Scale (NPRS) 5 nutritional support alcohol-induced neuropathy 65, 67, 70 diabetic neuropathy 57 HIV neuropathy 73, 77 knee osteoarthritis 247 opioid-induced constipation 469–470 urine drug screening 411 obesity knee osteoarthritis 244 postmastectomy lymphedema 367–368, 370 obstructive sleep apnea (OSA) and insomnia 460 total knee arthroplasty 495 occipital nerve 304, 304, 309–310 occipital neuralgia cervicogenic headache 86–89, 88 differential diagnosis 81–82, 312 treatments 89–93, 90, 91 occipital-atlanto-axial joint complex 81–82, 86–89, 88 occupational risk factors, myofascial pain syndrome 226–227 Occupational Safety and Health Administration. see OSHA occupational therapy alcohol-induced neuropathy 69 thoracic outlet syndrome 106 OCD (osteo-chondral defects), ankle pain 236–237 OIH (opioid-induced hyperalgesia) 219 older patients, postherpetic neuralgia 9–10, 9–11 O-Log (Orientation Log) 85 OMT. see osteopathic manipulation therapy

521

Index

onabotulinum. see botulinum toxin opiates, definitions 400–401 Opioid Assessment tool for Patients with Pain (SOAPP-R) 404–405 opioid-induced constipation 467–472, 467–472, 468 opioid-induced hyperalgesia (OIH) 219 opioid receptor agonists 61 opioid receptor antagonists 450, 471 opioid receptors 468, 468 Opioid Risk Tool (ORT) 404–405, 408–409 opioids alcohol-induced neuropathy 68 bone metastases 208 brachial plexopathy 33 cancer pain 343–346, 345, 355 chronic pelvic pain 269–270 complex regional pain syndrome 48 definitions 400–401 dependence 442–453. see also substance abuse diabetic neuropathy 59 drug screens 410–416, 414–415 epidural catheters 487–488 fibromyalgia 216, 218–219 functional restoration programs 227 hematological cancer 363 HIV neuropathy 76 infant pain 382 and insomnia 461 intrathecal drug delivery 178–179 knee osteoarthritis 247 lumbar spinal stenosis 169 metabolic pathways 411 migraine headache 301 myofascial pain syndrome 225, 227 pancreatitis treatment 256 patient-controlled analgesia 478 phantom limb pain 40, 41 postherpetic neuralgia 2–3, 2–4, 2–15, 7–13, 9–10 postpartum pain 294 poststroke pain 25 pregnancy pain 287 rotation 471 side effects 3–7, 346–347, 347 thoracic outlet syndrome 107 total knee arthroplasty 492, 493–494 treatment agreement 404 urine drug testing 505–507 see also fentanyl; hydrocodone; hydromorphone; methadone; morphine; oxycodone; tapentadol; tramadol opiophobia 406

522

oral contraceptives, endometrial fibrosis 263–264 oral hydration, dural puncture headache 436 organic brain syndrome, aberrant drug use 408, 413 Orientation Log (O-Log) 85 ORT (Opioid Risk Tool) 404–405, 408–409 orthotics. see braces OSA. see obstructive sleep apnea OSHA (Occupational Safety and Health Administration), radiation exposure, standards 417–418 Osler, William 402 osmotic laxatives 470 ossification of posterior longitudinal ligament (OPLL) 99 osteoarthritis, temporomandibular joint disorders 334–339. see also knee osteoarthritis osteochondral defects (OCD), ankle pain 236–237 osteopathic manipulation therapy (OMT) 230–232, 231, 285 osteoporosis lumbar compression fracture 182, 183–184, 183–184, 191 pregnancy pain 283 and sacral insufficiency fracture 204 osteitis pubis, pregnancy pain 283 Oucher pain scale 383, 383 outcome measures anesthesia-assisted opiate detoxification 451–452 complementary and alternative medicine 391, 397 opioid dependence 445 see also Cochrane reviews; systematic reviews outpatient psychological interventions 377 overdose, accidental methadone prescription 499 statistics 409, 413 oxidative stress, alcohol use 64–65, 67, 69 oxycodone anesthesia-assisted opiate detoxification 447–453 cancer pain 345 phantom limb pain 40 postherpetic neuralgia 6–7 pregnancy pain 285 thoracic outlet syndrome 107 urine drug testing 414, 505–507 oxygen treatment, cluster headaches 309

Paget’s disease 202 Pain Medication Questionnaire (PMQ) 408–409 painful diabetic peripheral neuropathy (PDPN) 52. see also diabetic neuropathy palliative care, bone metastases 208 palpation, knee osteoarthritis 245 pancreas cancer pain. see cancer pain pancreatitis 253–255, 253–258 PAP. see postamputation pain paracetamol. see acetaminophen paralysis following epidural steroid injections 423–427, 429–433, 430, 432 paresthesia, epidural catheters 484–486 paroxetine, pregnancy pain 285 passive range of motion testing (PROM), ankle pain 236 patellofemoral disorder, pregnancy pain 283 patents, anesthesia-assisted opiate detoxification 453 patient-controlled analgesia (PCA) 476–480 patient-controlled epidural analgesia (PCEA) 482–488, 484, 485 patient expectations, postlaminectomy pain syndrome 175 patient global impression of change (PGIC) 5 patient information. see education patient-surgeon communication. see therapeutic relationship Patrick test (FABER test) 198 PDMP. see Prescription Drug Monitoring Programs PDPN. see painful diabetic peripheral neuropathy pediatric pain, 379–387, 381, 382, 383, 384 pelvic belts 285, 292. see also braces pelvic floor muscle dysfunction, chronic pelvic pain 268 pelvic girdle pain (PGP) postpartum pain 291 pregnancy pain 282–283 see also chronic pelvic pain pentosan polysulfate 269 percutaneous disc surgery, lumbar disc herniation 133–135, 134–136 percutaneous electrical nerve stimulation (PENS) 61 percutaneous radiofrequency neurotomy. see radiofrequency ablation percutaneous radiofrequency thermocoagulation 319–320

Index

percutaneous radiofrequency trigeminal rhizotomy (PTR) 319 percutaneous vertebroplasty (VP) 188–191 perineal pain, postpartum 291 peripheral catheters. see continuous peripheral catheters peripheral nerve blocks chronic pelvic pain 270 migraine headache 304 total knee arthroplasty 492–495, 493 peripheral nerve stimulation cervicogenic headache/occipital neuralgia 92–93 chronic rectal pain 279 peripheral neuropathy chemotherapy-induced 342, 353 lumbar spinal stenosis 168 PET. see positron emission tomography PGIC (patient global impression of change) 5 PGP. see pelvic girdle pain phantom limb pain 16, 38–42, 39–42 pharmacokinetics/pharmacodynamics buprenorphine 443–444 methadone 500–501 pharmacological treatments alcohol-induced neuropathy 67–69 brachial plexopathy 33, 36 cancer pain 343–346, 344, 345 cervical radiculopathy 112 chronic pelvic pain 268–270 chronic rectal pain 277 cluster headaches 309 complex regional pain syndrome 48 diabetic neuropathy 57–60 endometrial fibrosis 264 epidural catheters 487–488 fibromyalgia 217–218 glossopharyngeal neuralgia 327 hematological cancer 362–364 HIV neuropathy 75–76 insomnia 462 knee osteoarthritis 247 lumbar disc herniation 134 lumbar spinal stenosis 169 migraine headache 300–301, 303, 304–305 myofascial pain syndrome 225 pancreatitis 256 phantom limb pain 39, 40 postherpetic neuralgia 1–2 postlaminectomy pain syndrome 177–178 postpartum pain 293–295 poststroke pain 25, 26

post-thoracotomy pain 44 pregnancy pain 284, 285 spasticity 233–234 sphenopalatine neuralgia 331 spinal cord injury pain 19 temporomandibular joint disorders 338 tension headaches 313–314, 314 thoracic outlet syndrome 106, 107 total knee arthroplasty 492, 493–494 trigeminal neuralgia 319 see also alpha-2 adrenergics; alpha-2 delta ligands; antidepressants; anticonvulsants; beta blockers; calcium channel blockers; corticosteroids; local anesthetics; muscle relaxants; N-methyl-D-aspartate antagonists; non-steroidal antiinflammatory agents; opioids pharming, definitions 400–401 phencyclidine, urine drug testing 410–416, 505–507 phenol, spasticity 233 phenoxybenzamine/phentolamine, complex regional pain syndrome 48 phenytoin alcohol-induced neuropathy 67–68 poststroke pain 26 pregnancy pain 285 PHN. see postherpetic neuralgia physical dependence definitions 400–401 neurobiological processes underlying addiction 401 physical therapy alcohol-induced neuropathy 69 ankle pain 237 cervical radiculopathy 113 cervicogenic headache/occipital neuralgia 89 complex regional pain syndrome 48 diabetic neuropathy 60 knee osteoarthritis 247 lateral epicondylosis 241 lumbar disc herniation 134 lumbar spinal stenosis 169 lumbar spondylosis 148–149 myofascial pain syndrome 226 postlaminectomy pain syndrome 179 postmastectomy lymphedema 367–368, 370 postpartum pain 292 pregnancy pain 284 sacral insufficiency fracture 204 Scheuermann’s kyphosis 127 thoracic outlet syndrome 106 thoracic spine pain 128

see also exercise physician, gut instinct 506–507 phytotherapy, chronic pelvic pain 269 pilonidal cyst, differential diagnosis 275 PIN (posterior interosseous nerve) 240–241 PIPP (Premature Infant Pain Profile) 381, 381 piriformis syndrome 195–197, 290 Pittsburgh Sleep Quality Index 460–461 placebo effects, complementary and alternative medicine 397 platelet rich plasma (PRP) ankle pain 238 knee osteoarthritis 247–249 ankle pain 238 lateral epicondylosis 241 pleuritis 43–44, 207 PLPS. see postlaminectomy pain syndrome PMQ (Pain Medication Questionnaire) 408–409 pneumocephalus, differential diagnosis 435 pocket fills, intrathecal drug delivery 179 Polyanalgesic Consensus Guidelines, North American Neuromodulation Society 178–179 polymethylmethacrylate (PMMA) bone metastases 210–211 percutaneous vertebroplasty 188–189 polymyalgia rheumatica (PMR), differential diagnosis 214, 216–342 polyol pathway, diabetic neuropathy 58 polysomnography 460 population-based study, cluster headaches 310 positron emission tomography (PET), diabetic neuropathy 54 postamputation pain (PAP) 38. see also phantom limb pain postdural puncture headache 175, 435–438, 486 postherpetic neuralgia (PHN) 1–2, 13 current guidelines limitations 4–5, 9–10, 9–12 differential diagnosis 329 dosing/onset of analgesia 8, 9, 10–13 guidelines for treatment 2–3, 2–4, 2–15 new clinical data 5–7 side effects of treatment 7–8 systematic reviews/meta-analyses 4

523

Index

postherpetic neuralgia (PHN) (cont.) treatment selection 7–13, 9 postlaminectomy pain syndrome (PLPS) 174–179, 176–178 drug screens, abnormal 410–416, 411 factors associated with 175 postmastectomy lymphedema 367–371 postpartum pain 290–295 poststroke pain. see central poststroke pain post-thoracotomy pain 43–45 post-tonsillectomy pain 325–328, 326–327 posterior interosseous nerve (PIN) 240–241 posterior rami syndrome 124–125, 127–128 posterior talo-fibular ligament (PTFL) 235–236 PI (protease inhibitors) 500 PP (progressive polyradiculopathy) 72–74 prednisolone, pregnancy pain 285 prednisone postpartum pain 293 pregnancy pain 285, 287 pregabalin alcohol-induced neuropathy 67–68 brachial plexopathy 33, 36 cancer pain 344 combination therapeutic approaches 6–7 diabetic neuropathy 58 fibromyalgia 218 migraine headache 303 pancreatitis treatment 256 phantom limb pain 40 postherpetic neuralgia 4, 7–8, 9, 8–12 postpartum pain 294 poststroke pain 26 side effects 7–8 spinal cord injury pain 19 thoracic outlet syndrome 107 thoracic spine pain 128 total knee arthroplasty 492, 494 pregnancy anesthesia-assisted opiate detoxification 447–448 epidural hematoma 430 and sacral insufficiency fracture 202 pregnancy pain 282–288 low back pain 195 neurodiagnostic imaging guidelines 284–289 pharmacological treatments 285 see also postpartum pain

524

Premature Infant Pain Profile (PIPP) 381, 381 Prescription Drug Monitoring Programs (PDMP), aberrant drug use 412 pressure algometers 43 prevalence. see epidemiology/ demographics prevention strategies lateral epicondylosis 241 migraine headache 300–314, 303 PRICEMM (protect, ice, compression, elevation, medication, modalities), lateral epicondylosis 241 primary insomnia 459 principle of beneficence 402–403 principle of Daubert 83 principle of justice 403 principle of non-maleficence 402–403 proinflammatory cytokines, HIV neuropathy 75 proctalgia fugax 275–276 progestational agents, endometrial fibrosis 263–264 progressive muscle relaxation (PMR). see relaxation training progressive polyradiculopathy (PP) 72–74 prolotherapy ankle pain 237–238, 238 elbow pain 242 lateral epicondylosis 241 low back pain 395–396 PROM (passive range of motion) testing, ankle pain 236 prophylactic medication, migraine headache 300–314, 303 propranolol migraine headache 303 pregnancy pain 285 Proprionibacterium acnes 146, 148 prosapepide, HIV neuropathy 77 prostatitis 267, 272 prostheses phantom limb pain 40–41 temporomandibular joint disorders 338 protease inhibitors (PI) 500 protein kinase C inhibitors, diabetic neuropathy 58 proximal radioulnar joint 240 PRP. see platelet rich plasma pseudoaddiction 400–401, 408, 413 pseudogout. see calcium pyrophosphate dihydrate disease PSTIM (pulse stimulation treatment) 393

psychiatric complications, anesthesia assisted opiate detoxification 450 psychological interventions alcohol-induced neuropathy 69 chronic pain syndrome 373–377 complex regional pain syndrome 48 diabetic neuropathy 60 hematological cancer 364 HIV neuropathy 77 knee osteoarthritis 247 myofascial pain syndrome 226–227 temporomandibular joint disorders 337–338 see also cognitive behavioral therapy; cognitive therapy; mental health disorders psychometric tools, insomnia 460–461. see also screening tools psychosocial factors, postpartum pain 291–292 psychostimulants, cancer pain 344 psychotropic drugs, postpartum pain 294 pterygopalatine ganglion 329–330 PTFL (posterior talo-fibular ligament) 235–236 PTR (percutaneous radiofrequency trigeminal rhizotomy) 319 pubic symphysis separation, pregnancy pain 283 pudendal nerve, male pelvis 266 pudendal nerve block chronic pelvic pain 271 chronic rectal pain 279, 347 pudendal neuralgia, differential diagnosis 275 pulse stimulation treatment (PSTIM) 393 pulsed mode fluoroscopy, radiation exposure 419–420 pulsed radiofrequency ablation 113, 200. see also radiofrequency ablation pulvinar nucleus, thalamus 24 qualified medical evaluator (QME), disability benefits 224 quality of life (QOL) 62 phantom limb pain 41–42 postmastectomy lymphedema 367–368 poststroke pain 26–27 thoracic outlet syndrome 108 quantitative sensory testing (QST) alcohol-induced neuropathy 66 diabetic neuropathy 53 HIV neuropathy 53

Index

Quantitative Sudomotor Axon Reflex Test (QSART) 75 Quebec Task Force (QTF) 116 radiation exposure, interventional pain physician, 417–422, 418, 419 radiation-induced fibrosis 368 radiation-induced pain hematological cancer 361–362 pancreatic cancer 353 radiation proctitis 275–277 radiation treatment, cancer pain 342 radicular pain lumbar facet mediated pain 138, 138 paralysis following epidural steroid injections 429–433 radiculopathy epidural catheters 484–486 paralysis following epidural steroid injections 423–427 secondary to lumbar disc herniation 132, 134 radiofrequency ablation ankle pain 237 Bertolotti’s syndrome 157 bone metastases 208–212 cervical radiculopathy 113 cervicogenic headache 89–90, 91 facet joints, cervical 119–120, 119–121 glossopharyngeal neuralgia 328 lumbar facet mediated pain 140–143, 142 pancreatic cancer pain 354 pancreatitis treatment 257 sacroiliac joint pain/arthritis 199–200 thoracic spine pain 128 radiography ankle pain 236–237 axial neck pain 118 Baastrup’s disease 161 brachial plexopathy 33 cervical radiculopathy 112 cervical stenosis 95, 97–99 diffuse idiopathic skeletal hyperostosis 126 lumbar compression fracture 185 lumbar facet mediated pain 139 radiohumeral joint 240 radionucleotide bone scan. see nuclear bone scan ramelteon 462 range of motion (ROM) testing 236, 245 RDC. see research diagnostic criteria rectal innervation, chronic rectal pain 275–276 rectal pain, chronic 275–280, 278, 279

red flags drug screens, abnormal 415 migraine headache 298–299 substance abuse 402 referred pain differential diagnosis 275 sacroiliac joint pain/arthritis 196, 196–197 see also visceral pain regenerative injection therapy (RIT) knee osteoarthritis 247–249 low back pain 395–396 Regenexx-SCP, ankle pain 238 regional anesthesia, postoperative 491–495, 492, 493, 495 rehabilitation brachial plexopathy 33 complex regional pain syndrome 48 epidural hematoma 433 spinal cord injury pain 18 see also physical therapy relaxation training chronic pain syndrome 374–375 insomnia 464 low back pain 394 postpartum pain 293 tension headaches 313 see also muscle relaxants relaxation-induced anxiety 375 remifentanil 478 renal function anesthesia-assisted opiate detoxification 447–448 buprenorphine 444 postherpetic neuralgia 9, 9–10, 11–12 research diagnostic criteria, temporomandibular joint disorders (RDC/TMD) 333, 334–339, 336 research studies, spinal manipulation 232. see also meta-analyses; systematic reviews residual-limb pain (RLP) 38–39. see also phantom limb pain resisted middle finger test 241 resisted wrist extension test 241 respect for patient autonomy 403 respiratory depression epidural catheters 486 opioids 443 respiratory system, postoperative analgesia 483 restless leg syndrome 216 retrospective studies, regional anesthesia 492 rheumatoid arthritis, differential diagnoses 99, 202, 240, 244–246, 336

sacroiliac joint pain 196 rhizotomy, glossopharyngeal neuralgia 327 riboflavin (vitamin B2), migraine headache 304 risk assessment, opioid use for chronic pain 403–404 risk factors for development aberrant drug use 408–409, 413 ankle pain 235 cancer pain 342–343 cervical stenosis/myelopathy 97–99 cluster headaches 308 dural puncture headache 437–438 lumbar spondylosis, discogenic pain 145 myofascial pain syndrome 226–227 opioid dependence 442–443 postmastectomy lymphedema 367–368 postpartum pain 291–292 sacral insufficiency fracture 202, 240 vasovagal response during pain procedures 474 RIT. see regenerative injection therapy RLP. see residual-limb pain ROM (range of motion) testing 236, 245 ropivacaine 487 sacral insufficiency fracture 202–205 differential diagnosis 202, 275 postpartum pain 290 risk factors for development 240 sacral kyphoplasty 204–205, 205–206 sacroiliac joint pain 195–200, 196 differential diagnosis 144, 155, 195–197, 202 postpartum 290 sacroplasty 204–205 St John’s wort 77 saline injections, dural puncture headache 437 salivary gland disorders 336 SAMHSA/CSAT. see Substance Abuse and Mental Health Services Administration Scheuermann’s kyphosis 126–127, 127–352 sciatic nerve blocks, total knee arthroplasty 492 scoliosis epidural catheters 488 postpartum pain 290 Screener for Opioid Assessment for Patients with Pain-Revised (SOAPP-R) 408–409

525

Index

screening tools aberrant drug use 408–409 insomnia 460–461 opioids, safe use for chronic pain 404–405 SCS. see spinal cord stimulation SCT. see stem cell transplantation secondary insomnia 460 Seddon classification, diabetic neuropathy 56, 56 selective serotonergic reuptake inhibitors. see SSRIs; selective serotonin and norepinephrine reuptake inhibitors, SNRIs selective sodium channel blockers, diabetic neuropathy 61 self-medication, aberrant drug use 408 self-report scales, children 383 Semmes Weinstein monofilament examination (SWME) 54 sennosides, opioid-induced constipation 470 sensitization, poststroke pain 24, 27 sepsis, anesthesia-assisted opiate detoxification 449 septic arthritis, differential diagnosis 244 seronegative spondyloarthropathies 196 serotonin syndrome 7, 439–441, 440, 440 postherpetic neuralgia 12 treatment complications 310 triptans 300 see also SNRIs; SSRIs Sharp-Purser test, cervicogenic headache 84 shingles 1. see also postherpetic neuralgia short-lasting unilateral neuralgiform headache attacks with conjunctival injection and tearing (SUNCT) 330 shoulder abduction relief test 111 shoulder pain 109 and axial neck injuries 116–117 poststroke pain 23, 23–26 spinal cord injury pain 18 side effects, medication opioids 346–347, 347 patient-controlled analgesia 479 tricyclic antidepressants 3–7 see also treatment complications sign language, hearing impaired patients 388–389 sinusitis, differential diagnosis 312 Sjogren’s syndrome, differential diagnosis 214 skeletal-related events (SRE) 207–208. see also bone metastases

526

skin biopsy alcohol-induced neuropathy 66 diabetic neuropathy 53 HIV neuropathy 74 sleep apnea. see obstructive sleep apnea sleep diaries 460, 463. see also insomnia sleep disorders, fibromyalgia 216 sleep restriction 463 SLN (superior laryngeal neuralgia) 327 slow-wave sleep, fibromyalgia 216 Sluder neuralgia 307–329 small fiber HIV neuropathy 74–75 SMT. see spinal manipulative therapy SNRIs (selective serotonin and norepinephrine reuptake inhibitors) 218 alcohol-induced neuropathy 68, 269 chronic pain syndrome 373 diabetic neuropathy 59 hematological cancer 363 insomnia 462 migraine headache 302–310, 303 phantom limb pain 40 postpartum pain 294 pregnancy pain 286 spinal cord injury pain 19 thoracic outlet syndrome 107 see also duloxetine; milnacipam; venlafaxine sodium channels, poststroke pain 24 somatic dysfunction, spinal manipulation 230–231 somatization/somatoform disorder differential diagnosis 214, 217 Munchausen syndrome 456–458 myofascial pain syndrome 226–227 postlaminectomy pain syndrome 174, 175 sorbitol diabetic neuropathy 58 opioid-induced constipation 470 spasticity 232, 232–234 poststroke pain 23, 23–24 spinal cord injury pain 17–18 Special Interest Group on Neuropathic Pain, International Association for the Study of Pain (NeuPSIG) 2–3, 2–3, 2–15, 6 sphenopalatine ganglion 307–329 sphenopalatine neuralgia 329–332 spinal cord injury pain 16–20, 17–21. see also thoracic spine pain; vertebral fractures spinal cord stimulation (SCS) 50, 60, 225

complex regional pain 50, 50 pancreatic cancer pain 354 pancreatitis treatment 257 postlaminectomy pain 177 thoracic spine pain 129 spinal degeneration 152, 161 spinal fusion 196 spinal manipulative therapy (SMT) ,230–232, 231, 394 spinal stenosis 144, 152 and chronic pain 459–464 constipation 467–472, 467–472 lumbar. see lumbar spinal stenosis postlaminectomy pain, 174–179 postpartum pain 290 spino-thalamo-cortical pathway 24 spinous processes, Baastrup’s disease 160–162, 161, 162 spiritual healing 394–395 spondylarthopathies, differential diagnosis 245 spondylosis 165 spontaneous intracranial hypotension 435 sprained ankle, differential diagnosis 235 Spurling’s maneuver 111 SRE. see skeletal-related events SSRIs (selective serotonergic reuptake inhibitors) chronic pain syndrome 373 fibromyalgia 218 migraine headache 299–300, 302–310, 303 postpartum pain 293–294 poststroke pain 25 pregnancy pain 286 St John’s wort 77 standards, OSHA radiation exposure 417–418 Staphylococcus aureus 484 state inspections, radiation exposure 417 stationary bicycle test 168, 172 stellate ganglion block 49 stem cell transplantation (SCT) hematological cancer 362 knee osteoarthritis 248–249, 249 stereotactic radiosurgery, trigeminal neuralgia 322 Sternbach’s criteria, serotonin syndrome 440 steroids. see corticosteroids, epidural steroid injections stimulants, opioid-induced constipation 470 stimulus control, insomnia 463 strains, ankle pain 235

Index

stress, psychological aberrant drug use 413 and insomnia 460–461 phantom limb pain 39 tension headaches 313 see also trauma stress fractures 155, 202–205 strokes, and migraine headache 299. see also central poststroke pain stump pain (residual-limb pain) 38–39. see also phantom limb pain subdural hematoma, differential diagnosis 435 substance abuse 399, 406 definitions 400–401, 408 differential diagnosis 399–400 epidemiology/demographics 399 ethics 402–403 methadone prescription 499 neurobiological processes underlying addiction 401–402 opioid dependence 442–443 and postherpetic neuralgia treatment 9–10, 12 safe use of opioids for chronic pain 403–406 see also addiction; drug screens Substance Abuse and Mental Health Services Administration/Center for Substance Abuse Treatment (SAMHSA/CSAT) 444 substance dependence, definition 408 substance misuse, definition 408 subtalar (talocalcaneal) joint 235–236 sufentanil epidural catheters 487 patient-controlled analgesia 478 suicidal ideation/suicide anesthesia-assisted opiate detoxification 447–448 chronic pain syndrome 373 cluster headaches 308 and insomnia 459 medication-overuse headache 442 phantom limb pain 41–42 sphenopalatine neuralgia 329 spinal cord injury pain 16, 20 trigeminal neuralgia 318 sumatriptan 486 SUNCT (short-lasting unilateral neuralgiform headache attacks with conjunctival injection and tearing) 330 Sunderland’s classification, diabetic neuropathy 56, 56 super-concentrated platelets, ankle pain 238

superior hypogastric plexus 264, 266, 271 superior laryngeal neuralgia (SLN) 327 superior radioulnar joint 240 suppositories, opioid-induced constipation 470–471 supraorbital nerve stimulation, migraine headache 304 sural nerve biopsy, alcohol-induced neuropathy 66–67 surgery ankle pain 239 axial neck pain 121 Baastrup’s disease 163 Bertolotti’s syndrome 157 cervical radiculopathy 113–114 cervical stenosis/myelopathy 98–99 diabetic neuropathy 60 endometrial fibrosis 264–265 epidural hematoma 427–433 knee osteoarthritis 249–250 lumbar disc herniation 133–135, 134–136 lumbar spinal stenosis 171–177 pancreatitis treatment 257–258 phantom limb pain 41 postmastectomy lymphedema 370 sacral insufficiency fracture 204–205 scoliosis surgery 488 temporomandibular joint disorders 338 thoracic outlet syndrome 107–108 see also compression/ decompression; postlaminectomy pain syndrome swallowing, glossopharyngeal neuralgia 325 SWME (Semmes Weinstein monofilament examination) 54 sympathetic nerve blocks. see nerve blocks sympathetic nervous system fibromyalgia 216 postoperative analgesia 482–483 symphysiolysis, pregnancy pain 283 syringomyelia 17, 96 systematic reviews acupuncture 393 anesthesia-assisted opiate detoxification 451–452 chronic rectal pain 277 herbalism 396 massage 393 patient-controlled analgesia 477 postherpetic neuralgia 4 postoperative analgesia 482–483

see also Cochrane reviews; meta-analyses systemic lupus erythematosis, differential diagnosis 214 systemic vasculitis, differential diagnosis 214 TAA (total ankle arthoplasty) 239 TAC (trigeminal autonomic cephalalgia) 308, 330 Tai Chi, low back pain 394 talocalcaneal joint 235–236 talocrural joint 235–236 tanezumab, diabetic neuropathy 61 tapentadol diabetic neuropathy 61 phantom limb pain 40 thoracic outlet syndrome 107 TCAs. see tricyclic antidepressants TEDS. see treatment episode data sets telescoping, amputated limbs 39 temporomandibular joint disorders (TMD) 333–338, 334–339, 335–337, 337 tender point survey, fibromyalgia 215 tennis elbow 240–242 TENS. see transcutaneous electrical nerve stimulation tension, nervous/muscular 374–375. see also relaxation training; stress tension type headaches (TTH) 312–314, 314, 336, 435 teriparitide 204 terminology. see definitions theophylline, dural puncture 486 therapeutic relationship cancer pain 343 postlaminectomy pain syndrome 175 psychoanalysis 375–376 thermal radiofrequency ablation. see radiofrequency ablation thiamine. see vitamin B1 thigh thrust test 197 thoracic outlet syndrome (TOS) 102–108, 103, 107 clinical features 104–105 differential diagnosis 31–32, 32–34, 104–105, 109 examination/work-up 105 thoracic outlet spaces 103–104 thoracic radiculopathy 43–44 thoracic spine pain 123, 123–129, 124–125, 126, 127–352 thoracolumbar junction syndrome 127–128 thoracotomy, postoperative pain 43–45, 482 thrombosis, postoperative analgesia 483

527

Index

tibia fracture 38. see also phantom limb pain tibial H-reflex, alcohol-induced neuropathy 66 tibiofibular joint 235–236 Tietze syndrome. see costochondritis tizanidine brachial plexopathy 36 hematological cancer 364 phantom limb pain 40 spasticity 233 spinal cord injury pain 18 thoracic outlet syndrome 107 TKA (total knee arthroplasty) 491–495, 492, 493 TLS (tumor lysis syndrome) 362 TMD. see temporomandibular joint disorders tolerance definitions 400–401 neurobiological processes underlying addiction 401 tonsillectomy pain 325–328, 326–327 topical agents alcohol-induced neuropathy 68, 269 cancer pain 344 chronic rectal pain 277 diabetic neuropathy 59 HIV neuropathy 76 see also capsaicin; lidocaine topiramate migraine headache 302, 303 poststroke pain 26 torsades de pointes 58–59, 499–500 TOS. see thoracic outlet syndrome total ankle arthoplasty (TAA) 239 total knee arthroplasty (TKA) 491–495, 492, 493 total neuropathy score (TNS), HIV neuropathy 74 traction/neck distraction test 111 traditional Chinese medicine (TCM) 392 tramadol alcohol-induced neuropathy 68 diabetic neuropathy 59 dosing/onset of analgesia 9 phantom limb pain 40 postherpetic neuralgia 2–4, 2–15, 3–7, 7–12 pregnancy pain 287 serotonin syndrome 439–441 side effects 3–7, 7–8 thoracic outlet syndrome 107 transcutaneous electrical nerve stimulation (TENS) brachial plexopathy 33 chronic pelvic pain 268

528

diabetic neuropathy 61 lumbar facet mediated pain 140 phantom limb pain 40–41 postherpetic neuralgia 6–7 postpartum pain 293 post-thoracotomy pain 44 pregnancy pain 285 spinal cord injury pain 19 thoracic spine pain 128 transdermal analgesic routes 345–346 transient receptor potential channelsvanilloid receptors (TRP V1) 61 transnasal drug delivery, sphenopalatine neuralgia 331 transplacental drug delivery, fetal pain 380 trapped nerves, postpartum pain 291. see also compression/ decompression trauma glossopharyngeal neuralgia 326 spinal cord injury pain 16 and temporomandibular joint disorders 335 see also motor vehicle collisions; stress treatment algorithms diabetic neuropathy 57 endometrial fibrosis 264–265 pancreatitis 255 treatment complications anesthesia-assisted opiate detoxification 449–450 axial neck pain 121 bone metastases 212 breast cancer, metastatic 370 cluster headaches 310 complementary and alternative medicine 397 corticosteroids 202 discography 147–148 dural puncture headache 438 epidural catheters 484–487 epidural steroid injections 424 interspinous spacers 171 lumbar compression fracture 186 lumbar facet mediated pain 143 methadone prescription 501 neuraxial analgesia 355 paralysis following epidural injections 423–427, 429–433, 430, 432 and patient-controlled analgesia 479–480 postlaminectomy pain syndrome 179 sacral insufficiency fracture 205 sphenopalatine neuralgia 331

vasovagal response during pain procedures 473–475 vertebral augmentation procedures 188 see also side effects treatment episode data sets (TEDS), opioid dependence 442 tricyclic antidepressants (TCAs) alcohol-induced neuropathy 68, 269 brachial plexopathy 33, 36 cancer pain 344 chronic pain syndrome 373 chronic pelvic pain 269 complex regional pain syndrome 48 diabetic neuropathy 58–59 fibromyalgia 218 glossopharyngeal neuralgia 327 hematological cancer 363 migraine headache 218–302, 303 pancreatitis treatment 256 phantom limb pain 40 postherpetic neuralgia 2–3, 2–4, 2–15, 7–11, 9 postpartum pain 293–294 poststroke pain 25, 26 pregnancy pain 286 side effects 3–7 spinal cord injury pain 19 temporomandibular joint disorders 338 tension headaches 313–314 thoracic outlet syndrome 107 thoracic spine pain 128 see also amitriptyline; desipramine; imipramine; nortriptyline trigeminal autonomic cephalalgia (TAC) 308, 330 trigeminal neuralgia 316–323, 317, 320 differential diagnosis 312, 327, 336 trigemino-vascular system cluster headaches 308–309 migraine headache 297–298 trigger point injections 270, 371 trigger points 43 myofascial pain syndrome 82 tension headaches 313 triptans cluster headaches 309 migraine headache 299–300, 300–301 treatment complications 310 TTH. see tension type headaches tumor lysis syndrome (TLS) 362 T-wave response, alcohol-induced neuropathy 66

Index

ulnohumeral (trochlear) joint 240 ultrasound 175 epidural catheters 484 pancreatitis 254–255 pregnancy pain 287–288 United States postherpetic neuralgia, incidence 1 shingles, incidence 1 spinal cord injury pain, prevalence 16 United States Drug Enforcement Administration 413 United States Environmental Protection Agency 421 universal precautions, opioid use 404 urethral strictures, differential diagnosis 267 urethritis, differential diagnosis 267 urinary tract infections, differential diagnosis 267 urine drug screening (UDS) 405, 504–507. see also drug screens Utah Early Neuropathy Scale (UENS) 75 valproate, alcohol-induced neuropathy 67–68 valproic acid, pregnancy pain 285 Valsalva maneuver 111 varicella-zoster virus (VZV) 1. see also postherpetic neuralgia vascular compression, glossopharyngeal neuralgia 326 vascular imaging studies, thoracic outlet syndrome 106 vasovagal response 473–475 venlafaxine migraine headache 303 poststroke pain 25 tension headaches 314 ventral posterior nucleus 24 ventral tegmental area (VTA) 401–402 ventrocaudalis portae nucleus 24 verapamil 302 cluster headaches 309

migraine headache 303 verbiest syndrome 164–165. see also lumbar spinal stenosis vertebral compression 124–125. see also compression/ decompression; lumbar compression fracture vertebral augmentation (VBA) procedures 186–191, 204–205 vertebral fractures differential diagnosis 202 postpartum pain 290 vertebroplasty, hematological cancer 364 visceral pain differential diagnosis 207 endometrial fibrosis 261–265 lumbar facet mediated pain 137–138 lumbar spinal stenosis 168 pancreatitis 253–255, 253–258 pelvic 275 spinal cord injury pain 16–17, 17–21 thoracic region 123 viscosupplementation, ankle pain 237 visual aura, migraine headache 297–299, 302 vitamin B1 alcohol-induced neuropathy 64–66, 70 HIV neuropathy 73 vitamin B2, migraine headache 304 vitamin B9 65, 73 vitamin B12 alcohol-induced neuropathy 65 HIV neuropathy 73 vitamin E alcohol-induced neuropathy 67, 70 migraine headache 304 vitamin pills, pregnancy 285 VP (percutaneous vertebroplasty) 188–191

VTA (ventral tegmental area) 401–402 VZV. see varicella-zoster virus Webster study, postherpetic neuralgia 5 whiplash 81–82, 116. see also cervicogenic headache white matter hyperintensities (WMH), migraine headache 299 willow bark, low back pain 396 withdrawal syndrome anesthesia-assisted opiate detoxification 450 opioid dependence 443, 447 Wong-Baker Pain Rating Scale 383 workers, compensation (WC) system 224–225 myofascial pain syndrome 223 World Health Organization (WHO) analgesic ladder cancer pain 343 pancreatitis treatment 256 x-ray findings. see radiography X-STOP Interspinous Process Decompression (IPD) System 170 X-STOP PEEK (polyetherketone) 170 yoga, low back pain 394 zaleplon 462 ziconotide, intrathecal drug delivery 178–179 zolpidem 462 fibromyalgia 218 insomnia 462 zonisamide migraine headache 303 poststroke pain 26 zopiclone 462 zygapophysial joint. see lumbar facet mediated pain

529